A PYQ Workbook of History of Modern India

You might also like

Download as pdf or txt
Download as pdf or txt
You are on page 1of 351

PREVIOUS YEAR QUESTIONS

(PYQ) WORKBOOK OF
HISTORY OF
MODERN INDIA
For Civil Services Prelims Examination

(1200+ Topic Wise Solved Objective Questions from UPSC CSE Prelims (1991 to 2023),
CAPF & CDS EXAM (2014 to 2023) and STATE PCS Prelims Exams (1991 to 2023))

PYQ WORKBOOK

Forum Learning Centres


DELHI (ORN) MUKHERJEE PATNA HYDERABAD GURGAON
NAGAR
19, Pusa Road, 2nd 862, Banda 2nd floor, AG 1st Floor, SM Plaza, Property No. 894,
Floor, IAPL House, Bahadur Marg, Palace, E Boring RTC X Rd, Indira G.F., Saraswati
Opposite Metro Ist Floor, Dr. Canal Road, Patna, Park Road, Jawahar Vihar, Chakkarpur,
Pillar #95-96, Mukherji Nagar, Bihar - 800001 Nagar, Hyderabad, Near MG Rd
Karol Bagh, New Near Batra Telangana - Metro Station,
Delhi-05 Cinema, Delhi - 500020 Sector-28,
110009 Gurgaon, Haryana
https://blog.forumias.com | www.academy.forumias.com | 9311740400
PREVIOUS YEAR QUESTIONS (PYQ) WORKBOOK OF
HISTORY OF MODERN INDIA
For Civil Services Prelims Examination

First Edition (Print): 2024

No part of this publication may be reproduced or transmitted, in any form or by any means,
electronic, mechanical, photocopying, recording or otherwise, or stored in any retrieval system of
any nature without the written permission of the copyright holder and the publisher, application
for which shall be made to the publisher.
©Forum (Flaviant Network Pvt. Ltd.)

This book/material is sold subject to the conditions that it shall not, by way of trade or otherwise,
be lent, re-sold, hired out or otherwise circulated without the publisher's prior consent in any
form of binding or cover other than that in which it is published and without a similar condition
including this condition being imposed on the subsequent purchaser.

We are continuously working on improving our quality. If you are unhappy or happy with the
quality of this study material, want to report an error, or want to leave us feedback, you can email
us at ravi@forumias.com

© 2024 1st Print Edition | For feedback email us : ravi@forumias.com


The PYQ WORKBOOK
Dear Aspirants,

ForumIAS is glad to inform you that we have come up with a PYQ Workbook Series
to provide the aspirants with a holistic idea regarding challenges and opportunities
associated with the changing pattern and difficulty level of UPSC CSE Prelims Exam.
The PYQ Workbook includes the questions asked in different objective questions-based
exams conducted by Union Public Service Commission like UPSC CSE Prelims Exam,
CAPF Exam and CDS Exam. It will also include the solved previous year questions from
different State PCS Prelims Examinations, to equip the aspirants with every relevant fact
and concept needed for their UPSC CSE Prelims Exam preparation. This PYQ Workbook
Series, along with ForumIAS Practicebooks and Comprehensive Testbooks, will help the
aspirants to get the required factual as well as conceptual clarity regarding the different
subjects and to crack their UPSC CSE Prelims Exam.

Trend of Modern History Questions in UPSC CSE Prelims in recent years:

HISTORY OF MODERN INDIA


14

11
No. of Questions

10
10
10 8
7
6
5 7 7 7
4 4 4

2012 2014 2016 2018 2020 2022 2023


Year
Number of Modern History questions asked in UPSC CSE GS-1 Prelims from 2011 till
2023

 As it can be seen from the above analysis, the weightage of questions of History
of Modern India has experienced a fluctuating trend in recent years. Somewhere
between 4 to 15 questions have been asked in History of Modern India each year and
the difficulty level of questions have varied.
 The questions in Modern History have been a good mix of traditional and not so
traditional ones in recent years. The traditional questions can be answered with a good
reading of NCERTs and standard books.
 On the other hand, there have been a few random and not so traditional questions
related to different personalities and aspects related to them, some isolated events in
History of Modern India etc. For example, a) Rakhmabai case b) question related
to archaeologists Alexander Rea, A. H. Longhurst, Robert Sewell, James Burgess
and Walter Elliot c) question related to Francis Xavier etc. Also, some questions on
books like Songs from Prision, Desher Katha and a question which relates National
Handloom Day with the Swadeshi Movement. These types of questions have been
asked mostly because these personalities or books or cases or events have been in news
for one reason or other.

Ultimately, one has to supplement the knowledge from standard books with current affairs.
With this basic strategy, aspirants will be able to answer 70 to 80 percent questions of History
of Modern India with ease.

Happy Learning and Happy Testing!!!


Your team @ForumIAS
INDEX
UNIT 1. ADVENT OF EUROPEANS AND CONSOLIDATION OF
BRITISH EMPIRE IN INDIA .....1 – 25
1.1. UPSC CSE Previous Years' Questions .....1
1.2. Other Examination Previous Years' Questions .....3
SOLUTIONS .....10

UNIT 2. REBELLIONS AND REFORM MOVEMENTS .....26 – 80


2.1. UPSC CSE Previous Years' Questions .....26
2.2. Other Examination Previous Years' Questions .....28
SOLUTIONS .....45

UNIT 3. EMERGENCE OF INDIAN NATIONALISM [1857-1919] .....81 – 135


3.1. UPSC CSE Previous Years' Questions .....81
3.2. Other Examination Previous Years' Questions .....85
SOLUTIONS .....100

UNIT 4. ERA OF MASS NATIONALISM [1919 - 1947] .....136 – 212


4.1. UPSC CSE Previous Years' Questions .....136
4.2. Other Examination Previous Years' Questions .....144
SOLUTIONS .....162

UNIT 5. GOVERNANCE AND OTHER ASPECTS .....213 – 258


5.1. UPSC CSE Previous Years' Questions .....213
5.2. Other Examination Previous Years' Questions .....216
SOLUTIONS .....230

UNIT 6. MISCELLANEOUS .....259 – 346


6.1. UPSC CSE Previous Years' Questions .....259
6.2. Other Examination Previous Years' Questions .....265
SOLUTIONS .....288
HISTORY OF MODERN INDIA

HISTORY OF MODERN INDIA


ADVENT OF EUROPEANS AND CONSOLIDATION OF BRITISH EMPIRE IN INDIA
*This unit consists of questions from Advent of Europeans and Consolidation of British empire in
India.

1.1. UPSC CSE Previous Years’ Questions (a) 1 and 2 only


(b) 2 and 3 only
1. With reference to Indian history, consider (c) 3 only
the following statements: (d) 1, 2 and 3
1. The Dutch established their factories/ 4. Which of the following statements correctly
warehouses on the east coast on lands explain the impact of Industrial Revolution
granted to them by Gajapati rulers. on India during the first half of the
2. Alfonso de Albuquerque captured Goa nineteenth century? [UPSC CSE Pre. 2020]
from the Bijapur Sultanate. (a) Indian handicrafts were ruined.
3. The English East India Company (b) Machines were introduced in the Indian
established a factory at Madras on a plot textile industry in large numbers.
of land leased from a representative of the (c) Railway lines were laid in many parts of
Vijayanagara empire. the country
Which of the statements given above are (d) Heavy duties were imposed on the
correct? [UPSC CSE Pre. 2022] imports of British manufactures
(a) 1 and 2 only 5. The staple commodities of export by the
(b) 2 and 3 only English East India Company from Bengal
(c) 1 and 3 only in the middle of the 18th century were:
(d) 1, 2 and 3 [UPSC CSE Pre. 2018]
(a) Raw cotton, oil-seeds and opium
2. In the first quarter of seventeenth century,
(b) Sugar, salt, zinc and lead.
in which of the following was/were the
(c) Copper, silver, gold, spices and tea
factory/factories of the English East India
(d) Cotton, silk, saltpetre and opium
Company located?
1. Broach 6. Economically, one of the results of the
2. Chicacole British rule in India in the 19th century was
the [UPSC CSE Pre. 2018]
3. Trichinopoly
(a) increase in the export of Indian
Select the correct answer using the code given handicrafts
below: [UPSC CSE Pre. 2021] (b) growth in the number of Indian owned
(a) 1 only factories
(b) 1 and 2 (c) commercialization of Indian agriculture
(c) 3 only (d) rapid increase in the urban population
(d) 2 and 3
7. Consider the following:
3. With reference to the history of India, [UPSC CSE Pre. 2012]
consider the following pairs: 1. Assessment of land revenue on the basis
[UPSC CSE Pre. 2020] of nature of the soil and the quality of
crops.
1. Aurang In-charge of treasury of the
2. Use of mobile cannons in warfare.
State
3. Cultivation of tobacco and red chillies.
2. Banian Indian agent of the East
Which of the above was/were introduced into
India Company
India by the English?
3. Mirasidar Designated revenue payer (a) 1 only
to the State (b) 1 and 2
Which of the pairs given above is/are correctly (c) 2 and 3
matched? (d) None

1 PYQ Workbook
HISTORY OF MODERN INDIA

8. During the time of which Mughal Emperor 3. Mir Jafar entered in conspiracy with the
did the English East India Company English for the defeat of Nawab Siraj-ud-
establish its first factory in India? daulah in the Battle of Plassey
[UPSC CSE Pre. 2009] Which of the statements given above is/are
(a) Akbar correct? [UPSC CSE Pre. 2004]
(b) Jahangir (a) 1, 2 and 3
(c) Shahjahan (b) 3 only
(d) Aurangzeb (c) 2 and 3
(d) None of the above
9. In the year 1613, where was the English East
India Company given permission to set up a 14. Which one of the statements is not correct?
factory (trading post)? [UPSC CSE Pre. 2003]
[UPSC CSE Pre. 2006] (a) Ali Mardan Khan introduced the system
(a) Bangalore of revenue farming in Bengal.
(b) Madras (b) Maharaja Ranjit Singh set up modern
(c) Masulipattam foundries to manufacture cannons at
(d) Surat Lahore.
(c) Sawai Jai Singh of Amber had Euclid’s
10. Which one of the following is the correct ‘Elements of Geometry’ translated into
statement? [UPSC CSE Pre. 2005] Sanskrit.
(a) The modern Kochi was a Dutch colony (d) Sultan Tipu of Mysore gave money for
till India’s Independence. the construction of the idol of Goddess
(b) The Dutch defeated the Portuguese and Sharda in the Sringeri temple.
built Fort Williams in the modern Kochi.
15. In India, among the following locations,
(c) The modern Kochi was first a Dutch
the French established their earliest factory
Colony before the Portuguese took over
at: [UPSC CSE Pre. 2003]
from them.
(d) The modern Kochi never became a part (a) Surat
(b) Pulicat
of the British colony.
(c) Cochin
11. Which of the following shifted his capital (d) Qasim Bazar
from Murshidabad to Munger?
16. With reference to the entry of European
[UPSC CSE Pre. 2005] powers into India, which one of the
(a) Alivardi Khan following statements is NOT correct?
(b) Siraj-ud-Daula [UPSC CSE Pre. 2003]
(c) Mir Jafar (a) The Portuguese captured Goa in 1499
(d) Mir Qasim (b) The English opened their first factory in
12. Which one of the following is the correct South India at Masulipatnam
chronological order of the battles fought in (c) In Eastern India, the English company
India in the 18th Century? opened its first factory in Orissa in 1633
[UPSC CSE Pre. 2005] (d) Under the leadership of Dupleix, the
(a) Battle of Wandiwash-Battle of Buxar- French occupied Madras in 1746
Battle of Ambur-Battle of Plassey 17. Which of the following pairs are correctly
(b) Battle of Ambur-Battle of Plassey-Battle matched?
of Wandiwash-Battle of Buxar
List-I (Period) List-II (War)
(c) Battle of Wandiwash-Battle of Plassey-
Battle of Ambur-Battle of Buxar A. AD 1767-69 1. First Anglo-Maratha
(d) Battle of Ambur-Battle of Buxar-Battle of War
Wandiwash-Battle of Plassey B. AD 1790-92 2. Third Mysore War
13. Consider the following statements: C. AD 1824-26 3. First Anglo-Burmese
1. In the Third Battle of Panipat, Ahmad War
Shah Abdali defeated Ibrahim Lodi D. AD 1845-46 4. Second Anglo-Sikh
2. Tipu Sultan was killed in the Third War
Anglo-Mysore War Code: [UPSC CSE Pre. 2004]

PYQ Workbook 2
HISTORY OF MODERN INDIA

(a) 2 and 4 A B C D
(b) 3 and 4 (a) 4 3 2 1
(c) 1 and 2 (b) 4 3 1 2
(d) 2 and 3 (c) 3 4 1 2
18. With which one of the following mountain (d) 3 4 2 1
tribes did the British first come into contact 22. Who among the following was the first
with after the grant of Diwani in the year European to initiate the policy of taking
1765? [UPSC CSE Pre. 2002] part in the quarrels of Indian Princess with
(a) Garo a view to acquire territories?
(b) Khasi [UPSC CSE Pre. 1996]
(c) Kuki (a) Clive
(d) Tipperah (b) Dupleix
19. Who among the following Indian rulers (c) Albuquerque
established embassies in foreign countries (d) Warren Hastings
on modern lines? [UPSC CSE Pre. 2001] 23. Hooghly was used as a base for piracy in the
(a) Haider Ali Bay of Bengal by: [UPSC CSE Pre. 1995]
(b) Mir Qasim (a) Portuguese
(c) Shah Alam-II (b) French
(d) Tipu Sultan (c) Danish
(d) British
20. Match list-I with list-II and select the
correct answer using the codes given below 24. Which one of the following pairs is correctly
the lists: matched? [UPSC CSE Pre. 1995]
List-I List-II List-I List-II
(Voyagers) (Countries) (a) Battle of Buxar Mir Jafar Vs. Clive
A. Vasco da Gama 1. Spain (b) Battle of French Vs. East
B. Christopher 2. Portugal Wandiwash India Company
Columbus (c) Battle of Dalhousie Vs.
C. Captain Cook 3. Holland Chilianwala Marathas
D. Tasman 4. Great Britain (d) Battle of Khurda Nizam Vs. East
India Company
Code: [UPSC CSE Pre. 2000]
A B C D 25. The first to start a joint stock company to
(a) 3 2 1 4 trade with India were:
(b) 2 1 4 3 [UPSC CSE Pre. 1994]
(c) 1 4 3 2 (a) Portuguese
(d) 4 3 2 1 (b) Dutch
(c) French
21. Match list-I with list-II and select the (d) Danish
correct answer using the codes given below
the lists: 1.2. Other Examination Previous Years’
List-I List-II Questions
(Year) (Event) 26. Consider the following events and arrange
A. 1775 1. First Anglo-Burmese War them in chronological order:
B. 1780 2. First Anglo-Afghan War I. Battle of Mudki
C. 1824 3. First Anglo-Maratha War II. Battle of Porto Novo
D. 1838 4. Second Anglo-Mysore III. Battle of Shakarkheda
War IV. Battle of Bedara
Code: [UPSC CSE Pre. 1999] Code: [U.P.P.C.S. (Pre) 2022]

3 PYQ Workbook
HISTORY OF MODERN INDIA

A B C D 32. In which of the following places the Dutch


(a) III IV II I established their trading centre in India?
(b) I II III IV [U.P.P.C.S. (Pre) 2017]
(c) II III IV I (a) Nagappattinam, Chinsura, Machilipatnam
(d) IV III II I (b) Surat, Bharuch, Agra
27. At which place did Raja Ranjit Singh set up (c) Cochin, Ahmedabad, Patna
the Adalat-i-Ala? [U.P.P.C.S. (Pre.) 2021] (d) All of the above
(a) Amritsar 33. Who remarked about the East India
(b) Lahore
Company that “The Company is an anomaly,
(c) Firozpur
but it is part of a system where everything is
(d) Multan
an anomaly”? [U.P.P.C.S. (Pre) 2017]
28. In India the First Municipal Corporation (a) Warren Hastings
was set up in which one among the following (b) G.B. Macaulay
places? [U.P.P.C.S. (Pre.) 2021] (c) Lord Clive
(a) Calcutta (d) Henry Dundas
(b) Madras
(c) Bombay 34. Match list-I with list-II and select the
(d) Delhi correct answer using the codes given below
the lists:
29. With reference to the Treaty of “Aix-la-
Chapelle-1748” which of the following List-I List-II
statements is/are correct? A. First Carnatic War 1. Ended by Treaty
1. The 1st Carnatic War was ended. of Paris
2. The English got back Madras.
B. Third Carnetic 2. Britishers Lost
Select the correct answer using the codes War
given below. [U.P.P.C.S. (Pre) 2020]
C. Second Carnatic 3. Inconclusive
(a) 1 only War War
(b) 2 only
(c) Both 1 and 2 D. First Mysore War 4. Ended by
(d) Neither 1 nor 2 Treaty of Aix La
Chapelle
30. Who among the following established a Code: [U.P.P.C.S. (Pre) 2016]
modern armory in Dindigal Mysore in
1755? [U.P.P.C.S. (Pre) 2020] A B C D
(a) Nanjraj (a) 1 3 4 2
(b) Haider Ali (b) 2 4 1 3
(c) Devraj (c) 4 1 3 2
(d) Chikka Krishnaraj (d) 3 1 4 2
31. Match list-I with list-II and select the 35. Which of the following is correctly matched?
correct answer using the codes given below [U.P.P.C.S. (Mains) 2014]
the lists:
List-I List-II
List-I List-II
(a) First Anglo- Hyder Ali was defeated
A. Treaty of Allahabad 1. 1782 Mysore War War
B. Treaty of Manglore 2. 1784 (b) Second Hyder Ali defeated the
C. Treaty of Salbai 3. 1769 Anglo-Mysore British
D. Treaty of Madras 4. 1765 War
Code: [U.P.P.C.S. (Pre) 2019] (c) Third Anglo- Tipu Sultan won the
Mysore War battle and did not cede
A B C D
his territory to British
(a) 4 2 3 1
(b) 2 4 3 1 (d) Fourth Anglo- Tipu was defeated and
(c) 4 2 1 3 Mysore War was killed in the thick
(d) 2 4 1 3 of battle

PYQ Workbook 4
HISTORY OF MODERN INDIA

36. Which one of the following statements is 1. The first European power to occupy
correct about Dalip Singh, the ex-Maharaja Pondicherry were the Portuguese.
of Punjab? [U.P.P.C.S. (Mains) 2013] 2. The second European power to occupy
(a) He died in Paris on 23 October 1893. Pondicherry were the French.
(b) He was cremated at Nasik. 3. The English never occupied Pondicherry.
(c) He never renounced the Sikh faith. Which of the statements given above is/are
(d) He had never visited Russia. correct? [U.P.P.C.S. (Mains) 2006]
37. Which one of the following was the last (a) 1 only
Governor of Bengal appointed by the (b) 2 and 3
Mughal Emperor? (c) 3 only
(d) 1, 2 and 3
[U.P.P.C.S. (Mains) 2013]
(a) Sarfraz Khan 44. Sirajuddaula was defeated by Lord Clive in
(b) Murshid Quli Khan the battle of- [U.P.P.C.S. (Mains) 2005]
(c) Alivardi Khan (a) Plassey
(d) Shujauddin Muhammad Khan (b) Buxar
(c) Munger
38. Begum Samru constructed most eminent (d) Wandiwash
church at: [U.P.P.C.S. (Mains) 2012]
45. In which Governor’s tenure, Diwani rights
(a) Mount Abu of Bengal, Bihar and Odisha was granted
(b) Nainital to East India Company by Emperor Shah
(c) Sardhana Alam? [U.P.P.C.S. (Spl) (Mains) 2004]
(d) Kanpur (a) Lord Clive
39. Which one of the following was the (b) Lord Cornwallis
Emperor of India when the British East (c) Lord Wellesley
India Company was formed in London? (d) Lord William Bentinck
[U.P.P.C.S. (Pre) 2012] 46. The kingdom of Ranjit Singh included–
(a) Akbar [U.P.P.C.S. (Spl) (Mains) 2004]
(b) Jahangir (a) Delhi
(c) Shahjahan (b) Kabul
(d) Aurangzeb (c) Makran
40. At which one of the following places in (d) Srinagar
India did the Portuguese build their first 47. Among the following factories in Bengal,
fortress? [U.P.P.C.S. (Mains) 2010] the one established by the Portuguese was:
(a) Anjidiv [U.P.P.C.S. (Pre) 2004]
(b) Cannanore (a) Bandel
(c) Cochin (b) Chinsurah
(d) Goa (c) Hooghly
(d) Shrirampur
41. Tipu Sultan defeated the British Army in
1780 at– [U.P.P..C.S. (Mains) 2009] 48. Who among the following was the founder
(a) Hyderabad of Calcutta?
(b) Pollilur [U.P.P.C.S. (Spl) (Mains) 2004]
(c) Srirangapatna (a) Charles Ayar
(d) Nizamabad (b) Job Charnok
(c) Garold Angiyar
42. Who were the first Europeans to come to (d) William Novris
India for trade? [U.P.P.C.S. (Mains) 2007]
49. The Anglo-Afghan relations in the 19th
(a) Dutch century were influenced by the fear of
(b) English whose expansion towards India?
(c) French
[U.P.P.C.S. (Spl.) (Mains) 2004]
(d) Portuguese
(a) Afghanistan
43. With reference to Pondicherry (Now (b) France
Puducherry), consider the following (c) Iran
statements: (d) Russia

5 PYQ Workbook
HISTORY OF MODERN INDIA

50. The most decisive battle that led to the (a) De Almeida
establishment of supremacy of the British (b) Albuquerque
in India was: [U.P.P.C.S. (Pre) 2003] (c) Dupleix
(a) The Battle of Buxar (d) Robert Clive
(b) The Battle of Plassey
57. Bombay was taken over by the English East
(c) The Battle of Wandiwash
India Company from–
(d) The Third Battle of Panipat
[Uttarakhand P.C.S. (Pre) 2010]
51. Who among the following is considered the (a) The Dutch
founder of French Company in India: (b) The French
[U.P.P.C.S. (Mains) 2003] (c) The Danish
(a) Richelieu (d) The Portuguese
(b) Mazarin
(c) Colbert 58. Capital of Kingdom of Maharaja Ranjit
(d) Franco Martin Singh was: [Uttarakhand P.C.S. (Pre) 2006]
(a) Amritsar
52. The French failed to establish power in the (b) Patiala
Deccan because: [U.P.P.C.S. (Pre) 1998] (c) Lahore
(a) Duplex was not a capable general (d) Kapurthala
(b) The English had a strong army
(c) The French were not liked by the Indians 59. With which misal was Ranjit Singh related
(d) Pondicherry was not a strategic Centre [M.P.P.C.S. (Pre) 2021]
53. The following European Powers entered (a) Aahluvaliyan
into Indian trade at one time or the other: (b) Dalevaliyan
1. The English (c) Kanhaiya
(d) Shukarchakiya
2. The Dutch
3. The French 60. Where was the Western presidency situated
4. The Portuguese in the early period of the East India
Company? [M.P.P.C.S. (Pre) 2018]
Select the correct chronological order of their
entry from the code given below: (a) Surat
[U.P.P.C.S. (Pre) 1997] (b) Satara
(c) Bombay
(a) 1, 2, 3, 4
(d) Panaji
(b) 4, 2, 1, 3
(c) 3, 4, 2, 1 61. Which English Governor of East India
(d) 2, 3, 4, 1 Company in India was expelled by
54. The secret of success of East India Company Aurangzeb- [M.P.P.C.S. (Pre) 2008]
in India was: [U.P.P.C.S. (Pre) 1994] (a) Aungier
(a) Absence of Nationalism in India (b) Sir John Child
(b) The company army received western (c) Sir John Gayer
training and they had modern arms. (d) Sir Nicholas Waite
(c) Indian Soldiers lacked fidelity 62. Who was the ruler of Delhi at the time of
consequently the one who paid them the battle of Buxar?
sufficient could have them on his service [M.P.P.C.S. (Pre) 2005]
(d) All the above three
(a) Aurangzeb
55. Who was the real founder of Portuguese (b) Shah Alam I
power in India? (c) Bahadur Shah Zafar
[U.P. Lower Sub. (Pre) 2003] (d) Shah Alam II
(a) Vasco da Gama 63. Which of the following war began the
(b) Albuquerque consolidation of British supremacy over
(c) Bartholomeu Dias India? [M.P.P.C.S. (Pre) 1994]
(d) George Oxdone
(a) Battle of Buxar
56. Which one of the followings is connected (b) Battle of Plassey
with ‘Blue Water’ policy? (c) 3rd Battle of Mysore
[Uttarakhand P.C.S. (Pre) 2016] (d) Independence Struggle of 1857

PYQ Workbook 6
HISTORY OF MODERN INDIA

64. The successor of Maharaja Ranjit Singh (c) Dost Mohammad


was: [Jharkhand P.C.S. (Pre) 2003] (d) Sher Ali
(a) Harsingh Nalwa 72. Tipu Sultan set up his capital at–
(b) Kharak Singh
[38th B.P.S.C. (Pre) 1992]
(c) Sher Singh
(d) Naunihal Singh (a) Srirangapatna
(b) Mysore
65. British general who defeated/beat Haider (c) Banglore
Ali in War of Porto Novo: (d) Coimbatore
[Jharkhand P.C.S. (Pre) 2003]
73. Which of the following statements about
(a) Captain Popham ‘Agency Houses’ is/are correct?
(b) Sir Eyer Coote 1. They were important commercial
(c) Sir Hector Munro formations in the late eighteenth and
(d) General Goddad nineteenth century India.
66. Who was appointed Deputy Diwan of 2. The operation of managing agencies
Murshidabad by Robert Clive after the remained confined to Calcutta.
Allahabad Treaty? Select the correct answer using the code given
[45th B.P.S.C. (Pre) 2001] below. [CAPF 2022]
(a) Mohd. Raza Khan (a) 1 only
(b) Shitab Rai (b) 2 only
(c) Rai Durlabh (c) Both 1 and 2
(d) Syed Ghulam Hussain (d) Neither 1 nor 2
67. Who was the first Portuguese Viceroy in 74. Consider the following statements about
India? [45th B.P.S.C. (Pre) 2001] the Treaty of Amritsar (1809) :
(a) Diaz 1. The Treaty fixed the Sutlej as the boundary
(b) Vasco da Gama between the Punjab and British India.
(c) Almeida 2. It was after this treaty that Maharaja
(d) Albuquerque Ranjit Singh was able to annex Jammu,
68. Which of the British officials defeated Multan and Kashmir to his domains.
Portuguese at Sowlley? Which of the statements given about is/are
[44th B.P.S.C. (Pre) 2000] correct? [CAPF 2020]
(a) William Hawkins (a) 1 only
(b) Thomas Best (b) 2 only
(c) Thomas Roe (c) Both 1 and 2
(d) Josiah Child (d) Neither 1 nor 2
69. Which one of the following was the 75. Examine these two statements carefully and
immediate cause of the First Carnatic War? select the correct answer using the codes
[44th B.P.S.C. (Pre) 2000] given below.
(a) Anglo-French Rivalry Statement I – In 1856, Nawab Wajid Ali Shah
(b) Austrian War of Succession was dethroned and exiled to Calcutta on the
(c) Issues of Carnatic Succession plea that the region was being misgoverned.
(d) Capture of French ships by the British Statement II – The Nawab was accused of
70. Englishmen made the Treaty of being unable to control the rebellious chiefs
Srirangapatna with– and Taluqdars.
[42nd B.P.S.C. (Pre) 1997] Codes: [CAPF 2018]
(a) Hyder Ali (a) Both the statements are individually
(b) Dupleix true and Statements II is the correct
(c) Tipu Sultan explanation of Statement I
(d) Nandraj (b) Both the statements are individually
true but Statement II is not the correct
71. Ranjeet Singh got famous Kohinoor explanation of Statement I
diamond from- [41st B.P.S.C. (Pre) 1996] (c) Statement I is true but Statement II is false
(a) Shah Shuja (d) Statement I is false but Statement II is
(b) Zaman Shah true

7 PYQ Workbook
HISTORY OF MODERN INDIA

76. Which one of the following commercial (a) Signature


centres declined after mid-eighteenth (b) Land document
century? [CAPF 2018] (c) Trade permit
(a) Calcutta (d) Tax on textiles
(b) Madras 81. Which one of the following was primarily
(c) Dhaka associated with ‘Dadni’ system?
(d) Bombay [CDS 2023 (I)]
77. Examine these two statements carefully and (a) Textile production
select the correct answer using the code (b) Warfare
given below. (c) Payment to officials
Statement I The British legal scholars (d) Revenue collection
relied on Indian Pandits and Maulavis for 82. The British policy towards Afghanistan in
understanding of canons of authoritative the first half of the 19th century aimed at
texts. [CDS 2022 (II)]
Statement II British codified the Hindu Laws (a) Strengthening Afghanistan as a buffer
in 1783 and the Muslim Laws in 1785. against Russia.
Code: [CAPF 2017] (b) Weakening Russian influence In
(a) Both the statements are individually true Afghanistan but not promoting a strong
and statement II is the correct explanation Afghanistan either.
of statement I. (c) Promoting Russian influence in
(b) Both the statements are individually Afghanistan to control insurgent tribal
true, but statement II is not the correct leaders.
(d) Annexing Afghanistan as a directly-
explanation of statement I.
controlled British Territory.
(c) Statement I is true, but statement II is
false. 83. The work Siyar-ul- Mutakherin, which
(d) Statement I is false, but statements II is describes the Battle of Plassey, 1757, was
true. written by [CDS 2020 (I)]
(a) Salabat Jung
78. Which one of the following statements
(b) Qasim Khan
about the Treaty of Allahabad is NOT
(c) Ghulam Husain
correct? [CAPF 2017]
(d) Ram Mohan Roy
(a) It was signed in the year 1765.
(b) The Mughal Emperor by a Farman 84. Which of the following features of the State
formally granted the Diwani of Bengal, of Arcot in 18th century South India are
Bihar and Orissa to the East India correct?
Company. 1. The founders of the dynasty that ruled
(c) Awadh was restored to its Nawab on Arcot were Daud Khan Panni and
payment of 50 lakhs. Saadatullah Khan.
(d) Banaras and the Surrounding tracts were 2. Arcot became the site of a protracted
detached from Awadh and handed over struggle between the English and Dutch
to Shah Alam II. East India Companies from the 1740s.
3. Decentralisation was a key feature of the
79. The First Anglo-Maratha War was State of Arcot in the 18th century.
concluded by which one of the following?
4. The other major State to emerge in South
[CDS 2023 (I)] India at this time was Mysore.
(a) The Treaty of Surat
Select the correct answer using the codes
(b) The Treaty of Purandar
given below: [CDS 2015 (I)]
(c) The Convention of Wadgaon
(a) 1 and 2
(d) The Treaty of Salbai
(b) 1, 2 and 4
80. In the context of eighteenth century India, (c) 3 and 4
what was Dastak? [CDS 2023 (I)] (d) 2 and 4

PYQ Workbook 8
HISTORY OF MODERN INDIA

85. Which of the following statement(s) about (c) Both 1 and 2


the penetration of English into Bengal is/ (d) Neither 1 nor 2
are correct? 86. Consider the following statements
1. Job Charnock arrived in Sutanati in 1. Battle of Buxar provided the key to the
August, 1690 and laid the foundation of English to establish their rule in India.
Calcutta which later became the heart of 2. The Treaty of Allahabad, concluded in
the British Indian empire. 1765, enabled the British to establish
2. The French East India Company built a their rule in Bengal.
fort near the Fort William in Calcutta. Which of the statement(s) given above is/are
correct? [CDS 2014 (I)]
Select the correct answer using the codes
(a) Only 1
given below: [CDS 2014 (II)] (b) Only 2
(a) Only 1 (c) Both 1 and 2
(b) Only 2 (d) Neither 1 nor 2

9 PYQ Workbook
HISTORY OF MODERN INDIA

SOLUTIONS
Indian markets and India supplied the raw material
1.1. UPSC CSE Previous Years’ Questions (Cotton) to Britain.
1. Solution: (b) Indian textiles now had to face stiff competition from
Exp) Option b is the correct answer. machine-made English textiles in the European and
Statement 1 is incorrect: Gajapati dynasty came to an end American markets. Heavy duties were imposed on imported
in around 1542, whereas the Dutch East India Company was Indian textiles in Britain. The machine-made cheap textiles
formed in 1602 through a charter. of Britain successfully captured the African markets. This
led to severe downfall of Indian handicraft.
Statement 2 is correct: Albuquerque acquired Goa from the
Sultan of Bijapur in 1510 with ease; the principal port of the 5. Solution (d)
Sultan of Bijapur became “the first bit of Indian territory Exp) Option d is the correct answer.
to be under the Europeans since the time of Alexander the
Great”. In 1600, the East India Company acquired a charter from the
ruler of England, Queen Elizabeth I, granting it the sole right
Statement 3 is correct: British East Indian company entered
to trade with the East.
a treaty with the native Cheiftaim of Madras (a representative
of Vijaynagar Empire) to lease a site in Madras in 1639. The fine qualities of cotton and silk produced in India, had a
big market in Europe. So, cotton and silk from Bengal were
2. Solution: (a) the major items of export from India. Indigo and saltpetre
Exp) Option a is the correct answer. were the other major imports from India, and the fact that
By 1st quarter of 17th century, English East India Company both products were produced in the eastern Gangetic plain,
had established factories at Surat, Broach, Ahmedabad, especially in Bihar and Bengal, stimulated British efforts to
Agra, and Masulipatnam and not at Chicacole and establish factories on the east coast as well as the west coast
Trichinopoly. of the Indian subcontinent. Pepper, cloves, cardamom and
cinnamon from India too were in great demand in Europe.
The timelines are:
Early in the 18th century the Portuguese found that they
1613- A permanent factory of the east India company
established at Surat could import opium from India and sell it in China at a
considerable profit. By 1773 the British had discovered the
1616- 1st factory in south at Masulipatnam
trade, and became the leading suppliers of the Chinese
1619- Sir Thomas Roe set up factories at Agra, Ahmedabad, market. The opium exports from British India to China
and Broach. increased from 4,000 chests per year at the beginning of the
3. Solution: (b) 19th Century to more than 60,000 chests by the 1880s.

Exp) Option b is the correct answer. 6. Solution (c)


Pair 1 is incorrect. Aurang was a Persian term for a Exp) Option c is the correct answer.
warehouse. One of the results of the British rule in India in the 19th
Pair 2 is correct. Banians acted as intermediaries for century was the emergence of the commercialisation
European merchants in Bengal particularly for East India of agriculture. So far, agriculture had been a way of life
Company. rather than a business enterprise. Now agriculture began
Pair 3 is correct. Before the Ryotwari settlement system in to be influenced by commercial considerations. Certain
Madras Presidency, the company recognized mirasidars as specialised crops began to be grown not for consumption in
the sole proprietors of land. They acted as the Designated the village but for sale in the national and even international
revenue payer to the State. markets. Commercial crops like cotton, jute, groundnut,
oilseeds, sugarcane, tobacco, etc., were more remunerative
4. Solution (a)
than foodgrains.
Exp) Option a is the correct answer.
Important Tips
During Mughal Period, Indian handicrafts produced
excellent quality of textile products and as per estimates Economic impacts of British Rule in India:
nearly two hundred varieties of cotton and silk fabrics were • After 1820, European markets were virtually closed
exported from India. to Indian handicrafts, as a result Indian export of
When Industrial Revolution took place in England, Britain handicrafts fell drastically. Cheap and machine-made
needed raw materials from India and a wide market for its imports flooded the Indian market due to one-way
machine-made goods. Thus, owing to industrial revolution free trade for the Britishers and imposition of heavy
cheap machine-made goods from Britain flooded the tariffs of nearly 80 per cent on Indian products.

PYQ Workbook 10
HISTORY OF MODERN INDIA

• There was a rapid decrement in the number of Indian • In 1616, the company established its first factory in the
owned industries. Indian industry was steadily south in Masulipattanam.
destroyed. The demand from Indian textile industry
9. Solution: (d)
was replaced by British made clothes. A thriving
ship-building industry was crushed. The British ships Exp) Option d is the correct answer.
contracted by the Company were given a monopoly In 1612 Sir Thomas Roe visited to Mughal Emperor
over trade routes, while even the Indian merchant Jahangir to arrange for a commercial treaty that would
ships plying along the coast were made to face heavy give the company exclusive rights to reside and build
duties. Also, The British did not allow the Indian steel factories in Surat and other areas. In return, the company
industry to grow. offered to provide the Emperor with goods and rarities from
• Another important feature of British rule in India the European market. Jahangir accepted the proposal and the
was Deurbanisation i.e. the decline of many cities company created trading posts in Surat (where a factory was
and a process of ruralisation of India. Many artisans, built in 1613), Madras (1639), Bombay (1668), and Calcutta
faced with diminishing returns and repressive policies (1690).
abandoned their professions, moved to villages and 10. Solution: (b)
took to agriculture.
Exp) Option b is the correct answer.
7. Solution (d) William Fort Thrissur, Kerala was constructed by the Dutch
Exp) Option d is the correct answer. East India Company with the permission from Kingdom of
Cochin in 1714 after defeating the Portuges.
Statement 1 is incorrect. During Mauryan times the
assessment of land revenue was done according to the William Fort was later occupied by Zamorin of Calicut and
quality of the land and the nature of the crop, and the rate thereafter Tipu Sultan. Tipu Sultan defeated Zamorin of
of land revenue was not on the gross produce of the land Calicut and renamed it as Tipu Sultan Fort. The fort was
but it was computed after taking into account the gross captured and recaptured many times by British Empire,
income and also expenditure per unit of land and also after Dutch East India Company, Kingdom of Mysore, Zamorin of
considering the producers’ surplus. Hence this practice of Calicut and Kingdom of Cochin. Hence statement a, c, and
revenue assessment is use in Indian since ancient times. d are incorrect.

Statement 2 is incorrect. The Mughals excelled in the use 11. Solution: (d)
of firearms. The Mughal emperor Babur conquered India Exp) Optio d is the correct answer.
in 1526 using mobile cannons drawn by bullocks, horses,
Mir Qasim was the Nawab of Bengal who shifted his capital
and camels. Thus, mobile cannons were not introduced into from Murshidabad to Munger during his reign. He ruled
India by the English. from 1760 to 1763 and is known for his efforts to modernize
Statement 3 is incorrect. The Portuguese contribution to the administration and military of Bengal. His reign was
the diversification of Indian agriculture was substantial. marked by conflicts with the British East India Company,
They are credited with introducing into India tobacco, leading to the Battle of Buxar in 1764, where he was defeated
sweet potatoes, red chillies, pineapples, papayas and by the British and their allies. His decision to shift the capital
cashewnuts. Thus, it was not introduced into India by the to Munger was part of his attempts to assert his authority and
English. break away from British influence.

8. Solution: (b) Important Tips


Exp) Option b is the correct answer. • Alivardi Khan (1671 – 9 April 1756), known as Nawab
Mughal Emperor Jahangir permitted the English East of Bengal from 1740 to 1756, rose to prominence by
India Company to establish their factory in 1613 at Surat. defeating Sarfaraz Khan in 1740, thereby ending the
The British East India Company came to India for trading in rule of the Nasiri dynasty of Nawabs and establishing
spices, an important commodity in Europe back then and as his own authority.
it was used to preserve meat. Apart from this, they primarily • Siraj ud-Daulah was the Nawab of Bengal from 1756
do trading in silk, cotton, indigo dye, tea, and opium. to 1757. He was defeated by the British in the Battle of
Plassey and was killed shortly afterwards.
Important Tips
• Mir Jafar was the Nawab of Bengal from 1757 to 1760.
Mughal and East India Company:
He was installed by the British for his favors to British
• Mughal emperor Jahangir granted a Farman to in winning Battle of Plassey.
Captain William Hawkins permitting the English to
erect a factory at Surat in 1613. 12. Solution: (b)
• In 1615, Thomas Roe, the Ambassador to James I, got Exp) Option b is the correct answer.
an imperial Farman from Jahangir for doing trading The correct chronological order of the battles fought in India
and establish factories all across the Mughal empire. in the 18th century is:

11 PYQ Workbook
HISTORY OF MODERN INDIA

1. Battle of Ambur (1746)- It was fought between the 16. Solution: (a)
Mughal Empire and the Maratha Empire. The Mughals Exp) Option a is the correct answer.
were defeated and the Marathas gained control of the
Afonso de Albuquerque acquired Goa from the Sultan of
Carnatic region. Bijapur Ismail Adil Shah with the help of Krishnadev raya
2. Battle of Plassey (1757)- It was fought between (ruler of Vijaynagar empire) in 1510. He is also known to
the British East India Company and the Nawab of have abolished the practice of Sati in Goa. Hence statement
Bengal. The British were victorious and this led to the a is incorrect.
establishment of British rule in Bengal. The British established their first factory in the South India
3. Battle of Wandiwash (1760)- It was fought between the in Masulipatnam in 1611. Hence statement b is correct.
British East India Company and the French East India In Eastern India, the English company opened its first factory
Company. The British were victorious and this led to the in Orissa in 1633. Hence statement c is correct.
end of French influence in India. Under the leadership of Dupleix, the French occupied
4. Battle of Buxar (1764)- It was fought between the Madras in 1746. Hence statement d is correct
British East India Company and the combined forces
17. Solution: (d)
of the Mughal Empire, Awadh, and Bengal. The British
were victorious and this led to the British gaining control Exp) Option d is the correct answer.
of Bengal, Bihar, and Orissa. Pair 1 is incorrectly matched: The First Anglo-Maratha
War (1775-1782) resulted from the power struggle in the
13. Solution: (b) Maratha Empire. The Treaty of Surat led to British support
Exp) Option b is the correct answer. for Raghunathrao against the Peshwa, but the British-
Statement 1 is incorrect- The Third Battle of Panipat was Calcutta Council annulled it. Battles followed, and the Treaty
of Salbai (1782) made the East India Company retain Salsette
fought in 1761 between the Maratha Empire and the Afghan
and Broach, secured a Maratha guarantee against Hyder Ali,
Empire, led by Ahmad Shah Abdali. The Afghans won the
and instilled Madhavrao II as Peshwa.
battle, and the Maratha Empire was severely weakened.
Ibrahim Lodi was not involved in this battle. Pair 2 is correctly matched: The Third Anglo-Mysore War
(1790-1792) arose due to strained Anglo-Mysore relations.
Statement 2 is incorrect- Tipu Sultan was killed in the
Tipu Sultan’s aggressive actions, refusal to release English
Fourth Anglo-Mysore War in 1799. prisoners, and alliances with the French led to war. The
Statement 3 is correct- Mir Jafar entered into a conspiracy conflict ended with Treaty of Seringapatam (1792), where
with the British for the defeat of Nawab Siraj-ud-daulah in Tipu ceded half his kingdom to the British, paid a substantial
the Battle of Plassey in 1757. indemnity, and surrendered two sons as surety.
Pair 3 is correctly matched: The First Anglo-Burmese
14. Solution: (a)
War (1824-1826) began over territorial disputes along the
Exp) Option a is the correct answer. Bengal-Burma borders. The British declared war in 1824,
The system of revenue farming in Bengal was introduced by leading to battles, including the Battle of Yangon. In 1825,
Murshid Quli Khan, the first Nawab of Bengal. Ali Mardan they entered Ava, the Burmese capital, but faced tough
Khan was a general in the army of Murshid Quli Khan. resistance. The war’s hardships, diseases, and rain led to
the Treaty of Yandabo in 1826, resulting in Burma ceding
15. Solution: (a) Assam, Manipur, Arakan, and coastal areas to the British and
Exp) Option a is the correct answer. accepting a British resident at Ava.
The French East India Company was the last to be formed Pair 4 is incorrectly matched: The Second Anglo-Sikh
among Dutch, Danish, Portuguese, and French. The company War (1848-1849) was fueled by the humiliation of the first
established its first factory in Surat under Francis Caron in war, mistreatment of Maharani Jindan Kaur, and conflicts in
1668. The second factory was established at Masulipatnam a Multan over tax assessments. Battles occurred at Ramnagar
year later, in 1669. and Chilianwala, with an indecisive outcome in Ramnagar
and a Sikh victory in Chilianwala. The British won the final
Important Tips battle at Gujrat in 1849. Punjab was annexed, the young
French East India Company: Maharaja Duleep Singh was sent to England, and the Koh-i-
• Jean-Baptiste Colbert founded the French East India Noor diamond came under British control.
Company in 1664. 18. Solution: (b)
• Francois Martin of the French East India Company Exp) Option b is the correct answer.
established a trading centre in 1674 at Pondicherry.
The Khasi people are an indigenous tribe of northeastern
• In 1674, the François Martin of the French East India India. They inhabit the Khasi Hills, which are located in the
Company established a trading centre at Pondicherry, state of Meghalaya. The British first came into contact with
which eventually became the chief French settlement the Khasi people in 1765, after the grant of Diwani of Bengal.

PYQ Workbook 12
HISTORY OF MODERN INDIA

The British were interested in the Khasi Hills because these a series between the British and Burmese empires. It resulted
were a source of timber and other resources. The Khasi in a decisive British victory, granting them control over
people were initially hostile to the British, but they were Northeastern India, including Assam, Manipur, Cachar, and
eventually subdued by the British military. more, while Burmese agreed to pay an indemnity and sign a
commercial treaty.
19. Solution: (d)
Exp) Option d is the correct answer. Option D in List-I matches Option 2 in List-II: The First
Anglo-Afghan War (1838-1842) was a British campaign in
Tipu Sultan, the ruler of the Kingdom of Mysore, was the
Afghanistan, initially supporting Shah Shujah in a succession
first Indian ruler to establish embassies in foreign countries
dispute but ending in a disastrous retreat from Kabul. It was a
on modern lines. He established embassies in France, Turkey,
key event in the Great Game, a 19th-century British-Russian
and Egypt in the late 18th century. These embassies were
staffed by trained diplomats, and they were responsible for rivalry for influence in Central Asia.
conducting relations between the Kingdom of Mysore and 22. Solution: (b)
these countries. Tipu Sultan’s decision to establish embassies
Exp) Option b is the correct answer.
in foreign countries was a sign of his forward-thinking and
his desire to modernize the Kingdom of Mysore. He believed Dupleix was a son of a director of the French East India
that by establishing embassies, he could learn from the West Company and joined the Company’s service at Pondicherry
and improve the military and technological capabilities of in 1720. He was the first who lend European troops to the
his kingdom. Indian state in the matter of quarrels.

20. Solution: (b) 23. Solution: (a)


Exp) Option b is the correct answer. Exp) Option a is the correct answer
Portuguese explorer Vasco da Gama was the first European Hooghly was used as a base for piracy in the Bay of
to reach India via the Atlantic Ocean at Calicut in India. Bengal by the Portuguese. Hooghly was one of the main
Columbus was born in the Italian seaport of Genoa in economically growing districts in West Bengal. Hooghly was
1451. In 1476 Columbus moved to Lisbon, Portugal, and for in a good position both in industry and agriculture. During
many years attempted to gain support for a journey he was the sixteenth and seventeenth centuries, Hooghly was
planning to find new trade routes to the Far East. Eventually controlled by the Portuguese settlers and traders. Portuguese
Ferdinand and Isabella, the King and Queen of Spain, merchants sailed from the ‘Bay of Bengal’ to Hooghly to
agreed to finance him. deceive the customs duties, and practised piracy in the
Abel Tasman was the greatest of the Dutch(Holland) region.
navigators and explorers, who was the first European to sight
Tasmania, New Zealand, Tonga, and the Fiji Islands. Important Tips

James Cook was a British naval captain, navigator, and Portuguese Decline in India:
explorer who sailed the seaways and coasts of and conducted • While most of India got its independence from the
three expeditions to the Pacific Ocean ranging from the British, the Portuguese still held on to its colonial
Antarctic ice fields to the Bering Strait and from the coasts outposts in India.
of North America to Australia and New Zealand. • On July 24, 1954, an organisation called the “United
Front of Goans’ seized the Dara naive while Nagar
21. Solution: (c)
Haveli was seized by Azad Gomantak Dal in August
Exp) Option c is the correct answer. of that year.
Option A in List-I matches Option 3 in List-II: The First • The decision by the International Court of Justice at
Anglo-Maratha War (1775-1782) began with the Treaty of The Hague to give access to Portugese territories in
Surat and ended with the Treaty of Salbai. It saw British defeat India was rendered useless.
and the restoration of positions before the war. The conflict
• Adopting a ‘wait and watch’ tactic from 1951 to
resulted from political turmoil in the Maratha Empire after
1961, the Indian government highlighted the issue of
the murder of Narayanrao Peshwa.
decolonisation before the international communities,
Option B in List-I matches Option 4 in List-II: The Second while at the same time enforcing an economic embargo.
Anglo-Mysore War (1780-1784) was influenced by Anglo-
French hostilities during the American Revolutionary War. • It was in December 1961, when the Indian military
Hyder Ali, ruler of Mysore, initially succeeded against the launched an invasion of Goa. Against overwhelming
British but faced setbacks. French and British forces from odds, the Portuguese tried to put up a fight but were
Europe joined the conflict. When France made peace, the swiftly defeated by the Indian Army.
British sought a resolution, resulting in the unfavorable 1784 • The Governor of Portuguese India signed the
Treaty of Mangalore. Instrument of Surrender on 19 December 1961,
Option C in List-I matches Option 1 in List-II: The First Liberating Goa after 450 years of Portuguese rule in
Anglo-Burmese War (1824-1826) was the initial conflict in India.

13 PYQ Workbook
HISTORY OF MODERN INDIA

24. Solution: (b) 26. Solution: (a)


Exp) Option b is the correct answer. Exp) Option a is the correct answer.
Pair a is incorrect- Battle of Plassey (and not Buxar) was Chronological order:
between Mir Jafar and Clive. It occurred on 23 June 1757. • Battle of Shakar Kherda (1724): The Battle of Shakar
On the other hand, Battle of Buxar occurred on October 22- Kherda occurred on 11 October 1724 near Sakhar
23, 1764 and it involved the British East India Company, Kherda in Berar, India, between Nizam-ul-Mulk and
led by Hector Munro, facing the combined forces of Mir Mubariz Khan, the Subedar of Deccan. Nizam-ul-Mulk
Qasim, the Nawab of Bengal; Shuja-ud-Daula, the Nawab emerged victorious, defeating and killing Mubariz Khan.
of Awadh; and the Mughal Emperor Shah Alam II. As a result, he was rewarded with the title of Asaf Jah and
Pair b is correct- The Battle of Wandiwash was a decisive reappointed as the Subedar of the Deccan province by
victory for the British East India Company over the French Emperor Muhammad Shah Rangeela in 1725.
East India Company. It was fought on 22 January 1760 at • Battle of Bedara (1759): The Battle of Chinsurah or
Vandavasi, in the present day Tiruvannamalai district of Bedara occurred on 25 November 1759 during the Seven
Tamil Nadu. The battle was part of the Seven Years’ War, Years’ War near Chinsurah, a Dutch outpost. British
which was being fought in Europe at the time. The British forces, led by Colonel Francis Forde, defeated the Dutch
and French were also fighting a parallel war in India, and the East India Company, who had been invited by the Nawab
Battle of Wandiwash was a major turning point in that war. of Bengal to help expel the British from Bengal.
The British were led by Sir Eyre Coote, and the French were • Battle of Porto Novo (1781): The Battle of Porto Novo
led by the Marquis de Bussy-Castelnau. The British had a occurred on 1 July 1781 during the Second Anglo-Mysore
numerical advantage, and they were also better equipped. War. Sir Eyre Coote’s British force, with over 8,000 men,
The French were defeated and were forced to withdraw from defeated an estimated 40,000-strong army led by Hyder
India. Ali of the Kingdom of Mysore in Porto Novo, India.
Pair c is incorrect- The Battle of Chilianwala was fought on • Battle of Mudki (1845): The Battle of Mudki was fought
13 January 1849 between the British East India Company on 18 December 1845, between the forces of the East India
and the Sikh Empire. It was a bloody and indecisive battle, Company and part of the Sikh Khalsa Army, the army of
the Sikh Empire of the Punjab. The British army won an
with both sides claiming victory. The battle took place in
untidy encounter battle, sustaining heavy casualties.
the Chillianwala region of Punjab, now in Pakistan.
Pair d is incorrect- The Battle of Khurda was fought on 11 27. Solution: (b)
March 1795 between the Maratha Empire and the Nizam Exp) Option b is the correct answer.
of Hyderabad. The Marathas were victorious, and the Raja Ranjit Singh set up the Adalat-i-Ala, the highest court
Nizam was forced to cede much of his territory and pay an of the Sikh Empire, in Lahore. Lahore was the capital of the
indemnity of Rs. 3 crores. Sikh Empire, and it was a major cultural and commercial
center. The Adalat-i-Ala was responsible for hearing appeals
25. Solution: (b)
from district and provincial courts and settling disputes
Exp) Option b is the correct answer. between individuals and communities. The final court of
Dutch were the first to start a joint-stock company to trade appeal lies with the Raja Ranjit Singh himself.
with India. The Dutch East India Company established the
28. Solution: (b)
world’s first stock market in Amsterdam in March 1602.
Exp) Option b is the correct answer.
Important Tips
In 1687, the first municipal corporation in India was
Dutch East India Company: set up at Madras. The Chennai Municipal Corporation
• The formation of the company took place in March (officially the Corporation of Chennai), formerly known as
1602. the Corporation of Madras, is the civic body that governs
• The Dutch East India Company established factories in the city of Chennai (formerly Madras), India. Inaugurated
on September 29, 1688, under a Royal Charter issued by
India at Masulipatnam in 1605 AD, Pulicat (1610 AD),
King James II on December 30, 1687, as the Corporation of
Surat (1616 AD), Bimlipatam (1641 AD), Karaikal
Madras, it is the oldest municipal body of the Commonwealth
(1645 AD), Chinsura, Kasimbazar, Patna, Balasore,
of Nations outside Great Britain. It is headed by a mayor,
Nagapatam, and Cochin.
who presides over 200 councillors, each of whom represents
• Pulicat was the main centre of Dutch in India till 1690 one of the 200 wards of the city. It is also the second oldest
AD. Later Nagapatam replaced it. corporation in the world.
• The Dutch conceded to the British after their defeat in
29. Solution: (c)
the Battle of Bedera in 1759 AD
Exp) Option c is the correct answer.
The First Carnatic War is remembered for the Battle of St.
1.2. Other Examination Previous Years’ Thome (in Madras) fought between the French forces and
Questions the forces of Anwar-ud-din, the Nawab of Carnatic, to whom

PYQ Workbook 14
HISTORY OF MODERN INDIA

the English appealed for help. The First Carnatic War was B. Treaty of Mangalore- 2. 1784
an extension of the Anglo-French War in Europe which was Executed on March 11, 1784, the Treaty of Mangalore
caused by the Austrian War of Succession. The Treaty of Aix- marked a pivotal agreement between Tipu Sultan and
La-Chapelle, (1748) ended the War of Austrian Succession. the British East India Company. This significant accord
Hence, statement 1 is correct. was established in the city of Mangalore and effectively
Under the terms of this treaty, Madras was handed back concluded the Second Anglo-Mysore War..
to the English. The French, in turn, got their territories in C. Treaty of Salbai- 1. 1782
North America. Hence, statement 2 is correct.
Formally executed on May 17, 1782, the Treaty of Salbai
Important Tips materialized as a result of extensive negotiations between
The War of Austrian Succession: delegates from the Maratha Empire and the British
East India Company. This treaty was the culmination of
• Between 1740 and 1748, most of Europe’s great powers
efforts to resolve the consequences of the First Anglo-
were involved in a conflict caused by the question of
Maratha War and was ultimately inked by Warren
Maria Theresa’s succession to the Austrian Habsburg
Hastings and Mahadaji Sindhia.
crown.
D. Treaty of Madras- 3. 1769
• The war involved all of Europe, with France, Prussia,
Spain, Bavaria and Saxony arrayed against Austria and On April 4, 1769, the Treaty of Madras materialized as
Britain. a significant pact that marked the conclusion of the First
Anglo-Mysore War. This peace accord, signed between
• The first two series of wars, the First Silesian War
Mysore and the British East India Company, led by
(1740–42) and the Second Silesian War (1744–45) were
Lord Harry Verelst, was a pivotal step towards ending
centered around Austria and Prussia.
the hostilities. The conflict, which had ignited in 1767,
• The third war was centred on the continued conflict saw Hyder Ali’s forces making advances that brought
between France and Britain over colonial possessions them close to capturing Madras during one phase of the
in India and North America. fighting.
• During the war, British troops proved their worth as
32. Solution: (d)
soldiers.
Exp) Option d is the correct answer.
The war was concluded with the Peace treaty of Aix-la-
Chapelle, signed in October 1748. In 1602, the United East India Company of the Netherlands
was formed and given permission by the Dutch government
• Under this treaty, France agreed to leave the Austrian
to trade in the East Indies including India. The Dutch
Netherlands and give back Madras to Britain in return
founded their first factory in Masaulipatam in Andhra
for Louisbourg.
Pradesh in 1605. Subsequently, they also established trading
• Maria Theresa was also confirmed as Austrian ruler. centres in various parts of India.
30. Solution: (b) Konkan (Northern part of Westcoast India) - Surat (1616-
Exp) Option b is the correct answer. 1795), Agra (1621-1720), Burhanpur, Kanpur (1650-1685),
Ahmadabad (1617-1744), Bharuch, Vengurla (1637-1685),
Haider Ali, the ruler of the Kingdom of Mysore, established
Kundapura (1667- ca.1682), Patna.
a modern armory in Dindigal in 1755. The armory
Malabar (Southern part of Westcoast India) - Veeramala
was established to manufacture and store weapons and
Hills,Cheruvathur , Cannanore (1663-1790) (taken from
ammunition for the Mysorean army. Haider Ali was a
Portugal), Ponnani (ca. 1663), Cochin, Purakkad (ca. 1680-
skilled military leader and innovator, and he modernized
1750), Kayamkulam (ca. 1645), Quilon (Coylan) (1661)
and expanded the Mysorean army during his reign. The
(taken from Portugal).
Dindigul armory was one of several military and industrial
establishments he established to support his military Coromandel (East coast of India) - Golkonda(1662-ca
1733), Jaggernaikpoeram , Nagelwanze (1669-1687); now
campaigns and improve the strength and capability of his
Nagulavancha, Machilipatnam, Petapoeli (1606-1668);
forces.
now Nizampatnam, Paliacatta (1610-1781/ 1785-1795/ 1805-
31. Solution: (c) 1825) to the English; now Pulicat, Tegenapatnam, Kudalur
Exp) Option c is the correct answer. (1608-1758); now Cuddalore, Nagapattinam (1658-
1781) to the English, Tuticorin or Tutucorim (1658); now
A. Treaty of Allahabad- 4. 1765. Thoothukudi, chinsura.
On August 16, 1765, the Treaty of Allahabad was
formalized, establishing an agreement between Shah 33. Solution: (b)
Alam II, the Mughal Emperor and successor of Alamgir Exp) Option b is the correct answer.
II, and Robert Clive, a representative of the East India “The Company is an anomaly but it is part of a system where
Company. This accord came into being following the everything is an anomaly” remark was given by Thomas
Battle of Buxar, which took place on October 22, 1764. Babington Macaulay.

15 PYQ Workbook
HISTORY OF MODERN INDIA

Important Tips Pair d is correct- The Fourth Anglo-Mysore War (1799)


was fought between the British East India Company and
Thomas Babington Macaulay.
Tipu Sultan. The war ended with the death of Tipu Sultan in
• He became India’s first Law Member of the Governor-
the Battle of Srirangapatna in 1799.
General Council after the Charter Act of 1833 was
passed. 36. Solution: (a)
• He ensured that English flourished in India.
Exp) Option a is the correct answer.
• He brought education policy in 1835 and was published
Dalip Singh, the last Maharaja of the Sikh Empire, died in
titled “Minute on Indian Education”.
Paris on 23 October 1893 at the age of 55. He was cremated
• He advocated imparting English to Indians and
at Elveden, Suffolk, England.
teaching them in English.
• English Education Act was passed in 1835 by Lord Important Tips
William Bentick. Here are some more details about Dalip Singh’s life:
34. Solution: (c) • He was born in Lahore, Punjab, on 6 September 1838.
Exp) Option c is the correct answer. • He was the only son of Maharaja Ranjit Singh and
The correct answer is A- 4, B-1, C-3, D-2. Maharani Jind Kaur.

First Carnatic War (1746-1748)- Between: On one side • He was proclaimed Maharaja of the Punjab in 1843, at
the age of five.
there was the Governor of France, Duplex, and the Nawab
of Karnataka, on the other, the Governor of the English East • He was deposed in 1849 after the Second Anglo-Sikh
India Company. The war ended with the Treaty of Aix-la War.
Chapelle, which was signed in 1748. • He was exiled to England, where he lived on a pension
Second Carnatic War (1748-1754) - Nizam of Hyderabad from the British government.
died resulting in a war of succession. Duplex supported • He converted to Christianity in 1853.
Muzzafar Jung for Hyderabad and Chanda Sahib for • He married Bamba Müller, a German woman, in 1864.
Karnataka. Whereas, British supported Nasir Jang for
• He had eight children.
Hyderabad and Nawab Anwar-ud-din for Karnataka. The
war was inconclusive. • He died in Paris in 1893.

Third Carnatic War (1756 - 1763) - The immediate cause 37. Solution: (b)
was the Seven Year War (1756-1763) in Europe. Robert
Exp) Option b is the correct answer.
Clive and Watson captured Chandranagar. The Battle of
Wandiwash happened in 1760. Between Eyre Coot( British Murshid Quli Khan was the last Governor of Bengal
General) and Count de Lally(French General). The French appointed by the Mughal Emperor. He was appointed in 1717
were defeated in this war.The Treaty of Paris was signed in by Mughal Emperor Farrukhsiyar. He was a very capable
1763. administrator and is credited with bringing stability and
First Mysore War (1767-1769): It was fought Between Hyder prosperity to Bengal. He also founded the Nasiri dynasty of
Ali and the Triple Alliance of Maratha, Niza, and Britishers. Nawabs of Bengal.
British lost the war. The Treaty of Madras was signed in 1769.
38. Solution: (c)
35. Solution: (d)
Exp) Option c is the correct answer.
Exp) Option d is the correct answer.
Pair a is incorrect- The First Anglo-Mysore War (1767- One of the most eminent churches that Begum Samru
69) was fought between the British East India Company constructed is the Basilica of Our Lady of Graces in
and Hyder Ali, the ruler of the Kingdom of Mysore. The Sardhana. The church was built in 1823 and it is considered
war ended in a stalemate with the signing of the Treaty of to be one of the finest examples of Gothic architecture in
Madras in 1769. India. The church is also known as the Samru Church or the
Pair b is incorrect- The Second Anglo-Mysore War (1780- Begum Samru’s Church.
84) was fought between the British East India Company
and Hyder Ali and his son Tipu Sultan. The war ended with Important Tips
the signing of the Treaty of Mangalore in 1784. Begum Samru, also known as Samru bibi, was a military
Pair c is incorrect- The Third Anglo-Mysore War (1790- leader and a philanthropist. She was born in 1770 in
Sardhana, Uttar Pradesh, India. She was the daughter
92) was fought between the British East India Company,
of a Hindu merchant and a Muslim woman. She was
the Maratha Empire, and the Nizam of Hyderabad against
married to a French soldier, Walter Reinhardt Sombre,
Tipu Sultan. The war ended with the signing of the Treaty
who died in 1778.
of Srirangapatna in 1792.

PYQ Workbook 16
HISTORY OF MODERN INDIA

43. Solution: (a)


After the death of her husband, Begum Samru took
control of his army and became a powerful military leader. Exp) Option a is the correct answer.
She fought against the Maratha Empire and the British Portuguese were the first European power to set up a factory
East India Company. She was also a philanthropist and in the Pondicherry region during the 16th century. Hence,
she built a number of schools, hospitals, and mosques in statement 1 is correct.
Sardhana.
After the Portuguese, the second European power to capture
39. Solution: (a) Pondicherry was the Dutch. Hence, statement 2 is Not
Exp) Option a is the correct answer. correct.

Akbar was the third Mughal emperor, who reigned from In the year 1761, the British captured Pondicherry from the
1556 to 1605. Akbar was the Emperor of India when the French but later had to return it back to the French under the
British East India Company was formed in London on 31 Treaty of Paris (1763). Hence, statement 3 is Not correct.
December 1600 CE. Important Tips
Important Tips The story of Pondicherry:
Facts about Akbar: • The French East India Company set up a trading center
• Akbar succeeded his father, Humayun, under a regent, at Pondicherry in the year 1674, and this later became
Bairam Khan, who helped the young emperor expand the most prominent French settlement in India. But the
and consolidate Mughal domains in India. French continuously faced opposition from the Dutch
and the English.
• With a strong personality and a successful general,
Akbar gradually enlarged the Mughal Empire to • In the year 1693, Pondicherry was again captured by
include much of the Indian subcontinent. the Dutch as the French Governor Francois Martin
surrendered to the Dutch Governor. Later, in the year
• His power and influence, however, extended over
1699, the French regained Pondicherry from the Dutch.
the entire subcontinent because of Mughal military,
political, cultural, and economic dominance. • Pondicherry was occupied several times by the British,
but it was ultimately the French who ruled the territory.
• To unify the vast Mughal state, Akbar established a
centralized system of administration throughout his • The de facto transfer of Pondicherry from the French
empire and adopted a policy of conciliating conquered government to the Indian union took place in 1954. But
rulers through marriage and diplomacy the treaty affecting the de jure transfer of Pondicherry
got finally ratified in the year 1962.
40. Solution: (c)
44. Solution: (a)
Exp) Option c is the correct answer.
Exp) Option a is the correct answer.
The first fort built by the Portuguese was Fort Emmanuel in
Kochi, Kerala. The permission for building it was granted to The Battle of Plassey took place on June 23, 1757. It was
them by their ally, the local maharaja of Kochi in 1503 AD. a pivotal battle in Indian history where the British East
The structure was simple and used local materials. India Company, led by Robert Clive, decisively defeated
Siraj-ud-Daulah, the Nawab of Bengal. The victory at the
41. Solution: (b) Battle of Plassey marked the beginning of British colonial
Exp) Option b is the correct answer. dominance in India.
Tipu Sultan defeated the British Army in 1780 at Pollilur. Important Tips
The Battle of Pollilur was fought on 10 September 1780 near
The Battle of Wandiwash happened on January 22, 1760,
Conjeevaram, the city of Kanchipuram in present-day Tamil
when the French tried to take control of the Vandavasi
Nadu state, India, as part of the Second Anglo-Mysore War.
Fort in Tamil Nadu. The British, commanded by British
It was fought between an army commanded by Tipu Sultan Lieutenant-General Sir Eyre Coote, stopped their attempt,
of the Kingdom of Mysore, and a British East India Company leading to the Battle of Wandiwash and playing a part in
force led by Colonel William Baillie. The Mysore army won the Third Carnatic War. The Treaty of Paris was signed
a decisive victory, capturing Baillie and his entire force. between the East India Company and French.
42. Solution: (d) 45. Solution: (a)
Exp) Option d is the correct answer.
Exp) Option a is the correct answer.
Portuguese explorer Vasco da Gama was the first European
The sixteenth Mughal Emperor, Shah Alam II, played a
to reach India via the Atlantic Ocean at Calicut in India.
pivotal role in the transfer of Diwani rights, specifically for
Portuguese were followed by the Dutch when they tried to Bengal, Bihar, and Orissa, to the East India Company on
enter the Indian market in the middle of the 16th century. 12th August 1765 under the tenure of Robert Clive. The
The British and the French came much later. battle of Buxar concluded with the signing of the Treaty

17 PYQ Workbook
HISTORY OF MODERN INDIA

of Allahabad in 1765. This treaty granted the East India Important Tips
Company the authority to collect taxes, which is referred
• The First Anglo-Afghan War (1839-42) was fought
to as Diwani rights, for the territories of Bengal, Bihar, and
because the British were worried that the Russians
Orissa. were gaining influence in Afghanistan. The British
Important Tips invaded Afghanistan and installed a puppet ruler, Shah
Shuja, but they were eventually forced to withdraw.
• Lord Cornwallis was the Governor of Bengal from
This conflict marked a key episode in the 19th-century
1786 to 1793. He was responsible for introducing
Great Game rivalry between Britain and Russia for
a number of reforms, including the Permanent
Central Asian influence.
Settlement of Bengal.
• The Second Anglo-Afghan War (1878-80) was also
• Lord Wellesley was the Governor-General of India
fought because of the British fear of Russian expansion.
from 1798 to 1805. He was responsible for expanding
The British invaded Afghanistan again and forced the
the British Empire in India.
Afghans to accept a British protectorate. The British
• Lord William Bentinck was the Governor-General secured their objectives with the Treaty of Gandamak
of India from 1828 to 1835. He was responsible for a and a decisive victory over Ayub Khan outside
number of reforms, including the abolition of sati and Kandahar. Abdur Rahman Khan, chosen by the British,
the introduction of English education in India. confirmed the treaty, establishing a buffer between the
British Raj and the Russian Empire.
46. Solution: (d)
• The Third Anglo-Afghan War (1919) was the last
Exp) Option d is the correct answer. of the Anglo-Afghan wars. It was fought because the
Maharaja Ranjit Singh, widely recognized as Sher-e-Punjab, Afghans wanted to regain their independence from
emerged as the prominent figure at the helm of the Sikh the British. The British were defeated in this war, and
Empire, a dominion that held sway over the northwestern Afghanistan was finally able to assert its independence.
region of the Indian subcontinent during the first half It ended in an armistice. The Anglo-Afghan Treaty of
of the 19th century. His realm encompassed a notable 1919 recognized the Durand Line as the Afghanistan-
expanse, incorporating Srinagar, Attock, Peshawar, Bannu, India border, granting Afghans control over foreign
Rawalpindi, Jammu, Gujrat, Sialkot, Kangra, Amritsar, affairs while ensuring border stability.
Lahore, and Multan. 50. Solution: (a)
47. Solution: (c) Exp) Option a is the correct answer.
Exp) Option c is the correct answer. The Battle of Buxar, occurred on 22 October 1764, as a
Vasco da Gama set up a trading factory at Cannanore. confrontation at Buxar in northeastern India. It pitted the
Gradually, Calicut, Cannanore and Cochin became the British East India Company’s forces, led by Major Hector
important trade centres of the Portuguese.They also set Munro, against a combined army comprising an alliance of
Indian states including Bengal, Awadh, and the Mughal
up factory at Hoogly in west Bengal. Gradually, under the
Empire. This crucial battle solidified British authority over
pretext of protecting the factories and their trading activities,
Bengal and Bihar, following their initial triumph at the
the Portuguese got permission to fortify these centres.
Battle of Plassey in 1757, and signified the conclusion of
48. Solution: (b) the endeavor to govern Bengal through a puppet nawab.
Exp) Option b is the correct answer. Subsequently, the company assumed control. The British
triumph at Buxar led to the expansion of British dominion
The founding of the modern city of Kolkata earlier known
over a substantial portion of the Indian subcontinent.
as Calcutta dates back to 1690. The foundation has been
ascribed to Job Charnock, an agent of the British East Important Tips
India Company, who landed in the village of Sutanuti and The Third Battle of Panipat was fought on 14 January
established a British factory there. Present day Kolkata 1761 between the Maratha Empire and the invading
stands laid its foundation originally on these three villages: army of the Durrani Empire. The battle took place in and
Sutanuti, Kalikata and Gobindapur. around the city of Panipat, approximately 97 kilometres
(60 mi) north of Delhi. The Afghans were supported
49. Solution: (d)
by three key allies in India: Najib-ud-Dawlah who
Exp) Option d is the correct answer. persuaded the support of the Rohilla chiefs, elements of
The Anglo-Afghan relations in the 19th century were the declining Mughal Empire, and most prized the Oudh
influenced by the fear of Russian expansion towards India. State under Shuja-ud-Daula. The Maratha army was led
This was known as the “Great Game”. The British were by Sadashivrao Bhau, who was third-highest authority
concerned that the Russians would try to conquer India, and of the Maratha Confederacy after the Chhatrapati and
the Peshwa.
they used Afghanistan as a buffer state to prevent this.

PYQ Workbook 18
HISTORY OF MODERN INDIA

XIV, the efforts of his minister Colbert resulted in the


The battle was a decisive victory for the Afghans. The
formation of the French trading company
Maratha army was routed and Sadashivrao Bhau was killed
in battle. The defeat of the Marathas at the Third Battle of 54. Solution: (d)
Panipat marked the end of their expansionist ambitions Exp) Option d is the correct answer.
and led to a decline in their power. The Afghans, under
Of all the European nations who came as traders to India after
Ahmad Shah Durrani, emerged as the dominant power in
new sea routes were discovered, England emerged as the most
the north of India.
powerful and successful by the end of the eighteenth century.
51. Solution: (c) The major factors which can be attributed for the success
of the English against other European powers—Portugal, the
Exp) Option c is the correct answer.
Netherlands, France and Denmark—in the world in general
French East India Company was established in the 17th and in India in particular were as follows.
century to oversee French commerce with India, eastern
• The British soldiers were a disciplined lot and they
Africa, and other territories of the Indian Ocean and the East
received western training and had modern arms.
Indies. It was established by Jean-Baptiste Colbert, finance
minister to King Louis XIV. • Indian Soldiers lacked fidelity consequently the one
who paid them sufficient could have them on his
Important Tips service.
French in India • With the exception of the Glorious Revolution of 1688,
• In 1668, the first French factory was established in Britain witnessed stable government with efficient
Surat. monarchs.
• In February, 1701, Pondicherry was made the capital of • There was absence of Nationalism in India.
the French settlements in India.
55. Solution: (b)
• French occupied Mahe in Malabar, Yanam in
Exp) Option b is the correct answer.
Coromandel and Karaikal in Tamil Nadu.
Alfonso de Albuquerque laid the foundation for the
• The arrival of Duplex as French governor saw the
Portuguese in India. He replaced Almeida as a governor in
beginning of Anglo-French conflicts in India.
the year 1509 AD. In the year 1510 AD, he captured Goa
52. Solution: (b) from the Sultan of Bijapur. Alfonso de Albuquerque was
considered as a real founder of the Portuguese power in
Exp) Option b is the correct answer.
India.
Though the British and the French came to India for trading
purposes, they were ultimately drawn into the politics of Important Tips
India. Both had visions of establishing political power over • Vasco Da Gama was the one who discovered the sea
the region. The French failed to establish power in Deccan route to India in 1498.
because they were drawn in war with British in southern • Francisco De Almeida was the first Portuguese Viceroy
states of India and also British had a strong army and base in in India, he constructed the fort Anjediva.
India than the French.
• Alfonzo De Albuquerque set-up his headquarters in
53. Solution: (b) Cochin.
Exp) Option b is the correct answer. • Nino Da Cunha was a Portuguese governor in India
The correct order is 4-2-1-3 from 1528 to 1538. He shifted Portuguese headquarters
from Cochin to Goa.
Portuguese in 1498 - The first Portuguese mission was led
by Vasco da Gama who reached Calicut via the Cape of Good 56. Solution: (a)
Hope in May 1498 where he was favourably received by the Exp) Option a is the correct answer.
local ruler Zamorin.
Francisco de Almeida followed the Blue Water Policy. As per
Dutch in 1602 - The Dutch also established their trade this policy, the Portuguese should be the sole trade power
centres in India. Cornelis de Houtman was the first Dutch in the Arabian sea and the Indian Ocean. It called for the
citizen to arrive in India. The Dutch East India Company Portuguese to be powerful at the sea instead of building
was formed in 1602, They overpowered the Portuguese and fortresses on the Indian mainland. In 1505, ‘Francisco de
established their control over the centres of spice cultivation Almeida’ was elected as the 1st Governor and Viceroy of
in India. They established their trade posts in Gujarat, India. The Blue water policy was reversed by Alfonso de
Bengal, Bihar and Orissa. Albuquerque.
English in 1608 - British emperor James I sent Captain
William Hawkins to the court of Jahangir in 1608, to seek 57. Solution: (d)
permission to establish trading posts in India. Exp) Option d is the correct answer.
French in 1664 - French were the last among the European King Charles II of Britain married Catherine of Braganza
companies to enter India. In 1664, during the reign of Louis in 1661. King “Charles II” was given a huge dowry by the

19 PYQ Workbook
HISTORY OF MODERN INDIA

Portuguese as a marriage treaty. Portugal handed over the • In 1687 Bombay became the capital of the Company’s
city of Bombay to Charles II as a part of that dowry. Portugal territories in India. The area administrated from
handed the city on 3rd July 1661. England acquired the 7 Bombay the Bombay Presidency roughly covers the
Islands of Bombay and Catherine’s dowry of Tangier. modern states of Sindh( Pakistan), Gujrat, and Western
58. Solution: (c) Maharashtra.

Exp) Option c is the correct answer. • The Presidency Includes: Konkan, Nashik, and
Pune divisions of the present-day Indian state of
Lahore was the capital of the Kingdom of Maharaja Ranjit Maharashtra.
Singh. He captured Lahore in 1799 and made it the capital
of his empire. Lahore was a strategically important city, 61. Solution: (b)
located at the crossroads of trade routes between Central Exp) Option b is the correct answer.
Asia, South Asia, and the Middle East. It was also a major
Sir John Child, the English Governor of East India Company
cultural center, with a rich history and heritage.
in India, was expelled by Aurangzeb. Child became involved
Important Tips in a war with the Mughal emperor Aurangzeb, whose troops
Maharaja Ranjit Singh: captured Surat and forced Child to make peace.
In his youthful years, Maharaja Ranjit Singh engaged in 62. Solution: (d)
multiple campaigns aimed at expelling the Afghans, and Exp) Option d is the correct answer.
he ascended to the title of “Maharaja of Punjab” when he
Shah Alam II was the ruler of Delhi at the time of the Battle
reached the age of 21. He was the solitary offspring of Maha
of Buxar (22 October, 1764). Following their victory in the
Singh. His birth transpired on the 13th of November 1780
Battle of Buxar (1764), the Mughal Emperor Shah Alam II
in Gujranwala, which is presently situated in Pakistan.
conferred the Diwani rights upon the East India Company.
The establishment of Patiala as the capital of the princely This granting of Diwani rights, specifically pertaining to
state of Patiala occurred in 1763. Notably, Amritsar, Bengal, occurred in 1765. In essence, Diwani rights signify
founded by Sri Guru Ramdass Ji, the fourth guru of the the authority to manage and collect land revenue.
Sikhs, took shape around 1574 A.D. Moving further,
Kapurthala State, previously under the dominion of 63. Solution: (b)
Ahluwalia Sikh rulers, held the status of a former Princely Exp) Option b is the correct answer.
state within Punjab.
The Battle of Plassey gave the British control of Bengal and
59. Solution: (d) marked the beginning of their gradual consolidation of British
Supremacy over much of India. The British chose Bengal,
Exp) Option d is the correct answer.
particularly Kolkata (Calcutta), due to its strategic location,
Ranjit Singh was the last Misaldar (commander) of the weak local ruler, rich resources, educated population, and
Sukerchakia Misl, which was one of the 12 Sikh misls that potential for trade. The region’s fertile land, abundant supply
existed in Punjab in the 18th century. The Sukerchakia Misl of resources, and access to the Ganges River facilitated trade
was founded by Charat Singh, the grandfather of Ranjit and business. The Battle of Plassey was fought on 23 June
Singh. Ranjit Singh eventually united all the Sikh misls and 1757 between the forces of the Nawab of Bengal, Siraj ud-
established the Sikh Empire. Daulah, and the British East India Company, led by Robert
Important Tips Clive. The British victory in the Battle of Plassey marked the
beginning of British rule in India. The battle was won by the
Facts abot Misl:
British due to a number of factors, including the betrayal of
• The Aahluvaliyan Misl was led by Jassa Singh Mir Jafar, the commander of the Nawab’s army, the superior
Ahluwalia. military training and equipment of the British forces, and
• The Dalevaliyan Misl was led by Hari Singh Dhillon. the support of the local Hindu population.
• The Kanhaiya Misl was led by Jai Singh Kanhaiya.
Important Tips
60. Solution: (a) Battle of Buxar- It occurred on October 22-23, 1764. It
Exp) Option a is the correct answer. involved the British East India Company, led by Hector
Munro, facing the combined forces of Mir Qasim, the
The first British settlement in the western Presidency was
in 1618 when the East India Company established a factory Nawab of Bengal; Shuja-ud-Daula, the Nawab of Awadh;
at Surat. The British were protected by a charter from the and the Mughal Emperor Shah Alam II. Mir Qasim
Mughal Emperor Jahangir. suffered defeat from the British.
3rd Battle of Mysore- The Third Anglo-Mysore War took
Important Tips
place in South India, involving the Kingdom of Mysore
British in India: and various forces including the British East India
• The East India Company transferred its Western India Company, the Kingdom of Travancore, the Maratha
Headquarter from Surat to Bombay in 1687. Then Empire, and the Nizam of Hyderabad. British won the
Bombay presidency came into existence. war and Treaty of Srirangapatnam was signed.

PYQ Workbook 20
HISTORY OF MODERN INDIA

Independence Struggle of 1857- Also known as the • Initiated British rule in India and established their
Sepoy Mutiny or the First War of Independence, it was a political influence.
major uprising against British rule in India that began on Diwani Rights Granted:
10 May 1857. The rebellion was triggered by a number of
• Under the treaty terms, Shah Alam granted the East
factors, including the introduction of new cartridges that
were rumored to be greased with cow and pig fat, which India Company Diwani rights.
was offensive to both Hindu and Muslim soldiers • Diwani rights allowed the Company to collect taxes on
behalf of the emperor from the eastern provinces of
64. Solution: (b) Bengal, Bihar, and Orissa.
Exp) Option b is the correct answer. • It gave the Company direct authority to collect revenue
Kharak Singh was the eldest legitimate son of Maharaja from the people of these regions.
Ranjit Singh. He was proclaimed Maharaja of the Punjab Exchange of Benefits:
on June 27, 1839, after the death of his father. However,
• In exchange for Diwani rights, the East India Company
Kharak Singh was a weak and indecisive ruler, and he was
paid an annual tribute of 26 lakh rupees to the emperor.
soon challenged by his younger half-brother, Sher Singh. In
October 1839, Kharak Singh was deposed and imprisoned • The Company secured the districts of Kada and
by Sher Singh. He died in prison in November 1840. Allahabad for Shah Alam II as part of the arrangement.

65. Solution: (b) 67. Solution: (c)


Exp) Option b is the correct answer. Exp) Option c is the correct answer.
Sir Eyre Coote was the British general who defeated Haider The Portuguese king Manuel I made Francisco de Almeida
Ali in the War of Porto Novo. The Battle of Porto Novo was viceroy of the newly conquered territories of India in March
fought on 1 July 1781 between forces of the Kingdom of 1505. Almeida was made First Viceroy in India because of his
Mysore and British East India Company in the place called achievement in the wars against the Moors.
Porto Novo (now known as Parangipettai) on the Indian
subcontinent, during the Second Anglo-Mysore War. The 68. Solution: (b)
British force, numbering more than 8,000 men under the Exp) Option b is the correct answer.
command of Sir Eyre Coote defeated a force estimated at
In 1612 the East India Company’s ships defeated the
40,000 under the command of Hyder Ali.
Portuguese in the Sowlley estuary off Surat. The naval
66. Solution: (a) Battle of Sowlley, also known as Battle of Suvali, took place
Exp) Option a is the correct answer. on 29–30 November 1612 off the coast of Suvali a village
near the Surat city and was a victory for four English East
Mohammed Raza Khan was appointed as the Deputy
India Company galleons over four Portuguese galleons.
Diwan of Murshidabad by Robert Clive after the Allahabad
Thomas best led the British forces in this decisive war.
Treaty. The Allahabad Treaty was signed in 1765 between
the East India Company and the Mughal Emperor Shah This relatively small naval battle is historically important as
Alam II. The treaty gave the East India Company the right to it marked the beginning of the ascent of the English East
collect revenue from Bengal, Bihar and Orissa. Mohammed India Company’s presence in India.
Raza Khan was a trusted advisor of Robert Clive and he 69. Solution: (d)
was instrumental in helping the British to consolidate their
power in Bengal. Exp) Option d is the correct answer.
Joseph Francis Dupleix the French governor of Chandarnagar
Important Tips
was not in favor of violence or war with English. At that time
Treaty of Allahabad: Commodore Barnet, the English commander captured some
• Signed in Allahabad on August 16, 1765. French ship. This was the immediate cause of war.
• Scripted by I’tisam-ud-Din, a Bengali Muslim scribe Important Tips
and Mughal Empire diplomat.
• The First Carnatic War was an extension of the
• Involved the Mughal Emperor Shah Alam II and Anglo-French War in Europe which was caused by the
Robert Clive of the East India Company. Austrian War of Succession.
• Resulted from the British victory over Shah Alam • The war was concluded with the Peace treaty of Aix-la-
and the Nawabs of Awadh and Bengal in the Battle of Chapelle, signed in October 1748
Buxar.
• The First Carnatic War is remembered for the Battle
Impact of the Treaty: of St. Thome (in Madras) fought between the French
• Marked the beginning of British political and forces and the forces of Anwar-ud-din, the Nawab of
constitutional involvement in Indian affairs. Carnatic, to whom the English appealed for help

21 PYQ Workbook
HISTORY OF MODERN INDIA

70. Solution: (c)


• Engaged in the Maratha–Mysore War and signed the
Exp) Option c is the correct answer. Treaty of Gajendragad.
The Treaty of Srirangapatna was signed on 18 March 1792, • Forced into the Treaty of Seringapatam during the
between the British East India Company and Tipu Sultan, Third Anglo-Mysore War, losing previously conquered
the ruler of the Kingdom of Mysore. The treaty ended the territories.
Third Anglo-Mysore War. Under the terms of the treaty, • Defeated in the Fourth Anglo-Mysore War by a
Tipu Sultan agreed to cede half of his kingdom to the British, combined British, Maratha, and Nizam force in 1799.
including the districts of Malabar, Coimbatore, and Salem. Contributions and Conflicts:
He also agreed to pay a large indemnity to the British. In
• Made endowments to Hindu temples, including
return, the British agreed to recognize Tipu Sultan as the
Ranganathaswami Temple at Srirangapatna.
ruler of the remaining half of his kingdom.
• Appointed Hindu officers and provided land grants to
71. Solution: (a) Hindu temples, showcasing religious tolerance.
Exp) Option a is the correct answer. Implemented Reforms:
Ranjit Singh got the famous Kohinoor diamond (along • Tippu is recognized for pioneering the use of iron-
with another diamond Darya-i-noor and Timur Ruby) cased rockets in warfare.
from Shah Shuja Durrani, the former king of Afghanistan. • In the Anglo-Mysore Wars, he introduced modern
Shah Shuja was forced to flee to Lahore in 1813 after being war rockets (though some sources attribute the initial
overthrown by his brother, Dost Mohammad. Ranjit Singh introduction to his father, Hyder Ali, and Tippu
offered Shah Shuja asylum in Lahore, but on the condition improved upon them).
that he give up the Kohinoor diamond. Shah Shuja agreed, Coinage System:
and the Kohinoor diamond became one of the most prized • Under Haidar Ali, coins featured pagodas with Persian
possessions of the Sikh Empire. inscriptions in the name of Mughal Emperor Shah
Alam II.
72. Solution: (a)
• Tippu Sultan continued the issuance of pagodas,
Exp) Option a is the correct answer.
mohurs, and rupees, with entirely new inscriptions.
Tipu Sultan, the ruler of the Kingdom of Mysore, set up
• In copper coinage, Haidar Ali initiated the larger
his capital at Srirangapatna. Srirangapatna is located on
paisa, which Tippu Sultan expanded by introducing
an island in the Cauvery River, in the Mandya district of
additional denominations.
Karnataka, India. It was a strategically important location,
Land Revenue and Sericulture:
as it was surrounded by water on three sides. Srirangapatna
was also a well-fortified city, making it difficult to attack. • Tippu introduced a novel land revenue system in
Mysore.
Important Tips • He also played a role in promoting sericulture, a
Tippu Sultan: practice that still provides employment for many
• Born as Sultan Fateh Ali Sahab Tippu on November Kannadigas.
10, 1750, in Devanahalli (present-day Bangalore).
73. Solution: (c)
• Son of Hyder Ali, who had risen in the ranks of the
Wodeyar dynasty’s army, rulers of Mysore. Exp) Option c is the correct answer.

• Earned the nickname “Tiger of Mysore.” Statement 1 is correct- Agency Houses were important
commercial formations in the late eighteenth and
Military Innovations and Anglo-Mysore Wars:
nineteenth century India. They were essentially private
• Expanded iron cased Mysorean rockets and companies that provided a range of services to the British
commissioned the military manual Fathul Mujahidin. East India Company (EIC), including banking, insurance,
• Utilized rockets against British and allied forces and shipping.
during the Anglo-Mysore Wars. Statement 2 is correct- The operation of managing agencies
• Fought in battles like the Battle of Pollilur and Siege of remained confined to Calcutta, which was the capital of
Srirangapatna. the EIC. This was because Calcutta was the main center
• Tipu sent an embassy to Constantinople and another of British trade and commerce in India. The managing
in 1787 to Paris to strengthen his position against the agencies were able to operate effectively in Calcutta because
English. they had access to a large pool of financial resources and
Treaties and Conflicts: skilled manpower.
• Negotiated the 1784 Treaty of Mangalore with the 74. Solution: (c)
British, ending the Second Anglo-Mysore War.
Exp) Option c is the correct answer.

PYQ Workbook 22
HISTORY OF MODERN INDIA

Statement 1 is correct- The Treaty of Amritsar was signed in Statement 1 is correct: British did rely on Indian Pandits
1809 between the British East India Company (represented and Maulavis for understanding the Hindu and Muslim
by Charles T. Metcalfe) and Maharaja Ranjit Singh, the laws respectively. They appointed them as court advisers and
ruler of the Sikh Empire. The treaty fixed the Sutlej River as translators of the ancient texts.
the boundary between the Sikh Empire and British India. It Statement II is incorrect: British did not codify the Hindu
also allowed the British to maintain a presence in the Punjab, and Muslim laws in 1783 and 1785 respectively. The actual
which was seen as a strategic buffer between the British and codification of Hindu and Muslim laws took place much later,
the Afghans. in the 19th century, through a series of legislative acts and
Statement 2 is correct- The treaty helped Maharaja Ranjit judicial decisions. The Charter Act of the British Parliament
of 1833 established a Law Commission to unify and codify
Singh to consolidate his power in the Punjab. After the
Indian laws, which it achieved between 1862 and 1872 with
treaty, he was able to annex Jammu, Multan, and Kashmir
the Indian Divorce Act of 1869, the Hindu Wills Act of 1870,
to his domains. These annexations made the Sikh Empire
the Special Marriage Act of 1872, the Indian Evidence Act
one of the most powerful states in South Asia
of 1872, the Christian Marriage Act of 1872, The Married
75. Solution: (a) Women’s Property Act 1874; The Indian Majority Act of
1875 and Divorces Registration Act 1876 all came out of this.
Exp) Option a is the correct answer.
With the implementation of the Avadh Laws Act in 1876,
Statement 1 is correct: In 1856, Nawab Wajid Ali Shah modifications were also introduced to Muslim personal law.
was dethroned and exiled to Calcutta on the plea that the The Kazis Act 1881, the Transfer of Property Act 1882, the
region was being misgoverned. Nawab Wajid Ali Shah was Guardians and Wards Act 1890, and the Bengal Protection
the last ruler of Awadh, a princely state in northern India. of Mohammadan’s Pilgrim Act 1896 were all key legislation
He was deposed by Lord Dalhousie, the Governor-General of that followed.
India, who accused him of neglecting the administration and
Important Tips
welfare of his subjects. The British East India Company then
annexed Awadh and brought it under its direct rule. This • The Calcutta Madrasa was founded by Warren
was one of the causes of the revolt of 1857, as many people in Hastings in 1781 for the study of Muslim law and
Awadh resented the British interference and injustice. related subjects, such as Persian and Arabic languages.

Statement II is correct: The Nawab was accused of being • The Sanskrit College was established by Jonathan
unable to control the rebellious chiefs and Taluqdars. The Duncan, the resident at Benaras, in 1791 for the study
chiefs and Taluqdars were the local landlords and nobles who of Hindu law and philosophy, as well as the Sanskrit
had considerable power and influence in Awadh. They often language.
defied the authority of the Nawab and resisted his attempts 78. Solution: (d)
to reform the revenue system and curb their privileges. The
Exp) Option d is the correct answer.
British used this as a pretext to justify their annexation of
Banaras and the surrounding tracts were not handed over to
Awadh, claiming that they would restore law and order and
Shah Alam II, but to Balwant Singh, the zamindar of Banaras.
protect the rights of the peasants.
Balwant Singh of Benares was granted these territories by the
Statement II is also the correct explanation of Statement I. East India Company as a part of the Treaty of Allahabad in
The British allegation of misgovernance against Nawab Wajid 1765. He later transferred them to Shah Alam II in 1775
Ali Shah was based on his supposed inability to control the
rebellious chiefs and Taluqdars, who posed a threat to the 79. Solution: (d)
stability and security of Awadh. The British argued that they Exp) Option d is the correct answer.
had to intervene and take over Awadh to ensure its proper The First Anglo-Maratha War was concluded by The Treaty
administration and development. of Salbai. The Treaty of Salbai, signed in 1782, brought an
end to the First Anglo-Maratha War. The treaty was signed
76. Solution: (c)
between the British East India Company and the Maratha
Exp) Option c is the correct answer. Empire, specifically the Peshwa (prime minister) of the
Dhaka was a leading commercial centre in Bengal that Marathas. The treaty established peace between the two
flourished in the seventeenth century due to its trade in parties and defined the territorial boundaries and terms of
cotton textiles, silk, muslin, saltpetre, sugar, etc. It had a engagement.
cosmopolitan population of merchants, artisans, bankers Important Tips
and weavers from different regions and communities. It also
• The Treaty of Surat: It was a peace treaty signed
had a thriving export market with Europe, West Asia and
between the British East India Company and the
Southeast Asia. However, Dhaka declined in the eighteenth
Marathas in 1775. It was an attempt to bring a
century when the British became the dominant political and
temporary cessation of hostilities during the First
economic power in India after the Battle of Plassey in 1757.
Anglo-Maratha War. However, the Treaty of Surat did
77. Solution: (c) not mark the conclusion of the war but rather a brief
pause in the fighting.
Exp) Option c is the correct answer.

23 PYQ Workbook
HISTORY OF MODERN INDIA

• The Treaty of Purandar: It was signed in 1776 between Siyar-ul-Mutakherin was written by Ghulam Husain Khan
the British East India Company and the Marathas. It Tabatabai, an Indian historian from the 18th century Bengal.
followed the Battle of Wadgaon, where the British He was an 18th-century Persian historian who served in
suffered a significant defeat. The treaty established the court of Nawab Siraj-ud-Daulah, the last independent
peace between the two sides and led to the restoration Nawab of Bengal.
of territories to the Marathas. Important Tips
• The Convention of Wadgaon took place in 1779 during The Siyar-ul-Mutakhkherin is in three volumes:
the First Anglo-Maratha War. It was a significant event
• Volume I gives an account of the geography, climate,
where the Marathas, led by their commander-in-chief
animals, forts, saints and Ulama of Hindustan, and
Mahadji Shinde, achieved a decisive victory over the
briefly surveys its history from the epic age to the reign
British forces.
of aurangzeb.
80. Solution: (c) • Volume II discusses the history of Bengal from the
Exp) Option c is the correct answer. death of Aurangzeb and deals with the nizamat of
Murshidabad, the relations of the English with the
The Dastak was a trade permit or pass issued by the local
nawabs, the battle of Palashi, and establishment of
authorities, granting the European traders permission to
English supremacy in Bengal and Bihar together with
conduct their trade within a specific region or territory. The
British administrative system.
Dastak served as a form of authorization and protection
for the European merchants, ensuring their safety and • Volume III discusses the history of the Nizam of
facilitating their commercial transactions. Haidarabad, Haidar Ali of Mysore, Nawab-Wazirs
of Oudh, Emperor Muhammad Shah, Ahmad Shah
81. Solution: (a) Abdali’s invasions and his wars with the Marathas.
Exp) Option a is the correct answer. The volume comes to an end after the affairs of the
The Dadni system was a phase of business management of English upto 1781 AD.
the English East India Company in Bengal in the eighteenth 84. Solution: (b)
century. The company used to engage local merchants to
procure goods, especially textiles, from the market on its Exp) Option b is the correct answer.
behalf. They were called dadni-merchants, because they Statement 1 is correct: Daud Khan Panni and Saadatullah
received advances from the company for delivering goods Khan were the founders of the dynasty that ruled Arcot, a
under stipulated terms. princely state that was established by the Mughal emperor
The Dadni system played a crucial role in promoting textile Aurangzeb in 1692 and later became a protectorate of the
production and craftsmanship in regions where it was British East India Company.
prevalent. Statement 2 is correct: Arcot became the site of a protracted
struggle between the English and Dutch East India
82. Solution: (b)
Companies from the 1740s, as part of their rivalry for trade
Exp) Option b is the correct answer. and influence in India.
British policy towards Afghanistan in the first half of the Statement 3 is incorrect: The State of Arcot in the 18th
19th century aimed at: weakening Russian influence in century was characterized by centralization rather than
Afghanistan but not promoting a strong Afghanistan decentralization. The rulers of Arcot sought to consolidate
either. their power and establish a strong central authority.
• The British were concerned that Russia would attack Statement 4 is correct: The other major State to emerge in
India via Afghanistan and the country’s northwestern South India at this time was Mysore. In the 18th century,
border, and wanted to keep Russia at a safe distance from the Kingdom of Mysore, under the leadership of rulers like
the Indian border. Hyder Ali and Tipu Sultan, emerged as a significant power
• The British adopted a forward policy, which implied in South India.
that they had to take initiatives to protect the boundary
of British India from a probable Russian attack, either 85. Solution: (a)
through treaties with the neighbouring countries or by Exp) Option a is the correct answer.
annexing them completely. Statement 1 is correct: Job Charnock arrived in Sutanati in
• The British also wanted to weaken and end Russian August 1690 and laid the foundation of Calcutta, which
influence in Afghanistan, but they did not want a strong later became the heart of the British Indian empire. Job
Afghanistan. They wanted to keep her a weak and Charnock is considered one of the founders of Calcutta (now
divided country which they could easily control. Kolkata) and played a significant role in establishing British
presence in Bengal.
83. Solution: (c)
Statement 2 is incorrect: The French East India Company
Exp) Option c is the correct answer.
did not build a fort near Fort William in Calcutta. Fort

PYQ Workbook 24
HISTORY OF MODERN INDIA

William was built by the British East India Company in their establishment of control over Bengal and other parts
the 18th century and served as a major British stronghold of India.
in Bengal. Statement 2 is correct: The Treaty of Allahabad was signed
86. Solution: (c) on 16 August 1765 between the Mughal Emperor Shah
Alam II and Robert Clive of the East India Company, in the
Exp) Option c is the correct answer.
aftermath of the Battle of Buxar. It granted the Company the
Statement 1 is correct: The Battle of Buxar, which took place diwani rights, or the right to collect revenue, in Bengal,
in 1764, was a decisive battle in the Indian subcontinent. Bihar, and Odisha. This effectively gave the British control
The victory of the British East India Company in this battle over these regions and marked a significant step in their
provided them with a key advantage and paved the way for establishment of political power in India.

25 PYQ Workbook
HISTORY OF MODERN INDIA

HISTORY OF MODERN INDIA


REBELLIONS AND REFORM MOVEMENTS
*This unit consists of questions from the Revolt of 1857, the Peasant and Trible Movements and the
Socio-Relegious Reform Movements.

2.1. UPSC CSE Previous Years’ Questions 6. After the Santhal Uprising subsided, what
was/were the measure/measures taken by
1. Who among the following was associated as the colonial government?
Secretary with Hindu Female School which [UPSC CSE Pre. 2018]
later came to be known as Bethune Female 1. The territories called ‘Santhal Parganas’
School? [UPSC CSE Pre. 2021] were created.
(a) Annie Besant 2. It became illegal for a Santhal to transfer
(b) Debendranath Tagore land to a non-Santhal.
(c) Ishwar Chandra Vidyasagar Select the correct answer using the code given
(d) Sarojini Naidu below:
2. The Self-Respect Movement was initiated (a) 1 only
by [UPSC CSE Pre. 2021] (b) 2 only
(a) B.R. Ambedkar (c) Both 1 and 2
(b) Jyotiba Phule (d) Neither 1 nor 2
(c) E.V. Ramaswamy Naicker 7. Which among the following events
(d) Jawaharlal Nehru happened earliest? [UPSC CSE Pre. 2018]
3. In the context of Indian history, the (a) Swami Dayanand established Arya Samaj.
Rakhmabai case of 1884 revolved around (b) Dinabandhu Mitra wrote Neel Darpan.
[UPSC CSE Pre. 2020] (c) Bankim Chandra Chattopadhyay wrote
1. women’s right to gain education Anandmath.
(d) Satyendranath Tagore became the first
2. age of consent Indian to succeed in the Indian Civil
3. restitution of conjugal rights Services Examination.
Select the correct answer using the code
8. The Trade Disputes Act of 1929 provided
given below:
for [UPSC CSE Pre. 2017]
(a) 1 and 2 only
(a) the participation of workers in the
(b) 2 and 3 only
management of industries.
(c) 1 and 3 only
(b) arbitrary powers to the management to
(d) 1, 2 and 3
quell industrial disputes.
4. Indigo cultivation in India declined by the (c) an intervention by the British Court in
beginning of the 20th century because of the event of a trade dispute.
[UPSC CSE Pre. 2020] (d) a system of tribunals and a ban on strikes.
(a) peasant resistance to the oppressive 9. Satya Shodhak Samaj organized
conduct of planters [UPSC CSE Pre. 2016]
(b) its unprofitability in the world market (a) a movement for upliftment of tribals in
because of new inventions Bihar
(c) national leaders’ opposition to the (b) a temple-entry movement in Gujarat
cultivation if indigo (c) an anti-caste movement in Maharashtra
(d) Government control over the planters (d) a peasant movement in Punjab
5. With reference to the history of India, 10. Consider the following:
“Ulgulan” or the Great Tumult is the 1. Calcutta Unitarian Committee
description of which of the following
events? [UPSC CSE Pre. 2020] 2. Tabernacle of New Dispensation
(a) The Revolt of 1857 3. Indian Reform Association
(b) The Mappila Rebellion of 1921 Keshav Chandra Sen is associated with the
(c) The Indigo Revolt of 1859-60 establishment of which of the above?
(d) Birsa Munda’s Revolt of 1899-1900 [UPSC CSE Pre 2016]

PYQ Workbook 26
HISTORY OF MODERN INDIA

(a) 1 and 3 only (a) Introduction of a new system of land


(b) 2 and 3 only revenue and taxation of tribal products.
(c) 3 only (b) Influence of foreign religious missionaries
(d) 1, 2 and 3 in tribal areas.
(c) Rise of a large number of money
11. The demand for the Tebhaga Peasant lenders, traders and revenue farmers as
Movement in Bengal was for middlemen in tribal areas.
[UPSC CSE Pre. 2013] (d) The complete disruption of the old
(a) the reduction of the share of the landlords agrarian order of the tribal communities.
from one-half of the crop to one-third
15. Which one of the following places was
(b) the grant of ownership of land to peasants associated with Acharya Vinoba Bhave’s
as they were the actual cultivators of the Bhoodan Movement at the beginning of the
land movement? [UPSC CSE Pre. 2007]
(c) the uprooting of Zamindari system and (a) Udaygiri
the end of serfdom (b) Raipur
(d) writing off all peasant debts (c) Pochampalli
12. Which of the following statements is/are (d) Venkatagiri
correct regarding Brahmo Samaj? 16. With reference to the ‘revolt of 1857’ who
[UPSC CSE Pre. 2012] of the following was betrayed by ‘friend’
1. It opposed idolatry. captured and put to death by the British?
2. It denied the need for a priestly class for [UPSC CSE Pre. 2006]
interpreting the religious texts. (a) Nana Sahib
3. It popularized the doctrine that the Vedas (b) Kunwar Singh
are infallible. (c) Khan Bahadur Khan
Select the correct answer using the codes (d) Tatya Tope
given below: 17. Which one of the following revolts was made
(a) 1 only famous by Bankim Chandra Chatterjee in
(b) 1 and 2 only his novel ‘Anand Math’?
(c) 3 only [UPSC CSE Pre. 2006]
(d) 1, 2 and 3 (a) Bhil uprising
(b) Rangpur and Dinajpur uprising
13. During Indian freedom struggle, the
(c) Bishnupur and Birbhum rebellion
National Social Conference was formed. (d) Sanyasi rebellion
What was the reason for its formation?
[UPSC CSE Pre. 2012] 18. Who was the Governor-General of India
(a) Different social reform groups or during the Sepoy Mutiny?
organizations of Bengal region united [UPSC CSE Pre. 2006]
to form a single body to discuss the (a) Lord Canning
issues of larger interest and to prepare (b) Lord Dalhousie
appropriate petitions/representations to (c) Lord Hardings
the government (d) Lord Lytton
(b) Indian National Congress did not want to 19. Who among the following was not associated
include social reforms in its deliberations with the formation of U.P. Kisan Sabha in
and decided to form a separate body for February 1918? [UPSC CSE Pre.2005]
such a purpose (a) Indra Narayan Dwivedi
(c) Behramji Malabari and M. G. Ranade (b) Gauri Shankar Misra
decided to bring together all the social (c) Jawaharlal Nehru
reform groups of the country under one (d) Madan Mohan Malviya
organization
20. Which one of the following territories was
(d) None of the statements (a) , (b) and (c) not affected by the Revolt of 1857?
given above is correct in this context.
[UPSC CSE Pre.2005]
14. Which amongst the following provided a (a) Jhansi
common factor for tribal insurrection in (b) Chittor
India in the 19th century? (c) Jagdishpur
[UPSC CSE Pre. 2011] (d) Lucknow

27 PYQ Workbook
HISTORY OF MODERN INDIA

21. Which one of the following places did 25. M.C. Setalvad, B.N. Rao, and Alladi
Kunwar Singh, a prominent leader of the Krishnaswamy Iyer were distinguished
Revolt of 1857 belong to? members of the: [UPSC CSE Pre. 1997]
[UPSC CSE Pre. 2005] (a) Swaraj Party
(a) Bihar (b) All India National Liberal Federation
(b) Madhya Pradesh (c) Madras Labour Union
(c) Rajasthan (d) Servants of India Society
(d) Uttar Pradesh
26. Consider the following passage:
22. Consider the following statements: His principle forte was social and religious
[UPSC CSE Pre. 2001] reform. He relied upon legislation to do away
1. Arya Samaj was founded in 1835. with social ills and worked unceasingly for
2. Lala Lajpat Rai opposed the appeal of the eradication of child marriage and the
Arya Samaj to the authority of Vedas in purdah system. To encourage consideration
support of its social reform programme. of social problems on a national scale, he
3. Under Keshab Chandra Sen, the Brahmo inaugurated the Indian National Social
Samaj campaigned for women’s education Conference, which for many years met for
4. Vinoba Bhave founded the Sarvodaya its annual sessions alongside the Indian
Samaj to work among refugees. National Congress. Their reference in this
Which of these statements are correct? passage is to: [UPSC CSE Pre 1996]
(a) 1 and 2 (a) Ishwar Chandra Vidyasagar
(b) 2 and 3 (b) Behramji Merwanji Malabari
(c) 2 and 4 (c) Mahadev Govind Ranade
(d) 3 and 4 (d) B.R. Ambedkar
23. Consider the following events: 27. The word Adivasi was used for the first time
1. Indigo Revolt to refer to the tribal people by:
2. Santhal Rebellion [UPSC CSE Pre. 1995]
3. Deccan Riot (a) Mahatma Gandhi
4. Mutiny of the Sepoys (b) Thakkar Bappa
(c) Jyotiba Phule
The correct chronological sequence of these
(d) B.R. Ambedkar
events is: [UPSC CSE Pre. 1999]
(a) 4, 2, 1, 3 28. Who among the following was the leader of
(b) 4, 2, 3, 1 some anti-British revolts in Sambhalpur?
(c) 2, 4, 3, 1 [UPSC CSE Pre.1994]
(d) 2, 4, 1, 3 (a) Utirat Singh
24. Match list-I with list-II and select the (b) Surendra Sai
correct answer using the codes given below (c) Kattabomman
the lists: (d) Sayyid Ahmad Barelvi

List-I List-II 29. Whose main aim was to uplift the backward
classes? [UPSC CSE Pre. 1993]
A. Moplah revolt 1. Kerala
(a) Prarthana Samaj
B. Pabna revolt 2. Bihar (b) Satyashodhak Samaj
C. Eka Movement 3. Bengal (c) Arya Samaj
D. Birsa Munda revolt 4. Oudh (d) Ramakrishna Mission
Code: [UPSC CSE Pre. 1997]
2.2. Other Examination Previous Year’s
A B C D Questions
(a) 1 3 4 2
(b) 2 3 4 1 30. Match list-I with list-II and select the
(c) 1 2 3 4 correct answer using the codes given below
(d) 3 4 1 2 the lists:

PYQ Workbook 28
HISTORY OF MODERN INDIA

List-I List-II Place Leadership


(Persons) (Associated works/
Posts) (a) Sambhalpur Surender Sahi
A. D.K. Karve 1. Establishment of (b) Ganjam Radhakrishna
Girls School in Dandsena
Calcutta (c) Kashmir Gulab Singh
B. J.E.D. Bethune 2. Secretary, Widow
Re-marriage (d) Lucknow Liyaqat Ali
Association
34. Which of the above pairs is NOT correctly
C. Ishwar Chandra 3. Starting a crusade
matched? [UPPCS (Pre) 2018]
Vidyasagar against child
marriage Rebellion Year
D. B.M. Malabari 4. Principal of (a) Santhal 1855
Sanskrit College in (b) Kol 1831
Calcutta (c) Khasi 1829
Code: [UPPCS (Pre) 2022]
(d) Ahom 1815
A B C D
(a) 2 1 4 3 35. Which one the following social reformers
strongly opposed Jury Act of 1826?
(b) 1 2 3 4
(c) 1 2 4 3 [U.P.P.C.S. (Pre) 2017]
(d) 2 1 3 4 (a) Ishwar Chandra Vidyasagar
(b) Raja Ram Mohan Roy
31. Which one of the following is correctly (c) Mahadev Govind Ranade
matched? [UPPCS (Pre) 2022] (d) Rajnarayan Basu
(a) Bharat Dharma Mahamandal - Delhi 36. Who has written ‘Satyarth Prakash’?
(b) Deva Samaj - Banaras [U.P.P.C.S. (Mains) 2017]
(c) Radhaswami Satsang - Lahore (a) Raja Ram Mohan Roy
(d) Sanatan Dharma Rakshini Sabha - (b) Bal Gangadhar Tilak
Calcutta (c) Swami Vivekananda
(d) Dayanand Saraswati
32. Match list-I with list-II and select the
correct answer using the codes given below 37. Which of the above following was started in
the lists: opposition to the religious/social ideas of
Ram Mohan Roy?
List-I List-II [U.P.P.C.S. (Mains) 2017]
(Movement) (Year) (a) Digdarshan
A. Pabna rebellion 1. 1855-56 (b) Samachar Chandrika
B. Eka Movement 2. 1873-85 (c) Samvad Kaumudi
(d) Bengal Gazette
C. Santhal Rebellion 3. 1922
38. With reference to “Santhal Hool” of 1855,
D. Tana Bhagat 4. 1914 which of the following statement/s is/are
Movement correct? Select the correct answer from the
Code: [UPPCS (Pre) 2019] codes given below:
A B C D 1. Major Burrough lost the battle with
(a) 1 2 4 3 Santhals near Bhagalpur.
(b) 2 3 1 4 2. Gokko was an important leader of Godda.
(c) 3 1 4 2 3. Mahajan Deen Dayal Rai is also an
important name in this context.
(d) 4 3 2 1
4. There was a time when Santhals enjoyed
33. Which of the followings pairs is NOT full sovereignty over the area of Ganga
correctly matched? [UPPCS (Pre) 2018] valley near Muzaffarpur.

29 PYQ Workbook
HISTORY OF MODERN INDIA

Codes: [U.P.P.C.S. (Mains) 2017] 46. In which State was the Bhoodan Movement
(a) 1 only first started? [UPPCS (Mains) 2013]
(b) 1, 3 and 4 only (a) In Andhra Pradesh
(c) 1, 2 and 3 only (b) In Karnataka
(d) 2 and 3 only (c) In Tamil Nadu
39. Whose name, amongst the following is not (d) In Uttar Pradesh
associated with the Revolt of 1857? 47. Who among the following prominently
[U.P.P.C.S. (Mains) 2017] fought for and got widow remarriage
(a) Col. St. Legar legalized? [U.P.P.C.S. (Mains) 2012]
(b) Lt. Col. Gibbs (a) Annie Besant
(c) Col. Wallace (b) Ishwar Chandra Vidyasagar
(d) All of the above (c) M.G. Ranade
40. Who amongst the following was the leader (d) Raja Ram Mohan Roy
of the 1857 uprising at Allahabad? 48. Who among the following had played
[U.P.P.C.S. (Mains) 2015] an important role in passing the Native
(a) Nana Saheb Marriage Act of 1872?
(b) Azimullah [U.P.P.C.S. (Mains) 2011]
(c) Tatya Tope (a) Devendra Nath Tagore
(d) Maulvi Liyaqat Ali (b) Ishwar Chandra Vidyasagar
41. The Tarkeshwar Movement of 1924 in (c) Keshav Chandra Sen
Bengal was against: (d) Shyam Chandra Das
[U.P.P.C.S. (Pre) (Re. Exam) 2015] 49. The statement “the so-called first national
(a) Corruption in temples war of independence was neither first, nor
(b) Violence national, nor a war of independence’ is
(c) Arrest of political leaders associated with- [U.P.P.C.S. (Mains) 2010]
(d) Communalism (a) R.C. Majumdar
42. ‘Hali System’ concerned: (b) S. N. Sen
[U.P.P.C.S. (Pre) 2015] (c) Tarachand
(a) Bonded labour (d) V.D. Savarkar
(b) Exploitation of peasants 50. Which one of the following conceptions is
(c) Untouchability true about the Revolt of 1857?
(d) Illiteracy [U.P.P.C.S. (Mains) 2010]
43. Dar-ul-Ulum was established by– (a) Indian historians have described it as an
[U.P.P.C.S. (Mains) 2014] Indian mutiny
(a) Maulana Shibli Numani (b) British historians have described it as a
(b) Maulavi Hussain Ahmad war of independence
(c) Maulavi Abdullah Chakralavi (c) It gave a death blow to the system of East
(d) Maulana Ahmad Riza Khan India Company’s rule in India
(d) It was for the improvement of the
44. Who among the following social reformers administrative machinery in India
was known for proficiency in the Sanskrit
language? [U.P.P.C.S. (Mains) 2013] 51. The official historian of Indian Freedom
(a) Dayanand Saraswati struggle was- [UPPCS (Pre) 2010]
(b) Iswar Chandra Vidyasagar (a) R.C. Mazumdar
(c) Raja Rammohan Roy (b) Tarachandra
(d) All of the above (c) V.D. Savarkar
(d) S.N. Sen
45. Who among the following had presided
over the All-India Trade Union Congress 52. Which among the following organizations
held at Nagpur in 1929? supported Shuddhi movement?
[U.P.P.C.S. (Mains) 2013] [U.P.P.C.S (Pre) 2010]
(a) Jawaharlal Nehru (a) Arya Samaj
(b) Acharya Narendra Dev (b) Brahmo Samaj
(c) Subhas Chandra Bose (c) Dev Samaj
(d) Yusuf Meherally (d) Prarthana Samaj

PYQ Workbook 30
HISTORY OF MODERN INDIA

53. Match these former revolts in 19th century 59. Match list-I with list-II and select the
with these related areas: correct answer using the codes given below
the lists:
List-I List-II
A. Kuki revolt 1. Punjab List-I List-II
B. Kuka revolt 2. Bengal A. Bardoli 1. Swami
Satyagraha Shraddhananda
C. Pabna revolt 3. Bihar Saraswati
D. Birsa Munda revolt 4. Tripura B. Bhartiya Kisan 2. Sardar Vallabhbhai
Code: [U.P.P.C.S. (Pre) 2010] Vidyalaya Patel
A B C D C. Bengal Praja 3. Fazlul Haq
(a) 4 2 1 3 Party
(b) 2 3 1 4 D. Bakashat 4. N.G. Ranga
(c) 4 1 3 2 Agitation
(d) 4 1 2 3 Code: [UPPCS (Mains) 2006]
54. The First Society founded by Raja Ram A B C D
Mohan Roy was: [U.P.P.C.S. (Mains) 2009] (a) 2 3 4 1
(a) Brahma Samaj (b) 2 4 3 1
(b) Atmiya Sabha (c) 1 4 2 3
(d) 4 1 3 2
(c) Brahma Sabha
(d) Tatvabodhini Sabha 60. Match list-I with list-II and select the
correct answer using the codes given below
55. The founder of ‘Bhartiya Brahmo Samaj’ the lists:
was: [U.P. P.C.S. (Spl.) (Mains) 2008]
List-I List-II
(a) Devendranath Tagore
(b) Ishwar Chandra Vidyasagar (Events) (Dates)
(c) Keshav Chandra Sen A. Barrackpur Mutiny 1. July, 1806
(d) Raja Ram Mohan Roy B. Berhampur Revolt 2. November,
1824
56. The first event relating to the war of
Independence of 1857 was– C. Santhal Revolt 3. 1855-56
[U.P.P.C.S. (Mains) 2008] D. Vellore Mutiny 4. February, 1857
(a) Kanpur’s Revolt and taking over the [UPPCS (Mains) 2006]
leadership by Nana Saheb A B C D
(b) Begum Hazrat Mahal’s leadership of (a) 2 4 3 1
Oudh (b) 2 1 4 3
(c) Marching of Sepoys to Delhi’s Red Fort (c) 3 4 2 1
(d) 1 2 3 4
(d) Revolt by Rani of Jhansi
61. Who among the following is known as
57. Who among the following was the leader of ‘Martin Luther of India’?
the revolution of 1857 in Assam? [U.P. P.C.S. (Mains) 2005]
[U.P.P.C.S. (Mains) 2007] (a) Swami Dayanand Saraswati
(a) Diwan Maniram Dutta (b) Raja Ram Mohan Roy
(b) Kandarpeshwar Singh (c) Swami Vivekanand
(c) Purandar Singh (d) Swami Shraddhanand
(d) Piali Barua 62. Who among the following represented
58. What did the Act V of 1843 make illegal? India at the World Parliament of Religions
at Chicago in 1893?
[U.P.P.C.S. (Mains) 2007] [U.P.P.C.S. (Spl) (Mains) 2004]
(a) Child marriage (a) Swami Dayanand Saraswati
(b) Infanticide (b) Ram Krishna Paramhans
(c) Sati (c) Swami Vivekanand
(d) Slavery (d) Raja Ram Mohan Roy

31 PYQ Workbook
HISTORY OF MODERN INDIA

63. Who was the founder of Ram Krishna (c) Shiv Narayan Agnihotri
Mission? [U.P.P.C.S. (Mains) 2004] (d) Swami Shraddhananda
(a) Ramkrishna Paramhansa 69. Which one of the following was the author
(b) M.N. Das Gupta of ‘Gulamgiri’? [U.P.P.C.S. (Pre) 2000]
(c) Swami Vivekananda (a) B.R. Ambedkar
(d) Swami Rangnath Nanda (b) Jyotiba Phule
64. Which one of the following is correctly (c) Mahatma Gandhi
matched? [UPPCS (Spl) (Mains) 2004] (d) Periyar

(a) Brahmo Samaj of Raja Ram 70. Which of the following places was the
India Mohan Roy Centre of the Gadakari rebellion?
[U.P.P.C.S. (Pre) 1999]
(b) Tatvabodhini Sabha Keshav Chandra
Sen (a) Bihar Sharif
(b) Kolhapur
(c) The Satyashodhak Devendranath (c) Surat
Samaj Tagore (d) Sylhet
(d) The Servants of Gopal Krishna 71. The Pagalpanthi rebellion was indeed a
India Society Gokhle rebellion of: [U.P.P.C.S. (Pre) 1999]
65. Who said that “If God were to tolerate (a) Bhils
untouchability, I would not recognize him (b) Garos
as God at all”? (c) Gonds
[U.P.P.C.S. (Spl) (Mains) 2004] (d) Koliyas
(a) B.R. Ambedkar 72. Who among the following was the leader of
(b) Bal Gangadhar Tilak the Farazi rebellion? [U.P.P.C.S. (Pre) 1999]
(c) Lala Lajpat Rai (a) Aga Muhammad Raza
(d) Mahatma Gandhi (b) Dadu Miyan
(c) Shamsher Gazi
66. Match list-I with list-II and select the
(d) Wazir Ali
correct answer using the codes given below
the lists: 73. In the 1930s, peasant movements were
started in different parts of the country at
List-I List-II the instance of various leaders. Match them
A. Brahmo Samaj 1. Mumbai with their respective areas of influence:
B. Manav Dharma Sabha 2. Surat List-I List-II
C. Arya Samaj 3. Kolkata A. Sahjanand Saraswati 1. Hyderabad
D. Nadwah-ul-Ulama 4. Lucknow B. Khudai Khidmatgars 2. Southern
Code: [UPPCS (Mains) 2003] Assam
A B C D C. Swami Ramanand 3. Bihar
(a) 4 1 3 2 D. Abdul Hamid Khan 4. NWFP
(b) 1 4 3 2
Code: [UPPCS (Pre) 1998]
(c) 3 1 4 2
(d) 3 2 1 4 A B C D
(a) 1 2 3 4
67. Velu Thampi led a rebellion against the (b) 3 4 1 2
British in- [U.P.P.C.S. (Mains) 2002] (c) 4 3 2 1
(a) Kerala (d) 2 3 1 3
(b) Maharashtra
74. Who among the following was the leader of
(c) Mysore
the revolt during 1857 at Bareilly?
(d) Telangana
[U.P.P.C.S. (Pre) 1998]
68. Who was the founder of Radha Swami (a) Khan Bahadur
Satsang? [U.P.P.C.S. (Pre) 2002] (b) Kunwar Singh
(a) Haridas Swami (c) Maulvi Ahmad Shah
(b) Shiv dayal Sahab (d) Birzis Kadir

PYQ Workbook 32
HISTORY OF MODERN INDIA

75. Which one of the following uprisings 82. Who was the British Prime minister during
occurred in Maharashtra? the revolt of 1857? [U.P.P.C.S. (Pre) 1991]
[U.P.P.C.S. (Pre) 1998] (a) Churchill
(a) Bhil uprising (b) Palmerston
(b) Kol Uprising (c) Attlee
(c) Rampa uprising (d) Gladstone
(d) Santhal uprising
83. Match list-I with list-II and select the
76. Who among the following was not related to
correct answer using the codes given below
the Revolt of 1857? [U.P.P.C.S. (Pre) 1996]
the lists:
(a) Begum Hazrat Mahal
(b) Kunwar Singh List-I List-II
(c) Uddham Singh A. Child Marriage 1. Ishwarchand
(d) Maulavi Ahmadullah Vidyasagar
77. Who of the following is responsible for the B. Suppression of 2. Colonel
revival of Vedas: [U.P.P.C.S. (Pre) 1995] Thuggee Sleimen
(a) Rama Krishna Paramhans C. Widows’ Remarriage 3. M.G.
(b) Ramanuja Ranade
(c) Swami Dayanand Saraswati
D. Suppression of 4. Lord
(d) Swami Vivekanand
Pindaris Hastings
78. Indians were assured of many things in Code: [UPPCS (Pre) 1990]
the declaration by Queen Victoria in 1858.
Which among the following assurances was A B C D
fulfilled by the British Government: (a) 3 2 1 4
[U.P.P.C.S. (Pre) 1994] (b) 4 2 1 3
(a) Policy of annexation will be abandoned (c) 1 4 3 2
(b) ‘Status quo’ of the native rulers would be (d) 2 3 1 4
maintained 84. What was the main cause of 1857 revolt?
(c) Equal treatment would be given to all
[U.P.P.C.S. (Pre) 1990]
subjects, Indians, and Europeans
(d) There would be no interference in the (a) Public Outrage
social and religious beliefs of the Indians (b) Military discontent
(c) Management of Christian Missionaries
79. Who among the following great men is
(d) Policy of the British Empire
called the ‘Father of Indian Renaissance’?
[U.P.P.C.S. (Pre) 1994] 85. The Rajamundri Social Reform Association
(a) Vivekananda to encourage widow re-marriage was
(b) Raja Ram Mohan Roy founded in 1878 by-
(c) Rabindra Nath Tagore [R.A.S./R.T.S. (Pre.) 2021]
(d) Dayanand Saraswati (a) Veeresalingam
80. Chief centre of Wahabi Movement during (b) K. Ramakrishan Pillai
19th century was: [U.P.P.C.S. (Pre) 1994] (c) K.T. Telang
(a) Lahore (d) Gopalachariar
(b) Patna
86. Which of the following pairs is NOT
(c) Amritsar
(d) Pune correctly matched? [RAS/RTS (Pre) 2018]

81. Which of the following were not protested Institution Founder


by Raja Ram Mohan Roy? (a) Servants of India Society G.K. Gokhale
[U.P. P.C.S. (Pre) 1992] (b) Social Service League N.M. Joshi
(a) Child marriage (c) Seva Samiti H.N. Kunzru
(b) Sati system
(c) Western education (d) Social Reform Shri Ram
(d) Idolatry Association Bajpai

33 PYQ Workbook
HISTORY OF MODERN INDIA

87. Consider the following statements: 92. What was common among Brahma Samaj,
[R.A.S./R.T.S. (Pre) 2016] Ram Krishna Mission, and Arya Samaj?
Assertion (A): Socio-religious movements of [R.A.S./R.T.S.(Pre) 2008]
the 19th century resulted in the modernization (a) None of these three had a political
of India. mission but they helped to develop a
Reason (R): Rationalism, scientific temper spirit of patriotism
and other such ideas which are the basis of (b) All the three originated from Bengal
modernization were at the core of the socio- (c) Founder of all the three was educated in
religious movements. England
Select the correct answer from the code given (d) Founder of all the three took active part
below: in politics
(a) (R) is true, but (A) is false. 93. Who amongst the following first used the
(b) Both (A) and (R) are true, and (R) is the word ‘Swaraj’ and accepted Hindi as the
correct explanation of (A). national language?
(c) (A) is true, but (R) is false.
[R.A.S./R.T.S.(Pre) 1999]
(d) Both (A) and (R) are true, but (R) is not
the correct explanation of (A). (a) Raja Ram Mohan Roy
(b) Swami Dayanand
88. Arrange the following uprisings against (c) Swami Vivekanand
the British colonial power in India prior to (d) Bal Gangadhar Tilak
1857 in the various parts of the country in
the correct order: 94. Who gave Vallabhbhai Patel the title of
(i) Sepoy mutiny in Bengal ‘Sardar’? [R.A.S./R.T.S. (Pre) 1997]
(ii) Kutch Rebellion (a) Mahatma Gandhi
(b) Pandit Nehru
(iii) Sepoy mutiny at Vellore (c) Maulana Azad
(iv) Santhal uprising (d) Kasturba
(v) Kol uprising
95. The British were able to suppress the
Code: [R.A.S./R.T.S. (Pre) 2013] uprising of 1857 in Rajput states because:
(a) (i), (iii), (ii), (v), (iv)
[R.A.S./R.T.S. (Pre) 1996]
(b) (ii), (iii), (i), (v), (iv)
(c) (iv), (i), (iii), (ii), (v) (a) The local Rajput rulers did not support
(d) (iii), (i), (ii), (iv), (v) revolutionaries.
(b) The educated middle class supported the
89. Who is considered as the first ‘Modern Man’ British.
of India? [R.A.S./R.T.S.(Pre) 2012] (c) The soldiers in cantonments were
(a) Nana Saheb not prepared to accept leadership of
(b) A. O. Hume revolution from outside Rajasthan.
(c) Raja Ram Mohan Roy (d) The newspaper could not project the true
(d) Swami Vivekanand aim of the revolution.
90. The Greatest Parsi Social reformer of the 96. The immediate cause of India’s first war of
19th century was:
independence was:
[R.A.S./R.T.S.(Pre) 2010]
[R.A.S./R.T.S. (Pre) 1993]
(a) Sir Jamshedji
(b) Sir Rustom Behramji (a) Doctrine of Lapse by Lord Dalhousie
(c) Navalji Tata (b) Suspicion about British interference in
(d) Behramji M. Mallabari religion
(c) Military discontent
91. Who launched the ‘Lasodia Movement’ for (d) Economic exploitation of India
social reforms amongst the Bhils of Mewar,
Bagar and nearby regions? 97. The first peasant movement of India was:
[R.A.S./R.T.S.(Pre) 2008] [R.A.S./R.T.S. (Pre) 1992]
(a) Mavji (a) Champaran
(b) Govind Giri (b) Bardoli
(c) Surmal Das (c) Begu
(d) Moti Lal Tejawat (d) Bijolia

PYQ Workbook 34
HISTORY OF MODERN INDIA

98. The largest number of soldiers participated (a) R.G. Bhandarkar


in the Struggle of 1857 came from– (b) M.G. Ranade
[U.P. Lower Sub. (Pre) 2015] (c) Pandit Ramabai
(a) Bengal (d) Gopal Ganesh Agarkar
(b) Oudh 106. Who among the following was the founder
(c) Bihar of ‘Dev Samaj’?
(d) Rajasthan [U.P. Lower Sub. (Pre) 2003]
99. Who among the following famous reformers (a) Vallabhbhai Patel
wrote the books Jnyana Yoga, Karma Yoga, (b) Dadabhai Naoroji
and Raja Yoga? (c) Shiv Narayan Agnihotri
[U.P. Lower Sub. (Pre) 2015] (d) Ramakrishna Paramhansa
(a) Swami Vivekananda 107. The first freedom struggle of 1857 failed
(b) Ranade because– [U.P. Lower Sub. (Pre) 1998]
(c) Raja Ram Mohan Roy (a) There was lack of the unity of purpose in
(d) Ramakrishna Paramahansa Indian sepoys
100. The campaign for widow remarriage in (b) Generally, Native Kings supported British
Maharashtra was led by: (c) British Soldiers were better equipped and
[U.P. Lower Sub. (Pre) 2013] organised rather than Indians
(a) Vishnu Parashuram Pandit (d) All of the above
(b) B.M. Malabari 108. Match list-I with list-II and select the
(c) Gopal Hari Deshmukh correct answer using the codes given below
(d) Dadabhai Naoroji the lists:
101. Who was the founder of Bahujan Samaj? List-I List-II
[U.P. Lower Sub. (Spl) (Pre) 2010] A. Raja Ram 1. Said that Brahmovad
(a) Sri Narayan Guru Mohan Roy should be made the
(b) Mukund Rao Patil religion of the world
(c) Dr. B.R. Ambedkar
(d) B.R. Shinde B. Keshav 2. Identified Hinduism
Chandra Sen with religion
102. The first Indian to write in Indian language instituted in Vedas
on the causes of the uprising of 1857 was:
C. Dayanand 3. Emphasized that
[U.P. Lower Sub. (Pre) 2009] Saraswati there are several
(a) Syed Ahmed Khan ways to reach God
(b) V. D. Savarkar
D. Ramakrishna 4. Purest form of
(c) Bankim Chandra Chatterjee
Paramhansa Hinduism is
(d) None of the above
contained in
103. Who was ‘commander-in-chief ’ of Nana Upanishads
saheb? [U.P. Lower Sub. (Spl) (Pre) 2008] Code: [UP Lower Sub. (Pre) 1998]
(a) Azimullah A B C D
(b) Birjis Qadir (a) 1 4 2 3
(c) Tatya Tope (b) 1 4 3 2
(d) None of the above (c) 4 1 3 2
104. The Bengali leader who opposed socio- (d) 4 1 2 3
religious reforms and supported orthodoxy 109. The Tebhaga Movement was started in 1946
was- [U.P. Lower Sub. (Pre) 2008] from bengal under the leadership of:
(a) Radhakant Deb [66th B.P.S.C. Re-Exam (Pre) 2020]
(b) Nemisadhan Bose
(a) Muslim League
(c) Hemchandra Biswas
(b) Kisan Sabha
(d) Hemchandra De
(c) Indian National Congress
105. Who was the chief director of Prarthana (d) Trade Union
Samaj in Maharashtra? (e) None of the above/More than one of the
[U.P. Lower Sub. (Pre) 2004] above

35 PYQ Workbook
HISTORY OF MODERN INDIA

110. Who wrote the famous play, Neel Darpan 115. The Indigo Rebellion was about-
in which oppression of Indigo farmers was [BPSC (Pre) 2018]
displayed? [66th B.P.S.C. (Pre) 2020] (a) the peasant not wanting to grow indigo
(a) Sharat Chandra Chatterjee but being forced
(b) Rabindranath Tagore (b) the peasant wanting to grow indigo but
(c) Barindra Ghosh being forced not to
(d) Dinabandhu Mitra (c) the peasant not wanting to grow indigo
(e) None of the above/More than one of the but being forced to grow at a price that
above was unacceptable
111. Poona Sarvajanik Sabha was founded in: (d) a protest movement that carried and
[66th B.P.S.C. Re-Exam (Pre) 2020] indigo-coloured flag
(a) 1858 by Mahadev Govind Ranade 116. Who among the following did not rebel
(b) 1870 by Mahadev Govind Ranade against the British East India Company’s
(c) 1870 by Pandita Ramabai Ranade control over them? [B.P.S.C. (Pre) 2018]
(d) 1870 by Surendranath Banerjee (a) Raja of Vijayanagar
(e) None of the above/More than one of the (b) Nizam of Hyderabad
above (c) Polygar of Tamil Nadu
112. Which of the following was not a reason (d) Dewan Velu Thampi of Travancore
for making the sepoys of the East India (e) None of the above/More than one of the
Company rebellious? [B.P.S.C. (Pre) 2018] above
(a) The efforts of the officers of the company 117. Swami Sahajanand Saraswati published a
to spread Christianity journal named: [63rd B.P.S.C (Pre.) 2017]
(b) The order to the sepoys to travel on ships (a) Jankranti
(c) The stoppage of Bhatta (b) Hunkar
(d) The inefficiency of the officers (c) Krishak Samachar
(e) None of the above/More than one of the (d) Vidrohi
above (e) None of the above/More than one of the
113. Identify the correct combination from the above
options given below for Prathana Samaj, 118. Who was elected the first President of All
Young India, Lokahitavadi, Satyashodhak India Kisan Sabha at Lucknow in 1936?
Samaj, Rehnumai Mazdayasan Sabha:
[60th to 62nd B.P.S.C. (Pre) 2016]
[BPSC (Pre) 2018]
(a) N.G. Ranga
(a) Gopal Hari Deshmukh, Atmaram
(b) E.M.S. Namboodripad
Pandurang, Mohandas Karamchand
(c) Swami Sahajanand Saraswati
Gandhi, Jyotiba Phule, Naoroji Furdonji
(d) Acharya Narendra Dev
(b) Atmaram Pandurang, Mohandas
(e) None of the above/More than one of the
Karamchand Gandhi, Gopal Hari
above
Deshmukh, Jyotiba Phule, Naoroji
Furdonji 119. The Waghera Uprising happened in–
(c) Atmaram Pandurang, Jyotiba Phule, [56th to 59th B.P.S.C. (Pre) 2015]
Mohandas Karamchand Gandhi, Gopal (a) Surat
Hari Deshmukh, Naoroji Furdonji (b) Poona
(d) Naoroji Furdonji, Atmaram Pandurang, (c) Calicut
Mohandas Karamchand Gandhi, Gopal (d) Baroda
Hari Deshmukh, Jyotiba Phule
(e) None of the above/More than one of the 120. Who was the founder of ‘Prarthana Samaj’?
above [53rd to 55th B.P.S.C. (Pre) 2011]
114. Who began the Faraizi Movement? (a) Atmaram Panduranga
(b) Tilak
[BPSC (Pre) 2018]
(c) Annie Besant
(a) Haji Shariatullah (d) Rasbihari Ghose
(b) Syed Ahmed
(c) Salimullah 121. Who founded the ‘Theosophical Society’?
(d) M.A. Jinnah [53rd to 55th B.P.S.C. (Pre) 2011]

PYQ Workbook 36
HISTORY OF MODERN INDIA

(a) Madame H.P. Blavatsky (c) Bankim Chandra Chatterji


(b) Raja Ram Mohan Roy (d) Raja Rammohan Roy
(c) Mahatma Gandhi
129. Which of the following groups did not
(d) Swami Vivekananda
participate in the revolt of 1857?
122. Kol Mutiny of 1831 under Buddho Bhagat [40th B.P.S.C. (Pre) 1995]
took place in which of the following regions? 1. Agricultural Labour
[66th B.P.S.C. (Pre) 2010] 2. Sahukar
(a) Kutch 3. Farmers
(b) Singhbhum 4. Landlords
(c) Western Ghats
(d) Satara Select your answer from the following codes:
(e) None of the above/More than one of the (a) Only 1
above (b) 1 and 2
(c) Only 2
123. The revolt of 1857 at Lucknow was led by: (d) 2 and 4
[48th to 52nd B.P.S.C. (Pre) 2008]
130. Who called the revolt of 1857 a conspiracy?
(a) Begum of Oudh
(b) Tatya Tope [40th B.P.S.C. (Pre) 1995]
(c) Rani Lakshmibai (a) Sir James Outram and W. Taylor
(d) Nana Saheb (b) Sir John K.
(c) Sir John Lawrence
124. After the revolt of 1857, the British recruited (d) T.R. Homes
the Soldiers from the-
[48th to 52nd B.P.S.C. (Pre) 2008] 131. Trace the tribe that rebelled against
Britishers because human sacrifice system
(a) Brahmins of U.P. & Bihar
was banned– [40th B.P.S.C. (Pre) 1995]
(b) Bengalis and Orria’s from the East
(c) Gurkhas, Sikhs, and Punjabis in the North (a) Kuki
(d) Madras presidency and Marathas (b) Khond
(c) Oraon
125. Which one of the following commissions is (d) Nackda
associated with the Army Reorganization
after the suppression of the Revolt of 1857? 132. Who demanded the right to enter temples
that caused a terrible riot in Tirunevali in
[45th B.P.S.C. (Pre) 2001]
1899? [40th B.P.S.C. (Pre) 1995]
(a) Public Service Commission
(a) Okalinga
(b) Peel Commission
(b) Nadars
(c) Hunter Commission
(c) Mahar
(d) Simon Commission
(d) Pali
126. The Revolt of 1857 was witnessed by the
133. Consider the following statements:
poet– [45th B.P.S.C. (Pre) 2001]
[39th B.P.S.C. (Pre) 1994]
(a) Mir Taqi Mir
(b) Zauq Assertion (A): The first war of Independence
(c) Ghalib in 1857 failed to secure freedom from the
(d) Iqbal British Government.
Reason (R): The leadership of Bahadur Shah
127. Kuka Movement was organized by–
Zafar did not have popular support and most
[45th B.P.S.C. (Pre) 2001] of the Indian rulers of important States shied
(a) Guru Ram Das away from the struggle.
(b) Guru Nanak
Select the correct answer from the code given
(c) Guru Ram Singh
below:
(d) Guru Gobind Singh
(a) Both (A) and (R) are true, and (R) is the
128. The best representative of Neo-Hindustan correct explanation of (A).
in the second half of the nineteenth century (b) Both (A) and (R) are true, but (R) is not
was– [41st B.P.S.C. (Pre) 1996] the correct explanation of (A).
(a) Ramkrishna Paramhans (c) (A) is true, and (R) is false.
(b) Swami Vivekanand (d) (A) is false, but (R) is true.

37 PYQ Workbook
HISTORY OF MODERN INDIA

134. What was the purpose of Barhiyataal (a) 1 and 2


resistance in Munger? (b) 1, 2 and 3
[39th B.P.S.C. (Pre) 1994] (c) 3 and 4
(a) Demand of Bakast land restoration (d) 1, 3 and 4
(b) Muslim peasants may not be exploited 139. Which of the following statements about
(c) Abolishment of Zamindari custom Prarthana Samaj are correct? [CAPF 2019]
(d) Beginning of class struggle 1. Its aim was to reform the Hindu society.
135. Who founded Ramosi Krishak Jatha in 2. It was based in Maharashtra.
Maharashtra? [39th B.P.S.C. (Pre) 1994] 3. It believed in Monotheism.
(a) Justice Ranade 4. It had greatly influenced the Brahmo
(b) Gopal Krishna Gokhale Samaj.
(c) Vasudeo Balwant Phadke Select the correct answer using the code given
(d) Jyotiba Phule below:
136. ‘Nai-Dhobi Band’ was a form of social (a) 1 and 2
boycott in 1919 which– (b) 2 and 3
[39th B.P.S.C. (Pre) 1994] (c) 1, 2 and 3
(d) 1 and 4
(a) was started by the farmers of Pratapgarh
district 140. Consider the following events: [CAPF 2019]
(b) was a movement started by saints for the 1. Foundation of the Ramakrishna Mission
emancipation of the people lower classes 2. Foundation of the Arya Samaj
(c) was a step against contractors of lower 3. Foundation of the Brahmo Samaj
castes by landlords 4. Foundation of the Paramhans Mandali
(d) was a movement against contractors by
lower castes Which one of the following is the correct
chronological order of the above?
137. Which of the following statement(s) is/are (a) 1, 2, 3, 4
correct? (b) 2, 3, 4, 1
1. Parmahansa Mandali was founded in (c) 3, 4, 1, 2
Bengal by Swami Vivekananda. (d) 3, 4, 2, 1
2. Prarthana Samaj was started in 141. Which one of the following statements
Maharashtra with the aim of reforming about the Indigo Revolt of 1859 is correct?
Hindu religious thought and practice. [CAPF 2017]
Select the correct answer using the codes (a) It broke out at Poona and Ahmadnagar.
given below [CAPF 2020] (b) It was directed against moneylenders.
(a) 1 only (c) It was supported by the intelligentsia of
(b) 2 only Bengal.
(c) Both 1 and 2 (d) It was directed against the petty Zamindars
(d) Neither 1 nor 2 and ex-employees of the planters.
138. Which of the following statements about 142. Which of the following is/are
the Arya Samaj, which emerged in the late characteristic(s) of the Ryotwari system?
19th century are correct? [CAPF 2019] 1. It was the brainchild of Thomas Munro.
1. Had a very strong support base amongst 2. It was meant to reduce intermediaries.
the trading castes. 3. Cultivating peasants were gradually
2. Introduced ‘Shuddhi’ or mass purification. impoverished by the system.
3. The moderates in the Arya Samaj were 4. It was introduced in parts of Madras and
headed by Hans Raj and Lajpat Rai and Bengal Presidency.
set up a chain of Dayanand Anglo-Vedic Select the correct answer using the code given
colleges. below. [CAPF 2016]
4. The Arya Samaj had fewer members than (a) 1 only
the Brahmo Samaj. (b) 1, 2 and 3
Select the correct answer using the code given (c) 2 and 4 only
below: (d) 2, 3 and 4

PYQ Workbook 38
HISTORY OF MODERN INDIA

143. Who among the following founded the C. Satyasodhak 3. Mundas


Atmiya Sabha in 1815? [CAPF 2016] Samaj
(a) Keshab Chandra Sen D. Ulgulan 4. Kunbi peasantry
(b) Debendranath Tagore
Codes: [CAPF 2015]
(c) Ram Mohan Roy
(d) Bijoy Krishna Goswami A B C D
(a) 2 4 1 3
144. Consider the following statements about (b) 2 1 4 3
the Santhal Hool of 1855-56. [CAPF 2016] (c) 3 4 1 2
1. The Santhals were in a desperate situation (d) 3 1 4 2
as tribal lands were leased out. 147. In 1893, there occurred a split in the Arya
2. The Santhal rebels were treated very Samaj on the issue(s) of [CAPF 2015]
leniently by British officials. 1. Meat-eating vs vegetarianism.
3. Santhal inhabited areas were eventually 2. Caste system and widow remarriage.
constituted into separate administrative 3. Shuddhi of converts.
units called Santhal parganas. 4. Anglicised vs Sanskrit-based education.
4. The Santhal rebellion was the only major Select the correct answer using the code
rebellion in mid 19th century India. given below.
Which of the statements given above is/are (a) 1, 3 and 4
correct? (b) 2 and 4 only
(a) 1 only (c) 1 and 4 only
(b) 2 and 3 (d) 1 only
(c) 1, 3 and 4 148. Who among the following were well known
(d) 1 and 3 only as champions of women’s education in
145. Match List I with List II and select the colonial India? [CAPF 2015]
correct answer using the code given below 1. Sister Subbalaksmi
the lists. 2. Begum Rokeya Sakhawat Hossain
3. Keshub Chandra Sen
List I (Peasant List II (Leader/ 4. Ananda Coomaraswamy
Movement) Follower)
Select the correct answer using the code
A. Bakasht Land 1. Baba Ramchandra given below.
Movement
(a) 1, 2, 3 and 4
B. Eka Movement 2. Kunhammed Haji (b) 1, 2 and 3 only
C. Mappila Rebellion 3. Madari Pasi (c) 3 and 4 only
D. Avadh Kisan Sabha 4. Karyanand Sharma (d) 1 and 2 only
Movement 149. Which one among the following is not true
Codes: [CAPF 2015] for the Brahmo Samaj? [CAPF 2014]
A B C D (a) It considered Vedic scriptures as infallible
(a) 4 3 2 1 (b) It opposed idolatry
(b) 4 2 3 1 (c) It emphasized the worship of one God
(c) 1 2 3 4 (d) It emphasized the worship of God without
(d) 1 3 2 4 the mediation of priests

146. Match List I with List II and select the 150. Which one among the following is not true
for the Theosophical Society of Madam H P
correct answer using the code given below
Blavatsky? [CAPF 2014]
the lists.
(a) It was founded outside India
List I (Movement) List II (Mass base/ (b) It advocated the revival of ancient
Following) religions
A. Bardoli 1. Bargadars (c) It recognized the doctrine of
Satyagraha transmigration of soul
(d) It was silent on the idea of Universal
B. Tebhaga 2. Kaliparaj Brotherhood

39 PYQ Workbook
HISTORY OF MODERN INDIA

151. Who among the following founded the (c) Statement I is true, but Statement II is
Hindu Dharma Sabha in the 19th century? false.
[CDS 2023 (I)] (d) Statement I is false, but Statement II is
(a) Radhakanta Deb true.
(b) Ram Mohan Roy 156. The elected President of the All-India Kisan
(c) Dwarkanath Tagore Sabha, which met in Vijayawada (1944), was
(d) Keshab Chandra Sen [CDS 2018 (II)]
152. The ‘Tattvabodhini Sabha’ was established (a) Sahajananda Saraswati
(b) Vinoba Bhave
by [CDS 2020 (I)]
(c) Achyut Rao Patwardhan
(a) Devendranath Tagore in 1839 (d) Narendra Dev
(b) Keshab Chandra Sen in 1857
(c) Akshay Kumar Datta in 1850 157. Which one of the following peasant
(d) Dwarakanath Tagore in 1840 struggles was an outcome of British opium
policy?
153. Which one of the following towns was not a [CDS 2017 (II)]
centre of the Revolt of 1857? (a) Phulaguri Dhawa [1861]
[CDS 2020 (II)] (b) Birsaite Ulgulan [1899-1900]
(a) Ayodhya (c) Pabna Revolt [1873]
(b) Agra (d) Maratha Peasant Uprising [1875]
(c) Delhi 158. Which of the following statements about
(d) Kanpur the Shiromani Gurudwara Prabandhak
154. Consider the following statements: The Committee (SGPC) are correct?
Azamgarh Proclamation refers to 1. It began as the political wing of the Singh
Sabhas in the late 19th century.
1. the declaration by the rebels of 1857.
2. It was formed in 1920 as part of the
2. the statement by the leader of the upcoming Akali movement.
underground movement in the Revolt of 3. It was founded to reclaim control of
1942. the Sikh shrines from the government
Which of the statements given above is/are manipulated loyalist committees.
not correct? [CDS 2020 (II)] 4. It formed the Akali Dal to coordinate
(a) Only 1 groups (Jathas) to reclaim control of the
(b) Only 2 shrines.
(c) Both 1 and 2 Select the correct answer using the codes
(d) Neither 1 nor 2 given below [CDS 2017 (I)]
(a) 1, 3 and 4
155. The following item consist of two
(b) 2, 3 and 4
statements, Statement I and Statement II. (c) 2 and 4 only
Examine these two statements carefully and (d) 2 and 3 only
select the correct answer using the code
given below: 159. Which of the following statement(s) about
the social base of the Arya Samaj in British
Statement I: The Oudh Kisan Sabha India is/are correct?
established in 1920 failed to bring under its
1. It was located mainly in Punjab and
wing any Kisan Sabhas. Western Uttar Pradesh.
Statement II: The Oudh Kisan Sabha asked 2. It mainly comprised the trading castes.
the Kisans to refuse to till Bedakhli land, not 3. It was much more limited than that of the
to offer hari and begar. [CDS 2019 (II)] Brahmo Samaj.
(a) Both the statements are individually true, Select the correct answer using the codes
and Statement II is the correct explanation given below [CDS 2017 (I)]
of Statement I. (a) 1 and 2
(b) Both the statements are individually (b) 2 and 3
true, but Statement II is not the correct (c) 1 only
explanation of Statement I. (d) 1, 2 and 3

PYQ Workbook 40
HISTORY OF MODERN INDIA

160. Which of the following statement(s) about 163. The following item consist of two
Jyotirao Phule’s Satyashodhak Samaj statements, Statement I and Statement II.
Movement in Maharashtra is/are true? Examine these two statements carefully and
1. The Satyashodhak Samaj was set-up in select the correct answer using the code
1873. given below:
2. Phule argued that Brahmins were the Statement I: Ram Mohan Roy in his famous
progeny of ‘alien’ Aryans. work Gift to Monotheism put forward weighty
3. Phule’s focus on the Kunbi peasantry in arguments against belief in many Gods and
the 1880s and 1890s led to a privileging for the worship of a single God.
of Maratha identity. Statement II: Ram Mohan Roy in his Precepts
Select the correct answer using the codes of Jesus tried to separate the moral and
given below: [CDS 2015 (II)] philosophic message of the New Testament.
(a) 1 and 2 [CDS 2014 (II)]
(b) Only 2 (a) Both the statements are individually true,
(c) 1 and 3 and Statement II is the correct explanation
(d) 1, 2 and 3 of Statement I.
161. Which of the following statement(s) about (b) Both the statements are individually
Maulvi Ahmadullah Shah, who played an true, but Statement II is not the correct
important part in the Revolt of 1857 is/are explanation of Statement I.
correct? (c) Statement I is true, but Statement II is
1. He was popularly known as Danka Shah false.
or the Maulvi with a drum. (d) Statement I is false, but Statement II is
true.
2. He fought in the famous Battle of Chinhat.
3. He was killed by British troops under the 164. The following item consist of two
command of Henry Lawrence. statements, Statement I and Statement II.
Select the correct answer using the codes Examine these two statements carefully and
given below: [CDS 2015 (II)] select the correct answer using the code
(a) Only 1 given below:
(b) 1 and 3 Statement I: The Bethune School, founded
(c) 2 and 3 in Calcutta in 1849 was the first fruit of the
(d) 1 and 2 powerful movement for women’s education
that arose in the 1840s and 1850s.
162. Which of the following is/are the
characteristic(s) of the Sannyasi and Fakir Statement II: The first step in giving
uprisings? modern education to girls was undertaken by
1. These uprisings refer to a series of Vidyasagar in 1800. [CDS 2014 (II)]
skirmishes between the English East (a) Both the statements are individually true,
India Company and a group of Sannyasis and Statement II is the correct explanation
and Fakirs. of Statement I.
2. One reason for the uprising was the ban (b) Both the statements are individually
on free movement of the Sannyasis along true, but Statement II is not the correct
pilgrimage routes. explanation of Statement I.
(c) Statement I is true, but Statement II is
3. In the course of the uprisings in 1773,
Warren Hastings issued a proclamation false.
banishing all Sannyasis from Bengal and (d) Statement I is false, but Statement II is
Bihar. true.
4. Are contemporaneous with the Non- 165. The following item consist of two
Cooperation Movement. statements, Statement I and Statement II.
Select the correct answer using the codes given Examine these two statements carefully and
below: [CDS 2015 (I)] select the correct answer using the code
(a) 1 Only given below:
(b) 1 and 3 Only Statement I: The annexation of Awadh by
(c) 1, 2 and 3 Lord Dalhousie in 1856 adversely affected the
(d) 2 and 4 Only financial conditions of the sepoys.

41 PYQ Workbook
HISTORY OF MODERN INDIA

Statement II: The sepoys had to pay higher (a) W.H. Russel
taxes on the land where their family members (b) Robert Peel
stayed in Awadh. [CDS 2014 (II)] (c) Gladoston
(a) Both the statements are individually true, (d) Palmerston
and Statement II is the correct explanation 172. ‘Indian nationalism in the middle of the
of Statement I. 19th century was in Embryonic stage’.
(b) Both the statements are individually Which historians are related to this fact?
true, but Statement II is not the correct [M.P.P.C.S. (Pre.) 2020]
explanation of Statement I. (a) Dr. R.C. Majumdar and Dr. S.N. Sen
(c) Statement I is true, but Statement II is (b) Sir James Outram and W. Tailor
false. (c) T.R. Homes and L.E.R. Reas
(d) Statement I is false, but Statement II is (d) Sir John Lawrence and Seale
true.
173. Who founded the Adi Brahma Samaj?
166. Who among the following was not associated [M.P.P.C.S. (Pre.) 2020]
with the activities of the Theosophical (a) Devendranath Tagore
Society? [CDS 2014 (I)] (b) Keshav Chandra Sen
(a) Madame HP Blavatsky (c) Raja Ram Mohan Roy
(b) Mr AO Hume (d) Ravindranath Tagore
(c) Col HS Olcott
174. Which of the following brave from Bhopal
(d) Mrs Annie Besant
led the main movement and sacrificed
167. Azimullah Khan was an advisor to- his life during the first struggle for
[Uttarakhand P.C.S. (Pre) 2012] Independence in 1857?
(a) Nana Saheb [MPPCS (Pre) 2019]
(b) Tatya Tope (a) Fazil Mohammad Khan
(c) Rani Laxmi Bai (b) Sheikh Ramzan
(d) Kunwar Singh (c) Dost Mohammad Khan
(d) Habibulla Khan
168. Which tribal leader is known as ‘Dharti
Aaba’ (Father of World)? 175. “The British empire is rotten to the core,
[Uttarakhand P.C.S. (Mains) 2002] corrupt in every direction and tyrannical
and mean.” This statement was made by-
(a) Jiriya Bhagat
(b) Kanu Sanyal [M.P.P.C.S. (Pre) 2018]
(c) Roop Nayak (a) Sister Nivedita
(d) Birsa Munda (b) Savitribai Phule
(c) Annie Besant
169. Mangal Pandey incident took place at: (d) Bal Gangadhar Tilak
[Uttarakhand P.C.S. (Mains) 2002]
176. Who adorned Ram Mohan Roy with the
(a) Meerut title of ‘Raja’? [M.P.P.C.S. (Pre) 2017]
(b) Barrackpore (a) Aurangzeb
(c) Ambala (b) Robert Clive
(d) Lucknow (c) Mahatma Gandhi
170. Sharda Act related to– (d) Mughal Emperor Akbar-II
[Uttarakhand P.C.S. (Mains) 2002] 177. Who among the following was the first
(a) Prevention of child marriage leader to organize labour movement in
(b) Prevention of inter-caste marriage India? [M.P.P.C.S (Pre) 2017]
(c) Prevention of widow marriage (a) B.P. Walia
(d) Prevention of tribal marriage (b) Lala Lajpat Rai
171. Who was the Correspondent of the (c) N.M. Lokhande
(d) N.G. Ranga
Newspaper ‘Times’ published from London
in 1857 wrote that “Nobody in the North 178. Modern Historian, who called the revolt of
India looks at white man’s car with friendly 1857 as the first Independence War was–
view”? [M.P.P.C.S. (Pre.) 2020] [M.P.P.C.S. (Pre) 2008]

PYQ Workbook 42
HISTORY OF MODERN INDIA

(a) Dr. R.C. Mazumdar 3. Revolt of Gumsur was led by Shrikar


(b) Dr. S.N. Sen Bhanja
(c) V. D. Savarkar 4. Dhananjay was father of Shrikar Bhanja
(d) Ashok Mehta
Which of the above statements is/are true?
179. Who of the following had struggled against [Chhattisgarh PCS (Pre) 2021]
Britishers in 1857? [M.P.P.C.S. (Pre) 2000]
(a) 1, 2 and 3
(a) Chandrasekhar Azad
(b) Ramprasad Bilsmil (b) 1, 2 and 4
(c) Saadat Khan (c) 1 and 2
(d) Makhan Lal Chaturvedi (d) 1 only
180. Which reformer of Maharashtra is also 186. Who has advocated ‘Practical Vedanta’?
known as ‘Lokahitvadi’? [Chhattisagarh P.C.S. (Pre) 2019]
[M.P.P.C.S. (Pre) 1995] (a) Dayanad
(a) M.G. Ranade (b) Raja Ram Mohan Roy
(b) Gopal Krishna Gokhale (c) Gandhi
(c) Pandit Ramabai (d) Vivekananda
(d) Gopal Hari Deshmukh
181. Which of the following statement is not 187. The following popular movements occurred
correct? [M.P.P.C.S. (Pre) 1995] after 1857:
(a) Dr. Annie Besant was a Theosophist 1. Santhal Rebellion
(b) The International headquarter of 2. Indigo Revolt
Theosophical society is in Madras 3. Deccan Agriculturists Riots
(c) Swami Dayanand founded Arya Samaj 4. Birsa Munda Uprising
(d) Mahatma Gandhi was born in
Gandhinagar Select the correct answer:
182. Which of the following statements is not [Chhattisgarh P.C.S. (Pre) 2017]
correct? [M.P.P.C.S. (Pre) 1994] (a) 1, 2, 3
(a) In 1829, William Bentinck declared Sati (b) 2, 3, 4
System an offence by law. (c) 1, 2, 4
(b) The Government had passed an Act in (d) 1, 3, 4
1856 under which Hindu widows could
remarry. 188. The revolt of Vellore occur during the
(c) Swami Dayanand Saraswati founded Arya regime of which Governor?
Samaj in 1875. [Chhattisgarh P.C.S. (Pre) 2016]
(d) Raja Ram Mohan Roy was the supporter (a) Wellesley
of Sati System. (b) Lord Minto
183. Maharani Laxmibai had combated in last (c) Lord Cornwallis
battle against– [M.P.P.C.S. (Pre) 1992] (d) Sir George Barlow
(a) Hugh Rose (e) None of the above
(b) Guff
189. Who was the composer of the song ‘Vande
(c) Niel
(d) Havlock Mataram’?
[Chhattisgarh P.C.S. (Pre) 2005]
184. The symbol of 1857 independence struggle
was– [M.P.P.C.S. (Pre) 1990] (a) Rabindra Nath Tagore
(a) Lotus and Chapatis (b) Ramdhari Singh Dinkar
(b) Eagle (c) Sarojini Naidu
(c) Scarf (d) Bankimchandra Chatterjee
(d) Two sword 190. Who was the leader of Bhumij Revolt?
185. Read the following statements and choose [Jharkhand P.C.S. (Pre.) 2021]
the correct option: (a) Bhagirath
1. Ganjam Revolt occurred during AD 1800 (b) Dubia Gosain
to 1805 (c) Jatra Bhagat
2. Ganjam Revolt was led by Dhananjay (d) Ganga Narain

43 PYQ Workbook
HISTORY OF MODERN INDIA

191. The first tribal leader who was inspired by (c) Both (a) and (b)
Mahatma Gandhi and his ideology was- (d) None of (a) and (b)
[Jharkhand PCS (Pre) 2013]
193. Who of the following said, ‘good
(a) Alluri Sitaram Raju
Government is no substitute for Self-
(b) Jodanang
(c) Jhabkar Bapa Government’?
(d) Rani Gaidinliu [Uttaranchal P.C.S. (Pre) 2005]
192. Who led the revolt of Santhals? (a) Lokmanya Tilak
[Jharkhand P.C.S. (Pre) 2003] (b) Swami Vivekananda
(a) Siddhu-Kanhu (c) Swami Dayanand
(b) Bhairav Chand (d) Ravindranath Tagore

PYQ Workbook 44
HISTORY OF MODERN INDIA

SOLUTIONS
4. Solution (b)
2.1. UPSC CSE Previous Years’ Questions
Exp) Option b is the correct answer.
1. Solution: (c) In the 18th and 19th century there was a significant increase
Exp) Option c is the correct answer. in the Indigo production in Bengal and Bihar due to
Ishwar Chandra Vidyasagar was the secretary of Bethune increasing demand and production. Though the revolts of
school established in 1849 also known as Hindu female farmers and oppressive zamindari system had some negative
school. He was one of the pioneers of higher education for impact on indigo farming, a major reason for its falling
women in India. cultivation was the invention of Synthetic indigo, which
made natural indigo non-profitable for farmers as well as
2. Solution: (c) traders.
Exp) Option c is the correct answer.
Important Tips
E.V. Ramaswamy Naicker, or Periyar, as he was called, came
Indigo (Neel) is a blue dye which comes from the Indigo
from a middle-class family. Interestingly, he had been an
plant. It was widely grown in India firstly for the East India
ascetic in his early life and had studied Sanskrit scriptures
Company and later for the British Raj. In the nineteenth
carefully. Later, he became a member of the Congress, only
century, it was one of the most profitable commodities
to leave it in disgust when he found that at a feast organised
traded in the European market often called “Blue Gold”.
by nationalists, seating arrangements followed caste
distinctions – that is, the lower castes were made to sit at a 5. Solution (d)
distance from the upper castes. Convinced that untouchables Exp) Option d is the correct answer.
had to fight for their dignity, Periyar founded the Self
Munda Rebellion is one of the prominent 19th century
Respect Movement. He felt that all religious authorities saw
tribal rebellions in the subcontinent. Birsa Munda led this
social divisions and inequality as God-given. Untouchables
movement in the region of south Ranchi in 1899-1900.
had to free themselves, therefore, from all religions in order
Aimed at establishing Munda Raj or Munda rule in this
to achieve social equality.
region, revolt came to be known as The Ulgulan (meaning
3. Solution (b) total revolt) or “the Great Tumult”.
Exp) Option b is the correct answer. Important Tips
Statements 2 and 3 are correct. Dr. Rukhmabai (1864 Significant causes of Munda rebellion:
–1955) was an Indian physician and feminist. She is best
In 1874, the British replaced the traditional Khuntkari
known for being one of the first practising women doctors
system of Munda tribals by the zamindari system. The
in colonial India as well as being involved in a landmark
introduction of zamindari system created the classes of
legal case involving her marriage as a child bride between
zamindars (landlords) and ryots (tenants). Following the
1884 and 1888.
monetisation of economy, the tribals had to depend on
Statement 1 is incorrect. Education wasn’t an issue in this cash for paying the rent and for meeting their daily needs.
case. This made them dependent on the money-lenders.
Important Tips 6. Solution (c)
Dr. Rukhmabai was married off at the young age of eleven Exp) Option c is the correct answer.
with Dadaji Bhikaji, who was nineteen years old. She
stayed at her parent’s house and she educated herself under Statement 1 is correct. The Santals belonged to Bengal
the guidance of her stepfather. In 1885, after 12 years of Region and were among the worst sufferers due to the
marriage, Bhikaji filed a petition in the court and sought permanent settlement. The Santhals under the leadership
“restitution of conjugal rights”. of Sidhu and Kanhu raised in 1856 with a view to put an
end to colonial rule in India. They disrupted the railway and
Rakhmabai Defense Committee was formed by Behramji
postal communications between Bhagalpur and Rajmahal.
Malabari and Pandita Ramabai in her defence. In March
1887, Rukhmabai was ordered to go live with her husband The Rebellion was pacified with the creation of a separate
or spend six months in Jail. Rukhmabai wrote to Queen district consisting of the Santhal Parganas. The British
Victoria about her situation. The Queen overruled the hoped that by creating a new territory for the Santhals and
court’s verdict and dissolved the marriage. In July of 1888, imposing some special laws within it, the Santhals could be
Dadaji accepted monetary compensation of two thousand conciliated.
rupees to dissolve the marriage. The case continued for Statement 2 is correct. After the rebellion was suppressed,
4 years until Dadaji was compensated in 1888, outside of the British passed the Santhal Pargana Tenancy Act, 1876
court. The case was instrumental in the drafting of the which prohibit the transfer of santhal owned land to a
Age of Consent Act in 1891. non-Santhal.

45 PYQ Workbook
HISTORY OF MODERN INDIA

7. Solution (b) Brahmo Samaj, a socio-religious movement that advocated


Exp) Option b is the correct answer. for monotheism and social reforms.
• Neeldarpan was written by Dinabandhu Mitra - 1858- • He founded the Tabernacle of New Dispensation,
59 (During the Indigo Revolt) also known as Naba Bidhan, in 1868 as a new religious
institution that combined elements of Hinduism,
• Satyendranath Tagore became the first Indian to
Christianity, Islam, and Buddhism.
succeed in the Indian Civil Services Examination – 1863
(Surendranath Banerjee, Subash Chandra Bose also • He also founded the Indian Reform Association in
cleared ICS in later years) 1870 as a secular organization that aimed to promote
education, women’s rights, legal reforms, and social
• Swami Dayanand established Arya Samaj - 1875 welfare
• Bankim Chandra Chattopadhyay wrote Anandmath – Keshav Chandra Sen was not associated with the
1882 establishment of the Calcutta Unitarian Committee, which
8. Solution (d) was founded by Raja Ram Mohan Roy, Dwarkanath Tagore,
and William Adam in 1823 as a platform to propagate
Exp) Option d is the correct answer.
Unitarianism and rationalism
The British were alarmed by the frequent strikes and rising
influence of the communists in the trade unions which gave 11. Solution (a)
it a militant character. To control the Indian working class Exp) Option a is the correct answer.
the Trade Disputes Act, 1929 was passed. The Tebhaga movement (1946) was a major peasant
The 1929 Act made provisions for establishment of movement that took place in Bengal led by All India Kisan
Courts of Inquiry and Consultation Board with a view Sabha. the peasants challenged the prevailing system of share
to investigate and settle trade disputes. It also stated cropping, demanding two thirds of the produce from land
that incitement to participate in an unlawful strike was a for themselves and one third (tebhaga) for the landlords.
criminal offence, punishable by imprisonment for up to two Tebhaga literally means ‘one-third share’ of harvests.
years. For a lawful strike 15 day’s notice was to be given. Traditionally, sharecroppers held their tenancy on fifty-
fifty basis of the share of the produce, known as barga, adhi,
9. Solution (c) bhagi, etc., all meaning half share.
Exp) Option c is the correct answer. As a result of this movement, Bargardari Bill was passed,
Satya Shodhak Samaj was founded by Jyotirao Phule on which provided that the share of the harvest given to the
September 24, 1873. The goal of the group was fight against landlords would be limited to one third of the total, but the
Brahaminical domination and upliftment of downtrodden Act was not fully implemented.
groups such as women and shudras. He demanded
12. Solution (b)
representation of all classes of the Hindus in all the local
bodies, in services and institutions and also established a Exp) Option b is the correct answer.
primary school. Statement 1 is correct. Brahmo samaj was established by
Raja Ram Mohan Roy in 1828. It supported monotheism
Important Tips
of Vedas and Unitarianism of Christianity. It opposed idol
The aims and objectives of SatyaShodhak Samaj were as worship. Prayers, meditation and readings of the Upanishads
follows: were to be the forms of worship and no graven image, statue
• Spread of education among Shudras to make them or sculpture, carving, painting, picture, portrait, etc., were
aware of their rights and get them out of the influence to be allowed in the Samaj buildings, thus underlining the
of sacred books made by the Brahmins. Samaj’s opposition to idolatry and meaningless rituals.
• To unite all shudras and ati-shudras against Brahminical Statement 2 is correct. It denied the need for priestly class
order and culture. for interpreting religious writings. It discarded the faith in
• Establishing ideal society based on Faith in one god divine incarnation. It denounced polytheism and criticized
without intermediaries like priest and rejection of caste system.
caste system and four varna division of society. The long-term agenda of the Brahmo Samaj was to purify
• Membership of the Satyashodhak Samaj was open to Hinduism and to preach monotheism.
all those who accepted it ideology irrespective of their It took no definite stand on the doctrine of Karma and
caste, sect and religion. transmigration of soul and left it to individual Brahmos to
believe either way.
10. Solution: (b)
Statement 3 is incorrect. The Brahmo Samaj contested the
Exp) Option b is the correct answer. doctrine that the Vedas are infallible. It believed that no
Keshav Chandra Sen is associated with the establishment scripture or man is infallible. The doctrine that the Vedas
of Tabernacle of New Dispensation and Indian Reform are infallible was given by Dayanand Saraswati who founded
Association. He was a social reformer and leader of the the Arya Samaj.

PYQ Workbook 46
HISTORY OF MODERN INDIA

13. Solution (b) Important Tips


Exp) Option b is the correct answer. Cause of death of others Leader:
National Social Conference also referred to social reform • Nana Sahib escaped to Nepal after British took the
cell of Indian National Congress (INC) was formed in 1887 control of Kanpur and he probably perished in the
by Mahadev Govind Ranade. Leaders of INC were of the jungle of Nepal.
view that Congress platform should be devoted to political • Kunwar Singh succumbed to injuries he sustained in
topics only and formed National Social Conference as an the fight against the British on 26 April 1858.
adjunct to INC for this purpose. The Conference advocated • Khan Bahadur Khan escaped to Nepal where the
inter-caste marriages and opposed kulinism and polygamy. Nepal army captured him and returned to the British.
He was tried, sentenced to death, and hanged in the
14. Solution (d) Kotwali (Police Station) on 24 February 1860.
Exp) Option d is the correct answer.
17. Solution: (d)
The British enforced the Forest Act of 1865 and 1878
Exp) Option d is the correct answer.
and established monopoly over the forested land. Also,
the extension of settled agriculture into tribal areas The novel “Anand Math” by Bankim Chandra
Chattopadhyay made the Sanyasi Rebellion famous. The
was contrary to the tribal practice of Shifting Cultivation.
novel, released in 1882, is set during the Bengal famine
Extension of Settled Agriculture led to influx of non-tribals
of 1770 and revolves around the events of the Sanyasi
in the tribal area. This reduced the tribals to labourers in
Rebellion. It follows the story of Mahendra and Kalyani,
lands they had been owning for generations.
a couple facing hardships during the famine, and their
Thus, with the arrival of British, the social and economic involvement with the Sannyasi uprising against British rule.
structure which existed for centuries was disrupted. The novel depicts the rebellion’s objectives, struggles, and the
The handover of tribal lands and the encroachment of characters’ contributions, contributing to the popularization
moneylenders, merchants and British laws generated a lot of the Sanyasi Rebellion in the context of India’s struggle
of pressure on the tribal life. This provided for the common against colonial rule.
grounds for tribal insurrection.
Important Tips
15. Solution: (c) Sanyasi Revolt:
Exp) Option c is the correct answer. • Overview: The Sanyasi Revolt (1770-1779) was a
significant rebellion against British colonial rule in
Bhoodan movement was first started in Andhra Pradesh.
Bengal during the late 18th century. Led by sanyasis
The movement was initiated by Vinoba Bhave in 1951 at
and sadhus in Murshidabad and Baikunthapur
Pochampally village in Nalgonda district, Andhra Pradesh forests of Jalpaiguri, it aimed to challenge British
(now Telangana). Some of the largest donations came from exploitation and oppressive policies. Pandit Bhabani
the Raja of Ranka, who donated 102,001 acres, and the Charan Pathak emerged as a key leader.
Nizam of Hyderabad, who donated 14,000 acres. By 1969, • Cause: The revolt was triggered by a severe famine in
the movement had collected about 4 million acres of land, 1770 and discontent over British economic policies,
which was distributed among 5.3 million families. pushing sanyasis and sadhus to rebel against British
rule.
Important Tips
• Nature: The revolt was marked by armed resistance
The Bhoodan movement was a remarkable example
by these ascetics, symbolizing widespread popular
of voluntary and peaceful social change in India. It
discontent against British authorities.
challenged the feudal system and empowered the poor
and marginalized sections of society. It also inspired many • Objectives: The revolt sought relief from oppressive
other movements such as Gramdan (village gift), Sarvodaya British taxation and demanded local magistrate
(welfare of all), and Antyodaya (upliftment of the last). appointments.
The movement showed the potential of Gandhian ideals • Spread: The uprising spread through incidents like the
and methods in transforming India’s rural landscape. 1770 Baikunthpur revolt and multiple Sanyasi Revolts
in Bengal until 1779.
16. Solution: (d)
• Importance: It was one of the earliest organized
Exp) Option d is the correct answer. protests against British oppression, influencing later
Tatya Tope was betrayed by Man Singh, one of his closest revolts like the Santhal Revolt.
associates. He was captured by the British forces on April 07, • Failure: The revolt faced challenges due to internal
1859, and after a hurried trial for waging war against the East caste conflicts and lack of a unified leadership, leading
India Company, he was hanged on April 18, 1859. to its eventual failure.

47 PYQ Workbook
HISTORY OF MODERN INDIA

18. Solution: (a) 23. Solution: (d)


Exp) Option a is the correct answer. Exp) Option d is the correct answer.
Lord Canning was the Governor-General of India during The correct chronological sequence of the events is:
the Sepoy mutiny of 1857. He had the unique opportunity • Santhal Rebellion (1855-56)
to become the Governor-General as well as the first Viceroy
• Mutiny of the Sepoys (1857)
according to the Government of India Act 1858.
• Indigo Revolt (1859)
19. Solution: (c) • Deccan Riot (1875)
Exp) Option c is the correct answer.
24. Solution: (a)
Jawaharlal Nehru was not associated with the formation of
U.P. Kisan Sabha in February 1918. The U.P. Kisan Sabha Exp) Option a is the correct answer.
was a peasant organization that was set up by Gauri Shankar The correct matching of the movements with their respective
Misra and Indra Narayan Dwivedi, with the support of locations is:
Madan Mohan Malviya, who were members of the Home A. Moplah revolt - 1. Kerala
Rule League. The U.P. Kisan Sabha aimed to mobilize the B. Pabna revolt - 3. Bengal
peasants against the oppressive zamindari system and the
C. Eka Movement - 4. Oudh
colonial government. Jawaharlal Nehru joined the peasant
movement later in 1920, when he visited the villages D. Birsa Munda revolt - 2. Bihar
in U.P. to look at the farmers’ conditions at the behest of Important Tips
Baba Ramchandra, who was another leader of the peasant
• The 1921 Malabar Rebellion, also known as Moplah
movement.
Rebellion or Mappila Rebellion, was an uprising
20. Solution: (b) in Kerala against British colonial rule and the
prevailing feudal system. It encompassed attacks on
Exp) Option b is the correct answer.
colonial symbols, clashes between Mappila peasants
Chittor was not at all affected by the revolt of 1857. Jhansi, and landlords, and resulted in widespread casualties
Jagdishpur and Lucknow actively revolted against the and imprisonment.
atrocious rule of British. • The Eka Movement emerged in 1921 in Hardoi,
21. Solution: (a) Bahraich, and Sitapur (in oudh princely state),
initially supported by Congress and the Khilafat
Exp) Option a is the correct answer.
movement. Led by Madari Pasi, it protested high rents,
Kunwar Singh, a prominent leader of the Revolt of 1857, thekedars’ oppression, and sharecropping. Religious
belonged to Jagdishpur, Bihar. At the age of 80, he led a rituals marked Eka meetings, uniting peasants against
selected band of armed soldiers against the troops under the unjust practices. Leadership shift to Pasi and turning
command of the British East India Company. to violence caused the loss of Congress support.
Repression led to its end by March 1922.
22. Solution: (d)
• The Pabna Peasant Uprising (1873–76) in Bengal,
Exp) Option d is the correct answer. led by Ishan Chandra Roy (“Bidrohi Raja”), was a
Statement 1 is incorrect: Arya Samaj was founded in 1875 peaceful resistance by peasants against oppressive
by Swami Dayananda Saraswati, not in 1835. zamindars. It emerged due to enhanced land taxes,
evictions, and denial of occupancy rights. Peasants
Statement 2 is incorrect: Lala Lajpat Rai was a supporter
declared some areas independent, forming a rebel
of Arya Samaj and its appeal to the authority of Vedas in
army. Famine and police action eventually subdued
support of its social reform programme. He was a member of
the revolt, with authorities ensuring peace and legal
Arya Samaj and also edited its journal, Arya Gazette. resolution.
Statement 3 is correct: Under Keshab Chandra Sen, the
Brahmo Samaj campaigned for women’s education and 25. Solution: (d)
other social reforms. He organised the Bamabodhini Sabha Exp) Option d is the correct answer.
in 1862 to facilitate women’s education and also published M.C. Setalvad, B.N. Rao, and Alladi Krishnaswamy Iyer
the Bamabodhini Patrika, a journal that carried articles on were distinguished members of the Servants of India
women’s issues. Society. They were all eminent lawyers and jurists who
Statement 4 is correct: Vinoba Bhave founded the Sarvodaya contributed to the drafting of the Constitution of India.
Samaj (Society for the Welfare of All) in 1952, which aimed M.C. Setalvad was the first Attorney General of India and
to promote the principles of Sarvodaya through education, the chairman of the First Law Commission of India. B.N.
social service, and community development activities. He Rao was the chief advisor to the Constituent Assembly and
also worked among refugees during the Partition of India later a judge at the International Court of Justice. Alladi
and persuaded them to adopt non-violence and constructive Krishnaswamy Iyer was one of the chief architects of the
work. Constitution and a member of the Drafting Committee.

PYQ Workbook 48
HISTORY OF MODERN INDIA

Important Tips Important Tips


• The Swaraj Party was a political party that emerged • Tirot Sing, also known as U Tirot Sing Syiem, born in
from the Congress in 1923, led by C.R. Das and Motilal 1802 in Mairang, was a Khasi chief who led resistance
Nehru. It aimed to enter the legislative councils and against British attempts to control the Khasi Hills
obstruct the functioning of the British government in in the early 19th century. He fought using native
India. weapons, engaging in guerrilla warfare. Captured in
• The All-India National Liberal Federation was 1833, he died on 17 July 1835 in Dacca. His defiance
another political party that was formed in 1919 by is honored annually as U Tirot Sing Day in Meghalaya.
Surendranath Banerjee and others. It supported • Veerapandiya Kattabomman, an 18th-century
constitutional reforms and cooperation with the Palayakarrar of Panchalankurichi, Tamil Nadu,
British government. resisted British taxation and influence. He led a
• The Madras Labour Union was one of the earliest rebellion, fighting initially from his fort and later
trade unions in India that was founded in 1918 by through guerrilla tactics. Captured with local support,
B.P. Wadia and others. It represented the workers of he was executed on 16 October 1799 in Kayathar after
Buckingham and Carnatic Mills in Madras a 15-day interrogation.
• Syed Ahmad Barelvi, was a 19th-century Indian
26. Solution: (c) Islamic revivalist and military leader who initiated
Exp) Option c is the correct answer. a jihad movement against British colonial rule. He
advocated for puritanical interpretations of Islam,
Mahadev Govind Ranade was a scholar, social reformer,
denounced practices like saint veneration, and aimed
judge, and author who was one of the founding members to establish an Islamic state. His teachings influenced
of the Indian National Congress. He relied on legislation Ahl-i Hadith and Deobandi movements.
to do away with social ills and worked unceasingly for
29. Solution: (b)
the eradication of child marriage, the purdah system, and
other social evils. He also inaugurated the Indian National Exp) Option b is the correct answer.
Social Conference in 1887, which was a forum for discussing Satyashodhak Samaj was a social reform society founded
by Jyotirao Phule in Pune, Maharashtra, on 24 September
social issues and reforms alongside the Indian National
1873. The main aim of Satyashodhak Samaj was to promote
Congress.
education and social rights for the underprivileged groups,
27. Solution: (b) especially women, Shudras, and Dalits, in Maharashtra. The
society also opposed the Brahminical domination and the
Exp) Option b is the correct answer. caste system, and advocated for the equality of all human
Thakkar Bapa (Amritlal Vithaldas Thakkar) is credited beings as the children of one God.
with coining the term “Adivasi” to refer to tribal people in Important Tips
India. Although the term gained popularity later, Thakkar
• The main aim of Prarthana Samaj was to reform
Bapa first used it in his writings and speeches. He was an Hinduism by removing idolatry, casteism, superstition,
Indian social worker who worked for the upliftment of tribal and ritualism, and promoting monotheism, rationalism,
communities, particularly in Gujarat, and played a significant and social service.
role in empowering Harijans. In 1922, he founded the Bhil • The main aim of Arya Samaj was to revive the Vedic
Seva Mandal. Later, he became the general secretary of religion and culture by rejecting the later accretions
the Harijan Sevak Sangh founded by Mahatma Gandhi in of Hinduism, such as idolatry, polytheism, casteism,
1932. The Bharatiya Adimjati Sevak Sangh was founded on and untouchability, and promoting the authority of the
Vedas.
24 October 1948 on his initiative.
• The main aim of Ramakrishna Mission was to spread
28. Solution: (b) the teachings of Sri Ramakrishna Paramahamsa, who
Exp) Option b is the correct answer. was a mystic and a saint who preached the harmony of
all religions and the divinity of all souls.
Surendra Sai, a valiant leader, led anti-British revolts in
Sambalpur. Born in 1809, he fought against British rule
2.2. Other Examination Previous Years’
even before and after the 1857 Revolt. Sai opposed British-
Questions
installed rulers, resisted oppressive policies, and conducted
guerrilla warfare. Despite imprisonment and betrayals, his 30. Solution: (a)
struggle persisted until his death in 1884. His efforts played Exp) Option a is the correct answer.
a significant role in the fight against British colonialism in • D.K. Karve stands for Dhondo Keshav Karve, a social
the region. reformer and educator who was a pioneer in promoting

49 PYQ Workbook
HISTORY OF MODERN INDIA

widows’ education and remarriage. He founded the 32. Solution: (b)


first women’s university in India, the SNDT Women’s Exp) Option b is the correct answer.
University, in 1916. In 1893, Karve established the
The matching of movements with their respective years is as
Widow Re-marriage Association. He also received the
Bharat Ratna, the highest civilian award of India, in 1958. follows:

• J.E.D. Bethune stands for John Elliot Drinkwater A. Pabna Rebellion - 2. 1873-85
Bethune, an English lawyer and educator who was a B. Eka Movement - 3. 1922
supporter of women’s education in India. He founded C. Santhal Rebellion - 1. 1855-56
the Calcutta Female School (now known as Bethune D. Tana Bhagat Movement - 4. 1914
College) in 1849, which was one of the first schools for
girls in India. Important Tips
• The Santhal Rebellion (1855-1856) in Jharkhand
• Ishwar Chandra Vidyasagar was a scholar, writer, and
and West Bengal, India, opposed British East India
social reformer who is considered the father of Bengali
Company’s oppressive revenue and zamindari
prose. He was also a champion of women’s rights and systems. Led by Sidhu and Kanhu Murmu, around
fought for widow remarriage, abolition of child marriage, 60,000 Santhals rebelled, seeking to end exploitation.
and female education. He was the principal of Sanskrit The revolt involved clashes with British troops and
College in Calcutta from 1851 to 1858. local forces, resulting in thousands of casualties. The
• B.M. Malabari stands for Behramji Merwanji Malabari, rebellion was quelled, leading to significant casualties
and destruction, with the rebels lacking firepower
a Parsi poet, author, and publicist who campaigned
against British military.
against child marriage and for the protection of women’s
rights. He wrote a series of letters titled Notes on Infant • The Tana Bhagat Movement (1914-1920) in
Chhotanagpur, British India, opposed British
Marriage and Enforced Widowhood, which influenced
policies and exploitative zamindars, using
the passage of the Age of Consent Act of 1891.
Satyagraha. They, led by Jatra Bhagat, resisted taxes,
31. Solution: (d) zamindars, banias, Muslims, Christians, and British,
aligning with Gandhi’s non-violent principles. In
Exp) Option d is the correct answer. 1920, they joined Indian nationalist and Satyagraha
Sanatan Dharma Rakshini Sabha was a society formed in movements, supporting non-cooperation.
1873 in Calcutta by orthodox Hindus who wanted to defend
33. Solution: (d)
their religion from the criticism and reform of the Brahmo
Samaj and other movements. The society aimed to preserve Exp) Option d is the correct answer.
the eternal (sanatan) principles of Hinduism and to promote Pair 1 is correct: Surendra Sai was the leader from
education, charity, and social service among the Hindus. It Sambhalpur (Odisha) during 1857 revolt. He and his
also opposed the conversion of Hindus to other faiths and supporters carried on with their guerrilla warfare till 1862.
supported the revival of Vedic learning. On 16 May 1862, Surendra Sai surrendered to the British and
the rebellion came to an end.
Important Tips
Pair 2 is correct: The revolt in Ganjam erupted in 1856-
• Bharat Dharma Mahamandala was a prominent
1857 under the active support of the sabaras, a local tribe.
Hindu organization founded by Pandit Din Dayalu
Sharma in Haridwar in 1887. Its objective was to bring It was led by Radhakrishna Dandsena. Captain Wilson was
together all leaders of the orthodox Hindu community sent to control the situation. Wilson set fire to many sabara
and to work together for the preservation of Sanatan villages and destroyed their crops. Finally, Radhakrishna
Dharma. Dandasena and some of his rebel friends were arrested and
executed in 1857.
• Deva Samaj was a religious and social reform society
founded by Shiv Narayan Agnihotri in Lahore in 1887. Pair 3 is correct: Maharaja Gulab Singh Jamwal (1792–
Agnihotri was a former follower of the Brahmo Samaj, 1857) was the founder of Dogra dynasty and the first
but later developed his own system of philosophy and Maharaja of the princely state of Jammu and Kashmir. He
ethics based on the concept of Deva or the Supreme remained loyal to Britishers. The Gulab Singh send his son
Being. Ranbir Singh with 2,000-foot soldiers, 200 cavalry and six
• Radhaswami Satsang was a spiritual movement heavy guns to help the British in the siege of Delhi during
founded by Shiv Dayal Singh in Agra in 1861. Singh 1857 revolt.
claimed to be a manifestation of Radhaswami or the Pair 4 is incorrect: Maulvi Liyaqat Ali led the Revolt of
Lord of the Soul, and taught a method of meditation 1857 in Allahabad (not in Lucknow). The revolt of 1857 in
that involved listening to the inner sound or shabd. Lucknow was led by Begum Hazrat Mahal.

PYQ Workbook 50
HISTORY OF MODERN INDIA

Important Tips 36. Solution: (d)

Places of revolt of 1857 and their leaders: Exp) Option d is the correct answer.

Place of Indian Leaders British Officials “Satyarth Prakash” was written by Swami Dayanand
Revolt who suppressed the Saraswati, a prominent Hindu reformer and the founder of
revolt the Arya Samaj. The book, which translates to “The Light
Delhi General Bakht John Nicholson of Truth,” outlines his philosophical and theological ideas,
Khan promoting the principles of rationalism, Vedanta, and social
Kanpur Nana Sahib Sir Colin Campbell reform.
Lucknow Begum Hazrat Henry Lawrence
Important Tips
Mahal and Sir Colin
Campbell Other books written by Swami Dayanand Saraswati:
Bihar Kunwar Singh William Taylor • Rigvedadi Bhashya Bhumika (Introduction to the
Commentary on the Vedas), which is an introduction
Bareilly Khan Bahadur Sir Colin Campbell
to his commentary on the four Vedas
Jhansi Rani Laxmibai General Hugh Rose
• Vedang Prakash (The Light of the Vedangas), which is
Allahabad Maulvi Liaquat General Neil a treatise on the six auxiliary disciplines of the Vedas
Ali
• Sanskar Vidhi (The Method of Sacraments), which
is a book on the sixteen Hindu sacraments, such as
34. Solution: (d) naming ceremony, initiation, marriage, etc.
Exp) Option d is the correct answer.
37. Solution: (b)
The Ahom Rebellion took place in 1828 and not 1815
in Assam, as a response to British control following the Exp) Option b is the correct answer.
First Anglo-Burmese War. Led by Gomdhar Konwar, the The publication that was started in opposition to the
Ahom prince and his followers rebelled against British
religious and social ideas of Ram Mohan Roy was Samachar
encroachment, formally installing Konwar as king. Their
Chandrika. It was a Bengali newspaper that was founded
attempt to seize Rangpur failed, leading to arrests and trials.
in 1822 by Gangakishore Bhattacharya, a follower of the
Ultimately, the British relented, restoring a portion of the
kingdom to the Assamese ruler Maharaja Purandar Singh orthodox Hindu leader Radhakanta Deb. It was the first
Narendra. newspaper to be printed in Bengali type and the second
oldest Bengali newspaper after Samachar Darpan. It was a
35. Solution: (b)
conservative and reactionary newspaper that criticized
Exp) Option b is the correct answer. the reformist and progressive views of Ram Mohan Roy
The social reformer who strongly opposed the Jury Act of and his Brahmo Samaj. It defended the practices of idolatry,
1826 was Raja Ram Mohan Roy. He opposed the Jury Act of caste system, polygamy, and sati, and attacked the western
1826 as a flagrant injustice and a violation of the principle
education and culture that Roy advocated.
of equality before the law. He prepared and sent a petition
to both houses of the British Parliament, protesting against Important Tips
the act and demanding its repeal. He argued that the act was • Digdarshan was a Bengali monthly magazine that
based on the false assumption that Indians were incapable was founded in 1818 by William Carey, a Baptist
of judging their fellow citizens impartially, regardless of missionary and linguist. It was the first Bengali
their religion. He also pointed out that the act would create periodical and the first Indian magazine to deal
resentment and distrust among the different religious with science and general knowledge. It was edited
communities in India. by Ram Mohan Roy’s brother Ramdulal Dey and
had contributions from Roy himself. It propagated
Important Tips
rationalism, monotheism, and social reforms among
• The Jury Act of 1826 was a law passed by the British the Bengali readers.
government in India that introduced religious
• Samvad Kaumudi was a Bengali weekly newspaper
distinctions in the judicial system of the country.
that was founded in 1821 by Ram Mohan Roy
• According to the act, only Christians had the right to himself. It was the first Indian-owned newspaper and
sit as jurors in trials involving Christians, while Hindus the first to express Indian public opinion. It dealt with
and Muslims were excluded from such cases. various topics such as religion, philosophy, politics,
• However, Christians could also sit as jurors in trials economics, education, and social issues. It advocated
involving Hindus and Muslims, thus creating an for freedom of press, constitutional reforms, abolition
unequal and discriminatory situation of sati, women’s rights, and modern education.

51 PYQ Workbook
HISTORY OF MODERN INDIA

• Bengal Gazette was an English weekly newspaper Important Tips


that was founded in 1780 by James Augustus Hicky, Maulvi Liyaqat Ali:
an Irishman who worked as a surgeon in the British
• Maulvi Liaqat Ali (1817-1892) was born on 05 October
East India Company. It was the first newspaper to be
1817.
printed in India and the first to be published by an
Indian. It was a radical and sensationalist newspaper • Maulvi Liaqat Ali’s father’s younger brother Dayem Ali
that exposed the corruption and scandals of the British married Chanchal Bai who was Rani Lakshmi Bai’s
officials and criticized the policies of Governor- father Moropant’s sister living in Banaras.
General Warren Hastings. It also supported the cause • For this reason, Maulvi Liaqat Ali often addressed Rani
of Indian independence and nationalism Laxmi Bai as Chhabili Behn or Chhoti Behn (younger
sister). Later, to help Rani Laxmi Bai, Maulvi Liaqat
38. Solution: (c) Ali sent his famous Topchi (cannon operator)
Exp) Option c is the correct answer. Khuda Baksh from Allahabad to Jhansi where he got
Statement 1 is correct: Major Burrough was defeated by martyrdom in war.
the Santhals in 1855. A small contingent of force under • On the night of 06 June 1858, the mess of Sixth
Major Burrough was called to suppress the rebels but he Infantry Cantonment of Allahabad was attacked by
met tremendous resistance and was defeated at Pirpainati, the rebellions of Banaras.That was the beginning of
the uprising and the city was thrown at the mercy of
Bhagalpur. The victory of the Santhals over the all mighty
uncontrolled rebellions.
British further fueled the rebellion and it spread like wildfire.
• On 07 June 1858, Maulvi Liaqat Ali took control of
Statement 2 is correct: Gokko was one of the chief rebel the situation.
leaders of the Santhals. He led attacks, organized resistance,
• Maulvi Liaqat Ali made Khusro Bagh his military
and fought against colonial oppression, becoming a
operational headquarters since it was the only fortified
prominent figure in the Santhal rebellion. spacious place available in the city.
Statement 3 is correct: Mahajan Deen Dayal Rai was a • Neill led the attack with his powerful artillery on
moneylender who exploited the Santhals. He was one of the Khusro Bagh on 16 June. Neill successfully seized the
main targets of the Santhal Hool. Khusro Bagh but the Maulvi was out of his reach.
Statement 4 is incorrect: The Santhals never enjoyed full • People were hanged from the trees. Many young
sovereignty over the area of Ganga valley near Muzaffarpur. children were hanged simply because they were beating
They were the tribal people who were subject to the authority drums and parading with green flags in their hands.
of the zamindars and the British. For this reason, the people named Lt. Col. Neill a
Butcher of Allahabad. Many English historians like
39. Solution: (d) Kaye, Malleson, and Charles Ball have written about
Exp) Option d is the correct answer. the barbaric killing of Allahabad citizens.

Option a is incorrect: Col. St. Legar was one of the • Maulvi was able to dodge English officers for a full
fifteen years which was impossible for any ordinary
commander of British army who fought against the rebellions
rebellion.
of Dewan Velu Thampi in 1809.
Option b is incorrect: Lt. Col. Gibbs is also not associated 41. Solution: (a)
with the Revolt of 1857. Exp) Option a is the correct answer.
Option c is incorrect: Col. Wallace took part in Cornwallis’s The Tarkeshwar Movement of 1924 in Bengal was against
operations against Tipu Sultan in 1791–92, including the the corruption in temples. It was a religious reform
siege of Seringapatam in 1792. movement that challenged the abuse of power by the Mahant
(head priest) of the Tarakeswar Shiva temple. The movement
40. Solution: (d) was led by Swami Satyananda and Swami Viswananda, who
Exp) Option d is the correct answer. entered into Satyagraha (non-violent resistance) on 20th
May 1924. The movement soon turned anti-British and anti-
Maulvi Liyaqat Ali led the Revolt of 1857 in Allahabad. He
zamindari, as the Mahant sought help from the colonial
set up his own government and made a series of political authorities and the landlord of British connections.
and military appointments. But soon the city was recaptured
by the Brtish forces under the command of Lt. Col. Neill. 42. Solution: (a)
But, this great freedom fighter escaped from there and was Exp) Option a is the correct answer.
caught after 14 years on September 1871 at Bombay Railway The Hali System was related to bonded labour. It was
Station due to the treachery of his friends. He was tried in the a system where the Dubla or the Hali borrowed money
Court of Law and sentenced to death, but died in captivity from Patidar or other Ulji Paraj and in repayment of it, he
in Rangoon on May 17, 1892. worked as his master’s permanent agricultural labourer for

PYQ Workbook 52
HISTORY OF MODERN INDIA

a lifetime, simply because he could never repay the loan. Important Tips
Consequently, for a Hali the chain of bondage continued
The Bhoodan movement:
from one generation to another.
• The Bhoodan movement was a voluntary land reform
43. Solution: (a) movement in India that was initiated by Gandhian
Exp) Option a is the correct answer. Vinoba Bhave in 1951.
Maulana Shibli Numani was one of the founders of Dar- • The movement aimed to persuade wealthy landowners
ul-Ulum Nadwatul Ulama, an Islamic seminary in Lucknow, to donate a part of their land to the landless peasants,
India, that aimed to reform and uplift the Muslim society who would cultivate it for their own subsistence.
on orthodox lines. He was also a prominent scholar, poet, • One landlord, Ramachandra Reddy, offered to donate
historian, and critic of orientalists. 100 acres of his land, which inspired others to follow
44. Solution: (d) suit. Bhave named this act of generosity as Bhoodan,
meaning land gift
Exp) Option d is the correct answer.
All the three social reformers mentioned; Dayanand 47. Solution: (b)
Saraswati, Iswar Chandra Vidyasagar and Raja Rammohan Exp) Option b is the correct answer.
Roy were known for their proficiency in the Sanskrit
Ishwar Chandra Vidyasagar was a social reformer who
language. They used their linguistic skills to study various
fought for and got widow remarriage legalized in India.
religious and philosophical texts and to write their own
He was an Indian educator, writer, philosopher, and social
works on social and educational reforms.
reformer. He was a key figure of the Bengal Renaissance
• Dayanand Saraswati was a great scholar of Sanskrit, and a pioneer of modern Bengali prose. He advocated for
Persian and English languages and also knew Arabic, the widow remarriage act and cited Hindu scriptures to
Latin and Greek. He wrote his magnum opus Satyarth show that widow remarriage was well within the folds of
Prakash (1875) in Hindi, which was later translated into Hinduism. Through his efforts, Lord Canning enacted the
many languages Widow Remarriage Act throughout British India in 1856.
• Ishwar Chandra Vidyasagar was the principal of Sanskrit
48. Solution: (c)
College in Calcutta from 1851 to 1858. He was a brilliant
student at Sanskrit College, where he received the title Exp) Option c is the correct answer.
Vidyasagar (“Ocean of Learning”), and in 1850 he was Keshav Chandra Sen was a Hindu philosopher and social
appointed head pandit (scholar-teacher) of Fort William reformer who attempted to incorporate Christian theology
College, Calcutta. within the framework of Hindu thought. He was a member
• Raja Rammohan Roy was a great scholar of Sanskrit, of the Brahmo Samaj and later formed his own society called
Persian and English languages and also knew Arabic, the Bharatvarshiya Brahmo Samaj in 1866. He played an
Latin and Greek. He translated the Upanishads into important role in passing the Native Marriage Act of 1872,
Bengali, Hindi, and English and wrote commentaries on which legalized inter-caste and inter-religious marriages
them. among Hindus, Buddhists, Jains, Sikhs, and Christians.

45. Solution: (a) Important Tips


Exp) Option a is the correct answer. • Devendra Nath Tagore: He was a Hindu philosopher
and religious reformer who founded the Brahmo
Jawaharlal Nehru presided over the All-India Trade Union
religion in 1848. He was a leader of the Brahmo Samaj
Congress held at Nagpur in 1929. The split occurred in
and a pioneer of modern Bengali prose. He campaigned
AITUC at the Nagpur session in 1929, where the communists
against sati, polygamy, child marriage, and caste
captured the leadership of the organization and supported
discrimination. He also supported widow remarriage
the affiliation with the Red International of Labour Unions
and women’s rights.
(RILU), also known as Profintern. The reformers, who
wanted to affiliate with the International Federation of Trade • Ishwar Chandra Vidyasagar: He was an Indian
Unions (IFTU), also known as Amsterdam International, educator and social reformer who fought for and got
opposed this move and formed a rival federation called the widow remarriage legalized in India. He also opposed
National Trade Union Federation (NTUF) in 1929. The split child marriage and polygamy and did much to promote
weakened the trade union movement in India and affected the education of girls. He was a key figure of the Bengal
its role in the freedom struggle. Renaissance and a Sanskrit scholar. He translated the
Katha Upanishad and the Bhagavad Gita into Bengali.
46. Solution: (a)
Exp) Option a is the correct answer. 49. Solution: (a)
Bhoodan movement was first started in Andhra Pradesh. Exp) Option a is the correct answer.
The movement was initiated by Vinoba Bhave in 1951 at R.C. Majumdar tried to summarize the revolt of 1857
Pochampally village in Nalgonda district, Andhra Pradesh differently. According to him, the revolt was neither the
(now Telangana). first, nor was it national, nor was it a war of independence.

53 PYQ Workbook
HISTORY OF MODERN INDIA

Important Tips • Book written by Karl Marx: The First Indian War of
Views of otherIndian historians about the Revolt of Independence 1857-1859
1857: • Book written by Sashi Bhusan Chaudhuri: Civil
• Tarachand: It has to be admitted that the war against Rebellion in the Indian Mutinies, 1857-1859
the British was not inspired by any sentiment of • Book written by Puran Chand Joshi: Rebellion in
nationalism, for in 1857 India was not yet politically 1857: A Symposium
a nation.
52. Solution: (a)
• V.D. Savarkar: The upsiring of 1857 was not a sepoy
mutiny. It was a India’s first war of independence. Exp) Option a is the correct answer.
• S. N. Sen: What began as a fight for religion ended as a Shuddhi Movement was started by the Arya Samaj founded
war of independence. by Swami Dayanand Saraswati in 1875 in Bombay to bring
back the people who converted to Islam and Christianity
50. Solution: (c)
from Hinduism. The movement aimed at abolishing the
Exp) Option c is the correct answer. practice of untouchability by converting outcasts from
Option 1 and 2 are incorrect: While British historians other religions to Hinduism and integrating them into the
(not Indian historian) called the revolt the Sepoy Mutiny, mainstream community. The movement also strove to reduce
whereas Indian historians (not British historians) named it the conversions of Hindus to Islam and Christianity, which
the Revolt of 1857 or the First War of Indian Independence. were underway at the time.
It was the first expression of organised resistance.
53. Solution: (d)
Option 3 is correct: Revolt of 1857gave a death blow to
the system of East India Company’s rule in India. The Exp) Option d is the correct answer.
East India Company’s rule in India came to an end as a The correct match between the former revolts in the 19th
result of the uprising. India was now directly governed by century and their related areas is:
the British Crown. The Doctrine of Lapse was disregarded,
A. Kuki revolt - 4 (Tripura)
and the claims of Indian kings were acknowledged.
B. Kuka revolt - 1 (Punjab)
Option 4 is incorrect: The revolt was started due to
British interference in religion and hurting the religious C. Pabna revolt - 2 (Bengal)
sentiments of the Indians. However, there were many D. Birsa Munda revolt - 3 (Bihar)
other reasons. All groups having their own concerns behind
participating in the revolt of 1857. For instances, Zamindars Important Tips
and some rulers wanted to claim their authority in their • The Kuki Rebellion (1860-61) in Tripura was a tribal
territory and peasants wanted the relaxation in taxes, uprising against British and oppressive rule. Led
etc. Hence, it was not only for the improvement of the by Ratan Pooya, Kukis attacked villages, prompting
administrative machinery in India. British intervention. Despite Kuki resistance, British
forces punished leaders, and the rebellion subsided.
51. Solution: (d) The revolt highlighted tribal resilience and defiance
Exp) Option d is the correct answer. against colonial forces.
S.N. Sen can be called as official historian of Indian freedom • The Kuka Movement, led by Baba Ram Singh,
struggle. The book written by the S.N. Sen was the book was a religio-political uprising in Punjab against
published by the Government of India. Book “Eighteen British rule, rooted in opposition to new policies and
Fifty-Seven” was written by well-known Indian Historian religious beliefs. Despite government suppression, the
Dr. Surendra Nath Sen. It was published in May 1957 by movement persisted, leading to social and political
the publication division, Ministry of Information and change. Baba Ram Singh’s leadership and Guru Hari
Broadcasting, Government of India. The book gives an Singh’s continuation shaped its legacy.
unbiased record of the first uprising against the British.
54. Solution: (b)
Books written by other historians given in option are not
officially published by the Government of India. Exp) Option b is the correct answer.
Raja Ram Mohan Roy founded the Atmiya Sabha in 1815,
Important Tips
in Calcutta, as a religious and social reform movement.
The important books and authors on the Revolt of 1857
The Atmiya Sabha aimed to propagate his doctrines of
are as follows:
monotheistic Hinduism and to oppose the superstitions,
• Book written by R.C. Mazumdar: The Sepoy Mutiny
orthodoxy, and oppression that prevailed in Indian society
and the Revolt of 1857
and religion. The Atmiya Sabha later evolved into the
• Book written by Syed Ahmed Khan: Asbab-e- Brahmo Sabha in 1828, which became the Brahmo Samaj,
Baghawat-e-Hind (The Causes of the Indian Revolt) a more influential and organized socio-religious movement.

PYQ Workbook 54
HISTORY OF MODERN INDIA

Important Tips • Begum Hazrat Mahal took over the reigns at Lucknow
• Brahma Samaj was the original name of the Brahmo where the rebellion broke out on June 4, 1857 and
Samaj, a socio-religious movement founded by Raja popular sympathy was overwhelmingly in favour of the
Ram Mohan Roy in 1828. The Brahma Samaj aimed to deposed Nawab of Awadh Wajid Ali Shah. Her son,
propagate the doctrine of monotheism and to reform Birjis Qadir, was proclaimed the nawab and a regular
the Hindu society and religion from superstitions, administration was organised with important offices
orthodoxy, and oppression. shared equally by Muslims and Hindus.
• Tatvabodhini Sabha was a splinter group of the • Rani Lakshmi Bai defended Jhansi against British
Brahmo Samaj, founded by Debendranath Tagore in troops when Sir Hugh Rose besieged Jhansi on 23
1839. The Tatvabodhini Sabha aimed to encourage March 1858. The bombardment of Jhansi began on
religious inquiries and disseminate the essence of the 24 March but was met by heavy return fire and the
Upanishads. damaged defences were repaired.
• The Tatvabodhini Sabha also published a newspaper
called Tattvabodhini Patrika, which propagated the 57. Solution: (a)
new creed and advocated for social reforms. Exp) Option a is the correct answer.
• In 1843, Debendranath Tagore and his followers
Diwan Maniram Dutta was the leader of the revolution of
joined the Brahmo Samaj and merged the
Tatvabodhini Sabha with it. 1857 from Assam. He belonged to a prominent Assamese
family and was employed as a clerk in the British
55. Solution: (c) administration. Maniram Dewan and Piyali Barua were the
Exp) Option c is the correct answer. masterminds behind the execution of the anti-British plot
The founder of ‘Bhartiya Brahmo Samaj’ was Keshav during the revolt of 1857 in Assam. Maniram was the leader
Chandra Sen. He was a social reformer and leader of the and directed the revolt from Calcutta and Piyali Barua
Brahmo Samaj, a socio-religious movement that advocated
executed all the plans as the Chief Lieutenant in Assam.
for monotheism and social reforms. He founded the
Bharatvarshiya Brahmo Samaj, also known as the Brahmo Important Tips
Samaj of India, in 1866 as a separate organization from Maniram Diwan:
the original Brahmo Samaj, which was led by Debendranath
• Maniram Dutta Baruah, popularly known as
Tagore. He introduced some radical changes in the doctrine
Maniram Dewan, was born on 17 April, 1806 in
and practice of Brahmoism, such as accepting Christ as
Charing in Sibsagar district.
an incarnation of God, performing inter-caste and inter-
religious marriages, and allowing women to participate in • He was a counsellor to the Ahom King Purundar
religious ceremonies. He also founded the Indian Reform Singha.
Association in 1870 as a secular organization that aimed to • He was the first Indian to set up a tea garden at
promote education, women’s rights, legal reforms, and social Chenimore in Jorhat.
welfare. • With the discovery of tea bushes in Assam, the
56. Solution: (c) British decided to start tea cultivation there. They
deposed Purandar Singha and took control of the
Exp) Option c is the correct answer.
administration.
On 11 May 1857, a band of rebel sepoys and civilians marched
• He along with Piyali Barua was arrested and found
to Delhi’s Red Fort to seek the leadership of Bahadur Shah
guilty. They were publicly executed by hanging at
Zafar, the Mughal Emperor, for mutiny. Actually the Sepoys
Jorhat jail on 26 February, 1858.
needed someone to legitimise the revolt and they thought
Bahadur Shah Zafar would be the right person at that 58. Solution: (d)
moment. With him on their side, the Sepoys knew that the
Exp) Option d is the correct answer.
mutiny would become a political struggle which is exactly
what the Sepoys wanted. Slavery was made illegal by Act V of 1843, also known as
the Indian Slavery Act, which outlawed many economic
Important Tips
transactions associated with slavery. The act stated that no
Other events related to the revolt of 1857:
public officer shall sell or cause to be sold any person or the
• Kanpur’s Revolt started on June 6, 1857 and Nana
right to the compulsory labour or services of any person on
Saheb took over the leadership. On June 6, 1857 Nana
Saheb and his soldiers attacked the British soldiers the ground that such person is in a state of slavery. The act
of 53rd native infantry at Kanpur. He along with his also declared that no rights arising out of an alleged property
soldiers attacked the entrenchment of the British East in the person and services of another as a slave shall be
India Company which was under General Sir Hugh enforced by any civil or criminal court or magistrate within
Wheeler. the territories of the East India Company.

55 PYQ Workbook
HISTORY OF MODERN INDIA

Important Tips Important Tips


• Child marriage was made illegal by the Age of Consent • The Barrackpore mutiny of Nov 1824 saw Indian
Act in 1891, which raised the age of consent for sexual sepoys rebelling against British officers during the
intercourse for girls from 10 to 12 years. First Anglo-Burmese War. The mutiny arose due to
• Infanticide was made illegal by the Bengal Regulation cultural insensitivity, negligence, and poor supply,
XXI of 1795, which declared infanticide as murder and causing resentment. Troops refused to march, leading
punishable by death to a confrontation. Loyal soldiers and British regiments
surrounded the camp, resulting in casualties. Mutineers
• Sati was made illegal by the Bengal Sati Regulation
were punished, the 47th Regiment disbanded, and its
in 1829, which declared the practice of sati or widow
officers dismissed.
immolation as illegal and punishable by law.
• The Berhampore Mutiny of 27 February 1857
59. Solution: (b) occurred when the 19th Regiment of Bengal Native
Exp) Option b is the correct answer. Infantry refused greased cartridges. Confronted by
cavalry and artillery, the regiment disbanded under
• Bardoli Satyagraha: This was a farmers’ agitation and
censure, led by Lt-Col Mitchell. They were subsequently
nationalist movement in India against the increased disbanded in Barrackpore on 31 March.
taxation of farmers by the colonial government. It
• The Vellore Mutiny on July 10, 1806, marked a
demanded a cancellation of the 22% tax hike being levied
significant Indian sepoys’ uprising against the East
in Bombay Presidency. The movement began on 12 June India Company. Triggered by grievances over changes
1928 and was eventually led by Sardar Vallabhbhai Patel, in dress code and disrespect for religious practices,
who earned the title of ‘Sardar’ (leader) for his role. mutineers seized Vellore Fort, leading to clashes and
• Bhartiya Kisan Vidyalaya: This was not a political party, about 350 mutineer deaths.
but an educational institution established by N.G. Ranga,
61. Solution: (a)
a freedom fighter, classical liberal, parliamentarian and
farmers’ leader. He founded the Bharatiya Vidya Bhavan’s Exp) Option a is the correct answer.
Andhra Pradesh Kendra in Hyderabad in 1956 and Swami Dayanand Saraswati is often referred to as the
started the Bhartiya Kisan Vidyalaya, a school for rural “Martin Luther of India” for his efforts for religious reforms.
children, in 1959. He founded the Arya Samaj, a socio-religious organization
that aimed to promote social equality, rationalism, and the
• Bengal Praja Party: This was a political party of Bengal in
revival of Vedic principles.
the latter days of British India. It was formed by a coterie
of eighteen (of the total of thirty) Muslim members of 62. Solution: (c)
the Bengal Legislative Council following a snap election Exp) Option c is the correct answer.
held in June 1929 with A.K. Fazlul Huq as its leader, and
The person who represented India and Hinduism at the
Abdur Raheem, a Delhi-born Urdu-speaking Calcutta
World Parliament of Religions at Chicago in 1893 was
merchant (not to be confused with Sir Abdur Rahim) as
Swami Vivekananda. He was a Hindu monk and philosopher
the deputy leader.
who propagated the teachings of Vedanta and Yoga. He also
• Bakasht Agitation: This was a peasant movement travelled extensively across India and abroad, spreading his
launched for the abolition of the zamindari system in message of universal brotherhood, spiritual awakening, and
Bihar during 1937-39. The movement was organised social service.
by the Bihar Provincial Kisan Sabha led by Swami
Sahajanand Saraswati. The movement was against the 63. Solution: (c)
eviction of tenants from Bakasht lands by zamindars Exp) Option c is the correct answer.
and led to the passing of the Bihar Tenancy Act and the The founder of Ram Krishna Mission was Swami
Bakasht Land Tax. Vivekananda. He was a disciple of Ramakrishna Paramhansa,
who was a mystic and saint.
60. Solution: (a)
Exp) Option a is the correct answer. Important Tips
The correct match between List-I (Events) and List-II • Swami Vivekananda set up the Ramakrishna Math
(Dates) is: and its twin organisation, Ramakrishna Mission,
in 1886 with the help of other sanyasin disciples of
A. Barrackpur Mutiny - 2. November, 1824 Ramakrishna Paramhansa.
B. Berhampur Revolt - 4. February, 1857 • The headquarters of Ramakrishna Math and
C. Santhal Revolt - 3. 1855-56 Ramakrishna Mission is at Belur Math in West
D. Vellore Mutiny - 1. July, 1806 Bengal, India.

PYQ Workbook 56
HISTORY OF MODERN INDIA

• The Ramakrishna Math and Ramakrishna Mission socio-religious reform organizations in Gujarat and
have several centres and institutions all over the world, British India. It aimed to expose the hypocritical acts
which are engaged in various spiritual, educational, present in Christian, Muslim, and Hindu religions. It also
and social service activities accepted the concept of monotheism and advocated for
social reforms.
64. Solution: (d) • Arya Samaj was founded in Mumbai in 1875 by
Exp) Option d is the correct answer. Dayananda Saraswati. It was a revivalist movement
The Servants of India Society was founded by Gopal within Hinduism that promoted the authority of the
Krishna Gokhale in 1905 as a secular organization that Vedas as the source of true knowledge. It opposed
dedicated itself to the welfare of the Indian people and the polytheism, idol worship, caste system, and other
cause of national liberation. Gokhale was a moderate leader practices. It also introduced proselytization and social
who believed in constitutional methods and cooperation with reforms in Hinduism.
the British government. He also worked for social reforms • Nadwah-ul-Ulama is a council of Muslim theologians in
such as female education, abolition of untouchability, and India that was formed in 1893 in Kanpur. The council
prevention of child marriage. established the Darul Uloom Nadwatul Ulama, a famous
Islamic seminary in Lucknow, on 26 September 1898.
Important Tips The council aimed to reform the prevalent educational
• The Brahmo Samaj of India was founded by Keshav system and to eliminate the differences among Muslim
Chandra Sen in 1866 as a splinter group of the groups on different religious issues
original Brahmo Samaj, which was founded by Raja
Ram Mohan Roy in 1828. Sen was a radical reformer 67. Solution: (a)
who advocated for inter-caste and inter-religious Exp) Option a is the correct answer.
marriages, women’s education, and social service. Velu Thampi led a rebellion against the British in Kerala.
• The Tatvabodhini Sabha was founded by Velu Thampi, also known as Thampi Chempakaraman
Debendranath Tagore in 1839 as a branch of the Velayudhan, was the Dalawa (Prime Minister equivalent) of
Brahmo Samaj, which he later merged with Brahamo the Kingdom of Travancore. He spearheaded an insurrection
Samaj in 1850. Tagore was a conservative reformer who against the British East India Company in the early 18th
emphasized the authority of the Upanishads and the century. Velu Thampi’s rebellion aimed to address internal
Vedanta philosophy. issues, corruption, and British interference in Travancore’s
affairs, ultimately resulting in his confrontation with British
• The Satyashodhak Samaj was founded by Jyotiba
forces.
Phule in 1873 as an independent social movement
that aimed to uplift the oppressed and marginalized 68. Solution: (b)
sections of society, especially the Dalits, women, and Exp) Option b is the correct answer.
peasants.
The founder of Radha Swami Satsang was Shiv Dayal
65. Solution: (b) Singh, also known as Shivdyal Sahab or Soami Ji Maharaj.
Exp) Option b is the correct answer. He was a Hindu banker of Agra, who believed that human
beings could perfect their highest capabilities only through
In 1916, Bal Gangadhar Tilak delivered a speech at the All
repetition of the shabda (“sound”), or nam (“name”), of
India Depressed Classes Conference in Bombay, where God. He revealed himself as the sant satguru (true teacher
he denounced untouchability as a sin and a crime against of spirituality) in 1861 and began instructing a group of
humanity. He said, “If a God were to tolerate untouchability, followers. He wrote two books, both titled Sar Bachan
I would not recognize him as God at all”. He also quoted the (“Essential Utterances”), one in prose and one in verse.
Vedas and the Upanishads to show that untouchability had
no scriptural sanction and was a later invention by selfish 69. Solution: (b)
people. Exp) Option b is the correct answer.
66. Solution: (d) Jyotiba Phule was the author of Gulamgiri, a book that
exposed the oppression and discrimination faced by the
Exp) Option d is the correct answer.
lower castes in India. He dedicated his book to the American
• Brahmo Samaj was founded in Calcutta (now Kolkata) movement to end slavery and compared the plight of the
in 1828 by Raja Ram Mohan Roy. It was a reformist Dalits to that of the African Americans. He was also a social
movement within Hinduism that rejected the authority reformer who founded the Satya Shodhak Samaj, a society
of the Vedas, idol worship, caste system, and other that aimed to uplift the marginalized groups in Maharashtra.
practices. It also adopted some elements of Christianity
and Islam in its beliefs. 70. Solution: (b)
• Manav Dharma Sabha was founded in Surat in 1844 Exp) Option b is the correct answer.
by Durgaram Manchharam Mehta, Dadoba Pandurang The Gadakari rebellion centered in Kolhapur, Maharashtra,
Tarkhadkar, and a few others. It was one of the earliest was a response to British land acquisition following their

57 PYQ Workbook
HISTORY OF MODERN INDIA

occupation of Satara. In 1844, the agrarian Maratha- is credited for having created a revolutionary movement
militia, known as Gadkaris, rebelled against British actions. to integrate Hyderabad State with the Indian Union in
They established a parallel government, taking control of 1948.
forts like Samangarh and Bhudargarh. Although eventually • Abdul Hamid Khan: Popularly known as Maulana
suppressed, this revolt symbolized resistance against colonial
Bhasani, Abdul Hamid Khan was self-educated, village-
rule, driven by issues like unfair land policies and indigenous
based, a fire-brand, and skeptical about colonial
industry decline.
institutions. In 1919, Bhasani joined the non-cooperation
71. Solution: (b) movement and khilafat movement to mark the launching
Exp) Option b is the correct answer. of his long and colourful political career. He went to
The Pagalpanthi rebellion was a rebellion of Garos. The Santosh in Tangail (Southern Assam) to take up the
movement originated in the northern Mymensingh and leadership of the oppressed peasants during the Great
Sherpur District area, mainly inhabited by tribal groups Depression period.
like the Garos. The Pagal Panthis, a socio-religious order
74. Solution: (a)
emerging in late 18th-century Bengal, sought to uphold
religious principles and landless peasants’ rights. Led by Exp) Option a is the correct answer.
Tipu Shah, the movement evolved into armed resistance Khan Bahadur Khan Rohilla was the grandson of Hafiz
against the British and the zamindar system. It was Rahmat Khan, who was the Nawab of Rohilkhand in
predominantly influenced by the Hodi, Garo, and Hajong Uttar Pradesh. On 31 May 1857, Khan Bahadur Khan
tribes, and led by Hodi leader Janku Pathar and Debraj
Rohilla declared independence at Bareilly, the capital of
Pathar. It was eventually suppressed by the British in 1833.
Rohilkhand. He revolted against British when he was 70
72. Solution: (b) years old. Various steps were taken by him to ensure harmony
Exp) Option b is the correct answer. among the Hindus and Muslims. Cow slaughter was banned
by him during the celebration of Hindu festivals. Due to
The leader of the Farazi rebellion was Dudu Miyan, also
the efforts made by Khan Bahadur, the Britishers could not
known as Muhsin Uddin Ahmad. Dudu Miyan was a
significant figure in the Faraizi Movement in Bengal. He split the Hindus and Muslims in this region to fulfill their
took over the movement after the death of his father, Haji imperialist motives.
Shariatullah, and guided the Faraizi movement towards
75. Solution: (a)
a more radical and agricultural focus, establishing an
efficient organizational structure. He aimed to establish Exp) Option a is the correct answer.
a separate state within British-ruled territory and actively The Bhil uprising, which took place in Maharashtra in
participated in the Indian Rebellion of 1857. Dudu Miyan 1818, was a notable tribal revolt against British colonial
was imprisoned by the British administration and eventually rule. The Bhil tribe, deprived of their longstanding forest
released before his death in 1862. rights and subjected to mistreatment, rose in rebellion.
73. Solution: (b) The British responded with military force to quell the
uprising, ultimately leading to a peace treaty, which resulted
Exp) Option b is the correct answer.
in certain concessions being afforded to the Bhil community.
• Sahjanand Saraswati: He was the leader of the peasant
movement in Bihar, where he formed the Bihar Important Tips
Provincial Kisan Sabha in 1929 and mobilised peasant • The Rampa Rebellion (1922-1924), led by Alluri
grievances against the zamindari attacks on their Sitarama Raju, was a tribal uprising in Madras
occupancy rights. He also founded the All-India Kisan Presidency, British India. Triggered by colonial forest
Sabha in 1936 and issued a Kisan Manifesto demanding policies, economic hardships, and forced labor, Raju,
abolition of zamindari and cancellation of rural debts. a charismatic leader, united tribes and sympathetic
• Khudai Khidmatgars: They were a predominantly muttadars against British rule. The rebellion, marked
Pashtun nonviolent resistance movement against the by guerrilla warfare and Raju’s capture in May 1924,
British Raj in colonial India, based in the North- revealed local discontent and ended his messianic anti-
West Frontier Province (now Khyber Pakhtunkhwa, colonial struggle.
Pakistan). They were led by Abdul Ghaffar Khan, also • The Kol uprising (1831-1832) in Chhota Nagpur
known as Bacha Khan or Badshah Khan, who was a was a tribal revolt against East India Company
close associate of Mahatma Gandhi. They wore red exploitation. Led by Budhu Bhagat and others,
shirts as their uniform and participated in various civil Kols rebelled due to encroachments by non-tribals,
disobedience campaigns against the British. forced labor, and oppressive land policies. Tribes
• Swami Ramanand: He participated in Satyagrahas (“non- like Munda, Oraon, and Chero joined. British officer
violent resistance” campaigns) and was imprisoned for Thomas Wilkinson suppressed it by killing leaders and
111 days by Osman Ali Khan. Swami Ramanand Tirtha followers.

PYQ Workbook 58
HISTORY OF MODERN INDIA

76. Solution: (c) • The Age of Consent Act, 1891 was a legislation
Exp) Option c is the correct answer. enacted in British India on 19 March 1891 which
Udham Singh was an Indian revolutionary belonging to raised the age of consent for sexual intercourse for all
the Ghadar Party, who is well known for the assassination girls, married or unmarried, from ten to twelve years
of Michael O’Dwyer on 13 March 1940. Michael O’Dwyer in all jurisdictions, its violation subject to criminal
was the lieutenant governor of Punjab at the time of the prosecution as rape.
Jallianwala Bagh massacre in Amritsa. All others were related 79. Solution: (b)
to the Revolt of 1857.
Exp) Option b is the correct answer.
77. Solution: (c) Raja Ram Mohan Roy was a prominent Indian reformer
Exp) Option c is the correct answer. and leader who brought about social, religious, and
Swami Dayanand Saraswati was responsible for the revival educational reforms in India in the 19th century. He was
of Vedas. He was the founder of Arya Samaj, a reform influenced by the western ideas of rationalism, humanism,
movement of Hinduism that advocated the supremacy of the and democracy and challenged the superstitions, orthodoxy,
and oppression that prevailed in Indian society and
Vedas and opposed idolatry, untouchability, child marriage,
religion. He founded the Brahmo Samaj, a socio-religious
and other social evils. He also gave the slogan of ‘let’s go
movement that advocated for monotheism, abolition of caste
back to Vedas’ and attributed all the beliefs from the Vedic
discrimination, women’s rights, and modern education. He
period until today as a false religion.
also campaigned against the practice of sati, child marriage,
78. Solution: (a) and polygamy and supported widow remarriage. He is
Exp) Option a is the correct answer. widely regarded as the father of Indian renaissance for his
visionary and progressive ideas that shaped modern India.
Option a is correct: After the declaration by Queen Victoria
in 1858, Policy of annexation was abandoned. Policies such Important Tips
as Doctrine of Lapse was disregarded. • Rabindra Nath Tagore was a Bengali poet, writer,
Option b is incorrect: Though it was said in the musician, painter, and Nobel laureate. He also founded
proclamation that ‘Status quo’ of the native rulers would be the Visva-Bharati University at Santiniketan, where he
maintained. But Britishers did not abide by it. For instances, experimented with new forms of education, art, and
rural development.
British wanted to include princely state of Manipur under
British rule in 1891 after the Anglo-Manipur war of • Dayanand Saraswati was an Indian reformer and
1891. Manipur was the last of the independent states to be founder of the Arya Samaj, a reform movement of
Hinduism. He was influenced by the western ideas of
incorporated into British India.
rationalism, humanism, and democracy and challenged
Option c is incorrect: It was said in the proclamation that the superstitions, orthodoxy, and oppression that
equal treatment would be given to all subjects, Indians, and prevailed in Indian society and religion.
Europeans. But it did not happen in reality. For instances, • Vivekananda propagated the teachings of Vedanta and
British classified Indian castes into the ‘martial’ caste and Yoga. He was a disciple of Ramakrishna Paramahamsa
the ‘non-martial’ caste based of allegiance shown during the and the founder of the Ramakrishna Mission and Math.
revolt of 1857. Caste, religion and ethnicity or race became He is best known for his speech at the Parliament of
more crucial while enlisting a soldier. Indians were still not the World’s Religions in Chicago in 1893, where he
promoted to the rank of officers in any of the army station. introduced Hinduism to the western world.
Option d is incorrect: It was said in the proclamation that 80. Solution: (b)
there would be no interference in the social and religious
Exp) Option b is the correct answer.
beliefs of the Indians. But they passed many legislations
to interfere in the social and religious order of the society. The chief center of the Wahabi Movement during the 19th
For instances, The Criminal Tribes Act of 1871, Transfer of century was Patna. The movement, led by Sayyid Ahmad of
Property Act 1882, The Age of Consent Act, 1891, etc. Rae Bareli, aimed to purify and reform Islam in response to
Western influences, emphasizing the Quran and Hadith. It
Important Tips sought to restore the true spirit of Islam and eliminate un-
• In 1871, the British government in India passed the Islamic practices.
Criminal Tribes Act. This Act classified many mobile
craftsmen, traders, and pastoralists as criminal tribes. Important Tips
• The Transfer of Property Act 1882 is an Indian Wahabi Movement:
legislation which regulates the transfer of property in • The Wahabi Movement emerged as a revivalist
India. It contains specific provisions regarding what response to Western influences and the decline
constitutes a transfer and the conditions attached to it. among Indian Muslims, inspired by teachings of
It came into force on 1 July 1882. Abdul Wahab and sermons of Shah Walliullah.

59 PYQ Workbook
HISTORY OF MODERN INDIA

• Shah Walliullah, an 18th-century leader, aimed for child marriage prevented girls from getting an education
unity among four Muslim jurisprudence schools, and from participating in the workforce. Ranade persisted
integrating their best elements, and recognizing in his efforts, and he eventually succeeded in getting the
individual conscience in religious interpretations. Indian government to pass the Child Marriage Restraint Act
• Shah Abdul Aziz and Syed Ahmad Barelvi further in 1929.
popularized Walliullah’s teachings and added a political Option B in List-I matches Option 2 in List-II: In
context. suppression of Thugs, along with William Bentinck, one
• The movement aimed to eliminate un-Islamic practices more name is cherished. This able officer was William
and restore Arabia-like society. Henry Sleimen. Initially he was a soldier and later became
• India was considered Dar-ul-Harb (the land of the the administrator. In 1835, the ‘Thuggee and Dacoity Dept’
kafirs), and it needed to be converted to Dar-ul- was created by William Bentinck and William Henry Sleeman
Islam (land of Islam). was made its superintendent.
• Initially targeting Punjab Sikhs, the movement
Option C in List-I matches Option 1 in List-II: Pandit
shifted focus to British after Punjab’s annexation
Ishwar Chandra Vidyasagar (1820-91), the principal of
(1849).
Calcutta’s Sanskrit College, was largely responsible for
• During the 1857 Revolt, Wahabis played a role in anti-
widow remarriage act 1856. He pushed tirelessly to establish
British sentiment. The movement declined against
a widow remarriage culture in the nation.
British military strength by the 1870s.
Option D in List-I matches Option 4 in List-II: Lord
81. Solution: (c) Hastings played a pivotal role in suppressing the Pindaris
Exp) Option c is the correct answer. by organizing a large army. He personally led the force
Raja Ram Mohan Roy did not protest against western from the north, while Sir Thomas Hislop commanded the
education, but rather supported it and advocated for south. Under his leadership, the British forces successfully
its introduction in India. He himself was well-versed dismantled the Pindari bandits by 1818, ending their menace
in several languages, including English, Persian, Arabic, in the region.
Sanskrit, Bengali, and Hindi. He wrote several books and
84. Solution: (d)
articles in English and Bengali on various topics such as
religion, philosophy, history, politics, and economics. He Exp) Option d is the correct answer.
also established several schools and colleges in India that Various policies of the British Empire were the main cause of
imparted western education along with Indian culture. 1857 revolt. The immediate cause was the cartridge of the
He believed that western education would help Indians to new ‘Enfield Rifle’ which was greased with cow and pig fat.
progress and modernize their society and nation. Other than that their economic policies, political strategies,
He was a prominent social reformer who protested against and administrative factors also played key role behind the
practices such as child marriage, the Sati system, and idolatry. revolt of 1857.
He advocated for social and religious reforms to modernize Important Tips
and uplift Indian society.
Cause of 1857 revolt:
82. Solution: (b) • Political Causes:
Exp) Option b is the correct answer. • Suspicion in the minds of almost all the ruling
Viscount Palmerston was British Prime Minister during the princes against British, through such policies as
revolt of 1857. His tenure was from 1855 to 1858. Similarly, of ‘Effective Control’, ‘Subsidiary Alliance’, and
Winston Churchill (1940-45) was the Prime Minister during ‘Doctrine of Lapse’.
Quit India Movement of 1942 and Clement Attlee (1945- • Economic Causes:
51) was the Prime Minister when India got independence • Heavy taxation
in 1947. • Discriminatory tariff policy against Indian products
83. Solution: (a) • Destruction of traditional handicrafts industry
Exp) Option a is the correct answer. • Socio-Religious Causes:

Option A in List-I matches Option 3 in List-II: M.G Ranade • Racial and a superiority complex characterised the
was a strong advocate for the abolition of child marriage. British administrative attitude towards the native
Indians.
He believed that child marriage was harmful to both the
girl child and the society as a whole. He argued that child • British interference in the socio-religious affairs of
marriage led to early pregnancy, which could have serious the Indian public like abolition of sati, support to
health consequences for the girl child. He also argued that widow-marriage, etc

PYQ Workbook 60
HISTORY OF MODERN INDIA

• Influence of Outside Events: • Social Service League : N.M. Joshi. This was a social
• British suffered serious losses in the the First welfare organization founded by Narayan Malhar Joshi
Afghan War (1838–42), Punjab Wars (1845–49), in 1911 in Bombay. The league worked for improving
and the Crimean Wars (1854–56). The British were the living and working conditions of the urban poor,
seen to be not so strong and it was felt that they especially the industrial workers.
could be defeated. • Seva Samiti : H.N. Kunzru. This was another
organization founded by a prominent member of
85. Solution: (a) the Servants of India Society, Hridayanath Kunzru.
Exp) Option a is the correct answer. Founded in 1914 at Allahabad, Uttar Pradesh, the Seva
The Rajamundri Social Reform Association to encourage Samiti had as its objective to organize social service
widow re-marriage was founded in 1878 by Veeresalingam. during natural disasters like floods and epidemics,
He was a social reformer who fought against hiring nautch to promote the spread of education, cooperation,
girls and spearheaded a campaign for widow remarriage. He sanitation.
also constructed Brahmo Mandir, Widows’ Home, Social 87. Solution: (b)
Reform Association in Madras, Hitakarini Samajam, Girls’
Exp) Option b is the correct answer.
School. He donated his property to these institutions and
inspired an army of youth with his zeal. Assertion (A) is correct: Socio-religious movements of
the 19th century resulted in the modernization of India
Important Tips by introducing reforms in various aspects of society and
• K. Ramakrishan Pillai: He was an Indian nationalist religion, such as education, women’s rights, caste system,
writer, journalist, editor, and political activist. He social evils, and religious tolerance. These movements also
edited Swadeshabhimani (The Patriot), the newspaper contributed to the growth of nationalism, democracy, and
which became a potent weapon against the rule of the secularism in India.
British and the erstwhile princely state of Travancore
Reason (R) is correct explanation of Assertion (A): The
(Kerala, India) and a tool for social transformation.
reason why the socio-religious movements resulted in
• K.T. Telang: He was an Indian lawyer and judge at
the modernization of India was because they adopted
Bombay High Court. He was also an Indologist, a
rationalism, scientific temper and other such ideas as their
Sanskrit scholar, and a social reformer. He was one of
guiding principles. These ideas enabled them to critique
the founding members of the Indian National Congress
and reform the existing social and religious practices and
and the first secretary of INC.
institutions that were oppressive, backward, and stagnant.
• Gopalachariar: He was an Indian lawyer and
These ideas also inspired them to embrace the values of
administrator who acted as the Diwan of Travancore
liberty, equality, fraternity, justice, and humanism that were
from 16 August 1906 to 26 October 1907. He was born
essential for the development of a modern nation.
into a Vaishnavite Brahmin family in Sarukkai, Madras
Presidency in 1850. He worked as a vakil in Madurai 88. Solution: (a)
before being appointed a sub-judge in 1885.
Exp) Option a is the correct answer.
86. Solution: (d) The correct order of the uprisings against the British colonial
Exp) Option d is the correct answer. power in India prior to 1857 is:
The Social Reform Association was founded by Kandukuri • Sepoy mutiny in Bengal (1764)
Veeresalingam Pantulu, a Telugu writer and social reformer. • Sepoy mutiny at Vellore (1806)
He established the association in 1878 in Rajahmundry,
• Kutch Rebellion (1816-32)
Andhra Pradesh, to promote widow remarriage, women’s
education, and social service. • Kol uprising (1831-32)

Shri Ram Bajpai was the founder of the Seva Samiti Boy • Santhal uprising (1855-56)
Scouts Association, a scouting organization for Indian boys. 89. Solution: (c)
He started the association in 1914 in Bombay, on the lines
Exp) Option c is the correct answer.
of the worldwide Baden-Powell Organizations, which at that
time banned Indians from joining it. Raja Ram Mohan Roy, often referred to as the “Father
of the Indian Renaissance,” played a significant role in
Important Tips advocating for social, educational, and religious reforms
• Servants of India Society : G.K. Gokhale. This was in the 19th century. He championed causes such as widow
a society of social workers and reformers founded by remarriage, abolition of the Sati practice, and the promotion
Gopal Krishna Gokhale in 1905 in Pune, Maharashtra. of modern education. His efforts laid the foundation for the
The society aimed to promote education, sanitation, modernization and reform movements in India, earning
health care, and self-reliance among the masses. him the title of the first ‘Modern Man’ of the country.

61 PYQ Workbook
HISTORY OF MODERN INDIA

90. Solution: (d) upliftment of the people. However, they indirectly helped
Exp) Option d is the correct answer. to develop a spirit of patriotism and nationalism among
the Indians by creating a sense of pride in their culture
Behramji M. Malabari was the greatest Parsi social
and heritage. They also inspired many freedom fighters
reformer of the 19th century. He was born in 1853 in a
and leaders who participated in the Indian independence
Persian family in Baroda. He edited a circular against child
movement.
marriage and in support of widow remarriage. He also
wrote several books and articles on social issues, such as 93. Solution: (b)
The Indian Eye on English Life, Gujarat and the Gujaratis, Exp) Option b is the correct answer.
and Infant Marriage and Enforced Widowhood in India. He
The word “Swaraj” was first used in the Indian political
campaigned for the enactment of the Age of Consent Act,
context by Dayanand Saraswati, inspired by the Irish “Home
1891, which raised the minimum age of marriage for girls
Rule” movement. Swami Dayanand’s talks of Swadeshi Raj
from 10 to 12 years. He also founded the Indian Spectator, a
or Swaraj at a time when the country was so backward in
weekly newspaper that advocated for social reforms.
political consciousness. He had already given the slogan
Important Tips ‘India for the Indians, this slogan became the basis of the
• Sir Jamshedji Jeejeebhoy was a Parsi merchant, Indian National Congress during the Calcutta Session.
philanthropist, and knight. He was one of the first The idea of Swaraj have been fully discussed in the sixth
Indians to be knighted by Queen Victoria in 1842. chapter of the Satyartha Prakash. It was Swami Dayanand
• Sir Rustom Behramji Karanjia was a Parsi journalist, Saraswati whose major contribution to Indian Nationalism
editor, and publisher. He was the founder of the Blitz, was firstly for promoting Hindi as national language.
a weekly tabloid that covered politics, entertainment,
94. Solution: (a)
and sports. He was also known for his anti-imperialist
and pro-socialist views. Exp) Option a is the correct answer.
• Navalji Tata was a Parsi industrialist and philanthropist. Mahatma Gandhi gave the title of Sardar to Vallabhbhai
He was the father of J.R.D. Tata, the founder of Tata Patel for his great organizational skill in The Bardoli
Group. He was also involved in various social welfare Satyagraha. It was during the struggle and after the Indian
activities, such as supporting the Indian Institute of National Congress victory in Bardoli Satyagraha, that Patel
Science, the Tata Memorial Hospital, and the Lady Tata first began to be referred to as Sardar. He was a prominent
Memorial Trust. leader of the Indian independence movement and a social
reformer.
91. Solution: (b)
Exp) Option b is the correct answer. 95. Solution: (a)

Govindgiri (1858–1931), also known as Govind Guru Exp) Option a is the correct answer.
Banjara, launched the ‘Lasodia Movement’ for social The British were able to suppress the uprising of 1857 in
reforms amongst the Bhils of Mewar, Bagar, and nearby Rajput states because the local Rajput rulers did not support
regions. He was a social and religious reformer who revolutionaries. The British government suppressed the
aimed to improve the moral character, habits, and tribal rebellions in Rajput states as they had the support
religious practices of the tribal communities. He preached of the Rajput rulers of Mewar, Partapgarh and Dungarpur
monotheism, temperance, forsaking crimes, and encouraged States, etc.
adopting the ways of the upper castes. He faced opposition
from ruling classes but gained a significant following. He was 96. Solution: (b)
arrested and imprisoned for his activities, and the movement Exp) Option b is the correct answer.
had a lasting impact on tribal communities. The immediate cause of India’s first war of independence
was the suspicion about British interference in religion.
92. Solution: (a)
The cartridge of the new ‘Enfield Rifle’ was greased with
Exp) Option a is the correct answer. cow and pig fat and had to be bitten before loading. This
Brahma Samaj, Ramakrishna Mission, and Arya Samaj were was unacceptable to the Hindus and Muslims. The cow
three socio-religious reform movements that emerged in the was sacred to the Hindus, while the pig was taboo for
19th century in India. None of these three had a political the Muslims. This paved the way for immediate protest by
mission but they helped to develop a spirit of patriotism. Mangal Pandey, and the revolt broke out.
They aimed to reform and revitalize Hinduism by rejecting
the superstitions, rituals, caste system, and other social 97. Solution: (d)
evils that had corrupted it. They also promoted rationalism, Exp) Option d is the correct answer.
humanism, and universalism as the core values of Hinduism. Peasant movement in Rajasthan began for the first time
None of these three movements had a political mission from Bijoliya in 1897. It continued in 3 phases between
or agenda, as they focused on the spiritual and moral 1847-1941. Bijolia was a Jagir of the Udaipur State.

PYQ Workbook 62
HISTORY OF MODERN INDIA

• The first phase- Between 1897-1915. It was marked by • The large majority of the sepoys of the Bengal Army
a spontaneous movement that was advanced by local were recruited from the villages of Awadh and eastern
leadership. Important Persons: Initially, Nanji Patel of Uttar Pradesh. Awadh was, in fact, called the “nursery
Berisal and Thakari Patel of Gopal Niwas and later Sadhu of the Bengal Army’’.
Sitaram Das. 99. Solution: (a)
• The second phase- Between 1915-1923. Marked a new Exp) Option a is the correct answer.
state of consciousness among the peasants and it was Swami Vivekananda was a famous reformer and philosopher
led by trained and mature leaders of national status. who wrote the books Jnyana Yoga, Karma Yoga, and Raja
Not only this but this movement also linked up with Yoga, based on his lectures and teachings on the different
the mainstream of the nation. Important Persons: Vijay paths to liberation in Hinduism. These books explain the
Singh Pathik, Manik Lal Verma. concepts and practices of Jnana Yoga (the path of knowledge),
Karma Yoga (the path of action), and Raja Yoga (the path
• The third phase- continued up to 1941 of meditation), respectively. Swami Vivekananda also
Important Tips introduced the term Raja Yoga as a synonym for classical
yoga, which is one of the six orthodox schools of Hindu
• Champaran: This was a peasant movement in Bihar
philosophy. He was a disciple of Ramakrishna Paramahansa,
in 1917, led by Mahatma Gandhi against the British
who was a mystic and saint.
indigo planters who exploited the farmers and forced
them to grow indigo at low prices. 100. Solution: (a)
• Bardoli: This was a peasant movement in Gujarat in Exp) Option a is the correct answer.
1928, led by Sardar Vallabhbhai Patel against the British The campaign for widow remarriage in Maharashtra
government’s decision to increase the land revenue by was led by Vishnu Parashuram Pandit. He was a social
22% despite the crop failure and famine. reformer and a scholar who founded the Widow Remarriage
• Begun movement: It was one of the peasant movements Association in 1850 and also launched the widow re-marriage
in Rajasthan during the British Raj in India. It was a movement. He was inspired by Jyotirao Phule, another social
protest against the high taxes and oppression imposed reformer who advocated for the rights of women and Dalits.
by the feudal lord (jagirdar) of Begun, who was one He also wrote several articles and books on the subject of
of the 16 nobles (solah umrao) of Mewar state. The widow remarriage and social reform.
movement started in 1921 and lasted until 1941, when 101. Solution: (b)
the peasants achieved some relief and reforms.
Exp) Option b is the correct answer.
98. Solution: (b) The founder of Bahujan Samaj was Mukund Rao Patil,
a rich peasant and a patil (village headman) from Satara
Exp) Option b is the correct answer.
district. He was the adopted son of Krishnarao Bhalekar,
The largest number of soldiers who participated during a close associate of Jyotirao Phule, the pioneer of the
struggle of 1857 came from Awadh (Oudh). Due to Satyashodhak (Truth-Seeking) movement. Mukund Rao
unemployment, more than 75% of people participated in Patil was influenced by Phule’s teachings and continued his
Awadh. legacy of fighting for social justice and equality. Mukund
Rao Patil launched a village newspaper called Din Mitra
Important Tips (Friend of the Poor) in 1910, which became the mouthpiece
• Nawab Wajid Ali Shah was dethroned and exiled of Bahujan Samaj.
to Calcutta on the plea that the region was being
Important Tips
misgoverned. The British government also wrongly
assumed that Wajid Ali Shah was an unpopular ruler. • Bahujan Samaj was a social and political movement
On the contrary, he was widely loved. that emerged in Maharashtra in 1910 to oppose the
exploitation of the lower castes by the upper castes,
• The removal of the Nawab led to the dissolution
especially the Brahmins, landlords, and moneylenders.
of the court and its culture. Thus a whole range of
people – musicians, dancers, poets, artisans, cooks, • It also opposed the Indian National Congress, which
retainers, administrative officials and so on – lost their it considered to be dominated by the upper castes and
livelihood. indifferent to the problems of the masses

• In Awadh, more than anywhere else, the revolt became 102. Solution: (a)
an expression of popular resistance to an alien order. Exp) Option a is the correct answer.
• In Awadh where resistance during 1857 was intense Sir Syed Ahmed Khan was the first Indian to have written
and long lasting, the fighting was carried out by in an Indian language on the causes of the Revolt of 1857.
taluqdars and their peasants. In 1859, Sir Syed published the booklet Asbab-e-Baghawat-

63 PYQ Workbook
HISTORY OF MODERN INDIA

e-Hind (The Causes of the Indian Revolt) written in the as the Depressed Classes Mission Society of India and the
Urdu language and later translated in English. The booklet National Social Conference.
analyzes the reasons for the 1857 revolt as such as aggressive
Important Tips
British expansion and ignorance of local customs by British
as the chief reasons • R.G. Bhandarkar was an Indian scholar, orientalist,
and social reformer.
103. Solution: (c) • He was a Sanskrit scholar and a pioneer of
Exp) Option c is the correct answer. Indological research in India.
Tatya Tope was the Commander in Chief of the Indian troops • He was also a member of the Prarthana Samaj and
at Kanpur under the leadership of Nana Saheb. advocated for religious and social reforms among
the Hindus.
Important Tips
• He was awarded the title of Companion of the Order
Tatya Tope: of the Indian Empire in 1911.
• He was also known as Ramachandra Pandurang Tope. • Pandit Ramabai was an Indian social reformer,
• He was born in 1814 in Nashik, Maharashtra. educationist, and feminist.
• He was an intimate friend and the right hand of Nana • She was the first woman to be awarded the titles of
Saheb, the adopted son of Peshwa. Pandita and Sarasvati as a Sanskrit scholar by the
University of Calcutta.
• He forced General Windham to retreat from the city
of Gwalior. • She was also one of the ten women delegates of the
Congress session of 1889.
• He collaborated with Rani Lakshmi Bai of Jhansi to
• She converted to Christianity and founded the
seize Gwalior.
Pandita Ramabai Mukti Mission, a charity for
• He was defeated by Sir Colin Campbell on December destitute women and children.
6, 1857.
• Gopal Ganesh Agarkar was an Indian social reformer,
• He was hanged on April 18, 1859, in Shivpuri in educationist, and thinker.
Madhya Pradesh.
• He was a close associate of Bal Gangadhar Tilak and
104. Solution: (a) co-founded educational institutes such as the New
English School, the Deccan Education Society and
Exp) Option a is the correct answer.
Fergusson College.
Radhakant Deb was a scholar and a leader of the Calcutta • He was the first editor of Kesari newspaper and
conservative Hindu society, who opposed the socio- later started his own periodical, Sudharak, in which
religious reforms initiated by Raja Ram Mohan Roy and he campaigned for social reforms such as widow
others. He defended the practice of sati and formed the remarriage, caste abolition, and female education.
Dharma Sabha, a society that protested against the abolition
of sati by Lord William Bentinck in 1829. He also resisted 106. Solution: (c)
the efforts of Christian missionaries to convert Hindus and Exp) Option c is the correct answer.
advocated for the preservation of orthodox Hinduism. The founder of Dev Samaj was Shiv Narayan Agnihotri.
Important Tips He was a Hindu social reformer who emerged as a leading
• Nemisadhan Bose was an academic and a former vice- member of the Brahmo Samaj, a Hindu reform movement.
chancellor of Visva-Bharati University. He was not a He broke from Brahmoism in 1887 to form his own religious
leader of the Bengali society and did not oppose socio- group, the Dev Samaj (Divine Society). He advocated
religious reforms. reforms of the social system of the Hindus, such as the
elimination of caste discrimination, the education of women,
• Hemchandra Biswas was a freedom fighter and a bomb
widow remarriage and the abolition of child marriage. He
expert. He was associated with the revolutionary group
also claimed to have attained the highest possible plane of
led by Barindra Kumar Ghosh and Aurobindo Ghosh.
existence and that eternal bliss could only be attained with
He went to France to learn the science of bombs and
designed the first Indian national flag in Paris. He was his guidance.
arrested and sentenced to life imprisonment in the 107. Solution: (d)
Alipore Bomb Case.
Exp) Option d is the correct answer.
105. Solution: (b) The first freedom struggle of 1857 failed because of many
Exp) Option b is the correct answer. reasons. Some of them are:
M.G. Ranade was a prominent social reformer and a judge • There was lack of the unity of purpose in Indian sepoys.
of the Bombay High Court. He joined the Prarthana Samaj There was no all India veneer about the revolt. The
in 1869 and became its chief director in Maharashtra. He eastern, southern, and western parts of India remained
also played a key role in other social reform movements such more or less unaffected.

PYQ Workbook 64
HISTORY OF MODERN INDIA

• Many native kings supported British such as Rulers of simply put, meant that two-thirds of the crops, tilled by the
Gwalior, Kashmir, Bhopal and Hyderabad, etc. bargadars and adhiars would have to go to them.
• British Soldiers were better equipped and organised Important Tips
rather than Indians. The Indian soldiers were fighting
The Tebhaga movement:
generally with swords and spears and very few guns and
muskets. On the other hand, the European soldiers were • The Tebhaga movement was a significant peasant
equipped with the latest weapons of war like the Enfield agitation that was started in Bengal by the All India
rifle. Kisan Sabha of peasant front of the Communist Party
of India (1946–1947)
108. Solution: (d) • The movement aimed at improving the share of the
Exp) Option d is the correct answer. peasants who worked as sharecroppers on the lands of
• Raja Ram Mohan Roy was a social and religious the landlords.
reformer who founded the Brahmo Samaj, a movement • The movement sparked off in an area under PS
that advocated for monotheism, rationalism, and Chirirbandar in the district of Dinajpur. The assembled
human rights. He was also a pioneer of modern Indian peasants refused to give fifty per cent to the Zamindars;
education and journalism. He opposed the idolatry instead, they offered thirty-three per cent out of the
and ritualistic worship, and worked towards reviving total yield.
Vedic ideologies. He believed that the purest form of
110. Solution: (d)
Hinduism is contained in the Upanishads, which are
the philosophical texts that expound the essence of the Exp) Option d is the correct answer.
Vedas. The famous play “Neel Darpan” (The Indigo Mirror)
• Keshav Chandra Sen was a Hindu philosopher and was written by Dinabandhu Mitra in 1858–1859. The
social reformer who attempted to incorporate Christian play depicted the oppression of indigo farmers and was
theology within the framework of Hindu thought. He was a significant influence on the Indigo Revolt of 1859 in
a follower of Raja Ram Mohan Roy and later became the Bengal. It also played a pivotal role in the development of
leader of the Brahmo Samaj of India, a splinter group of theatre in Bengal and influenced the establishment of the
the original Brahmo Samaj. He also tried to introduce National Theatre by Girish Chandra Ghosh.
Christian doctrines such as Trinity, Incarnation, and 111. Solution: (b)
Atonement in his teachings. He said that Brahmovad or
Exp) Option b is the correct answer.
the worship of one Supreme Being should be made the
religion of the world. • Poona Sarvajanik Sabha was a socio-political
organization in British India that started with the aim
• Dayanand Saraswati was a Hindu ascetic and founder
of working as a mediating body between the government
of the Arya Samaj, a reform movement of Hinduism. He
and the people of India and to popularize the peasants’
advocated the doctrine of Karma and Reincarnation.
legal rights.
He emphasized the Vedic ideals of brahmacharya,
including celibacy and devotion to God. He believed • It was founded on 2 April 1870 in Pune, Maharashtra,
in the infallible authority of the Vedas as the source of by a group of 95 members elected by 6000 persons
true knowledge and rejected all other scriptures and • Among its founders were Mahadev Govind Ranade, a
traditions as corruptions. He identified Hinduism with scholar, social reformer, judge, and author, and Ganesh
religion instituted in Vedas. Vasudeo Joshi, a lawyer and activist.
• Ramakrishna Paramhansa was a Hindu mystic and • The organization was a precursor to the Indian National
spiritual leader who had a profound influence on the Congress, which started with its first session from
modern Hindu renaissance. He was a devotee of Kali and Maharashtra itself. In 1875, the Sabha sent a petition
experienced various forms of ecstasy in his worship. He to the House of Commons demanding India’s direct
realized that all religions are true and lead to the same representation in the British Parliament.
ultimate reality. He emphasized that there are several
112. Solution: (d)
ways to reach God and one should follow one’s own
chosen path with sincerity and devotion. Exp) Option d is the correct answer.
Statement a is correct: The officers of the company wanted
109. Solution: (b) to spread Christianity in India. Christian missionaries
Exp) Option b is the correct answer. were allowed to enter India and carry on with their mission
The Tebhaga movement was a notable peasant agitation of proselytizing. These activities were looked upon with
that was started in Bengal by the All India Kisan Sabha suspicion and made the sepoys of the East India Company
of the peasant front of the Communist Party of India rebellious.
(1946–1947). It was a fierce peasant uprising on the eve of Statement b is correct: The General Service Enlistment
India’s independence and the partition of Bengal. Tebhaga, Act of 1856 was passed by the British Parliament in 1856.

65 PYQ Workbook
HISTORY OF MODERN INDIA

It required every soldier of Bengal army to go overseas for Wahhabi movement of Arabia and the teachings of Shah
deployment if required. To the religious Hindu of the time, Waliullah of Delhi.
crossing the seas meant loss of caste.
Important Tips
Statement c is correct: The Indian sepoy was equally unhappy
• The Wahabi movement was an Islamist revivalist
with his emoluments compared to his British counterpart.
movement in the 19th century. The movement was
A more immediate cause of the sepoys’ dissatisfaction was
influenced by the teachings of Abdul Wahab of Arabia
the order that they would not be given the foreign service
and Shah Walliullah of India, who advocated for a
allowance (bhatta) when serving in Sindh or in Punjab.
return to the pure and simple Islam of the Prophet
Statement d is incorrect: The inefficiency of the officers Muhammad and his companions.
was not the reason for making the sepoys of the East India
• The Ahmadiyya movement is a modern Muslim
Company rebellious. Officers were mainly British. They
messianic movement that originated in Punjab, India,
had shown their leadership prior to revolt of 1857 such
in 1889. It was founded by Mirza Ghulam Ahmad,
as Robert Clive during Plassey war, Hector Munro during
who claimed to be the Mahdi (Guided One) and
battle of Buxar, and others. Similarly, East India Company
Messiah expected by Muslims and other religions. The
was fortunate in having the services of men of exceptional
movement believes in the finality of prophethood with
abilities in the Lawrence brothers, John Nicholson, James
Muhammad, but also considers Ahmad as a subordinate
Outram, Henry Havelock, etc during the revolt of 1857.
prophet who came to revive Islam.
113. Solution: (b) • The Deobandi movement is a revivalist movement
Exp) Option b is the correct answer. within Sunni Islam that adheres to the Hanafi school of
law. It formed in the late 19th century around the Darul
• Prathana Samaj: This was a social reform society
Uloom Madrassa in Deoband, India, by Muhammad
founded by Atmaram Pandurang in Bombay in 1867. It
Qasim Nanautavi, Rashid Ahmad Gangohi, and several
was inspired by the Brahmo Samaj of Bengal and aimed
others. The movement aimed to purify Islam from
to promote monotheism, rationalism, and human rights
foreign influences and innovations, and to revive the
among the Hindus.
original teachings of the Quran and Sunnah.
• Young India: This was a weekly journal in English
• The Aligarh movement was a socio-religious reform
founded by Lala Lajpat Rai in 1916 and later published
movement among Muslims that began in response
by Mohandas Karamchand Gandhi from 1919 to 1931.
to western influences and British rule in India. It
It was a platform for Gandhi to spread his views on non-
was organized by Syed Ahmad Khan. The movement
violence, civil disobedience, Swaraj, and social issues.
advocated for Western education, scientific rationalism,
• Lokahitavadi: This was the pen name of Gopal Hari and cultural modernization for Muslims
Deshmukh, a social reformer and writer from
Maharashtra. He wrote articles on various social issues 115. Solution: (c)
such as education, women’s rights, caste system, religious Exp) Option c is the correct answer.
superstitions, and economic development.
The Indigo Rebellion, also known as Nil bidroha, was
• Satyashodhak Samaj: This was a social movement a peasant movement that occurred in Bengal in 1859.
founded by Jyotiba Phule in Pune in 1873. It aimed Peasants resisted being forced to grow indigo at prices they
to liberate the oppressed and marginalized sections of found unacceptable. Village leaders and ryots were at the
society, especially the Dalits, women, and peasants. forefront, along with former employees of European planters.
• Rehnumai Mazdayasan Sabha: This was a religious The movement spread across different districts and resulted
reform association founded by Naoroji Furdonji and in violent attacks against indigo factories and planters. It
other progressive Parsis in 1851. It sought to purify the ended with the formation of the Indigo Commission in 1860,
Zoroastrian religion from the influence of Hindu elements which introduced reforms to address the exploitative system.
such as child marriage, polygamy, separate dining of men
and women, and use of nirang for sterilizing customs. 116. Solution: (b)
Exp) Option b is the correct answer.
114. Solution: (a)
Option a is incorrect: Raja of Vijayanagar was not
Exp) Option a is the correct answer.
contemporary of Britishers. So it is incorrect.
Haji Shariatullah was the founder of the Faraizi movement,
Option b is correct: Nizam of Hyderabad did not rebel
a religious reform movement that aimed to purify Islam
against the British East India Company. Hyderabad became
from un-Islamic practices and to uphold the obligatory
duties (farz) of Muslims. He started the movement in 1818 a British protectorate from 1798 until India’s independence
in East Bengal, where he mobilized the peasants against the in 1947. Hyderabad was the British protectorate from 1798.
oppression of the landlords, the indigo planters, and the Option c is incorrect: The poligars of Tamil Nadu gave a
British government. He also opposed the Hindu influence stiff resistance to the British between 1795 and 1805. The
on Muslim culture and society. He was influenced by the problem started in 1781, when the Nawab of Arcot gave

PYQ Workbook 66
HISTORY OF MODERN INDIA

the management and control of Tinneveli and the Carnatic The Waghera Uprising occurred in Baroda. Lasting from
Provinces to the East India Company. This arrangement 1818 to 1820, the Waghera chiefs of Okha Mandal rebelled
caused resentment among the poligars who considered due to both foreign domination and the oppressive demands
themselves as independent sovereign authorities within their imposed by the Gaekwad of Baroda, supported by the British
respective territories. administration. The uprising was pacified through a peace
Option d is incorrect: Dewan Velu Thampi is known for agreement in November 1820.
his role in the rebellion against the British East India 120. Solution: (a)
Company in 1809. He led a group of soldiers and civilians
Exp) Option a is the correct answer.
in an attempt to overthrow British rule in Travancore, but
the rebellion was ultimately suppressed, and he was captured Atmaram Panduranga was the founder of Prarthana Samaj,
and executed. a movement for religious and social reform in Bombay, India,
based on earlier reform movements. Prarthana Samaj was
117. Solution: (b) founded by Atmaram Pandurang in 31 March 1867 when
Exp) Option b is the correct answer. Keshub Chandra Sen visited Maharashtra, with an aim to
Swami Sahajanand Saraswati published a journal called make people believe in one God and worship only one God.
“Hunkar”. He was a peasant leader, a nationalist, and an It became popular after Mahadev Govind Ranade joined.
ascetic who formed the Bihar Provincial Kisan Sabha in 1929 The main reformers were the intellectuals who advocated
and the All India Kisan Sabha in 1936. reforms of the social system of the Hindus.

118. Solution: (c) 121. Solution: (a)


Exp) Option c is the correct answer. Exp) Option a is the correct answer.
Swami Sahajanand Saraswati was elected the first President Madame Helena Petrovna Blavatsky, along with Henry
of All India Kisan Sabha at Lucknow in 1936. He also led Steel Olcott and William Quan Judge, founded the
the Bakasht Movement in Bihar in 1937-1938 against the Theosophical Society in 1875. The society aimed to promote
eviction of tenants from self-cultivated lands by zamindars. spiritual and philosophical exploration, study of ancient
He was arrested during the Quit India Movement and was wisdom traditions, and the search for deeper truths in the
honoured by Subhash Chandra Bose and All India Forward realms of religion, science, and philosophy.
Bloc as a freedom fighter.
122. Solution: (b)
Important Tips Exp) Option b is the correct answer.
• N.G. Ranga: He was an Indian freedom fighter, classical The Kol Mutiny of 1831, led by Buddho Bhagat, occurred
liberal, parliamentarian and farmers’ leader. He was in Singhbhum, Chhota Nagpur, including areas of Ranchi,
the founding president of the Swatantra Party, and an
Hazaribagh, and Palamau. The uprising was sparked by
exponent of the peasant philosophy. He received the
economic exploitation, land tenure changes, and oppressive
Padma Vibhushan award for his contributions to the
practices of the British administration, which led to the
Peasant Movement.
Kols and other tribal communities rebelling against these
• E.M.S. Namboodripad: He was an Indian communist injustices. The rebellion involved violent clashes and
politician and theorist, who served as the first Chief resistance against outsiders, especially moneylenders and
Minister of Kerala in 1957–1959 and then again in landlords, who were seen as exploiting the tribal population.
1967–1969. As a member of the Communist Party of The rebellion was eventually suppressed by British forces.
India (CPI), he became the first non-Congress Chief
Minister in the Indian republic. 123. Solution: (a)
• Acharya Narendra Dev: He was one of the leading Exp) Option a is the correct answer.
theorists of the Congress Socialist Party in India. The revolt of 1857 in Lucknow was led by Begum Hazrat
His democratic socialism renounced violent means Mahal of Oudh (Awadh). She took over the reigns at Lucknow
as a matter of principle and embraced the satyagraha where the rebellion broke out on June 4, 1857.
as a revolutionary tactic. He was active in the Hindi
language movement. He was a key leader of the Important Tips
Congress Socialist Party from its founding in 1934 Revolt of 1857 in Lucknow:
and was imprisoned several times during the freedom • Birjis Qadir, son of Begum Hazrat Mahal, was
struggle. He served as Vice Chancellor of University proclaimed the nawab and a regular administration
of Lucknow from 1947-1951 and then served as was organised with important offices shared equally
Vice Chancellor of Banaras Hindu University from by Muslims and Hindus.
December 1951 to 31 May 1954.
• Henry Lawrence, the British resident, the European
119. Solution: (d) inhabitants, and a few hundred loyal sepoys took
Exp) Option d is the correct answer. shelter in the residency to protect themselves.

67 PYQ Workbook
HISTORY OF MODERN INDIA

• The residency was besieged by the Indian rebels, and • This movement held deep-seated roots in both
Henry Lawrence was killed during the siege. political and religious spheres, with its primary
• The early attempts of Sir Henry Havelock and Sir goal being a resistance against the newly established
James Outram to recover Lucknow met with no British governmental framework.
success. • The movement’s origins can be traced back to the
• Finally, Sir Colin Campbell, the new commander- emotionally charged chanting practices of the
in-chief, evacuated the Europeans with the help of Namdharis, which led to their moniker as the
Gorkha regiments. “Kukas” or shouters.
• In March 1858, the city was finally recovered by the • The Kukas advocated for civil disobedience, the
British, but guerrilla activity continued till September cessation of cow slaughter, and non-cooperation as
of the same year. forms of opposition against the British regime.
• At the forefront of this movement was Baba Ram
124. Solution: (c) Singh, a former soldier, who succeeded the leadership
Exp) Option c is the correct answer. of Baba Balak Singh and Bhagat Jawar Mal.
After the revolt of 1857, the British recruited the Soldiers • The British government took stringent measures
from the Gurkhas, Sikhs, and Punjabis in the North. to suppress Kuka activities, resulting in arrests and
British classified them into Martial Caste. These soldiers confrontations.
supported the British during the revolt of 1857. • The Kukas lent their support to Gandhiji’s non-
cooperation movement and initiated newspapers
125. Solution: (b)
such as Satyug and Kuka.
Exp) Option b is the correct answer.
• The movement’s enduring impact resonates in how
After the Sepoy Mutiny of 1857, the British government it shaped Mahatma Gandhi’s strategies for India’s
sought to bring changes in the military system. Thus, the profound social and political transformation.
Peel Commission was set up in 1857. Peel Commission had
the task of identifying social groups and regions from which 128. Solution: (b)
‘loyal’ soldiers could be recruited. Exp) Option b is the correct answer.
The best representative of Neo-Hinduism in the second
126. Solution: (c)
half of the nineteenth century was Swami Vivekananda.
Exp) Option c is the correct answer. He was a Hindu monk and philosopher who propagated
Mirza Ghalib (1797–1869), the eminent Urdu poet who the teachings of Vedanta and Yoga. He was a disciple
belonged to Delhi witnessed the historic First War of of Ramakrishna Paramahamsa and the founder of the
Independence of 1857 and the subsequent re-occupation Ramakrishna Mission and Math. He is best known for his
of the city by the British. Mirza Ghalib has provided a speech at the Parliament of the World’s Religions in Chicago
fascinating eyewitness account of the uprising of 1857 in his in 1893, where he introduced Hinduism to the western
Persian diaries Dastambu as well as his letters addressed to world. He also travelled extensively across India and abroad,
various friends and confidantes. spreading his message of universal brotherhood, spiritual
awakening, and social service.
127. Solution: (c)
Important Tips
Exp) Option c is the correct answer.
• Ramkrishna Paramhans was a mystic and saint who
The Kuka Movement, also known as the Namdhari practiced various forms of Hinduism, Islam, and
Movement, was organized by Guru Ram Singh. Baba Ram Christianity. He was the guru of Swami Vivekananda
Singh (1816-1885) founded the movement in 1857, after and inspired him to spread his teachings. He is regarded
being a disciple of Balak Singh since 1841. The movement as an avatar of God by some of his followers.
aimed to establish a set of rituals similar to Guru Gobind • Bankim Chandra Chatterji was a Bengali novelist
Singh’s Khalsa, advocating for wearing the five symbols and journalist who wrote several influential works of
while carrying a stick instead of a sword. The movement fiction and non-fiction. He is considered as the father
emphasized abandoning idol worship, maintaining moral of modern Bengali literature and one of the pioneers
virtues, refraining from beef consumption, and protecting of Indian nationalism. He wrote Anandamath, a novel
cattle. that introduced the song Vande Mataram, which
became a symbol of the freedom movement.
Important Tips
Kuka Movement: 129. Solution: (d)
• The Kuka Movement was initiated in Punjab by Baba Exp) Option d is the correct answer.
Ram Singh as a direct response to British authority Sahukar and Landlords did not participate in the revolt
following the year 1849. of 1857. Big zamindars acted as ‘break-waters to storm’.

PYQ Workbook 68
HISTORY OF MODERN INDIA

Most Indian rulers refused to join and often gave active help executed and shot dead by the Britisher. By the end of 1859,
to the British. Whereas Agricultural Labour and Farmers British authority over India was fully re-established. We failed
participated. There were several reforms in the revenue mainly because we lacked unity. The leadership of Bahadur
system and taxation that affected the peasants and labours Shah Zafar did not have popular support as he was just
heavily. This economic factor of Peasants was one of the a symbolic leader. Most of the Indian rulers of important
main reasons for revolt. States shied away from the struggle such as the Scindia of
Gwalior; the Holkar of Indore; the rulers of Patiala, Sindh,
130. Solution: (a)
and other Sikh chieftains; and the Maharaja of Kashmir.
Exp) Option a is the correct answer. Hence, Assertion (A) and Reason (R) both are correct and
British historians had always tried to downplay the Great (R) is the correct explanation of (A).
Revolt of 1857 in its very character. Sir James Outram and
William Tayler said, “ The 1857 revolt was the conspiracy 134. Solution: (a)
of Hindu and Muslims against Britishers. Sir James Outram Exp) Option a is the correct answer.
is best known for his role during the relief and capture of The Barhiyataal resistance in Munger, Bihar, was primarily
Lucknow during the Indian Mutiny. William Tayler was a aimed at the demand for the restoration of Bakasht lands
civil servant of the East India Company who lived in India to the tenants. Bakasht lands were originally held by tenants
from 1829 until 1867. He became commissioner of Patna in but were taken over by landlords in lieu of non-payment of
1855 and in 1857 was involved in the suppression of the 1857 dues. The movement, which took place during the 1930s, was
Sepoy Mutiny. organized in response to the eviction of tenants from these
131. Solution: (b) lands by Zamindars. Led by Swami Sahajanand Saraswati
and supported by the Kisan Sabha, the movement aimed to
Exp) Option b is the correct answer.
protect the rights of the peasants and resist the exploitation
The tribe that rebelled against the Britishers due to the by landlords. The agitation eventually led to the settlement
banning of the human sacrifice system was the Khond of Bakasht lands with the tenants, securing their tenancy
tribe. The Khond Uprising took place in the regions of
rights.
Orissa and Andhra Pradesh, specifically between 1837 and
1856. The Khonds rebelled against British rule because of 135. Solution: (c)
various factors, including the imposition of additional Exp) Option c is the correct answer.
taxes, the intrusion of Zamindars and moneylenders into
Vasudev Balwant Phadke (4 Nov 1845 – 17 Feb 1883)
tribal areas, and the outlawing of the “Mariah” practice,
which involved human sacrifice and was prohibited by the founded the Ramosi Krishak Jatha in Maharashtra. As an
British government. Chakra Bisoi, a young raja and leader Indian freedom fighter and revolutionary, he aimed to end
of the Khonds, led the uprising against the British forces. colonial rule. The Jatha, a peasant organization, protested
The Khonds resisted using traditional weapons like axes, the exploitation of the Ramosi community by zamindars.
hatchets, bows, and arrows, but their efforts were eventually It effectively raised awareness and demanded improved
suppressed by the technologically superior British forces. treatment for the Ramosis.

132. Solution: (b) 136. Solution: (a)


Exp) Option b is the correct answer. Exp) Option a is the correct answer.
Nadars are a Tamil caste of India who were predominantly Nai-Dhobi Band was a form of social boycott in 1919. It was
involved in the cultivation of palmyra trees and jaggery. organised by the panchayats of Awadh to deprive landlords
They also faced discrimination and oppression from the of the services of even barbers and washermen in Pratapgarh
upper castes, especially the Brahmins and the Maravars, who district. This was part of the peasant movement in Awadh
denied them the right to enter temples and worship. led by Baba Ramchandra, who established the Awadh Kisan
In 1899, a group of Nadars from Kamudi or Kamuthi, a town Sabha in 1920.
in Tirunelveli district of Tamil Nadu, attempted to enter the 137. Solution: (b)
Meenakshi Sundareswarar Temple, which was under the
trusteeship of the Raja of Ramnad, Baskara Sethupathy. The Exp) Option b is the correct answer.
Nadars forcefully entered the temple on the night of May 14, Statement 1 is incorrect- Paramahansa Mandali was
1899, and performed rituals in the sanctum sanctorum. The founded in Bombay (now Mumbai) in 1849 by Durgaram
temple entry agitation was an important step in the history Mehtaji and Dadoba Pandurang. It was a secret socio-
of Kamudi and the Nadar community’s struggle for social religious group that aimed to reform Hindu religious thought
justice and equality. and practice. The Mandali was disbanded in 1860 after its
existence was revealed to the public.
133. Solution: (a)
Statement 2 is correct- The Prarthana Samaj was initiated
Exp) Option a is the correct answer. in Maharashtra by Atmaram Pandurang. The primary
The first war of Independence in 1857 failed to secure objective of the Prarthana Samaj was to bring about reforms
freedom from the British Government. The leaders were in Hindu religious thought and practice. One of their key

69 PYQ Workbook
HISTORY OF MODERN INDIA

beliefs was the advocacy of monotheism while strongly was founded after Keshab Chunder Sen, a leader of the
criticizing the practice of idol worship. Brahmo Samaj, visited Bombay in 1864.

Important Tips 140. Solution: (d)


Paramhansa Mandali was based on seven principles of Exp) Option d is the correct answer.
Dadoba:
• Foundation of the Brahmo Samaj: This was a socio-
• Worship of God alone religious reform movement founded by Raja Ram Mohan
• Emphasis on love and moral conduct Roy and Dwarkanath Tagore in 1828 in Calcutta.
• Unity of spiritual religions • Foundation of the Paramhans Mandali: This was a
• Promotion of individual freedom of thought socio-religious group founded by Atmaram Pandurang
• Importance of proper actions and speech in 1848 in Bombay. He was helped by Dadoba Pandurang
and Bal Krishna Jaykar.
• Belief in the unity of mankind
• Foundation of the Arya Samaj: This was a Hindu
• Provision of the right knowledge to all
reformist movement founded by Swami Dayanand
138. Solution: (b) Saraswati in 1875.
Exp) Option b is the correct answer. • Foundation of the Ramakrishna Mission: This was
Statement 1 is correct: The Arya Samaj did have a strong a Hindu religious organization founded by Swami
support base among the trading castes. The movement Vivekananda in 1897. It was inspired by the teachings
gained popularity among the mercantile communities and of Sri Ramakrishna Paramahansa, a mystic saint who
trading castes, particularly in northern India. preached the harmony of all religions and paths to God.
Statement 2 is correct: The Arya Samaj introduced the 141. Solution: (c)
concept of ‘Shuddhi,’ which means mass purification.
Exp) Option c is the correct answer.
Shuddhi aimed at reconverting individuals who had
previously converted to other religions back to Hinduism. Indigo Revolt of 1859 was supported by the intelligentsia of
Bengal. Many educated middle-class Bengalis sympathized
Statement 3 is correct: The moderates in the Arya Samaj
with the cause of the indigo farmers and exposed the
were headed by Hans Raj and Lajpat Rai. They played
atrocities of the British indigo planters. Indian newspapers,
significant roles in the Arya Samaj movement. Additionally,
such as the Hindu Patriot and the Som Prakash, reported on
the Arya Samaj did establish a chain of educational
institutions called Dayanand Anglo-Vedic (DAV) colleges. the plight of the peasants and the violence of the planters.
The play Nil Darpan (Mirror of Indigo) by Dinabandhu
Statement 4 is incorrect: The Arya Samaj had more
Mitra, which depicted the oppression and exploitation of the
members than the Brahmo Samaj, which was another
indigo cultivators, became a powerful tool of propaganda
Hindu reform movement that emerged in the 19th century.
and mobilization.
The Arya Samaj was more popular and influential than the
Brahmo Samaj, especially in northern and western India. Option a is incorrect: The Indigo Revolt did not break out
The Arya Samaj had about 4 million members by 1947, while at Poona and Ahmadnagar, but in Bengal, especially in the
the Brahmo Samaj had less than 100,000 members by then. districts of Nadia, Jessore, Murshidabad, Pabna, Rajshahi
and Malda.
139. Solution: (c)
Option b is incorrect: It was directed against the British
Exp) Option c is the correct answer. indigo planters, who forced the peasants to grow indigo on
Statement 1 is correct: Prarthana Samaj aim was to reform their lands under oppressive and exploitative conditions.
the Hindu society. It advocated for social reforms such The peasants were paid very low prices for their indigo crop
as abolition of caste system, child marriage, and widow and were often trapped in a cycle of debt and bondage.
remarriage, and promotion of female education and
Option d is incorrect: The revolt was not directed against
intercaste marriage.
the moneylenders or the petty zamindars, although they
Statement 2 is correct: It was based in Maharashtra. It was were also part of the oppressive system that supported the
founded in Bombay (now Mumbai) in 1867 by Atmaram planters.
Pandurang and others. It had its greatest influence in and
around Maharashtra state. Important Tips
Statement 3 is correct: It believed in Monotheism. It Facts regarding Indigo Revolt:
followed the teachings of the Marathi saint-poets who • The indigo revolt was led by Bishnu Biswas and
worshipped one God and rejected idolatry and ritualism. It Digambar Biswas who belonged to the Nadia district
also emphasized the unity of all religions and paths to God. of Bengal. Later they were joined by peasant leaders
Statement 4 is incorrect: Prarthana Samaj was influenced from other parts of Bengal, such as Shyamal Mondal
by the Brahmo Samaj, not vice versa. The Prarthana Samaj from Kalna, Burdwan.

PYQ Workbook 70
HISTORY OF MODERN INDIA

• The play Nil Darpan or the ‘Mirror of Indigo’ was Statement 2 is incorrect: The British suppressed the
written by Dinabandhu Mitra. It remains a classic rebellion brutally and many Santhals were killed.
because it was written during the movement in 1859. Statement 3 is correct: The British government constituted
It narrated the sufferings, oppression and struggle of separate administrative units for the Santhals in order to
indigo cultivators. protect them from exploitation.
• The play Nil Darpan was translated into English by Statement 4 is incorrect: There were other major rebellions
poet Michael Madhusudan Dutta and published by in mid 19th century India, such as the Sepoy Mutiny of 1857.
Anglican priest James Long.
Important Tips
142. Solution: (b) The Santhals:
Exp) Option b is the correct answer. • The Santhals were the tribal people who lived in the
Statement 1 is correct: Munro was a British administrator Chota Nagpur plateau in present-day Jharkhand.
who served as Governor of Madras from 1820 to 1827. He
• They were in a desperate situation as their tribal lands
was the architect of the Ryotwari system.
were being leased out to outsiders. The Santhals also
Statement 2 is correct: The Ryotwari system was designed to felt that they were being exploited by the British
eliminate the intermediaries who had traditionally collected officials and moneylenders.
land revenue from the cultivators. This was done in order
• The rebellion was led by Sidhu and Kanhu Murmu,
to give the cultivators direct ownership of their land and to
reduce the amount of revenue that was lost to intermediaries. two Santhal leaders. The rebels quickly gained control
of large parts of the Chota Nagpur plateau. However,
Statement 3 is correct: The Ryotwari system led to high
the British eventually suppressed the rebellion.
tax rates, 50% in dryland and 60% in the wetland, which
gradually impoverished many cultivators. The rates were • After the rebellion, the British government
high and unlike the Permanent System, they were open to constituted separate administrative units called
being increased. Additionally, tax rates were fixed, but the Santhal Parganas for the Santhals. These areas were
prices of agricultural produce fluctuated. As a result, the given special status and the Santhals were given certain
cultivators often found themselves unable to pay the land concessions.
revenue.
145. Solution: (a)
Statement 4 is incorrect: The Ryotwari system was
introduced in parts of Madras and Bombay presidencies, as Exp) Option a is the correct answer.
well as in Assam and Coorg. However, it was not introduced A. The Bakasht Land Movement was led by Karyanand
in Bengal presidency. Sharma (4).

143. Solution: (c) B. The Eka Movement was associated with Madari Pasi (3)
Exp) Option c is the correct answer. C. Mappila Rebellion led by Kunhammed Haji (2).
In 1815, Raja Ram Mohan Roy founded the Atmiya Sabha, D. Avadh Kisan Sabha Movement was associated with Baba
a philosophical discussion circle in Kolkata. The Atmiya Ramchandra (1).
Sabha was a precursor to the Brahmo Samaj, which was
founded by Roy in 1828. Important Tips
Moplah Rebellion,1921:
Important Tips
• Moplah Rebellion was led by the Mappila Muslims
• Ram Mohan Roy established the Brahmo Sabha
that targeted British Officers and Hindu landowners.
in 1828, which evolved into the Brahmo Samaj,
promoting monotheism, rationalism, and social • The succession of the riots was headed by
reforms to challenge orthodox Hindu practices. Variyamkunnath Kunjahammed Haji.

• Debendranath Tagore founded the Adi Brahmo Samaj • The Khilafat movement in Malabar stirred up the
in 1848, advocating monotheism, social equality, and communal sentiments of the Mappila Muslims and led
the abolition of caste practices. to the armed revolt in Malabar.

• Keshab Chandra Sen formed the Brahmo Samaj of • The riots led to the damage of the temples, homes, and
India in 1866, promoting monotheistic worship and the oppression of the Hindus and official authorities of
universal brotherhood. He later split and formed the the British.
Sadharan Brahmo Samaj in 1878, embracing a wider 146. Solution: (b)
spectrum of people and advocating social reforms.
Exp) Option b is the correct answer.
144. Solution: (d) • Bardoli Satyagraha led by Sardar Vallabhbhai Patel was
Exp) Option d is the correct answer. a peasant movement benefiting the Kaliparaj in Gujarat.
Statement 1 is correct: The Santhals were losing their land • Tebhaga Movement led by Kisan Sabha fought for fair
to outsiders and they were feeling increasingly desperate. sharecroppers’ (Bargardars) crops in Bengal.

71 PYQ Workbook
HISTORY OF MODERN INDIA

• Satyashodhak Samaj by Jyotirao Phule uplifted Kunbi • Keshub Chandra Sen was a Bengali social reformer and
peasantry in Maharashtra. religious leader. He founded several schools for girls,
• Ulgulan led by Birsa Munda benefited the Munda tribe and he worked to improve the status of women in
society.
in Jharkhand.
• Ananda Coomaraswamy was an art historian,
Important Tips philosopher, and critic. He was a strong advocate for
Tebhaga Movement(1946-47): Indian culture and heritage, and he worked to promote
• In the early 19th century, affluent peasants called the study of Indian art and philosophy in the West. He did
jotedars gained prominence in Bengal. They amassed not work specifically in the field of women’s education.
vast land holdings, controlled local markets, and 149. Solution: (a)
dominated money lending, exerting power over poor
Exp) Option a is the correct answer.
cultivators.
The Brahmo Samaj was a religious reform movement founded
• Jotedars, concentrated in North Bengal, employed
by Raja Ram Mohan Roy in 1828. The Brahmo Samaj
sharecroppers (bargadars) who cultivated large
rejected the authority of the Vedas and emphasized the
agricultural areas and handed over half of the harvest
worship of one God. It also opposed idolatry and emphasized
to jotedars.
the worship of God without the mediation of priests.
• In late 1946, bargadars challenged the system,
demanding to pay only one-third of the produce and 150. Solution: (d)
store it in their own godowns, backed by the Floud Exp) Option d is the correct answer.
Commission’s recommendation.
The Theosophical Society was founded in New York City in
• The movement was led by the All India Kisan Sabha, 1875 by Helena Petrovna Blavatsky, Henry Steel Olcott,
the peasant wing of the CPI. Initially, participation and William Quan Judge. The society’s stated goals include
was limited, but it grew when the Bengal Bargadars the formation of a “nucleus of the universal brotherhood
Temporary Regulation Bill incorporated their of humanity.” The society has also published numerous
demands in January 1947. works on the subject of Universal Brotherhood, and it has
• Police, at the behest of jotedars, suppressed the held numerous conferences and events on the topic.
sharecroppers, leading to a decline in the movement
Important Tips
by March 1947.
Theosophical Society of India:
• Despite government promises, the bill wasn’t
• Madame H. P. Blavatsky and Colonel Olcott founded
immediately passed. It was only in 1950 that the bill
the Theosophical Society in New York in 1875.
was finally enacted into law.
• Theosophy gained prominence in the Indian
147. Solution: (c) community and civilization in 1879.
Exp) Option c is the correct answer. • The Society relocated its headquarters to Adyar, near
The Arya Samaj split in 1893 over meat-eating vs Madras (Chennai), India, in 1882.
vegetarianism and Anglicised vs Sanskrit-based education. • Annie Besant played a crucial role in popularizing
The Mahatma group, led by Mahatma Hansraj, was vegetarian Theosophy in India and became a valuable member.
and supported Sanskrit education, while the Cultured Party, • Theosophy was a philosophy that merged mysticism,
led by Swami Shraddhanand, was meat-eating and supported spiritualism, and metaphysics, drawing heavily from
Anglicised education. Caste and widow remarriage were Buddhist and Hindu thought.
not major factors. • The Society aimed to foster unity and brotherhood
among its members.
148. Solution: (b)
151. Solution: (a)
Exp) Option b is the correct answer.
Exp) Option a is the correct answer.
• Sister Subbulakshmi was a social reformer and
educationist who played a key role in the development Radhakanta Deb founded the Hindu Dharma Sabha in
of women’s education in India. She founded the Sharada Calcutta in 1830. The main purpose of the organization
was to oppose the current social reform movements led by
Sadan in Chennai, which was one of the first schools for
prominent figures like Raja Ram Mohan Roy and Henry
girls in India.
Derozio.
• Begum Rokeya Sakhawat Hossain was a Bengali writer,
social reformer, and women’s rights activist. She is best Important Tips
known for her utopian novel, “Sultana’s Dream”, which • Ram Mohan Roy was a social reformer and the founder
imagines a world where women are equal to men. She of Brahmo Samaj. He supported Western education
founded the Mahila Samiti, which was one of the first and culture and advocated for the abolition of sati and
organizations to promote women’s education in Bengal. other social evils

PYQ Workbook 72
HISTORY OF MODERN INDIA

• Dwarkanath Tagore was a businessman and a • The author reasons that, ‘both Hindoos and
philanthropist. He was a close associate of Ram Mohammedans are being ruined under the tyranny
Mohan Roy and supported his reformist ideas. He also and oppression of the infidel and treacherous
promoted Western education and trade in India English’.
• Keshab Chandra Sen was a religious leader and a • By making them aware of the injustice done by
social reformer. He was a member of Brahmo Samaj the English the manifesto appeals to landlords,
and later founded his own sect called Naba Bidhan. merchants, public servants, artisans, and people
He advocated for women’s rights, education and social of different religions to accept the Badshahi
service. Government and assures a better economic, social,
and cultural future after the defeat of the British.
152. Solution: (a)
• Finally, warnings are made that anyone who accepts
Exp) Option a is the correct answer. Colonial rule even after the circulation of this
The Tattvabodhini Sabha was a religious organization ishtihar will have his property confiscated and
founded by Devendranath Tagore in 1839. The goal of the imprisoned with his family, and sentenced to death.
Sabha was to promote a more rational and humanist form
155. Solution: (d)
of Hinduism based on the Upanishads. The Sabha published
Exp) Option d is the correct answer.
a journal called Tattvabodhini Patrika, which was a major
forum for the discussion of religious and philosophical Statement I is false: The Oudh Kisan Sabha did bring
issues. under its wing several other Kisan Sabhas. For example,
the Azad Kisan Sabha of Gorakhpur, the Kisan Sabha of
153. Solution: (a) Faizabad, and the Kisan Sabha of Bahraich were all affiliated
to the Oudh Kisan Sabha.
Exp) Option a is the correct answer
Statement II is true: The Oudh Kisan Sabha asked the Kisans
The Revolt of 1857 began in Meerut on May 10, 1857, and
to refuse to till Bedakhli land, not to offer hari and begar.
spread to other parts of the country, including Delhi, Agra,
Bedakhli land was land that had been taken away from
and Kanpur. Ayodhya was not a center of the Revolt of 1857. the Kisans by the landlords. Hari and begar were forms of
The city was under the control of the Nawab of Awadh, forced labor that the Kisans were forced to perform for the
who was a loyal ally of the British. The Nawab suppressed landlords.
any attempts at rebellion in Ayodhya, and the city remained
calm throughout the revolt. 156. Solution: (a)
Exp) Option a is the correct answer
154. Solution: (a)
Swami Sahajanand Saraswati was elected as the president
Exp) Option a is the correct answer. of the All India Kisan Sabha at its eighth session in
Statement 1 is correct: The Azamgarh Proclamation refers Vijayawada in 1944. He was a leading figure in the Indian
to the declaration by the rebels during the Indian Rebellion peasant movement, and he played a major role in organizing
of 1857 (also known as the Sepoy Mutiny or First War of and mobilizing the peasantry against the zamindari system.
Independence). Important Tips
Statement 2 is incorrect: The Azamgarh Proclamation is All India Kisan Sabha:
not associated with the Quit India Movement of 1942 or any
• The All India Kisan Sabha was formed in 1936 at
underground movement during that time. Lucknow.
Important Tips • All India Kisan Sabha is also known as ‘Akhil Bhartiya
The Azamgarh Proclamation: Kisan Sabha.’
• The Azamgarh Proclamation was authored mostly • Swami Sahajanand Saraswati was the head of the
by Firoz Shah, the grandson of Mughal Emperor Sabha.
Bahadur Shah II. The English translation of the • The secretary of this association was NG Ranga.
manifesto was published in the Central Delhi district • The motives of the All India Kisan Sabha were:
of Delhi, by the Delhi Gazette on September 29, 1857.
• To abolish the Zamindari system,
• The manifesto provides a detailed list of the different
• Provide free land to agricultural and other rural
grievances of different sections of the society against
laborers.
the unjust colonial rule, and asserts the ability of the
Mughals to establish their suzerainty over India. • To institutionalize credit.
• In the proclamation, the rulers and chieftains are • To raise the rural masses’ standard of living.
advised to accept the throne of Delhi and stand up for • To put an end to the exploitation of agricultural
their subjects in times of crisis. and other rural laborers.

73 PYQ Workbook
HISTORY OF MODERN INDIA

157. Solution: (a) 159. Solution: (a)


Exp) Option a is the correct answer. Exp) Option a is the correct answer.
The Phulaguri Dhawa, or Phulaguri Uprising, was a Statement 1 is correct- The Arya Samaj was initially located
peasant revolt that occurred in Assam, India, in 1861. It mainly in Punjab and Western Uttar Pradesh. The Arya
was a direct response to the oppressive practices enforced Samaj was a Hindu reform movement that was founded
by the British colonial authorities as part of their opium in 1875 by Dayananda Saraswati. The Samaj was based
policy. The indigenous peasants, primarily from the Ahom on the principles of the Vedas, and it sought to reform
community, were forced to cultivate opium poppy plants Hinduism by removing practices that were considered to
and meet specific quotas imposed by the British. The revolt
be superstitious or harmful.
emerged as a resistance against this exploitation, with the
peasants refusing to comply and actively protesting against Statement 2 is correct- It was popular among the trading
forced labor and economic hardships. It was the Assam’s first castes, as the Samaj’s emphasis on education and social
organized peasant movement. reform appealed to these groups.
Statement 3 is incorrect- The Brahmo Samaj was a more
Important Tips
elite movement than the Arya Samaj, and it had a smaller
• Birsaite Ulgulan [1899-1900]- The Birsaite Ulgulan following. However, the Brahmo Samaj was more influential
was a tribal uprising led by Birsa Munda in the late in the intellectual circles of British India, and it played a
19th century. It was primarily a movement against
significant role in the development of modern Hinduism.
the exploitation and oppression faced by the tribal
communities under British colonial rule. 160. Solution: (d)
• Pabna Revolt [1873]- The Pabna Revolt was an Exp) Option d is the correct answer.
agrarian uprising that occurred in Bengal in 1873. It
Statement 1 is correct: Jyotirao Phule founded the
was mainly a protest against the harsh and exploitative
Satyashodhak Samaj (Society of Seekers of Truth) in 1873
indigo plantation system imposed by the British.
in Maharashtra, India. The organization aimed to challenge
• Maratha Peasant Uprising [1875]- The Maratha and eradicate social inequalities, particularly the caste
Peasant Uprising, also known as the Deccan Riots,
system prevalent in society at that time.
took place in the late 19th century in the Deccan
region of India. It was a protest against the oppressive Statement 2 is correct: Jyotirao Phule challenged the
revenue policies and land revenue collection practices prevailing Brahminical dominance and the hierarchical
introduced by the British. caste system in Indian society. He argued that Brahmins,
who held the highest social status in the caste system, were
158. Solution: (b) not indigenous to India but were descendants of “alien”
Exp) Option b is the correct answer. Aryans who invaded the country.
Statement 1 is incorrect- The Singh Sabhas were a reformist Statement 3 is correct: In the later years of his activism,
movement that began in the late 19th century. The SGPC Jyotirao Phule shifted his focus towards the Kunbi
was not formed until 1920, and it was not the political wing peasantry, a farming community in Maharashtra. Phule
of the Singh Sabhas. believed that the Kunbi community, traditionally considered
Statement 2 is correct- The SGPC was formed in 1920 part of the lower castes, had a strong historical and cultural
as part of the upcoming Akali movement. The Akali connection to the Maratha empire and should be recognized
movement was a protest against the control of Sikh shrines and uplifted.
by the government-appointed mahants, who were often
corrupt and had little regard for Sikh religious practices. 161. Solution: (d)
Statement 3 is correct- The SGPC was founded to Exp) Option d is the correct answer.
reclaim control of the Sikh shrines from the government Statement 1 is correct: Ahmadullah Shah was also known
manipulated loyalist committees. The loyalist committees as Danka Shah or Nakkar Shah, because he used to carry
were appointed by the government to control the shrines, but a drum (danka or nakkar) with him and beat it to gather
they were often corrupt and did not have the best interests people for his speeches and rallies.
of the Sikh community at heart. The SGPC was formed to
replace these loyalist committees with a body that would be Statement 2 is correct: Ahmadullah Shah fought in the
accountable to the Sikh community. Battle of Chinhat on June 30, 1857, in which the British
forces under Sir Henry Lawrence were defeated by the rebels
Statement 4 is correct- The SGPC formed the Akali Dal to
led by Nana Sahib, Bakht Khan and Ahmadullah Shah.
coordinate groups (Jathas) to reclaim control of the shrines.
The Akali Dal was a political party that was formed to support Statement 3 is incorrect: Ahmadullah Shah was not killed by
the goals of the SGPC. The Akali Dal played a key role in the British troops, but by a local zamindar named Raja Jagannath
Akali movement, and it eventually succeeded in reclaiming Singh of Powayan, who betrayed him and shot him when he
control of the Sikh shrines for the Sikh community. came to meet him on June 5, 1858.

PYQ Workbook 74
HISTORY OF MODERN INDIA

162. Solution: (c) Statement II is incorrect: The first step in giving modern
Exp) Option c is the correct answer. education to girls in India was not undertaken by Vidyasagar
in 1800. Vidyasagar was born in 1820 and died in 1891, so he
Statement 1 is correct: The Sannyasi and Fakir uprisings
could not have undertaken any step in 1800.
were a series of rebellions that took place in Bengal and
Bihar between 1767 and 1800, involving a group of Hindu Important Tips
ascetics (Sannyasis) and Muslim mendicants (Fakirs) who Ishwar Chandra Vidhyasagar:
opposed the British rule and exploitation. • Among Vidyasagar’s social reform works are
Statement 2 is correct: One of the reasons for the uprising ‘Bidhobabivah’ on widows’ right to remarry (1855),
was the ban imposed by the British on the free movement and ‘Bahubivah’ on the prohibition of polygamy
of the Sannyasis along their traditional pilgrimage routes (1871), and Balyabivah on the flaws of child marriage.
across Bengal and Bihar. • As a result of his tireless efforts, the then-Government
Statement 3 is correct: Warren Hastings was the Governor- of India passed the Widow Remarriage Act in 1856.
General of Bengal from 1772 to 1785, who faced several • Vidyasagar began working as an ‘Assistant Secretary’ at
challenges from the Sannyasi and Fakir rebels during his the Sanskrit College in 1846.
tenure. In 1773, he issued a proclamation banishing all
Sannyasis from Bengal and Bihar, declaring them as outlaws 165. Solution: (a)
and enemies of the state. Exp) Option a is the correct answer.
Statement 4 is incorrect: The Non-Cooperation Movement Statement I is correct: The annexation of Awadh by Lord
was a mass civil disobedience campaign launched by Dalhousie in 1856 did have adverse effects on the financial
Mahatma Gandhi and the Indian National Congress conditions of the sepoys. The annexation resulted in the
against the British rule in India in 1920-22. The Sannyasi dissolution of the princely state of Awadh and the elimination
and Fakir uprisings took place much earlier, in the late of the ruling Nawab’s authority. As a consequence, the sepoys,
18th century, soon after the British acquired political power who were predominantly recruited from Awadh, lost their
in Bengal following their victory at the Battle of Buxar in privileged positions and the economic benefits they received
1764. from the Nawab’s court.
Statement II is correct: After the annexation, the British
163. Solution: (b) implemented new land revenue policies in Awadh, which
Exp) Option b is the correct answer. included higher taxation. The sepoys, who were landowners
Both the statements are individually true but Statement II or had family members residing in Awadh, had to bear the
is not the correct explanation of Statement I. burden of increased land taxes. This financial burden further
worsened their economic conditions and added to their
Statement I is correct: Ram Mohan Roy, a prominent social grievances.
and religious reformer in 19th-century India, put forward
weighty arguments against belief in many Gods and 166. Solution: (b)
advocated for the worship of a single God. His work “Gift Exp) Option b is the correct answer.
to Monotheism” reflects his critique of polytheism and his Mr AO Hume was not associated with the activities of the
promotion of monotheistic ideas. Theosophical Society. Mr AO Hume was a British civil
Statement II is correct: Ram Mohan Roy, in his work servant and political reformer who founded the Indian
“Precepts of Jesus,” aimed to separate the moral and National Congress in 1885. He was influenced by the ideas
philosophical teachings of the New Testament from the of Theosophy, but he was not a member of the Theosophical
theological aspects of Christianity. He emphasized the Society.
universal moral principles and ethical teachings of Jesus
167. Solution: (a)
while downplaying specific Christian doctrines.
Exp) Option a is the correct answer.
While both statements describe different aspects of
Ram Mohan Roy’s works, there is no direct explanatory Azimullah Khan served as the Chief Advisor and Minister
of Nana Saheb. He played a key role in the negotiations that
relationship between the two statements.
terminated the Siege of Cawnpore (Kanpur) . Representing
164. Solution: (c) Nana Sahib, Azimullah met with the British commander
Exp) Option c is the correct answer. of the garrison Major-General Sir Hugh Wheeler and
agreement was reached that the garrison and their families
Statement I is true but Statement II is false would be evacuated and taken by boat to safety in Allahabad.
Statement I is correct: The Bethune School, founded in His powerful poetic expressions brought about an
Calcutta in 1849 by John Elliot Drinkwater Bethune, was atmosphere of closely integrated coexistence of Hindu and
one of the earliest institutions established as a result of Muslim communities. The patriotic song was composed by
the growing movement for women’s education in the 19th revolutionary Azimullah Khan and published in a news daily
century. It played a significant role in promoting education “ Payme Azadi” patronized by Nana Sahib. One original
for girls and women in India. copy of it has been preserved by British Museum, London.

75 PYQ Workbook
HISTORY OF MODERN INDIA

168. Solution: (d) named after its sponsor Harbilas Sharda, a judge and Arya
Exp) Option d is the correct answer. Samaj member.
Birsa Munda, known as ‘Dharti Aaba’ (Father of Earth), The act aimed to prevent child marriage, which was a
revived traditional tribal culture and opposed British prevalent and harmful practice in Indian society. Child
Christian missionary activities. He preached against marriage had adverse effects on the health, education, and
religious conversions and taxes imposed by the Church. rights of girls and boys. The act made it illegal for anyone
Birsa became a healer, miracle-worker, and preacher, to perform, conduct, or direct any child marriage and
gaining a significant following among the Mundas, Oraons, prescribed penalties for those who violated it.
and Kharias. His teachings emphasized the importance 171. Solution: (a)
of returning to their original tribal religious practices,
Exp) Option a is the correct answer.
earning him the title ‘Dharti Aaba.’
William Howard Russell was an Irish reporter with the
Important Tips “Times” and is considered to have been one of the first
Munda Ulgulan: Tribal Uprising modern war correspondents. He was sent to India by The
• Origin and Provocation: Times to report on the conflict of 1857–1859 known as the
• The rebellion emerged among the Munda tribe in Indian Mutiny. He written in the newspaper “Times” that
Chhota Nagpur, Jharkhand, triggered by British after the 1857 revolt, nobody in the North India looks at
colonialism, zamindars, and missionaries. white man’s car with friendly view.

• Disruption of Traditional Land Ownership: 172. Solution: (a)


• The Munda’s Khuntkatti System, involving Exp) Option a is the correct answer.
collective land ownership, was undermined by R.C. Majumdar and S.N. Sen conclude that the revolt of 1857
outsiders becoming landowners, forcing Mundas was “Not an organized ‘national’ revolt”. They consider
into labor roles. that Indian nationalism in the middle of the 19th century
• Leadership of Birsa Munda: was in the Embryonic stage. The wave of Nationalism
• Birsa Munda emerged as a leader, championing was emerged in 19th-century in British India as a reaction
Munda rights, traditions, and independence. against the consolidation of British rule and due to spread of
The 1894 revolt aimed to establish a ‘Munda Raj,’ Western civilization.
targeting police stations and government property.
173. Solution: (a)
• Influence and Outcomes:
Exp) Option a is the correct answer.
• Though suppressed in 1900 with Birsa’s capture and
The founder of the Adi Brahma Samaj was Devendranath
demise, the rebellion left a lasting impact on tribal
Tagore. He was a social reformer and leader of the Brahmo
movements.
Samaj, a socio-religious movement that advocated for
• The British enacted the Chotanagpur Tenancy Act monotheism and social reforms. He founded the Adi
in 1908, safeguarding tribal land rights. Brahmo Samaj in 1866 as a separate organization from the
• “Khuntkatti” rights were recognized, and “Beth original Brahmo Samaj. He introduced some conservative
begari” was prohibited, securing legal protection changes in the doctrine and practice of Brahmoism, such as
for tribal land rights. rejecting Christ as an incarnation of God, opposing inter-
caste and inter-religious marriages, and emphasizing the
169. Solution: (b)
authority of the Vedas.
Exp) Option b is the correct answer.
174. Solution: (a)
Mangal Pandey incident took place at Barrackpore. Mangal
Pandey, a sepoy of the 34th Bengal Native Infantry (BNI), Exp) Option a is the correct answer.
attacked two British officers in Barrackpore on March 29, Fazil Mohammad Khan from Bhopal led the main
1857. He was overpowered and executed on April 8, 1857 movement and sacrificed his life during the first struggle
while his regiment was disbanded in May. However, the for Independence in 1857. Adil Mohammed Khan and Fazil
revolt of 1857 is formally supposed to be begun from Meerut Mohammed Khan, two brothers, became one of the chief
on May 10, 1857. rebel leaders from Bhopal region during 1857 revolt. They
were in the service of the Bhopal state which was loyal to the
170. Solution: (a)
British, Adil Khan and his brother Fazil Khan took up arms
Exp) Option a is the correct answer. against the East India Company. They urged the sipahis of
The Sharda Act is the commonly used name for the Child the Bhopal regiment to join the uprising and throw out the
Marriage Restraint Act, which was passed in 1929 by the foreigners. Encouraged by his call, the native soldiers seized
Imperial Legislative Council of India. The act fixed the age the Sehore cantonment on 5th August 1857. Shortly after,
of marriage for girls at 14 years and boys at 18 years. It he attacked Garhi-Ambapani and Pathari and captured both
was a result of the social reform movement in India and was territories. In December, a military unit under Hugh Rose

PYQ Workbook 76
HISTORY OF MODERN INDIA

was dispatched to deal with Khan’s forces, and by January control of the region again. He managed to evade capture and
1858, they besieged the Rahatgarh fort, arrested and executed continued to wage further struggle till September 1874, when
Fazil Khan on 29 January 1858. he was finally captured and executed. Chandrasekhar Azad
175. Solution: (a) and Ramprasad Bilsmil were associated with Hindustan
Republican Association (HRA) whereas Makhan Lal
Exp) Option a is the correct answer.
Chaturvedi was an Indian poet, writer, essayist, playwright
Sister Nivedita was a disciple of Swami Vivekananda and a
and a journalist. All these three were born after 1857.
social worker, author, and teacher. She wrote many letters
to her friend Miss MacLeod, expressing her hatred for British 180. Solution: (d)
rule. One such letter says “The British empire is rotten to the
Exp) Option d is the correct answer.
core, corrupt in every direction, and tyrannical and mean.”
She was also known as ‘Lokmata’ (Mother of People) by Gopal Hari Deshmukh was a social reformer and writer
Rabindra Nath Tagore, ‘Real Lioness’ by Vivekananda, and from Maharashtra, who wrote under the pen name
‘Agnishikha’ (the flame of fire) by Aurobindo Ghose. Lokahitwadi (meaning “one who speaks for the welfare of
the people”). He was a judge and a member of the Governor-
176. Solution: (d)
General’s Council in 1880. He advocated for self-reliance,
Exp) Option d is the correct answer. Western learning, humanitarianism, and social service as
Ram Mohan Roy was adorned with the title of ‘Raja’ by the means to improve the condition of India. He also wrote
Mughal Emperor Akbar-II. Akbar II was the 19th Mughal against the evils of casteism, superstition, and orthodoxy,
emperor who ruled from 1806 to 1837. He was the son of and supported the cause of women and education.
Shah Alam II and the father of Bahadur Shah Zafar. He sent
Ram Mohan Roy as his ambassador to Britain in 1831 to Important Tips
plead for his pension and restoration of his powers. He also • Gopal Hari Deshmukh also known as “Lokahitawadi”,
conferred him with the title of ‘Raja’ in recognition of his was a writer and social reformer from Maharashtra. He
services, contribution and talent. was born on 18 February 1823 and died on 9 October
1892.
177. Solution: (c)
• He began writing articles aimed at social reform in
Exp) Option c is the correct answer.
Maharashtra at the age of 25, under the pen name
The first leader to organize the labor movement in India Lokhitawadi, for the weekly Prabhakar.
was N.M. Lokhande. Narayan Meghaji Lokhande played
• In 1880, he was a member of the Governor General’s
a significant role in improving the working conditions of
Council and a judge.
textile mill workers in the 19th century. He founded the
Bombay Mill Hands Association and later established the • He was a strong supporter of women’s rights and a
Mumbai Kamgar Sangh. His efforts led to important labor proponent of female education.
rights for workers, such as weekly holidays, designated work
181. Solution: (d)
hours, and timely salaries. He is widely recognized as the
Father of the Trade Union Movement in India. Exp) Option d is the correct answer.
Mahatma Gandhi was born in Porbandar, a coastal town
178. Solution: (c)
in Gujarat, on 2 October 1869. Gandhinagar, the capital
Exp) Option c is the correct answer.
city of Gujarat, was named after Mahatma Gandhi, but it
In India, the term First War of Independence was first was established in 1960, more than a decade after his death
popularized by Vinayak Damodar Savarkar in his 1909 Gandhinagar is about 400 km away from Porbandar.
book “The History of the War of Indian Independence”,
which was originally written in Marathi. He wrote about Important Tips
the guerilla warfare tricks used in the 1857 Sepoy Mutiny. • Dr. Annie Besant was a British socialist, theosophist,
Jawaharlal Nehru, the first Prime Minister of India, insisted women’s rights activist, and supporter of Indian self-
on using the term First War of Independence to refer to rule. She became a member of the Theosophical Society
the event of 1857, and the terminology was adopted by the in 1890 and its president in 1907. The Theosophical
Government of India. Society was a religious movement that promoted the
study of ancient wisdom and the unity of all religions.
179. Solution: (c)
• The International headquarters of the Theosophical
Exp) Option c is the correct answer.
Society is located in Adyar, Madras (now Chennai),
Saadat Khan was the leader of the revolution of 1857 in India. It was established there in 1882 by Helena
Indore. He is revered for not only defeating the British but Blavatsky, one of the founders of the society, who
also compelling them to flee their base within a few hours of moved from New York to India. The headquarters
the encounter that took place on 01 July 1857. It was only consists of a campus with various buildings, gardens,
on 15 December, 1857 that the British forces managed to get libraries, and a temple.

77 PYQ Workbook
HISTORY OF MODERN INDIA

• Swami Dayanand Saraswati was an Indian philosopher, Important Tips


social reformer, and founder of the Arya Samaj, a Chapati movement, 1857:
Hindu reform movement that advocated the authority
• In March 1857, Dr. Gilbert Hadow, an army surgeon
of the Vedas and rejected idolatry, caste system, and
in the employ of East India Company, wrote the
ritualism.
following lines describing a strange movement afoot
182. Solution: (d) in 1857, in a letter to his sister in Britain. “There is
a most mysterious affair going on throughout the
Exp) Option d is the correct answer.
whole of India at present. No one seems to know the
Raja Ram Mohan Roy was a social and religious reformer meaning of it. It is not known where it originated, by
who opposed the practice of sati and worked tirelessly whom or for what purpose, whether it is supposed to
to abolish it. He witnessed the death of his sister-in-law be connected to any religious ceremony or whether it
has to do with some secret society. The Indian papers
through sati when he was a child and was deeply affected
are full of surmises as to what it means. It is called the
by it. He wrote several articles and petitions against sati
chapati movement.”
and challenged the orthodox Brahmins who justified it on
• Thousands of unmarked chapatis were distributed
religious grounds. He also formed an association of like-
to homes and police outposts throughout India by
minded people who helped him in his crusade against sati. runners at night, and the people who accepted the
offerings would quietly make more batches and pass
Important Tips
them on.
Other important facts:
• Extensive enquiries were conducted for understanding
• William Bentinck was the Governor-General of
the meaning of this bizarre distribution but nothing
India from 1828 to 1835. He passed the Bengal Sati
concrete was found. As there was not a word written on
Regulation in 1829, which declared the practice of sati
or sign made on the chapatis, the British were unable
or widow immolation as illegal and punishable by law.
to find grounds for stopping or arresting the chapati
• The Hindu Widows’ Remarriage Act was passed in runners who were quite often police chowkidars
1856 by the British Parliament, which allowed Hindu
themselves.
widows to remarry legally and inherit their deceased
husbands’ property. 185. Solution: (a)
• Swami Dayanand Saraswati was a Hindu reformer Exp) Option a is the correct answer.
and revivalist who founded the Arya Samaj in 1875 in
Bombay. Statement 1 is correct: The Ganjam Revolt was a tribal
uprising that took place in the Ganjam district of Odisha in
183. Solution: (a) 1800-1805. The revolt was caused by the oppressive policies
Exp) Option a is the correct answer. of the British East India Company. The British had taken
over the administration of the Ganjam district and had
Along with the help of Tatya Tope and other rebel soldiers, the
begun to impose heavy taxes on the people. The tribal people
Rani Laxmibai captured the fort of Gwalior. Afterwards,
of Ganjam were also being exploited by the zamindars. The
she proceeded to Morar near Gwalior to confront a British Ganjam Revolt was suppressed by the British in 1805.
counterattack led by Hugh Rose. Rani Laxmibai died
Statement 2 is correct: The Ganjam Revolt was led by
while fighting in Gwalior on 18th June 1858, aged 23. She
Dhananjay Bhanja, the last king of Ghumsur. Dhananjay
was dressed as a soldier when she died. Hugh Rose led the resisted British annexation, inspiring tribal support. Despite
British forces against Rani Laxmi Bai and on her death said several attempts, British troops couldn’t capture him, and the
that “here lay the woman who was the only man among the revolt continued even after his death in 1835.
rebels”. Hugh rose crushed the revolt in Jhansi and regions Statement 3 is correct: The Revolt of Gumsur was initiated
around it. by Shrikar Bhanja. He led the rebellion against British
authority in Ghumsur, assisted by tribal chiefs. The revolt
184. Solution: (a)
aimed to resist British attempts to annex the estate and
Exp) Option a is the correct answer.
depose loyal village headmen. Dhananjay Bhanj, successor of
Lotus and Chapatis (Bread) are considered as the symbol Shrikar Bhanja, later continued the resistance against British
of the revolt of 1857. Lotus is pure and is related to God. rule.
Bread is the basic need of everyone. There are opinion of Statement 4 is incorrect: Dhananjay Bhanj succeeded
some scholars that the slices of bread and lotus were passed his father, Strikara Bhanj, as the zamindar of Gumsur and
to different villages to carry forward some messages. continued the resistance against the British.

PYQ Workbook 78
HISTORY OF MODERN INDIA

Important Tips Important Tips


Uprisings in Ganjam and Gumsur (1800, 1835–37): • The Deccan Riots of 1875 were a peasant uprising
in Maharashtra, targeting oppressive debt peonage
• The region of Ganjam witnessed significant uprisings
imposed by moneylenders. Peasants attacked
during the early 19th century. Strikara Bhanj, the
moneylenders’ properties, burning debt bonds and
zamindar of Gumsur in Ganjam district, initiated
deeds. The riots arose due to deceitful practices,
a rebellion in 1800 by refusing to pay revenues and high-interest rates, and exploitation by moneylenders
garnering support from fellow zamindars. This who profited from post-American Civil War cotton
uprising led the British authorities to allocate certain demands. It reflected agrarian distress and interplay
districts under Strikara’s control. between credit, commerce, and colonial policies.
• Rebellions in Ghumsur and Parlakhemndi arose due
188. Solution: (d)
to factors like alienation and irregular revenue
payment. Exp) Option d is the correct answer.
The Vellore mutiny occurred during the regime of Sir
• In 1807-08, Dhananjay Bhanj succeeded his father,
George Barlow. The mutiny took place on July 10, 1806,
Strikara Bhanj, and continued the resistance against
and was a significant early instance of a large-scale revolt
British rule. Despite his efforts, Dhananjay eventually
by Indian sepoys against the British East India Company.
surrendered in 1815 due to mounting pressure.
The mutineers seized the Vellore Fort and engaged in violent
• In 1830, burdened by unpaid arrears, Dhananjay conflict with British troops. The rebellion was subdued by
Bhanj once again rose against the British. British cavalry and artillery from Arcot.
forces responded by occupying Gumsur and Kolaida
in November 1835. 189. Solution: (d)
• The uprisings severely challenged the authority of the Exp) Option d is the correct answer.
British government in the region. Although Dhananjay The composer of the song “Vande Mataram” is Bankim
Bhanj passed away in December 1835, his followers Chandra Chatterjee. The song was written in the 1870s and
persisted in their resistance efforts. first appeared in Chatterjee’s Bengali novel “Anandmath”
• The struggle persisted until February 1837 when a in 1882. It became an integral part of India’s independence
prominent leader named Doora Bisayi was arrested. movement and was adopted as the national song of India
This marked a turning point, and the zamindari of in 1950 by the Constituent Assembly. The song is an ode
to the motherland and played a significant role in inspiring
Gumsur was subsequently forfeited.
patriotism during the struggle for independence.
186. Solution: (d)
190. Solution: (d)
Exp) Option d is the correct answer. Exp) Option d is the correct answer.
Practical Vedanta was advocated by Vivekananda, a Hindu Ganga Narain Singh led the Bhumij Rebellion during
1832-1833 in the Dhalbhum and Jungle Mahal areas of the
monk and philosopher who propagated the teachings of
Midnapore district, Bengal Presidency. The rebellion was in
Vedanta and Yoga. Practical Vedanta refers to a modern response to British exploitation and policies, triggered by the
interpretation of Vedanta philosophy that emphasizes its dispute over succession to the Barabhum Raj. Ganga Narain
application in daily life and social service. It is based on formed the Sardar Gorilla Vahini Sena, gaining support from
various regions. His guerilla campaigns and strategic attacks
the idea that all beings are divine and that serving them is
led to significant unrest and casualties among British forces.
the highest form of worship. The rebellion compelled the British to withdraw oppressive
laws and policies.
187. Solution: (b)
191. Solution: (b)
Exp) Option b is the correct answer.
Exp) Option b is the correct answer.
• The Santhal rebellion took place before 1857 in around The first tribal leader inspired by Mahatma Gandhi’s
1855. ideology was Haipou Jadonang. Jadonang, a Naga spiritual
leader and political activist from Manipur, established the
• The popular movements that occurred after 1857 are:
Heraka religious movement and sought to unite his people
• Indigo revolt (1859-1860) against foreign imperialism. He admired Gandhi’s civil
disobedience movement and even attempted to meet him.
• Deccan Agriculturists Riots (1875)
Jadonang’s leadership was marked by his efforts to preserve
• Birsa Munda uprising (1899-1900) Naga culture and fight against British colonial rule.

79 PYQ Workbook
HISTORY OF MODERN INDIA

Important Tips and the zamindari system in present-day Jharkhand and


West Bengal, India. It was led by the four sibling brothers -
• Alluri Sitarama Raju (1897/98–1924) was an Indian
revolutionary who led the Rampa Rebellion (1922- Sidhu, Kanhu, Chand, and Bhairav Murmu. The rebellion
1924) against British colonial rule. He fought for aimed to resist the oppressive revenue system, usury
tribal rights, opposing the 1882 Madras Forest Act. practices, and exploitation by local zamindars. It began
Raju, known as “Manyam Veerudu,” orchestrated on June 30, 1855, and lasted until January 1856 when it was
guerilla campaigns in Andhra Pradesh and Odisha. He suppressed by the British forces.
raided police stations, seeking weapons for his forces.
Captured and executed by the British, he remains a 193. Solution: (c)
symbol of resistance. Exp) Option c is the correct answer.
• Gaidinliu Pamei (1915–1993), known as Rani
Swami Dayanand observed that “Good government was
Gaidinliu, was a Naga leader who led a revolt against
never a good substitute for self-government”. This quote
British rule. Joining the Heraka movement at 13, she
later fought to drive out the British from Manipur and reflect his vision of a free and prosperous India, where the
Naga areas. Arrested at 16, she was released in 1947 people would govern themselves according to the principles
after Nehru’s intervention. She championed Naga of Vedic dharma, without depending on any external
traditions and was honored with a Padma Bhushan. authority or power. He wanted to revive the ancient glory of
India, which he thought was lost due to centuries of foreign
192. Solution: (c)
invasions and domination. He also wanted to create a sense of
Exp) Option c is the correct answer. national unity and identity among the diverse communities
The Santhal rebellion, also known as the Santhal Hool, was and regions of India, by emphasizing their common heritage
a significant revolt against the British East India Company and culture.

PYQ Workbook 80
HISTORY OF MODERN INDIA

HISTORY OF MODERN INDIA


EMERGENCE OF INDIAN NATIONALISM [1857-1919]
*This unit consists of questions from Emergence of Indian Nationalism from the 1st war of independence
in 1857 to various events till 1919.

3.1. UPSC CSE Previous Years’ Questions 3. Surendranath Founder of the Indian
Banerjee Association
1. Consider the following freedom fighters:
Which of the above pairs is/are correctly
1. Barindra Kumar Ghosh matched?
2. Jogesh Chandra Chatterjee (a) 1 only
3. Rash Behari Bose (b) 1 and 3 only
Who of the above was/were actively associated (c) 2 and 3 only
with the Ghadar Party? (d) 1, 2 and 3
[UPSC CSE Pre 2022] 5. What was the main reason for the split in
(a) 1 and 2 the Indian National Congress at Surat in
(b) 2 only 1907? [UPSC CSE Pre. 2016]
(c) 1 and 3 (a) Introduction of communalism into
(d) 3 only Indian politics by Lord Minto.
2. Which one of the following is not a political (b) Extremists’ lack of faith in the capacity
of the moderates to negotiate with the
method of the moderates in the National
British Government.
Movement? [UPSC CSE Pre. 2021]
(c) Foundation of Muslim League.
(a) Overthrow of alien rule (d) Aurobindo Ghosh’s inability to be elected
(b) Constitutional agitation as the President of the Indian National
(c) Slow, orderly political progress Congress.
(d) Mobilisation of public opinion
6. The ‘Swadeshi’ and ‘Boycott’ were adopted
3. With reference to Swadeshi Movement as methods of struggle for the first time
consider the following statements: during the- [UPSC CSE Pre. 2016]
[UPSC CSE Pre. 2019] (a) agitation against the Partition of Bengal
1. It contributed to the revival of the (b) Home Rule Movement
indigenous artisan crafts and industries. (c) Non-Cooperation Movement
2. The National Council of Education (d) visit of the Simon Commission to India
was established as a part of Swadeshi 7. Who of the following was/were economic
Movement. critic/ critics of colonialism in India?
Which of the statements given above is/are [UPSC CSE Pre. 2015]
correct? 1. Dadabhai Naoroji
(a) 1 only 2. G. Subramania Iyer
(b) 2 only 3. R. C. Dutt
(c) Both 1 and 2 Select the correct answer using the code given
(d) Neither 1 nor 2 below.
4. Consider the following pairs: (a) 1 only
[UPSC CSE Pre. 2017] (b) 1 and 2 only
(c) 2 and 3 only
1. Radhakanta First President of (d) 1, 2 and 3
Deb the British Indian
8. Which one of the following movements has
Association
contributed to a split in the Indian National
2. Gazulu Founder of the Madras Congress resulting in the emergence of
Lakshminarasu Mahajana Sabha ‘moderates’ and ‘extremists’?
Chetty [UPSC CSE Pre. 2015]

81 PYQ Workbook
HISTORY OF MODERN INDIA

(a) Swadeshi Movement (c) removal of disqualifications imposed on


(b) Quit India Movement the Indian magistrates with regard to the
(c) Non-Cooperation Movement trial of the Europeans.
(d) Civil Disobedience Movement (d) removal of a duty on imported cotton
cloth.
9. Consider the following statements
13. Consider the following statements:
[UPSC CSE Pre. 2015]
The most effective contribution made by
1. The first woman President of the Indian Dadabhai Naoroji to the cause of Indian
National Congress was Sarojini Naidu. National Movement was that he
2. The first Muslim President of the Indian [UPSC CSE Pre. 2012]
National Congress was Badruddin Tyabji. 1. exposed the economic exploitation of
Which of the statements given above is / are India by the British.
correct? 2. interpreted the ancient Indian texts and
restore the self-confidence of Indians.
(a) 1 only
3. stressed the need for eradication of all the
(b) 2 only
social evils before anything else.
(c) Both 1 and 2
Which of the statements given above is/are
(d) Neither 1 nor 2
correct?
10. The Partition of Bengal made by Lord (a) 1 only
Curzon in 1905 lasted until (b) 2 and 3 only
[UPSC CSE Pre. 2014] (c) 1 and 3 only
(d) 1, 2 and 3
(a) the First World War when Indian troops
were needed by the British and the 14. With reference to the period of colonial
partition was ended. rule in India, “Home Charges” formed
(b) King George V abrogated Curzon’s Act at an important part of drain of wealth
from India. Which of the following funds
the Royal Durbar in Delhi in 1911.
constituted “Home Charges’’ ?
(c) Gandhiji launched his Civil Disobedience
[UPSC CSE Pre. 2011]
Movement.
1. Funds used to support the India office in
(d) the Partition of India in 1947 when East London.
Bengal became East Pakistan. 2. Funds used to pay salaries and pensions
11. The Ghadr (Ghadar) was a of British personnel engaged in India.
[UPSC CSE Pre. 2014] 3. Funds used for waging wars outside India
by the British.
(a) revolutionary association of Indians with
(a) 1 only
headquarters at San Francisco.
(b) 1 and 2 only
(b) nationalist organization operating from (c) 2 and 3 only
Singapore. (d) 1, 2 and 3
(c) militant organization with headquarters
15. What was the purpose with which Sir
at Berlin.
William Wedderburn and W.S. Caine had
(d) communist movement for India’s freedom set up the Indian Parliamentary Committee
with headquarters at Tashkent. in 1893? [UPSC CSE Pre. 2011]
12. The Ilbert Bill controversy was related to (a) To agitate for Indian political reforms in
the [UPSC CSE Pre. 2013] the House of Commons.
(b) To campaign for the entry of Indians into
(a) imposition of certain restrictions to carry
the Imperial Judiciary.
arms by the Indians. (c) To facilitate a discussion on India’s
(b) imposition of restrictions on newspapers Independence in the British Parliament.
and magazines published in Indian (d) To agitate for the entry of eminent Indians
languages. into the British Parliament.

PYQ Workbook 82
HISTORY OF MODERN INDIA

16. What was the immediate cause for the Which of the statement (s) given above is/are
launch of the Swadeshi Movement? correct? [UPSC CSE Pre. 2006]
[UPSC CSE Pre. 2010] (a) 1, 2 and 3
(a) The partition of Bengal done by Lord (b) 2 and 3 only
Curzon. (c) 1 and 2 only
(b) A sentence of 18 months rigorous (d) 3 only
imprisonment imposed on Lokmanya
Tilak. 21. What was Kamagatamaru?
(c) The arrest and deportation of Lala Lajpat [UPSC CSE Pre. 2005]
Rai and Ajit Singh; and passing of the (a) A political party based in Taiwan
Punjab Colonization Bill. (b) Peasant Communist leader of China
(d) Death sentence pronounced on the (c) A naval ship of Voyage to Canada
Chapekar brothers. (d) A Chinese village where Mao Tse lung
17. In the context of the Indian Freedom began his long march.
Struggle, 16th October 1905 is well-known 22. Where were the Ghadar revolutionaries,
for which one of the following reasons? who became active during the outbreak of
[UPSC CSE Pre. 2009] World War I based? [UPSC CSE Pre. 2005]
(a) The formal proclamation of Swadeshi (a) Central America
Movement was made in Calcutta town (b) North America
hall. (c) West America
(b) Partition of Bengal came into effect.
(d) South America
(c) Dadabhai Naoroji declared that the goal
of Indian National Congress was Swaraj. 23. The name of the famous person of India
(d) Lokmanya Tilak started Swadeshi who returned the Knighthood conferred on
Movement in Poona. him by the British Government as a token
18. Who was the Viceroy of India when the of protest against the atrocities in Punjab in
Rowlatt Act was passed? 1919 was: [UPSC CSE Pre. 2004]
[UPSC CSE Pre. 2008] (a) Tej Bahadur Sapru
(a) Lord Irwin (b) Ashutosh Mukherjee
(b) Lord Reading (c) Rabindra Nath Tagore
(c) Lord Chelmsford (d) Syed Ahmed Khan
(d) Lord Wavell 24. With reference to the period of extremist
19. The first conference of Indian National nationalist movement in India with its spirit
Congress held at– [UPSC CSE Pre. 2008] of Swadeshi, which one of the following
(a) Kolkata statements is NOT correct?
(b) Lahore [UPSC CSE Pre. 2002]
(c) Mumbai (a) Liyakat Hussain led the Muslim peasants
(d) Pune of Barisal in their agitations.
20. Consider the following statements about (b) In 1889, the scheme of national education
Madam Bhikaji Cama: was formulated by Satish Chandra
1. Madam Cama unfurled the National Flag Mukherjee.
at the International Socialist Conference (c) The Bengal National College was founded
in Paris in the year 1907. in 1906 with Aurobindo as the Principle.
2. Madam Cama served as private secretary (d) Tagore preached the cult of Atmasakti,
to Dadabhai Naoroji. the main plank of which was social and
3. Madam Cama was born to Parsi. economic regeneration of the villages.

83 PYQ Workbook
HISTORY OF MODERN INDIA

25. Which one of the following submitted in D. Kuka Movement 4. Aurobindo


1875 a petition to the House of Commons Ghosh
demanding India’s direct representation in Code: [UPSC CSE Pre. 2000]
the British Parliament? A B C D
[UPSC CSE Pre. 2002] (a) 1 3 4 2
(a) The Deccan Association (b) 1 3 2 4
(b) The Indian Association (c) 3 1 2 4
(c) The Madras Mahajan Sabha (d) 3 1 4 2
(d) The Poona Sarvajanik Sabha 30. While delivering the presidential address,
the Congress President who advocated the
26. The Hunter Committee was appointed after introduction of Roman script for the Hindi
the: [UPSC CSE Pre. 2001] language was: [UPSC CSE Pre. 2000]
(a) Blackhole incident (a) Mahatma Gandhi
(b) Jalianwalla Bagh massacre (b) Jawaharlal Nehru
(c) Uprising of 1857 (c) Abul Kalam Azad
(d) Partition of Bengal (d) Subhash Chandra Bose
27. Who among the following organized the 31. Which one of the following Indian leaders
famous Chittagong armoury raid? was dismissed by the British from the
[UPSC CSE Pre. 2001] Indian Civil Service?
(a) Laxmi Sehgal [UPSC CSE Pre. 1999]
(b) Surya Sen (a) Satyendranath Tagore
(c) Batukeshwar Datta (b) Surendranath Banerji
(d) J.M. Sengupta (c) R.C. Dutt
(d) Subhash Chandra Bose
28. Consider the following statements about
32. The first venture of Gandhi in all-India
the Indian National Congress: politics was the: [UPSC CSE Pre. 1999]
1. Sarojini Naidu was the first woman to be (a) Non-Cooperation Movement
the President of the Congress. (b) Rowlatt Satyagraha
2. C.R. Das was in prison when he functioned (c) Champaran Movement
as the President of the Congress. (d) Dandi March
3. The first Britisher to become the President 33. Which one of the following defines
of the Congress was Alan Octavian Hume. extremist ideology during the early phase
4. Alfred Webb was the President of the of Indian freedom movement?
Congress in 1894. [UPSC CSE Pre. 1998]
Which of these statements are correct? (a) Stimulating the production of indigenous
[UPSC CSE Pre. 2000] articles by giving them preference over
(a) 1 and 3 imported commodities.
(b) Obtaining Self-Government by
(b) 2 and 4
aggressive means in place of petitions and
(c) 2, 3 and 4 constitutional ways.
(d) 1, 2, 3 and 4 (c) Providing national educational according
29. Match list-I with list-II and select the to the requirements of the country.
correct answer using the codes given below (d) Organizing coups against the British
the lists: empire through military revolt.
34. The Indian Muslims, in general, were
List-I List-II NOT attracted to the extremist movement
A. Chhittagaon 1. Kalpana Dutt because of the: [UPSC CSE Pre. 1998]
Armoury Raid (a) influence of Sir Sayyid Ahmed Khan
B. Abhinav Bharat 2. Guru Ram Singh (b) anti-Muslim attitude of extremist leaders
(c) the indifference is shown to Muslim
C. Anushilan Samiti 3. Vinayak aspirations
Damodar (d) extremist’s policy of harping on Hindu
Savarkar part

PYQ Workbook 84
HISTORY OF MODERN INDIA

35. Who was the leader of the Ghadar Party? 40. What was common among Madam Bhikaji
[UPSC CSE Pre. 1998] Cama, M. Barkahtulla, V.V.S Iyer and M.N.
(a) Bhagat Singh Roy? [UPSC CSE Pre. 1994]
(b) Lala Hardayal (a) All of them were leading members of the
(c) Bal Gangadhar Tilak International Communist Movement.
(d) V.D. Savarkar (b) M. Barkatulla was the Prime Minister and
the rest were ministers in the Provisional
36. Match list-I with list-II and select the Government of free India established by a
correct answer using the codes given below group of revolutionaries at Kabul.
the lists: (c) All of them were among the prominent
revolutionaries outside India operating
List-I List-II in different countries during the freedom
A. Abhinav Bharat 1. Sri Aurobindo movement.
Society Ghosh (d) All of them were accused in the case
relating to the throwing of a bomb on
B. Anushilan Samiti 2. Lala Hardayal
Lord Hardinge.
C. Gadar Party 3. C.R. Das
3.2. Other Examination Previous Years’
D. Swaraj Party 4. V.D. Savarkar Questions
Code: [UPSC CSE Pre. 1996]
A B C D 41. Consider the following events and arrange
them in chronological order:
(a) 4 1 3 2
1. The foundation of Gadar Party.
(b) 1 4 3 2
(c) 1 4 2 3 2. Chittagong Armoury Raid.
(d) 4 1 2 3 3. Set up of the ‘Indian Independence
Committee’ at Berlin.
37. The Anarchical and Revolutionary Crime 4. Central Assembly Bomb Case.
Act (1919) was popularly known as the: Select the correct answer from the codes
[UPSC CSE Pre. 1996] given below. [UPPCS (Pre) 2022]
(a) Rowlatt Act (a) 1, 3, 4 and 2
(b) Pitt’s India Act (b) 3, 1, 2 and 4
(c) Indian Arms Act (c) 3, 1, 4 and 2
(d) Ilbert Bill (d) 1, 3, 2 and 4
38. The Barrah dacoity was the first major 42. Who among the following was NOT
venture of the revolutionary terrorists of associated with the Kanpur Conspiracy
the freedom movement in: Case of 1924?
[UPSC CSE Pre. 1995] [U.P.P.C.S. (Pre.) 2021]
(a) Bombay-Karnataka (a) Muzaffar Ahmed
(b) Punjab (b) Nalini Gupta
(c) East Bengal (c) Shaukat Usmani
(d) The Madras Presidency (d) M.A. Ansari
43. Who among the following Governor-
39. Which of the following international
Generals ridiculed Congress as representing
events influenced the course of the national only a ‘microscopic minority’ of people?
movement in India before the advent of
[U.P.P.C.S. (Pre) 2018]
Mahatma Gandhi?
(a) Lord Dufferin
1. Italian-Abyssinian War, 1898 (b) Lord Curzon
2. Boxer Movement in China (c) Lord Minto
3. Revolutionary Movement in Ireland (d) Lord Lansdown
4. Victory of Japan in the Russo-Japanese 44. Who founded the Asiatic Society of Bengal
War in Calcutta in 1784?
Code: [UPSC CSE Pre. 1994] [U.P.P.C.S. (Mains) 2017]
(a) only 1, 2 and 3 (a) Jonathan Duncan
(b) only 1, 2 and 4 (b) William Jones
(c) only 3 and 4 (c) Warren Hastings
(d) 1, 2, 3 and 4 (d) Charles Grant

85 PYQ Workbook
HISTORY OF MODERN INDIA

45. Among the following who established All 52. The first President of Indian National
India Harijan Sevak Sangh? Congress was- [U.P.P.C.S. (Pre) 2015]
[UPPCS (Mains) 2017] (a) Dadabhai Naoroji
(a) B.G. Gokhale (b) Surendra Nath Bannerji
(b) M.K. Gandhi (c) Womesh Chandra Bannerjee
(c) B.R. Ambedkar (d) A. O. Hume
(d) None of the above 53. In which one of the following sessions,
46. Which of the following pairs is not correctly Congress declared its policy towards the
matched? [U.P.P.C.S. (Mains) 2017] Indian States for the first time?
(a) Delhi Conspiracy Case - Amirchand [U.P.P.C.S. (Pre) 2015]
(b) Kakori Conspiracy Case - Ashfaqualla (a) Nagpur session
(c) Lahore Conspiracy Case - Jatin Das (b) Gaya session
(d) Nasik Conspiracy Case - Ras Bihari Bose (c) Calcutta session
(d) Lucknow session
47. Who amongst the following was one of
the founders of the Bombay Presidency 54. In which of the following sessions of Indian
Association in 1885? National Congress, Jawaharlal Nehru spoke
[U.P.P.C.S. (Mains) 2016] of socialism as the key to the solution of
(a) Feroz Shah Mehta India’s problems?
(b) P. Anand Charlu [U.P.P.C.S. (Mains) 2015]
(c) M. V. Raghav Cheriyar (a) Lahore
(d) S. N. Banerjee (b) Lucknow
(c) Allahabad
48. Which of the following sessions of the (d) Ramgarh
Indian National Congress was presided
over by C. Vijay Raghav Chariar? 55. Who among the following was Government
[U.P.P.C.S. (Mains) 2016] advocate in Kakori Conspiracy Case?
(a) Lucknow Session (1916) [U.P.P.C.S. (Mains) 2015]
(b) Nagpur Session (1920) (a) Mohan Lal Saxena
(c) Gaya Session (1922) (b) Jagat Narain Mulla
(d) None of the above (c) Krishna Bahadur
(d) Prabhat Chandra
49. Who among the following was the President
of Indian National Congress continuously 56. Who amongst the following was the first
for six years? [U.P.P.C.S. (Mains) 2016] Chairman of the Ghadar Party?
(a) Jawaharlal Nehru [U.P.P.C.S. (Mains) 2015]
(b) Abul Kalam Azad (a) Lala Hardayal
(c) G. K. Gokhale (b) Sohan Singh Bhakna
(d) Dadabhai Naoroji (c) Pandit Kashi Ram
(d) Keshar Singh
50. Who of the following continuously from
1904 onwards emphasized on the grant of 57. Match list-I with list-II and select the
‘self-rule’ to India? correct answer using the codes given below
[U.P.P.C.S. (Mains) 2016] the lists:
(a) S. N. Banerjee List-I List-II
(b) Aurobindo Ghosh (Organizations) (Founders)
(c) Firoz Shah Mehta
(d) Dadabhai Naoroji A. Land Holders’ 1. S.N. Banerji
Society
51. Who among the following was not an
B. British India 2. Anand Mohan Bose
extremist nationalist leader?
Society
[U.P.P.C.S. (Mains) 2016]
(a) Bipin Chandra Pal C. Indian Society 3. William Adams
(b) B.G. Tilak D. Indian 4. Dwarkanath Tagore
(c) Lala Lajpat Rai Association
(d) G.K. Gokhale Code: [UPPCS (Mains) 2014]

PYQ Workbook 86
HISTORY OF MODERN INDIA

A B C D Code: [UPPCS (Mains) 2014]


(a) 4 3 2 1 A B C D
(b) 4 2 3 1 (a) 1 2 3 4
(c) 3 2 4 1 (b) 2 3 1 4
(d) 4 1 2 3 (c) 4 2 3 1
58. Who of the following were elected to (d) 2 1 4 3
collaborate with Mahatma Gandhi who
62. Under whose chairmanship a committee
was to author the new Constitution of the
was formed to defend the accused in Kakori
Congress as per the resolution of Indian
National Congress at Amritsar Session, Case? [U.P.P.C.S. (Pre) 2014]
1919? (a) Acharya Narendra Dev
1. B.G. Tilak (b) Govind Ballabh Pant
2. N.C. Kelkar (c) Chandrabhanu Gupta
(d) Motilal Nehru
3. C.R. Das
4. I.B. Sen 63. Who said, “criticism and independent
Select the correct answer from the code given thought are the two characteristics of a
below: [U.P.P.C.S. (Mains) 2014] revolutionary”?
(a) 2 and 4 [U.P.P.C.S. (Mains) 2014]
(b) 1 and 2 (a) Bhagat Singh
(c) 3 and 4 (b) Ram Prasad Bismil
(d) 1 and 3 (c) Sachindranath Sanyal
59. The last session of Indian National Congress (d) Bhagwati Charan Vohra
attended by Bal Gangadhar Tilak was: 64. India House in London was established by:
[U.P.P.C.S. (Pre) 2014] [U.P.P.C.S. (Mains) 2014]
(a) Calcutta Session, 1906 (a) Shyamji Krishna Verma
(b) Surat Session, 1907 (b) Barkat Ullah
(c) Calcutta Session, 1917 (c) Virendranath Chattopadhyay
(d) Amritsar Session, 1919 (d) Lala Har Dayal
60. Who amongst the following revolutionaries 65. The Lieutenant Governor of Bengal at the
refused to drink the given milk on the eve of
time of Partition of Bengal was:
his execution and said “Now, I shall take my
mother’s milk only.”? [U.P.P.C.S. (Pre) 2014]
[U.P.P.C.S. (Pre) 2014] (a) Sir Andrew Fraser
(a) Rajguru (b) H.H. Risley
(b) Ashfaqualla (c) Brodrick
(c) Ramprasad Bismil (d) A.T. Arundel
(d) Bhagat Singh 66. The British journalist H.W. Nevinson was
61. Match list-I with list-II and select the associated with: [U.P.P.C.S. (Pre) 2014]
correct answer using the codes given below (a) Non-Co-operation Movement
the lists: (b) Civil Disobedience Movement
(c) Swadeshi Movement
List-I List-II
(d) Quit India Movement
(Organizations) (Founders)
67. The task of drafting Congress Inquiry
A. Abhinava Bharat 1. V.D. Savarkar
Committee report on Jallianwala Bagh
B. Mitra Mela 2. G.D. Savarkar Massacre was entrusted to:
C. Indian Republican 3. S.N. Sanyal [U.P.P.C.S. (Pre) 2014]
Army (a) Jawaharlal Nehru
D. Hindustan 4. S. Sen (b) Mahatma Gandhi
Republic (c) C.R. Das
Association (d) Fazlul Haq

87 PYQ Workbook
HISTORY OF MODERN INDIA

68. Match list-I with list-II and select the List-I (President) List-II (Session of
correct answer using the codes given below Congress)
the lists:
A. Dr. M.A. Ansari 1. Haripura
List-I List-II B. Purushottam Das 2. Kanpur
(President) (Places, where Tandon
Meetings of Indian C. Sarojini Naidu 3. Madras
National Congress
was held) D. Subhash Chandra 4. Nasik
Bose
A. Abul Kalam Azad 1. Amritsar, 1919
Code: [UPPCS (Pre) 2012]
B. Sarojini Naidu 2. Bombay, 1934
A B C D
C. Motilal Nehru 3. Kanpur, 1925 (a) 1 2 4 3
D. Dr. Rajendra 4. Ramgarh, 1940 (b) 2 3 1 4
Prasad (c) 3 4 2 1
Code: [UP RO/ARO (Mains) 2014] (d) 4 1 3 2
A B C D 74. Mahatma Gandhi presided over only in one
(a) 1 3 2 4 session of the Indian National Congress in
(b) 2 4 3 1 1924. Where was the session held?
(c) 3 2 4 1 [U.P.P.C.S. (Pre) 2011]
(d) 4 3 1 2 (a) Gaya
69. Bhagat Singh’s memorial is situated at– (b) Amritsar
[U.P.P.C.S. (Mains) 2013] (c) Belgaon
(a) Ferozpur (d) Kanpur
(b) Amritsar 75. Who among the following started the ‘Mitra
(c) Ludhiana Mela’ union? [U.P.P.C.S. (Pre) 2011]
(d) Gurdaspur (a) Shyamji Krishna Verma
70. When did Bal Gangadhar Tilak get the title (b) Vinayak Damodar Savarkar
Lokmanya? [UPPCS (Mains) 2012] (c) Lala Hardayal
(a) Swadeshi movement (d) Sohan Singh Bhakana
(b) Revolutionary movement 76. Who among of the following led the
(c) Home Rule Movement agitation against the partition of Bengal
(d) Quit India Movement (1905)? [U.P.P.C.S. (Pre) 2011]
71. Aurobindo Ghosh was brilliantly defended (a) Surendranath Bannerjee
in the Alipore Conspiracy Case by– (b) C.R. Das
[U.P.P.C.S. (Mains) 2012] (c) Ashutosh Mukherjee
(a) Chittaranjan Das (d) Rabindranath Tagore
(b) W.C. Banerjee 77. Who was the first to suggest the boycott of
(c) Motilal Nehru British goods in Bengal?
(d) Tej Bahadur Sapru [U.P.P.C.S. (Mains) 2011]
72. A provisional Government of India with (a) Aurobindo Ghosh
Raja Mahendra Pratap as its President was (b) Krishna Kumar Mitra
established during the first World War in? (c) Motilal Ghosh
[U.P.P.C.S. (Pre) 2012] (d) Satish Chandra Mukherjee
(a) Afghanistan 78. Who among the following led the Swadeshi
(b) Germany Movement at Delhi?
(c) Singapore [U.P.P.C.S. (Pre) 2011]
(d) Turkey (a) Bal Gangadhar Tilak
73. Match list-I with list-II and select the (b) Ajit Singh
correct answer using the codes given below (c) Lajpat Rai
the lists: (d) Syed Haidar Raja

PYQ Workbook 88
HISTORY OF MODERN INDIA

79. Who among the following leaders was a (a) East India Association
supporter of ‘Swadeshi’? (b) London India Society
[U.P.P.C.S. (Pre) 2009] (c) Indian Association
(a) Aurobindo Ghosh (d) Indian National Conference
(b) Feroz Shah Mehta 87. Who among the following transformed the
(c) Dadabhai Naoroji traditional Ganapati festival of Maharashtra
(d) Subhash Chandra Bose into a national festival and gave it a political
80. Yugantar Party was led by– character? [U.P.P.C.S. (Mains) 2007]
[U.P.P.C.S. (Mains) 2009] (a) Ramdas
(b) Shivaji
(a) Jatindranath Mukherjee
(c) Mahadev Govind Ranade
(b) Sachindranath Sanyal (d) Bal Gangadhar Tilak
(c) Rasbehari Bose
(d) Subhash Chandra Bose 88. The Indian Home Rule Society in London
was started by– [U.P.P.C.S. (Mains) 2007]
81. Who among the following was never elected
(a) Annie Besant
as a President of the ‘Indian National
(b) B.G. Tilak
Congress’? (c) M.K. Gandhi
[U.P.P.C.S. (Spl) (Mains) 2008] (d) Shyamji Krishna Verma
(a) Lala Lajpat Rai
(b) Annie Besant 89. Who among the following had founded the
(c) Moti Lal Nehru ‘Indian Society of Oriental Art’ to revive
(d) Bal Gangadhar Tilak ancient art traditions of India?
[U.P.P.C.S. (Mains) 2007]
82. The Haripura Session of Indian National (a) Abanindranath Tagore
Congress, 1938 was presided over by: (b) Nand Lal Bose
[U.P.P.C.S. (Spl) (Mains) 2008] (c) Asit Kumar Haldar
(a) Abul Kalam Azad (d) Amrita Shergill
(b) J.B. Kripalani
90. Who resigned from the membership of
(c) Rajendra Pradesh
Viceroy’s Executive Council as a protest
(d) Subhash Chandra Bose against Jallianwala Bagh Massacre?
83. Who founded the ‘Anushilan Samiti’? [U.P.P.C.S. (Mains) 2007]
[U.P.P.C.S. (Spl) (Pre) 2008] (a) Mahatma Gandhi
(a) P. Mitra (b) Rabindranath Tagore
(b) Varindra Ghosh (c) Shankaran Nair
(c) V.D. Savarkar (d) Jamnalal Bajaj
(d) Narendra Gosain 91. Match list-I with list-II and select the
84. Who had established “India Independence correct answer using the codes given below
League”? [U.P. P.C.S. (Spl) (Pre) 2008] the lists:
(a) Motilal Nehru
(b) Mahatma Gandhi List-I List-II
(c) Rashbihari Bose (Organizations) (Founders)
(d) Lala Lajpat Roy A. British Indian 1. Gopal Krishna
85. Who suggested launching a no tax Association Gokhale
campaign as a protest against the Rowlatt B. Bombay 2. K.T. Telang
Act? [U.P.P.C.S. (Mains) 2008] Presidency
(a) Abul Kalam Azad Association
(b) Gandhiji C. Central 3. Radha Kant Dev
(c) Rabindranath Tagore Mohammadan
(d) Swami Shraddhanand National
86. Name the organization formed by Association
Surendranath Banerjee which merged with D. Servants of India 4. Sayyid Amir Ali
the Indian National Congress in 1886: Society
[U.P.P.C.S. (Mains) 2007] Code: [UPPCS (Mains) 2006]

89 PYQ Workbook
HISTORY OF MODERN INDIA

A B C D (a) Rajguru and Sukhdev


(a) 1 2 4 3 (b) Khudiram Bose and Surya Sen
(b) 3 2 4 1 (c) Madan Lal Dhingra and Udham Singh
(c) 3 1 2 4 (d) Kartar Singh Sarabha and Ashfaqullah
(d) 4 3 2 1 Khan
92. Who among the following was not the 97. Seeds of discard were in which event during
member of the Hindustan Republican National Movement, and which eventually
Association (HRA)? divided the country, was–
[U.P.P.C.S (Mains) 2006] [U.P.P.C.S. (Spl) (Pre) 2004]
(a) Bhagat Singh (a) Establishment of Muslim League in 1906.
(b) Chandra Shekhar Azad (b) Division of Bengal in 1905.
(c) Ram Prasad Bismil (c) Khilafat Movement Supported by
Gandhiji.
(d) Shiv Verma
(d) Reservation of seats and separate
93. In which of the following Sessions of the electorates for Muslims in legislative
Indian National Congress Bal Gangadhar assemblies.
Tilak had expressed “Swaraj is my 98. Who is regarded as the “Mother of Indian
birthright; I shall have it”? Revolution”? [U.P.P.C.S. (Pre) 2003]
[U.P.P.C.S. (Spl) (Mains) 2004] (a) Annie Besant
(a) Banaras Session, 1905 (b) Sarojini Naidu
(b) Calcutta Session, 1906 (c) Rama Bai
(c) Surat Session, 1907 (d) Bhikaji Rustam Cama
(d) Lucknow Session, 1916
99. “Daro-deewar pe hasrat ki nazar karte hain,
94. Given below is a list of persons who became khush raho ahle-vatan ham to safar karte
Presidents of Indian National Congress. hain.’’ was said by: [U.P.P.C.S. (Pre) 2003]
Arrange them in chronological order. (a) Ashfaqullah Khan
1. Mahatma Gandhi (b) Bahadur Shah Zafar
2. Jawaharlal Nehru (c) Ramprasad Bismil
3. Vallabh Bhai Patel (d) Wazid Ali Shah
4. Smt. Sarojini Naidu 100. During the Indian freedom struggle, a large
Select your answer using the code given in unarmed crowd gathered in the Jallianwala
the list. Bagh at Amritsar on April 13, 1919 to
Code: [U.P.P.C.S. (Mains) 2004] protest against the arrest of:
(a) 1, 2, 3 and 4 [U.P.P.C.S. (Pre) 2002]
(b) 1, 3, 4 and 2 (a) Swami Shraddhanand and Mazharul Haq
(c) 1, 4, 2 and 3 (b) Madan Mohan Malviya and Mohammad
(d) 4, 3, 1 and 2 Ali Jinnah
(c) Mahatma Gandhi and Abul Kalam Azad
95. Given below is a list of organizations. Select (d) Dr. Saifuddin Kitchlu and Dr. Satyapal
those engaged in revolutionary activities,
using the codes given below the lists: 101. “The Congress is faltering to its fall and
one of my great ambitions while in India
1. Abhinav Bharat
is to assist it to a peaceful demise.” This
2. Anushilan Samiti declaration was made by:
3. New Nationalist Party [U.P.P.C.S. (Mains) 2002]
4. Indian Patriot Association (a) George Hamilton
Code: [U.P.P.C.S. (Pre) 2004] (b) Lord Curzon
(a) 1, 2 and 3 (c) Lord Dufferin
(b) 1 and 2 (d) Lord Minto
(c) 2, 3 and 4
102. Which of the following occurred last?
(d) 1, 2 and 4
[U.P.P.C.S. (Pre) 2000]
96. Which of the following pair got sentence to (a) Annexation Policy
be hanged for assassination of the British (b) Partition of Bengal
officers in England? (c) Permanent Settlement
[U.P.P.C.S. (Spl) (Pre) 2004] (d) Subsidiary Alliance

PYQ Workbook 90
HISTORY OF MODERN INDIA

103. Which one of the following events was (c) Valentine Chirol
characterized by Montague as ‘Preventive (d) Henry Cotton
Murder’? [U.P.P.C.S. (Pre) 1998]
111. Who among the following threw a bomb in
(a) Killing of INA activities
(b) Massacre of Jallianwala Bagh the Central Legislative Assembly on April 8,
(c) Shooting of the Mahatma 1929, to make the deaf British Government
(d) Shooting of Curzon-Wythe hear?
1. Bhagat Singh
104. Who among the following was not present at
the founding session of the Indian National 2. Sukhdev
Congress? [U.P.P.C.S. (Pre) 1997] 3. Rajguru
(a) Dadabhai Naoroji 4. Batukeshwar Datta
(b) G. Subramaniya Iyer Select the answer from the code given below:
(c) Justice Ranade [U.P. Lower Sub. (Pre) 2013]
(d) Surendranath Bannerjee
(a) 1 and 2
105. Hindustan Socialist Republican Association (b) 2 and 3
was formed by: [U.P.P.C.S. (Pre) 1996] (c) 1 and 3
(a) Subhash Chandra Bose (d) 1 and 4
(b) Ras Bihari Bose
(c) Chandra Shekhar Azad 112. Who among the following were the critics
(d) Sardar Bhagat Singh of the Swadeshi movement and pleaded for
a better understanding between the East
106. On what charge was Jatin Das arrested: and the West?
[U.P.P.C.S. (Pre) 1994]
[U.P. Lower Sub. (Pre) 2009]
(a) Meerut Conspiracy
(b) Peshawar Conspiracy (a) W.C. Banerjee
(c) Lahore Conspiracy (b) S.N. Bannerjee
(d) Chittagong Armed Dacoity (c) R.N. Tagore
(d) B. G. Tilak
107. Who from the following killed Michael O’
Dwyer responsible for Jallianwala Bagh 113. Which of the following pairs do NOT
massacre? [U.P.P.C.S. (Pre) 1994] match?
(a) Prithvi Singh Azad 1. British India Society - London
(b) Sardar Kishan Singh 2. East India Association - Calcutta
(c) Udham Singh 3. National India Association - London
(d) Sohan Singh Josh
4. Indian Association - Bombay
108. Who launched the first Home Rule League Find your answer with the help of given code:
Movement? [U.P.P.C.S. (Pre) 1993]
[U.P. Lower Sub. (Spl) (Pre) 2008]
(a) Annie Besant
(b) Sarojini Naidu (a) 1 and 2
(c) Surendranath Banerjee (b) 3 and 4
(d) Tilak (c) 1 and 3
(d) 2 and 4
109. Who was the Viceroy of India at the time of
Jallianwala Bagh Massacre in 1919? 114. Match list-I with list-II and select the
[U.P.P.C.S. (Pre) 1990] correct answer using the codes given below
(a) Lord Chelmsford the lists:
(b) Lord Minto
List-I List-II
(c) Lord Dalhousie
(d) Lord Canning A. Indian League 1. Shishir Kumar
Ghosh
110. Who called Bal Gangadhar Tilak the ‘Father
of Indian Unrest’? B. Indian Association 2. Anand Mohan
Bose
[U.P. Lower Sub. (Pre) 2013]
(a) Lord Curzon C. India National 3. Syyad Ahmad
(b) Vincent Smith Liberal Federation Khan

91 PYQ Workbook
HISTORY OF MODERN INDIA

D. United Indian 4. Surendranath 120. Who established Indian Association in 1876


Patriotic Bannerjee AD? [67th BPSC (Pre) 2022]
Association (a) Surendranath Banerjee
(b) Chittaranjan Das
Code: [UP Lower Sub. (Pre) 2008] (c) W.C. Banerjee
A B C D (d) Aurobindo Ghosh
(a) 1 3 4 2 (e) None of the above/More than one of the
(b) 2 1 4 3 above
(c) 3 2 4 1 121. In which place Khudiram Bose tried to kill
(d) 1 2 4 3 Kingsford?
115. Who was the leader of the Swadeshi [67th BPSC (Pre) (Re-Exam) 2022]
Movement in Madras? (a) Patna
(b) Darbhanga
[U.P. Lower Sub. (Pre) 2008] (c) Muzaffarpur
(a) Srinivas Shastri (d) Gaya
(b) Rajagopalachari (e) None of the above/More than one of the
(c) Chidambaram Pillai above
(d) Chintamani 122. Who has said about Bengal Partition, “The
116. Name the first recorded Muslim who was partition announcement fell like a bomb
hanged for Indian freedom- shell”?
[67th BPSC (Pre) (Re-Exam) 2022]
[U.P. Lower Sub. (Pre) 2004]
(a) Surendra Nath Bannerjee
(a) Mohammad Ali (b) Gopal Krishna Gokhale
(b) Shaukat Ali (c) Bipin Chandra Pal
(c) Ashfaq Ullah Khan (d) Chittaranjan Das
(d) Azizuddin (e) None of the above/More than one of the
above
117. Select the correct sequence of the following
events by using the codes given below: 123. Which of the following was NOT objective
1. Jallianwala Bagh Massacre of the Indian National Congress as pointed
out by W.C. Banerjee in 1885?
2. Dr. Satyapal’s incarceration [66th B.P.S.C. Re-Exam (Pre) 2020]
3. Amritsar Congress Session, 1919 (a) To promote contact and friendship
Select the answer from the code given below: among the protectors of the interests of
Code: [U.P. Lower Sub. (Pre) 2003] the country.
(a) 2, 1, 3 (b) To promote the feeling of nationalism
among fellow countrymen by removing
(b) 1, 2, 3 the feeling of casteism, communalism
(c) 2, 3, 1 and regional biases.
(d) 3, 2, 1 (c) To express the viewpoint on the necessary
118. Who accused Indian National Congress of social issues with due consent of the
educated class.
practising ‘politics of prayer, petition and
(d) To determine on which direction and on
protest’? [U.P. Lower Sub. (Spl) (Pre) which basis the work has to be done in
2002] the coming years for the Indian public
(a) Lala Hardayal interest.
(b) Bal Gangadhar Tilak (e) None of the above/More than one of the
(c) Subhash Chandra Bose above.
(d) Sardar Bhagat Singh 124. Who was the Second Muslim President in
119. Partition of Bengal was done mainly for: the Indian National Congress?
[U.P. Lower Sub. (Pre) 1998] [66th B.P.S.C. Re-Exam (Pre) 2020]
(a) Abul Kalam Azad
(a) To divide the Hindu and Muslims (b) Zakir Hussain
(b) For administrative convenience (c) Rahimtulla Mahomed Sayani
(c) To weaken the growth of Nationalism in (d) Badruddin Tyabji
Bengal (e) None of the above/More than one of the
(d) For the development of Bengal above

PYQ Workbook 92
HISTORY OF MODERN INDIA

125. Who is the author of the popular song, 131. Which movement started after the Partition
Sarfaroshi Ki Tamanna Ab Hamare Dil of Bengal?[59th to 59th B.P.S.C. (Pre) 2015]
Mein Hai? [66th B.P.S.C. (Pre) 2020] (a) Civil Disobedience
(a) Surya Sen (b) Swadeshi Movement
(b) Chandra Shekhar Azad (c) Quit India Movement
(c) Sardar Bhagat Singh (d) Non-Cooperation Movement
(d) Ram Prasad Bismil 132. The protest method of moderate leaders of
(e) None of the above/More than one of the the Congress was:
above [48th to 52nd B.P.S.C. (Pre) 2008]
126. Who founded Abhinav bharat society in (a) Non-Co-operation
1906 in London? (b) Constitutional agitation
[66th B.P.S.C. Re-Exam (Pre) 2020] (c) Passive resistance
(a) Vinayak Damodar Savarkar (d) Civil disobedience
(b) Shyamji Krishna Varma 133. Who said that “Congress Movement was
(c) Sohan Singh neither inspired by the people, nor devised
(d) P.N. Bapat or planned by them”?
(e) None of the above/More than one of the [47th B.P.S.C. (Pre) 2005]
above (a) Lord Dufferin
127. Which revolutionaries were hanged in the (b) Sir Syed Ahmed
‘Kakori Train Robbery Case’? (c) Lord Curzon
[65th B.P.S.C. (Pre) 2019] (d) Lala Lajpat Rai
(a) Ram Prasad Bismil and Ashfaqualla 134. Who propounded the doctrine of ‘Passive
(b) Veer Savarkar and Vasudev Chapekar Resistance’? [47th B.P.S.C. (Pre) 2005]
(c) Prafulla Chandra Chaki and Khudiram (a) Mahatma Gandhi
Bose (b) Bipin Chandra Pal
(d) Surya Sen and Udham Singh (c) Bal Gangadhar Tilak
(e) None of the above/More than one of the (d) Aurobindo Ghosh
above 135. The second session of the Indian National
128. The Jallianwala Bagh Massacre happened in Congress was presided over by-
the context of which Gandhian Satyagraha? [45th B.P.S.C. (Pre) 2001]
[B.P.S.C. (Pre) 2018] (a) Ganesh Agarkar
(a) Swadeshi Satyagraha (b) Surendranath Bannerjee
(b) Rowlatt Satyagraha (c) Dadabhai Naoroji
(c) Bardoli Satyagraha (d) Feroz Shah Mehta
(d) Individual Satyagraha 136. Which one of the following leaders belonged
(e) None of the above/More than one of the to the Extremist wing of the Congress?
above [45th B.P.S.C. (Pre) 2001]
129. Who established a branch of Anushilan (a) Aurobindo Ghosh
Samiti at Patna in 1913? (b) Dadabhai Naoroji
[63rd B.P.S.C (Pre.) 2017] (c) G.K. Gokhale
(a) Rewati Nag (d) S.N. Bannerjee
(b) Yadunath Sarkar 137. Who gave the ‘Inquilab Zindabad’ slogan?
(c) Sachindra Nath Sanyal [45th B.P.S.C. (Pre) 2001]
(d) Mazharul Haque (a) Iqbal
(e) None of the above/More than one of the (b) M.K. Gandhi
above (c) Bhagat Singh
130. Who was the President of Gaya Session of (d) S.C. Bose
the Indian National Congress held in 1922? 138. The founder of the Indian National
[56th to 59th B.P.S.C. (Pre) 2015] Congress was a: [43rd B.P.S.C. (Pre) 1999]
(a) Chittaranjan Das (a) Civil Servant
(b) S.N. Banerjee (b) Scientist
(c) Dr. Rajendra Prasad (c) Social Worker
(d) Hakim Ajmal Khan (d) Commander

93 PYQ Workbook
HISTORY OF MODERN INDIA

139. Most of the moderate leader hailed from: 145. What was the cause of the outbreak of
[43rd B.P.S.C. (Pre) 1999] Ghadar Revolution?
(a) Rural areas [39th B.P.S.C. (Pre) 1994]
(b) Urban areas (a) Arresting of Lala Hardyal
(c) Both rural and urban area (b) Komagata Maru incident
(d) Punjab (c) Beginning of first World War
(d) Hanging of Kartar Singh Sarabha
140. Name the Viceroy of India concerned with
Bengal partition–[42nd B.P.S.C. (Pre) 1997] 146. Who called the Congress Conference as
(a) Lord Hardinge the ‘Annual National Festival of Educated
(b) Lord Curzon Indians’? [MPPCS (Pre) 2021]
(c) Lord Lytton (a) Lala Lajpat Rai
(d) Lord Minto (b) Bal Gangadhar Tilak
(c) Gopal Krishna Gokhale
141. Home Rule Movement was symbol of (d) Vipin Chandra Pal
starting a new phase of freedom movement
in India because:[41st B.P.S.C. (Pre) 1996] 147. An important event of Lord Dufferin’s
tenure as Viceroy was-
(a) It put a perfect project for the self-
Government before the nation [M.P.P.C.S. (Pre) 2017]
(b) Leadership of the movement came in the (a) Establishment of Ramakrishna Mission
hands of Gandhiji (b) Establishment of Muslim league in Dhaka
(c) Hindus and Muslims started to struggle (c) Establishment of Indian National
jointly Congress
(d) It established co-ordination between (d) Beginning of the First Census
extremists and moderates 148. Who among the following was the youngest
142. Indian National Congress opposed the person to become the President of the
Rowlatt Act because it aimed- Indian National Congress?
[M.P.P.C.S. (Pre) 2017]
[41st B.P.S.C. (Pre) 1996]
(a) Jawaharlal Nehru
(a) To limit the individual liberty
(b) Abul Kalam Azad
(b) To ban on Indian National Congress
(c) Ananda Mohan Bose
(c) To enlarge the communal delegations
(d) Bhupendra Nath Bose
(d) To imprison national leaders charging the
traitors to the nation 149. Consider the following statements and
select the correct answer from the codes
143. Which of the following classes was mainly given below:
unaffected by the Swadeshi Movement of
1. Subhash Chandra Bose formed the
1905?
Forward Block
(i) Women 2. Bhagat Singh was one of the founders
(ii) Farmers of Hindustan Socialist Republican
(iii) Muslim Association,
(iv) Intellectuals Code: [M.P.P.C.S. (Pre) 2008]
(a) Only 1 is correct
Select your answer to the following codes–
(b) Only 2 is correct
[40th B.P.S.C. (Pre) 1995] (c) Both 1 and 2 are correct
(a) i and ii (d) None of these
(b) i, ii and iii
(c) ii and iii 150. Which implication was taken in Lucknow
(d) i and ii session of Congress in 1916?
[M.P.P.C.S. (Pre) 1994]
144. Who among the following did not contribute (a) Separate electorate demand of Muslim
to Home Rule League? League was accepted.
[40th B.P.S.C. (Pre) 1995] (b) Temporary merger of Muslim League and
(a) Bal Gangadhar Tilak Congress occurred
(b) Annie Besant (c) A Muslim person was elected as President
(c) S. Subrahmanyam Ayer of Congress
(d) T.S. Alcott (d) None of the above

PYQ Workbook 94
HISTORY OF MODERN INDIA

151. Which of the following revolutionaries was [R.A.S./R.T.S. (Re. Exam) (Pre) 2013]
not hanged for Lahore Conspiracy? (a) (i), (ii), (iii) and (iv)
[M.P.P.C.S. (Pre) 1990] (b) Only (iii) and (iv)
(a) Batukeshwar Dutt (c) Only (i), (iii) and (iv)
(b) Sukhdev (d) Only (i) and (iv)
(c) Sardar Bhagat Singh 155. Who among the following became President
(d) Rajguru of Indian National Congress?
152. Match list-I with list-II and choose the [R.A.S./R.T.S. (Pre) 2012]
correct answer from the code given below– (a) Sucheta Kriplani
(b) Aruna Asaf Ali
List-I (c) Annie Besant
A. Howrah Conspiracy case (d) Vijay Lakshmi Pandit
B. Lahore Conspiracy case 156. In 1889, a committee was set up in Britain
C. Delhi Conspiracy case to gain support for the Congress, who
D. Alipore Conspiracy case amongst the following was its President–
[R.A.S./R.T.S. (Pre) 1996]
List-II
(a) Sir W. Wadderburn
(i) Master Ami Chand (Amir Chand) (b) Mr. Digby
(ii) Arvindo Ghosh (c) Dadabhai Naoroji
(iii) Jatindranath Mukherjee (d) W.C. Banerjee
(iv) Rajguru 157. The following programmes were launched
Code: [R.A.S./R.T.S. (Pre.) 2021] by the nationalist against the partition of
Bengal:
(a) A-(iv), B-(iii), C-(ii), D-(i)
(b) A-(i), B-(ii), C-(iii), D-(iv) 1. Boycott
(c) A-(iii), B-(iv), C-(i), D-(ii) 2. Swadeshi
(d) A-(ii), B-(iii), C-(iv), D-(i) 3. Non-cooperation
4. National education
153. Which of the following statements is/are
Select the correct answer using the codes
true? given below:
I. After the Nagpur Session (1920) of Congress, [Chhattisgarh P.C.S. (Pre) 2014]
the Provincial Congress Committees were (a) 1, 2 and 3
constituted on linguistic basis. (b) 2, 3 and 4
II. In 1948, Congress rejected the demand of (c) 1, 3 and 4
formation of provinces on linguistic basis. (d) 1, 2 and 4
Select the correct answer from the code (e) None of these
below: [R.A.S./R.T.S. (Pre) 2018] 158. Match list-I with list-II and select the
(a) Only I correct answer using the codes given below
(b) Only II the lists:
(c) Neither I nor II List-I List-II
(d) Both I & II
A. Jatin Das 1. During life
154. Which amongst the following statements is/ imprisonment
are true about V.D. Savarkar?
B. Chandrashekhar 2. Hunger strike
(i) He founded Abhinav Bharat - A Azad
revolutionary organization.
C. Bhagat Singh 3. During encounter
(ii) In order to inspire Indian nationalist, he
wrote the biography of Mazzini. D. Kalpana Datt 4. Hanged
Code: [Chhattisgarh PCS (Pre) 2013]
(iii) He also authored “The Indian War of
Independence - 1857” which provides a A B C D
nationalist view of the revolt of 1857. (a) 3 2 4 1
(b) 2 3 4 1
(iv) He jumped into the sea from the sailing (c) 2 3 1 4
ship in order to escape British captivity. (d) 3 4 1 2
Choose the correct answer: (e) 1 2 4 3

95 PYQ Workbook
HISTORY OF MODERN INDIA

159. Who among the following was not associated 165. Who among the following suggested the
with the Kakori incident? winding up of the Indian National Congress
[Uttarakhand P.C.S. (Pre) 2016] after India attained independence?
(a) Ram Prasad Bismil [Uttarakhand P.C.S. (Pre) 2005]
(b) Surya Sen (a) C. Rajagopalachari
(c) Rajendra Lahiri (b) Acharya Kripalni
(d) Ashfaq Ullah (c) Mahatma Gandhi
(d) Jayaprakash Narain
160. Muzaffar Ahmad, S.A. Dange, Shaukat
Usmani and Nalini Gupta were jailed for 166. Consider the following Statements:
which conspiracy? 1. The First Session of the Indian National
Congress was held in Calcutta.
[Uttarakhand P.C.S. (Pre) 2012]
2. The Second Session of the Indian
(a) Kakori train heist
National Congress was held under the
(b) Chattagaon Armoury raid presidentship of Dadabhai Naoroji.
(c) Meerut conspiracy case
3. Both Indian National Congress and
(d) Kanpur Bolshevik conspiracy case Muslim League held their sessions at
161. The first Political Organization established Lucknow in 1916 and concluded the
in India in 1838 was known as- Lucknow Pact.
[Uttarakhand P.C.S. (Pre) 2010] Which of the statements given above is/are
(a) British India Society correct? [Jharkhand P.C.S. (Pre) 2011]
(b) Bengal British India Society (a) 1 and 2
(c) Settlers Association (b) 2 only
(d) Zamindari Association (c) 2 and 3
President of Indian National Congress (d) 3 only
at the time of Independence was? 167. Arrange the following organization on the
[Uttarakhand P.C.S. (Pre) 2010] basis of their correct order:
(e) Mahatma Gandhi 1. Bombay Association
(f ) Jawaharlal Nehru 2. Madras Mahajan Sabha
(g) J.B. Kripalani 3. Indian Association
(h) Dr. Rajendra Prasad 4. Indian League
162. Bhagat Singh, Sukhdev, and Rajguru were Code: [Jharkhand P.C.S. (Pre) 2003]
sentenced to death in– (a) 1, 2, 3, 4
[Uttarakhand P.C.S. (Pre) 2010] (b) 2, 3, 1, 4
(a) Alipur conspiracy case (c) 3, 4, 2, 1
(d) 1, 4, 3, 2
(b) Lahore conspiracy case
(c) Kakori conspiracy case 168. Activities of Barindra Ghosh had given
(d) Kanpur conspiracy case birth of secret revolutionary organization
named as- [Jharkhand P.C.S. (Pre) 2003]
163. When was Bengal reunited due to the
(a) Anushilan Samiti
protests by Indians?
(b) Swadeshi Bandhawa Samiti
[Uttarakhand P.C.S. (Pre) 2010] (c) Brati Samiti
(a) 1905 (d) Sadhawa Samaj
(b) 1911
169. Which of the following are true in the
(c) 1947
context of Swadeshi movement in 1905?
(d) 1971
1. Massive demonstrations were held in
164. Vande Mataram became the theme song of Bengal.
the Indian National Movement during– 2. Hartal was observed in Calcutta.
[Uttarakhand P.C.S. (Mains) 2006] 3. The movement was initiated by the
(a) Swadeshi Movement moderates but was taken over by the
(b) Champaran Movement revolutionary nationalists at a later stage.
(c) Anti-movement of Rowlatt Act Select the correct answer using the code given
(d) Non-cooperation movement below: [CAPF 2019]

PYQ Workbook 96
HISTORY OF MODERN INDIA

(a) 1, 2 and 3 3. It advocated a constructive programme of


(b) 1 and 2 self help or ‘atmashakti”.
(c) 2 and 3
(d) 1 and 3 4. In the sphere of education, national
schools were established.
170. Which among the following statements
about the Jallianwala Bagh massacre is/ are Select the correct answer using the code
correct? givenbelow. [CAPF 2016]
1. In Jallianwala Bagh troops opened fire (a) 1, 2 and 3 only
upon an unarmed crowd.
(b) 1 and 4 only
2. The troops were under the command of
General Dyer. (c) 2, 3 and 4 only
3. The troops did not issue any warning to (d) 1, 2, 3 and 4
the people before opening fire.
173. Which of the following statements about the
Select the correct answer using the code given
below: [CAPF 2019] founding of the Indian National Congress
(a) Only 1 (INC) is/are correct?
(b) 1 and 2 1. The INC was formed in Bombay in 1885.
(c) 1, 2 and 3
2. The first President of the INC was WC
(d) 2 and 3
Bannerjee.
171. Examine these two statements carefully and
select the correct answer using the code Select the correct answer using the code given
given below. below. [CAPF 2016]
Statement I: Dadabhai Naoroji argued that (a) Only 1
what was being drained out was ‘potential (b) Only 2
surplus’ that could generate more economic
(c) Both 1 and 2
development in India if invested in India.
Statement II: Imperialists believed that (d) Neither 1 nor 2
India was brought into the large capitalist 174. Which of the following statements about
world market and that was in itself a progress
the Champaran Satyagraha is/are correct?
towards modernisation.
Code: [CAPF 2017] 1. The Champaran region had a long
(a) Both the statements are individually true tradition of anti-planter discontent and
and statement II is the correct explanation agitation.
of statement I.
2. Mahatma Gandhi gave all India publicity
(b) Both the statements are individually
true, but statement II is not the correct to the grievances of Champaran
explanation of statement I. cultivators.
(c) Statement I is true, but statement II is 3. The cultivators of Champaran had
false.
(d) Statement I is false, but statements II is protested against excessive taxation on
true. sugar.
172. Which of the following statements about Select the correct answer using the code given
the Swadeshi Movement in Bengal in 1905 below. [CAPF 2016]
are correct?
(a) 1 only
1. It was marked by the mass mobilisation
through the organisation of ‘Samitis’. (b) 1 and 2 only
2. It was marked by the mass mobilisation (c) 2 and 3 only
through labour strikes. (d) 1, 2 and 3

97 PYQ Workbook
HISTORY OF MODERN INDIA

175. Which one of the following was NOT the Punjab countryside was stirred by the
feature of the Home Rule Movement in powerful Akali upsurge.
(a) Both the statements are individually true,
India? [CAPF 2015] and Statement II is the correct explanation
(a) Boycott and Picketing of Statement I.
(b) Two separate Home-Rule Leagues (b) Both the statements are individually
true, but Statement II is not the correct
rather than one All-India Home-Rule explanation of Statement I.
Organisation (c) Statement I is true, but Statement II is
false.
(c) Setting up of discussion groups and (d) Statement I is false, but Statement II is
reading rooms true.
(d) Sale and circulation of pamphlets 180. Which of the following statements about the
176. Which one among the following explains Non-Cooperation Movement is/are correct?
the meaning of the Safety Valve Theory in 1. It was marked by significant participation
the context of Indian National Movement? of peasants from Karnataka.
[CAPF 2014] 2. It was marked by non-brahmin lower
(a) A theory that argued that the Congress caste participation in Madras and
was formed as a safe outlet to the Maharashtra.
discontented educated Indians. 3. It was marked by the lack of labour unrest
(b) A theory that argued that the Congress in places like Assam, Bengal and Madras.
was formed as a sale outlet to the hostile 4. It was badly shaken by the Chauri Chaura
attitude of the British towards the Indian incident in 1922 after which Gandhiji
people. decided to continue with the movement
(c) A theory which beloved that the British on a much smaller scale.
and the ex-rebels of 1857 should amicably Select the correct answer using the codes
resolve their differences against each given below. [CDS 2018 (I)]
other. (a) 1 only
(d) As idea aimed at promoting goodwill (b) 1, 2 and 4
among the Indian people. (c) 2 and 3 only
177. The headquarters of Ghadar Movement/ (d) 2 only
Party were at [CDS 2022 (I)] 181. Which of the following statement(s) about
(a) San Francisco the formation of the Indian National
(b) Stanford University Congress is/are true?
(c) Portland 1. The Indian National Congress was formed
(d) Florida at a national convention held in Calcutta
178. Which one of the following was NOT a in December 1885 under the presidency
demand made by the Congress moderates? of Motilal Nehru.
[CDS 2020 (II)] 2. The Safety Valve Theory regarding the
(a) Universal Adult Franchise formation of the Indian National Congress
(b) Repeal of the Arms Act emerged from a biography of AO Hume
(c) Extension of Permanent Settlement written by William Wedderburn,
(d) Higher Jobs for Indians in the army 3. An early decision was that the President
would be from the same region where the
179. The following item consist of two session was to be held.
statements, Statement I and Statement II.
4. WC Bannerjee was the first President of
Examine these two statements carefully and
the Indian National Congress.
select the correct answer using the code
given below: [CDS 2019 (II)] Select the correct answer using the codes
given below: [CDS 2015 (II)]
Statement I: Non-cooperation began in
Punjab with the student movement inspired (a) 1, 2 and 4
by Lala Lajpat Rai in January 1921. (b) 2 and 3
(c) 2 and 4
Statement II: The Sikh dominated central (d) 1 and 3

PYQ Workbook 98
HISTORY OF MODERN INDIA

182. Consider the following statements about 183. Consider the following statements
the First Session of the Indian National 1. BG Tilak founded the Home Rule League
Congress
in April 1916, in Maharashtra.
1. It was held in Bombay in 1885.
2. Surendranath Bannerji could not attend 2. NC Kelkar was not associated with Home
the session due to the simultaneous Rule Movement.
session of the Indian National Conference. Which of the statement(s) given above is/are
Which of the statement(s) given above is/are correct? [CDS 2014 (I)]
correct? [CDS 2014 (I)]
(a) Only 1
(a) Only 1
(b) Only 2 (b) Only 2
(c) Both 1 and 2 (c) Both 1 and 2
(d) Neither 1 nor 2 (d) Neither 1 nor 2

99 PYQ Workbook
HISTORY OF MODERN INDIA

SOLUTIONS
contributed to the revival of the indigenous artisan crafts
3.1. UPSC CSE Previous Years’ Questions and industries. The Swadeshi Movement celebrated our
1. Solution: (d) indigenous fabrics, weavers and the country’s artisans.
Exp) Option d is the correct answer. August 7 has been chosen as National Handloom Day as
it was the day that the Swadeshi Movement was started in
Indian revolutionaries in the United States of America and
1905.
Canada had established the Ghadar (Rebellion) Party in
1913. While most of the members of the party were Sikh Self-reliance or ‘atma shakti’ was encouraged. The swadeshi
peasants and soldiers, their leaders were mostly educated spirit also found expression in the establishment of swadeshi
Hindus or Muslims. textile mills, soap and match factories, tanneries, banks,
insurance companies, shops, etc.
Some of the prominent Ghadar leaders were: Baba Gurmukh
Singh, Kartar Smgh Saraba, Sohan Singh Bhakna, Rahmat Ali Statement 2 is correct. The National Council of Education
Shah, Bhai Parmanand, and Mohammad Barkatullah. Other - Bengal (or NCE - Bengal) was an organisation founded
revolutionaries were active in India and abroad. by Indian nationalists in Bengal in 1906 as part of the
Swadeshi Movement.
In 1915, during an unsuccessful revolutionary attempt,
Jatin Mukerjea popularly known as Bagha Jatin gave his life It was set up to organise a system of education— literary,
fighting a battle with the police at Balasore. Rash Bihari scientific and technical—on national lines and under
Bose, Raja Mahendra Pratap, Lala Hardayal, Abdul national control.
Rahim, Maulana Obaidullah Sindhi, Champak Raman
Important Tips
Pillai, Sardar Singh Rana, and Madam Cama were some
of the prominent Indians Ghadar Leaders who carried on Swadeshi Movement:
revolutionary activities and propaganda outside India • The Swadeshi Movement had its genesis in the anti-
partition movement which was started to oppose the
2. Solution: (a) British decision to partition Bengal. On August 7,
Exp) Option a is the correct answer. 1905, with the passage of the Boycott Resolution in
The national leaders like Dadabhai Naoroji, Pherozshah a massive meeting held in the Calcutta Townhall, the
Mehta, D.E. Wacha, W.C. Bonnerjea, and S.N. Banerjea who formal proclamation of Swadeshi Movement was made.
dominated the Congress policies during this period were • During Swadeshi Movement, a Bengal Institute of
staunch believers in ‘liberalism’ and ‘moderate’ politics and Technology was set up for technical education and
came to be labelled as Moderates to distinguish them from funds were raised to send students to Japan for
the neo-nationalists of the early twentieth century who were advanced learning. Also, the Bengal National College,
referred to as the Extremists. inspired by Tagore’s Shantiniketan, was set up with
The moderate political activities involved constitutional Aurobindo Ghosh as its principal.
agitation within the confines of law and showed a slow but
4. Solution (b)
orderly political progress. The Moderates believed that the
British basically wanted to be just to the Indians but were not Exp) Option b is the correct answer.
aware of the real conditions. Option 1 is correct. The British Indian Association was
Therefore, if public opinion could be created in the country established on 31 October 1851 with entirely Indian
and public demands be presented to the Government through membership to promote government efficiency and
resolutions, petitions, meetings, etc., the authorities would ameliorate the condition of Indias. The first committee of
concede these demands gradually. the association was composed of: Raja Radhakanta Deb –
To achieve these ends, they worked on a two-pronged President, Raja Kalikrishna Deb – Vice-President.
methodology one, create a strong public opinion to arouse Option 2 is incorrect. In 1884, M. Veeraraghavachariar,
consciousness and national spirit and then educate and unite G.S Iyer and P. Anandacharlu established the Madras
people on common political questions; and two, persuade the Mahajana Sabha. Gazulu Lakshminarasu Chetty founded
British Government and British public opinion to introduce the Madras Native Association.
reforms in India on the lines laid out by the nationalists. Option 3 is correct. Indian Association was the first
Overthrow of Alien Rule was not a political method of the Nationalist Organization founded in British India by
moderates in the National Movement. Surendranath Banerjee and Anand Mohan Bose in 1876.
The association found an issue in opposing the lowering of
3. Solution (c)
the age limit for civil service examinations (1877) , which
Exp) Option c is the correct answer. was disadvantageous to Indian candidates; and in 1878
Statement 1 is correct. The Swadeshi Movement it objected to the Vernacular Press Act, which stifled the
included boycott and public burning of foreign goods and Indian press. It displaced the Indian League, which had been

PYQ Workbook 100


HISTORY OF MODERN INDIA

founded the year before, and criticised the British Indian 7. Solution (d)
Association, which it regarded as a reactionary body of Exp) Option d is the correct answer.
landlords and industrialists.
Dadabhai Naoroji in his book titled Poverty and Un-British
5. Solution (b) Rule in India talked about the drain theory. R. C. Dutt’s
Exp) Option b is the correct answer. The economic history of India, Ranade’s Essay on Indian
Economics, G. Subramania Iyer’s Economic Aspect of British
There was a fundamental difference between the political
Rule were also critics of economic policies of British.
ideologies of the extremists’ and moderates which led to
their split at Surat Session of Congress in 1907. 8. Solution (a)
The veteran moderate politicians refused to accommodate Exp) Option a is the correct answer.
the new extremist’s trend within the Congress policies and Surat split happened in the year 1907 just after the Swadeshi
programmes, and this led to the split in the Congress in its
Movement. The Swadeshi movement has its genesis in the
Surat session in 1907.
anti-partition movement which was started to oppose the
The Extremists, during the Swadeshi Movement, decided to British decision to divide Bengal. With the start of the
give a call for wide passive resistance and boycott of schools, Swadeshi movement at the turn of the century, the Indian
colleges, legislative councils, municipalities, law courts, etc. National Movement took a major leap forward. Within the
The Moderates, encouraged by the news that council reforms congress, there was rift between several members regarding
were on the anvil, decided to tone down the scheme adopted the movement.
at Calcutta Session.
9. Solution (b)
The Extremists thought that the people had been aroused
and the time had come for direct confrontation with the Exp) Option b is the correct answer.
British. They considered the Moderates to be a drag on Statement 1 is incorrect. Annie Besant who was instrumental
the movement and no longer have faith in the capacity of in creation of Home Rule League presided over the 1917
the moderates to negotiate with the British Government. session in Calcutta and became the first woman president
They decided that it was necessary to Part Company with of INC.
the Moderates, even if it meant a split in the Congress. On Sarojini Naidu joined the Indian National Movement in the
the other hand, the Moderates thought that it would be wake of the 1905 partition of Bengal. She was elected as the
dangerous that stage to associate with the Extremists whose President of the Indian National Congress Party in 1925,
anti-imperialist agitation would be ruthlessly suppressed by Kanpur Session. Thus, Sarojini Naidu was the first Indian
the mighty colonial forces. women President of INC.
Both sides adopted rigid positions, which ultimately led to Statement 2 is correct. Badruddin Tyabji came back to India
split. The moderates after the Surat Split in 1907 demanded from England in December 1867 and became the first Indian
colonial self-government, as against the extremist demand of barrister in the High Court of Bombay.
complete independence.
Badruddin Tyabji became the first Muslim president
6. Solution (a) of the Indian National Congress. Overall, he was the
Exp) Option a is the correct answer. third President of the Congress after WC Bannerjee and
Dadabhai Naoroji (1886).
To meet the growing nationalist challenge in eastern India,
on 19 July, 1905 the Government of India announced its 10. Solution (b)
decision to form the new province of “Eastern Bengal and
Exp) Option b is the correct answer.
Assam”, comprising the Chittagong, Dacca and Rajshahi
divisions, Hill Tippera (Tripura) , Malda and Assam. The The Partition of Bengal made by Lord Curzon in 1905 lasted
province came into existence on 16 October, 1905, by until 1911, when King George V announced to annul the
breaking up Bengal and its Bengali speaking people. partition of Bengal at the Royal Darbar in Delhi in 1911. It
was decided to annul the partition of Bengal in 1911 mainly
In the beginning the leaders resorted to sharp press
to curb the menace of revolutionary activities.
campaigns against the partition scheme, numerous public
meetings in opposition to it and the drafting of petitions to On 22 March 1911, a royal proclamation announced that the
the Government for annulment. However, they realized it Durbar would be held in December to commemorate the
made no effect to the authorities. This led to a search for coronation in Britain a few months earlier of King George V
new techniques from the middle of 1905 and resulted in and Queen Mary and allow their proclamation as Emperor
the discovery of the boycott of British goods as an effective and Empress of India. Without public forewarning, the
weapon. The boycott of British products was followed announcement of the move of India‘s capital from Calcutta
by the advocacy of swadeshi (Indian-made) or exhorting to Delhi was also made at the Durbar. King George V also
purchasers to buy indigenously produced goods as a announced to annul the partition of Bengal
patriotic duty. Thus, Swadeshi and Boycott started against
11. Solution (a)
Partition of Bengal in 1905. It also showcase the rich swadeshi
tradition of culture, art, literature, music and science. Exp) Option a is the correct answer.

101 PYQ Workbook


HISTORY OF MODERN INDIA

The Ghadar party was found in San Francisco by Lala 15. Solution (a)
Hardayal, Sohan Singh Bakhna, Mohammed Barkattullah Exp) Option a is the correct answer.
etc. Shortly after the outbreak of World War I, many of the
The role of Sir William Wedderburn and W.S. Caine were
Ghadrites returned to India and for several months during
instrumental in setting up the Indian Parliamentary
1915 carried on terrorist activities in central Punjab. Ghadar Committee in 1893. Wedderburn played a key role in
Party was associated with the Ghadar mutiny which was passing the resolution for simultaneously conducting Civil
meant to overthrow the British rule from India. Services Exam in India.
12. Solution (c) The British Committee of Indian National Congress decided
Exp) Option c is the correct answer. to carry Indian Propaganda on three fronts:
• In House of Common by organising the Indian
The Criminal Procedure Code of 1873 laid down that except
Parliamentary Committee
in Presidency towns no Magistrate or Sessions Judge could
try a European British subject unless he himself was a • On platform, by arranging meetings and lectures
European by birth. • In the press, by founding the journal ‘India’
The Illbert Bill (1883) , proposed by viceroy Lord Ripon Important Tips
intended to give jurisdiction to Indian judges over Europeans.
Indian Parliamentary Committee:
It sought to empower the Indian district Magistrates and
sessions judges to try Europeans in criminal cases. The British Committee of Indian National Congress was
founded in 1889 by Sir William Wedderburn as Chairman
Ripon’s Government had sought to abolish “judicial
and William Digby as it’s secretary to achieve political
disqualification based on race distinctions” and to give the
reforms in India by gathering support from British
Indian members of the covenanted civil service the same
public. Dadabhai Naroji and W.S.Caine were some of its
powers and rights as those enjoyed by their European
important members. Sir William Wedderburn entered
colleagues. Ripon had to modify the bill, thus almost
House of Commons in British Parliament in 1893 as a
defeating the original purpose, because of the stiff opposition
Liberal member.
from the European community.
16. Solution: (a)
13. Solution (a)
Exp) Option a is the correct answer.
Exp) Option a is the correct answer.
The immediate cause for the launch of the Swadeshi
Naoroji’s major contribution to the Indian independence Movement was the partition of Bengal done by Lord
movement was his “Drain of Wealth” theory, a detailed Curzon.
analytical study of how the colonial rulers of the
Swadeshi Movement was a comprehensive movement that
subcontinent pillaged its economic resources and
lasted 6 years.
shattered its industrial capacity. The theory was most
cogently demonstrated in his 1901 work Poverty and Un- Important Tips
British Rule in India.
Swadeshi Movement:
Naoroji used the British imperial state’s own data to prove
• Although Swadeshi Movement was regarded as a
its historical impoverishment of the subcontinent by
cultural movement celebrating the rich harvest of
mapping Indian net profit alongside different ventures being
history, the rich traditions like folk music, paintings,
undertaken by the British Raj. He calculated that drain of
the culture of Bengal were highlighted
India’s wealth, resulted in a yearly loss of approximately £30-
40 million with only £250,000’s worth of capital injected • Tilak called this movement “Bahishkar Yoga“ and most
back into India per annum. of the Bengali intelligentsia that was initially not in
favor of the boycott movement got integrated with the
14. Solution (b) Swadeshi movement. This was primary learning for the
Exp) Option b is the correct answer. national movement.
Home Charges’ referred to the expenditure incurred in • According to Bal Gangadhar Tilak, and Lala Lajpat
England by the Secretary of State on behalf of India. Rai, it was training in ‘self determination’, ‘self-help’,
The following constituted the Home Charges: and ‘self-reliance’

Dividends on East India Stock, Interest on Public debt raised 17. Solution: (b)
abroad, the salaries of the officers, establishments of the and
Exp) Option b is the correct answer.
building connected with the Home Department of Indian
Government, furlough and pensions to British officers in The Partition of Bengal on 16 th October 1905, despite
facing strong opposition from Indian nationalists, marked a
Military and Civil departments in India. Charges of all
significant turning point for the Indian National Congress.
descriptions paid to the British troops serving in India and
Lord Curzon, the Viceroy of British India, proposed the
portion of the cost of transporting the British troops to and
partition as a means to improve governance in the vast
from India.
province.

PYQ Workbook 102


HISTORY OF MODERN INDIA

Important Tips Important Tips


Partition of Bengal: Rowlatt Act
• Bengal, which included present-day West Bengal, • The act authorised the government to imprison any
Bihar, Odisha, Bangladesh, and Assam, was under suspect without trial and conviction in the court of law.
British rule since 1765. • It enables the government to suppress the habeas
• The province became challenging to administer due to corpus.
its large size and rapidly growing population, reaching • The Government wanted to replace the provision of
almost 80 million by the early 20th century. repression in Defense of India Act 1915 by a permanent
• Initially, Curzon proposed the partition as a purely law.
administrative measure but later saw it as a political • It was based on the report of Justice S.A.T. Rowlatt’s
tool to undermine growing nationalism. Committee of 1918.
• The proposed partition would result in two provinces: • Mahatma Gandhi organized satyagraha against Rowlett
Bengal (including West Bengal, Odisha, and Bihar) and Act. This was the first nationwide mass uprising under
Eastern Bengal and Assam. leadership of Mahatma Gandhi.
• Bengal would have a Hindu majority, while Eastern • The act was never actually implemented.
Bengal and Assam would have a Muslim majority.
Calcutta would remain the capital. 19. Solution: (c)
Exp) Option c is the correct answer.
18. Solution: (c)
The first conference of the Indian National Congress was
Exp) Option c is the correct answer. held in Bombay from December 28 to 31, 1885. It was
attended by 72 delegates from all over India. The session was
The Rowlatt Act was passed by the government under the presided over by Womesh Chandra Bonnerjee, a lawyer from
period of Lord Chelmsford. It was passed in February 1919. Calcutta. The main topics discussed at the session were the
need for greater Indian representation in the government,
All Indian member of the Central Legislative Council the improvement of education and the development of the
opposed the bill. Indian economy.
Important Tips
List of all the Indian National Congress Session from 1886 till Independence:
Year Location President Importance
1886 Calcutta Dadabhai Naoroji National Conference and National Congress.
1887 Madras Syed Badruddin Tyabji Muslims were asked to join hands with national leaders.
1888 Allahabad George Yule First Non-Indian/English President.
1889 Bombay Sir William Wedderburn –

1890 Calcutta Feroz Shah Mehta –

1891 Nagpur P. Ananda Charlu –

1892 Allahabad W C Bonnerjee –

1893 Lahore Dadabhai Naoroji –

1894 Madras Alfred Webb –

1895 Poona Surendranath Banerjee –

1896 Calcutta Rahimtullah M. Sayani National song ‘Vande Mataram’ sung for the first time

1897 Amravati C. Sankaran Nair –

1898 Madras Ananda Mohan Bose –

1899 Lucknow Romesh Chandra Dutt –

1900 Lahore N G Chandavarkar –

1901 Calcutta Dinshaw E. Wacha –

103 PYQ Workbook


HISTORY OF MODERN INDIA

1902 Ahmedabad Surendranath Banerjee –

1903 Madras Lal Mohan Ghosh –

1904 Bombay Sir Henry Cotton –

1905 Benares Gopal Krishna Gokhale Expressed resentment against the partition of Bengal

1906 Calcutta Dadabhai Naoroji The word ‘Swaraj’ was mentioned for the first time

1907 Surat Rash Behari Ghosh Party splits into extremists and moderates

1908 Madras Rash Behari Ghosh Previous session continued

1909 Lahore Madan Mohan Malaviya Indian Councils Act, 1909

1910 Allahabad Sir William Wedderburn –

1911 Calcutta Bishan Narayan Dhar ‘Jana Gana Mana’ sung for the first time

1912 Bankipore Raghunath Narasinha –


(Patna) Mudholkar

1913 Karachi Syed Mohammed –

1914 Madras Bhupendra Nath Basu –

1915 Bombay Satyendra Prasanna Sinha –

1916 Lucknow Ambica Charan Mazumdar Lucknow Pact – joint session with the Muslim League

1917 Calcutta Annie Besant (1847 – 1933) First woman president of the INC

1918 Bombay And Syed Hasan Imam (Bombay) Two sessions were held. First in Bombay in August/
Delhi And Madan Mohan Malaviya September Second in Delhi in December
(Delhi)

1919 Amritsar Motilal Nehru Jallianwala Bagh massacre strongly condemned

1920 Nagpur C Vijayaraghavachariar –

1921 Ahmedabad Hakim Ajmal Khan (acting –


President For C R Das)

1922 Gaya C R Das –

1923 Kakinada Maulana Mohammad Ali, –

1924 Belgaum M K Gandhi –

1925 Kanpur Sarojini Naidu (1879 – 1949) First Indian woman president

1926 Guwahati S Srinivasa Iyengar –

1927 Madras M A Ansari –

1928 Calcutta Motilal Nehru All India Youth Congress formed

1929 Lahore Jawaharlal Nehru Resolution for ‘Poorna Swaraj.’ Civil Disobedience
movement for complete independence to be launched, 26
January to be observed as ‘Independence Day’.

1930 No Session – –

1931 Karachi Vallabhbhai Patel Resolution on fundamental rights and national economic
progress. Gandhi-Irwin pact endorsed. Gandhi nominated
to represent INC in the second round table conference

PYQ Workbook 104


HISTORY OF MODERN INDIA

1932 Delhi Amrit Ranchhorddas Seth –

1933 Calcutta Malaviya Was Elected But –


Mrs Nellie Sengupta Presided

1934 Bombay Rajendra Prasad –

1937 Lucknow Jawaharlal Nehru –

1936 Faizpur Jawaharlal Nehru First rural session/first session to be held in a village

1938 Haripura Subhas Chandra Bose National planning committee set up under Nehru

1939 Tripuri Subhas Chandra Bose Bose was elected but had to resign since Gandhi supported
Pattabhi Sitaramayya. Instead, Rajendra Prasad was
appointed

1940 Ramgarh Abul Kalam Azad –

1941-45 – – No session because of arrest

1946 Meerut Acharya Kripalani Last session before independence

1948 Jaipur Pattabhi Sitaramayya First session after independence

20. Solution: (b) the distant East. The Ghadar Party, an Indian revolutionary
Exp) Option b is the correct answer. group, originated mainly with Indian individuals. It
exhibited a diverse composition, featuring leaders from
Statement 1 is incorrect- Bhikhaiji Cama put up the “Flag
Sikh, Hindu, and Muslim backgrounds. This organization’s
of Indian Independence” on August 22, 1907, during a
headquarters were located in San Francisco, United States
meeting in Germany. This flag looked like the Calcutta Flag
and had green, yellow, and red parts that stood for Islam, (West America).
Hinduism, and Buddhism. 23. Solution: (c)
Statement 2 is correct- She got to know Dadabhai Naoroji, Exp) Option c is the correct answer.
who was leading the British Committee of the Indian National
Rabindranath Tagore returned his Knighthood award as a
Congress. She started working as his private secretary. She,
protest when he came to know about the Jallianwala Bagh
along with Naoroji and Singh Rewabhai Rana, helped start
Massacre on 22 May 1919. He arranged a protest meeting
Varma’s Indian Home Rule Society in February 1905.
in Calcutta, post which, he finally gave off his British
Statement 3 is correct- Born on September 24, 1861, in
Knighthood as ‘a symbolic act of protest’.
Bombay (which is now Mumbai), Madam Cama came from
a well-off Gujarati Parsi family. Her parents were Sorabji 24. Solution: (a)
Framji Patel and Jaijibai Sorabji Patel. Her dad was a Exp) Option a is the correct answer.
merchant and an important person in the Parsi community.
The nationalist movement in India was a period of intense
They were a famous couple in the city.
struggle against British colonial rule, marked by various
21. Solution: (c) movements and agitations. The Swadeshi movement was
Exp) Option c is the correct answer. one such movement that aimed at promoting Indian-made
goods and boycotting British goods. Liaquat Hussain was not
The Komagata Maru, a steamship built in Scotland in 1890
associated with any such agitation in Barisal. He took the
and later owned by Germany and Japan, saw various names
movement to Patna and organised the East Indian Railway
including Stubbenhuk, Sicilia, Komagata Maru, and Heian
strike in 1906. He also wrote fiery articles in Urdu to rouse
Maru. In 1914, it transported 376 Sikh and other migrants
from the Far East to Canada, resulting in the Komagata nationalist sentiments in Muslims. He was supported by
Maru incident when Canadian authorities in Vancouver other Muslim swadeshi agitators like Ghaznavi, Rasul, Din
refused most of them entry and they were on ship for the 2 Mohammed, Dedar Bux, Moniruzzaman, Ismail Hussain,
months. Later, they were forced to return to India where the Siraji, Abdul Hussain and Abdul Gaffar. Hence Option a is
Imperial Police attempted to arrest the group leaders. incorrect.
Satish Chandra Mukherjee was a prominent educationist
22. Solution: (b)
and social reformer who formulated the scheme of national
Exp) Option b is the correct answer. education in 1889. The scheme aimed at promoting
Situated in San Francisco, the Ghadr revolutionaries education among the masses and was an important aspect of
established branches not only along the US Coast but also in the nationalist movement. Hence Option b is correct.

105 PYQ Workbook


HISTORY OF MODERN INDIA

The Bengal National College was founded in 1906 by Important Tips


Aurobindo Ghosh and other nationalist leaders. Aurobindo Hunter Commission
was the Principal of the college and the institution played
• On October 14, 1919, the Government of India
an important role in promoting nationalist ideology and announced the formation of the Disorders Inquiry
education. Hence Option c is correct. Committee.
Rabindranath Tagore was a prominent nationalist leader • The committee was commonly known as Hunter
Commission after the name of chairman, Lord William
who promoted the concept of Atmasakti or self-reliance. He
Hunter. It also had Indian members.
believed that the regeneration of Indian society could only
• In the final report submitted in March 1920, the
be achieved through the upliftment of the villages. Hence committee unanimously condemned Dyer’s actions.
Option d is correct. • However, the Hunter Committee did not impose any
penal or disciplinary action against General Dyer.
25. Solution: (d)
Exp) Option d is the correct answer. 27. Solution: (b)
In 1875 the Poona Sarvajanik Sabha submitted a petition Exp) Option b is the correct answer.
to the House of Commons signed by 21,713 persons On 18 April 1930, Chittagong armoury raid led by Surya
demanding direct representation of India in Parliament. Sen. In 1929, Surya Sen was the Secretary of the Chittagong
The influential Poona Sarvajanik Sabha, formed in April District Congress Committee. In early 1929, they formed
1870 in Maharashtra, aimed to mediate between the British a plan to organize an armed rebellion. They plan to capture
two main armouries in Chittagong and the arming of a
government and Indians, addressing public grievances and
large band of revolutionaries with the seized arms. The raid
fostering patriotism. Led by figures like G.V. Joshi and
was undertaken inthe name of Indian Republican Army,
Mahadev Govind Ranade, it supported local communities, Chittagong Branch. Surya Sen was formally declared the
established arbitration courts, and organized famine relief. President of the Provisional Revolutionary Government.
Important Tips The Union Jack was pulled down and the National Flag was
hoisted.
The Deccan Association:
• Vishnushastri Chiplunkar, Bal Gangadhar Tilak, 28. Solution: (b)
and Gopal Ganesh Agarkar established the Deccan Exp) Option b is the correct answer.
Education Society in 1884, which founded Fergusson
Statement 1 is incorrect- The first woman to be the President
College in Pune in 1885. The society played a vital
of the Congress was Annie Besant, who was elected president
role in promoting education, establishing schools
in 1917 and 1933.
and colleges, with significant figures like Gokhale
and Karve contributing. It expanded its educational Statement 2 is correct-C.R. Das was in prison when he
endeavors across Maharashtra, gaining support from functioned as the President of the Congress in 1922. He was
various princely states. arrested by the British government for his role in the Non-
Madras Mahajana Sabha: Cooperation Movement.
• Founded in 1884 by M. Veeraraghavachariar, G. Statement 3 is incorrect-George Yule, a Scottish merchant
Subramania Iyer, and P. Anandacharlu, Madras living in England, was the first Britisher to hold the position
Mahajana Sabha was a significant Indian nationalist of President in the Indian National Congress (INC). In
organization based in the Madras Presidency. It 1888, he became the fourth President of the INC during the
aimed to address Indians’ rights and welfare, advocating session held in Allahabad.
education, social reform, and political consciousness. Statement 4 is correct-Alfred Web presided the session of
It played a role in preparing the ground for the Indian
Indian National Congress in 1894 at Madras.
National Congress and actively participated in freedom
movements, including the Salt Satyagraha. 29. Solution: (a)
Exp) Option a is the correct answer.
26. Solution: (b) • Kalpana Dutt was a remarkable Indian revolutionary
Exp) Option b is the correct answer. who participated in the Chittagong Armory Raid led by
Surya Sen on 18 April 1930. In the jail, she was visited
After the terrible massacre at Jallianwala Bagh on 13th April,
by Gandhi and received letters from Rabindranath
1919, the Legislative Council of the government of India Tagore and C F Andrews. The latter addressed her as his
constituted the Hunter Committee to examine the mishap daughter from his previous life. Kalpana was given a life
that occurred there. sentence but was eventually released in 1939.

PYQ Workbook 106


HISTORY OF MODERN INDIA

• Baba Ram Singh Kuka started the Kuka Movement in Important Tips
1872. The Kukas were also referred Namdharis, who
Gandhi and Rowlatt act:
were a sect within Sikhism. They started as a group for
• Mahatma Gandhi wanted non-violent civil
religious purification in Sikhism under Ram Singh. He
disobedience against Rowlatt Act, which would start
was the first Indian who used Non-Cooperation as an
with a hartal on 6 April.
ideology and boycotted British services and goods.
• Rallies were organized in various cities, workers went
The movement received a political overtone with the
on strike in railway workshops, and shops were closed
established aim of restoring Sikh rule in Punjab and down.
ousting the foreign powers.
• Alarmed by the popular upsurge, and scared that lines
• Vinayak Damodar Savarkar was the founder of of communication such as the railways and telegraph
theAbhinav Bharat society. It was one among several would be disrupted, the British administration decided
such revolutionary societies functioning in Maharashtra to clamp down on nationalists.
at that time, which believed in the overthrow of British • Local leaders were picked up from Amritsar, and
rule through armed rebellion. Mahatma Gandhi was barred from entering Delhi.
• Aurobindo Ghosh was closely associated with the
Anushilan Samiti. Anushilan Samiti was a well-known 33. Solution: (b)
covert revolutionary organisation that operated out of Exp) Option b is the correct answer.
Bengal in the 20th century with the goal of removing Option a is incorrect: Stimulating the production of
colonial authority. indigenous articles by giving them preference over imported
commodities was a part of the Swadeshi Movement, which
30. Solution: (d) was associated with both moderate and extremist phases
Exp) Option d is the correct answer. of the freedom movement. It aimed at promoting domestic
Subhash Chandra Bose in his presidential address (1938), industries to reduce economic dependence on British goods.
advocated the introduction of Roman script for the Hindi Option b is correct: The extremist ideology during the early
language. He argued that this would make it easier for phase of the Indian freedom movement focused on achieving
people to learn Hindi and that it would help to promote self-government through assertive means rather than relying
unity among the Indian people. Bose’s proposal was not on petitions and constitutional ways. This approach involved
popular with all Congress members. Some people felt that it active resistance, mass mobilization, boycotts, and non-
cooperation to challenge oppressive British colonial rule
would be a betrayal of the Indian heritage to abandon the
and advance India’s independence.
Devanagari script. However, Bose’s proposal was eventually
adopted by the Congress. Option c is incorrect: Providing national education
according to the requirements of the country was a goal
31. Solution: (b) shared by both moderate and extremist leaders. The focus
Exp) Option b is the correct answer. was on fostering education that met the needs of the Indian
society and promoted a sense of national identity.
Surendranath Banerji was dismissed by the British from
the Indian Civil Service due to a minor judicial error. Option d is incorrect: Organizing coups against the British
empire through military revolt doesn’t accurately represent
Despite his appeals in England, he believed that racial
the ideology of the early phase of the Indian freedom
discrimination was a factor in his dismissal. This incident
movement. Extremists focused on non-violent methods of
contributed to his growing disillusionment with the British
protest and active resistance, rather than outright military
administration and fueled his nationalist sentiments. He
coups. They aimed to challenge British dominance through
became a prominent Indian nationalist leader, founding civil disobedience and mass mobilization.
the Indian National Association and advocating for Indian
political rights. Important Tips
Extremist Movement during the early phase of Indian
32. Solution: (b)
freedom movement:
Exp) Option b is the correct answer. The Extremist leaders (1905-1920)diverged from the
Rowlatt Satyagraha was the first venture of Gandhi in all- Moderates in their skepticism towards British rule and
India politics. On 24th February, 1919 at Bombay, Mahatma belief in British justice. They rejected petitioning and
Gandhi started a campaign and formed a Satyagraha Sabha embraced agitation. Their approach included:
against the Rowlatt Act. Rowlatt Act permitted few political • Boycott and Swadeshi Movement: They urged the
cases to be tried without juries and allowed internment of boycott of foreign goods and promotion of Swadeshi
suspects without trial. products to stimulate indigenous industries and trade.

107 PYQ Workbook


HISTORY OF MODERN INDIA

• Non-Cooperation: Extremists refused cooperation 36. Solution: (d)


with the British bureaucracy, even boycotting Exp) Option d is the correct answer.
governmental activities. • Abhinav Bharat society was founded by Vinayak
• Nationalistic Education: They established schools Damodar Savarkar in 1904. It was one among several
and colleges using Indian languages, instilling pride such revolutionary societies functioning in Maharashtra
in India’s heritage, fostering nationalism, and self- at that time, which believed in the overthrow of British
reliance in students. rule through armed rebellion.
• Passive Resistance: Non-payment of revenue and taxes, • Anushilan Samitiwas one of the secret revolutionary
coupled with setting up indigenous administrative organisations operating in Bengal founded by
units parallel to British ones at local levels. Satish Chandra Basu. It was led by the nationalists
• Mass Mobilization: Public meetings and processions Pramathanath Mitra,Aurobindo Ghosh and his brother
galvanized public sentiment and engagement. Barindra Ghosh and others.
• Swadeshi Enterprises: Extremists established • The Ghadar Party, initially named the Pacific Coast
Swadeshi textile mills, soap factories, banks, and more, Hindustan Association, was formed in was formed on 15
driven by patriotism rather than business intent. July 1913 in the United States under the leadership of
Lala Har Dayal, Baba Jawala Singh, Santokh Singh and
• Non-Violence: Extremists discouraged violence
Sohan Singh Bhakna as its president.
against British rule and disapproved of political
assassinations. • The swaraj party was formed by Chittaranjan Das,
Motilal Nehruand other members of the Indian National
• Support for Revolutionaries: They sympathized with
Congress on 1 January 1923. Chittaranjan Das was the
revolutionary activities but did not directly engage in
President of the Swaraj Party. The swaraj party was
them.
formed to fight British colonialism.
34. Solution: (d) 37. Solution: (a)
Exp) Option d is the correct answer. Exp) Option a is the correct answer.
The Indian Muslims, in general, were not attracted to the Anarchical and Revolutionary Crimes Act of 1919,
extremist movement due to the extremist leaders’ policy popularly known as the Rowlatt Act was a legislative council
of harping on Hindu aspects. The extremist movement in act passed by the Imperial Legislative Council in Delhi in
India was largely Hindu-centric and focused on reviving March 1919. The act indefinitely extended the emergency
Hindu culture and traditions. This alienated many Indian measures of preventive indefinite detention, incarceration
Muslims, who felt that the movement did not represent without trial.
their interests. The extremist movement also used religious
symbolism and rhetoric that was offensive to many Muslims. 38. Solution: (c)
For example, the movement celebrated Hindu festivals and Exp) Option c is the correct answer.
figures, such as Shivaji and Ganesha. This further alienated The Barrah dacoity was the first major venture of the
Muslims from the movement. revolutionary terrorists of the freedom movement in 1908,
in Dhaka district of East Bengal. Barrah dacoity was
35. Solution: (b)
organised by Dacca Anushilan under Pulin Das to raise
Exp) Option b is the correct answer. funds for revolutionary activities.Revolutionaries attacked
Lala Har Dayal was the leader of the Ghadar Party, a the residence of the Zamindar of Barrah.
revolutionary organization founded in 1913 by Indian
39. Solution: (d)
immigrants in the United States and Canada. The Ghadar
Party was committed to overthrowing British rule in India Exp) Option d is the correct answer.
through armed revolution. All of the mentioned international events influenced the
course of the Indian national movement before Mahatma
Important Tips
Gandhi’s arrival:
Ghadar Party:
• Italian-Abyssinian War, 1898: The Italian invasion
Lala Hardayal, along with individuals like Baba Sohan of Abyssinia (now Ethiopia) in 1898 captured the
Singh Bhakna, Sant Baba Wasakha Singh Dadehar, attention of Indian nationalists. The invasion exposed
Pandit Kanshi Ram Maroli, Baba Jawala Singh, and the imperialistic ambitions of European powers and their
Santokh Singh, played crucial roles. Notably, Lala disregard for the sovereignty of smaller nations, serving
Hardayal initiated the formation of the Hindi Association as a reminder of British colonial rule in India.
of the Pacific Coast, which would later be recognized as
• Boxer Movement in China: The Boxer Rebellion (1899-
the Hindustan Ghadar Party in May 1913. The purpose
1901) against foreign influence and Christianity in
behind this initiative was to exert influence over the
China resonated with Indians fighting for independence.
Indian immigrant population in Portland.
The rebellion highlighted the consequences of foreign

PYQ Workbook 108


HISTORY OF MODERN INDIA

dominance and sparked discussions about anti- 2. Chittagong Armoury Raid- On April 18, 1930, a group
imperialism and self-determination. of 63 young individuals, many of them teenagers, under
• Revolutionary Movement in Ireland: The Irish struggle
the leadership of Surjya Sen, also known as Surya Sen
against British colonialism, particularly during the
late 19th and early 20th centuries, inspired Indian and Master-da, conducted a raid on the police and
nationalists. The Irish movement’s tactics and challenges auxiliary forces’ arsenal in the Chittagong district
were often compared to those faced by Indians, fostering
solidarity and shared understanding. of undivided Bengal. They successfully freed it from

• Victory of Japan in the Russo-Japanese War: Japan’s British authority and raised the Indian flag.
victory in the Russo-Japanese War (1904-1905) showed
that an Asian nation could successfully challenge a 42. Solution: (d)
European power. This boosted the self-confidence of
Exp) Option d is the correct answer.
Indians and led them to believe that they too could
confront British colonial rule. Dr. Mukhtar Ahmed Ansari was a doctor and received

40. Solution: (c) his degree in medicine from Madras Medical College. The
Exp) Option c is the correct answer. Madras session of INC was presided over by M.A. Ansari in
• Bhikaiji Rustom Cama held a significant role in India’s the year 1927. He also served as the President of the Muslim
fight for independence. On August 22, 1907, she achieved
League in the year 1920. He was not associated with Kanpur
the historic feat of raising the Indian flag on foreign land
in Stuttgart, Germany. Conspiracy Case of 1924.
• Abdul Hafiz Mohamed Barakatullah was an Indian
Important Tips
freedom fighter who felt aligned with the Pan-Islamic
movement. Kanpur Bolshevik Conspiracy Case:
• Varahaneri Venkatesa Subramaniam Aiyar, hailing • Kanpur Conspiracy Case of 1924was responsible for
from Tamil Nadu, was an Indian patriot who actively actively introducing communism to the Indian masses.
opposed the British rule in India. He shared his time with • A series of prominent activists including Muzaffar
fellow revolutionaries like Subramanya Bharathi and Ahmad, S. Chettiar,Nalini Gupta, Shaukat
V.O. Chidambaram Pillai, who advocated for a more Usmani,S.A. Dange, Ghulam Hussain among others
militant approach to resist the British presence. were arrested and charged with conspiring to separate
• In the 20th century, Manabendra Nath Roy emerged India from Britain through armed revolution.
as an Indian revolutionary, radical campaigner, and • Ghulam Hussain turned a British informer and was
political theorist, alongside being a distinguished pardoned.
philosopher. Roy played a pivotal role in establishing • Through these arrests and trials, the government hoped
both the Mexican Communist Party and the Communist to crush the Socialist movement in India entirely.
Party of India.
• The newspapers covered the matter exhaustively and
this was for the first time the people of India could
3.2. Other Examination Previous Years’
know the communist doctrine in details.
Questions
41. Solution: (a) 43. Solution: (a)
Exp) Option a is the correct answer. Exp) Option a is the correct answer.
1. The Foundation of Ghadar Party- The Ghadar Party’s
establishment took place when the Pacific Coast Lord Dufferin was the Governor-General of India from
Hindustan Association, later known as the Ghadar Party, 1884 to 1888. He was a conservative who was skeptical of
was formed on July 15, 1913, in the United States.
the Indian National Congress. He ridiculed the Congress as
3. Setup of the ‘Indian Independence Committee’
representing only a “microscopic minority” of people and
at Berlin- In 1915, the Berlin committee was set
up with the aim of achieving Indian Independence. argued that it did not represent the views of the majority
Notably, instrumental figures in its establishment were of Indians. In a speech in 1888, Lord Dufferin said, “The
Virendranath Chattopadhyay, Bhupendranath Dutta,
and Lala Hardayal. Congress is to my mind to a great extent a microscopic
4. Central Assembly Bomb Case- On April 8, 1929, Bhagat minority, representing the views and wishes of a very small
Singh and Batukeshwar Dutt, revolutionaries, hurled section of the population.” He also said that the Congress
bombs at the Central Legislative Assembly in Delhi,
leading to an event referred to as the Central Assembly was “an artificial creation” and that it did not represent the
Bombing Case in modern Indian history. “true feelings of the people.”

109 PYQ Workbook


HISTORY OF MODERN INDIA

was killed. It is said that the Delhi Conspiracy was hatched


Important Tips
by Ras Bihari Bose, but was never proved. In the trial of this
• Lord Curzon: He was the Governor-General of India
from 1899 to 1905. Some of the major reforms done Delhi Conspiracy Case, Basant Kumar Biswas, Amir Chand
by him includes: and Avadh Behari were convicted and executed. Delhi
 Relief to famine affected people, Conspiracy Case is also known as Delhi-Lahore Conspiracy
 Education reforms (Indian Universities Act, 1904), Case or Hardinge Bomb Case.
 Reduction in Salt tax Rate, Pair 2 is correct: The major revolutionaries involved in
 Cooperative Credit Societies Act, the Kakori Conspiracy Caseof 1925 included Ram Prasad
 Number of judges in Calcutta High Court Bismil, Ashfaqulla Khan, Rajendra Lahiri, and others.
increased, and
Pair 3 is correct: Many young revolutionaries including
 Improvement in railway infrastructure.
Bhagat Singh, Jatin Das, Raj guru and Sukh Dev were charged
• Lord Minto: He was the Governor-General of India
from 1905 to 1910. He introduced the Morley-Minto for involvement in the murder of police officer J P Saunders
reforms, which gave Indians more representation in in 1929, which is famously known as Lahore Conspiracy
the government and introduced the system of separate Case. Jatin Das waspassed away after a sixty-three-day
electorate for the first time.
hunger strike while he was imprisoned in the Lahore jail.
• Lord Lansdowne: He was the Governor-General of
India from 1888 to 1894. Some of the event happened Pair 4 is incorrect: Rash Bihari Bose was not associated
during his tenure: with Nasik Conspiracy Case. Anant Laxman Kanhere, a
 Factory Act (1891), member ofAbhinav Bharat Society, shot A M T Jackson, a
 Indian Councils Act (1892), magistrate at Nasik, on 21 December 1909 at a theater where
 Setting up of Durand Commission (1893). a drama was to stage in his honor on the eve of his transfer.
This sensational murder is known as Nasik Conspiracy
44. Solution: (b) Case.
Exp) Option b is the correct answer.
47. Solution: (a)
The Asiatic Society of Bengal in Calcutta was founded by
the philologist William Jones in 1784 during the Company Exp) Option a is the correct answer.
rule in India. It aimed to promote “Oriental research” and Feroz Shah Mehta was one of the founders of the Bombay
the study of India and surrounding regions. Renamed over
Presidency Association in 1885. Along with K.T. Telang and
the years, it became “The Royal Asiatic Society of Bengal.”
Badruddin Tyabji, he played a significant role in establishing
The society’s headquarters is located in Kolkata (Calcutta),
housed in a building at Park Street. the association as a response to the policies of Lord Lytton
and the Ilbert Bill controversy. The association aimed to
45. Solution: (b)
represent the interests and concerns of the people of the
Exp) Option b is the correct answer.
Bombay Presidency during the colonial period.
Mahatma Gandhi established the All India Harijan Sevak
Sangh (AIHSS) in 1932 to work for the upliftment of the 48. Solution: (b)
untouchables, now known as Dalits. The AIHSS is a non-
Exp) Option b is the correct answer.
profit organization that works to improve the social,
economic, and political status of Dalits through education, In 1920, at a meeting of the Indian National Congress in
health care, and other programs. The AIHSS has been active Nagpur, C. Vijay Raghav Chariar became the President.
in India for over 80 years and it has made significant progress
This was the same event where people talked about and
in improving the lives of Dalits. The organization has helped
to build schools and hospitals, provide scholarships, and agreed with Gandhiji’s idea of getting complete freedom
promote social awareness about the problems faced by (Poorna Swaraj) using non-violence and non cooperation
Dalits. with the British.

46. Solution: (d) 49. Solution: (b)


Exp) Option d is the correct answer.
Exp) Option b is the correct answer.
Pair 1 is correct: On 23 December 1912, a Bomb was thrown
at the Viceroy Lord Hardinge when his procession was Abul Kalam Azad was the President of the Indian National
moving from Chandni Chowk. The Viceroy wounded in the Congress continuously for six years from 1940 to 1946. He
attempt, but his Mahavat (driver and keeper of an elephant) was the longest-serving President of the Congress.

PYQ Workbook 110


HISTORY OF MODERN INDIA

Important Tips Dadabhai Naoroji:


Maulana Abul Kalam Azad: • Dadabhai Naoroji, revered as India’s Grand Old
• Born on November 11th, 1888, in Mecca, Saudi Man, was a founding member of the Indian National
Arabia. Congress and served as its president thrice. He
coined the term “drain theory” in his book ‘Poverty
• Authored weekly journals Al-Hilal and Al-Balagh.
and Un-British Rule in India’ to expose British
• Gained the nickname “Congress Showboy.” exploitation. He was the first Indian MP in the British
• Participated actively in movements like Non- House of Commons, advocating for Indian interests.
Cooperation and Quit India. His extensive contributions spanned education,
• Held the position of the first Education Minister. nationalism, economic thought, social reform, and
more.
• His birth anniversary, November 11th, is celebrated as
National Education Day. 51. Solution: (d)
• Posthumously awarded the Bharat Ratna in 1992. Exp) Option d is the correct answer.
Gopal Krishna Gokhale was a moderate nationalist leader.
50. Solution: (d) He believed in the methods of persuasion and cooperation
Exp) Option d is the correct answer. with the British. He was a strong advocate for social reforms,
Dadabhai Naoroji continuously championed “self-rule” for such as education and widow remarriage. Bipin Chandra Pal,
India starting from 1904. Often referred to as the “Grand Bal Gangadhar Tilak, and Lala Lajpat Rai and Aurobindo
Old Man of India,” he held a prominent role in politics and Ghose were the prominent extremist nationalist leaders.
writing. In the 1906 Calcutta session under his leadership, the They believed that the British could not be persuaded to
Congress embraced Swaraj as India’s aspiration. Resolutions give India independence, and that they had to be forced to
on Bengal’s partition, Swadeshi, national education, and do so. They advocated for a more militant approach to the
boycott were adopted during this session. His influential independence struggle, including swadeshi (self-reliance)
work, “Poverty and Un-British Rule in India,” exposed the and swaraj (self-rule).
economic exploitation of India by Britain, fueling the call for Important Tips
self-rule within the Indian nationalist movement.
Prominent Extremist leaders, Lal-Bal-Pal Trio:
Important Tips Lala Lajpat Rai:
Prominent Moderate Leaders: • Lala Lajpat Rai, known as Punjab Kesari, was a
Surendranath Banerjea: prominent freedom fighter and a key leader in
India’s struggle for independence. He studied law in
• Surendranath Banerjea, known as Rashtraguru,
Lahore, influenced by Swami Dayananda Saraswati’s
founded the Indian National Association in 1876,
Arya Samaj. Part of the Lal-Bal-Pal trio, he staunchly
advocating political reforms. He initiated the concept
opposed British policies, supported non-cooperation,
of India’s political unity through the association. As a
and founded the Home Rule League in the USA. Rai
prominent speaker and educator, he stirred national
championed social causes, founded institutions,
consciousness among students. Establishing the Indian
authored significant works, and co-founded Punjab
Association marked his second major contribution.
National Bank. His tragic death followed a lathi charge
Banerjea’s leadership in the Indian National Congress,
during a protest against the Simon Commission in
including two presidencies, highlighted his role in
Lahore.
shaping India’s political landscape.
Bal Gangadhar Tilak:
Pherozeshah Mehta:
• Bal Gangadhar Tilak, known as Lokmanya Tilak,
• Pherozeshah Mehta, hailed as “Lion of Bombay,”
was a freedom fighter, lawyer, and educationist born
was a notable Indian leader. As President of the
on 23rd July 1856 in Ratnagiri, Maharashtra. He
Indian National Congress in 1890, he aimed to prevent
co-founded Deccan Education Society and Fergusson
extremists from dominating. Mehta played a pivotal
College. Advocating self-rule and swadeshi, he led the
role in establishing the modern Bombay Municipal
Extremist faction during the Surat Split, striving for
Corporation and co-founded the Bombay Presidency
complete independence. He initiated the Indian Home
Association. He founded the Bombay Chronicle,
Rule Movement and founded All India Home Rule
an influential English newspaper, and received
League. Tilak played a pivotal role in India’s struggle
knighthood for his contributions to governance and
for freedom, spending 6 years in Mandalay Prison
journalism.
and contributing through newspapers, writings, and
political actions before passing away on 1st August
1920.

111 PYQ Workbook


HISTORY OF MODERN INDIA

Bipin Chandra Pal: • Maulana Abul Kalam Azad became the President of
• Bipin Chandra Pal, a secular orator, freedom fighter, the Indian National Congress at the age of 35, making
and social reformer, played a vital role in India’s him the youngest to hold this position. He held this
independence movement. A member of the Lal- role continuously for six years, from 1940 to 1946.
Bal-Pal trio, he advocated Swadeshi and boycotts to He holds the record for being the Indian National
combat poverty and nationalism. He opposed British Congress’s longest-serving President before the
colonialism through strong nationalism, distinct from country gained independence.
Gandhi’s approach. As a journalist, he worked for • The list of Congress session Presidents featuring
Bengal Public Opinion, New India, and The Tribune, foreign individuals comprises George Yule (1888),
warning about geopolitical changes and India’s future William Weddenburn (1889, 1910), Alfred Webb
challenges. (1894), Henry Cotton (1904), and Annie Besant
Aurobindo Ghose: (1917).
• Sri Aurobindo (1872–1950), an Indian nationalist, • The first female President of the Indian National
initially championed independence from British Congress was Annie Besant, who led the 1917
rule and led extremist activities. Among the Calcutta session of the party, whereas the first Indian
extremist his approach was more radical and he had female President of the Indian National Congress was
ties with revolutionary groups like Anushilan Samiti Sarojini Naidu at the Kanpur Session, 1925.
(Extremists and revolutionaries were different
groups with different strategies). He faced trial for 53. Solution: (a)
the Alipore Conspiracy but was only convicted for
Exp) Option a is the correct answer.
anti-colonial articles. Mystical experiences in jail led
him to Pondicherry, where he embraced spiritual The Indian National Congress declared its policy
transformation and founded Sri Aurobindo Ashram towards the Indian States for the first time at the Nagpur
with Integral Yoga, envisioning human evolution session in 1920. The session was presided over by C.
into a divine existence. Notable works include “The Vijayaraghavachariar. The Congress declared that it would
Life Divine,” “Synthesis of Yoga,” and the epic poem work for the establishment of responsible government in
“Savitri.” the Indian States. It also declared that it would support
the people of the Indian States in their struggle for self-
government.
52. Solution: (c)
54. Solution: (b)
Exp) Option c is the correct answer.
Exp) Option b is the correct answer.
Womesh Chandra Bonnerjee was the first president of the Jawaharlal Nehru spoke of socialism as the key to the
Indian National Congress. He was a lawyer and a social solution of India’s problems in his presidential address to
the Lucknow session of the Indian National Congress in
reformer who was concerned about the lack of political
1936. He said that socialism was “the only ideology which
representation for Indians under British rule. He was elected can solve the problems of the world today” and that it was
president of the Indian National Congress at its first session “the only way to achieve real freedom and equality for the
people of India”.
in Bombay in 1885.
55. Solution: (b)
Important Tips
Exp) Option b is the correct answer.
• Dadabhai Naoroji was the second president of the
Indian National Congress. The second Session was Pandit Jagat Narain Mulla was a prominent lawyer and
held at Calcutta in 1886. public prosecutor in the United Provinces during the British
Raj. He is noted for contesting the Kakori conspiracy case
• Surendra Nath Bannerji was the ninth president of
on behalf of the British. He was a public prosecutor.He
the Indian National Congress. The ninth session was
was also a member of the Hunter Commission appointed
held at Poona in 1895.
to inquire into the events in Punjab following the Jallianwala
• Allan Octavian Hume was the founder of the Indian Bagh massacre.
National Congress. He never presided the Indian
National Congress Session. 56. Solution: (b)
• Madan Mohan Malviya served as the President of Exp) Option b is the correct answer.
Indian National Congress for highest number of The Ghadar Party’s inaugural president and founder was
times before independence. He held the position of Baba Sohan Singh Bhakna. Lala Har Dayal held the
the INC president in the years 1909, 1918, 1930, and
position of general secretary, while Pandit Kanshi Ram
1932.
Maroli undertook the role of treasurer.

PYQ Workbook 112


HISTORY OF MODERN INDIA

57. Solution: (a)


• Bismil wrote his autobiography in Gorakhpur Central
Exp) Option a is the correct answer. Jail which apparently was taken out from jail by
The correct match between List-I (Organizations) and List- Shiv Verma and later published by Ganesh Shankar
II (Founders) is: Vidyarthi.
A. Land Holders’ Society - 4. Dwarkanath Tagore • He was found guilty of waging war against the British
B. British India Society - 3. William Adams empire, given the death sentence and hanged on 19th
December 1927.
C. Indian Society - 2. Anand Mohan Bose
D. Indian Association - 1. S.N. Banerji 61. Solution: (d)
58. Solution: (a) Exp) Option d is the correct answer.
Exp) Option a is the correct answer. • Abhinav Bharat society was founded by Vinayak
The Amritsar Session of the Indian National Congress was Damodar Savarkar in 1904. It was one among several
held in 1919. It was a historic session as it saw the passage such revolutionary societies functioning in Maharashtra
of the Non-Cooperation Resolution, which was a major at that time, which believed in the overthrow of British
turning point in the Indian independence movement. The rule through armed rebellion.
session also passed a resolution to amend the Congress • Mitra Mela was the precursor of Abhinav Bharat Society.
Constitution. This amendment was to be drafted by Mahatma
It was established by Vinayak Damodar Savarkar and his
Gandhi and two other members, N.C. Kelkar and I.B. Sen.
brother Ganesh Damodar Savarkar at Nashik in the year
59. Solution: (d) 1899.
Exp) Option d is the correct answer. • Indian Republican Army was a short-lived revolutionary
Bal Gangadhar Tilak attended the Amritsar Session of army that was created by Surya Sen and the members of
the Indian National Congress in 1919. He was arrested the Anushilan Samiti in 1930. The army was created with
by the British government shortly after the session and he an aim of liberating the city of Chittagong and the Bengal
was imprisoned for two years. He died in 1920. Tilak was Presidency from British rule.
a prominent Indian nationalist and freedom fighter who
• Hindustan Republican Association (HRA) was a
was a leading figure in the Indian independence movement.
revolutionary organization formed in 1924. It was
He was a strong advocate of Swaraj (self-rule) and he was a
critic of the British Raj. formed by Ram Prasad Bismil and Sachindra Nath
Sanyal. The main leaders of the Hindustan Republican
60. Solution: (c) Association are Chandra Shekar Azad, Ram Prasad
Exp) Option c is the correct answer. Bismil, Ashfaq Ullah Khan, Thakur Roshan Singh
Ram Prasad Bismil was an Indian revolutionary who and Rajendra Lahiri. Sachindra Nath Sanyal wrote a
participated in the Manipuri conspiracy of 1918, and the manifesto for the HRA entitled ‘Revolutionary’.
Kakori conspiracy of 1925. He refused to drink the given
milk on the eve of his execution and said “Now, I shall take 62. Solution: (d)
my mother’s milk only”. Exp) Option d is the correct answer.
British authority managed to arrest the revolutionaries
Important Tips
involved in Kakori incident as there were some insiders, who
Ram Prasad Bismil:
had betrayed them and provided information to the British
• He was born on 11th June 1897 in a simple family in the regime. As the case had generated tremendous interest
Shahjahanpur district of present-day Uttar Pradesh.
in the country, efforts were also launched to provide the
• He joined the Arya Samaj Youth Association and revolutionaries legal help and fight their case in the court.
began spreading the teachings of Swami Dayanand. A committee was formed under the chairmanship of freedom
• He was a prolific writer and wrote patriotic poems in fighter Motilal Nehru for this purpose. The Members in the
Urdu and Hindi under various pen names: ‘Bismil’, committee were Acharya Narendra Dev, Govind Ballabh
‘Ram’, and ‘Agyat’. Pant, Chandra Bhai Gupta, Jawaharlal Nehru, Sriprakasha.
• He was involved in the Manipuri Conspiracy of
1918, wherein the police found a few young people, 63. Solution: (a)
including Bismil selling books that were banned by the Exp) Option a is the correct answer.
government. Bhagat Singh said, “Merciless criticism and independent
• He was the mastermind behind the infamous Kakori thinking are the two necessary traits of revolutionary
Conspiracy Case of 1925, where they looted a train thinking”. These constitute an integral part of his monumental
carrying government money, as they needed the funds essay ‘Why I am an Atheist’, authored by him in prison a
to support their activities.
mere five months before his execution.

113 PYQ Workbook


HISTORY OF MODERN INDIA

Important Tips Important Tips


Bhagat Singh: Sir Andrew Fraser
• Bhagat Singh was born in the Rurhiwal village of • Born on 14 November 1848, Andrew Henderson Fraser
Amritsar district. was educated at Edinburgh Academy.
• In 1923, Bhagat Singh joined the National College, • He became a Barrister-at-Law from Middle Temple
Lahore which was founded and managed by Lala and later joined the Indian Civil Service in 1871.
Lajpat Rai and Bhai Parmanand. • Before he joined as Lieutenant Governor of Bengal,
• In 1924 in Kanpur, he became a member of the Fraser held the position of Chief Commissioner of
Hindustan Republican Association (HRA), started Central Provinces (1899) and President of Police
by Sachindranath Sanyal a year earlier. The main Commission (1902).
organiser of the Association was Chandra Shekhar • After the partition of Bengal, he continued to remain
Azad and Bhagat Singh became very close to him. Lieutenant Governor of the other part of Bengal, what
• In 1925, he and his colleagues started a militant youth came to be known as Western Bengal (with Orissa and
organisation called the Naujawan Bharat Sabha. Bihar)
• To take revenge for the death of Lala Lajpat Rai, Bhagat • The Indian Police Commission of 1902-03, also
Singh and his associates plotted the assassination of known as the Frazer Commission, was established by
James A. Scott, the Superintendent of Police. However, the British government in India to reform the Indian
the revolutionaries mistakenly killed J.P. Saunders. police system.
The incident is famously known as Lahore Conspiracy  It was headed by Sir Andrew Frazer and Lord
case (1929). Curzon, and its main goal was to improve the
• His famous writings were - Why I Am an Atheist: An efficiency and effectiveness of the police force in
Autobiographical Discourse and The Jail Notebook India.
and Other Writings.  One of its key recommendations was the
• He was re-arrested for the murder of J.P. Saunders establishment of a centralized police intelligence
and bomb manufacturing in the Lahore Conspiracy system, which was considered to be a major success
case. He was found guilty in this case and was hanged in improving police intelligence gathering and
on 23 rd March, 1931 in Lahore along with Sukhdev and analysis.
Rajguru.  The Commission also recommended the separation
of the executive and judicial functions of the police,
64. Solution: (a) and the creation of a police training school.
 Overall, the Frazer Commission’s recommendations
Exp) Option a is the correct answer. had a significant impact on the Indian police
Shyamji Krishna Verma founded India House in London system and many of its recommendations were
implemented.
in 1905. India House was a hostel and meeting place for
Indian students and political activists in London. It was a 66. Solution: (c)
major center of Indian nationalist activity, and it played a Exp) Option c is the correct answer.
significant role in the Indian independence movement. The British journalist H.W. Nevinson was associated with
Swadeshi Movement. The Swadeshi Movement, now known
65. Solution: (a) as the ‘Make in India’ campaign was officially proclaimed
on 7 August 1905 at the Calcutta Town Hall, in Bengal.
Exp) Option a is the correct answer. The Boycott movement was also launched along with
Sir Andrew Fraser was the Lieutenant Governor of Bengal the Swadeshi movement. The movements included using
goods produced in India and burning British-made goods.
from 1903 to 1908. It is said that the idea of partitioning
Bal Gangadhar Tilak encouraged Swadeshi and Boycott
Bengal into two provinces in order to improve administrative movement after the British government decided on the
efficiency came actually from Andrew Fraser. Scholars also partition of Bengal.

believe that the partition of Bengal (1905) was not really 67. Solution: (b)
Curzon’s idea. However, official documents do indicate that Exp) Option b is the correct answer.

Curzon had great reliance on Fraser who had a number of The Jallianwala Bagh massacre took place on 13 April 1919.
Mahatma Gandhi was entrusted with the task of drafting
correspondences with him regarding the partition of Bengal Congress Inquiry Committee report on the Jallianwala Bagh
for the sake of better governance. Massacre.

PYQ Workbook 114


HISTORY OF MODERN INDIA

68. Solution: (d)


Political Contributions:
Exp) Option d is the correct answer. • Accompanied Gandhiji to second Round Table
A. Abul Kalam Azad - 4. Ramgarh, 1940. Conference (1931).
Maulana Azad reflected on the course of the freedom struggle • Became Governor of Uttar Pradesh post-independence.
after the end of the Second World War in the INC meeting. • Poetry and Literary Works:
B. Sarojini Naidu - 3. Kanpur, 1925. • Renowned poetess in English and Urdu.
The speech she delivered in the 1925 INC session sparked a • Famous works include “In the Bazaars of Hyderabad”,
sense of patriotism among those who attended the session, “The Golden Threshold”, “The Bird of Time”, and “The
which was honored by prominent politicians such as Motilal Broken Wing”.
Nehru, Madan Mohan Malaviya, and M.R. Jayakar. Abdullah Women’s Empowerment:
Abdur Rehman, the leader of the Indian community in • Advocate for women’s rights and empowerment.
South Africa, also participated in the session with the aim of
• Member of All-India Women’s Conference.
highlighting the plight of Indians living abroad
C. Motilal Nehru - 1. Amritsar, 1919. 69. Solution: (a)
Jallianwala Bagh Massacre was condemned strongly in 1919 Exp) Option a is the correct answer.
INC session. The Jallianwala Bagh massacre was a horrific The Hussainiwala National Martyrs’Memorial in memory
event that took place on April 13, 1919, in Amritsar, Punjab. of Indian freedom fighters Shaheed Bhagat Singh, Sukhdev
The crowd had gathered at the bagh to protest the arrests and Rajguru is located near the village of Hussainiwal
of two respected freedom struggle leaders, Dr. Saifuddin in Ferozepur district of Punjab near the India-Pakistan
Kitchlew and Dr. Satyapal. British troops on the order of international border. This monument shows the place on the
General Dyer opened fire on a peaceful gathering of unarmed banks of the river Sutlej where Bhagat Singh, Sukhdev and
Indians, killing hundreds and injuring thousands. Rajguru were cremated on 23-03-1931.
D. Dr. Rajendra Prasad - 2. Bombay, 1934
70. Solution: (c)
Jawaharlal Nehru presented the resolution advocating the
establishment of a Constituent Assembly tasked with crafting Exp) Option c is the correct answer.
an appropriate Constitution for an independent India in • Bal Gangadhar Tilak earned the title of ‘Lokamanya’
1934 session. Furthermore, during this session, decisions during the Home Rule Movement.
were taken to reshape the structure of the Congress. The • Bal Gangadhar Tilak earned this title because of his
number of Congress delegates was limited to 2000, effectively leadership and contribution to India’s freedom struggle.
reducing the strength of the All India Congress Committee
“Lokamanya” mean “accepted by the people” in Marathi.
(AICC) by half.
Important Tips
Important Tips
Books written by Bal Gangadhar Tilaka are:
About Sarojini Naidu:
• The Orion or Researches into the Antiquity of the
• Known as Bharatiya Kokila (The Nightingale of India). Vedas (1893)
• Indian independence activist, poet, and politician. • Arctic Home In The Vedas
• Born on February 13, 1879, in Hyderabad, India. • Gita Rahasya
• Joined Indian national movement due to Bengal • Vedic Chronology And Vedanga Jyotisha
partition in 1905.
• Received ‘Kaisar-i-Hind’ Medal for service during 71. Solution: (a)
plague epidemic.
Exp) Option a is the correct answer.
Contribution to Indian Freedom Movement:
Chittaranjan Das appeared as a lawyer for Aurobindo
• First Indian Woman President of INC in 1925 (Kanpur
Ghosh in the Alipore conspiracy case. Chittaranjan Das is
Session).
popularly called ‘Deshbandhu’. The Alipore Bombay Case
• Participated in Non-Cooperation Movement (1920).
Trial refers to an attempt to murder the district judge of
• Led Salt Satyagraha in 1930 against British salt Muzaffarpur. The Alipore Bomb case conspiracy took place
monopoly.
in the year 1908. It is also known as the Manicktolla bomb
• Arrested during “Quit India” movement in 1942, jailed conspiracy or Muraripukar conspiracy. The case saw the
for 21 months.
trial of a number of Indian nationalists of the Anushilan
• Traveled globally to raise awareness about India’s Samiti in Calcutta, under charges of “Waging war against the
struggle. Government” of the British Raj.

115 PYQ Workbook


HISTORY OF MODERN INDIA

Important Tips C. Sarojini Naidu - 2. Kanpur

Aurobindo Ghosh: D. Subhash Chandra Bose - 1. Haripura

• Aurobindo Ghosh, born on 15 August 1872 in 74. Solution: (c)


Calcutta district (present-day Kolkata), West Exp) Option c is the correct answer.
Bengal, was an Indian nationalist, poet, philosopher,
Mahatma Gandhi presided over only one session of the
and yogi.
Indian National Congress in 1924. The session was held
• He played an active role in the Indian Freedom in Belgaum, Karnataka. During his presidential address,
Movement till 1910, after which he became a Mahatma Gandhi discussed the importance of unity between
spiritual reformer. Hindus and Muslims, the boycott of foreign products,
• Ravindra Nath Tagore painted him as the Messiah the promotion of spinning and weaving Khadi, efforts to
of Indian Culture and Civilization and C R Das eliminate untouchability, and various other matters.
hailed him as the ‘poet of patriotism, the prophet
of nationalism and the lover of humanity.’ 75. Solution: (b)
• Aurobindo associated himself with journals and Exp) Option b is the correct answer.
periodicals like the Jugantar, Bande Mataram and Mitra Mela was a revolutionary organisation founded by
the Karmayogi, through which he criticised British Vinayak Damodar Savarkar along with his brother Ganesh
imperialism and preached a revolutionary concept Damodar Savarkar. They founded the Mitra Mela, a secret
of nationalism. organization of revolutionaries, in Nashik in 1899. In 1904,
• In 1908, he was arrested on the charge of the Alipore it was renamed AbhinavBharat Society.
Bomb Conspiracy Case and was acquitted in 1909
76. Solution: (a)
after a long trial.
Exp) Option a is the correct answer.
• Some of his important writings are The Life Divine,
Savitri, Essay on the Gita, Defense of Indian Surendranath Banerjee and Krishna Kumar Mitra led the
Culture, etc. agitation against the Bengal partition in 1905. The decision
to partition the state of Bengal was announced by Lord
• He passed away on 5 December 1950.
Curzon, the then Viceroy of India in July 1905.
Chittaranjan Das:
• He believed in extremist ideology and supported 77. Solution: (b)
the beliefs of the Lal-Bal-Pal trio. Exp) Option b is the correct answer.
• He also contributed to the English weekly ‘Bande Krishna Kumar Mitra was the first to suggest the boycott
Mataram’ along with Aurobindo Ghosh. of British goods in Bengal. The Swades Movement had its
• He championed the cause of Khadi and cottage genesis in the anti-partition movement which was started to
industries and gave up his own western clothes and oppose the British decision to partition Bengal.
luxurious lifestyle. Important Tips
• He, along with Motilal Nehru, established the Krishna Kumar Mitra:
Swaraj Party in January 1923. • Krishna Kumar Mitra (1852-1936) was one of the
• He was a prolific writer and poet. He published leaders of the Brahma Samaj and the swadeshi
his collection of poems in two volumes titled movement,
‘Malancha’ and ‘Mala’. • In 1883, Krishna Kumar launched a nationalist Bengali
weekly, Savjibani, in cooperation with Kalishankar and
72. Solution: (a) others. As editor, he endeavoured to raise the latent
Exp) Option a is the correct answer. nationalistic consciousness of the people
A provisional government of India was established in • He spread the Swadeshi idea through his paper,
Kabul, Afghanistan in December 1915 by a group of ‘Sanjeevani’. In one of his issues, he suggested that
Indian revolutionaries, led by Raja Mahendra Pratap. The people should pressurise the British government to
government was intended to be a government-in-exile that repeal the partition by boycotting British goods.
would overthrow British rule in India. The provisional
government was not recognized by any other countries, but 78. Solution: (d)
it did receive some support from Germany and Turkey. Exp) Option d is the correct answer.
73. Solution: (c) Swadeshi movement started on 7 August 1905. It started
with the partition of Bengal by the Viceroy of India, Lord
Exp) Option c is the correct answer.
Curzon in 1905 and continued up to 1911. It was the most
A. Dr. M.A. Ansari - 3. Madras successful of the pre-Gandhian movement. It was led by
B. Purushottam Das Tandon - 4. Nasik Syed Haider Raja in Delhi, Chidambaram Pillai in Madras,

PYQ Workbook 116


HISTORY OF MODERN INDIA

Lala lajpat Rai and Ajit Singh in Punjab and Uttar Pradesh, figurehead of the Yugantar party, the primary organization
Lokmanya Tilak in Mumbai and Pune. of Bengal’s revolutionary independence activists. In 1903,
Jatin met Aurobindo Ghose and highly inspired by his
79. Solution: (c)
teachings. He accepted to join a project of organizing secret
Exp) Option c is the correct answer. society aimed at an armed uprising all over India. Bhagha
Dadabhai Naoroji, positioned between moderates and Jatin assumed leadership of Yugantar party in 1908.
extremists, supported swadeshi in a distinct manner.
Important Tips
While not as radical as Tilak, Naoroji endorsed swaraj and
swadeshi in his 1906 Calcutta Congress speech. He shared Jugantar party:
the end goal with extremists but differed on tactics. Although • The Jugantar party was established by Barindra
criticized by both groups, his support for swaraj and the Kumar Ghosh (Younger brother of Aurobindo Ghosh),
swadeshi movement aligned with extremist sentiments. His Bhupendranath Datta, Raja Subodh Mallik in April
endorsement spurred them to pursue evolving ideas beyond 1906. Barindra Kumar Ghosh and Bagha Jatin were the
moderate strategies. main leaders.
Important Tips • Yugantar was one of the two major secret revolutionary
Leaders stand on swadeshi movement: movements of Anushilan Samiti fighting for Indian
independence in Bengal.
• While Aurobindo initially endorsed passive
resistance during the Swadeshi movement, he • During the First World War, the Jugantar party arranged
believed in the ethical validity of armed struggle to import German arms and ammunition through
if circumstances allowed. He criticized labeling all sympathizers and revolutionaries abroad. Bagha Jatin
violent methods as criminal, advocating strategic asked Rashbehari Bose to take charge of Upper India,
choices. His focus was on achieving national aiming to bring about an all-India insurrection in
emancipation above all else, considering liberty what has come to be called the ‘German Plot’ or the
paramount. Aurobindo’s approach blended both ‘Zimmerman Plan’.
violence and non-violence as methods for freedom,
distancing himself from being an active proponent of 81. Solution: (d)
the Swadeshi movement’s principles. Exp) Option d is the correct answer.
• Pherozeshah Mehta, despite being associated Bal Gangadhar Tilak was never elected president of the
with the Liberal School of Indian politics, had Indian National Congress. He was nominated for the
a more cautious approach towards the Swadeshi
presidency several times, but he was always defeated by more
movement. While he supported modernization and
moderate leaders. This was because Tilak was seen as too
education, Mehta was not as fervently aligned with
radical by many Congress members.
the radical aspects of the movement. His focus was
on constitutional means and administrative reform, Important Tips
rather than the more assertive tactics like strikes and
Significant Presidents of INC:
boycotts. Mehta’s aversion to violent methods and his
desire for gradual, constructive change set him apart • Lala Lajpat Rai was elected president of the Special
from the more extremist proponents of Swadeshi Session of the Indian National Congress at Kolkata in
during his time. 1920.
• Subhas Chandra Bose, while appreciating the • Annie Besant was elected president of the Indian
Swadeshi Movement’s economic objectives, believed National Congress in 1917 and 1933.
in a more assertive and proactive approach. He • Moti Lal Nehru was elected president of the Indian
advocated for a direct confrontation with British National Congress in 1929.
colonial rule through radical means, emphasizing
on aggressive mass mobilization, militant actions, 82. Solution: (d)
and organized resistance. Bose aimed for a swift and Exp) Option d is the correct answer.
forceful expulsion of the British rather than gradual
The Haripura Session of the Indian National Congress was
economic boycotts, emphasizing the need for more
presided over by Subhash Chandra Bose in 1938. National
active and assertive strategies to achieve India’s
independence. Planning Committee was set up under the chairmanship of
Jawaharlal Nehru.

80. Solution: (a) 83. Solution: (a)


Exp) Option a is the correct answer. Exp) Option a is the correct answer.
Jatin Mukherjee, better known as Bagha Jatin, was an Indian Anushilan Samiti (Calcutta) was founded on 24 March 1902.
independence activist who was born in 1879 in the division Pramathanath Mitra, known widely as P. Mitra, was a Bengali
of Bengal’s undivided Nadia district. He was the main Indian barrister and Indian nationalist who was among the

117 PYQ Workbook


HISTORY OF MODERN INDIA

earliest founding members of the Indian revolutionary • After hearing Dayanand’s views on Vedic religion,
organisation Anushilan Samiti. The people associated with Shraddhanand’s mind completely changed. He took up
this samiti were Sri Aurobindo Ghosh, Chittaranjan Das,
the mission to propagate the principles of Arya Samaj
Surendranath Tagore, Jatindranath Banerjee, Bagha Jatin,
and Vedic ideas among the people.
Bhupendra Natha Datta, Barindra Kumar Ghosh etc.
• He also raised his voice for Hindu-Muslim unity.
84. Solution: (c)
Exp) Option c is the correct answer. 86. Solution: (c)
In 1915, Rash Behari Bose fled from India to avoid arrest Exp) Option c is the correct answer.
by the British government. He went to Japan and then The organization formed by Surendranath Banerjee that
to Germany. In Germany, he established the Indian merged with the Indian National Congress in 1886 was
Independence League in 1915. The league was committed the Indian Association. The Indian Association played a
to promoting Indian independence through political and significant role in the early Indian nationalist movement,
diplomatic means. The Indian Independence League was advocating for Indian political rights. It initiated the idea
active in Europe and Asia during World War I. The league of an all-India conference, leading to the Indian National
also had branches in India. Conference, and eventually merged with the Indian
National Congress in 1886 to work collectively for Indian
Important Tips
interests.
• Captain Mohan Singh, an Indian soldier who fought
for the British, was captured and held as a prisoner 87. Solution: (d)
of war by the Japanese forces. While in captivity, a Exp) Option d is the correct answer.
Japanese army officer persuaded him to collaborate
Bal Gangadhar Tilak, a prominent figure in India’s
with them for the cause of India’s independence. As
freedom struggle, transformed the traditional Ganapati
a result, he was appointed as the leader of the Indian
festival into a national event with political significance.
National Army.
Recognizing the power of unity, Tilak initiated public
• Subhash Chandra Bose joined the ranks of the Indian celebrations in 1893 to foster a sense of togetherness. He
National Army in 1943. One of his most renowned introduced large hoardings and immersion of massive
pronouncements was, “Tum Mujhe khun do, main Ganesha statues on the tenth day, emphasizing collective
tumhe Azadi doonga” (You give me blood, and I will participation. This move played a crucial role in channeling
give you freedom). Under Subhash Bose’s guidance, the festival’s spirit towards India’s independence movement.
the INA played a pivotal role in the struggle for
independence 88. Solution: (d)
Exp) Option d is the correct answer.
85. Solution: (d)
Shyamji Krishna Verma was an Indian freedom fighter and
Exp) Option d is the correct answer. social reformer who founded the Indian Home Rule Society
Swami Shraddhanand was a great Arya Samajist, social in London in 1905. The society was a radical organization
worker, educationalist, and freedom fighter of the country. that advocated for complete independence from British rule.
He started working with Congress for the liberation of
Bharat Mata. He strongly condemned the Jallianwala Bagh 89. Solution: (a)
Massacre of 13 April 1919 and joined the nationwide protest Exp) Option a is the correct answer.
against the Rowlatt Act in Delhi. He also suggested launching Abanindranath Tagore established the Indian society of
a no tax campaign as a protest against the Rowlatt Act. Oriental art along with his brother Gagendranath Tagore in
Shraddhanand led a procession at Chandni Chowk, Delhi,
1907 in Calcutta to revive the ancient art traditions of India.
against the oppressive Rowlatt Act.
This Indian society of Oriental art sponsored by Europeans,
Important Tips much popularized Tagore’s Bengal School.

Swami Shraddhanand Important Tips


• He was born on 22 February 1856 at the village Talwan, Abanindranath Tagore (1871 -1951)
district Jalandhar of Punjab. • He was the gifted landscape and portrait painter.
• Swami Shraddhanand’s childhood name was Brihaspati • His first painting was Radha Krishna series and his set
but his father Lala Nanak Chand always called him of paintings called Krishna Lila is the synthesis of the
Munshi Ram. Indian and European style.
• He began his career as a lawyer and then practiced at • His Arabian Nights series is among his most renowned
Phillaur and Jalandhar. work.
• He met Swami Dayanand in September 1879 at Bareilly • Tagore best-known painting is Bharat Mata which
where he visited the place for the promotion of Vedic depicted Young women portrayed with 4arms in a
religion. manner of Hindu tidy holding objects.

PYQ Workbook 118


HISTORY OF MODERN INDIA

• He was the principal of government school of art and a 92. Solution: (d)
great artist of modern India. Exp) Option d is the correct answer.
Shiv Verma was the member of Hindustan Socialist
90. Solution: (c) Republican Association (HSRA), which was founded in 1928.
Exp) Option c is the correct answer. He was not the member of HRA which was formed in 1924.
Ram Prasad Bismil was the founding member of HRA.In
Shankaran Nair became a member of the Viceroy’s Executive
1924 – 25, many young people joined the HRA including
Council in 1915. Jalianwala Bagh Massacre took place on
Bhagat Singh, Sukhdev and Chandrasekhar Azad.
April 13,1919. About 20,000 protestors had assembled at
bagh to demonstrate against the Rowlatt act. General Dyer 93. Solution: (d)
open fired at peaceful demonstrators. About 379 people
Exp) Option d is the correct answer.
were killed and more than 1000 were injured. Following the
Jalianwala Bagh Massacre Shankaran Nair resigned from In 1916, Bal Gangadhar Tilak voiced the phrase “Freedom
Viceroy’s Council in 1919 as a protest. is my birthright and I shall obtain it!” in Lucknow. During
the same year and place, he also established the Home Rule
91. Solution: (b) League in April 1916.
Exp) Option b is the correct answer.
94. Solution: (c)
A. British Indian 3. Radha Kant Dev
Exp) Option c is the correct answer.
Association
M.K Gandhi- He presided over the Belgaum Session of the
B. Bombay Presidency 2. K.T. Telang
Indian National Congress in 1924.
Association
Smt. Sarojini Naidu- She presided over the Kanpur Session
C. Central Mohammadan 4. Sayyid Amir Ali of the Indian National Congress in 1925.
National Association
Jawahar Lal Nehru- He presided over the Lahore Session
D. Servants of India Society 1. Gopal Krishna Gokhale (1929), Lucknow Session (1937), Faizpur Session (1936),
Delhi Session (1951), Hyderabad Session (1953), and
Important Tips
Kalyani Session (1954).
British Indian Association:
Vallabh Bhai Patel- He presided over the Karachi Session of
• The British Indian Association, established in the Indian National Congress in 1931.
1851 in Kolkata, India, was a political organization
advocating for Indian welfare and rights. Radhakanta 95. Solution: (b)
Deb served as its first President, and it included Exp) Option b is the correct answer.
members like Peary Chand Mitra. The association
Option 1 is correct: Abhinav Bharat society was founded
demanded Indian education, end to East India
by Vinayak Damodar Savarkar in 1904. It was engaged
Company’s monopoly, support for local industry, and
Indian civil service inclusion. Its demands influenced in revolutionary activities. It was one among several such
the Indian National Congress. The association ceased revolutionary societies functioning in Maharashtra at that
in 1954 with the end of the Zamindari system. time, which believed in the overthrow of British rule through
armed rebellion. Vinayak Savarkar and Ganesh Savarkar
Central National Muhamedan Association:
founded the revolutionary secret society Mitra Mela in
• The Central National Muhamedan Association, Nasik in 1899. Vinayak Savarkar renamed it Abhinav Bharat
founded by Syed Ameer Ali in Calcutta in 1877, in 1904.
aimed to uplift Muslims through lawful means,
Option 2 is correct: Anushilan Samiti was formed by Satish
promoting education, moral values, and political
consciousness. It countered Hindu-dominated Chandra Basu in 1902.He named it after Bankimchandra’s
organizations, seeking Muslim welfare and loyalty to play Anushilan-Tattva or theory of discipline and
the British government. With branches across India Pramathanath Mitra was its main patron. The society’s goal
and London, it played a role in modernizing Muslims was to organise and train the youth to start a revolutionary
and addressing their issues. war against the British.
Servants of India Society: Option 3 is incorrect: New Nationalist Party was not
any such famous organisation which was engaged in
• The Servants of India Society, founded in Pune,
revolutionary activities.
Maharashtra, in 1905 by Gopal Krishna Gokhale,
aimed to promote social and human development Option 4 is incorrect: Indian Patriotic Association was
while opposing British rule. It focused on formed in 1888 by Sir Syed Ahmed Khan and Raja Shiv Prasad
education, health care, sanitation, and social issues Singh of Benaras. The main objective of this association was
like untouchability and women’s rights. Remaining to create a platform for the Indian Muslims to express their
apolitical, it declined after Gokhale’s death, with loyalty towards the British Raj and to promote the interest
Gandhi’s movement gaining prominence. It continues of the Indian Muslims. It was not engaged in revolutionary
with limited activities today. activities against British Raj.

119 PYQ Workbook


HISTORY OF MODERN INDIA

96. Solution: (c)


Important Tips
Exp) Option c is the correct answer. Ramabai Ambedkar:
Madan Lal Dhingra was a revolutionary who assassinated Ramabai Bhimrao Ambedkar (7 February 1898 – 27 May
Sir Curzon Wyllie, a British political figure, in London in 1935) was the first spouse of B. R. Ambedkar, whose
1909. He was sentenced to death and hanged. unwavering support played a pivotal role in enabling
him to pursue advanced education and realize his full
Udham Singh was a revolutionary who assassinated Michael
potential. B. R. Ambedkar dedicated his 1941 publication,
O’Dwyer, a British official who was responsible for the
‘Thoughts on Pakistan,’ to Ramabai. In the preface,
Jallianwala Bagh massacre, in London in 1940. He was also Ambedkar attributes to her his evolution from an ordinary
sentenced to death and hanged. Bhiva or Bhima to Dr. Ambedkar.
Important Tips Pandita Rama Bai:
• Rajguru and Sukhdev were revolutionary associated Pandita Ramabai was born into an upper-caste Hindu
with Bhagat Singh. They were hanged in India for household, where her father worked as a skilled narrator
their involvement in the murder of John Saunders, a of Hindu epics and mythical tales. She wrote books
British police officer. such as The High Caste Hindu Woman, published
• Khudiram Bose and Surya Sen were revolutionary in1887. A staunch promoter of women’s education and
associated with the Anushilan Samiti. They empowerment, she went on to establish the Arya Mahila
were hanged in India for their involvement in the Samaj, an institution committed to enhancing the status
Muzaffarpur Conspiracy Case. of women in India. Ramabai’s deep understanding and
analyses of diverse Sanskrit texts led to her being honored
• Kartar Singh Sarabha and Ashfaqullah Khan were
with the prestigious epithet ‘Saraswati,’ underscoring her
revolutionary associated with the Ghadar Party. They
exceptional intellectual capabilities and mastery in the
were hanged in India for their involvement in the
field.
Lahore Conspiracy Case.
Annie Besant:
Dr. Annie Besant took the initiative to establish two
97. Solution: (d)
newspapers, namely Commonweal and New India. In the
Exp) Option d is the correct answer. era of the home rule movement, she founded the National
Morley-Minto reforms introduced Reservation of seats and College in Madanapalle, with J.H. Cousins serving as the
separate electorates for the Muslims. Some constituencies principal in 1916. This institution is now recognized as
Besant Theosophical College. Notably, she attained the
were earmarked for Muslims and only Muslims could vote
distinction of becoming the first female president of the
their representatives. This event sow the seeds of discard
Indian National Congress in 1917. Her contributions
which eventually caused to divide the country extended to the founding of the Central Hindu College in
Benaras, which was later elevated to the status of Benaras
98. Solution: (d)
Hindu University by Pandit Madan Mohan Malaviya in
Exp) Option d is the correct answer. 1916.
Bhikaiji Rustom Cama, acclaimed as the Mother of the Indian Sarojini Naidu:
Revolution, played a pivotal role in the overseas Indian A trailblazer, she holds the honor of being the first Indian
freedom struggle. On August 22, 1907, she accomplished a woman to preside over the Indian National Congress. Her
historic feat by being the first to unfurl the Indian flag on elevation to this position transpired during the Kanpur
foreign soil in Stuttgart, Germany. Her powerful advocacy session in 1925. Acknowledged for her remarkable
for human rights, equality, and self-governance from prowess in poetry, she earned the title “Nightingale of
India” due to her substantial contributions in the realm
British rule resonated through her compelling descriptions
of poetry. She was often referred to as ‘Bharat Kokila’.
of the devastation wrought by a famine in the Indian
Pioneering yet again, she became the inaugural woman
subcontinent. She eloquently proclaimed, ‘This is the flag to assume the role of governor in the dominion of India,
of independent India. I appeal to all gentlemen to stand specifically taking up the position of governor of the
and salute the Flag.’ Collaborating with Shyamji Krishna United Provinces in 1947
Varma, she designed the altered ‘Calcutta Flag,’ which she
raised. Additionally, from Paris, she authored, published, 99. Solution: (d)
and disseminated revolutionary materials. When the British Exp) Option d is the correct answer.
proscribed ‘Vande Mataram,’ she responded by composing On 7th February 1856, Lord Dalhousie ordered to depose
‘Bande Mataram.’ Her unyielding spirit extended to creating Wajid Ali Shah on the account of alleged internal misrule.
‘Madan’s Talwar’ in response to the execution of Madan Lal Dar-o-Deewar par Hasrat Ki Nazar Karte Hain,Khush Raho
Dhingra. Ahle Watan hum to safar karte hein” was composed when

PYQ Workbook 120


HISTORY OF MODERN INDIA

Wajid Ali Shah was forced to leave Oudh. The mind of Wajid 103. Solution: (b)
Ali Shah was full of sorrow but still he felt that his country Exp) Option b is the correct answer.
and countrymen should be well and happy. Wajid Ali Shah
The massacre of the Jallianwala Bagh event was, characterized
himself composed Ghazals under the pen name of ‘Akbtar
by Montagu as ‘Preventive Murder’. According to him, it was
Piya’.
to save the public interest.
100. Solution: (d)
Important Tips
Exp) Option d is the correct answer.
After the Incident of Jallianwala Bagh:
People were gathered at Jallianwala Bagh in Amritsar to
• The Bengali poet and Nobel laureate Rabindranath
protest against the arrest of Saifuddin Kitchlew and Satyapal
Tagore renounced the knighthood that he had received
on 13th April 1919. The Jallianwala Bagh massacre or the
in 1915.
Amritsar Massacre took place when many villagers gathered
• Gandhi soon began organizing his first large-scale and
in the park for the celebration of Baisakhi. The gatherers
sustained nonviolent protest (satyagraha) campaign,
wanted to also peacefully protest the arrest and deportation
the Non-Cooperation Movement (1920–22).
of two national leaders, Satyapal and Saifuddin Kitchlew.
Important Tips 104. Solution: (d)
Consequence of Jallianwala Bagh: Exp) Option d is the correct answer.
• On october 14, 1919, the Government of India During the second session of the National Conference
announced the formation of the Disorders Inquiry in 1885, Surendranath Bannerjee was in Calcutta. The
Committee, famously known as Hunter Committee. inaugural session of the Indian National Congress, took place
• The purpose of the commission was to investigate the in Bombay on December 28th, 1885, immediately following
disturbance in Bombay, Delhi and Punjab. the conclusion of the National Conference. Consequently, he
was unable to attend the inaugural session in Bombay.
101. Solution: (b) Important Tips
Exp) Option b is the correct answer. About Surendranath Banerjee:
Lord Curzon was the Viceroy of India from 1899 to 1905. Basic Information:
He was a critic of the Indian National Congress and he
• Surendranath Banerjee, known as Rashtraguru
believed that it was a threat to British rule in India. He
made the statement about the Congress- “The Congress is • Founded Indian National Association in 1876 for
faltering to its fall and one of my great ambitions while political reforms
in India is to assist it to a peaceful demise.” Curzon’s • Established “The Bengalee” newspaper
statement was made in a private letter to his friend, Lord • Passed Indian civil service exams in 1869 and 1871-
George Hamilton, the Secretary of State for India. The Disqualified in 1869 due to age dispute- Dismissed in
letter was leaked to the press, and it caused a great deal of 1871 due to racial discrimination.
anger and resentment among Indian nationalists. Curzon’s • Advocated for civil disobedience movement
statement was also a reflection of the British government’s
Background:
attitude towards the Indian National Congress at the time.
• Born on November 10, 1848, in Calcutta
The British government was concerned about the growing
popularity of the Congress, and it was trying to find ways to • Graduated from Calcutta University in 1868
undermine its influence. • Attempted Indian Civil Services competition in
England
102. Solution: (b)
• Became a Professor of English in 1875
Exp) Option b is the correct answer.
Contributions & Achievements:
The correct order is Permanent Settlement- Subsidiary
• Inspired Indian students, eloquent speaker
alliance - Annexation Policy - Partition of Bengal.
• Founded Indian Association on July 26, 1876,
• The Partition of Bengal was a territorial reorganization
promoting political unity
of the Bengal Presidency implemented by the authorities
of the British Raj in 1905. • Organized National Conference sessions, emphasizing
unity
• Annexation Policy was introduced by Lord Dalhousie
that remained effective from 1848 - 56 • First Indian National Congress session held in Bombay
in 1885
• Lord Wellesley introduced Subsidiary alliance from 1798
• Played leading role, President of Congress in 1895 and
- 1805
1902
• Lord Cornwallis introduced Permanent Settlement in
• Peak in political career in 1906, followed by decline
1793

121 PYQ Workbook


HISTORY OF MODERN INDIA

105. Solution: (c) • Tilak was ready to assume leadership after his release
Exp) Option c is the correct answer. in June 1914. He also admitted that the acts of violence
After the Kakori setback, the younger revolutionaries, had only served to retard the pace of political progress
inspired by socialist ideas, set out to reorganise Hindustan in India.
Republic Association (HRA) at a historic meeting in • Annie Besant, the Irish theosophist based in India
the ruins of Ferozshah Kotla in Delhi in September 1928. since 1896, had decided to enlarge the sphere of her
The name of HRA was changed to Hindustan Socialist activities to include the building of a movement for
Republican Association (HSRA) under the leadership of home rule on the lines of the Irish Home Rule Leagues.
Chandrashekhar Azad. The participants included Bhagat
Singh, Sukhdev Thapar, Bhagwati Charan Vohra, Bejoy 109. Solution: (a)
Kumar Sinha, Shiv Verma, and Jaidev Kapur The HSRA Exp) Option a is the correct answer.
decided to work under a collective leadership and adopted Lord Chelmsford was the Viceory of India when the
socialism as its official goal. Jallianwala Bagh massacre 1919 took place. Jallianwala Bagh
massacre took place in Amritsar when people congregated in
106. Solution: (c)
Jallianwala Bagh on the eve of Baisakhi day.
Exp) Option c is the correct answer.
Jatin Das arrested for his involvement in Lahore Conspiracy 110. Solution: (c)
case of 1929. Many young revolutionaries including Bhagat Exp) Option c is the correct answer.
Singh, Jatin Das, Raj guru and Sukh Dev were charged for Bal Gangadhar Tilak was famously called the ‘Father of
involvement in the murder of police officer J P Saunders in Indian Unrest’ by Valentine Chirol of The Times (London).
1929, which is famously known as Lahore Conspiracy Case. He was arrested for 18 months for writing in his newspaper
Jatin Das passed away after a sixty-three-day hunger strike criticising the measures adopted by the government to
while he was imprisoned in the Lahore jail. tackle the bubonic plague epidemic. This title was used to
characterize his role in the Indian freedom movement and
107. Solution: (c)
his efforts in advocating for self-governance and challenging
Exp) Option c is the correct answer. British rule through his nationalist activities and writings.
Udham Singh was a Indian revolutionary and freedom
fighter belonging to the Ghadar Party. He shot and killed 111. Solution: (d)
Michael O’ Dwyer on 13 March 1940 in Britain. Michael O’ Exp) Option d is the correct answer.
Dyer was Lieutenant General of Punjab when Amritsar’s On 8th April, 1929, Bhagat Singh and Batukeshwar Datta
Jallianwala Bagh Massacre happened on April 13, 1919. threw a bomb in the Central Legislative Assembly. The aim
was not to kill but “to make the deaf hear”, and to remind the
108. Solution: (d)
foreign government of its callous exploitation.
Exp) Option d is the correct answer.
The Home Rule Movement was the Indian response to the 112. Solution: (c)
First World War in a less charged but a more effective way. Exp) Option c is the correct answer.
Tilak started his efforts to spark a revolutionary shift by R. N. Tagore was critical of the developments during
establishing a Home rule league in Maharashtra (excluding the Swadeshi movement, considering them “xenophobic,
Bombay), Karnataka, and the central provinces in April chauvinistic” Around 1908 he wrote to friend Aurobindo
1916. Later that year, Annie Besant founded her Home Rule Mohan Bose, saying, “Patriotism cannot be our final
movement in September 1916. spiritual center, I will never allow patriotism to triumph over
Important Tips humanity.” Tagore’s 1925 essay, Cult of the Charkha, severely
criticised the Swadeshi movement and Gandhi’s idea of
Some of the factors leading to the formation of the
‘charkha spinning’ was a means to achieve independence. “A
Home Rule Movement were as follows:
nation,” he wrote during this period, “ is that aspect which a
• A section of the nationalists felt that popular whole population assumes when organized for a mechanical
pressure was required to attain concessions from the purpose”, a purpose often associated with a “selfishness” that
government. “can be a grandly magnified form” of personal selfishness.
• The Moderates were disillusioned with the Morley- Important Tips
Minto reforms.
Tagore’s View:
• People were feeling the burden of wartime miseries
• The Swadeshi movement, which gained momentum as
caused by high taxation and a rise in prices, and were
a reaction to Lord Curzon’s decision to divide Bengal
ready to participate in any aggressive movement of
along communal lines in 1905, saw Tagore as a leading
protest.
figure against the division.
• The world war I exposed the myth of white superiority.

PYQ Workbook 122


HISTORY OF MODERN INDIA

• In this phase of tumult, where Tagore envisioned Indian Association:


change would come with unity and education, he wrote • The Indian Association, founded in 1876 by
songs of patriotism such as Banglar Maati Banglar Jol Surendranath Banerjea and Ananda Mohan Bose,
(Earth of Bengal, Water of Bengal) that bolstered the emerged in Bengal as a nationalist political entity
movements and also epitomised Hindu-Muslim unity. advocating local self-governance. It replaced the
• During this time, the British faced pecuniary loss Indian League and competed with the British Indian
and Indian industries got a chance to flourish in an Association, mostly gaining support from Bengal’s
environment made favourable by nationalist zeal. younger intelligentsia. Expanding nationwide while
• Among those who benefitted are Prafulla Chandra retaining its Bengal identity, it found provincial
Ray’s Bengal Chemicals, J.N. Tata’s Iron and Steel branches through expatriate Bengali communities.
Company, the Bengal Luxmi Cotton Mills and the
114. Solution: (d)
Mohini Mill, etc.
Exp) Option d is the correct answer.
The correct match between List-I and List-II is:
113. Solution: (d)
A. Indian League - 1. Shishir Kumar Ghosh
Exp) Option d is the correct answer. B. Indian Association - 2. Anand Mohan Bose
C. India National Liberal Federation - 4. Surendranath
The East India Association was not based in Calcutta but Bannerjee
was a London-based organization. Similarly, the Indian D. United Indian Patriotic Association - 3. Syyad Ahmad
Khan
Association was not based in Bombay but was founded in
Important Tips
Bengal.
The Indian National Liberal Federation:
• The Indian National Liberal Federation (INLF),
Important Tips
founded in 1919, emerged from a schism within the
British India Society: Indian National Congress. Led by Surendra Nath
• The British India Society, established in 1839 in Banerjea and prominent leaders like Tej Bahadur
London, aimed to promote ethical practices in Sapru and V. S. Srinivasa Sastri, it aimed to engage
British India through collaboration between British in constitutional discussions with British officials.
and American abolitionists, East India Company The INLF played a role in advocating for India’s
members, and Bengal’s elite. Advocating against constitutional representation, leading to initiatives like
slavery, EIC misrule, and for private enterprise, it was the Nehru Report and participation in the Round Table
founded by individuals like William Adam, George Conferences.
Thompson, William Ednis, and Major General Briggs United Indian Patriotic Association:
after Adam’s visit to Raja Ram Mohan Roy in India.
• The United Indian Patriotic Association, founded in
East India Association: 1888 by Sir Syed Ahmad Khan and Raja Shiv Prasad
• The East India Association (EIA), founded by Singh of Benaras, aimed to strengthen Muslim-British
Dadabhai Naoroji in 1866, was a London-based ties and counter Indian National Congress influence.
organization engaging with Indian matters. It Encouraged by British officials, it opposed Congress
included Indian members and retired British officials. propaganda and promoted their interests.
A precursor to the Indian National Congress, the EIA
later merged with other groups, evolving into the 115. Solution: (c)
Britain, India, and Pakistan Association. It published Exp) Option c is the correct answer.
the Journal of the East India Association and the V. O. Chidambaram Pillai was popularly known as
Asiatic Quarterly Review. Kappalottiya Tamilan (The Tamil Helmsman) and
National Indian Association: Sekkizuththa Semmal (scholarly gentry who suffered at the
oil press). He entered politics in 1905 following the partition
• The National Indian Association (NIA), founded
of Bengal. Towards the end of 1905, he visited Madras and
in 1870 by Mary Carpenter with Keshub Chunder
was drawn closer to the Swadeshi Movement initiated by
Sen’s assistance, aimed to promote female education
Bal Gangadhar Tilak and Lala Lajpat Rai. He was leader of
and raise British awareness about Indian affairs.
Swadeshi movement in Madras.
Its London branch, established by Mrs. Manning and
Elizabeth Manning in 1871, later became headquarters. 116. Solution: (c)
The NIA facilitated social interaction between Indians Exp) Option c is the correct answer.
and the British, organizing events, lectures, and
Ashfaque-ullah Khan was the first recorded Muslim
publishing a journal. revolutionary who was hanged for Indian freedom. He was

123 PYQ Workbook


HISTORY OF MODERN INDIA

involved in the Kakori incident of 1925. He met Ram Prasad The Indian National Association, later known as the
Bismil in the year 1920, and their friendship that blossomed Indian Association of Calcutta, was established in 1876
ended only with their death in 1927. Ashfaqullah and by Surendranath Banerjee and Anand Mohan Bose. This
Bismil worked together in the Non-Cooperation Movement, nationalist organization aimed to promote legitimate means
campaigned for the Swaraj party, and carried out missions for India’s political, intellectual, and material advancement.
for the HRA, which was founded in 1924. He wrote several It was a significant pre-Congress organization, advocating
poems and couplets under the pen name of ‘hasrat’ and for public opinion on political matters and uniting Indians
‘warsi’. Along with his fellow comrades Ramprasad Bismil
behind a common political agenda. The association
and Thakur Roshan Singh, Ashfaqullah Khan was hanged on
eventually merged with the Indian National Congress in
19th December 1927 in Faizabad.
1886.
117. Solution: (a)
121. Solution: (c)
Exp) Option a is the correct answer.
Exp) Option c is the correct answer.
The correct sequence is 2- 1 - 3
Khudiram Bose, along with Prafulla Chaki, tried to
• On 10th April, 1919, two leaders of Punjab Dr. Saifuddin assassinate Kingsford in Muzaffarpur on the eve of 30 th April
Kitchlu and Dr. Satyapal were incarcerated (arrested) 1908 AD. Douglas H Kingsford was the Chief Presidency
and deported to an undisclosed place without any trial. It Magistrate of Calcutta at the time. He was a target of the
created a turmoil situation in Punjab.
revolutionaries as he was known for his harsh treatment and
• Jallianwala Bagh Massacre took place on 13 April 1919 vindictiveness towards the freedom fighters.
(on Baisakhi) at Jallianwala Bagh in Amritsar, Punjab
Important Tips
province. This gathering was organized to oppose the
“Rowlatt Act”. The British officer “General Dyer” opened Khudiram Bose:
fire on the crowd present in the meeting, in which more • Khudiram joined the Anushilan Samiti in 1908.
than 1000 people died as reported. Jallianwala Bagh • Khudiram Bose and Prafulla Kumar Chaki were
massacre happened during the tenure of governor- appointed to carry out the mission of assassinating
general Lord Chelmsford Kingsford. But they failed and mistakenly assassinated
• Indian National Congress’s Amritsar session (34th the wife and daughter of a barrister called Pringle
Session) was held from 27 December, 1919 till 1 January, Kennedy.
1920 in Amritsar, Punjab. The session was presided over • Young Khudiram, at the age of eighteen, was executed
by Pandit Motilal Nehru. The session took place in the on 11 August 1908, making him one of the youngest
aftermath of the Jallianwala Bagh massacre (April 1919), revolutionaries in India to be hanged by the British.
the air-bombing of Gujranwala and the imposition of • The Poet Pitambar Das made his sacrifice immortal
martial law in Punjab. in the popular Bengali song Ek Baar Bidaye De Ma, a
118. Solution: (b) song that resonates with the passion the young boy had
for his motherland.
Exp) Option b is the correct answer.
Bal Gangadhar Tilak accused the Indian National
122. Solution: (a)
Congress of practising ‘politics of prayer, petition and
protest’. He believed that this method was not effective in Exp) Option a is the correct answer.
achieving independence for India. He advocated for a more ‘The partition announcement fell like a bomb shell. We felt
militant approach, which included swadeshi (self-reliance) that we had been insulted, humiliated, and tricked’. This
and swaraj (self-rule). statement was said by Surendra Nath Banerjee.

119. Solution: (c) Important Tips


Exp) Option c is the correct answer. Surendranath Banerjee:
Curzon came up with the idea to suppress the rising tide • Surendranath Banerjee was the founder of the Indian
of nationalism in Bengal. According to him the Bengali Liberation Federation (1919) and Indian National
intelligentsia was primarily responsible for the spread of Association.
nationalism in different parts of India. Thus Curzon wanted • He was the co-founder of the Indian National Congress.
to ensure that the minority consciousness should be used • He was born on 10th November 1848 in Bengal and
against the majority and that this would then break the died on 6th August 1925 in Barrackpore Calcutta at the
solidarity of Indian society, and in turn, break the solidarity age of 76 years.
of the Indian National Movement as wells. The ‘Partition of
• He was influenced by the writings of Mazzini
Bengal’ was at the core of his policies.

120. Solution: (a) 123. Solution: (c)


Exp) Option a is the correct answer. Exp) Option c is the correct answer.

PYQ Workbook 124


HISTORY OF MODERN INDIA

To express the viewpoint on the necessary social issues 126. Solution: (a)
with due consent of the educated class, was not one of the Exp) Option a is the correct answer.
objectives of the Indian National Congress as pointed out by
V.D. Savarkar founded the ‘Abhinav Bharat,’ a covert
W.C. Banerjee in 1885. This is because the Indian National
association established by him and his sibling Ganesh
Congress was initially formed as a moderate organization
that aimed to work within the existing British system. Damodar Savarkar in 1904. Initially known as Mitra Mela
The moderates believed that it was important to win the and originating in Nasik, this society collaborated with
consent of the British government before making any major numerous revolutionaries and political advocates, extending
changes to the social and political structure of India. They its presence to multiple locations in India as well as London.
also believed that it was important to involve the educated Important Tips
class in the freedom struggle, as they were seen as the
V.D. Savarkar:
natural leaders of the Indian people. However, as the Indian
National Congress became more radical in the early 20th Born on May 28, 1883, in Bhagur, a village close to Nashik
century, it began to take a more active role in addressing in Maharashtra, V.D. Savarkar embarked on an influential
social issues. For example, the Congress played a major role journey. He established the covert Abhinav Bharat Society
in the passage of the Hindu Code Bill in 1956, which gave and ventured to the United Kingdom, affiliating with
women equal rights in marriage, divorce, and inheritance. groups like India House and the Free India Society. His
involvement extended to shaping the Hindu Mahasabha
Important Tips and holding its presidency from 1937 to 1943. Notably,
The objectives of the Indian National Congress as Savarkar authored ‘The History of the War of Indian
pointed out by W.C. Banerjee in 1885 were: Independence,’ which delved into the guerrilla tactics
• To promote contact and friendship among the people employed during the 1857 Sepoy Mutiny. He also penned
of all parts of India. ‘Hindutva: Who is Hindu,’ contributing to significant
• To promote the feeling of nationalism among the ideological discussions.
people of India.
• To remove all racial, religious and provincial prejudices 127. Solution: (a)
among the people of India. Exp) Option a is the correct answer.
• To present the views of the people of India to the The Kakori Train Action involved the robbery of the
government. Number 8 down-train travelling from Shahjahanpur to
• To work for the improvement of the social and Lucknow on 9 August, 1925. The idea was to attack the
economic conditions of the people of India. guard cabin that was carrying state money to be deposited
in Lucknow. A train was robbed at Kakori (near Lucknow)
by Bismil and other members of the Hindustan Socialist
124. Solution: (c)
Republican Army. The loot was to be used for purchasing
Exp) Option c is the correct answer.
weapons for fighting the British. However, all the key people
Rahimtulla Mahomed Sayani was the second Muslim involved in the robbery were arrested. Ramprasad Bismil,
president of the Indian National Congress. He was a Ashfaqulla Khan, Rajendranath Lahiri and Roshan Singh
Khoja Muslim who was born in Bombay in 1847. He was were sentenced to death.
a lawyer and a social reformer who was active in the Indian
independence movement. Sayani was elected president of 128. Solution: (b)
the Indian National Congress in 1896. He was a moderate Exp) Option b is the correct answer.
leader who believed in working within the British system to
Mahatma Gandhi opposed Rowlatt’s act via non-violent
achieve self-rule for India. He was also a strong advocate for
Satyagraha. He called a hartal on 6 April 1919. On 13th
social reform, and he worked to improve the condition of
April, Baisakhi day, a large crowd of people unaware of the
the poor and the marginalized.
prohibitory orders in the Amritsar gathered in the Jallianwala
125. Solution: (e) Bagh to Protest against the arrest of two congressmen Dr.
Exp) Option e is the correct answer. Saifuddin Kitchlu and Dr. Satyal by British Officials. The
troops surrounded the gathering under orders from General
Bismil Azimabadi is the author of the popular song,
Sarfaroshi Ki Tamanna Ab Hamare Dil Mein Hai. He was an Dyer and blocked the only exit point and opened fire on
Urdu poet from Patna, Bihar. Bismil wrote the Sarfaroshi ki the unarmed crowd killing more than 1000 unarmed men,
Tamanna after the Jallianwala Bagh Massacre. The poem was women, and children.
first recited in 1920 at the Congress session of Kolkata. 129. Solution: (c)
In 1921, it was published in the Urdu daily Sabah. It later
became a war cry for several other freedom fighters. In 1927, Exp) Option c is the correct answer.
Ram Prasad Bismil chanted this poem on the way to the Sachindra Nath Sanyal established a branch of Anushilan
gallows before being hanged by the British force. Hence, Samiti at Patna in 1913. Bankimchandra Mitra of BN College
none of the above is the answer. was given responsibilities to lead the organisation.

125 PYQ Workbook


HISTORY OF MODERN INDIA

Important Tips 132. Solution: (b)

Anushilan Samiti: Exp) Option b is the correct answer.


• The society’s goal was to organise and train the youth The protest method of the moderate leaders of the
to start a revolutionary war against the British. Congress was constitutional agitation. They believed
• It had two branches: the Jugantar Group in Kolkata that the best way to achieve self-rule for India was through
and the Dhaka Anushilan Samiti in Dhaka. persuasion and cooperation with the British. They used
• The Samiti published “Bhavani Mandir” in 1905, methods such as petitions, protests, and delegations to the
which included a thorough plan for constructing a British government to voice their demands.
religious sanctuary as the focal point of revolutionary
Important Tips
operations in a remote location.
The Moderates, in pursuit of their objectives, employed
• In order to instill the required revolutionary mindset
among Indians, Aurobindo Ghosh and Bipin Chandra a range of methods:
Pal launched the Bengali nationalist weekly “Jugantar” • Demand for Reforms and Critique: They articulated
(New Era) and its English counterpart “Bande demands for reform and critiqued government policies,
Mataram” in March 1906. Both publications openly aiming to improve Indian conditions.
preached armed revolt.
• Preference for Non-Violence: Instead of violence,
• Anushilan Samiti was a well-known covert they advocated patience and reconciliation, focusing
revolutionary organisation that operated out of Bengal on peaceful means to achieve their goals.
in the 20th century with the goal of removing colonial
authority and igniting India’s independence movement. • Constitutional and Peaceful Approaches: They relied
on constitutional methods and peaceful strategies,
seeking change through legal and established channels.
130. Solution: (a)
Exp) Option a is the correct answer. • Educational Initiatives: Their efforts centered on
educating and awakening public political awareness,
Chittaranjan Das was the President of the Gaya Session
fostering informed opinions.
of the Indian National Congress held in 1922. He was a
prominent Indian nationalist and lawyer who was a close • Creating British Public Opinion: To garner support
associate of Mahatma Gandhi. He was a strong advocate of in England, they organized lectures and published a
the Non-Cooperation Movement and he played a key role weekly journal named ‘India’ for British readership.
in its success. • Newspapers and Journals: They utilized newspapers
Important Tips like the Bengali press, Bombay Chronicle, Hindustan
Times, Induprakash, Rast Goftar, and the weekly
The Gaya Session of the Indian National Congress
journal ‘India’ to criticize government policies.
was a significant event in the history of the Indian
independence movement. It was at this session that the • Public Meetings and Discussions: They convened
Congress adopted the Swaraj Constitution, which was a meetings across locations like England, Mumbai,
blueprint for self-government in India. The session also Allahabad, Pune, and Calcutta, addressing social,
saw the Congress declare its policy of non-violence and economic, and cultural issues.
its commitment to Hindu-Muslim unity. • Memoranda and Petitions: The Moderates drafted
and submitted memoranda and petitions to both the
131. Solution: (b)
Indian and British officials, along with the British
Exp) Option b is the correct answer.
Parliament. These documents aimed to enlighten the
Swadeshi Movement was started in 1905 to oppose the British British public and leaders about Indian conditions,
decision of partitioning of Bengal. It involved boycotting fostering understanding and support for change.
British products and the revival of domestic products and
production processes. The objective was to exert sufficient
pressure on the Government to prevent the unjust partition 133. Solution: (d)
of Bengal from being implemented. But still the government Exp) Option d is the correct answer.
went on with the decision and announced the partition in
Lala Lajpat Rai said “Congress Movement was neither
July 1905.
inspired by the people, nor devised or planned by them”
Important Tips because he believed that the Congress Movement was
Events and their occurrence years: dominated by a small group of elite Indians who were out
• Civil Disobedience Movement - March 12, 1930. of touch with the needs of the masses. He also believed that
• Swadeshi Movement - August 7, 1905. the Congress Movement was too focused on achieving self-
rule through constitutional means and that it was not doing
• Quit India Movement - August 8, 1942.
enough to address the social and economic problems facing
• Non-Cooperation Movement - August 1, 1920.
India.

PYQ Workbook 126


HISTORY OF MODERN INDIA

134. Solution: (d) Most of the moderate leaders hailed from urban areas.
Exp) Option d is the correct answer. They were mostly educated in English and had a good
understanding of the British system. They believed that
The Doctrine of Passive Resistance proposed by Aurobindo
the best way to achieve self-rule for India was through
Ghose (1872- 1950). It was an alternative to expelling the
persuasion and cooperation with the British. They believed
colonial power from the country. He calls for organized
political resistance to achieve liberty for India. For this goal, that the British could be persuaded to give India self-rule if
he proposes the concept of “Passive Resistance”, with the the Indian people were united and presented a strong case.
connected ideas of self-development and self-help. According Some of the prominent moderate leaders included Dadabhai
to Aurobindo, through this concept, Indians could effectively Naoroji, Gopal Krishna Gokhale, Surendranath Banerjea,
dispute every facet of colonial rule, without engaging in and Pherozeshah Mehta. They were all from urban areas and
armed rebellion. A four-tier program was suggested for this; had a good education.
development of India as a self-governing country, evolving
140. Solution: (b)
the defense capability, driving out the foreign rule from
every sphere of society and establishing self-rule. Exp) Option b is the correct answer.
Partition of Bengal was carried out by the British viceroy
135. Solution: (c) in India, Lord Curzon on 16 October 1905. The Partition
Exp) Option c is the correct answer. of Bengal separated the largely Muslim eastern areas from
The second session of the Indian National Congress was the largely Hindu western areas. Lord Curzon served as the
held in Calcutta in 1886. It was attended by 150 delegates Governor General and Viceroy of India from 6 January 1899
from all over India. The session was presided over by to 18 November 1905.
Dadabhai Naoroji.
141. Solution: (a)
136. Solution: (a) Exp) Option a is the correct answer.
Exp) Option a is the correct answer. Home Rule Movement was symbol of starting a new phase of
Aurobindo Ghosh, born in 1872, was a key figure in freedom movement in India because it put a perfect project
the Extremist wing of the Indian National Congress. for the self-Government before the nation. They put forth
Advocating for India’s freedom, he led revolutionary efforts demand of self-government or home rule for all of India
through youth clubs like Anushilan Samiti. As principal within the British commonwealth. They clearly mentioned
of Kolkata’s National College, Ghosh championed radical their ultimate objective.
means for independence. His role in the 1907 Congress split
between Moderates and Extremists underscored his stance. Important Tips
Arrested in the 1908 Alipore Bomb Case, Ghosh’s resolute Activities and programmes conducted during Home
commitment to India’s liberation remains notable in the Rule Movement:
nation’s history.
• B.G. Tilak through Home Rule put his demands before
137. Solution: (c) British government that included swarajya, formation
of linguistic states and education in the vernacular.
Exp) Option c is the correct answer.
Inquilab Zindabad, can be translated as “Long Live • The Movement attracted the hitherto ‘politically
Revolution” was coined by Moulana Hasrat Mohani in the backward’ regions of Gujarat and Sindh.
year 1921. It was one of the most famous slogans during the • Activities and programmes conducted during
Indian freedom struggle. It was popularized by Bhagat Singh movements were - promoting political education
during the late 1920s through his speeches and writings. This and discussion through public meetings, organising
slogan was shouted by Bhagat Singh and his companion B. libraries and reading rooms containing books on
K. Dutt after bombing the Central Legislative Assembly in national politics, holding conferences, and organising
1929. classes for students on politics.
• The Programmes also carried out propaganda through
138. Solution: (a)
newspapers, pamphlets, posters, illustrated post-cards,
Exp) Option a is the correct answer. plays, religious songs, etc.
The founder of the Indian National Congress was Allan • The activities also included collecting funds,
Octavian Hume, a retired British Indian Civil Service (ICS) organising social work, and participating in local
officer. He was a social reformer and a political activist who
government activities.
was concerned about the lack of political representation for
Indians under British rule. He founded the Indian National 142. Solution: (a)
Congress in 1885 as a forum for educated Indians to discuss
Exp) Option a is the correct answer.
political and social issues.
The British introduced the Rowlatt Act in 1919 to curb
139. Solution: (b) fundamental rights such as the freedom of expression
Exp) Option b is the correct answer. and strengthened police powers. It aims at limiting the

127 PYQ Workbook


HISTORY OF MODERN INDIA

individual Liberty. This was the prime reason for the protest interests of the masses. He also criticized the Congress for its
by the Indian National Congress against the Bill. focus on constitutional methods, arguing that these methods
were not effective in achieving self-government.
143. Solution: (c)
Exp) Option c is the correct answer. 147. Solution: (c)
The Swadeshi Movement 1905 had its roots in the anti- Exp) Option c is the correct answer.
partition movement which was started to oppose Lord Lord Dufferin was the Viceroy of India from 1884 to
Curzon’s decision of dividing the province of Bengal. Its 1888. During his tenure, the Indian National Congress was
chief architects were Aurobindo Ghosh, Lokmanya Bal established in 1885. The Congress was founded by Allan
Gangadhar Tilak, Bipin Chandra Pal and Lala Lajpat Rai, Octavian Hume, a retired British Indian Civil Service (ICS)
O. Chidambaram Pillai, Babu Genu. Farmers and Muslims officer. The Congress was a forum for educated Indians to
were not involved much in the movement; thus they were discuss political and social issues. It eventually became the
mainly unaffected by the Swadeshi movement. Womens and leading organization in the Indian independence movement.
the Intellectuals took active part in swadeshi movement.
Important Tips
144. Solution: (d)
Establishment of Ramakrishna Mission- In 1897,Swami
Exp) Option d is the correct answer. Vivekananda started the Ramakrishna Mission near
T.S. Alcott was not associated with the Home Rule League Calcutta in a place called Belur.Lord Elgin II was the
in India. Whereas, Bal Gangadhar Tilak, Annie Besant, viceroy at that time.
S. Subrahmanyam Ayer, Joseph Baptista, and Muhammad Establishment of Muslim League in Dhaka- It was
Ali Jinnah contributed to the Home Rule League. These established in 1906 by Nawab Salimullah. Lord Minto II
members got together and decided that it was necessary to was the Viceroy at that time.
have a national alliance that would work throughout the year Beginning of the First Census- The first census of India
(unlike the Congress which had annual sessions). The main was conducted in 1872.Lord Mayo was the Viceroy at that
objective of the Home Rule League was demanding self- time.
government or home rule for all of India within the British
commonwealth.
148. Solution: (b)
145. Solution: (b) Exp) Option b is the correct answer.
Exp) Option b is the correct answer. Abul Kalam Azad was the youngest person to become the
The Komagata Maru was a Japanese steamship that was President of the Indian National Congress. He was 35 years
chartered by Gurdit Singh, a Sikh businessman from old when he was elected President in 1923.
Punjab, India. Singh had organized the voyage to Canada
Important Tips
in order to transport Indian immigrants who were seeking
a better life. The Komagata Maru arrived in Vancouver, Abul Kalam Azad
British Columbia, on May 23, 1914. The Canadian Azad was a prominent Indian scholar, educationist,
government denied the immigrants entry, claiming that and freedom fighter. He was a close associate of
they did not have the necessary paperwork. The immigrants Mahatma Gandhi and he was a leading figure in the Non-
were forced to stay on board the ship for two months, while Cooperation Movement and the Quit India Movement.
the Canadian government debated their fate. On July 23, Azad was a firm believer in secularism and he worked
1914, the Komagata Maru was forced to leave Vancouver to promote religious harmony in India. He was also a
and return to India. The return journey was also difficult, strong advocate of education and he played a key role in
and the ship was attacked by a British warship. The incident the development of the Aligarh Muslim University.
galvanized support for the Ghadar Party, a revolutionary
organization that was committed to overthrowing British 149. Solution: (c)
rule in India. The Ghadar Party launched a series of armed
Exp) Option c is the correct answer.
uprisings in India in 1915, but they were all unsuccessful.
Statement 1 is correct: The Forward Bloc party founded
146. Solution: (a) by Netaji Subhas Chandra Bose on May 3, 1939. It is a
Exp) Option a is the correct answer. revolutionary party which accepts Scientific Socialism as
Lala Lajpat Rai was a Punjabi lawyer, politician, and freedom its objective. It believes in the theory of class. It aims at the
fighter. He was also a critic of the moderate leadership establishment of a Union of Socialist Republics in India
of the Congress, which he believed was too willing to leading to classless society through socialist revolution.
compromise with the British. In 1907, Rai described the Statement 2 is correct: Hindustan Socialist Republican
Congress Conference as the “Annual National Festival Association (HSRA) was a revolutionary organisation
of Educated Indians”. He argued that the Congress was established in 1928 at Feroz Shah Kotla in New Delhi by
a gathering of the elite, and that it did not represent the Chandrasekhar Azad, Ashfaqulla Khan, Bhagat Singh,

PYQ Workbook 128


HISTORY OF MODERN INDIA

Sukhdev Thapar and Jogesh Chandra Chatterjee. Bhagat nationalists who were associated with the Anushilan
Singhwas one of the founders of HSRA. Samiti. These arrests followed the assassination of
150. Solution: (a) Inspector Shamsul Alam on January 24, 1910, in
Exp) Option a is the correct answer. Calcutta. Among those found guilty and sentenced to
Separate electorate demand of Muslim League was formally one-year imprisonment were Jatindranath Mukherjee
accepted by the Congress during the Lucknow session of and Narendranath Bhattacharjee.
Congress in 1916. While the effort of the Congress and the
Muslim League to put up a united front was a far-sighted one, • In Lahore Conspiracy Case many young revolutionaries
but the acceptance of the principle of separate electorates by including Bhagat Singh, Raj guru and Sukh Dev were
the Congress implied that the Congress and the League had charged for involvement in the murder of police officer J
become separate political entities to represent the voice
P Saunders in 1929.
of Indians.
• In Delhi Conspiracy Case, a Bomb was thrown at
151. Solution: (a)
the Viceroy Lord Hardinge when his procession was
Exp) Option a is the correct answer.
moving from Chandni Chowk. The Viceroy wounded in
Batukeshwar Dutt was not hanged for Lahore Conspiracy.
He was caught for throwing bomb in the Central the attempt, but his Mahavat (driver and keeper of an
Legislative Assembly on April 8, 1929 and sentenced to life elephant) was killed. In the trial of this Delhi Conspiracy
imprisonment. Case, Basant Kumar Biswas, Amir Chand and Avadh
Important Tips Behari were convicted and executed.
Batukeshwar Dutt:
• The Alipore Bombay Case Trial refers to an attempt to
• He graduated from Prithvinath College, Kanpur. In his
murder the district judge of Muzaffarpur. The Alipore
college days, he came in contact with Shahid Bhagat
Singh and was inspired by him to join the Hindustan Bomb case conspiracy took place in the year 1908. It
Socialist Republican Association. is also known as the Manicktolla bomb conspiracy or
• He was a member of the “Naujawan Bharat Sabha” Muraripukar conspiracy. Aurobindo Ghosh, Barindra
founded by Bhagat Singh in March 1926.
Kumar Ghosh, and 35 accused were tried in the case.
• After throwing the bombs in the Central Legislative
Assembly on April 8, 1929, the iconic duo was arrested 153. Solution: (d)
on the same day.
Exp) Option d is the correct answer.
• He continued his fight for human rights even in
prison. Statement A is correct- The Nagpur Session of the Indian
• He joined Bhagat Singh in one of the longest ever National Congress was held in 1920. The session was
hunger strikes, running up to 114 days, to support their presided over by C. Vijayaraghavachariar. One of the
demand for a better living standard for the prisoners.
important decisions taken at the session was to constitute the
• He was sentenced to life imprisonment and was sent
to the Andaman Cellular Jail. Dutta was later shifted Provincial Congress Committees on a linguistic basis. This
to Hazaribag jail, Delhi jail and Patna jail. was done in order to make the Congress more representative
• He was released from Patna jail in 1938. He was of the people of India.
suffering from several ailments including tuberculosis.
Statement B is correct- In 1948, the Congress rejected the
Despite his frail health, he joined Gandhiji in the
Quit India movement in 1942. He was again arrested demand of formation of provinces on linguistic basis. The
and imprisoned for another four years. Congress was concerned that the formation of linguistic
• He succumbed to cancer at AIIMS, Delhi on 20th July provinces would lead to the break-up of India. The Congress’s
1965.
position was based on the following arguments:
• Honouring his last will, he was cremated at
Hussainiwala, where Bhagat Singh, Rajguru, and • Linguistic provinces would lead to the dominance of
Sukhdev were laid to rest. one language over other languages.
• Linguistic provinces would create problems for the
152. Solution: (c)
minorities.
Exp) Option c is the correct answer.
• The Howrah-Sibpur Conspiracy case centres around the • Linguistic provinces would make it difficult to maintain
apprehension and subsequent trials of 47 Bengali Indian national unity.

129 PYQ Workbook


HISTORY OF MODERN INDIA

leadership as president from 1889 to 1913, the committee


Important Tips
actively advocated for Indian self-rule through pamphlets,
SK Dhar Commission:
articles, and awareness campaigns in Britain, significantly
• In June 1948, Rajendra Prasad formed a group contributing to the recognition of the Indian independence
called the Linguistic Provinces Commission (Dhar movement.
Commission) to decide if states should be changed
based on the languages spoken there. 157. Solution: (d)
• The group included SK Dhar (a retired judge), Exp) Option d is the correct answer.
Jagat Narain Lal (a lawyer in the group that made When Lord Curzon, then Viceroy of India, announced the
the Indian Constitution), and Panna Lal (a retired partition of Bengal in July 1905, Indian National Congress,
government officer). initiated the Swadeshi movement in Bengal. Swadeshi
• In their report, the Commission suggested that making movement was launched as a protest movement which also
states only about languages might not be the best idea gave a lead to the Boycott movement in the country. It
for India’s overall good. also promoted National education among the masses. Bal
Gangadhar Tilak, Bipin Chandra Pal, Lala Lajpat Rai are the
154. Solution: (a)
key people in the swadeshi movement.
Exp) Option a is the correct answer.
V.D. Savarkar founded Abhinav Bharat - A revolutionary Important Tips
organization in 1904, which was a new name of Mitra Mela. Non-cooperation Movement:
He also authored “The Indian War of Independence - 1857” • The non-cooperation movement by Gandhiji was
which provides a nationalist view of the revolt of 1857. He followed from September 1920 to February 1922.
was the first person to call 1857 revolt India’s first war of • The Jallianwala Bagh Massacre in Amritsar in 1919
Independence. He founded the Free India Society, based on was one of the major reasons for the start of the Non-
the thoughts of the Italian nationalist Giuseppe Mazzini. He cooperation movement.
had also written a biography of Mazzini in Marathi in order
• The Chauri Chaura incident occurred at Chauri
to inspire Indian nationalist. He jumped into the sea from
Chaura in the Gorakhpur district (Uttar Pradesh) on
the sailing ship in order to escape British captivity. Hence,
4 February 1922, when a large group of protesters,
all the options are correct.
participating in the Non-cooperation movement,
155. Solution: (c) clashed with police, who opened fire.
Exp) Option c is the correct answer. • As the incident turned violent Mahatma Gandhi called
Annie Besant was elected president of the Indian National off the ‘Non-cooperation Movement’ on 12 February
Congress in 1917 and 1933. She was a strong advocate for 1922, as a direct result of this incident.
Self-rule for India and she worked to unite the Hindu and
158. Solution: (b)
Muslim communities. She was also a supporter of the Home
Rule Movement. Exp) Option b is the correct answer.
• Jatin Das arrested for his involvement in Lahore
Important Tips
Conspiracy case of 1929. He was passed away after a
Annie Besant was a British-born Indian Theosophist, sixty-three-day hunger strike while he was imprisoned
educationist, women’s rights activist, and political leader in the Lahore jail.
who was the first female President of the Indian National
• Chandra Shekhar Azad, a revolutionary freedom fighter
Congress. Besant was born in London in 1847. She was
a strong advocate for women’s rights, and she founded was engaged in encounter with the British in Alfred
the Women’s Indian Association in 1917. She was also a park in Prayagraj on 27 February 1931. He shot himself
Theosophist, and she founded the Theosophical Society dead because he had vowed he would never be arrested
in India in 1879. by British Police.
• Bhagat Singh, Shivaram Hari Rajguru and Sukhdev
156. Solution: (a) Thapar were hanged at the Lahore Central Jail in Lahore
Exp) Option a is the correct answer. on March 23, 1931. India observes 23 rd March as ‘Shaheed
Sir William Wedderburn, a retired British Indian Civil Diwas’ to mark the death anniversary of these heros.
Service (ICS) officer, strongly endorsed Indian self- • Kalpana Dutt was a remarkable Indian revolutionary
rule and founded the British Committee of the Indian who participated in the Chittagong Armory Raid led
National Congress in 1889 to garner British support for the by Surya Sen on 18 April 1930. Kalpana was given a life
Congress. This committee, composed of British individuals sentence.
concerned about Indian affairs, held regular meetings to
discuss Indian politics and promote awareness of the Indian 159. Solution: (b)
independence movement in Britain. Under Wedderburn’s Exp) Option b is the correct answer.

PYQ Workbook 130


HISTORY OF MODERN INDIA

Surya Senwas not associated with the Kakori incident of 161. Solution: (d)
1925. Other than him, Ramprasad Bismil, Ashfaqulla Khan, Exp) Option d is the correct answer.
Rajendranath Lahiri and Roshan Singh were associated with
Kakori incident. They were hanged in 1927 in Gorakhpur The first Political Organization established in India in 1838
Central Jail. was known as the Zamindari Association, later renamed
the Landholders’ Society. Founded by influential zamindars
Important Tips
like Bhabani Charan Bandyopadhyay and Dwarkanath
Surya Sen
Tagore, it aimed to advocate for the implementation of
• He is more popularly known as Master Da.
Permanent Settlement across India. It had ties with the
• In 1916, buoyed by the spirit of nationalism and
British India Society in London and worked to secure
inspired by his teacher in college, Sen joined the
Anushilan Samiti. concessions for tax exemptions. The association’s influence
• Sen was also closely involved with the Non- was limited to Bengal, and it declined and closed by 1850,
Cooperation Movement (1920-1922) and was arrested eventually merging with the British Indian Association.
for his anti-colonial activities in the late 1920s.
162. Solution: (c)
• Surya Sen headed the Indian Revolutionary Army
(IRA). The main aim of the IRA was to lead an Exp) Option c is the correct answer.
organised struggle against the British and to challenge J.B. Kripalani was the President of the Indian National
their authority. One of the most notable actions of the
Congress at the time of India’s independence in 1947. He
IRA was the Chittagong Armoury Raid of 1930.
was a Gandhian socialist and a moderate leader of the
• The raid was quite successful. Sen hoisted the national
flag, took salute, and proclaimed a provisional Congress. Kripalani was elected President of the Indian
revolutionary government. National Congress in 1946. He was a moderate leader who
• On 12 January 1934, Sen was hanged along with his opposed the partition of India. He resigned as President in
associate, Tarakeswar Dastidar. 1947 after the partition was announced.

163. Solution: (b)


160. Solution: (d)
Exp) Option d is the correct answer. Exp) Option b is the correct answer.

Muzaffar Ahmad, S.A. Dange, Shaukat Usmani and Nalini Bhagat Singh, Sukhdev, and Rajguru were sentenced to death
Gupta were jailed for Kanpur Bolshevik conspiracy case in Lahore conspiracy case. To avenge the killing of Lala
of 1924. This case was responsible for actively introducing Lajpat Rai, Bhagat Singh, Raj guru, Jai Gopal and Sukh Dev
communism to the Indian masses. conspired to kill the police chief, Jams Scott but they shot the
Important Tips DSP J. P. Saunders, who was killed on the spot. This case was
Communist Movement in India: known as the Lahore Conspiracy Case.
• The Communist Movement in India was established
164. Solution: (b)
following the 2nd World Congress of the Communist
International of 1920 by the M.N. Roy, Abani Exp) Option b is the correct answer.
Mukherji, and others. Owing to mass political protests, the Bengal partition was
• It was established in Tashkent in 1920 (now the capital annulled in 1911. King George V abrogated Curzon’s Act at
of Uzbekistan).
the Royal Durbar in Delhi in 1911. East and West Bengal were
• The first chosen leader was M.N. Roy. reunited while Bihar and Orissa became separate provinces.
• The Communist Movement in India proclaimed that A separate Assam province was created. The capital of British
socialism in India would be built through revolution.
India was moved to Delhi from Calcutta (now Kolkata).
• M.N. Roy established the Communist Party of India
(CPI) in December 1925 after the Indian Communist 165. Solution: (a)
Conference was convened in Kanpur that year.
Exp) Option a is the correct answer.
• In the year 1929, the Government arrested 31 labour
union leaders and charged them with conspiracy to During the Swadeshi Movement, “Vande Mataram”
establish communist international in India. This Case became the theme song of Indian National Movement. The
is famously known as the Meerut Conspiracy Case. Swadeshi movement began in 1905 with the Viceroy of India,
• The Communist party was acknowledged by the Lord Curzon, partitioning Bengal and continued until 1911.
Communist International as the Indian branch in This was the strongest in Bengal and was also recognized in
1934. India as the Vande mataram Movement.

131 PYQ Workbook


HISTORY OF MODERN INDIA

Important Tips Important Tips


Swadeshi movement Indian National Congress and Muslim League put
• Among the movement’s various forms of struggle, it was forward shared requests to the British government,
the boycott of foreign-made goods that encountered including:
the greatest visible success on the practical and popular • Increasing the elected seats on councils.
level. • Ensuring protection for minorities in the provinces.
• Boycott and public burning of foreign clothes, picketing • Granting autonomy to all provinces.
of shops selling foreign goods, all became common in
• Separating the executive and judiciary.
remote corners of Bengal as well as in many major
cities and towns across the country.
• Another form of mass mobilization widely used by 168. Solution: (d)
the Swadeshi movement was the corps of volunteers Exp) Option d is the correct answer.
(samitis). The correct chronological order of the organizations is:
• Ashwini Kumar Dutt, a school teacher, set up the • Bombay Association (1852)
Swadesh Bandhab Samiti in Barisal which was the
• Indian League (1875)
best-known volunteer organization of all of them.
• Indian Association (1876)
166. Solution: (c) • Madras Mahajan Sabha (1884)

Exp) Option c is the correct answer. Important Tips

Mahatma Gandhi suggested the winding up of the Indian Bombay Association:

National Congress after India attained independence. He • In 1852, Jagannath Shankarshet established the
Bombay Association with notable members like
felt that the Congress had served its purpose and that it
Naoroji Fursungi, Vinayak Shankarshet, and
was no longer needed. He wrote in a note dated January 27, Dadabhai Naoroji. Sir Jamshedji Jejibhai served as the
1948, three days before he was assassinated: first president. The association aimed to voice public
“The Congress has outlived its use. It has no reason to grievances against British policies and actions during
the colonial period.
exist. The atmosphere is charged with communal poison.
Indian League:
Congress is the only organization which can still command
• The Indian League was founded in 1875 by Sisir
the confidence of all communities. But if it tries to function,
Kumar Ghosh with the goal of “stimulating a sense
it will only add fuel to the fire. Let it disband. Let all of nationalism among the people” and encouraging
Congressmen retire to their respective homes. Let them wait political education. This organization was associated
until the atmosphere is cleared. Then they can come together with nationalist leaders such as Ananda Mohan Bose,
and reorganize the Congress, if they so desire.” Durgamohan Das, Nabagopal Mitra, Surendranath
Banerjee, and others.
167. Solution: (c)
Exp) Option c is the correct answer. 169. Solution: (a)
Exp) Option a is the correct answer.
Statement 1 is incorrect- The first session of the Indian
National Congress was held in Bombay in 1885 under the Anushilan Samiti was a secret revolutionary organization
which was led by Barindra Ghosh, Pramathanath Mitra,
presidency of Womesh Chunder Bonnerjee.
Aurobindo Ghosh and Jatindranath Bannerjee, etc.
Statement 2 is correct- The second session of the Indian
Important Tips
National Congress was held in Calcutta in 1886 under the
Swadesh Bandhab Samiti was founded by Ashwini Kumar
presidency of Dadabhai Naoroji.
Dutta during Swadeshi movement. It aimed to promote
Statement 3 is correct- In December 1916, the Indian the consumption of indigenous products and the boycott
National Congress and the Muslim League got together in of foreign goods.
Lucknow and made a deal known as the Lucknow Pact. Brahma Mohan Ghosh was the leader of the Brati Samiti
This agreement was signed by Bal Gangadhar Tilak and of Fariddpur (a district of Bangladesh). Later on, he
realized the decline of Brati Samiti which led him to join
Muhammad Ali Jinnah. Because of this, Muslim League
Pulin Behari Das at Dacca in his Anushilan Samiti.
leaders decided to join the Congress movement that aimed
for India’s freedom. The Lucknow Pact was seen as a hopeful 170. Solution: (a)
sign of Hindus and Muslims coming together. Exp) Option a is the correct answer.

PYQ Workbook 132


HISTORY OF MODERN INDIA

Statement 1 is correct: The Swadeshi movement in 1905 from the free trade and the infrastructure development
witnessed massive demonstrations and protests across under British rule.
Bengal. People came out in large numbers to voice their However, statement II is not the correct explanation of
opposition to the partition of Bengal and to support the statement I, because Naoroji’s argument was not based on
cause of Swadeshi (indigenous production) and Boycott the imperialist view of India’s integration into the world
(boycott of foreign goods). market, but on his own calculation of India’s national income
Statement 2 is correct:As part of the Swadeshi movement, and expenditure. He did not accept the imperialist claim that
a hartal (general strike and closure of businesses) was India was progressing under British rule, but rather exposed
observed in Calcutta (now Kolkata) as a form of protest the adverse effects of British exploitation on India’s economy
against the partition of Bengal. It was an expression of unity and society.
and solidarity among the people. Important Tips
Statement 3 is correct: The Swadeshi movement was initially National Critique of colonial Economy:
initiated by the moderates within the Indian National • Dadabhai Naoroji wrote several books on this topic,
Congress, such as Surendranath Banerjee. However, it later such as Poverty and Un-British Rule in India and The
gained momentum and was embraced by more radical or Economic Condition of India.
revolutionary nationalists like Aurobindo Ghosh and
• Ramesh Chandra Dutt wrote a comprehensive work
Bal Gangadhar Tilak, who advocated for more assertive
on the economic history of India, titled The Economic
methods in the struggle for independence.
History of India under Early British Rule.
171. Solution: (c) • Rajani Palme Dutt wrote several books on the colonial
Exp) Option c is the correct answer. and imperialist aspects of British rule in India, such as
Statement 1 is correct: On April 13, 1919, British Indian India Today and India: A Nation.
Army troops, under the command of Brigadier General 173. Solution: (d)
Reginald Dyer, opened fire on a large gathering of unarmed
Exp) Option d is the correct answer.
civilians who had gathered at Jallianwala Bagh in Amritsar.
Statement 1 is correct: The Swadeshi Movement was a mass
The crowd had gathered for a peaceful protest against the
movement that was marked by the organization of samitis,
repressive Rowlatt Act imposed by the British colonial
or associations. These samitis were responsible for organizing
authorities.
protests, boycotts, and other forms of civil disobedience.
Statement 2 is correct:General Dyer was in command of
Statement 2 is correct: The Swadeshi Movement witnessed
the troops deployed in Amritsar during the Jallianwala Bagh
widespread labor strikes in various industries and sectors.
massacre. He ordered the troops to open fire on the crowd
Workers actively participated in the movement, demanding
without issuing any warning or giving them an opportunity better working conditions and higher wages.
to disperse.
Statement 3 is correct: The Swadeshi Movement emphasized
Statement 3 is correct: Prior to opening fire, General Dyer the concept of ‘atmashakti,’ which means self-help or
did not issue any warning or give the crowd an opportunity self-reliance. It aimed to promote indigenous industries,
to disperse peacefully. The sudden and brutal attack resulted encourage economic self-sufficiency, and reduce dependency
in the loss of hundreds of lives and caused widespread on imported British goods.
outrage and condemnation. Statement 4 is correct: In the sphere of education, national
172. Solution: (b) schools were established. These schools were designed to
provide a nationalist education that would help to promote
Exp) Option b is the correct answer.
Indian self-rule.
Statement I is correct: Dadabhai Naoroji did argue that
what was being drained out of India by the British was the 174. Solution: (c)
‘potential surplus’ that could generate more economic Exp) Option c is the correct answer.
development in India if invested in India. He proposed the Statement 1 is correct: The Indian National Congress was
‘drain of wealth’ theory in 1867, in which he estimated the indeed founded in Bombay (now Mumbai) on December
net national profit of India and the effect of colonial rule 28, 1885. The founding session of the Congress was attended
on it. He identified six factors that caused external drain by 72 delegates representing various parts of India.
and criticized the British for exploiting India’s resources, Statement 2 is correct: Womesh Chandra Bonnerjee,
markets, and manpower. commonly known as W.C. Banerjee, was elected as the first
Statement II is correct: Imperialists did believe that India President of the Indian National Congress.
was brought into the large capitalist world market and
175. Solution: (b)
that was in itself a progress towards modernization.
They justified their colonial rule by claiming that they were Exp) Option b is the correct answer.
civilizing and modernizing India, which they considered Statement 1 is correct: Champaran region had a long
backward and stagnant. They also argued that India benefited history of anti-planter discontent and agitation. The indigo

133 PYQ Workbook


HISTORY OF MODERN INDIA

planters had been exploiting the peasants for many years, and Important Tips
the peasants had been protesting against this exploitation.
Ghadar Party:
Statement 2 is correct: Mahatma Gandhi gave all India • It was founded in 1913 by Sohan Singh Bhakna.
publicity to the grievances of the Champaran cultivators.
• It was initially called as Pacific Coast Hindustan
He went to Champaran himself to investigate the situation, Association.
and he wrote articles and gave speeches about the plight of
• It published newspaper called ‘The Ghadar’.
the peasants. This publicity helped to raise awareness of the
• The Yugantar Ashram was the its headquarters.
issue and to put pressure on the government to take action.
• The organization was dissolved in 1948.
Statement 3 is incorrect: The main issue in Champaran
was the forced cultivation of indigo, not taxation on sugar.
179. Solution: (a)
176. Solution: (a) Exp) Option a is the correct answer.
Exp) Option a is the correct answer. The demand for universal adult franchise was not a part
of the Congress moderates’ agenda during the early 20th
Boycott and picketing were not features of the Home
century. They primarily focused on political reforms,
Rule Movement in India. These were features of the Non- repealing oppressive laws, extending the land revenue system,
Cooperation Movement, which was launched by Mahatma and increasing employment opportunities for Indians in the
Gandhi in 1920. military.
Important Tips 180. Solution: (b)
The Home Rule Movement: Exp) Option b is the correct answer.
• The home rule movement was the Indian response to Statement I is true: Non-cooperation movement in Punjab
the First World War in a less charged but in a more did begin with the student movement inspired by Lala
effective way. Lajpat Rai in January 1921.
• With people already feeling the burden of war time Statement II is true: The central countryside of Punjab,
miseries caused by high taxation and a rise in which had a Sikh majority population, was indeed stirred by
prices, Tilak and Annie Besant ready to assume the the powerful Akali upsurge.
leadership the movement started with great vigour.
However, Statement II does not provide a correct
• Two Indian Home Rule Leagues were organised on explanation for Statement I. While both statements are
the lines of the Irish Home Rule Leagues and they individually true, the Sikh-dominated central Punjab
represented the emergence of a new trend of aggressive countryside being stirred by the Akali upsurge is not directly
politics.
related to the student movement inspired by Lala Lajpat
• The League campaign aimed to convey to the common Rai that initiated the non-cooperation movement. The
man the message of home rule as self-government. two statements represent different aspects of the overall
• The movement used a variety of methods, including movement in Punjab.
setting up discussion groups and reading rooms,
181. Solution: (d)
selling and circulating pamphlets, and organizing
public meetings. Exp) Option d is the correct answer.
Statement 1 is incorrect- The Non-Cooperation Movement,
177. Solution: (a) which took place in India from 1920 to 1922, aimed to resist
British colonial rule through nonviolent means. While
Exp) Option a is the correct answer.
the movement garnered widespread support from various
The Safety Valve Theory was first proposed by R. Palme sections of society, including students, urban middle class,
Dutt, a Marxist historian. Dutt argued that the British and some sections of the working class, the participation
government allowed the formation of the Congress in of peasants from Karnataka was not particularly significant.
order to channel the growing discontent of the Indian The Non-Cooperation Movement largely gained momentum
in regions where the impact of British rule was more visible
people into a safe and non-violent outlet. This would help
and oppressive, such as Punjab and Bengal.
to prevent the Indian people from resorting to more radical
Statement 2 is correct- The movement was marked by
methods of protest, such as violence.
the participation of non-Brahmin lower castes in Madras
178. Solution: (a) and Maharashtra. In Madras, leaders like E.V. Ramasamy
(Periyar) mobilized non-Brahmin communities, emphasizing
Exp) Option a is the correct answer.
the need to fight against caste-based discrimination and
The headquarters of the Ghadar Party were located in San social inequality. Similarly, in Maharashtra, leaders like
Francisco, USA. B.R. Ambedkar, who hailed from the Dalit (formerly

PYQ Workbook 134


HISTORY OF MODERN INDIA

known as untouchable) community, actively advocated for Statement 2 is correct: The Safety Valve Theory was
the rights and welfare of lower-caste individuals. proposed by Lala Lajpat Rai in his book The Story of My
Statement 3 is incorrect- The Non-Cooperation Movement Deportation (1908), based on a biography of AO Hume by
did witness labor unrest in places like Assam, Bengal, William Wedderburn, who was also a founding member of
and Madras. While the movement primarily focused on the Congress.
boycotting British goods and institutions, labor issues The theory suggested that Hume founded the Congress to
were also part of the larger socio-economic grievances that provide a peaceful outlet for the discontent among educated
motivated people to join the movement. Indians and to prevent a violent uprising against British rule.
Statement 4 is incorrect- The Non-Cooperation Movement Statement 3 is incorrect: In an effort to reach all regions, it
was indeed shaken by the Chauri Chaura incident in 1922. was decided to rotate the Congress session among different
After the incident, in which protesters attacked and killed parts of the country. The President belonged to a region
policemen, Mahatma Gandhi decided to withdraw the other than where the Congress session was being held.
movement as he believed it had turned violent and lost its Statement 4 is correct: WC Bannerjee, a prominent lawyer
non-violent character. and public figure from Calcutta (now Kolkata), was elected
Important Tips as the first president of the Indian National Congress at its
inaugural session in Bombay in 1885.
Some facts about the Non-Cooperation Movement:
• Mahatma Gandhi led the Non-Cooperation 183. Solution: (c)
Movement and announced its manifesto in 1920. Exp) Option c is the correct answer.
• C.R. Das presented the main resolution on non- Statement 1 is correct: The first session of the Indian
cooperation during the Congress session in Nagpur in National Congress was held in Bombay (now Mumbai) in
1920, and his subordinates played key roles in Hindu- 1885. It took place from 28th to 31st December 1885.
Muslim unity. Statement 2 is correct: Surendranath Banerjee, a
• Jawaharlal Nehru encouraged the formation of Kisan prominent leader, was unable to attend the first session
Sabhas and disagreed with Gandhi’s decision to of the Indian National Congress due to his commitment to
withdraw the movement. the simultaneous session of the Indian National Conference.
• Subhash Chandra Boseresigned from the civil
184. Solution: (a)
service and became the Principal of the National
College in Calcutta during the movement. Exp) Option a is the correct answer.

• The Ali brothers, Shaukat Ali and Muhammad Ali, Statement 1 is correct: BG Tilak was a prominent nationalist
emphasized the religious unlawfulness of Muslims leader who advocated for self-government or swaraj within
serving in the British Army during the All India the British Empire for India. He founded the Indian Home
Khilafat Conference. Rule League in April 1916 at Belgaum, Maharashtra, with
Joseph Baptista as the president and NC Kelkar as the
182. Solution: (c) secretary.
Exp) Option c is the correct answer. Statement 2 is incorrect: NC Kelkar was a journalist and a
Statement 1 is incorrect: The first session of the Indian political activist who was closely associated with BG Tilak
National Congress was held in Bombay (now Mumbai) in and his Home Rule Movement. He was the secretary of the
December 1885 under the presidency of WC Bannerjee. Indian Home Rule League founded by Tilak and also edited
Motilal Nehru became the president of the Congress in 1919. his newspapers Kesari and Mahratta.

135 PYQ Workbook


HISTORY OF MODERN INDIA

HISTORY OF MODERN INDIA


ERA OF MASS NATIONALISM [1919 - 1947]
This unit consist of questions from 1919 - 1947 CE of Indian National Freedom Movement.

4.1. UPSC CSE Previous Years’ Questions 5. In the context of Colonial India, Shah Nawaz
Khan, Prem Kumar Sehgal and Gurbaksh
1. With reference to the proposals of Cripps Singh Dhillon are remembered as:
Mission, consider the following statements: [UPSC CSE Prelims 2021]
1. The Constituent Assembly would have (a) leaders of Swadeshi and Boycott
members nominated by the Provincial
Movement.
Assemblies as well as the Princely States.
(b) members of the Interim Government in
2. Any Province, which is not prepared to
1946.
accept the new Constitution would have
the right to sign a separate agreement (c) members of the Drafting Committee in
with Britain regarding its future status. the Constituent Assembly.
(d) officers of the Indian National Army.
Which of the statements given above is/are
correct? [UPSC CSE Pre 2022] 6. The Gandhi-Irwin Pact included which of
(a) 1 only the following? [UPSC CSE Pre. 2020]
(b) 2 only 1. Invitation to Congress to participate in
(c) Both 1 and 2 the Round Table Conference.
(d) Neither 1 nor 2 2. Withdrawal of Ordinances promulgated
2. With reference to 8th August, 1942 in in connection with the Civil Disobedience
Indian history, which one of the following Movement.
statements is correct? [UPSC CSE Pre 2021] 3. Acceptance of Gandhiji’s suggestion for
(a) The Quit India Resolution was adopted enquiry into police excesses.
by the AICC.
4. Release of only those prisoners who were
(b) The Viceroy’s Executive Council was
not charged with violence.
expanded to include more Indians.
(c) The Congress ministries resigned in Select the correct answer using the code given
seven provinces. below:
(d) Cripps proposed an Indian Union with (a) 1 only
full Dominion Status once the Second (b) 1, 2 and 4 only
World War was over. (c) 3 only
3. In India, the first major public appearance (d) 2, 3 and 4 only
of Mahatma Gandhi was in 7. Consider the following pairs:
[UPSC CSE Pre.2021]
[UPSC CSE Pre. 2019]
(a) Champaran (1917)
(b) Kheda (1918) Movement/
Leader
(c) Inauguration of Banaras Hindu University Organization
(1916) 1. All India Anti-
(d) Rowlatt Satyagraha (1919) Untouchability Mahatma Gandhi
League
4. Who among the following founded the Bhil
Seva Mandal in 1922? [UPSC CSE Pre.2021] 2. All India Kisan Swami Sahajanand
Sabha Saraswati
(a) Dayaram Gidumal
(b) Gurusaday Dutt 3. Self-Respect
(c) Dhondo Keshav Karve Movement E.V. Ramaswami
(d) Amritlal Vithaldas Thakkar Naicker

PYQ Workbook 136


HISTORY OF MODERN INDIA

Which of the pairs given above is/are correctly (c) Having two sets of rulers; one in London
matched? and another in Delhi.
(a) 1 only (d) Division of the subjects delegated to the
(b) 1 and 2 only provinces into two categories.
(c) 2 and 3 only 12. With reference to Indian freedom struggle,
(d) 1, 2 and 3 consider the following events :
8. With reference to the British colonial rule [UPSC CSE Pre. 2017]
in India, consider the following statements: 1. Mutiny in Royal Indian Navy
[UPSC CSE Pre. 2019] 2. Quit India Movement launched
1. Mahatma Gandhi was instrumental in 3. Second Round Table Conference
the abolition of the system of ‘indentured What is the correct chronological sequence of
labour’. the above events ?
2. In Lord Chelmsford’s ‘War Conference’, (a) 1-2-3
Mahatma Gandhi did not support the (b) 2-1-3
resolution on recruiting Indians for (c) 3-2-1
World War. (d) 3-1-2
3. Consequent upon the breaking of 13. The objective of the Butler Committee of
Salt Law by Indian people, the Indian 1927 was to? [UPSC CSE Pre. 2017]
National Congress was declared illegal by (a) Define the jurisdiction of the Central and
the colonial rulers. Provincial Governments.
Which of the statements given above are (b) Define the powers of the Secretary of
correct? State for India.
(a) 1 and 2 only (c) Impose censorship on national press.
(b) 1 and 3 only (d) Improve the relationship between the
(c) 2 and 3 only Government of India and the Indian
(d) 1, 2 and 3 States.
9. In 1920, which of the following changed its 14. The plan of Sir Stafford Cripps envisaged
name to “Swarajya Sabha”? that after the Second World War
[UPSC CSE Pre. 2018] [UPSC CSE Pre. 2016]
(a) All India Home Rule League (a) India should be granted complete
(b) Hindu Mahasabha independence.
(c) South Indian Liberal Federation (b) India should be partitioned into two
(d) The Servants of India Society before granting independence.
(c) India should be made a republic with
10. Which one of the following is a very the condition that she will join the
significant aspect of the Champaran Commonwealth.
Satyagraha? [UPSC CSE Pre. 2018] (d) India should be given Dominion status.
(a) Active all-India participation of lawyers, 15. With reference to Rowlatt Satyagraha,
students and women in the National which of the following statements is/ are
Movement. correct? [UPSC CSE Pre. 2015]
(b) Active involvement of Dalit and Tribal 1. The Rowlatt Act was based on the
communities of India in the National recommendations of the ‘Sedition
Movement. Committee’.
(c) Joining of peasant unrest to India’s
2. In Rowlatt Satyagraha, Gandhiji tried to
National Movement.
utilize the Home Rule League.
(d) Drastic decrease in the cultivation of
plantation crops and commercial crops. 3. Demonstrations against the arrival of
Simon Commission coincided with
11. In the context of Indian history, the Rowlatt Satyagraha.
principle of `Dyarchy (diarchy) ’ refers to Select the correct answer using the code given
[UPSC CSE Pre. 2017] below:
(a) Division of the central legislature into (a) 1 only
two houses. (b) 1 and 2 only
(b) Introduction of double government i.e., (c) 2 and 3 only
Central and State governments. (d) 1, 2 and 3

137 PYQ Workbook


HISTORY OF MODERN INDIA

16. With reference to Congress Socialist Party, 21. Quit India Movement was launched in
consider the following statements: response to [UPSC CSE Pre. 2013]
[UPSC CSE Pre. 2015] (a) Cabinet Mission Plan
1. It advocated the boycott of British goods (b) Cripps Proposals
and evasion of taxes. (c) Simon Commission Report
2. It wanted to establish the dictatorship of (d) Wavell Plan
proletariat.
22. The Congress ministries resigned in the
3. It advocated separate electorate for mi-
seven provinces in 1939, because
norities and oppressed classes.
[UPSC CSE Pre. 2012]
Which of the statements given above is / are
correct? (a) the Congress could not form ministries in
(a) 1 and 2 only the other four provinces.
(b) 3 only (b) emergence of a ‘left wing’ in the Congress
(c) 1, 2 and 3 made the working of the ministries
(d) None impossible.
(c) there were widespread communal
17. Who of the following organized a march on disturbances in their provinces.
the Tanjore coast to break the Salt Law in
(d) None of the statements (a), (b) and (c)
April 1930? [UPSC CSE Pre. 2015]
given above is correct.
(a) V. O. Chidambaram Pillai
(b) C. Rajagopalachari 23. Which of the following parties were
(c) K. Kamaraj established by Dr. B. R. Ambedkar?
(d) Annie Besant [UPSC CSE Pre. 2012]
18. The Radcliffe Committee was appointed to 1. The Peasants and Workers Party of India.
[UPSC CSE Pre. 2014] 2. All India Scheduled Castes Federation.
(a) solve the problem of minorities in India. 3. The Independent Labour Party.
(b) give effect to the Independence Bill. Select the correct answer using the codes
(c) delimit the boundaries between India and given below:
Pakistan.
(a) 1 and 2 only
(d) enquire into the riots in East Bengal.
(b) 2 and 3 only
19. The 1929 Session of Indian National Con- (c) 1 and 3 only
gress is of significance in the history of the (d) 1, 2 and 3
Freedom Movement because the
[UPSC CSE Pre. 2014] 24. Mahatma Gandhi undertook fast unto
(a) attainment of Self-Government was de- death in 1932, mainly because
clared as the objective of the Congress . [UPSC CSE Pre. 2012]
(b) attainment of Poorna Swaraj was adopted (a) Round Table Conference failed to satisfy
as the goal of the Congress. Indian political aspirations.
(c) Non-Cooperation Movement was (b) Congress and Muslim League had
launched. differences of opinion.
(d) decision to participate in the Round Ta- (c) Ramsay Macdonald announced the
ble Conference in London was taken. Communal Award.
20. The people of India agitated against the ar- (d) None of the statements (a) , (b) and (c)
rival of Simon Commission because given above is correct in this context.
[UPSC CSE Pre. 2013] 25. The Lahore Session of the Indian National
(a) Indians never wanted the review of the Congress (1929) is very important in
working of the Act of 1919. history, because [UPSC CSE Pre. 2012]
(b) Simon Commission recommended the
1. the Congress passed a resolution
abolition of Dyarchy (Diarchy) in the
demanding complete independence.
Provinces.
(c) there was no Indian member in the Si- 2. the rift between the extremists and
mon Commission. moderates was resolved in that session.
(d) the Simon Commission suggested the 3. a resolution was passed rejecting the two-
partition of the country. nation theory in that session.

PYQ Workbook 138


HISTORY OF MODERN INDIA

Which of the statements given above is/are 30. After Quit India Movement, C.
correct? Rajagopalachari issued a pamphlet entitled
(a) 1 only ‘The Way Out’. Which one of the following
(b) 2 and 3 was a proposal in this pamphlet?
(c) 1 and 3 [UPSC CSE Pre 2010]
(d) None of the above (a) The establishment of a “War Advisory
26. The Rowlatt Act aimed at Council” composed of representatives of
[UPSC CSE Pre. 2012] British India and the Indian States.
(a) compulsory economic support to war (b) Reconstitution of the Central Executive
efforts. Council in such a way that all its
(b) imprisonment without trial and summary members, except the Governor General
procedures for trial. and the Commander-in-Chief, should be
(c) suppression of the Khilafat Movement. Indian leaders.
(d) imposition of restrictions on freedom of (c) Fresh elections to the Central and
the press. Provincial Legislatures to be held at the
end of 1945 and the Constitution-making
27. Which one of the following observation is body be convened as soon as possible.
not true about the Quit India Movement of (d) A solution to the Constitutional deadlock.
1942 ? [UPSC CSE Pre. 2011]
(a) It was a non-violent movement 31. Who among the following were official
(b) It was led by Mahatma Gandhi Congress negotiators with Cripps Mission?
(c) It was a spontaneous movements [UPSC CSE Pre 2010]
(d) It did not attract the labour class in (a) Mahatma Gandhi and Sardar Patel.
general (b) Acharya J.B. Kripalani and C.
28. What was the reason for Mahatma Gandhi Rajgopalachari.
to organize a satyagraha on behalf of the (c) Pandit Jawaharlal Nehru and Maulana
peasants off Kheda ? [UPSC CSE Pre. 2011] Azad.
1. The administration did not suspend (d) Rajendra Prasad and Rafi Ahamad
the land revenue collection in spite of a Kidwai.
drought. 32. Who among the following drafted the
2. The administration proposed to introduce fundamental rights resolution for the
permanent settlement in Gujarat. Karachi Session of Congress in 1931?
Which of the statements given above is/are [UPSC CSE Pre 2010]
correct? (a) Mahatma Gandhi
(a) 1 only (b) Pandit Jawaharlal Nehru
(b) 2 only (c) Dr. Rajendra Prasad
(c) Both 1 and 2 (d) Dr. B.R. Ambedkar
(d) Neither 1 nor 2
33. With reference to the Simon Commission’s
29. With reference to the period of Indian recommendations, which one of the
freedom struggle, which of the following following statements is correct?
was/were recommended by the Nehru [UPSC CSE Pre 2010]
report? [UPSC CSE Pre. 2011] (a) It recommended the replacement of
1. Complete Independence for India. diarchy with responsible Government in
2. Joint electorates for reservation of seats the provinces.
for minorities. (b) It proposed the setting up of inter-
3. Provision of fundamental rights for the provincial council under the Home
people of India in the Constitution. Department.
Select the correct answer using the codes (c) It suggested the abolition of bicameral
given below legislature at the centre.
(a) 1 Only (d) It recommended the creation of Indian
(b) 2 and 3 only Police Service with a provision for
(c) 1 and 3 only increased pay and allowances for British
(d) 1, 2 and 3 recruits as compared to Indian recruits.

139 PYQ Workbook


HISTORY OF MODERN INDIA

34. Who of the following founded the Reason (R): The Congress did not accept the
Ahmedabad Textile Labour Association? decision of the Viceroy to declare war against
[UPSC CSE Pre 2009] Germany in the context of the Second World
(a) Mahatma Gandhi War.
(b) Sardar Vallabhbhai Patel Code: [UPSC CSE Pre 2008]
(c) N.M. Joshi (a) (A) and (R) are individually true, and (R)
(d) J.B. Kriplani is the correct explanation of (A).
(b) (A) and (R) are individually true, but (R)
35. With which one of the following movements is not the correct explanation of (A).
is the slogan ‘Do or Die’ associated? (c) Both (A) and (R) are false.
[UPSC CSE Pre 2009] (d) (R) is not the correct explanation of (A).
(a) Swadeshi Movement
(b) Non-Cooperation Movement 41. Which one of the following suggested the
reconstitution of the Viceroy’s Executive
(c) Civil Disobedience Movement
Council in which all the portfolios including
(d) Quit India Movement
that of War Members were to be held by the
36. In the ‘Individual Satyagraha,’ Vinoba Indian leaders? [UPSC CSE Pre 2008]
Bhave was chosen as the first Satyagrahi. (a) Simon Commission, 1927
Who was the second? [UPSC CSE Pre 2009] (b) Shimla Conference, 1945
(a) Dr. Rajendra Prasad (c) Cripps Mission, 1940
(b) Pandit Jawaharlal Nehru (d) Cabinet Mission, 1946
(c) C. Rajagopalachari 42. Which Portfolio was held by Dr. Rajendra
(d) Sardar Vallabhbhai Patel Prasad in the Interim Government formed
37. Consider the following statements: The in the year 1946? [UPSC CSE Pre 2006]
‘Cripps’ proposals include the provision (a) Defence
for: [UPSC CSE Pre 2009] (b) External Affairs and Commonwealth
1. Full independence for India. Relations
2. Creation of Constitution-making body. (c) Food and Agriculture
(d) None of the above
Which of the statement(s) given above is/are
correct? 43. Who among the following was the President
(a) Only 1 of the Indian National Congress when the
(b) Only 2 resolution of ‘Purna Swaraj’ was passed?
(c) Both 1 and 2 [UPSC CSE Pre 2006]
(d) Neither 1 nor 2 (a) Dadabhai Naoroji
(b) Jawaharlal Nehru
38. Who of the following Prime Ministers sent
(c) Lala Lajpat Rai
Cripps Mission to India?
(d) Surendra Nath Banerjee
[UPSC CSE Pre 2009]
(a) James Ramsay MacDonald 44. Which of the following pairs are correctly
(b) Stanley Baldwin matched?
(c) Neville Chamberlain Movement/ Person Activity
(d) Winston Churchill Satyagraha Associated with
39. During the Indian Freedom Struggle, who 1. Champaran Rajendra Prasad
of the following raised an army called ‘Free 2. Ahmedabad Morarji Desai
Indian Legion’? [UPSC CSE Pre 2008] Mill Workers
(a) Lala Hardayal 3. Kheda Vallabhbhai Patel
(b) Ras Behari Bose Select the correct answer using the code given
(c) Subhash Chandra Bose below: [UPSC CSE Pre 2005]
(d) V.D. Savarkar
(a) 1 and 2
40. Consider the following statements: (b) 2 and 3
Assertion (A): The Congress Ministries in all (c) 1 and 3
the provinces resigned in the year 1939. (d) 1, 2 and 3

PYQ Workbook 140


HISTORY OF MODERN INDIA

45. Consider the following statements: 50. Consider the following statements-
1. Lord Mountbatten was the Viceroy when [UPSC CSE Pre 2005]
the Shimla Conference took place. On the eve of launch of Quit India Movement,
2. Indian Navy Revolt, 1946 took place when Mahatma Gandhi:
the Indian sailors in the Royal Indian 1. Asked Government servants to resign.
Navy at Bombay and Karachi rose against
the Government. 2. Asked the soldiers to leave their posts.
3. Asked the Princess of the Princely states
Which one of the statements given above is/
to accept the sovereignty of their people.
are correct? [UPSC CSE Pre 2005]
(a) 1 only Which of the statements is/are correct?
(b) 2 only (a) 1 and 2
(c) Both 1 and 2 (b) 2 and 3
(d) Neither 1 nor 2 (c) 3 only
(d) 1, 2 and 3
46. In October, 1920, who of the following
headed a group of Indians gathered at 51. Which party was founded by Subhash
Tashkent to set up a Communist Party of Chandra Bose in the Year 1939 after he
India? [UPSC CSE Pre 2005] broke away from the Congress?
(a) H.K. Sarkar [UPSC CSE Pre 2005]
(b) P.C. Joshi (a) Indian Freedom Party
(c) M.C. Chagla (b) Azad Hind Fauj
(d) M.N. Roy (c) Revolutionary Front
47. At which Congress Session was the (d) Forward Bloc
working committee authorized to launch a 52. Which one of the following statements is
programme of Civil Disobedience? correct? [UPSC CSE Pre 2004]
[UPSC CSE Pre 2005] (a) The Constituent Assembly of India was
(a) Bombay elected by the Provincial Assemblies in
(b) Lahore the year 1946.
(c) Lucknow (b) Jawaharlal Nehru, M.A. Jinnah and
(d) Tripura
Sardar Vallabhbhai Patel were members
48. Consider the following statements: of the Constituent Assembly of India.
1. In the First Round Table Conference, Dr. (c) The first session of the Constituent
Ambedkar demanded separate electorates Assembly of India was held in January
for the depressed classes. 1947.
2. In the Poona Pact, special provisions for (d) The Constitution of India was adopted on
representation of the depressed people in 26th January 1950.
the local bodies and civil services were
53. During the Indian Freedom Struggle,
made.
who among the following proposed that
3. The Indian National Congress did not
Swaraj should be defined as Complete
take part in the Third Round Table
Independence free from all foreign control?
Conference.
[UPSC CSE Pre 2004]
Which of the statements given above is/are
correct? [UPSC CSE Pre 2005] (a) Mazharul Haque
(b) Maulana Hasrat Mohani
(a) 1 and 2
(b) 2 and 3 (c) Hakim Ajmal Khan
(c) 1 and 3 (d) Abul Kalam Azad
(d) 1, 2 and 3 54. When Congress leaders condemned the
49. In which one of the following provinces was Montagu-Chelmsford Report, many
the Congress ministry not formed under the moderates left the party to form the:
Act of 1935? [UPSC CSE Pre 2005] [UPSC CSE Pre 2003]
(a) Bihar (a) Swarajya Party
(b) Madras (b) Indian Freedom Party
(c) Orissa (c) Independence Federation of India
(d) Punjab (d) Indian Liberal Federation

141 PYQ Workbook


HISTORY OF MODERN INDIA

55. An important aspect of the Cripps Mission 59. The last opportunity to avoid the partition
of 1942 was: [UPSC CSE Pre 2003] of India was lost with the rejection of:
(a) that all Indian States should join the [UPSC CSE Pre 2002]
Indian Union as a condition to consider (a) Cripps Mission
any degree of autonomy for India. (b) Rajagopalachari Formula
(b) the creation of an Indian Union with (c) Cabinet Mission
Dominion status after the Second World (d) Wavell Plan
War.
(c) the active participation and cooperation 60. The President of Indian National Congress
of the Indian People, communities and at the time of partition of India was–
political parties in the British war efforts [UPSC CSE Pre 2002]
as a condition for granting Independence (a) C. Rajagopalachari
with full Sovereign status to India after (b) J.B. Kriplani
the war. (c) Jawaharlal Nehru
(d) the framing of a constitution for the (d) Maulana Abul Kalam Azad
entire Indian Union, with no separate 61. “In this instance, we could not play off the
constitution for any province, and a Mohammedans against the Hindus.” To
Union Constitution to be accepted by all which one of the following events did this
provinces. remark of Aitchison relate?
56. Who headed the Interim Cabinet formed in [UPSC CSE Pre 2000]
the year 1946? [UPSC CSE Pre 2003] (a) Revolt of 1857
(a) Rajendra Prasad (b) Champaran Satyagraha (1917)
(b) Jawaharlal Nehru (c) Khilafat and Non-Cooperation Movement
(c) Sardar Vallabhbhai Patel (1919-22)
(d) Rajagopalachari (d) August Movement of 1942
57. With reference to Indian freedom struggle, 62. The Balkan Plan for fragmentation of India
which one of the following statements in was the brainchild of– [UPSC CSE Pre 2000]
not correct? [UPSC CSE Pre 2003] (a) W. Churchill
(a) The Rowlatt Act aroused a wave of popular (b) M.A. Jinnah
indignation and led to the Jallianwala (c) Lord Mountbatten
Bagh massacre. (d) V.P. Menon
(b) Subhash Chandra Bose formed the
Forward Bloc. 63. As an alternative to the partition of India,
(c) Bhagat Singh was one of the founders Gandhiji suggested to Mountbatten that
of Hindustan Republican Socialist he– [UPSC CSE Pre 2000]
Association. (a) Postpone granting of independence.
(d) In 1931, the Congress Session at Karachi (b) Invite Jinnah to form the Government.
opposed Gandhi-Irwin Pact. (c) Invite Nehru and Jinnah to form the
Government together.
58. During the Indian freedom struggle, the
(d) Invite the army to take over for some
Khudai Khidmatgars, also known as Red
time.
Shirts called for: [UPSC CSE Pre 2002]
(a) The Union of Pakhtun tribal areas in 64. Which Indian nationalist leader looked
northwest with the Afghanistan. upon a war between Germany and Britain
(b) The adoption of terrorist tactics and as a Godsent opportunity which would
methods for terrorizing and finally enable Indians to exploit the situation to
ousting the colonial rulers. their advantage? [UPSC CSE Pre 1999]
(c) The adoption of communist revolutionist (a) C. Rajagopalachari
ideology for political and social reform. (b) M.A. Jinnah
(d) The Pathan regional nationalist unity and (c) Subhash Chandra Bose
a struggle against colonialism. (d) Jawaharlal Nehru

PYQ Workbook 142


HISTORY OF MODERN INDIA

65. Consider the following statements: 70. What is the correct sequence of the
Assertion (A): Lord Linlithgow described the following events?
August Movement of 1942 as the most serious 1. The August offer
rebellion since Sepoy Mutiny. 2. The I.N.A. trial
Reason (R): There was a massive upsurge of 3. The Quit India Movement
the peasantry in certain areas. 4. The Royal Indian Naval Ratings Revolt
Code: [UPSC CSE Pre 1999] Select the correct answer using the code given
below: [UPSC CSE Pre 1998]
(a) Both (A) and (R) are true, and (R) is the
correct explanation of (A). (a) 1, 3, 2, 4
(b) 3, 1, 2, 4
(b) Both (A) and (R) are true, but (R) is not a
(c) 1, 3, 4, 2
correct explanation of A. (d) 3, 1, 4, 2
(c) (A) is true, but (R) is false.
(d) (A) is false, but (R) is true. 71. Consider the following statements:
The Non-Cooperation Movement led to the:
66. “It made its proposals in May. It still wanted
1. Congress becoming a mass Movement for
a united India. There was to be the Federal
the first time.
Union composed of British provinces.” The
2. Growth of Hindu-Muslims unity.
above quotation is related to:
3. Removal of fear of the British might from
[UPSC CSE Pre 1999] the minds of the people.
(a) Simon Commission 4. British Government’s willingness to grant
(b) Gandhi-Irwin Pact political concessions to Indians.
(c) Cripps Mission Select the correct answer using the code given
(d) Cabinet Mission below: [UPSC CSE Pre 1996]
67. When the Indian Muslim League was (a) 1, 2, 3 and 4 are correct
inducted into the Interim Government in (b) 1, 2 and 3 are correct
1946, Liyaqat Ali Khan was assigned the (c) 1 and 3 are correct
Portfolio of? [UPSC CSE Pre 1998] (d) 3 and 4 are correct
(a) Foreign Affairs 72. The meeting of Indian and British political
(b) Home leaders during 1930-32 in London has
(c) Finance often been referred to as the First, Second
(d) Defence and Third Round Table Conferences. It
would be incorrect to refer to them as such
68. Lord Mountbatten came to India as a because: [UPSC CSE Pre 1996]
Viceroy along with specific instruction to– (a) The Indian National Congress did not
[UPSC CSE Pre 1998] take part in two of them.
(a) Balkanize the Indian sub-continent. (b) Indian parties other than the Indian
(b) Keep India United if possible. National Congress participating in
(c) Accept Jinnah’s demand for Pakistan. the Conference represented sectional
(d) Persuade the Congress to accept partition. interests and not the whole of India.
(c) The British Labour Party had withdrawn
69. The Indian National Congress agreed in from the Conference there making the
1947 to the partition of the country mainly proceeding of the Conference partisan.
because– [UPSC CSE Pre 1998] (d) It was an instance of a Conference held
(a) The principle of Two-nation Theory was in three sessions and not that of three
then acceptable to them. separate conferences.
(b) It was imposed by the British Government 73. Which one of the following is not correct
and the Congress was helpless in this about the Cabinet Mission Plan?
regard. [UPSC CSE Pre 1996]
(c) They wanted to avoid large-scale (a) Provincial Grouping
communal riots. (b) Interim Cabinet of Indians
(d) India would have otherwise lost the (c) Acceptance of Pakistan
opportunity to attain freedom. (d) Constitution Framing Right

143 PYQ Workbook


HISTORY OF MODERN INDIA

74. Given below are two statements, one I. Poona Pact


labelled as Assertion (A) and the other II. Gandhi-Irwin Agreement
labelled as Reason (R).
III. Cripps Mission
Assertion (A): The British sovereignty
continued to exist in free India. IV. Civil Disobedience Movement
Codes: [U.P.P.C.S. (Pre) 2019]
Reason (R): The British sovereign appointed
the last Governor-General of free India. (a) IV, II, III, I
(b) II, IV, I, III
In the context of the above two statements, (c) IV, II, I, III
which one of the following is correct? (d) III, I, IV, II
[UPSC CSE Pre 1996]
(a) Both (A) and (R) are true, but (R) is the 79. Given below are two statements, one is
correct explanation of (A). labelled as Assertion (A) and the other as
(b) Both (A) and (R) are true, but (R) is not Reason (R). [U.P.P.C.S. (Pre) 2018]
the correct explanation of (A). Assertion (A): In 1946, the Muslim League
(c) (A) is true, but (R) is false. withdrew its acceptance of the Cabinet
(d) (A) is false, but (R) is true. Mission Plan.
75. The radical wing of the Congress Party with Reason (R): The Muslim League joined the
Jawaharlal Nehru as one of its main leaders interim government formed in 1946.
founded the independence for India League Select the correct answer from the codes
in opposition to- [UPSC CSE Pre 1995] given below.
(a) The Gandhi-Irwin Pact (a) Both (A) and (R) are true, but (R) is the
(b) The Home Rule Movement correct explanation of (A).
(c) The Nehru Report (b) Both (A) and (R) are true, but (R) is not
(d) The Montford reforms the correct explanation of (A).
76. Which of the following were the main (c) (A) is true, but (R) is false.
objectives of the Khilafat Movement? (d) (A) is false, but (R) is true.
[UPSC CSE Pre 1993] 80. Who among the following supported Swaraj
1. To rouse anti-British feelings among the Party in Central Legislative Assembly in
Muslim of India. pre-independent India?[UPPCS (Pre) 2017]
2. To reform the Muslim society. (a) M.A. Jinnah
3. To demand separate electorates and (b) Maulana Abul Kalam Azad
preserve the Khilafat. (c) Dr. Rajendra Prasad
4. To save the Ottoman empire and preserve (d) Jawaharlal Nehru
the Khilafat.
81. Who among the following founded the All
Choose the correct answer from the codes India Harijan Sevak Sangh in 1932?
given below: [U.P.P.C.S. (Mains) 2017]
(a) 1 and 2 (a) B.G. Gokhale
(b) 2 and 3 (b) M.K. Gandhi
(c) 3 and 4
(c) B.R. Ambedkar
(d) 1 and 4
(d) None of the above
4.2. Other Examination Previous Year
82. Which one of the following was NOT a
Question cause for starting the Non-Cooperation
77. The name of the foreign journalist who Movement? [U.P.P.C.S. (Mains) 2017]
reported about Satyagraha at Dharasana (a) Question of Khilafat
salt works was- [U.P.P.C.S. (Pre.) 2021] (b) Salt Law
(a) Francis Louis (c) Atrocities in Punjab
(b) Mark Tully (d) Rowlatt Act
(c) Web Miller 83. Which of the following trials attracted
(d) Philip Sprat worldwide publicity and drew sympathetic
78. Arrange the following events in comments from Albert Einstein. H.G.
chronological order and select the correct Wells, Harold Laski and Roosevelt in favour
answer from the codes given below: of the convicts? [U.P.P.C.S. (Pre) 2017]

PYQ Workbook 144


HISTORY OF MODERN INDIA

(a) INA trial 91. Who of the following had regarded the
(b) Lahore conspiracy case Karachi Session of Indian National Congress
(c) Meerut conspiracy case (1931) as the ‘pinnacle of Mahatma Gandhi’s
(d) None of the above popularity ‘and prestige’?
84. In which of the following movements [U.P.P.C.S. (Pre) 2014]
women’s participation is considered to be (a) S.C. Bose
the maximum? [U.P.P.C.S. (Mains) 2016] (b) Sitaramaiya
(a) Non-Cooperation Movement (c) Sardar Vallabh Bhai Patel
(b) Salt Satyagraha (d) Sardar Kishan Singh
(c) Bardoli March 92. Who among the following had seconded the
(d) Quit India Movement resolution on the partition in the meeting
85. What is the name of the party founded by of All India Congress Committee held in
Saumyendra Nath Tagore? New Delhi in 1947? [U.P.P.C.S. (Pre) 2014]
[U.P.P.C.S. (Re. Exam) (Pre) 2015] (a) Govind Ballabh Pant
(a) Indian Bolshevik Party (b) Sardar Vallabhbhai Patel
(b) Revolutionary Communist Party (c) J.B. Kripalani
(c) Bolshevik Leninist Party (d) Abul Kalam Azad
(d) Radical Democratic Party 93. Who of the following took the passing of a
86. The ‘Cabinet-Mission’ of 1946 was led by: resolution on the partition in the meeting
[U.P.P.C.S. (Re. Exam) (Pre) 2015] of the Congress Committee (1947) as a
(a) Sir Pethick-Lawrence “Surrender of Nationalism in favour of
(b) Lord Linlithgow Communalism”? [U.P.P.C.S. (Pre) 2014]
(c) Lord Wavell (a) Dr. Kitchlew
(d) Sir John Simon (b) Purushottam Das Tandon
(c) Jawaharlal Nehru
87. Who headed the Boundary Commission
(d) G.B. Pant
appointed in 1947 in the context of India’s
partition? `[U.P.P.C.S. (Mains) 2015] 94. Which one of the following statements
(a) Mountbatten is not correct about Non-Cooperation
(b) Radcliffe Movement? [U.P.P.C.S. (Main) 2013]
(c) James Bolt (a) The period of the Movement was from
(d) Richardson 1920 to 1921.
88. The ‘Deepavali Declaration’ issued in the (b) Attainment of Swaraj within a year was its
year 1929 was related to the: motto.
[U.P.P.C.S. (Re. Exam) (Pre) 2015] (c) It comprised the programme of the
boycott.
(a) Communal problem
(d) M.A. Jinnah had supported this
(b) Dominion status
Movement.
(c) Labour leaders
(d) Untouchability 95. Who among the following had opposed the
89. Who one of the following had renounced burning of foreign clothes during the Non-
the title of Haziq-ul-Mulk during Khilafat Cooperation Movement?
agitation? [U.P.P.C.S. (Mains) 2014] [U.P.P.C.S. (Mains) 2013]
(a) Maulana Abul Kalam Azad (a) Rabindranath Tagore
(b) Mohammad Ali (b) Subhas Chandra Bose
(c) Shaukat Ali (c) Shaukat Ali
(d) Hakim Ajmal Khan (d) C.R. Das
90. The former name of Harijan Sevak Sangh 96. Who among the following members of
was? [U.P.P.C.S. (Mains) 2014] Simon Commission belonged to the liberal
(a) All India Anti-Untouchability League party? [U.P.P.C.S. (Mains) 2013]
(b) All India Depressed Classes Association (a) Sir John Simon
(c) Depressed Classes Association for Social (b) Major Attlee
Reforms (c) Stephen Walsh
(d) Association of Untouchables (d) Viscount Burnham

145 PYQ Workbook


HISTORY OF MODERN INDIA

97. The statement: “I want world sympathy 103. After, the arrest of Gandhiji during salt
in this battle of Right against Might,” is Satyagraha, who took his place as the leader
associated with: [U.P.P.C.S. (Mains) 2013] of the movement?[U.P.P.C.S. (Mains) 2012]
(a) Non-Cooperation Movement (a) Abbas Taiyabji
(b) Gandhi’s Dandi March (b) Abul Kalam Azad
(c) Individual Satyagraha (c) Jawahar Lal Nehru
(d) Quit India Movement (d) Sardar Patel
98. The Commander-in-Chief of the Indian 104. The Muslim League Annual Session which
Army during Quit India Movement was: gave effect to Jinnah’s Two Nation Theory
[U.P.P.C.S. (Mains) 2013] was held in: [U.P.P.C.S. (Mains) 2012]
(a) Lord Wavell (a) Lahore
(b) Lord Linlithgow (b) Karachi
(c) Lord Mountbatten (c) Bombay
(d) None of the above (d) Lucknow
99. Who among the following had demanded 105. Who of the following was the President of
first the dominion status for India? Indian National Congress for consecutive
[U.P.P.C.S. (Pre) 2013] six years? [U.P.P.C.S. (Mains) 2012]
(a) Rajagopalachari and Sardar Patel (a) Jawahar Lal Nehru
(b) Pt. Motilal Nehru and Govind Ballabh (b) Dadabhai Naoroji
Pant (c) Abul Kalam Azad
(c) Sir Tej Bahadur Sapru and Jaykar (d) Gopal Krishna Gokhale
(d) Jawaharlal Nehru and Jagjeevanram 106. The National Leader who was elected
100. By whom the ‘Quit India’ resolution was President (Speaker) of the Central
moved in the Bombay Session of the Legislative Assembly in 1925 was:
Congress in the year 1942? [U.P.P.C.S. (Pre) 2012]
[U.P.P.C.S. (Pre) 2013] (a) Motilal Nehru
(a) Jawaharlal Nehru (b) C.R. Das
(b) Narendra Deo (c) Vallabhbhai Pate
(c) Rajendra Prasad (d) Vitthalbhai Patel
(d) J.B. Kripalani 107. After the Poona Pact of 1932, Harijan Sewak
101. Select the correct chronological order of Sangh was established. Its President was:
the following events connected with India’s [U.P.P.C.S. (Mains) 2011]
struggle for independence from the code (a) Jagjiwan Ram
given below: (b) Ghanshyam Das Birla
1. Second Round Table Conference (c) B.R. Ambedkar
2. Karachi Session of Indian National (d) Amrit Lal Thakkar
Congress
108. In which of the following session of Indian
3. Execution of Bhagat Singh National Congress, Subhash Chandra Bose
4. Gandhi-Irwin Pact was elected as President for the second
Code: [U.P.P.C.S. (Mains) 2013] time? [U.P.P.C.S. (Mains) 2011]
(a) 1, 2, 3, 4 (a) Haripura Session
(b) 2, 3, 1, 4 (b) Madras Session
(c) 4, 3, 2, 1 (c) Tripuri Session
(d) 3, 4, 2, 1 (d) Calcutta Session
102. Who among the following has pleaded 109. Who among the following had moved the
on the side of the Indian National Army Non-cooperation resolution in the Nagpur
officers in their Red Fort trial? session of the Indian National Congress in
[U.P.P.C.S. (Pre) 2013] 1920? [U.P.P.C.S. (Pre) 2011]
(a) C.R. Das (a) C.R. Das
(b) Motilal Nehru (b) Annie Besant
(c) M.A. Jinnah (c) B.C. Pal
(d) Sir T.B. Sapru (d) Madan Mohan Malviya

PYQ Workbook 146


HISTORY OF MODERN INDIA

110. Which one of the following had proposed a 116. Who represented the Indian National
three-tier polity for India? Congress in the Partition Council headed
[U.P.P.S.C. (GIC) 2010] by Lord Mountbatten?
(a) Nehru Committee [U.P.P.C.S. (Mains) 2009]
(b) Simon Commission 1. Abul Kalam Azad
(c) Cripps Mission 2. Jawaharlal Nehru
(d) Cabinet Mission 3. Sardar Patel
111. The Tenure of the Congress ministries 4. Rajendra Prasad
formed in the elections of 1937 after the Act Select the correct answer from the code given
of 1935 was: [U.P. P.C.S. (Mains) 2010] below:
(a) 20 Months (a) 1 and 2
(b) 22 Months (b) 2 and 3
(c) 24 Months (c) 3 and 4
(d) 28 Months (d) 1 and 4
112. Which one of the following is NOT true 117. The Poorna Swaraj Resolution adopted at
about the First Round Table Conference? the Lahore Session of the Indian National
[U.P.P.C.S. (Mains) 2010] Congress was moved by:
(a) It was held in 1930 [U.P.P.C.S. (Mains) 2009]
(b) It was to discuss the Report of the Simon (a) B.G. Tilak
Commission (b) J.L. Nehru
(c) It was held in London (c) M.K. Gandhi
(d) It was attended by the Congress delegation (d) Sardar Patel
113. Consider the following statements: 118. Who among the following called Subhash
Assertion (A): The Congress boycotted the Chandra Bose as ‘Desh Nayak’?
Simon Commission. [U.P.P.C.S. (Pre) 2009]
Reason (R): The Simon Commission did not (a) Mahatma Gandhi
have a single Indian member. (b) Ram Manohar Lohia
Select the correct answer by using the code (c) Rabindranath Tagore
given below: [U.P.P.C.S. (Pre) 2010] (d) Sardar Vallabhbhai Patel
(a) Both (A) and (R) are true, and (R) is the 119. During the freedom struggle, Aruna
correct explanation of (A). Asaf Ali was a major woman organizer of
(b) Both (A) and (R) are true, and (R) is not underground activities in:
the correct explanation of (A). [U.P.P.C.S. (Pre) 2009]
(c) (A) is true, but (R) is false. (a) Civil Disobedience Movement
(d) (A) is false, but (R) is true. (b) Non-Co-operation Movement
114. Who of the following supported Non- (c) Quit India Movement
Cooperation Movement, but could not see (d) Swadeshi Movement
it’s result– [U.P.P.C.S. (Pre) 2010] 120. Who was the President of Indian National
(a) Bal Gangadhar Tilak Congress during the visit of Cabinet
(b) Lala Lajpat Rai Mission to India?[U.P.P.C.S. (Mains) 2008]
(c) Motilal Nehru (a) Jawaharlal Nehru
(d) Chittaranjan Das (b) J.B. Kripalani
115. Where was Mahatma Gandhi when a raid (c) Maulana Abul Kalam Azad
was carried out by Congress volunteers on (d) Sarojini Naidu
Dharsana Salt Depot? 121. Muslim League observed the ‘Direct Action
[U.P.P.C.S. (Pre) 2010] Day’ on: [U.P.P.C.S. (Spl) (Mains) 2008]
(a) In Yerawada Jail (a) 13 August, 1946
(b) In Sabarmati Jail (b) 14 August, 1946
(c) Agha Khan Palace Poona (c) 15 August, 1946
(d) In Ahmadnagar Fort Jail (d) 16 August, 1946

147 PYQ Workbook


HISTORY OF MODERN INDIA

122. Jiatrang Movement started in: (c) 3 and 4


[U.P.P.C.S. (Pre) 2008] (d) 2 and 4
(a) Nagaland 129. Which of the following institutions were
(b) Tripura founded during the Non-cooperation
(c) Manipur Movement (1920-22)?
(d) Mizoram 1. Kashi Vidyapeeth
123. Who among the following called the 2. Gujarat Vidyapeeth
Movements of Gandhiji as ‘Political 3. Jamia Millia
Blackmail’? [U.P.P.C.S. (Mains) 2007] 4. Kashi Hindu Vishva Vidyalaya
(a) Lord Chelmsford Select the correct answer from the codes
(b) Lord Wavell given below: [U.P.P.C.S. (Mains) 2005]
(c) Lord Linlithgow
(a) 1 and 2 only
(d) Lord Montague
(b) 2 and 3 only
124. The first Trade Union was founded in India (c) 1, 2, and 3 only
in the year 1918 by- (d) All of the above
[U.P.P.C.S. (Mains) 2007] 130. Consider the following statements:
(a) N.M. Joshi
Assertion (A): The Poona Pact defeated the
(b) B.P. Wadia
purpose of Communal Award.
(c) V.V. Giri
(d) S.A. Dange Reason (R): It paved the way for reservation
of seats in the Parliament and the State
125. Who formed the Radical Democratic Party Assemblies for the SC and ST people.
in 1940? [U.P.P.C.S. (Mains) 2007]
Select the correct answer from the code given
(a) Indrasen below: [U.P.P.C.S. (Mains) 2005]
(b) M.N. Roy
(a) Both (A) and (R) are true, and (R) is the
(c) Somendranath Tagore
correct explanation of (A).
(d) Shachindranath Sanyal
(b) Both (A) and (R) are true, but (R) is not a
126. Who became the President of the Indian correct explanation of (A).
National Congress after the resignation of (c) (A) is true, but (R) is false.
Subhash Chandra Bose? (d) (A) is false, but (R) is true.
[U.P.P.C.S. (Pre) 2007] 131. Which of the following statements are true
(a) Maulana Abul Kalam Azad about the Simon Commission?
(b) Pattabhi Sitaramaiya [U.P.P.C.S. (Mains) 2005]
(c) Rajendra Prasad
1. It was appointed to enquire into the
(d) Sardar Patel
working of the 1919 Act.
127. Who of the following presided over the 2. It was headed by Sir John Simon.
Haripura session of the Indian National 3. It recommended a Federal Form of
Congress? [U.P.P.C.S. (Mains) 2006] Government.
(a) C.R. Das 4. It was opposed by the Indian leaders
(b) J.B. Kriplani
Select the correct answer using the code given
(c) S.C. Bose
below:
(d) J.L. Nehru
(a) 1 and 2 only
128. Who among the following decided to launch (b) 1, 2 and 3 only
the Independent Party on 16 December (c) 2, 3and 4 only
1922? (d) All the above
1. Lala Har Dayal
132. The main outcome of 1921-22 Non-
2. Madan Mohan Malviya Cooperation Movement was-
3. Mohammed Ali Jinnah [U.P.P.C.S. (Pre) 2005]
4. Moti Lal Nehru (a) Hindu-Muslim unity
Select the correct answer from the codes (b) More powers to provinces
given below: [U.P.P.C.S. (Mains) 2006] (c) Increase in elected members to the
(a) 1 and 2 Central Legislative Assembly
(b) 2 and 3 (d) Division in the Indian National Congress

PYQ Workbook 148


HISTORY OF MODERN INDIA

133. Who among the following presided over 139. Who among the following resigned from
the Karachi Session of the Indian National the presidency of the Congress for making
Congress? [U.P.P.C.S. (Pre) 2005] Swaraj Party? [U.P.P.C.S. (Spl) (Pre) 2004]
(a) Jawahar Lal Nehru (a) C.R. Das
(b) J.M. Sengupta (b) Motilal Nehru
(c) S.C. Bose (c) Vithalbhai Patel
(d) Vallabhbhai Patel (d) Pherozeshah Mehta
134. Consider the following statements: 140. Lala Lajpat Rai was injured-
[U.P.P.C.S. (Pre) 2005] [U.P.P.C.S. (Spl.) (Pre) 2004]
Assertion (A): The Quit India Movement (a) In lathi charge in protest to Simon
marked the culmination of Indian National Commission
Movement. (b) In lathi charge in protest to Rowlett Act
(c) In lathi charge in protest to Quit Indian
Reason (R): After the Quit India Movement Movement
it was a need of time to find a suitable (d) In lathi charge in protest to Government
mechanism for transfer of power. of India Act
Select the correct answer from the code given
below: 141. Consider the following statements:
(a) (A) and (R) are individually true, and (R) Assertion (A): The salt agitation was
is the correct explanation of (A). launched by Mahatma Gandhi in 1930.
(b) (A) and (R) are individually true, but (R) Reason (R): Mahatma Gandhi’s object was to
is not the correct explanation of (A). make salt available free to the poor.
(c) (A) is true, but (R) is false. In the context of the above statements which
(d) (A) is false, but (R) is true. of the following is correct:
135. The first commander of “Azad Hind Fauj” [U.P.P.C.S. (Pre) 2003]
was– [U.P.P.C.S. (Pre) 2005] (a) Both (A) and (R) are true, and (R) is the
(a) Mohan Singh correct explanation of (A).
(b) Pritam Singh (b) Both (A) and (R) are true, but (R) is not
(c) Subhash Chandra Bose the correct explanation of (A).
(c) (A) is true, but (R) is false.
(d) Shahnawaz Khan
(d) (A) is false, but (R) is true.
136. The soldiers of Garhwal Regiment refused
142. Who warned Gandhiji not to encourage
to fire on the revolutionaries in-
fanaticism of Muslim religious leaders and
[UPPCS (Spl.) (Pre) 2004] their followers: [U.P. P.C.S. (Pre) 2002]
(a) Khilafat Movement (a) Agha Khan
(b) Non-Cooperation Movement (b) Ajmal Khan
(c) Civil Disobedience Movement (c) Hasan Imam
(d) Quit India Movement (d) Mohammad Ali Jinnah
137. Who among of the following was NOT a 143. Who tabled a vote of censure against
member of the interim Cabinet? Gandhiji for calling off the Non-
[U.P.P.C.S. (Spl) (Pre) 2004] Cooperation Movement at the meeting
(a) Jawaharlal Nehru of All India Congress Committee held on
(b) Baldev Singh February 24, 1922, at Delhi?
(c) Ali Zaher [U.P.P.C.S. (Mains) 2002]
(d) B. R. Ambedkar (a) K.T. Shah
138. Which one of the following is correctly (b) Bipin Chandra Pal
matched? [U.P.P.C.S. (Mains) 2004] (c) Subhash Chandra Bose
(a) 1940 - Lahore Session of Indian National (d) Dr. Moonje
Congress 144. Given below is a list of places. Name
(b) 1931 - Execution of Rajguru those where parallel Governments were
(c) 1921 - Commencement of Non- established during ‘Quit India Movement.’
cooperation Movement Select your answer from the code given
(d) 1920 - Rowlatt Satyagraha below the list:

149 PYQ Workbook


HISTORY OF MODERN INDIA

1. Balia 150. Which were not allocated separate electorate


2. Satara and reserved seats by Communal Award of
3. Hazaribagh MacDonald? [U.P.P.C.S. (Pre) 2001]
4. Meerut (a) Muslims
(b) Sikhs
5. Agra (c) Depressed Castes
Code: [U.P.P.C.S. (Mains) 2002] (d) Buddhists
(a) 1 and 2
151. The American publicist who was with
(b) 1, 2 and 3 Mahatma Gandhi during his ‘Quit India’
(c) 2, 3 and 5 movement was: [U.P.P.C.S. (Pre) 2001]
(d) 1, 3 and 4 (a) Louis Fischer
145. In the meeting of All India Committee on (b) William L. Shiver
June 14-15, 1947; who among voted against (c) Web Miller
the partition of India? (d) Negley Farson
[U.P. P.C.S. (Pre) 2002] 152. Consider the following statements:
(a) Abul Kalam Azad Assertion (A) : Britain made India free in
(b) Khan Abdul Gaffar Khan 1947.
(c) Sardar Patel Reason (R) : Britain had become weak during
(d) Gobind Vallabh Pant the second World War.
146. Which Indian revolutionary helped Subhash In the context of the above two statements,
Chandra Bose in the establishment of Azad which of the following is/are correct?
Hind Fauz? [U.P.P.C.S. (Pre) 2002] [U.P.P.C.S. (Pre) 2001]
(a) Batukeshwar Dutt (a) Both (A) and (R) are true, and (R) is the
(b) Ras Bihari Bose correct explanation of (A).
(c) Ram Prasad Bismil (b) Both (A) and (R) are true, but (R) is not a
correct explanation of (A).
(d) Suryasen
(c) (A) is true, but (R) is false.
147. Kanpur conspiracy case was against the (d) (A) is false, but (R) is true.
leaders of: [U.P.P.C.S. (Pre) 2001] 153. The Congress supported the Khilafat
(a) Khilafat Movement Movement mainly for:
(b) Non-cooperation Movement 1. Reinstatement of Caliph
(c) Communist Movement 2. Removal of Caliph
(d) Revolutionary Movement 3. Getting the sympathy of the Muslims
148. Choose the correct sequence of the events 4. Marginalising Jinnah in the Congress
with the help of codes given below- Code: [U.P.P.C.S. (Pre) 2000]
1. Chauri-Chaura incident (a) 1 and 3
2. Suspension of Non-Cooperation (b) 2 and 4
Movement (c) 3 and 4
(d) 1 and 4
3. Bardoli Resolutions
Code: [U.P.P.C.S. (Pre) 2001] 154. Which one of the following began with the
(a) 1, 2, 3 Dandi March? [U.P.P.C.S. (Pre) 2000]
(b) 2, 3, 1 (a) Home Rule Movement
(b) Non-Cooperation Movement
(c) 1, 3, 2
(c) Civil Disobedience Movement
(d) 2, 1, 3
(d) Quit India Movement
149. Who of the following persons called Irwin 155. Who represented Indian Christians in first
and Gandhi ‘The Two Mahatmas’? Round Table Conference held in London?
[U.P.P.C.S. (Pre) 2001] [U.P.P.C.S. (Pre) 2000]
(a) Mira Bahan (a) Rao Bahadur Srinivas
(b) Sarojini Naidu (b) Sir Akbar Haidari
(c) Madan Mohan Malviya (c) Sir A.P. Patro
(d) Jawaharlal Nehru (d) K.T. Paul

PYQ Workbook 150


HISTORY OF MODERN INDIA

156. ‘Depressed Classes League’ was established (c) Hindu-Muslim riots increased
by: [U.P.P.C.S. (Pre) 2000] (d) Hindus were suppressed
(a) Dr. B.R. Ambedkar 163. Who was the President of the Indian
(b) Babu Jagjiwan Ram National Congress when it passed the ‘Quit
(c) N.S. Kajrolkar India’ resolution? [U.P.P.C.S. (Pre) 1996]
(d) Mahatma Jyotiba Phule
(a) Mahatma Gandhi
157. Who has moved a resolution for the creation (b) Maulana Abul Kalam Azad
of Pakistan in the Muslim League Session of (c) Sardar Patel
1940: [U.P.P.C.S. (Pre) 2000] (d) Pt. Jawahar Lal Nehru
(a) M.A. Jinnah
164. The formula for transfer of sovereignty to
(b) Mohammed Iqbal
India in 1947 was known as–
(c) Rahmat Ali
(d) Khaliquzzaman [U.P.P.C.S. (Pre) 1994]
(a) Durand Plan
158. Who among the following gave up his (b) Morley-Minto Reforms
legal practice during the Non-Cooperation (c) Mountbatten Plan
Movement? [U.P.P.C.S. (Pre) 1999] (d) Wavell Plan
(a) Mahatma Gandhi
(b) M.M. Malviya 165. Who was elected as President of the All-
(c) Tej Bahadur Sapru India Khilafat Conference in 1919?
(d) Chittaranjan Das [U.P.P.C.S. (Pre) 1993]
159. Who among the following issued the (a) Mahatma Gandhi
‘Communal Award’? [U.P.P.C.S. (Pre) 1999] (b) Muhammad Ali Jinnah
(c) Maulana Shaukat Ali
(a) Ramsay Macdonald
(b) Stanley Baldwin (d) Motilal Nehru
(c) Neville Chamberlain 166. Motilal Nehru was the leader of Swaraj
(d) Winston Churchill Party. Who of the following was NOT in the
160. Who among the following were associated Party? [U.P.P.C.S. (Pre) 1993]
with the formation of ‘Swaraj Party’? (a) Srinivas Iyer
1. Subhash Chandra Bose (b) Chittaranjan Das
2. C.R. Das (c) Vithalbhai Patel
(d) C. Rajagopalachari
3. Jawaharlal Nehru
4. Motilal Nehru 167. Among the following whose adjournment
Select the correct answer from the codes was proposed in Gandhi-Irwin Pact?
given below: [U.P.P.C.S. (Pre) 1998] [U.P.P.C.S. (Pre) 1993]
(a) 1, 2, 3 and 4 (a) Non-Cooperation Movement
(b) 1, 2 and 3 (b) Khilafat Movement
(c) 2 and 3 (c) Round Table Conference
(d) 2 and 4 (d) Civil Disobedience Movement
161. Who among the following ran ‘Congress 168. What was the policy of Indian National
Radio’ during the ‘Quit-India’ movement: Congress during 2nd World War?
[U.P.P.C.S. (Pre) 1997] [U.P.P.C.S. (Pre) 1993]
(a) Aruna Asaf Ali (a) Cooperation of Britain in the case of
(b) Jai Prakash Narayan assurance of Complete Independence
(b) Active collaboration of British (Britain)
(c) Usha Mehta
(c) Neutrality
(d) None of the above (d) None of the above
162. Which one of the following was the result of 169. Simon Commission in 1928 came to India
the Khilafat movement? with the purpose– [U.P.P.C.S. (Pre) 1990]
[U.P.P.C.S. (Pre) 1996] (a) To consider administrative reform
(a) Hindu-Muslim differences were narrowed (b) To improve Education
down (c) To improve Agricultural sector
(b) Language problem became acute (d) To evaluate Military capacity

151 PYQ Workbook


HISTORY OF MODERN INDIA

170. Who gave the slogan ‘Jai Hind’? (a) Nehru Report
[U.P.P.C.S. (Pre) 1990] (b) Cripps Mission Plan
(a) J.L. Nehru (c) Beveridge Report
(b) S.C. Bose (d) Mountbatten Plan
(c) Bhagat Singh 176. Which one of the following statements
(d) B.G. Tilak about Cripps Mission is NOT correct?
171. Consider the following statements: [CAPF 2017]
1. The growth of socialist tendency in the (a) In March 1942, Cripps persuaded War
national movement led to the foundation Cabinet to agree to a draft declaration
of the Congress Socialist party in 1934 (b) Jawaharlal Nehru and Sardar Patel
under the leadership of Acharya Narendra represented Congress to negotiate with
Deva and Jayaprakash Narayan. the mission
2. In 1936, Jawaharlal Nehru urged the (c) The mission offered Domination status
Congress to accept socialism as its goal after the war
and to bring itself closer to the peasantry (d) The mission offered formation of
and the working class. Constitution making body after the war
Which of the statements given above is/are 177. Which among the following was not the
correct? [CAPF 2021] recommendation of the Nehru Report?
(a) 1 only [CAPF 2017]
(b) 2 only (a) Dominion Status
(c) Both 1 and 2 (b) Separate Electorate
(d) Neither 1 nor 2 (c) Unitary and Democratic Centre
(d) Adult Suffrage
172. Which one of the following is NOT correct
about Mahatma Gandhi’s Dandi Yatra or 178. Which of the following was/were the
Salt Match? [CAPF 2020] features of the Quit India Movement?
(a) It was the first national activity in which [CAPF 2016]
women participated in large numbers. 1. From the very beginning, it was a rural
(b) Gandhiji allowed the women’s revolt.
participation at the request of Kamaladevi 2. It was essentially a moderate movement.
Chattopadhyay. 3. It was marked by the unprecedented use
(c) The march was fully covered by the of violence.
Western press. 4. It was marked by acute state repression.
(d) The march began after the failure of the
First Round Table Conference in London. Select the correct answer using the code given
below.
173. Who among the following were jailed in the (a) 1 only
Kanpur Bolshevik Conspiracy case in 1924? (b) 1 and 2 only
[CAPF 2019] (c) 3 and 4 only
(a) Muzaffar Ahmad, SA Dange. Shaukat (d) 1, 3 and 4
Usmani, Nalini Gupta
179. Which of the following represents the main
(b) Muhammad Ali and Shaukat Usmani phases of the Quit India Movement?
(c) SA Dange and SV Ghate
(d) Muzaffar Ahmad and SS Mirajkar [CAPF 2015]
1. Strikes, boycotts and picketing in urban
174. Who among the following was NOT centres.
associated with the Kakori Conspiracy case? 2. Widespread attacks in rural areas on the
[CAPF 2019] many symbols and means of colonial rule
(a) Ramprasad Bismil and authority. e.g., railways, telegraph
(b) Rajendra Lahiri lines and Government buildings.
(c) Ashfaqulla Khan 3. Formation of Home Rule leagues.
(d) Surya Sen 4. The Karnataka method.
175. According to which one of the following Select the correct answer using the code given
plans/reports, the decision with regard to below.
the partition of Punjab and Bengal was to be (a) 1 and 2 only
taken on the basis of voting of the members (b) 2, 3 and 4
of the respective Legislative Assemblies? (c) 1, 2 and 4
[CAPF 2019] (d) 1 and 4 only

PYQ Workbook 152


HISTORY OF MODERN INDIA

180. Which of the following was/were NOT part/ 184. Who among the following did NOT belong
parts of the RIN Mutiny? [CAPF 2015] to the group of ‘No-Changers’?
1. Indian National Army [CDS 2022 (I)]
2. HMIS Talwar (a) M.A. Ansari
3. Hunger strike by naval ratings (b) Vithalbhai Patel
4. Lord Irwin (c) Chakravarti Rajagopalachari
(d) Kasturi Ranga Iyengar
Select the correct answer using the code given
below. 185. Which one of the following statements
(a) 1 and 2 about the Gandhi-Irwin Pact of 1931 is
(b) 2 and 3 correct? [CDS 2021 (I)]
(c) 1 and 4 (a) The Government would not release those
(d) 4 only prisoners who had been non-violent.
(b) The Government would not release
181. The proposals of the Cabinet Mission did those prisoners who had indulged in the
NOT include [CAPF 2015] peaceful picketing of liquor and foreign
1. a three-tier structure for the Union of cloth shops.
India, with both the Provinces and the (c) The Congress did not agree to the
Princely States. suspension of the Civil Disobedience
2. formation of a Constituent Assembly. Movement.
3. creation of a sovereign Pakistan with sit (d) The Congress agreed to take part in the
Muslim-majority provinces. Second Round Table Conference.
Select the correct answer using the code giver 186. On 31st December 1929 in which one
below. of the following Congress Sessions was
(a) 1 and 3 proclamation of Purna Swaraja made?
(b) 3 only [CDS 2020 (I)]
(c) 2 only (a) Ahmedabad,
(d) 1 and 2 (b) Calcutta
(c) Lahore
182. Which one of the following statements (d) Lucknow
concerning the Freedom and Partition of
India is NOT correct? [CAPF 2015] 187. Which one among the following was
demanded by the All-India Depressed
(a) In February 1947, British Prime Minister
Classes Leaders’ Conference at Bombay in
Attlee announced that the British were
1931? [CDS 2020 (I)]
not going to transfer power to Indians,
(a) Universal adult suffrage
come what may.
(b) Separate electorates for untouchables
(b) Lord Wavell advocated total withdrawal
(c) Reserved seats for the minorities
of the British from India by 31 March,
(d) A unitary state in India
1948.
(c) It was Lord Mountbatten who speeded up 188. Which political party formally accepted
the process of transfer of power. the Cabinet Mission Plan on 6th June
(d) The Plan Balkan was abandoned on 1946, which had rejected the demand for a
account of Nehru’s opposition to it. sovereign Pakistan? [CDS 2018 (II)]
(a) The Hindu Mahasabha
183. Which one of the following statements (b) The Congress
about the Gandhi-Irwin Pact is NOT (c) The Muslim League
correct? [CAPF 2015] (d) The Unionist Party
(a) Gandhi wrote a letter seeking an interview
with the Viceroy. 189. Match List I with List II and select the
(b) Civil Disobedience movement was to be correct answer using the code given below
discontinued. the list.
(c) The imprisoned freedom fighters were to List I (Act/Event) List II (Year)
be released. A. Rowlatt Act 1922
(d) All the lands that had been confiscated
were be returned to the original owners. B. Salt March 1931

153 PYQ Workbook


HISTORY OF MODERN INDIA

C.Chauri Chaura 1930 (c) initiate discussions regarding anti-


Incident imperialism among workers.
(d) help the formation of a Trade Union
D.Second Round 1919
Movement in Punjab.
Table Conference
Codes [CDS 2017 (I)] 193. The Ghadar party, formed in the USA, was
A B C D determined to start a revolt in India. Which
(a) 4 3 1 2 among the following provinces did the
(b) 3 1 4 2 party choose to begin its armed revolt?
(c) 4 3 2 1 [CDS 2014 (II)]
(d) 3 4 2 1 (a) Punjab
(b) Bengal
190. Consider the following statements about
(c) United Provinces
the Salt March:
(d) Bihar
1. The Salt March was deliberately ignored
by the European media. 194. Which among the following was the reason
2. The Salt March was widely covered by the of the resignations of the Indian Ministers
American and European press. in all the provinces in the year 1939?
3. The Salt March was the first nationalist [CDS 2014 (I)]
activity in which women participated in (a) The Governors refused to act as
large numbers. constitutional heads.
4. The Salt March impressed upon the (b) The Centre did not provide the required
British the urgent need to devolve more financial help to provinces.
power to Indians. (c) The Governor-General converted Indian
Which of the statements given above are administration from federal to unitary
correct? [CDS 2017 (I)] one because of the beginning of the
(a) 1, 2 and 4 World War II.
(b) 2, 3 and 4 (d) India was declared a party to the World
(c) 3 and 4 only War II without the consent of the
(d) 2 and 3 only provincial government.
191. In the elections to the provincial legislatures 195. During the Quit India Movement, a girl
in 1937 in British India named Kanaklata Barua led a public
1. only about 10 to 12 per cent of the procession and defying the police, tried to
population had the right to vote. enter the police station but was shot dead.
2. the untouchables had no right to vote. This incident took place at–
3. the Congress won an absolute majority in [R.A.S./R.T.S. (Pre.) 2021]
five out of eleven provinces. (a) Sonitpur
4. the Muslim League won more than 80% (b) Midnapur
of the seats reserved for Muslims. (c) Korapur
Select the correct answer using the codes (d) Gohpur
given below: [CDS 2017 (I)] 196. Choose the correct chronological order of
(a) 1, 3 and 4 the following events:
(b) 1 and 4 only (i) Lucknow Pact
(c) 1 and 3 only
(d) 2, 3 and 4 (ii) Formation of Swaraj Party
(iii) Jallianwala Massacre
192. Which would be the most appropriate
description concerning the Punjab (iv) Death of Bal Gangadhar Tilak
Naujawan Bharat Sabha? It aspired to Select the answer from the following code:
[CDS 2015 (I)] [R.A.S./R.T.S. (Pre) 2018]
(a) do political work among youth, peasants (a) (i), (iv), (iii) & (ii)
and workers. (b) (iv), (iii), (i) & (ii)
(b) spread the philosophy of revolution (c) (i), (iii), (iv) & (ii)
among students. (d) (i), (ii), (iii) & (iv)

PYQ Workbook 154


HISTORY OF MODERN INDIA

197. Consider the following events and select 203. Who among the following had primarily
their correct chronological order from the used the word ‘Pakistan’ for separate State
code given below: for Indian Muslims?
(i) Formation of Naujawan Bharat Sabha [R.A.S./R.T.S. (Pre) 1996]
(a) Sir Mohammad Iqbal
(ii) Formation of Swaraj Party (b) Sir Aga Khan
(iii) The Dandi March (c) M. A. Jinnah
(iv) The Jallianwala Bagh Tragedy (d) Chaudhary Rehmat Ali and his friends
Code: [R.A.S./R.T.S. (Re. Exam) (Pre) 2013] 204. In 1937 elections, Congress secured a clear
(a) (iv), (iii), (ii), (i) majority in the State numbering:
(b) (ii), (i), (iv), (iii) [R.A.S./R.T.S. (Pre) 1996]
(c) (ii), (iv), (iii), (i) (a) Three
(d) (iv), (ii), (i), (iii) (b) Four
(c) Five
198. Where did the Congress Working Committee (d) Six
meet to pass the ‘Quit India Resolution’ of 205. Before 1947, 26th January was called as-
14th July 1942? [R.A.S./R.T.S.(Pre) 2010]
[Jharkhand P.C.S. (Pre.) 2021]
(a) Bombay (a) Republic Day
(b) Wardha (b) Martyr’s Day
(c) Lucknow (c) Constitution Day
(d) Tripura (d) Poorna Swaraj Day
199. Who represented Congress in the Second 206. The “Pakistan Resolution” was drafted by-
Round Table Conference? [Jharkhand P.C.S. (Pre) 2016]
[R.A.S./R.T.S.(Pre) 2008] (a) Rehmat Ali
(a) Jawaharlal Nehru (b) Sikandar Hayat Khan
(b) Moti Lal Nehru (c) Muhammad Ali Jinnah
(d) Fazlul Haque
(c) Abul Kalam Azad
(d) Mahatma Gandhi 207. Mountbatten Plan (June 1947) provided
for a referendum in which of the following
200. The whole dispute between Subhash province(s)? [Jharkhand P.C.S. (Pre) 2016]
Chandra Bose and right-wing, after the (a) Sindh
Tripuri session of Congress, centred round (b) Baluchistan
to the question of- [R.A.S./R.T.S.(Pre) 2008] (c) North-West Frontier Province
(a) Formation of Congress Working (d) All of these
Committee 208. In which Session Indian National Congress
(b) Policy towards princely States passed Complete Independence Resolution?
(c) Attitude towards Central Government [67th BPSC (Pre) (Re-Exam) 2022]
(d) Double membership of Congress Socialist (a) 1920 AD Nagpur
Party members (b) 1924 AD Belgaon
201. In which province Muslim League formed (c) 1929 AD Lahore
(d) 1931 AD Karachi
their government after 1946 elections?
(e) None of the above/More than one of the
[R.A.S./R.T.S.(Pre) 2008] above
(a) Bengal
209. Who was the third Satyagrahi of Individual
(b) North-East province
Satyagraha launched by Mahatma Gandhi
(c) Punjab in 1940? [67th BPSC (Pre) (Re-Exam) 2022]
(d) Bihar
(a) Vinoba Bhave
202. Muslim League celebrated the day of (b) Jawaharlal Nehru
Deliverance in: [R.A.S./R.T.S. (Pre) 1999] (c) Rajendra Prasad
(a) 1939 (d) Brahma Dutt
(b) 1942 (e) None of the above/More than one of the
(c) 1946 above
(d) 1947 210. In which country was the honorific ‘Netaji’

155 PYQ Workbook


HISTORY OF MODERN INDIA

applied to the name of Subhash Chandra 216. Who led cultivators in Bihar during the
Bose? [67th BPSC (Pre) 2022] Non-cooperation Movement?
(a) India [65th BPSC (Pre) 2019]
(b) Italy
(a) Swami Vidyanand
(c) Germany
(d) Japan (b) Raj Kumar Singh
(e) None of the above/More than one of the (c) Shri Krishna Singh
above (d) J.B. Sen
(e) None of the above/More than one of the
211. Subhas Chandra Bose renamed Nicobar
Island as:[67th BPSC (Pre) (Re-Exam) 2022] above
(a) Swaraj Island 217. Gandhiji’s Non-Cooperation Movement
(b) Shaheed Island urged people to avoid alcohol. This
(c) Balidan Island resulted in a serious loss of revenue for the
(d) Navodaya Island
government. The government of a province
(e) None of the above/More than one of the
above circulated a list of prominent individuals
who drank alcohol as a device to persuade
212. Who was allotted the Portfolio of the Labour people to start drinking again. Name the
Department in the Interim Government’s
province: [64th BPSC (Pre) 2018]
Cabinet of 1946 AD?
[67th BPSC (Pre) (Re-Exam) 2022] (a) Andhra Pradesh
(a) Jagjivan Ram (b) Bihar
(b) Dr. Rajendra Prasad (c) Bombay
(c) Dr. Anugraha Narayan Singh (d) Gujarat
(d) Syed Ali Zaheer (e) None of the above/More than one of the
(e) None of the above/More than one of the above
above
218. Who was NOT the member of the Cabinet
213. Who was given the princely portfolio in the Mission? [63rd B.P.S.C (Pre.) 2017]
National Provisional Government?
(a) Pethick-Lawrence
[67th BPSC (Pre) (Re-Exam) 2021]
(b) Wavell
(a) V.P. Menon
(b) Sardar Vallabhbhai Patel (c) Stafford Cripps
(c) Krishna Shahi (d) A.V. Alexander
(d) C. Rajgopalachari (e) None of the above/More than one of the
above
214. All India Azad Dasta at Bakri Ka Tapu was
organised during the Quit India Movement 219. What was the ultimate goal of Gandhiji’s
by: [66th B.P.S.C. Re-Exam (Pre) 2020] Salt Satyagraha? [63rd B.P.S.C (Pre.) 2017]
(a) Ram Manohar Lohia (a) Repeal of Salt laws
(b) Usha Mehta (b) Curtailment of the government’s power
(c) Chittu Pandey
(d) Jayprakash Narayan (c) Economic relief to the common people
(e) None of the above/More than one of the (d) ‘Purna Swaraj’ for India
above (e) None of the above/More than one of the
above
215. The Congress Socialist Party was founded
in 1934 by: 220. Home department was establishment under
[66th B.P.S.C. Re-Exam (Pre) 2020] whose presidentship?
(a) Jawaharlal Nehru and Vinoba Bhave [63rd BPSC (Pre) 2017]
(b) Jawaharlal Nehru and Jay Prakash (a) Mahatma Gandhi
Narayan
(b) J.L. Nehru
(c) Jayprakash Narayan and Acharya
Narendra Dev (c) J.B. Pant
(d) Ashok Mehta and Dr. Rajendra Prasad (d) Sardar Patel
(e) None of the above/More than one of the (e) None of the above/More than one of the
above above

PYQ Workbook 156


HISTORY OF MODERN INDIA

221. Who joined Subhas Chandra Bose to 227. Who played an important role in the signing
find All India Forward Bloc and actively of Gandhi-Irwin Pact?
associated with the INA movement? [47th B.P.S.C. (Pre) 2005]
[60th to 62nd B.P.S.C. (Pre) 2016] (a) Motilal Nehru
(a) Jai Prakash Narayan (b) Madan Mohan Malviya
(b) Baikunth Shukla (c) Tej Bahadur Sapru
(c) Sheel Bhadra Yajee (d) Chintamani
(d) Ram Narain Prasad
(e) None of the above/More than one of the 228. The Prime Minister of England during Quit
above India Movement was-
[46th B.P.S.C. (Pre) 2005]
222. Who among the following participated in
the Salt Satyagraha of Gandhi? (a) Chamberlain
(b) Churchill
[60th to 62nd B.P.S.C. (Pre) 2016]
(c) Clement Attlee
(a) Sarojini Naidu (d) Macdonald
(b) Rajkumari Amrit Kaur
(c) Kamla Devi Chattopadhyaya 229. Which Congress President negotiated with
(d) All of these both Cripps Mission and Lord Wavell?
223. Fill in the blanks: The Nehru Report was [47th B.P.S.C. (Pre) 2005]
drafted by a committee headed by ........ and (a) Abul Kalam Azad
the subject was ........ (b) Jawaharlal Nehru
[56th to 59th B.P.S.C. (Pre) 2015] (c) J.B. Kriplani
(a) Motilal Nehru and Jawaharlal Nehru; (d) C. Rajagopalachari
India’s relationship with the British 230. Due to which of the event, Mahatma Gandhi
Empire halted the Non-Cooperation Movement?
(b) Jawaharlal Nehru; Local Self-Government [46th B.P.S.C. (Pre) 2004]
in India
(c) Motilal Nehru; Constitutional (a) Kakori Case
arrangements in India (b) Chauri-Chaura incident
(d) Jawaharlal Nehru; Constitutional (c) Jallianwala Bagh Massacre
arrangements in India (d) Muzaffarpur case
224. Who was the first President of All India 231. The Second Round Table Conference in
Trade Union Congress? London was held in the backdrop of the:
[48th to 52nd B.P.S.C. (Pre) 2008] [46th B.P.S.C. (Pre) 2004]
(a) B.T. Ranade (a) Emerson-Gandhi Pact
(b) Satya Bhakta (b) Hailey-Gandhi Pact
(c) Lala Lajpat Rai (c) Irwin-Gandhi Pact
(d) N.M. Joshi (d) Gandhi-Simon Pact
225. The Revolutionary phase of the trade union 232. The word ‘Sarvodaya’ was primarily used
movement in India was: by: [46th B.P.S.C. (Pre) 2004]
[48th to 52nd B.P.S.C. (Pre) 2008] (a) Mahatma Gandhi
(a) 1939-45 (b) Pandit Nehru
(b) 1926-39 (c) Vinoba Bhave
(c) 1918-26 (d) Jai Prakash Narayan
(d) 1914-18 233. At the Congress Working Committee
226. Why did Mahatma Gandhi support the meeting at Allahabad (April-May, 1947),
Khilafat Movement? who of the following members was able to
[48th to 52nd B.P.S.C. (Pre) 2008] secure a majority for his plan of a Guerilla
(a) The Khalifa had given shelter to Indian War against the Japanese due to India’s
revolutionaries unequivocal stand against Nazism, Fascism
(b) Gandhiji wanted to win the support of the and imperialism?[46th B.P.S.C. (Pre) 2004]
Indian Muslims against the British (a) Subhash Chandra Bose
(c) The Khalifa supported Indian struggle (b) Sarojini Naidu
for freedom (c) Maulana Azad
(d) The Khalifa was a friend of Gandhiji (d) Jawaharlal Nehru

157 PYQ Workbook


HISTORY OF MODERN INDIA

234. Where was Azad Hind Fauz set up in 1943? (a) Rising influence of leaders like Hardayal
[45th B.P.S.C. (Pre) 2001] and Lajpat Rai
(a) Japan (b) Postponement of non-cooperation
(b) Burma movement by Gandhiji
(c) Singapore (c) Influences of foreign events
(d) Malaya (d) Refusal of Indian demand by Britishers
235. Which of the following Indian leaders 242. After the failure of the Civil Disobedience
attended the First Round Table Conference movement, Gandhiji gave importance to:
in London? [44th B.P.S.C. (Pre) 2000] [41st B.P.S.C. (Pre) 1996]
(a) Maulana Mohammad Ali (a) Constructive programmes
(b) Maulana Abul Kalam Azad (b) Limited use of violence
(c) Mahatma Gandhi (c) Negotiation with British
(d) Pt. Jawaharlal Nehru (d) None of the above
236. Who viewed Cripps’ proposal as a post- 243. On which issue did the Second Round Table
dated cheque upon a crashing bank’? Conference fail? [41st B.P.S.C. (Pre) 1996]
[44th B.P.S.C. (Pre) 2000] (a) Communal Delegations
(a) Mahatma Gandhi (b) Granting Dominion Status
(b) Jawaharlal Nehru (c) The date of transfer of ruling power
(c) J.B. Kriplani (d) Postponement of Civil Disobedience
(d) Jaiprakash Narayan Movement
237. Moplah Rebellion in 1921 was an offshoot 244. After passing the resolution of ‘Quit India
of: [43rd B.P.S.C. (Pre) 1999] Movement’ Gandhiji was arrested–
(a) Khilafat Movement [41st B.P.S.C. (Pre) 1996]
(b) Mutiny of 1857
(a) in Yarawada Jail
(c) Swadeshi Movement
(b) in Naini Jail
(d) Non-Cooperation Movement
(c) in the fort of Ahmednagar
238. On 6th July 1942, at the Working Committee (d) in Aga Khan Palace
of the Congress, Mahatma Gandhi discussed
for the first time his ‘Quit India Policy’. The 245. In which context did Jai Prakash Narayan
President of the Committee was: get the recognition of national leader?
[43rd B.P.S.C. (Pre) 1999] [41st B.P.S.C. (Pre) 1996]
(a) Rajagopalachari (a) Quit India Movement
(b) Maulana Abul Kalam Azad (b) Establishment of Congress Socialist Party
(c) Pandit Jawaharlal Nehru (c) Bhoodan Movement
(d) Dr. Annie Besant (d) Making the member of Congress
Executive Committee
239. The famous slogan ‘Swaraj should be for
common people not only for classes’ was 246. Who was the first Indian to be the member
given by- [42nd B.P.S.C. (Pre) 1997] of Communist International?
(a) C.R. Das [40th B.P.S.C. (Pre) 1995]
(b) C. Rajagopalachari (a) M.N. Roy
(c) Motilal Nehru (b) Muzaffar Ahmad
(d) Gopi Nath Saha (c) S.S. Dange
240. Agitation against Chaukidari Tax in (d) None of the above
Begusarai was a part of- 247. In which city was the conference of Khilafat
[42nd B.P.S.C. (Pre) 1997] Committee in 1920 held that requested
(a) Non-cooperation movement Gandhiji to assume the leadership of Non-
(b) Civil disobedience movement co-operation Movement?
(c) Quit India movement [39th B.P.S.C. (Pre) 1994]
(d) Khilafat movement (a) Lucknow
241. During 1923-28, the repetition of (b) Lahore
revolutionary activities in Indian politics (c) Allahabad
was due to: [41st B.P.S.C. (Pre) 1996] (d) Karachi

PYQ Workbook 158


HISTORY OF MODERN INDIA

248. In which working Committee did the 254. Netaji Subhash Chandra Bose became the
Congress adopt the policy of abolishing President of India National Congress by
land ownership? [39th B.P.S.C. (Pre) 1994] defeating which leader?
(a) Working Committee, 1937 [Chhattisgarh P.C.S (Pre) 2013]
(b) Working Committee, 1942 (a) Sardar Vallabhbhai Patel
(c) Working Committee, 1945 (b) J.L. Nehru
(d) Working Committee, 1946 (c) Maulana Abul Kalam Azad
(d) C. Rajagopalachari
249. The result of whose brain is I.N.A. and who (e) P. Sitaramaiya
founded it? [39th B.P.S.C. (Pre) 1994]
255. During which movement the two brothers
(a) Subhash Chandra Bose
Gangaram and Kheemdev were martyred?
(b) Mohan Singh
[Uttarakhand PCS (Pre) 2021]
(c) Chandra Shekhar Azad
(d) Bhagat Singh (a) Salt Satyagraha Movement
(b) Dola-Palki Movement
250. Who among following led the movement (c) Kuli-Begar Movement
for the separate State Pakistan? (d) Quit India Movement
[38th B.P.S.C. (Pre) 1992] 256. Which event occurred first?
(a) Aga Khan [Uttarakhand P.C.S. (Mains) 2006]
(b) Nawab Salim Ullah (a) Dandi March
(c) Liaqat Ali Khan (b) Quit India Movement
(d) Md. Ali Jinnah (c) Arrival of Simon Commission
251. Which of the following were the main events (d) Gandhi-Irwin Pact
of the year 1945: 257. Who was selected as first Satyagrahi in
1. Wavell Plan Individual Satyagraha Movement by
2. Shimla Conference Mahatma Gandhi:
3. Naval Mutiny [Uttarakhand P.C.S. (Mains) 2006]
4. INA Trial (a) Jawahar Lal Nehru
(b) Sardar Patel
Select the correct answer using the codes (c) Sarojini Naidu
given below: (d) Vinoba Bhave
[Chhattisgarh P.C.S. (Pre) 2017]
258. The idea of a separate Nation for Muslims
(a) 1, 2, 3
was given by:
(b) 2, 3, 4
[Uttarakhand P.C.S. (Mains) 2006]
(c) 1, 2, 4
(d) 1, 3, 4 (a) Mohammad Ali Jinnah
(b) Sir Syed Ahmad Khan
252. Who of the following was the President of (c) Sir Mohammad Iqbal
‘All Parties’ Conference held in February (d) Liaqat Ali Khan
1928? [Chhattisgarh P.C.S. (Pre) 2016]
259. The draft of the “Quit India” resolution was
(a) Motilal Nehru prepared by:
(b) Dr. M.A. Ansari [Uttarakhand P.C.S. (Pre) 2002]
(c) Subhash Chandra Bose (a) Jawaharlal Nehru
(d) M.K. Gandhi (b) Mahatma Gandhi
(e) None of these (c) Maulana Abul Kalam Azad
253. In which ground of Bombay the proposal (d) Sarojini Naidu
was passed for ‘Quit India Movement’? 260. On whose suggestions were the Indians kept
[Chhattisgarh P.C.S. (Pre) 2015] out of the Simon Commission?
(a) Marine drive ground [U.P. Lower Sub. (Pre) 2013]
(b) Kala Ghoda ground (a) Lord Reading
(c) Chembur ground (b) Lord Chelmsford
(d) Gowalia Tank (c) Sir John Simon
(e) None of the these (d) Lord Irwin

159 PYQ Workbook


HISTORY OF MODERN INDIA

261. Consider the following reports connected (a) Hind Kesari


with Muslim grievances in the Congress (b) Kaiser-i-Hind
administered provinces: (c) Rai Bahadur
[U.P. Lower Sub. (Pre) 2013] (d) Rt. Honorable
1. Pirpur Report
267. Who hoisted the Indian flag at midnight of
2. Shareef Report
December 31, 1929?
3. Muslim sufferings under Congress rule
[U.P. Lower Sub. (Pre) 2004]
Select the correct chronological order of the
reports from the code given below: (a) Motilal Nehru
(a) 1, 2, 3 (b) Dr. Rajendra Prasad
(b) 2, 1, 3 (c) Mahatma Gandhi
(c) 3, 1, 2 (d) Jawaharlal Nehru
(d) 1, 3, 2 268. Which of the following parties did not
262. Which one of the following had supported support ‘Quit India Movement’?
the Quit India Resolution’ of 1942? [U.P. Lower Sub. (Pre) 2004]
[U.P. Lower Sub. (Pre) 2009] (a) The Hindu Mahasabha
(a) A.K. Azad (b) The Communist Party of India
(b) Rajendra Prasad (c) The Unionist Party of Punjab
(c) Sardar Vallabhbhai Patel (d) All of the above
(d) Jawaharlal Nehru
263. Who amongst the following made regular 269. Who among the following had NOT
broadcasted on Congress Radio operated advocated in INA Red Fort Trial in 1945?
during Quit India Movement? [U.P. Lower Sub. (Pre) 2004]
[U.P. Lower Sub. (Pre) 2008] (a) Bhaulabhai Desai
(a) Jay Prakash Narayan (b) Pandit Jawaharlal Nehru
(b) Subhash Chandra Bose (c) Sardar Vallabh Bhai Patel
(c) Ram Manohar Lohia (d) Dr. Kailash Nath Katju
(d) Sucheta Kriplani
270. For which of the following reason/reasons,
264. Who amongst the following was responsible Swaraj party founded in India–
for the formation of the Independence of
1. Withdraw of non-cooperation movement
India League in 1928?
by Mahatma Gandhi.
1. Jawahar Lal Nehru
2. Subhash Chandra Bose 2. Entering in the council and resection the
3. Acharya Narendra Dev Government of India’s Act of 1919 by
don’t let them work.
4. Jaya Prakash Narayan
3. Repression by the British Government.
Select the correct answer using the code given
below: [U.P. Lower (Spl) (Pre) 2008] 4. It is believed by the Indian’s that
(a) 1, 2 and 3 they should feel the experience of
(b) 2, 3 and 4 administration.
(c) 1 and 2 Select the correct answer using the code given
(d) 3 and 4 below: [U.P Lower Sub. (Pre) 1998]
265. In the interim Government (1946) who held (a) only 1
the Railways Portfolio? (b) 1 and 2
[U.P. Lower Sub. (Pre) 2008] (c) 1, 2 and 3
(a) Baldev Singh (d) 1, 3 and 4
(b) T.T. Chundrigar
271. Which of the following leaders participated
(c) Asaf Ali
(d) Abdul Rab Nishtar in 2nd Round Table Conference-
1. Mahatma Gandhi
266. The title given by the British Government
to Mahatma Gandhi which he surrendered 2. Sarojini Naidu
during the Non-Cooperation Movement 3. Madan Mohan Malviya
was: [U.P. Lower Sub. (Pre) 2004] 4. Maulana Azad

PYQ Workbook 160


HISTORY OF MODERN INDIA

Select correct answer using code given below: (b) Pyarelal Nayyar
[U.P. Lower Sub. (Pre) 1998] (c) Madan Mohan Malaviya
(a) 1 and 2 (d) Jawaharlal Nehru
(b) 1 and 3 275. The head office of ‘Azad Hind Fauj’ is
(c) 1, 2 and 3 situated at– [M.P.P.C.S. (Pre) 2013]
(d) 1, 3 and 4 (a) Tokyo
272. From which ashrama did Mahatma Gandhi (b) Rangoon
start the ‘Dandi March’? (c) Berlin
[MPPCS (Pre) 2021] (d) Delhi
(a) Sabarmati 276. Who among the following officers of Azad
(b) Pavnar Hind Fauj did NOT face famous Red Fort
(c) Sewagram trials? [M.P.P.C.S. (Pre) 2010]
(d) Ramanandiya (a) Gurdayal Singh
(b) Prem Sehgal
273. Who coined the slogan ‘Quit India’?
(c) Mohan Singh
[M.P.P.C.S. (Pre) 2018] (d) Shahnawaz
(a) Mahatma Gandhi
(b) Pt. Jawaharlal Nehru 277. Who among the following held the Office
(c) Yousuf Meher Ali of Speaker in Central Legislative Assembly
(d) Aruna Asaf Ali of British era and the Parliament of free
India? [M.P.P.C.S. (Pre) 2010]
274. Who among the following had NOT (a) Sir Abdur Rahim
participated in the Second Round Table (b) G.V. Mavlankar
Conference? [M.P.P.C.S (Pre) 2017] (c) Ananat Shayanam Ayanger
(a) Mahadev Desai (d) Vitthal Bhai Patel

161 PYQ Workbook


HISTORY OF MODERN INDIA

SOLUTIONS
Malaviya on the Occasaion of opening Banaras Hindu
4.1. UPSC CSE Previous Years’ Questions University. Among the invitees to this event were the princes
1. Solution: (b) and philanthropists whose donations had contributed to the
founding of the BHU. Also present were important leaders of
Exp) Option b is the correct answer.
the Congress, such as Annie Besant.
In March 1942, a mission headed by Stafford Cripps was sent
to India with constitutional proposals to seek Indian support 4. Solution: (d)
for the war. Exp) Option d is the correct answer.
The main proposals of the Cripps mission were as follows: Amritlal Vithaldas Thakkar, popularly known as Thakkar
• An Indian Union with a dominion status would be Bapa was an Indian social worker who worked for upliftment
set up; it would be free to decide its relations with the of tribal people in Gujarat state in India. He became a
Commonwealth and free to participate in the United member of the Servants of India Society founded by Gopal
Nations and other international bodies. Krishna Gokhale in 1905. In 1922, he founded the Bhil
• After the end of the war, a constituent assembly would Seva Mandal. Later, he became the general secretary of the
be convened to frame a new constitution. Members of Harijan Sevak Sangh founded by Mahatma Gandhi in 1932.
this assembly would be partly elected by the provincial Important Tips
assemblies through proportional representation and
Dayaram Gidumal Shahani (30 June 1857 – 7 December
partly nominated by the princes. (Hence, Statement 1 is
1927) or Rishi Dayaram, was a social reformer, judge,
incorrect)
poet, and scholar in British India. He helped establish the
• The British government would accept the new constitution
D.J. Sindh College in Karachi and supported education for
subject to two conditions: (i) any province not willing to
women. He was a scholar of Arabic, Persian, and Sanskrit.
join the Union could have a separate constitution and
He founded the Seva Sadan along with Behramji Malabari.
form a separate Union, and (ii) the new constitution-
He published numerous books including Sata saheliyun
making body and the British government would negotiate
and Chabak mana lai ain rihan (Lashes for the mind,
a treaty to effect the transfer of power and to safeguard
racial and religious minorities. (Hence, Statement 2 is 1923–29).
correct) Gurusaday Dutt (1882–1941) was a civil servant,
• In the meantime, defence of India would remain in folklorist, and writer. He was the founder of the Bratachari
British hands and the governor-general’s powers would Movement in the 1930s. In October 1925, he started
remain intact. Bangalakshmi, a monthly magazine that is still published.
In 1929, he started a magazine called Gramer Daak that
2. Solution: (a)
dealt with agrarian and rural matters of concern.
Exp) Option a is the correct answer.
Dhondo Keshav Karve (18 April 1858 – 9 November
Statement a is correct. The Quit India resolution was 1962), popularly known as Maharshi Karve, was a social
adopted in Bombay in August, 1942, by AICC (All India reformer in India in the field of women’s welfare. He
Congress committee) at Gowalia Tank. founded the first women’s university in India - SNDT
Statement b is incorrect. More Indians were added to Women’s University.
Viceroy’s executive council by Government of India Act
1919. First time, an Indian was added in this council by 5. Solution: (d)
Government of India act 1909. Exp) Option d is the correct answer.
Statement c is incorrect. Congress resigned from ministries Prem Kumar Sehgal, Shah Nawaz Khan and Gurubaksh
in 7 provinces in 1939 as they decided not to support war Singh Dhillon were officers of Indian National Army whose
as India could not be party to a war for democratic freedom trial were held in 1945 at the Red Fort in Delhi. This was the
while that freedom was being denied to India. first trial of INA Prisoners of wars.
Statement d is incorrect. Yes, Cripps proposed a dominion
6. Solution (b)
status to India, but this was not related to 8 August 1942.
Exp) Option b is the correct answer.
3. Solution: (c) Statements 1, 2 and 4 are correct.
Exp) Option c is the correct answer. The Gandhi-Irwin Pact was signed by Gandhiji on behalf of
Mohandas Karamchand Gandhi’s first major public the Congress and by Lord Irwin on behalf of the British India
appearance was at the opening of the Banaras Hindu Government. It placed the Congress on an equal footing with
University (BHU) in February 1916. He was invited by the Government.

PYQ Workbook 162


HISTORY OF MODERN INDIA

The terms of the agreement included the following: that north Indians and Brahmins are Aryans.
• Immediate release of all political prisoners not The Self-Respect Movement was aimed at nothing short of
convicted for violence, a rejection of the brahminical religion and culture which
• withdrawal of emergency ordinances, which were Naicker felt was the prime instrument of exploitation of
promulgated in connection with the Civil Disobedience the lower castes. He sought to undermine the position of
Movement brahmin priests by formalising weddings without brahmin
priests.
• the remission of all fines not yet collected,
• the return of confiscated lands not yet sold to third 8. Solution (b)
parties, Exp) Option b is the correct answer.
• Lenient treatment for those government employees who Statement 1 is correct. Indian immigration to South
had resigned. Africa had begun in 1890 when the White settlers recruited
• The Government also conceded the right to make salt for indentured Indian labour, mainly from South India, to work
consumption to villages along the coast, as also the right on the sugar plantations. They faced racial discrimination in
to peaceful and non-aggressive picketing. South Africa
• Invitation to Congress for participation in Second During the early 1900s, Gandhi was instrumental in
Round Table Conference. getting the indentured labour system abolished in the
Statement 3 is incorrect. However, the Congress demand British Empire through the use of the method of passive
for a public inquiry into police excesses was not accepted, resistance or civil disobedience, named Satyagraha.
but Gandhiji’s insistent request for an inquiry was recorded in Statement 2 is incorrect. The Viceroy of India Lord
the agreement. Gandhi’s demand of commutation of Bhagat Chelmsford, invited various Indian leaders to attend a
Singh and his comrades’ death sentence to life sentence War conference during World War I. Gandhi accepted the
was also not accepted. The Congress, on its part, agreed to invitation and went to Delhi. Gandhi was not happy that
discontinue the Civil Disobedience Movement. leaders like Tilak or the Ali brothers had not been invited to
the conference, so he felt unwilling attended the conference.
7. Solution (d) After meeting the Viceroy, who was very keen that Gandhi
Exp) Option d is the correct answer. should support the resolution on recruiting, Gandhi
Pair 1 is correctly matched. The All-India Anti- supported the government’s resolution on recruiting.
Untouchability League was established in 1932 by M.K. He believed that whole-hearted co-operation with the
Gandhi for elevating the social status of the lower and government will bring India within sight of goal of Swaraj
backward classes. It was born out of the historic Poona Pact as nothing else will.
between Mahatma Gandhi and Babasaheb Ambedkar. Statement 3 is correct. Gandhiji began his salt march from
It was set up as part of his efforts to eradicate the concept of Ahmedabad to Dandi on 12 th March 1930. Gandhi and his
‘untouchability’ from India’s caste system. It was later came selected followers reached Dandi beach and broke the salt
to be known as the Harijan Sewak Sangh. He also started the law by picking up salt left on the shore by the sea.
weekly Harijan in January 1933. He wanted the people to break the salt law openly and to
Pair 2 is correctly matched. Between 1920 and 1940 peasant prepare themselves for non-violent resistance to police
organisations arose. The first organisation to be founded was action.
the Bihar Provincial Kisan Sabha (1929) and in 1936 the All British government took many other leaders under custody.
India Kisan Sabha. In dealing with the breakers of the salt law, the police
All India Kisan Sabha was founded in Lucknow in April resorted to their usual brutal methods and The Indian
1936 with Swami Sahjanand Saraswati as the president and National Congress was declared illegal.
N.G. Ranga as the general secretary. A kisan manifesto was
issued and a periodical under Indulal Yagnik started. The 9. Solution (a)
AIKS and the Congress held their sessions in Faizpur in 1936. Exp) Option a is the correct answer.
The Congress manifesto (especially the agrarian policy) for When Gandhiji and his followers gained controlled of the
the 1937 provincial elections was strongly influenced by the All India Home Rule League. They changed its name to
AIKS agenda. ‘Swaraj Sabha’ (Hindi for Home Rule League) in 1920 .
Pair 3 is correctly matched. E.V. Ramasami Naicker, At the same time they also changed its constitution to allow
Known as Periyar (the respected) , was a strong supporter it to serve as vehicle for Non-cooperation movement. This
of atheism; famous for his anticaste struggle and rediscovery enabled Gandhi to combine cadre of the Home Rule League
of Dravidian identity. Initially a worker of the Congress and Muslim league for the Non-cooperation movement.
party, he started the self-respect movement (1925) and led
10. Solution (c)
the anti-Brahmin movement. He also worked for the Justice
party and later founded Dravidar Kazhagam; opposed to Exp) Option c is the correct answer.
Hindi and domination of north India; propounded the thesis Champaran Satyagraha (1917) was the first peasant

163 PYQ Workbook


HISTORY OF MODERN INDIA

movement to have garnered nationwide attention. It was first without their consent to any democratically elected
Satyagraha movement led by Gandhi in India. government in British India; but at the same time,
Gandhi played a critical role by opening a new phase in it reaffirmed the supremacy of paramountcy with
the national movement by joining it to the great struggle unlimited power-even to suggest constitutional changes
of the Indian peasantry by creating awareness among the in a particular state if there was widespread demand for
peasants against the European Planters. In the subsequent such reforms.
years, Gandhi’s localised movements in Ahmedabad (for 14. Solution (d)
mill workers) and Kheda (where he supported distressed
Exp) Option d is the correct answer.
peasants) thus, creating grounds for the massive nationwide
protests. When the Second World War situation worsened, the British
Government wanted to seek the active cooperation of
11. Solution (d) Indians in the War. To secure this cooperation the British
Exp) Option d is the correct answer. Government sent to India in March 1942 a mission headed
Dyarchy means ‘rule of two’-executive councillors and by a Cabinet minister Stafford Cripps. Cripps announced
popular ministers. The Government of India Act 1919 that the aim of British policy in India was ‘the earliest
introduced dyarchy for the executive at provincial level. The possible realization of self- government in India’. The plan
provincial subjects were further subdivided into transferred unveiled by Cripps promised India Dominion Status and
and reserved. a constitution-making body after the War whose members
would be elected by the provincial assemblies and nominated
12. Solution (c) by the rulers in case of the princely states.
Exp) Option c is the correct answer.
15. Solution (b)
Event 3: The Second Round Table Conference, held in
Exp) Option b is the correct answer.
London in 1931, was convened by the British to discuss
constitutional reforms in India. The INC, led by Gandhi, Statement 1 is correct. The Rowlatt committee was a
participated in the conference. However, the conference ‘Sedition Committee’ appointed in 1918 by the British
failed to resolve major differences between the Indian and Indian Government and it was chaired by Justice Rowlatt.
British delegations, particularly over the issue of separate The recommendation of the committee was the basis for
electorates for minority communities. This led to the Rowlatt Act.
resumption of the Civil Disobedience Movement in India. The Rowlatt Act authorized the authorities to imprison a
Event 2: On 8 August 1942, the All-India Congress person suspected of terrorism for maximum of two years
Committee (AICC) met at the Gowalia Tank Maidan in without trial.
Bombay (now Mumbai) to pass the Quit India Resolution, Statement 2 is correct. In 1919 Gandhiji gave a call for a
demanding the immediate withdrawal of British rule from satyagraha against the Rowlatt Act that the British had just
India. The resolution was drafted by Jawaharlal Nehru and passed. It was first attempt of Gandhiji for a nation-wide
moved by Yusuf Meherally. It was seconded by Vallabhbhai movement. Under the leadership of Gandhi, the newly
Patel and passed unanimously. established Home League was the leading force in this
Event 1: Mutiny in Royal Indian Navy began on 18 satyagraha along with pan-islamists and Satyagraha Sabha
February 1946 in Bombay (now Mumbai) and quickly spread members.
to other parts of British India. The mutiny was eventually Statement 3 is incorrect. The Simon Commission was formed
suppressed by the British, but it had a significant impact on to look into the of the constitutional system facilitated by the
the Indian independence movement. 1919 Act in India. It arrived in India in 1928 and was met
with huge protests as it did not have any Indian member.
13. Solution (d) Gandhi gave call for Rowlatt Satygraha in 1919. Thus, the
Exp) Option d is the correct answer. two demonstrations were separated by a period of almost
The Butler Committee (1927) also known as the Indian a decade.
states committee was set up to examine the nature of
16. Solution (d)
relationship between the princely states and government.
Exp) Option d is the correct answer.
It recommended that:
Within Congress, ‘Congress Socialist Party’ in 1934 was
1. Paramountcy must remain supreme and must fulfil its
founded J P Narayan, Acharya Narendra Dev and Minoo
obligations, adopting and defining itself according to the
Masani. Its members were critical of tactics and strategy
shifting necessities of time and progressive development
of Gandhi as well as of Communists. They wanted to give
of states.
Congress a socialist direction by working from within the
2. States should not be handed over to an Indian Government congress and organising peasants and workers outside the
in British India, responsible to an Indian legislature, congress. The party mainly consisted of three ideologies –
without the consent of states. Maxian Socialism, Fabianism and Gandhian Socialim. The
3. It thus gave concession to Indian States in the form of stood for abolition of Capitalism, Zamindari and princely
a promise that paramountcy would not be transferred states.

PYQ Workbook 164


HISTORY OF MODERN INDIA

Statement 1 incorrect: As it is found post Civil Disobedience 21. Solution (b)


Movement in 1934 they were silent on boycott of british Exp) Option b is the correct answer.
goods and evasion of taxes.
In March 1942, Britain dispatched Sir Stafford Cripps, a
Statement 2 is incorrect: They want to establish government member of the War Cabinet, to India to discuss the British
with socialistic principles not dictatorship of proletariat like
Government’s Draft Declaration. The draft granted India
communists.
Dominion status after the war but otherwise conceded few
Statement 3 is incorrect: It advocated demoractric socialism changes to the British Government Act of 1935. The draft
not seprate electorate. was unacceptable to the Congress Working Committee
17. Solution (b) who rejected it.
Exp) Option b is the correct answer. Thus, Cripps Mission failed to satisfy the demands of
Congress leaders. On 7 to 8 August 1942, the All India
In 1930, Gandhiji declared that he would lead a march
Congress Committee met in Bombay and ratified the ‘Quit
to break the salt law. According to this law, the state had
a monopoly on the manufacture and sale of salt. Mahatma India’ resolution. Gandhi ji called the Cripps Proposals a
Gandhi along with other nationalists reasoned that it was ‘post-dated’ cheque. Muslim league also rejected proposals
sinful to tax salt since it is such an essential item of our food. of Cripps Mission.
The defiance of the salt laws occurred all over India. K. 22. Solution (d)
Kelappan organised salt marches in Malabar, Gopalbandhu
Exp) Option d is the correct answer.
Chaudhuri organised salt salt satyagraha in coastal regions
of Orrisa. The elections to the provincial assemblies were held in
February 1937 in eleven provinces and Congress ministries
C. Rajagopalachari organized a march from Trichinopoly
were formed in Bombay, Madras, Central Provinces, Orissa,
to Vedaranniyam on the Tanjore coast to break the salt law
United Provinces, Bihar and later in the NWFP and Assam
in April 1930. It was modelled on the lines of Dandi March
also.
which was led by Gandhi.
The Congress ministries resigned in protest against
18. Solution (c) Viceroy Lord Linlithgow’s action of declaring India to be
Exp) Option c is the correct answer. a belligerent in the Second World War without consulting
Sir Cyril Radcliffe was the Chairman of the two Border for the Indian people.
demarcation border between India and Pakistan in Punjab
23. Solution (b)
and Bengal.
Exp) Option b is the correct answer.
The boundary demarcation line between India and Pakistan
known as the Radcliffe Line came into force on 17 August Option 1 is incorrect. The Peasants and Workers Party of
1947. The Radcliffe Line was named after its architect, Sir India was founded in 1948 by Marxist leaders such as Keshav
Cyril Radcliffe. Radcliffe divided India into three parts: West Rao Jedhe, Nana Patil and others. Thus, it was not established
Pakistan, East Pakistan and India. by Dr. B.R.Ambedkar.

19. Solution (b) Option 2 is correct. The second political party established
by Ambedkar was the All India Scheduled Castes Federation.
Exp) Option b is the correct answer.
Founded in 1942, it was first all India political party
It was in the Lahore Session (1929) that the Congress for exclusively for Scheduled Castes.
the first time adopted complete independence or Poorna
Option 3 is correct. In 1936, Babasaheb Ambedkar founded
Swaraj as the goal of INC and it was declared that 26 January
1930, would be celebrated as the Independence Day. Such the Independent Labour Party, which contested the 1937
demand was not raised from the Congress platform earlier. Bombay election to the Central Legislative Assembly for
The 1929 Session of Indian National Congress was presided the 13 reserved and 4 general seats, securing 11 and 3 seats
by the Jawaharlal Nehru. respectively.

20. Solution (c) 24. Solution (c)


Exp) Option c is the correct answer. Exp) Option c is the correct answer.
The Indian Statutory Commission, also known as Simon The British Premier Ramsay MacDonald, announced the
Commission was a seven member commission of British Communal Award which provided for separate electorates
parliamentarians formed to look into the of the constitutional for the ‘Depressed Classes’. Gandhi was in Yervada Jail,
system facilitated by the 1919 Act in India and suggest protesting against the separate electorate, he commenced
changes. Major reason behind its opposition was that – all his fast unto death. A solution was found to the communal
its members were white and there was no Indian member. In problem in form of Poona Pact.
1928, when the Simon Commission arrived in India, it was
greeted with black flags and the slogan ‘Go back Simon’. It 25. Solution (a)
was boycotted by majority of political parties in India. Exp) Option a is the correct answer.

165 PYQ Workbook


HISTORY OF MODERN INDIA

Statement 1 is correct. It was in the Lahore Session (1929) • It did not provide for separate electorates for any
that the Congress for the first time raised the demand for community or weightage for minorities.
complete independence. Such demand was not raised from • It advocated federal structure.
the Congress platform earlier. The Lahore Congress accepted
• Separation of state from religion
the new interpretation for Swaraj which was now to mean
complete independence. • A parliamentary form of Government
Statement 2 is incorrect. The rift between the extremists • Residual powers with federal/central government
and moderates was resolved in Lucknow Session held in • Adult franchise
1916. • Redistribution of provincial boundaries on a linguistic
Statement 3 is incorrect. Jinnah’s Two Nations Theory basis
came up in March 1940. It was not rejected in the Lahore
30. Solution: (d)
Session (1929) of the Indian National Congress.
Exp) Option d is the correct answer.
26. Solution (b)
C. Rajagopalachari, also known as Rajaji, was a prominent
Exp) Option b is the correct answer. leader of the Indian National Congress and a close associate
Rowlatt Act was introduced in 1919, indefinitely extending of Mahatma Gandhi. He issued a pamphlet entitled ‘The
‘emergency measures’ (of the Defence of India Regulations Way Out’ in 1944, in which he proposed a solution to the
Act, 1915) enacted during the First World War in order to Constitutional deadlock that had arisen due to the British
control public unrest and root out conspiracy. The Rowlatt government’s refusal to grant independence to India and the
Act authorized the authority to imprison a person Muslim League’s demand for a separate state of Pakistan for
suspected of terrorism for maximum of two years without the Muslims of India.
trial. It allowed the political actvists without jury or in
Important Tips
summary trials.
Rajaji’s proposal, which came to be known as the CR
27. Solution (b) Formula or the Rajaji Formula, was as follows:
Exp) Option b is the correct answer. The Muslim League should endorse the Indian demand for
It was non-violent like earlier movements of Gandhi. The independence and cooperate with the Congress in forming
Quit India movement was launched on August 8 1942. In a provisional interim government for a transitional period.
the yearly hours of August 9, 1942 in a single sweep of all After the end of the Second World War, a commission
top leaders were arrested including Gandhi. This movement should be appointed to demarcate the districts having a
was guided by leaders like Ram Manohar Lohia, Jai Prakesh Muslim majority population in the north-west and eastern
Narayan through underground activities. zones of India.
28. Solution (a) The people of these districts should then decide by a
Exp) Option a is the correct answer. plebiscite whether they wanted to form a separate sovereign
state or remain with the Indian Union.
Statement 1 is correct. Gandhi organized a Satyagraha
to support the peasants of the Kheda district of Gujarat. The terms of the treaty between the two states should
Affected by crop failure due to drought in 1918 and a plague include clauses for safeguarding the rights and interests
epidemic, the peasants of Kheda could not pay the revenue, of the minorities, ensuring free movement and trade, and
and were demanding that revenue collection be relaxed. maintaining common defense and foreign affairs.
Despite their difficulties, British government raised the 31. Solution (c)
revenue.
Exp) Option c is the correct answer
Statement 2 is incorrect. The Permanent Settlement System
Jawaharlal Nehru and Maulana Azad were official Congress
introduced in 1793 was implemented in Bengal, Bihar, and
negotiators for Cripps Mission. On 11 March 1942, Churchill
some part of UP. Government agreed to suspend the tax for
announced that the British government has prepared a plan
the year in question and next.
to overcome India’s political deadlock and Sir Stafford Crips
29. Solution (b) would be sent to India. Sir Cripps was also a member of the
Exp) Option b is the correct answer. British Cabinet and actively supported the Indian National
Movement. The Cripps Mission provided for a separate
The ‘Nehru Report’ was prepared by a committee of the
constitution for other provinces of India, whose hidden goal
All-Parties Conference chaired by Motilal Nehru with
was to divide India into pieces. This proposal was against the
Jawaharlal Nehru acting as secretary. Its recommendations
principle of national unity of India.
included:
• It called for Dominion Status not complete 32. Solution: (b)
Independence. Exp) Option b is the correct answer.
• Unlike the eventual Government of India Act 1935 it Pandit Jawaharlal Nehru drafted the fundamental rights
contained a Bill of Rights. resolution and economic program for the Karachi Session

PYQ Workbook 166


HISTORY OF MODERN INDIA

of the Indian National Congress in 1931. The resolution was Important Tips
adopted by the Congress and became an important milestone
Some of the leaders of the Quit India Movement were:
in the struggle for India’s independence. The resolution
Aruna Asaf Ali: She was a prominent woman leader and
outlined a number of fundamental rights that the Congress
a member of the Congress Socialist Party. She hoisted
believed should be guaranteed to all Indians.
the Indian flag at Gowalia Tank Maidan in Bombay
Important Tips after Gandhi’s arrest. She went underground and led the
These Fundamental Rights in the Karachi resolution movement in Delhi
included: Ram Manohar Lohia: He was a socialist leader and a
• Freedom of speech and expression. follower of Gandhi. He organised underground activities
and published anti-British pamphlets. He was arrested
• Freedom of association and assembly
several times during the movement
• Freedom of conscience and religion Sucheta Kriplani: She was a Congress leader and a disciple
• Equality of all citizens before the law of Gandhi. She worked as a secretary for Gandhi and later
• The right to work, education, and a living wage went underground to lead the movement in Uttar Pradesh.
She also edited a clandestine newspaper called ‘Quit India’
• The right to property
Biju Patnaik: He was an adventurous leader and a pilot.
• The right to participate in the government.
He flew several secret missions to drop leaflets and carry
33. Solution: (a) messages for the movement. He also helped some leaders
escape from British custody.
Exp) Option a is the correct answer.
Kushal Konwar: He was a freedom fighter from Assam. He
The Simon Commission’s recommendations included the
was accused of derailing a train and killing a British officer.
suggestion to abolish the Provincial Dyarchy system that
He was hanged by the British on 15 June 1943, becoming
was in place. The Dyarchy system involved the division of
the only martyr to be executed during the movement.
powers and responsibilities between elected Indian ministers
and appointed British officials in the provincial governments. 36. Solution: (b)
The Commission recommended replacing this system Exp) Option b is the correct answer
with responsible government in the provinces, where the
Gandhi decided to initiate a limited satyagraha in 1940 on
elected ministers would have greater authority and control
an individual basis by a few selected individuals in every
over the administration. This transition aimed to provide
locality. The demands of Satyagrahi would be freedom of
more autonomy to the Indian provinces and move towards a speech to preach against the participation against the war.
responsible and representative form of governance. The Satyagrahi publicly declares it is wrong to help British
34. Solution: (a) with men or money. They would beforehand inform the
district magistrate of the time and place where he or she
Exp) Option a is the correct answer was going to make the anti-war speech. Gandhi chose the
On 25th February, 1920, Ahmedabad Textile Labour Vinoba Bhave as the first Satyagrahi on 17 October 1940
Association was established by Mahatma Gandhi along and Jawahrlal Nehru the second.
with Anasuya Sarabhai, who was a pioneer of the women’s
37. Solution: (b)
labour movement in India.
Exp) Option b is the correct answer
Important Tips
Cripps Mission was sent in India by the British Government
Significant facts about Ahmedabad Mills Strike: in March 1942. It’s aimed to secure Indian cooperation
Mahadev Desai wrote a book “ The Righteous Struggle” and support for British War Effort. Cripps Mission
about Ahmedabad Mills Strike conducted by Mahatma recommended that ‘India would be a dominion associated
Gandhi in 1918 . with the United Kingdom’. Cripps’s mission does not include
complete independence for India. Also, it recommended for
First President of Ahmedabad Textile Labor Association
the creation of a Constituent Assembly to frame a new
was Anasuyabehn Patel with Mahatma Gandhi and Shri
constitution for the country
Shankerlal Banker as the Advisers.
Important Tips
35. Solution: (d)
Cripps Mission:
Exp) Option d is the correct answer.
• The Cripps Mission was a failed attempt in late March
The slogan ‘Do or Die’ is associated with the Quit India 1942 by the British government to secure full Indian
Movement. This movement was launched by Mahatma cooperation and support for their efforts in World War
Gandhi on 9 August 1942 at the Bombay session of the All- II.
India Congress Committee. He gave a ‘do or die’ call to the
• Sir Stafford Cripps was sent by the British cabinet to
people of India in a final push to make the British quit2. The
India to settle terms with Indian leaders who were
movement was also known as August Kranti Movement or
forthwith released.
Bharat Chodo Andolan.

167 PYQ Workbook


HISTORY OF MODERN INDIA

• Cripps proposed Dominion Status after the war but 40. Solution: (a)
his proposal was rejected by all the political leaders. Exp) Option a is the correct answer
• As no party was agreed to accept these proposals, the The Congress Ministries in all the provinces resigned in the
Cripps mission ended in failure. year 1939 because Indian Viceroy Lord Linlithgow declared
• Gandhi said that Cripps’ offer of Dominion Status after India against Germany in World War II and Congress decided
the war was a “post-dated cheque drawn on a failing not to support the world war against Germany.
bank”.
Important Tips
38. Solution: (d) Second World War:
Exp) Option d is the correct answer The Second World War took place between 1939 and 1945
Winston Churchill was the Prime Minister of England when and was the bloodiest conflict in human history.
Cripps Mission arrived in India in the year 1942. Winston It split the world’s nations into two opposing military
Churchill was Prime Minister of the United Kingdom from alliances.
1940 to 1945, during the Second World War, and again from The Allies, led by Britain and her Empire, the United
1951 to 1955. States, the Soviet Union and France, united against the
Axis Powers of Nazi Germany, Fascist Italy and Imperial
39. Solution (c)
Japan.
Exp) Option c is the correct answer.
During the Indian Freedom Struggle, Subhas Chandra 41. Solution: (b)
Bose raised an army called the ‘Free Indian Legion’. The Exp) Option b is the correct answer.
legion was intended to serve as a liberation force for British- Under Shimla Conference, 1945 also known as the
ruled India and was made up of Indian prisoners of war and Wavell Plan, the Viceroy’s Executive Council was to be
expatriates in Europe. Subhas Chandra Bose initiated the reconstituted with different communities getting their due
formation of this legion as part of his efforts to win India’s share in the council and parity was reserved for caste-Hindus
independence by seeking support from Axis powers during and Muslims. All the portfolios, including that of War
World War II. Members, were to be held by the Indian leaders.
Important Tips 42. Solution (c)
Free India Legion: Exp) Option c is correct answer.
The Indian Legion, also known as the Free India Legion
Rajendra Prasad was entrusted with the Food and
or Azad Hind Legion, was a military unit raised during
Agriculture Portfolio in the Interim Government of 1946.
World War II.
He strongly believed in increasing crop production and
Formation and Purpose: The Indian Legion was formed
improving farmers’ lives, and he promoted the slogan “Grow
in 1941 by Subhas Chandra Bose, an Indian independence
leader, during World War II. More Food.” He gave this slogan before Independence. With
his leadership, the Food and Agriculture Ministry started a
Origins: It originated in Berlin as part of Bose’s efforts to
gain support for India’s independence struggle by seeking successful campaign to achieve this goal.
aid from Axis powers. 43. Solution: (b)
Composition: The legion comprised Indian prisoners of
Exp) Option b is the correct answer.
war and expatriates in Europe, including volunteers from
Indian students in Germany and some prisoners of war The Purna Swaraj (Complete Independence) resolution
from North Africa. was passed by the Indian National Congress in its Lahore
Role: Initially intended as an assault group for a German- session in 1929. The session was presided over by Jawaharlal
Indian joint invasion of British India, its primary purpose Nehru. Nehru was a staunch advocate of independence, and
shifted over time. he played a major role in the drafting of the resolution. The
Actions: A portion of the Indian Legion was transferred to Purna Swaraj resolution was a major turning point in the
the Indian National Army in Southeast Asia, while others Indian independence struggle. It marked the end of the
served non-combat duties in the Netherlands and France. gradual approach to independence that had been adopted by
Engagements: It saw action during the Allied advance the Congress in the past. The resolution declared that India
across France, primarily against the French Resistance, would not be satisfied with anything less than complete
and a company was sent to Italy in 1944. independence.
Aftermath: After the surrender of Nazi Germany, legion
members attempted to march to neutral Switzerland but 44. Solution (c)
were captured by Allied troops. They were eventually Option c is the correct answer.
shipped back to India to face charges of treason, though Pair 1 is correctly matched: Champaran Satyagraha, 1917:
many trials were not completed due to public outcry and Gandhiji reached Champaran at the request of Rajkumar
circumstances. Shukla, where he started an effort to free the farmers

PYQ Workbook 168


HISTORY OF MODERN INDIA

suffering from the Tinkathia system. Other leaders involved 47. Solution: (b)
in champaran satyagraha were Rajendra Prasad, JB Kriplani Exp) Option b is the correct answer.
Mahadev Desai, Ramnavmi Prasad, Narahari Parekh,
The working committee of the Indian National Congress
Brajkishore Prasad, Mazhar-ul-Haq, Anugrah Narayan
was authorized to launch a program of Civil Disobedience
Sinha, and Shambhusharan Varma. at the Lahore Session of the Congress in December 1929.
Pair 2 is incorrectly matched: Ahmedabad mill workers The session was presided over by Jawaharlal Nehru. The
strike in 1918: Gandhi was invited by Anasuya Ben Sarabai resolution on Civil Disobedience was moved by Subhas
and her brother Ambalal Sarabhai, leading mill-owner of Chandra Bose and was passed by the Congress.
Ahmadabad towards the cause of the mill owners.
Important Tips
Pair 3 is correctly matched: Kheda satyagraha, 1918:
Resolution passed at Lahore Session in December 1929:
Mahatama Gandhi was assisted by the newly joined
Satyagrahi Sardar Vallabhbhai Patel and other local lawyers • Purna Swaraj- Complete Independence from British
and advocates namely Indulal Yagnik, Shankarlal Banker, Rule.
Mahadev Desai, Narhari Parikh, Mohanlal Pandya and Ravi • The rejection of the two-nation theory, which was
Shankar Vyas. They toured the countryside, organised the the idea that India should be divided into two separate
villagers and gave them political leadership and direction. countries, one for Hindus and one for Muslims.
• The adoption of the economic program of the Congress,
45. Solution: (b)
which called for land reform, industrialization, and the
Exp) Option b is the correct answer. development of cottage industries.
Statement 1 is incorrect: Lord Wavell was the Viceroy of • The establishment of a National Planning Committee
India when the Shimla Conference took place in 1945. to develop a plan for the economic development of
The Shimla Conference was convened in order to agree on India.
the Wavell Plan for Indian self-government, which provided
for separate representations on communal lines. Both the 48. Solution: (d)
plan and the conference failed on account of the Muslim Exp) Option d is the correct answer.
League and the Indian National Congress not coming to an Statement 1 is correct- Ambedkar advocated for a separate
agreement. electorate for the Depressed Classes during the First Round
Statement 2 is correct: The Royal Indian Navy mutiny, also Table Conference due to their distinct identity within
known as the 1946 Naval Uprising, was a failed insurrection Hindu society and their lack of integration, emphasizing
of Indian naval ratings, soldiers, police personnel, and that they needed special political representation to counter
civilians against the British government in India. It began their historical marginalization and untouchability. He
as a strike by naval ratings protesting poor living conditions believed that the existing political structure wouldn’t address
and food. The strike spread and gained support throughout their unique challenges, and a separate political mechanism
British India. was essential to empower them and secure their rightful
place in the constitutional reforms.
46. Solution: (d) Statement 2 is correct- The pact reserved 10% of the seats
Exp) Option d is the correct answer in the Central Public Service Commission and 7.5% of the
In October 1920, M. N. Roy headed a group of Indians seats in the provincial Public Service Commissions for the
gathered at Tashkent to set up a Communist Party of India. Depressed Classes. It also reserved seats for the Depressed
The other prominent persons include Abani Mukherjee and Classes in the local bodies, such as municipal councils and
Mohd. Ali Mohd. Shafiq. district boards.
Statement 3 is correct- The Indian National Congress
Important Tips
participated only in the Second Round Table Conference
Communist Party of India: (September 1931 – December 1931). Mahatma Gandhi
• The ground for the formation of Communist Party of represented the Indian National Congress.
India was prepared by the Second World Congress of
49. Solution: (d)
the Communist Third International (1920).
Exp) Option d is the correct answer.
• The First Congress of the Peoples of the East was
held at Baku in September 1920, specifically aimed at After the result of the Punjab provincial election of 1937, the
fighting imperialism in Asia. Unionist party got majority in the assembly on its own. The
Unionist Party won 95 out of 175 seats, whereas Congress
• This was followed by the formation of the Communist
won 18 seats. The Unionist Party leader Sikandar Hayat
Party of India on 17 October 1920 at Tashkent.
Khan claimed the formation of Government in the State.
• The seven members were M. N. Roy, Evelyn Roy-Trent, Sikandar Hayat Khan led a coalition government with the
Abani Mukherjee, Rosa Fitingov, Mohammad Ali, support of Khalsa National Board, Hindu Election Board,
Mohamad Shafiq and Acharya. Akali, etc.

169 PYQ Workbook


HISTORY OF MODERN INDIA

50. Solution (c) adopted by the assembly on 26th November 1949 (not
Exp) Option c is the correct answer. on 26th January 1950). Whereas, the Constitution came
On the eve of the launch of Quit India Movement, Mahatma into force on 26 January 1950 and this day is celebrated as
Gandhi did not ask government servants to resign or Republic Day.
soldiers to leave their posts. He only asked the princes of
the princely states to accept the sovereignty of their people 53. Solution: (b)
and join the movement.
Exp) Option b is the correct answer.
Statement 1 is not correct: On the eve of the launch of the
Quit India movement, Gandhi Ji asked government servants Maulana Hasrat Mohani, also known as Syed Fazl-
to openly declare their allegiance to the congress and not to ul-Hasan, proposed that Swaraj should be defined as
resign. Complete Independence free from all foreign control. This
Statement 2 is not correct: Gandhi Ji asked the soldiers to proposal was put forth during the Ahmedabad session of
refuse to fire on their own people and not to leave their posts.
the Congress in 1921, where a resolution to define Swaraj
Statement 3 is correct: Gandhi Ji asked the princes of the
in this manner was moved by Maulana Hasrat Mohani and
Princely states to accept the sovereignty of their own
people. Swami Kumaranand. However, Mahatma Gandhi opposed
this demand, and the resolution was defeated.
51. Solution: (d)
Exp) Option d is the correct answer. Important Tips
The Forward Bloc was founded by Subhas Chandra Bose in Maulana Hasrat Mohani:
the year 1939 after he broke away from the Indian National • Maulana Hasrat Mohani was born on 1 January 1875
Congress. This party was formed on May 3, 1939, in Makur in Qasba Mohan, Unnao district, Uttar Pradesh,
Unnao, Uttar Pradesh. Subhas Chandra Bose resigned from with the real name Syed Fazal-ul-Hasan. He later
the presidency of the Indian National Congress on April became popularly known as Hasrat Mohani due to his
29, 1939, after a disagreement with Mohandas Karamchand takhallus, which he used while writing poetry.
Gandhi. The aim of the Forward Bloc was to rally all the
• In 1903, Hasrat Mohani started the magazine
left wing sections within the Congress and develop an
‘Urdu-e-Mualla’ in Aligarh, which published articles
alternative leadership inside the party. Bose became the
against imperialist oppression and policies, promoting
president of the Forward Bloc, and the party’s constitution awareness about the freedom struggle.
and program were approved at a conference in Bombay.
Subhas Chandra Bose traveled around the country to gather • Due to his activism, he was imprisoned again in
support for his new political initiative and also started a 1907, and his magazine was banned by the British
newspaper called “Forward Bloc.” authorities. He was an active member of Congress
until 1907, closely associated with Bal Gangadhar
Important Tips Tilak.
• The Indian Freedom Party was founded by C. • He is credited by some historians with writing the
Rajagopalachari in 1942. slogan ‘Inquilab Zindabad’ in 1921, a slogan later
• The Revolutionary Front was a communist party immortalized by Bhagat Singh.
founded in 1938. • Maulana Hasrat Mohani’s poetry collection is famous
as ‘Kuliyat-e-Hasrat,’ in which he explored themes of
52. Solution: (a)
love, society, politics, and freedom.
Exp) Option a is the correct answer.
Statement 1 is correct: The members of the Constituent 54. Solution: (d)
Assembly were elected by the provincial assemblies Exp) Option d is the correct answer.
by a single transferable-vote system of proportional
representation in 1946. There were a total of 389 members in When Congress leaders criticized the Montagu-
the constituent assembly. Chelmsford Report in 1918, many moderates were
Statement 2 is incorrect: M.A. Jinnah was not the member dissatisfied. Consequently, some moderate leaders left
of the Constituent Assembly. Even Mahatma Gandhi was the party to form the “Indian Liberal Federation” in 1919.
not the member of the Constituent Assembly constituted in
Surendranath Banerjea, Tej Bahadur Sapru, V. S. Srinivasa
1946.
Sastri, and M. R. Jayakar were notable members. They aimed
Statement 3 is incorrect: The first session of the Constituent
to achieve constitutional reforms through dialogues and
Assembly of India was held in 9 December, 1946 (not in
January 1947) in Delhi. Whereas the last session of the cooperation. The Indian Liberal Federation played a role
constituent assembly was held on 24th January 1950. in initiatives like the Nehru Report and the Round Table
Statement 4 is incorrect: The constitution was passed and Conferences for India’s constitutional advancement.

PYQ Workbook 170


HISTORY OF MODERN INDIA

Important Tips • Formulating policies and programs for economic and


Indian Freedom Party or Swatantra Party: social development.
• The Swatantra Party, active from 1959 to 1974, was • Ensuring law and order in the country.
founded by C. Rajagopalachari as a response to • Preparing for the elections to the Constituent
the socialist and statist direction of the Jawaharlal Assembly, which would draft India’s constitution.
Nehru-led Indian National Congress.
57. Solution: (d)
• Its leadership included notable figures like Tanguturi
Exp) Option d is the correct answer.
Prakasam Pantulu, Minoo Masani, N.G. Ranga, and
K.M. Munshi. In 1931, the Congress Session at Karachi did not oppose
the Gandhi-Irwin Pact; in fact, it endorsed the pact. The
• Swatantra advocated a market-based economy and the
Karachi Congress Session, held following the Gandhi-
dismantling of the “Licence Raj,” opposing extreme
Irwin Pact, supported the accord. The session also
laissez-faire policies.
discussed various resolutions related to fundamental rights
• Unlike religious-based parties, it focused on economic and the national economic program, with an emphasis on
principles and stood on the right side of India’s achieving Swaraj and addressing economic issues.
political spectrum. Prime Minister Nehru criticized it
for its perceived association with feudalism. Important Tips
Background of the Karachi Session in 1931:
55. Solution: (b)
The Karachi Congress Session in 1931 took place
Exp) Option b is the correct answer following the Gandhi–Irwin Pact and shortly after the
In order to get the support of India in World War II, Cripps execution of Bhagat Singh.
Mission was sent to India in 1942. Few proposals of this Demonstrations with black flags were organized by the
mission are: Punjab Naujawan Bharat Sabha along Gandhi’s route to
• Setting up of an Indian dominion. This dominion Karachi to protest his failure to secure the commutation
would have the freedom to remain with the British of the death sentence for Bhagat Singh and his comrades.
Commonwealth or to secede from it. It would also be at As per the accord, Congress agreed to call off “civil
liberty to take part in international organisations. disobedience” in exchange for the release of all satyagrahi
• A Constituent Assembly would be formed to frame a prisoners, the freedom to collect salt in coastal areas,
new constitution for the country. This Assembly would and the return of forfeited lands. This compromise was
have members elected by the provincial assemblies and endorsed in the Karachi session.
also nominated by the princes. At the Second Round Table Conference in London,
Gandhi was to represent the Congress’s goal of Swaraj
• Any province unwilling to join the Indian dominion
(self-governance).
could form a separate union and have a separate
constitution. Bhagat Singh’s influence was significant at the Karachi
Congress, where he virtually shaped the agenda and
• The transfer of power and the rights of minorities would defined the draft resolution put together by Jawaharlal
be safeguarded by negotiations between the Constituent Nehru, even after his execution by the British.
Assembly and the British government.
• In the meantime, until this new constitution came into 58. Solution: (d)
force, India’s defence would be controlled by the British Exp) Option d is the correct answer.
and the powers of the Governor-General would remain The Khudai Khidmatgars, also known as Red Shirts, were
unaltered. a nonviolent Pashtun nationalist movement in the North-
West Frontier Province (now Khyber Pakhtunkhwa) of
56. Solution: (b)
British India. They were led by Khan Abdul Ghaffar Khan,
Exp) Option b is the correct answer. also known as Badshah Khan. The Khudai Khidmatgars
Jawaharlal Nehru was the head of the interim government called for the Pathan regional nationalist unity and a
formed on 2nd September, 1946. He was the vice-president struggle against colonialism. They opposed the British rule
of the Council of Ministers, and the powers of a prime in India and they also opposed the partition of India. They
minister were bestowed on him. He held this position until were known for their nonviolent methods of protest, such as
India gained independence on August 15, 1947. boycotts, strikes, and civil disobedience.
Important Tips Important Tips
The Interim Government, established in March 1946 Contribution of Khudai Khidmatgars in India’s freedom
under the Cabinet Mission Plan, had the following struggle:
functions: • In the 1930s, the Khudai Khidmatgars staged non-
• Negotiating with the British government on the terms violent protests against British salt monopolies,
of independence. garnering public support for independence.

171 PYQ Workbook


HISTORY OF MODERN INDIA

• In 1932, their non-cooperation campaign caused diverse political, economic, military, religious and social
officials to resign and British goods to be boycotted. causes. A mutiny in several sepoy companies of the Bengal
army was sparked by the issue of new gunpowder cartridges
• In 1937, they secured a majority in the Northwest
for the Enfield rifle February, 1857. A rumor was spread that
Frontier Province Legislative Assembly, advocating for
the cartridges were made from cow and pig fat.
Pashtun rights.
• In 1947, they opposed India’s partition and boycotted 62. Solution (c)
the referendum for the province’s fate. Exp) Option c is the correct answer.

59. Solution (c) Plan Balkan aimed at fragmentation of India was the
brainchild of Lord Mountbatten, the last Viceroy of India.
Exp) Option c is the correct answer.
Between March and May 1947, Mountbatten, realizing the
The Cabinet Mission was a mission sent by the British Cabinet Mission Plan wasn’t workable, came up with this
government to India in 1946 to try to find a solution to the plan. This plan suggested transferring power to individual
growing communal tensions between Hindus and Muslims. provinces or a confederation, provided one was formed
The mission proposed a plan for a united India with a before the transfer. Punjab and Bengal got to decide on
weak central government and a number of autonomous partition. The units and princely states, now independent
provinces. The Cabinet Mission Plan was rejected by both due to the end of paramountcy, could choose to join India,
the Indian National Congress and the Muslim League. The Pakistan, or stay independent. However, Nehru strongly
Congress felt that the plan did not give enough power to opposed this plan, and it was abandoned.
the central government, while the Muslim League felt that
it did not give enough autonomy to the Muslim-majority 63. Solution: (b)
provinces. Exp) Option b is the correct answer.
Gandhiji proposed to Mountbatten the idea of inviting
Important Tips
Jinnah to form a government as an alternative to India’s
Rajagopalachari Formula (1944)- The formula proposed partition. He thought that if Jinnah was given the
a three-tier federal system, with a central government, opportunity to lead, he might be able to convince the Muslim
provincial governments, and autonomous groupings of League to agree to a united India. However, Mountbatten
provinces. The central government would have control declined Gandhi’s suggestion, as he believed it was too late
over defense, foreign affairs, and communications. to prevent India’s partition. The communal tensions between
The provincial governments would have control over all Hindus and Muslims had reached a critical point, posing a
other matters. The autonomous groupings of provinces substantial risk of civil war if partition was not agreed upon.
would have control over certain specified matters, such as
education and agriculture. 64. Solution (c)
Wavell Plan (1945)- It was proposed by Lord Wavell, Exp) Option c is the correct answer.
the Viceroy of India, in 1945 for the establishment of an Subhash Chandra Bose looked upon a war between Germany
interim government in India. The was rejected by both and Britain as a godsent opportunity which would enable
the Indian National Congress and the Muslim League. Indians to exploit the situation to their advantage. Subhash
Chandra Bose and the socialists argued that the war was an
60. Solution: (b)
imperialist one since both sides were fighting for gaining
Exp) Option b is the correct answer. or defending colonial territories. Therefore the question
Acharya J.B. Kriplani was the President of Indian National of supporting either of the two sides did not arise instead
Congress at the time of partition of India. He presided advantage should be taken of the situation to win freedom by
the 54th session of the Indian National Congress held immediately starting a Civil Disobedience Movement.
at Meerut on November 23-24, 1946. Kripalani was an
65. Solution: (a)
environmentalist, mystic, and independence activist. He
was a follower of Gandhi’s socialist ideals for a long time, Exp) Option a is the correct answer.
but later in life, he shifted to the economically conservative Both (A) and (R) are true, and (R) is the correct explanation
Swatantra Party. of (A).

61. Solution (a) Assertion is correct: Lord Linlithgow, the viceroy of India
from 1936 to 1944, described the August Movement of
Exp) Option a is the correct answer 1942 as the most serious revolt after the Sepoy Mutiny of
In this instance we could not play off the Mohammedans 1857. The August Movement, also known as the Quit India
against the Hindus.” This remark of Aitchison relate to Movement, was a civil disobedience movement launched by
Revolt of 1857. The Indian Rebellion of 1857 is also called Mahatma Gandhi on 9 August 1942, demanding an end to
the Indian Mutiny or the Sepoy Mutiny. It began on 10 May British rule in India.
1857 at Meerut, as a mutiny of sepoys of the British East Reason is also correct and explains Assertion: The reason
India Company’s army. The Indian Rebellion of 1857 had why Lord Linlithgow considered the movement as such a

PYQ Workbook 172


HISTORY OF MODERN INDIA

serious threat was that there was a massive upsurge of the The Indian National Congress agreed to the partition of the
peasantry in certain areas, especially in Bihar, eastern country in 1947 mainly because they wanted to avoid large
Uttar Pradesh, Midnapore district of Bengal, and parts of scale communal riots between Hindus and Muslims. During
Maharashtra. The peasants joined the movement in large the fight for independence, tensions between these two
numbers and participated in various forms of resistance, groups had grown, and many Muslims, led by Muhammad Ali
such as strikes, boycotts, sabotage, attacks on police stations Jinnah and his Muslim League, were demanding a separate
and government offices, cutting of telegraph wires and rail Muslim state called Pakistan. Leaders like Mahatma Gandhi
tracks, and setting fire to government property. and Jawaharlal Nehru in the Congress were worried that
not agreeing some kind of partition would lead to a lot of
66. Solution: (d)
violence, like what happened during the Direct Action Day
Exp) Option d is the correct answer. in 1946. So, even though they did not wanted the division of
Cabinet Mission went to India in 1946 to discuss the transfer India into two countries- India and Pakistan, the Congress
of power from the British government to the Indian political leaders reluctantly agreed to the British plan for partition,
leadership with the aim of preserving India’s unity and called the Mountbatten Plan.
granting its independence. Formed at the initiative of British
Prime Minister Clement Attlee, the mission contained as 70. Solution: (a)
its members, Lord Pethick-Lawrence (Secretary of State Exp) Option a is the correct answer.
for India), Sir Stafford Cripps (President of the Board of
The correct sequence of the events is as follows:
Trade), and A. V. Alexander (First Lord of the Admiralty).
Event 1: The August Offer (1940)
The Cabinet Mission put forward its own proposals in May
1946, which included the idea of a Federal Union composed Event 3: The Quit India Movement (1942)
of British provinces and princely states, with a common Event 2: The I.N.A. Trial (1945)
center for defense, foreign affairs and communications. Event 4: The Royal Indian Naval Ratings Revolt (1946)
The Cabinet Mission hoped that this plan would preserve
the unity of India and accommodate the demands of the Important Tips
Muslim League for a separate homeland. However, the plan INA Trials (1945-46):
was rejected by both the Congress and the Muslim League,
• The Indian National Army trials, also known as
and eventually led to the partition of India in 1947.
the INA trials or Red Fort trials, occurred between
67. Solution (c) November 1945 and May 1946.
Exp) Option c is the correct answer. • They involved the court-martial of officers from the
Liyaqat Ali Khan from the Muslim League Political Party was Indian National Army (INA) on charges including
assigned the Portfolio of Finance in the Interim Government treason, torture, murder, and abetment to murder
of 1946. Liaquat Ali Khan’s 1947 budget marked a significant during World War II.
move towards social change. It included the creation of • Jawaharlal Nehru and the Congress party took
the Commodity Prices Board, tasked with shaping a clear responsibility for the defense of the INA soldiers.
pricing policy. However, conflicts between the Congress and • The defense committee was formed by the Congress
the Muslim League, particularly regarding industrial and Working Committee and included prominent figures
agricultural producers, hindered the board’s intended role. like Bhulabhai Desai, Asaf Ali, and Tej Bahadur
Moreover, the budget introduced measures like an excess Sapru.
profits tax, a wealth tax, and the idea of an Income Tax
• Initially, over 7,600 INA members were set for trial,
Investigation Commission, indicating a commitment to
but the number was significantly reduced due to
investigating financial misconduct by corporations.
difficulties in proving their crimes.
68. Solution: (b) • The first trial involved Colonel Prem Sahgal, Colonel
Exp) Option b is the correct answer. Gurbaksh Singh Dhillon, and Major-General Shah
During the Clement Attlee’s tenure as the Prime Minister Nawaz Khan, former officers of the British Indian
of England, he got Lord Mountbatten appointed by King Army who had joined the INA.
George VI as the final Viceroy of India and tasked him • These three were charged with “waging war against
with overseeing Britain’s withdrawal from India. Attlee’s the King-Emperor,” murder, and abetment of murder,
directive to Lord Mountbatten was to keep India united if while later defendants faced charges related to torture
possible or to prevent it from descending into chaos. This and murder.
was because the British government was facing increasing • The trials garnered significant public attention
pressure from the Indian independence movement, especially and sympathy for the defendants, especially as they
diminishing loyalty to the British crown within the ranks of occurred during the final stages of India’s independence
the Indian Army and Navy personnel, and they wanted to movement.
avoid a bloody and chaotic partition of India.
• Public outcry and unrest among troops prompted
69. Solution (c) Field Marshal Claude Auchinleck to commute the
Exp) Option c is the correct answer. sentences of the first trial’s three defendants.

173 PYQ Workbook


HISTORY OF MODERN INDIA

71. Solution: (b) 74. Solution: (d)


Exp) Option b is the correct answer. Exp) Option d is the correct answer.
Statement 1 is correct: The Non-Cooperation Movement Assertion (A) is false- The British sovereignty did not
marked a significant shift in the Indian National Congress continue to exist in free India. India became a dominion
(INC) as it transitioned from being a relatively moderate on August 15, 1947 and it officially became a Sovereign,
and elite-led organization to a mass movement involving a Democratic, and Republic nation upon the adoption of its
broader section of society in the struggle for independence. Constitution on January 26, 1950.
Statement 2 is correct: The Non-Cooperation Movement Reason (R) is true- The British sovereign appointed the
brought Hindus and Muslims together in a common cause last Governor-General of free India, Lord Mountbatten.
against British rule. The Khilafat Movement, which was However, this does not mean that the British sovereignty
linked to the Non-Cooperation Movement, aimed to support continued to exist in free India. The Governor-General was
the Ottoman Caliphate and garnered support from both a representative of the British monarch, but he was not the
Hindu and Muslim communities. ruler of India. The real power in India was vested in the
Statement 3 is correct: The Non-Cooperation Movement Indian government, which was elected by the Indian people.
instilled a sense of courage and defiance among the Indian 75. Solution (c)
people. The mass participation in protests, strikes, and
Exp) Option c is the correct answer.
boycotts showed that the people were willing to challenge
British authority, leading to a shift in the perception of The radical wing of the Congress Party, led by figures like
British power. Jawaharlal Nehru, established the “Independence for India
League” in opposition to the Nehru Report. The Nehru
Statement 4 is incorrect: While the Non-Cooperation
Report, despite being a significant document advocating
Movement did create pressure on the British government
constitutional reforms, was criticized by the more radical
and forced them to acknowledge the growing discontent, it
elements within the Congress for not explicitly demanding
did not lead to significant political concessions. Instead, the
complete independence and for being willing to settle for
movement was called off after the Chauri Chaura incident,
dominion status. The formation of the “Independence for
and its immediate goals were not fully achieved.
India League” represented a more assertive stance toward
72. Solution: (d) achieving full independence for India.
Exp) Option d is the correct answer. 76. Solution: (d)
The Round Table Conferences were a series of conferences Exp) Option d is the correct answer
held in London between 1930 and 1932 to discuss the future
The Khilafat movement was an agitation by Indian Muslims
of India. There were three sessions of the Round Table
allied with Indian nationalism in the years following World
Conferences, held in 1930, 1931, and 1932. However, it is
War I. Its purpose was to pressure the British government
incorrect to refer to them as the First, Second, and Third
to preserve the authority of the Ottoman Sultan as Caliph
Round Table Conferences because they were not three
of Islam following the breakup of the Ottoman Empire at the
separate conferences. Instead, they were three sessions of
end of the war. In 1920, they published the Khilafat Manifesto,
the same conference.
which called upon the British to protect the caliphate and for
73. Solution (c) Indian Muslims to unite and hold the British accountable
for this purpose.
Exp) Option c is the correct answer.
The Cabinet Mission Plan did not accept the demand of the 4.2. Other Examination Previous Year
Muslim League for a separate homeland for the Muslims Question
in India. Instead, it proposed a federal structure with three
groups of provinces, each having a high degree of autonomy. 77. Solution (c)
The Muslim-majority provinces would form Group B and Exp) Option c is the correct answer.
Group C, while the Hindu-majority provinces would form Webb Miller was an American journalist who reported
Group A. about the Salt Satyagraha at Dharasana salt works. He was
The groups could decide their own internal affairs, but the the only foreign journalist present at the time of the raid.
center would have control over defense, foreign affairs and His report was published in the New York Times and it helped
communications. The Muslim League initially accepted the to bring international attention to the Salt Satyagraha.
plan, hoping that it would pave the way for Pakistan in the Miller’s report was a moving account of the violence that
future, but later rejected it when it realized that the grouping was meted out to the peaceful protesters. He described how
was not compulsory and that the provinces could opt out of the volunteers were beaten with clubs and batons, and how
the groups after the first general election. The Congress also they were thrown to the ground and trampled on. He also
rejected the plan, as it wanted a strong central government described how the women volunteers were treated even more
and opposed the idea of grouping. Therefore, the Cabinet brutally than the men. Miller’s report shocked the world and
Mission Plan failed to resolve the constitutional deadlock it helped to turn public opinion against British rule in
and ultimately led to the partition of India in 1947. India.

PYQ Workbook 174


HISTORY OF MODERN INDIA

78. Solution (c) M.A. Jinnah supported the Swaraj Party in the Central
Exp) Option c is the correct answer. Legislative Assembly in pre-independent India. He backed
The correct chronological order of the events is: the Swaraj Party’s initiatives because it sought to negotiate
with the British for constitutional reforms while demanding
Event 4:. Civil Disobedience Movement (1930-31)
self-governance within the framework of the British
Event 2:. Gandhi-Irwin Agreement (1931) Empire., Jinnah was a proponent of achieving constitutional
Event 1:. Poona Pact (1932) progress through collaborative means with the British, and
Event 3:. Cripps Mission (1942) he recognized that effective participation in the legislative
council was imperative to realize these advancements.
Important Tips
This collaboration aimed to secure self-rule for India and
Poona Pact of 1932: meaningful representation in the legislative decision-making
• The Poona Pact of 1932 emerged as a solution to the processes, aligning with Jinnah’s strategic approach during
British Communal Award, which proposed separate that period.
electorates for various Indian communities.
81. Solution: (b)
• Mahatma Gandhi opposed this, fearing division.
Initially, Dr. B.R. Ambedkar supported separate Exp) Option b is the correct answer.
electorates to uplift the Depressed Classes. However, Mahatma Gandhi founded the Harijan Sevak Sangh in 1932
they reached a consensus known as the Poona Pact, as a part of his constructive program for the eradication
which reserved electoral seats for the Depressed of untouchability. The organization emerged from the All
Classes within the general electorate. India Anti Untouchability League, which Gandhi had earlier
• In the Central Legislature, the Depressed Classes’ established on 30 September 1932, and was subsequently
representation followed joint electorates, with 18% renamed the Harijan Sevak Sangh. The Harijan Sevak Sangh
of seats allocated to the general electorate reserved continues to exist as a non-governmental organization
for them. In provincial legislatures, their reserved dedicated to the well-being of Harijan or Dalit individuals
seats increased from 71 to 147. and the upliftment of the Depressed Class in India.
79. Solution: (b) 82. Solution: (b)
Exp) Option b is the correct answer. Exp) Option b is the correct answer.
The Cabinet Mission Plan was a proposal by the British
The cause that was not a reason for starting the Non-
government in 1946 to find a solution for the constitutional
Cooperation Movement was Salt Law. The Salt Law (Salt
deadlock and the transfer of power in India. The plan
Satyagraha) was a pivotal aspect of Mahatma Gandhi’s
suggested a federal structure with three groups of provinces,
Civil Disobedience Movement of 1930 and not the Non-
each having a high degree of autonomy, and a common center
Cooperation Movement of 1920. It involved the defiance of
for defense, foreign affairs, and communications.
the British monopoly on salt production and distribution.
Assertion (A) is correct: The Muslim League initially
accepted the plan, hoping that it would pave the way for Important Tips
Pakistan in the future, but later withdrew its acceptance Factors Leading to the Non-Cooperation Movement:
when it realized that the grouping was not compulsory and
• Question of Khilafat: The Khilafat movement was
that the provinces could opt out of the groups after the first
initiated to support the Ottoman Caliphate, whose
general election.
authority was under threat from the British following
Reason (R) is correct, but it is not the correct explanation World War I. Mahatma Gandhi allied with this cause
of Assertion (A): The interim government was formed in to garner Muslim support for the Indian freedom
September 1946 as a provisional government to oversee the movement.
transition of India from a British colony to an independent
republic. The interim government was composed of members 83. Solution (c)
from the Constituent Assembly, which was elected in August Exp) Option c is the correct answer
1946. The Congress had a majority in the assembly and also
Meerut conspiracy case was an extremely controversial case
in the interim government. The Muslim League boycotted
that happened in British India in March 1929. Many were
the interim government at first but later joined it in October
arrested including 3 Englishmen for organizing the Indian
1946 after being persuaded by the Viceroy Lord Wavell.
The Muslim League joined the interim government not Railway strike. British Government convicted 27 leaders
because it had withdrawn its acceptance of the Cabinet under the false lawsuit. This trial saw the rising communist
Mission Plan, but because it wanted to have a foothold to movement as convicts used the courtroom as a public
fight for its demand for Pakistan. platform to raise their voices. This trials attracted worldwide
publicity and drew sympathetic comments from Albert
80. Solution: (a) Einstein. H.G. Wells, Harold Laski and Roosevelt in favour
Exp) Option a is the correct answer. of the convicts.

175 PYQ Workbook


HISTORY OF MODERN INDIA

Important Tips The Cabinet Mission was sent to India in February 1946.
The Cabinet Mission had three British cabinet members –
Lahore Conspiracy Case - 1915:
Pethick Lawrence, Stafford Cripps, and A.V. Alexander.
• The Lahore Conspiracy Case trial, also known as the The mission aimed to discuss the transfer of power from
First Lahore Conspiracy Case for which the trials were the British government to the Indian political leadership
held in Lahore in the aftermath of the failed Ghadar with the aim of preserving India’s unity and granting its
conspiracy in 1915. independence.
• The trial was held by a Special tribunal constituted
Important Tips
under the Defence of India Act 1915.
• The principal charge against them was that they • Lord Linlithgow was the Viceroy of India from 1936 to
waged war against the British Empire and wanted to 1943 and was not a part of the Cabinet Mission.
overthrow the British government in India. • Lord Wavell succeeded him as the Viceroy of India
from 1943 to 1947 and was involved in some of the
84. Solution: (b) discussions of the Cabinet Mission, but was not a
Exp) Option b is the correct answer. member of it.
The Salt Satyagraha was a mass protest against the British • Sir John Simon was a British politician who led the
salt monopoly that began in 1930. It was a major turning Simon Commission in 1928 to review the constitutional
point in the struggle for India’s independence. Women played situation in India, but he had no role in the Cabinet
a significant role in the Salt Satyagraha. They participated in Mission.
protests, picketed shops that sold British goods, and even
87. Solution: (b)
went to jail. Some of the prominent women who participated
in the Salt Satyagraha include- Sarojini Naidu, Rajkumari Exp) Option b is the correct answer.
Amrit Kaur, Kamala Devi Chattopadhyaya, Aruna Asaf Sir Cyril Radcliffe, a British lawyer, became the chairman
Ali. of the Boundary Commission during India’s partition in
1947, despite having no prior experience in cartography.
85. Solution: (b) Under his guidance, India was divided into three parts: West
Exp) Option b is the correct answer Pakistan, East Pakistan, and India. The Radcliffe Line,
In the year 1934, the Revolutionary Communist Party of which he established, stretches from the Rann of Kutch in
India (RCPI) was founded by Saumyendranath Tagore. The Gujarat to the border in Jammu & Kashmir, marking the
party was founded by breaking away from the Communist division between India and Pakistan as separate nations.
Party of India (CPI). RCPI led armed uprisings after the Important Tips
independence of India, but later shifted to parliamentary
Some other important boundary lines in the world:
politics. The party is active in the Kerala, West Bengal and
Assam. Boundary Line Division between countries

Important Tips Hindenburg Line Germany and Poland


• The Bolshevik Leninist Party of India Durand Line Pakistan and Afghanistan
• Founders - Edmund Samarakkody, Leslie McMohan Line India and China
Goonewardene, N.M. Perera, Colvin R. de Silva,
17th Parallel North North Vietnam- South Vietnam
Philip Gunawardena.
• Founding Year- 1942 20th Parallel North Sudan- Libya.
• Headquarters - Calcutta 31st Parallel Iraq and Iran
• In India, Radical Democratic Party Mannerheim Line Russia-Finland.
• Founder - MN Roy
• Year-1940 88. Solution: (b)
• The Bolshevik Party of India Exp) Option b is the correct answer.
• Founder - N Dutta Mazumdar The ‘Deepavali Declaration,’ issued in 1929, was related
to the demand for Dominion status for India within the
• Year - 1939
British Empire. Lord Irwin, the Viceroy of India at the time,
• Headquater - Nagpur made this declaration to address the growing demands of
Indian nationalists. It clarified the British government’s
86. Solution: (a)
intention to work towards India achieving dominion
Exp) Option a is the correct answer. status in the future, although it did not specify a timeline.
The ‘Cabinet Mission’ of 1946 was led by Sir Pethick- This declaration marked a significant shift in India’s political
Lawrence. It was an initiative formulated by Clement landscape, with Indian leaders now actively pursuing
Attlee, the then Prime Minister of the United Kingdom. dominion status and a new constitution.

PYQ Workbook 176


HISTORY OF MODERN INDIA

89. Solution: (d) This sentiment was evident when Gandhi was met with black
Exp) Option d is the correct answer flag demonstrations on his way to the session, protesting
why he hadn’t refused to sign the pact over the issue of
Hakim Ajmal Khan had renounced the title of Haziq-ul-
commutation for the trio. The protest was spearheaded
Mulk during the Khilafat Agitation. He was honored with by the Punjab Naujawan Bharat Sabha. Despite these
this title in 1908 by the British Government. complexities, S.C. Bose perceived this session as a high point
Important Tips in Gandhi’s popularity and reputation.
Hakim Ajmal Khan: 92. Solution: (d)
• He was an eminent Indian Unani physician who was Exp) Option d is the correct answer.
a versatile genius, a great scholar, a social reformer, a The All-India Congress Committee meeting was held in
noted freedom fighter, an Unani medical educationist, New Delhi on June 14, 1947, to discuss the Mountbatten
and a founder of scientific research in the Unani Plan for the partition of India. The resolution on the
System of Medicine. partition was moved by Sardar Vallabhbhai Patel, and it was
• He was a trailblazer in educating women about medical seconded by Abul Kalam Azad.
knowledge through his Tibbiya Medical School.
Important Tips
• During the years of Martial Law 1919-20, Hakimji
Contribution of Maulana Abul Kalam Azad to the India’s
worked tirelessly with Swami Shraddhananda to keep freedom struggle:
peace in Delhi, while also witnessing the blatancy of
• He believed in Hindu-Muslim unity and opposed the
state violence.
idea of Partition.
• With the arrests of Gandhi and C. R. Das, the
• Azad supported the Non-Cooperation Movement led
responsibility of steering the movement fell upon him
by Gandhiji in 1920 and joined the Indian National
as he was unanimously made the Congress President.
Congress.
• A spirited Gandhian and champion of non-violent • In 1923, he became the President of the Indian
resistance, Hakim Ajmal Khan had dedicated himself National Congress, making him the youngest person
wholly to the service of the nation. to hold this position at the age of 35.
90. Solution: (a) • In 1930, Azad was arrested for breaking salt laws as
part of Gandhiji’s Salt Satyagraha. He was imprisoned
Exp) Option a is the correct answer.
in Meerut for a year and a half.
The former name of Harijan Sevak Sangh was the All India
• He was an advocate of universal and humanitarian
Anti-Untouchability League. It was founded by Mahatma education, and aimed to harmonize Eastern and
Gandhi in 1932 after the Poona Pact. The word “Harijan” Western ideals, blending spiritual excellence with
means “children of God” in Hindi. Gandhi used this word worldly achievements.
to refer to the Depressed Classes, whom he believed were
• He co-founded Jamia Millia Islamia University in
discriminated against and treated unfairly. Ambedkar left the 1920.
Poona Pact because the word “untouchability” was removed.
• Notable works- Basic Concept of Quran, Ghubar-
The removal of the word “untouchability” was a symbolic
eKhatir, Dars-e-Wafa, and India Wins Freedom.
gesture that angered Ambedkar. He felt that it showed that
• Journal- Al-Hilal and Al-Balagh
the upper caste Hindus were not serious about addressing
the issue of caste discrimination. 93. Solution: (a)
91. Solution: (a) Exp) Option a is the correct answer.
Exp) Option a is the correct answer. Dr Kitchlew took the passing of a resolution on partition in
the meeting of the Congress Committee (1947) meeting as
The person who regarded the Karachi Session of the Indian
a “Surrender of Nationalism in favour of Communalism”.
National Congress (1931) as the “pinnacle of Mahatma
He was a strong opponent of the India’s partition along
Gandhi’s popularity and prestige” was S.C. Bose. The
the religious lines. He believed that it was a betrayal of the
Karachi Session, held from March 26 to 31 in 1931, was
ideals of the Indian National Congress and that it would
a special gathering of the Indian National Congress in
Karachi, where Sardar Vallabhbhai Patel presided. This lead to communal violence. Kitchlew argued that British
session’s primary purpose was to endorse the Gandhi imperialism was India’s main adversary, emphasizing the
Irwin Pact. Notably, Mahatma Gandhi was nominated to need for collective action to defeat it.
represent the Indian National Congress at the Second Round Important Tips
Table Conference. However, the backdrop of this session
More information about Dr. Saifuddin Kitchlew:
was complex due to the recent execution of Bhagat Singh,
Sukhdev, and Rajguru by the British, which triggered • Dr. Saifuddin Kitchlew was a prominent freedom
widespread anger among the masses for Gandhi’s perceived fighter and barrister known for advocating Hindu-
failure to secure the commutation of their death sentences. Muslim unity.

177 PYQ Workbook


HISTORY OF MODERN INDIA

• He gained recognition for leading protests against the • Vernon Hartshorn,


Rowlatt Act, which was passed by the British in 1919. • George Lane-Fox
• He also made significant contributions to the • Donald Howard,
establishment of Delhi’s Jamia Millia Islamia
University. 97. Solution: (b)
• He played a crucial role in the Naujawan Bharat Exp) Option b is the correct answer.
Sabha, an organization founded by Sardar Bhagat The statement is associated with the Mahatma Gandhi.
Singh in 1926. In 1930, Gandhi carried out a remarkable act of civil
• His commitment to revolutionary ideals began during disobedience against a British colonial law that prohibited
his days at Cambridge, where he participated actively Indians from having salt not bought from the government.
in the majlis, a society formed by Indian students to Defying British rule, he led thousands on a 240-mile march
discuss India’s colonial challenges. to the sea, where he collected a pinch of salt. This ignited a
• It was through these gatherings that he crossed paths mass movement across the nation, with people gathering to
with India’s future Prime Minister, Jawaharlal Nehru. make their own salt. Despite Gandhi’s arrest, his followers
marched to take control of the government salt works. The
94. Solution: (d) marchers were met with violence from colonial troops, yet
Exp) Option d is the correct answer. they adhered to Gandhi’s non-violence principle, enduring
Muhammad Ali Jinnah did not support the Non- the attacks without retaliation. Then Gandhi stated, “I want
Cooperation Movement. Despite the movement’s promises, world sympathy in this battle of Right against Might.”
Jinnah considered the launch of the movement futile and 98. Solution: (a)
without any meaningful outcome. He believed in power-
Exp) Option a is the correct answer.
sharing through electoral politics rather than engaging in
mass protests. While Mahatma Gandhi, along with other Lord Wavell was the commander-in-chief of the Indian
leaders, aimed for Swaraj (self-governance) within a year, He Army during the Quit India Movement, which was launched
also believed that NCM might negatively impact the Muslim by Mahatma Gandhi on 8 August 1942 in Bombay. The
community’s interests. As a result, he did not endorse or movement demanded an immediate end to British rule in
support the Non-Cooperation Movement. India and sanctioned a civil disobedience movement against
British authority. The British responded by arresting tens
95. Solution: (a) of thousands of leaders and activists, including Gandhi,
Exp) Option a is the correct answer. and unleashing violence on the masses. The movement
ended in 1945 with the release of the jailed freedom fighters
During the Non-Cooperation Movement, Rabindranath
and the realization by the British that India was no longer
Tagore opposed the burning of foreign clothes. He
governable by them.
expressed his viewpoint in an article, “The Call of
Truth,” highlighting his refusal to obey orders blindly. Important Tips
He expressed that blindly obeying orders was not the way to • Lord Wavell: He was a British field marshal and colonial
fight, and he believed in using clothes to help those in need administrator who served as the penultimate Viceroy
rather than destroy them. He emphasized the importance of of India from 1943 to 1947. He was responsible for
battling the tyranny of blind obedience and illusion, aligning organizing the Simla Conference in 1945, which
with Mahatma Gandhi’s larger fight against oppression. aimed to reach a consensus among the Indian political
parties on the constitutional reforms. He also proposed
96. Solution: (a) the Wavell Plan, which envisaged the formation of an
Exp) Option a is the correct answer. executive council with equal representation of Hindus
and Muslims, and a balanced representation of other
The Simon Commission, officially known as the Indian
minorities.
Statutory Commission, was appointed in 1927 to study
• Lord Mountbatten: He was a British statesman, naval
constitutional reform in British India. Sir John Simon
officer, and cousin of King George VI. He served as
belonged to the Liberal Party among the members of the
the last Viceroy of India and the first Governor-
Simon Commission. He was the chairman of the commission
General of independent India from 1947 to 1948.
and a prominent British politician. He was tasked with overseeing the British withdrawal
Important Tips from India and ensuring a smooth transition to power.
However, he also presided over the partition of India
Members of the Commission: into India and Pakistan, which resulted in widespread
• Sir John Simon, MP for Spen Valley (Liberal, violence and displacement of millions of people. He
chairman) was also involved in the negotiations for the accession
• Clement Attlee, of princely states to either India or Pakistan. He
resigned as Governor-General in 1948 and returned
• Harry Levy-Lawson, to Britain, where he continued his naval career and
• Edward Cadogan, became the Chief of Defence Staff.

PYQ Workbook 178


HISTORY OF MODERN INDIA

99. Solution (c) National Army (INA) who were charged with various
Exp) Option c is the correct answer. offenses including treason, torture, murder, and abetment
The demand for dominion status for India was initially put to murder during World War II. The first trial took place
forth by Sir Tej Bahadur Sapru and M. R. Jayakar. These between November and December 1945. In this trial, which
two prominent figures were members of the Liberal party included defendants Shah Nawaz Khan, Gurubaksh Singh
and played an integral role in advocating for constitutional Dhillon, and Prem Sahgal, the defense was led by Sir Tej
reforms through peaceful dialogue and cooperation. Bahadur Sapru (Sir T.B. Sapru) along with Lt. Col. Horilal
Operating within the framework of the Indian Liberal Varma, who represented the accused officers.
Federation, they actively participated in significant
initiatives like the Nehru Report and the Round Table Important Tips
Conferences, both aimed at advancing India’s constitutional Sir T.B. Sapru:
progress. As skilled mediators, they were instrumental in • Tej Bahadur Sapru (1875 -1949) was an Indian
proposing the concept of dominion status for India as part freedom fighter, lawyer, and politician. He was a key
of their efforts to achieve greater autonomy within the figure in India’s struggle for independence, helping
British Commonwealth. draft the Indian Constitution. He was the leader of the
Important Tips Liberal party in British-ruled India.

• Mukund Ramrao Jayakar (1873-1959) was the first • He was knighted in 1922 and after receiving knighthood
Vice-Chancellor of the University of Poona, a lawyer, he was known as sir Tej Bahadur Sapru.
politician, and leader of the Swarajist Party. He served • He held positions in various legislative councils,
in various capacities, including as a member of the including the Legislative Council of the United
Bombay Legislative Council and Central Legislative Provinces and the Imperial Legislative Council, where
Assembly. Jayakar was a key figure in the Sapru he served as the law member of the Viceroy’s Council.
Committee, contributed to constitutional discussions, • Sapru mediated between different factions, playing
and authored his autobiography, “The Story of my a key role in facilitating the Gandhi-Irwin Pact and
Life.” resolving the dispute between Mahatma Gandhi and
Dr. Ambedkar regarding separate electorates for the
100. Solution: (a)
Depressed Classes, leading to the Poona Pact.
Exp) Option a is the correct answer.
• He became a member of the Privy Council in 1934
Jawaharlal Nehru was the one who moved the ‘Quit India’ and represented the Liberal Party in the Round Table
resolution in the Bombay Session of the Congress in the Conferences in London.
year 1942. He was the president of the All-India Congress
• Sapru supported India’s participation in the Second
Committee at that time, and he presented the resolution
World War without prior consultation with Indians,
on behalf of Mahatma Gandhi, who had drafted it. The
a stance that differed from most Indian political
resolution called for an immediate end to British rule in
activists at the time.
India and authorized Gandhi to lead a non-violent mass
• As a lawyer, he handled significant cases such as the
movement.
Meerut Conspiracy Case and defended rebel soldiers
101. Solution (c) of the Indian National Army.
Exp) Option c is the correct answer. • Sapru’s contributions extended to the Nehru
Event 4: Gandhi- Irwin Pact- It was signed on 5th March, Committee Report of 1928, and although his health
1931. prevented him from serving in the Constituent
Assembly after independence, his legal expertise
Event 3: Execution of Bhagat Singh- Execution of Bhagat
continued to influence constitutional reforms.
Singh, Rajguru, and Sukhdev took place on 23rd March,
• He established the Indian Council of World Affairs
1931.
in 1943 and served as its president until 1949.
Event 2: Karachi Session of Indian National Congress-
It took place on 29th March, 1931 and it was presided by 103. Solution: (a)
Sardar Vallabhai Patel.
Exp) Option a is the correct answer.
Event 1: Second Round Table Conference- It took place
In May 1930, Mahatma Gandhi chose Tyabji, who was
in 17th September, 1931 and Mahatma Gandhi was the
seventy-six years old, to take over as the leader of the Salt
representative of the Indian National Congress.
Satyagraha following Gandhi’s own arrest. Shortly after
102. Solution: (d) assuming leadership, Tyabji was also arrested and detained
Exp) Option d is the correct answer. by the British Indian Government. With admiration, Gandhi
The Red Fort trials, also known as the INA trials, involved and his colleagues affectionately referred to Tyabji as the
court-martial proceedings against officers of the Indian “Grand Old Man of Gujarat.”

179 PYQ Workbook


HISTORY OF MODERN INDIA

104. Solution: (a) 109. Solution: (a)


Exp) Option a is the correct answer. Exp) Option a is the correct answer.
The Lahore Resolution was passed by the All India Muslim
The Non-Cooperation resolution was moved by
League at its annual session in Lahore on March 24, 1940.
The resolution called for the creation of a separate Muslim Chittaranjan Das (C.R. Das) during the Nagpur session of
state in the north-west and north-east of India. It was the Indian National Congress in 1920. This session marked
presented by Muhammad Ali Jinnah, who was the leader a significant shift in the Congress’s approach to the freedom
of the All India Muslim League. The Lahore Session of the struggle, as it decided to pursue Swaraj (self-government)
Muslim League was held in 1940, and it was the first time
through non-cooperation with the British authorities,
that the Two-Nation Theory was put forward. The theory
was based on the idea that Hindus and Muslims were two boycotting foreign goods, schools, and institutions. This
separate nations, and that they could not live together in a approach was intended to be a peaceful and legitimate means
united India. of achieving independence.
105. Solution (c) Important Tips
Exp) Option c is the correct answer. 1920 INC Annual Session:
Maulana Abul Kalam Azad was the president of the Indian The Indian National Congress (INC) held its annual
National Congress for six consecutive years from 1940 session in December 1920 in Nagpur, Maharashtra,
to 1946. He was also the youngest person to become the with C. Vijayaraghavachariar presiding. This pivotal
president of the Indian National Congress at the age of 35 in session saw the adoption of the main resolution on Non-
1923. He was a prominent leader of the Indian independence Cooperation, which advocated for:
movement and a scholar of Islam. He also served as the first
• Boycott of Foreign Goods, Schools, and Courts: The
Minister of Education in independent India.
resolution urged Indians to boycott British goods,
He was the first Minister of Education in the Indian institutions, and the legal system.
government and established the University Grants
Commission (UGC) in 1953. He was also awarded the Bharat • Surrender of Titles: Indians were encouraged to give
Ratna, India’s highest civilian honour, posthumously in 1992. up honorary titles bestowed by the British government
as a sign of non-cooperation.
106. Solution: (d) • Harmony Between Muslims and Hindus: The session
Exp) Option d is the correct answer. emphasized promoting unity and understanding
Vithalbhai Patel, an Indian legislator and political leader, between the two major communities.
was elected as the President (Speaker) of the Central • Congress Working Committee: The formation of
Legislative Assembly in 1925. He co-founded the Swaraj a 15-member committee to guide the Congress’s
Party with leaders like Chittaranjan Das and Motilal Nehru. activities was a significant outcome.
Vithalbhai Patel played an influential role in the Indian
• Shift in Approach: The session marked a shift from
freedom struggle. He was known for his witty speeches and
constitutional means to achieving Swaraj (self-
efforts to challenge British rule through legislative means.
government) to a peaceful, non-cooperative mass
He introduced bills and amendments on various issues,
struggle.
including education and medical practice. As the President
of the Assembly, he established important practices and This session’s reforms elevated the INC’s political standing,
procedures for its functioning. shaping a new era in the freedom struggle’s history. The
Congress’s commitment to non-cooperation and its
107. Solution: (b) emphasis on unity set the stage for transformative
Exp) Option b is the correct answer. actions in India’s fight for independence.
Anti Untouchability League founded by Mahatma Gandhi
110. Solution: (d)
on 30 September 1932 was later renamed Harijan Sevak
Sangh. The aim of the Harijan Sevak Sangh was to uplift Exp) Option d is the correct answer.
the status of the Depressed Classes, also known as the
The Cabinet Mission was a group of three British ministers
Scheduled Castes. The first president of the Harijan Sevak
Sangh was Ghanshyam Das Birla, a prominent industrialist who came to India in 1946 to propose a plan for the transfer
and philanthropist. Amritlal Takkar was its first secretary. of power from the British to the Indian leaders. The Cabinet
Mission proposed a three-tier polity for India, consisting
108. Solution (c)
of a federal union of India, provincial groupings of
Exp) Option c is the correct answer.
states, and provincial governments. The federal union
In the 1939, Tripuri (near Jabalpur in present Madhya
would have limited powers over defence, foreign affairs,
Pradesh) Session, Subhash Chandra Bose was elected as
President of Indian National Congress for the second time. and communications, while the provincial groupings and
He defeated Pattabhi Sitaramayya. governments would have autonomy over other subjects.

PYQ Workbook 180


HISTORY OF MODERN INDIA

Important Tips 114. Solution: (a)

• Nehru Committee: The Nehru Committee was a Exp) Option a is the correct answer.
committee of Indian leaders appointed in 1928 to Bal Gangadhar Tilak was a prominent Indian nationalist
draft a constitution for India. The Nehru Committee and freedom fighter. He was one of the leaders of the Indian
proposed a dominion status for India within the British independence movement, and he supported the Non-
Commonwealth, with a federal system of government Cooperation Movement. However, he died in 1920, before
and a bill of rights. the movement could achieve its goals.

111. Solution: (d) 115. Solution: (a)


Exp) Option d is the correct answer. Exp) Option a is the correct answer.

On the 22nd of October 1939, all the Congress ministries Sarojini Naidu, Imam Sahib, and Manilal assumed the
applied their resignations after 28 months of rule in the responsibility of leading a raid on Dharasana salt works on
21 May 1930. This event took place while Mahatma Gandhi
provinces. The elections under the Government of India
was confined in Yerwada Jail.
Act, 1935 were held in 1936-37 which led to the formation
of Congress ministries. On the ‘ issue of World War II’, Important Tips
Congress decided to quit the government. Dharasana Salt Works in Gujarat:
Important Tips In May 1930, the Dharasana Salt Works in Gujarat, India,
became a symbol of resistance in the Indian independence
Work Done Under Congress Ministries:
struggle led by Mahatma Gandhi and the Indian National
• Eased curbs on civil liberties Congress. A peaceful protest against the British salt
• Restrictions on press lifted monopoly took place there, with around 2,500 nonviolent
• Political prisoners and revolutionaries released protesters marching to the facility. Faced with brutal
beatings by 400 British Indian police officers armed with
• Lifted ban from several illegal organisations, books,
batons, the satyagrahis did not retaliate, highlighting the
and journals
cruelty of British rule. This event garnered global support
• Restoration of pensions of officials associated with the for Indian independence and prompted negotiations
Civil Disobedience Movement (CDM) between the British government and the Indian National
• Controlled communal riots Congress, advancing the cause of independence.
• Encouragement given to indigenous enterprises
116. Solution (c)
• Measures for welfare of Harijans taken—temple entry,
Exp) Option c is the correct answer.
use of public facilities, scholarships, an increase in
their numbers in government service and police, etc Among the given options, Sardar Patel and Rajendra Prasad
represented Indian National Congress in the Partition Council
• Police powers were curbed
headed by Lord Mountbatten. Before the establishment of
112. Solution: (d) the Partition Council, a Partition Committee was formed.
Its members included prominent figures like Vallabh Bhai
Exp) Option d is the correct answer.
Patel, Dr. Rajendra Prasad, Liaqat Ali Khan, and Abdur
The First Round Table Conference was held in London from Rab Nishtar. This committee was later succeeded by the
November 12, 1930 to January 19, 1931. The conference Partition Council, where the Indian National Congress was
was to discuss the Report of the Simon Commission, but the represented by Sardar Patel, Dr. Rajendra Prasad, and
Indian National Congress boycotted the conference. The C. Rajgopalachari as an alternate member. The Muslim
Congress attended the second Round Table Conference, League had Mohammed Ali Jinnah, Liaqat Ali Khan, and
which took place from 7 September 1931 to 1 December Abdur Rab Nishtar as alternate members. Following India’s
1931 in London. Mahatma Gandhi represented Indian independence on August 15, 1947, the composition of the
National Congress in the Second Round Table Conference. partition council evolved to include two members from each
domain, and during this transitional period, Patel and Dr.
113. Solution: (a) Prasad continued to represent the Indian Domain.
Exp) Option a is the correct answer. 117. Solution: (b)
The Simon Commission’s lack of Indian representation Exp) Option b is the correct answer.
directly led to the Congress’s decision to boycott it. The The Poorna Swaraj Resolution adopted at the Lahore Session
absence of Indian members in the commission highlighted of the Indian National Congress was moved by Jawaharlal
the insensitivity of the British government towards involving Nehru. The resolution, a short 750-word document, called
Indians in shaping their own constitutional future. This fact for severing ties with the British and asserting complete
was a significant factor in the Congress’s decision to protest independence. It highlighted the economic, political, and
the commission and its work. cultural injustices caused by British rule in India. Nehru’s

181 PYQ Workbook


HISTORY OF MODERN INDIA

leadership in this historic declaration marked a significant to seek a separate Muslim homeland as the British prepared
step towards India’s struggle for independence and the to leave India. This day is also known as the 1946 Calcutta
subsequent civil disobedience movement. Killings/1946 Calcutta Riots, characterized by nationwide
communal violence for four days. It resulted in 5,000 to
118. Solution (c)
10,000 dead, and some 15,000 wounded.
Exp) Option c is the correct answer.
Rabindranath Tagore, the renowned poet and philosopher, 122. Solution (c)
referred to Subhas Chandra Bose as ‘Desh Nayak’, which Exp) Option c is the correct answer.
translates to “National Hero” or “Hero of the Nation”. He In Manipur, the Jiatarang movement began in 1930 with
even wrote a poem in his honor, titled “Desh Nayak”. The
Naga Woman Rani Gaidinliu as its leader. This movement
poem praises Bose as a “true son of India” and a “hero of
eventually transformed into a political effort aimed at
the nation”. The term reflects the high esteem and respect
removing the British from both Manipur and the nearby
with which Bose was regarded for his efforts in seeking
Naga regions.
India’s independence from British rule.
Important Tips
119. Solution (c)
Rani Gaidinliu ( “The Daughter of Hills”):
Exp) Option c is the correct answer.
• Rani Gaidinliu was a spiritual leader among the Naga
Aruna Asaf Ali was a major woman organizer of
people.
underground activity during the Quit India Movement.
The Quit India Movement, also known as the August • She belonged to the Rongmei clan of the Zeliangrong
Movement, was a mass civil disobedience movement tribe in Manipur’s Tamenglong district.
launched by Mahatma Gandhi in 1942, demanding an end to • Born on January 26, 1915.
British colonial rule in India. • At the age of 13, she joined the efforts of freedom
She is widely remembered for hoisting the Indian National fighter and religious leader, Haipou Jadonang, and
flag at the Gowalia Tank maidan in Bombay on 9 August became a key figure in his social, religious, and
1942, after most of the Congress leaders were arrested by the political movement.
British. She also worked alongside other leaders and activists • Jadonang, who was also from the Rongmei tribe,
to coordinate strikes, sabotage, and attacks on government initiated the ‘Heraka movement,’ based on traditional
offices and police stations. Naga beliefs, with a vision of an independent Naga
She remained underground for several months, evading kingdom (Naga-Raja).
arrest and inspiring many people to join the movement. • After Jadonang’s execution, Rani Gaidinliu continued
She also edited an underground newspaper called his work, leading the movement which gradually
Inquilab (Revolution), which reported on the activities shifted from religious to political focus.
and achievements of the movement. She was awarded the
• Rani launched a significant rebellion against the
International Lenin Peace Prize in 1964 and the Bharat
British colonial rule, leading to her imprisonment for
Ratna, India’s highest civilian honour, in 1997 for her life. She was eventually released after 14 years, in 1947.
contribution to the freedom struggle.
123. Solution (c)
120. Solution (c)
Exp) Option c is the correct answer
Exp) Option c is the correct answer.
The British case against Gandhi was based on the Western
The Cabinet Mission was a delegation sent by the British
interpretation of pragmatic justice. To the British, Gandhi
government to India in 1946 to discuss the transfer of
power from the British government to the Indian political was guilty of calling for a civil-disobedience campaign under
leadership with the aim of preserving India’s unity and Quit India Movement in 1942 which set off a mass outburst.
granting its independence. It was headed by three British Lord Linlithgow held Gandhi legally responsible for the
cabinet members: Pethick Lawrence, Stafford Cripps, and deaths that had occurred, the damage done. In the Viceroy’s
A.V. Alexander. The Viceroy of India, Lord Wavell, also words, Gandhi’s fast was “political blackmail”; as such
participated in some of the discussions. President of Indian it was Gandhi’s “sole responsibility.” This was the official
National Congress during the visit of Cabinet Mission to British view.
India was Maulana Abul Kalam Azad. He was the president
Important Tips
of the Congress from 1940 to 1946, and he advocated for a
united India with provincial autonomy. • Lord Chelmsford served as Viceroy and Governor
General of India from 1916 to 1921. He succeeded
121. Solution: (d) Lord Hardinge as Viceroy of India in 1916. His time
Exp) Option d is the correct answer. in India began in the aftermath of the British army’s
Muslim League observed the ‘Direct Action Day’ on 16th defeat in Mesopotamia, and Indian discontent grew as
August, 1946. It was the decision to call for ‘direct action’ the First World War progressed.

PYQ Workbook 182


HISTORY OF MODERN INDIA

• Edwin Montagu was appointed the Secretary of State Subhash Chandra Bose. It was the 51st session of the INC.
for India in 1917 and remained in that office till Haripura session adopted a resolution that the Congress
1922. On 20th August 1917, Montagu presented the would give moral support to those who were agitating against
historic Montagu Declaration (August Declaration) in the governance in the princely states.
the British Parliament. This declaration proposed the
increased participation of Indians in the administration Important Tips
and the development of self-governing institutions in Subhash Chandra Bose:
India. • In 1919, he had cleared the Indian Civil Services (ICS)
examination. Bose, however, resigned later.
124. Solution: (b)
• He was highly influenced by Vivekananda’s teachings
Exp) Option b is the correct answer
and considered him as his spiritual Guru. His political
India’s first labour union on modern lines was started mentor was Chittaranjan Das.
when workers of Buckingham and Carnatic Mills formed
• Netaji was the editor-in-chief of the newspaper named
the Madras Labour Union in April 1918. In 1918, the first
‘Forward’.
Registered Organized Trade Union was the “Madras Labour
• He even managed to open his own newspaper called
Union” founded by BP Wadia and V. Kalyanasundaram
‘Swaraj’.
Mudaliar. This landmark event is documented meticulously
by D. Veeraraghavan in his seminal book ‘Making of the • He established the National Planning Committee in
Madras Working Class’. October 1938. Its main objective was to focus on the
development of industries in India.
Important Tips
• S.C. Bose and his followers formed the Forward Bloc in
All India Trade Union Congress (AITUC) in the year 1939 as a new party within the Congress.
1920:
• He reached Japanese-controlled Singapore from
• The founding leaders were Lala Lajpat Rai, N M Germany in July 1943, issued from there his famous
Joshi & Joseph Baptista. call, ‘Delhi Chalo’, and announced the formation of
• Other leaders who participated were Netaji Subhash the Azad Hind Government and the Indian National
Chandra Bose, V. V. Giri, Sarojini Naidu, C.R. Das, Army on 21st October 1943.
Jawarharlal Nehru, Mohammad Ali Jinnah, Annie • His Jayanti is celebrated as ‘Parakram Diwas’ on 23rd
Besant, V.J. Patel, B.P. Wadia, Joseph Baptista, January.
Lalubhai Samaldas, Jamnadas, Dwarka Das, B W
Wadia, R R Karandikar, Col. J.C. Wedgwood. 128. Solution: (d)
• The AITUC was formed to secure a nominating Exp) Option d is the correct answer.
body for representing Indian Labour at the
Madan Mohan Malviya and Motilal Nehru made the
International Labour Conference.
decision to launch the Independent Party, also known
125. Solution: (b) as the Congress-Khilafat Swaraj Party, on 16 December
Exp) Option b is the correct answer 1922. This decision stemmed from disagreements that arose
In the year 1940, the Radical Democratic Party was following the suspension of the non-cooperation movement
founded by Manendra Nath Roy. It was developed from the in 1922. The party’s objective was to engage in electoral
League of Radical Congressmen which was founded in 1939 politics and secure representation in legislative bodies to
by M.N. Roy. Its main objective was to support the allies in effectively challenge the policies of the British government.
World War II. In the year 1948, it was dissolved. Inspired by the concept of Swaraj, which signifies
126. Solution (c) “independence” or “self-rule,” the Swaraj Party aimed to
attain greater self-governance and political freedom for the
Exp) Option c is the correct answer.
Indian people under British rule.
Rajendra Prasad appointed as the president of INC, soon
after Subash Chandra Bose resigned in 1939 in Tripuri 129. Solution (c)
Session. Subhas Chandra Bose was elected the President of
Exp) Option c is the correct answer.
Tripuri Session. But Gandhi was not happy with S.C. Bose’s
candidature. At the end S.C. Bose had to resign due to Kashi Hindu Vishva Vidyalaya was formed in 1916. During
getting lack of support even from leftists. the Non-cooperation Movement (1920-22), the institutions
founded were:
127. Solution (c)
• Kashi Vidyapith in 1921
Exp) Option c is the correct answer.
The Haripura session (in Gujarat) of Indian National • Gujarat Vidyapith in 1920
Congress (INC) held in the year 1938. It was presided over by • Jamia Millia Islamia in 1920

183 PYQ Workbook


HISTORY OF MODERN INDIA

Important Tips discarded separate electorates for the Depressed Classes


and instead provided for a system of joint electorates with
Kashi Vidyapith:
reserved seats. The pact increased the number of reserved
• Babu Shiv Prasad Gupta and Bhagwan Das established
seats for the depressed classes (SC and ST) in the Parliament
Kashi Vidyapith in Varanasi, on 10 February 1921,
and legislature to 147.
during the non-cooperation movement of the
freedom struggle. Originally named Kashi Vidyapith, 131. Solution: (d)
the university was renamed Mahatma Gandhi Kashi
Exp) Option d is the correct answer.
Vidyapith in 1995. It was inaugurated by Mahatma
Gandhi. Inspired by Gandhi’s principles, it was a hub of Statement 1 is correct: The Simon Commission, formally
the Indian National Movement, fostering self-reliance known as the Indian Statutory Commission, was indeed
and attracting notable nationalist scholars. appointed to review and enquire into the working of the
Gujarat Vidyapith: Government of India Act of 1919. The purpose was to
• Gujarat Vidyapith, a deemed university in assess the reforms that had been implemented and to make
Ahmedabad, India, was founded by Mahatma recommendations for potential further reforms.
Gandhi in 1920 to establish Indian-run educational Statement 2 is correct: It was chaired by Sir John Simon. He
institutions free from British control. It became was a British politician and lawyer who led the commission
a deemed university in 1963. While modernized, it during its study of the constitutional framework in India.
upholds Gandhian principles, emphasizing human
studies, social service, and development work. It Statement 3 is correct: Simon Commission advocated for a
played a pivotal role in promoting Indian education Federation-style structure at the centre, with a ‘Council of
during the struggle for independence. Greater India’ representing the interests of both British India
Jamia Millia Islamia: and the princely states.

• Jamia Millia Islamia, established in 1920 in Aligarh Statement 4 is correct: Yes, the Simon Commission faced
during British rule, is now a central university in significant opposition from Indian leaders and political
New Delhi. Founded by notable figures like Mahmud parties. The main reasons for opposition included the fact
Hasan Deobandi, Mohammad Ali Jauhar, Hakim Ajmal that the commission did not have any Indian members and
Khan, and others, its foundation stone was laid by its recommendations were seen as inadequate in addressing
Mahmud Hasan Deobandi. It gained central university Indian demands for self-governance and representation.
status in 1988, and its current location is Okhla.
Important Tips
Kashi Hindu Vishva Vidyalaya:
Recommendations of Simon Commission:
• Banaras Hindu University (BHU), also known as
kāśhī hindū viśvavidyālaya, established in 1916 in • Abolish Provincial Dyarchy: Expand ministers’
Varanasi, Uttar Pradesh, India, is a collegiate, central, responsibilities to provincial legislatures, eliminating
and research university. It was formed by incorporating the system of dual control.
the Central Hindu College founded by Annie Besant in • Governor’s Special Powers: Grant the Governor
1898. Madan Mohan Malaviya established BHU with special powers to protect minorities and safeguard
financial support from Maharaja Rameshwar Singh provinces.
of Dharbhanga, Maharaja Prabhu Narayan Singh of • Representation based on Population: Advocate
Benares, and lawyer Sunder Lal. representation in the Federal Assembly based on
130. Solution: (a) population.
• Burma’s Dominion Status: Grant Burma Dominion
Exp) Option a is the correct answer.
Status and its own Constitution.
Assertion (A) is true- The Poona Pact, established in
• Council of State Representation: Propose indirect
1932, marked an accord between Mahatma Gandhi and Dr.
election of Council of State members through
Babasaheb Ambedkar, representing the Dalits, depressed
Provincial Councils, akin to modern Proportional
classes, and upper caste Hindu leaders. This pact revolved
Representation.
around the reservation of electoral seats for the depressed
classes within the British India legislature. It defeated the • Federation-style Structure: Suggest a ‘Council of
Greater India’ to represent British India and princely
purpose of Communal Award which intended the separate
states.
electorates and reserved seats for minorities and depressed
classes. • Reservations for Poorest Classes: Recommend
reserved seats for the economically disadvantaged.
Reason (R) is true and (R) is the correct explanation
of (A)- The Poona Pact paved the way in facilitating the • Discretionary Powers for Communal Harmony:
participation of Dalits in the political arena and contributed Empower Governors with discretionary powers to
to diminishing the inequalities they encountered. It prevent communal tensions.

PYQ Workbook 184


HISTORY OF MODERN INDIA

• Increase in Legislative Council Members: Propose Reason is also correct and explains Assertions correctly:
an increase in the number of members in Legislative The Quit India Movement, with its mass protests and civil
Councils. disobedience, led to a severe crackdown by the British
authorities. It resulted in the arrest of many Indian leaders,
• Local Representation for Frontier Provinces:
including Mahatma Gandhi. The movement, coupled with
Establish local legislatures and representation rights
the changing dynamics of World War II, prompted the
for northwestern frontier provinces and Balochistan.
British government to reconsider its stance on India. This
• Administrative Divisions: Divide Sindh from Bombay eventually led to discussions and negotiations regarding the
and Burma from India.
transfer of power, culminating in India’s independence in
• Indianization of Army: Call for the Indianization of 1947.
the Indian army.
135. Solution: (a)
• Incorporation into Government of India Act 1935:
Integrate the Commission’s recommendations into the Exp) Option a is the correct answer.
1935 Act. The first commander of the “Azad Hind Fauj” or Indian
• Government Control over High Courts: Advocate full National Army (INA) was Mohan Singh. The INA was
governmental control over High Courts. founded on September 1, 1942, in Southeast Asia during
World War II. It was initially formed by Indian prisoners
132. Solution: (a) of war (PoWs) of the British Indian Army captured by
Exp) Option a is the correct answer. Japan in the Malayan campaign and at Singapore, under the
The main outcome of the 1921-22 Non-Cooperation leadership of Mohan Singh. Later, Subhas Chandra Bose took
Movement was the strengthening of Hindu-Muslim over its leadership and revived the INA after his arrival in
unity. The 1921-22 Non-Cooperation Movement, led by Southeast Asia in 1943.
Gandhi, aimed to unite Indians against British rule through
136. Solution (c)
nonviolence. The Khilafat Issue enhanced Hindu-Muslim
unity as Indian Muslims empathized with the Ottoman Exp) Option c is the correct answer.
Caliphate’s fate. Shared aspirations, leaders like Gandhi The Garhwal Regiment was a unit of the British Indian
and Nehru, mass participation, joint demonstrations, and Army. The soldiers of the Garhwal Regiment were deployed
interfaith symbols bridged divides. in Peshawar on 23rd April 1930, which was a hotbed of the
Civil Disobedience Movement. The soldiers were ordered
133. Solution: (d) to fire on the revolutionaries, but they refused to do so.
Exp) Option d is the correct answer. Chandra Singh Garhwali was one such soldier who refused
The Karachi Session of the Indian National Congress to open fire preventing another Jallianwala Bagh Massacre.
was held in March 1931. Vallabhbhai Patel presided over
137. Solution: (d)
the session. The session adopted the Karachi Resolution,
which outlined the economic and social goals of the Indian Exp) Option d is the correct answer.
National Congress. The resolution called for a number of Dr. B.R Ambedkar was not the member of the Interim
reforms, including the abolition of untouchability, the Cabinet but he later joined the Constituent Assembly.
introduction of a system of universal adult franchise, and Subsequently, he served as India’s first Law Minister in
the nationalization of key industries. Jawaharlal Nehru’s Cabinet. As a Law Minister, he led the
Drafting Committee of the Indian Constitution. He is also
134. Solution: (a) known as the ‘Father of the Indian Constitution.’
Exp) Option a is the correct answer.
Important Tips
(A) and (R) are individually true, and (R) is the correct
The Drafting Committee was set up on August 29, 1947.
explanation of (A).
Drafting Committee consisted of seven members. They
Assertion is correct: The Quit India Movement of 1942 were:
was one of the final and most intense phases of the
• Dr. B.R. Ambedkar (Chairman)
Indian freedom struggle against British colonial rule. It
represented a significant turning point in the struggle for • N. Gopalaswamy Ayyangar
independence. The movement was the most widespread • Alladi Krishnaswamy Ayyar
and intense phase of the freedom struggle, as it mobilized • Dr. K.M. Munshi
millions of people from all sections of society and regions • Syed Mohammad Saadullah
of the country. It also challenged the British authority and
• N. Madhava Rau (He replaced B.L. Mitter who resigned
legitimacy like never before, as it witnessed various forms
due to ill-health)
of resistance, such as strikes, boycotts, sabotage, attacks on
government offices and police stations, and formation of • T.T. Krishnamachari (He replaced D.P. Khaitan who
parallel governments. died in 1948)

185 PYQ Workbook


HISTORY OF MODERN INDIA

138. Solution: (b) against British policies and contributed to the growing
Exp) Option b is the correct answer. agitation for India’s independence. The event also played a
role in motivating Bhagat Singh and his associates to seek
Shivaram Hari Rajguru (1908–1931), an Indian revolutionary
retribution against the British officers, which eventually led
from Maharashtra, was a key figure in the Hindustan
to the tragic killing of J.P. Saunders in a case of mistaken
Socialist Republican Association (HSRA). Notably, he took
identity.
part in the assassination of British police officer John
Saunders. On March 23, 1931, he was executed by the 141. Solution (c)
British government, alongside Bhagat Singh and Sukhdev Exp) Option c is the correct answer.
Thapar, for their anti-colonial activities.
Assertion (A) is true- The salt agitation was launched by
Important Tips Mahatma Gandhi in 1930 to protest against the British salt
The Lahore Session of the Indian National Congress monopoly. The British had a monopoly on the production
in 1929 marked a turning point in India’s fight for and sale of salt, and they taxed it heavily. This made salt very
freedom. The session, under President Jawaharlal Nehru, expensive for the poor.
declared complete independence (Purna Swaraj) as the Reason (R) is false- Mahatma Gandhi’s object was not to
goal. It rejected the Simon Commission, authorized civil make salt available free to the poor. His object was to break
disobedience and tax non-payment, and fixed January 26, the British salt monopoly and to show that the Indian
1930, as the first Independence Day. people were not willing to be ruled by the British.
The Non-cooperation Movement was launched on 5th
142. Solution: (d)
September 1920 by the Indian National Congress (INC)
under the leadership of Mahatma Gandhi. In September Exp) Option d is the correct answer
1920, in Congress session in Calcutta, the party introduced According to KM Munshi, “Mohammad Ali Jinnah had
the Non-Cooperation programme. warned Gandhi not to encourage fanaticism of Muslim
The Rowlatt Satyagraha, led by Gandhi on April 6, 1919, religious leaders and their followers.
protested against the repressive Rowlatt Act passed by Important Tips
the British government. This Act allowed arrests without
Mohammad Ali Jinnah:
trial, indefinite detention, and curbed civil liberties. The
protest involved a nationwide hartal (strike), but violent • In 1916, he was elected as the president of the Muslim
clashes occurred, particularly in Punjab. The infamous league.
Jallianwala Bagh massacre occurred during these protests, • In 1920, when the Indian National Congress launched
leading to a heavy loss of life. The Act and other laws were a movement of non-cooperation, Jinnah opposed this
repealed in March 1922. policy and resigned from the congress.
• His continuous efforts and negotiations with the
139. Solution: (a)
British government resulted in the partition of India
Exp) Option a is the correct answer. and the formation of the state of Pakistan on 14 August
Chittaranjan Das, also known as C.R. Das, left his post as 1947.
the Indian National Congress president to establish the
Swaraj Party. A prominent leader from Bengal, he actively 143. Solution: (d)
participated in the Non-Cooperation Movement (1919-1922), Exp) Option d is the correct answer.
advocating non-violence and constitutional means for India’s Dr. B. S. Moonje tabled a vote of censure against Mahatma
freedom. Differing with Gandhi on “No Council Entry,” Gandhi for calling off the Non-Cooperation Movement
he resigned during the 1922 Gaya session. Forming the at the meeting of All India Congress Committee held
Congress-Khilafat Swaraj Party with figures like Motilal on February 24, 1922, in Delhi. Moonje was associated
Nehru and Narasimha Chintaman Kelkar, they sought to with the Hindu Mahasabha and held differing views from
engage in legislative councils for self-governance, offering Gandhi, especially regarding his principles of non-violence
an alternative approach within the broader independence and secularism. Moonje was critical of Gandhi’s decision to
movement. halt the movement. He was an advocate of a more militant
approach and was influenced by fascist ideologies.
140. Solution: (a)
Exp) Option a is the correct answer. 144. Solution: (a)
In October 1928, Lala Lajpat Rai led a demonstration Exp) Option a is the correct answer.
against the Simon Commission in Lahore. The police, Parallel governments were established in Balia and Satara
under the command of James A. Scott, subjected the during the Quit India Movement. The Quit India Movement,
protesters to a brutal lathi charge. Lala Lajpat Rai was also known as the August Movement, was a mass civil
injured in the police action, and he did not fully recover disobedience campaign launched by Mahatma Gandhi and
from his injuries, eventually passing away on November the Indian National Congress in 1942, demanding an end to
17, 1928. This incident further fueled the resentment British colonial rule in India.

PYQ Workbook 186


HISTORY OF MODERN INDIA

• Important Tips Army). Bose arrived in Singapore in 1943 and was invited
by Ras Bihari Bose to take over the leadership of the Indian
Parrallel Governments in 1942:
Independence League in East Asia. Ras Bihari Bose’s
• Four major parallel governments were established efforts and connections were instrumental in providing the
in Balliya (Uttar Pradesh), Tamluk (Bengal), Satara foundation and support for the formation of the Azad Hind
(Maharashtra) and Talcher (Orissa). Fauz, which aimed to fight for India’s independence from
• In Balia, a district in Uttar Pradesh, a parallel British rule.
government was formed on 19 August 1942 by Chittu
Important Tips
Pandey, a freedom fighter and a Congress leader. He
Some other Indian revolutionaries who helped Subhas
declared Balia as independent and appointed his own
Chandra Bose in the establishment of the Azad Hind
cabinet of ministers. He also issued his own currency
Fauj:
and stamps, collected taxes, ran courts, and maintained
law and order. He also organized a volunteer army to • Mohan Singh: Mohan Singh was the first commander
resist the British forces. The parallel government lasted of the Azad Hind Fauj. He was captured by the British
in 1942 and was imprisoned for several years.
for about a month, until the British army recaptured
Balia on 23 September 1942. • Captain Lakshmi Sahgal: Lakshmi Sahgal was a
doctor who joined the Azad Hind Fauj in 1943. She
• In Satara, a district in Maharashtra, a parallel
was known as the “Joan of Arc of India” for her bravery
government was formed on 10 September 1942 by and leadership.
Nana Patil, another freedom fighter and a Congress
• Captain Shah Nawaz Khan: Shah Nawaz Khan was
leader. He was inspired by the example of Balia and
a Muslim officer who joined the Azad Hind Fauj in
decided to replicate it in Satara. He declared Satara
1943. He was captured by the British and was tried for
as independent and appointed his own cabinet of
treason. He was acquitted of all charges in 1946.
ministers.
• In Tamluk in Midnapur district, parallel government, 147. Solution (c)
known as Jatiya Sarkar, was established on December Exp) Option c is the correct answer
17, 1942. It lasted until September 1944. The Kanpur Bolshevik conspiracy case was a controversial
• In Talcher in Orissa, parallel government was court case initiated in British India in 1924. Kanpur
established in 1943 by some local Congress workers. Conspiracy Case was also against the communists which
They took control of the police station and declared were abhorred by the British Government. Some newly
independence from British rule. turned communists named M N Roy, Muzaffar Ahamed,
S A Dange, Shaukat Usmani, Nalini Gupta, Singaravelu
145. Solution: (b) Chettiar, Ghulam Hussain were caught by the Government
Exp) Option b is the correct answer. and were trailed for conspiring against the Government. The
Charge on them was “to deprive the King-Emperor of his
In the meetings of All India Committee on June 14-15, 1947,
sovereignty of British India, by complete separation of India
Khan Abdul Ghaffar Khan voted against the partititon
from imperialistic Britain by a violent revolution.”
of India. He aligned himself with the pro-unity stance of
the Indian National Congress and All-India Azad Muslim 148. Solution: (a)
Conference, rather than the pro-partition All-India Muslim Exp) Option a is the correct answer.
League. However, his trust in the Indian National Congress The correct sequence of events is:
was shattered when they reluctantly accepted the partition
• Chauri-Chaura incident (February 5, 1922)
plan without consulting the Khudai Khidmatgar leaders,
leading him to express his disappointment by stating, “you • Suspension of Non-Cooperation Movement (February
have thrown us to the wolves.” In June 1947, Khan, along 12, 1922)
with other Khudai Khidmatgar leaders, formally presented • Bardoli Resolutions (1928):
the Bannu Resolution to the British authorities, demanding
Important Tips
the creation of an independent state of Pashtunistan
encompassing all Pashtun territories of British India, Chauri-Chaura incident (February 5, 1922): The Chauri
separate from the emerging state of Pakistan. Unfortunately, Chaura incident was a violent protest that took place in
the British government openly rejected the resolution’s the town of Chauri Chaura in Uttar Pradesh. A group
demands, and Khan’s vision of Pashtunistan as a separate, of protesters clashed with police and set fire to a police
independent entity was not realized. station, killing 22 policemen.
Suspension of Non-Cooperation Movement (February
146. Solution: (b) 12, 1922): The Chauri Chaura incident hocked Mahatma
Exp) Option b is the correct answer. Gandhi, who felt that the Non cooperation movement had
Ras Bihari Bose, an Indian revolutionary, played a become violent and that it was no longer following the
significant role in assisting Subhas Chandra Bose in the principles of non-violence. He called off the movement on
establishment of the Azad Hind Fauz (Indian National February 12, 1922.

187 PYQ Workbook


HISTORY OF MODERN INDIA

Bardoli Resolutions (1928): The Bardoli Resolution, • Negley Farson: He was an American adventurer,
drafted in 1928, was a pivotal document during the journalist, and author who traveled extensively around
Bardoli Satyagraha. It demanded the withdrawal of unfair the world and wrote about his experiences. He covered
tax hikes and fair treatment for farmers. The resolution’s various conflicts and revolutions, such as the Russian
impact extended to Saran, Bihar, inspiring locals to Revolution, the Chinese Civil War, the Spanish Civil
challenge oppressive British policies. Its courage and War, and World War II. He also met several influential
ideals continued to influence Saran’s involvement in the leaders, such as Winston Churchill, Joseph Stalin, Mao
struggle for freedom and justice. Zedong, and Chiang Kai-shek.

149. Solution: (b) 152. Solution: (a)


Exp) Option b is the correct answer. Exp) Option a is the correct answer.
Sarojini Naidu referred to Mahatma Gandhi and Lord Assertion (A) is true- On August 14-15, 1947, India
Irwin as “the two Mahatmas” due to their central roles in the achieved its independence from the United Kingdom of
events leading up to the Second Round Table Conference in Great Britain. The Indian Independence Act of 1947 shifted
1931 and the Salt Satyagraha of 1930. Gandhi’s leadership sovereign authority over British India from the British
of the Indian National Congress prompted the influential Parliament to the Constituent Assembly of India. Under the
Salt Satyagraha against the unjust salt law, resulting in his same Act, India underwent partition into Muslim majority
imprisonment and global attention. Lord Irwin, the Viceroy,
Pakistan and the democratic and secular India.
sought negotiations and released Gandhi, leading to a series
of eight meetings that culminated in the Gandhi-Irwin Reason (R) is true and (R) is the correct explanation of (A)-
Pact. This recognition by Naidu highlights their significance Britain suffered heavy losses in the Second World War. They
in striving for India’s independence through non-violent lost many soldiers, and they also had to pay a high financial
means and peaceful dialogue amid British imperialism. cost for the war. This weakened Britain economically and
militarily, and it made it difficult for them to maintain their
150. Solution: (d) rule in India.
Exp) Option d is the correct answer.
153. Solution: (a)
The Communal Award gave separate electorates and
reserved seats to Muslims, Sikhs, and Depressed Castes. Exp) Option a is the correct answer
However, it did not give separate electorates or reserved The Congress supported the Khilafat Movement mainly for
seats to Buddhists. The reason for this was that the number the Reinstatement of Caliph and getting the sympathy of the
of Buddhists in British India was relatively small. In Muslims in the upcoming struggle against the British.
addition, the Buddhists were not seen as a distinct religious
community by the British. 154. Solution (c)
Exp) Option c is the correct answer.
151. Solution: (a)
The Civil Disobedience Movement started with well-
Exp) Option a is the correct answer.
known Dandi March. The Dandi March was a 241-mile
Louis Fischer was an American journalist and author who
(385 km) protest march undertaken by Mahatma Gandhi
wrote extensively about world politics and history. He
and his followers in 1930 to protest against the British
was a close friend and admirer of Mahatma Gandhi, and
salt monopoly. The march began on March 12, 1930, from
he accompanied him during the Quit India Movement of
Gandhi’s Sabarmati Ashram in Ahmedabad, Gujarat, and
1942. He also wrote a biography of Gandhi, titled The Life
ended on April 6, 1930, at Dandi, Gujarat, where Gandhi
of Mahatma Gandhi, which was published in 1950 and later
adapted into the Oscar-winning film Gandhi in 1982. broke the salt law by making salt from seawater.

Important Tips Important Tips

• William L. Shirer: He was an American journalist and Mahatma Gandhi led the way in starting civil disobedience.
historian who reported on the rise of Nazi Germany This began after celebrating Independence Day in 1930.
and World War II. He is best known for his book The civil disobedience movement started with a well-
The Rise and Fall of the Third Reich, which is a known march to Dandi, when Gandhi, along with 78
comprehensive history of Nazi Germany based on his Ashram members, walked from Sabarmati Ashram in
personal observations and documents. Ahmedabad on 12 March 1930. At Dandi, Gandhi broke
the salt law, even though it was against the rules for
• Web Miller: He was an American reporter who covered
anyone other than the government to make salt. This act of
several major events in the 20th century, such as the
disobedience spread the idea of civil disobedience across
Mexican Revolution, the Irish War of Independence,
India. This event became a symbol of people standing up
the Spanish Civil War, and World War II. He also
against government rules.
interviewed several prominent figures, such as Adolf
Hitler, Benito Mussolini, Franklin D. Roosevelt, and 155. Solution: (d)
Jawaharlal Nehru. Exp) Option d is the correct answer.

PYQ Workbook 188


HISTORY OF MODERN INDIA

K. T. Paul was a prominent leader of the Indian Christian During the Non-Cooperation Movement, Chittaranjan
community and the General Secretary of the All India Das (also known as Deshbandhu) gave up his legal
Conference of Indian Christians. He represented Indian practice. He was a prominent Indian freedom fighter,
Christians in the First Round Table Conference in London political activist, and lawyer who played a vital role in the
in 1930. Indian Independence Movement. He suspended his legal
practice to actively participate in the movement against
156. Solution: (b)
British colonial rule. Chittaranjan Das was a key figure in
Exp) Option b is the correct answer.
advocating for non-cooperation with the British government
The Depressed Classes League was established in and promoting the use of indigenous goods, including Khadi
1935 by Babu Jagjiwan Ram, a prominent leader of the clothing.
Depressed Classes. The league was formed to demand more
representation for the Depressed Classes in the government 159. Solution: (a)
and in society. The league aimed to achieve equality for Exp) option a is the correct answer.
Dalits.
The Communal Award was a plan for the political
Important Tips representation of different religious communities in British
Babu Jagjivan Ram: India. It was announced by the British Prime Minister
Ramsay Macdonald on August 16, 1932. The Communal
• Babu Jagjivan Ram, popularly known as Babuji, was
Award was based on the principle of separate electorates,
a national leader, freedom fighter, and advocate for
social justice. which meant that different religious communities would
have their own electorates to elect their representatives to
• He championed the cause of depressed classes and
the legislature. This was done to protect the interests of
played a crucial role in the All India Depressed Classes
minority communities, such as Muslims and Sikhs.
League.
• In Calcutta (Kolkata), he organized Ravidas Sammelans 160. Solution: (d)
and celebrated Guru Ravidas Jayanti. Exp) Option d is the correct answer.
• In 1934, he established the Akhil Bhartiya Ravidas The Swaraj Party, also known as the Congress-Khilafat
Mahasabha in Calcutta. Swaraj Party, was formed by Chittaranjan Das and Motilal
• He appeared before the Hammond Commission in Nehru after the Gaya annual conference of the Indian
1935, advocating for voting rights for Dalits. National Congress in December 1922. Its primary goal was
• Inspired by Gandhiji, Babuji courted arrest in 1940 to advocate for greater self-governance and political freedom
and actively participated in the Civil Disobedience from British rule. Subhash Chandra Bose and Jawaharlal
Movement and Quit India Movement. Nehru were not directly associated with the formation of
• Babu Jagjivan Ram also served as the Deputy Prime the Swaraj Party.
Minister of India. Important Tips
157. Solution: (d) Background of formation of Swaraj Party:
Exp) Option d is the correct answer. The suspension of the non-cooperation movement in
1922 brought forth significant disagreements within the
The Lahore Conference Session took place from March
Congress Party’s leadership. A divide emerged among the
22 to 24, 1940, at Iqbal Park, Lahore, where the welcome
pioneers, with some advocating for the continuation of
speech was given by Sir Shah Nawaz Khan of Mamdot, who
non-cooperation while others aimed to conclude the
chaired the local reception committee. The Special Working
Committee of the All India Muslim League discussed different boycott of legislatures and engage in electoral contests.
versions of the final resolution text. The resolution, which The two factions were termed “no-changers” and “pro-
gained unanimous approval from the Subject Committee, changers.” The former group championed the persistence
endorsed the idea of a united homeland for Muslims and of non-cooperation, while the latter sought to transition
proposed the formation of an independent Muslim state. towards participating in elections and legislative bodies.
During the general session, A. K. Fazlul Huq, the chief During the Gaya session of the Congress in 1922, C R Das,
minister of undivided Bengal, introduced the resolution, who was presiding over the session, proposed the idea of
supported by leaders like Chaudhry Khaliquzzaman from entering the legislatures. However, this proposition was
the United Provinces, Zafar Ali Khan from Punjab, Sardar defeated, revealing the contrasting viewpoints within the
Aurangzeb Khan from North-West Frontier Province, and party.
Sir Abdullah Haroon from Sindh. Qazi Muhammad Essa
Consequently, C R Das and fellow leaders who supported
from Balochistan and other leaders also expressed their
participation in legislatures opted to break away from
support.
the Congress. They established the Congress-Khilafat
158. Solution: (d) Swarajaya Party, naming Das as its president and
Exp) Option d is the correct answer. enlisting Nehru as one of the secretaries.

189 PYQ Workbook


HISTORY OF MODERN INDIA

161. Solution (c) • British India would be divided into two dominions:
Exp) Option c is the correct answer. India and Pakistan.
Usha Mehta was a freedom fighter who was known for her • The constitution drafted by the Constituent Assembly
role in setting up the Congress Radio, an underground would not be applicable to Muslim-majority areas, as
radio station that operated for about three months during they would become part of Pakistan.
the Quit India Movement of 1942. Congress Radio was the • According to the plan, the legislative assemblies of
broadcasting mouthpiece of the Indian National Congress Bengal and Punjab convened and voted in favor of
and functioned from different locations in Bombay, present- partition.
day Mumbai, and briefly from Nashik. • The legislative assembly of Sind would make the choice
Congress Radio was organized by Usha Mehta, then a of joining the Indian constituent assembly or not. It
22-year-old student activist, with the help of amateur radio opted to join Pakistan.
operators. The radio station broadcast messages related to • A referendum was scheduled for NWFP (North-
the Indian independence movement and recorded speeches Western Frontier Province) to determine its dominion
from some of the prominent leaders of the movement. affiliation. NWFP chose to join Pakistan, although
The radio station broadcast from 27 August through 12 Khan Abdul Gaffar Khan boycotted and rejected the
November 1942 before being shut down by the authorities, referendum.
with the operators being arrested. • The transfer of power was set for August 15, 1947.
• To delineate the international boundaries between
Important Tips
the two nations, the Boundary Commission was
• Jai Prakash Narayan: He led the opposition against established, with Sir Cyril Radcliffe serving as
Prime Minister Indira Gandhi in the mid-1970s, and its chairperson. The commission was tasked with
he called for a “total revolution” to overthrow her. demarcating Bengal and Punjab into the two new
He was also involved in the Quit India Movement in countries.
1942, and he escaped from the Hazaribagh Central Jail
165. Solution: (a)
along with some other freedom fighters.
Exp) Option a is the correct answer
162. Solution: (a) Mahatma Gandhi was elected as President of the India
Exp) Option a is the correct answer Khilafat Conference (23rd November 1919). Khilafat
Committee was formed in Bombay On 20 March 1919. An
The Khilafat Movement had important consequences
all-India Khilafat Conference was organized in Delhi on 23
for India. The Khilafat Movement united Hindus and November 1919. Mahatma Gandhi saw it as an opportunity
Muslims. Before this movement, Hindus and Muslims did to bring together Hindus and Muslims on a common
not cooperate much with each other. Hindus supported the platform for the nationalist movement. For the upcoming
Muslims in this movement. This helped reduce communal Non-cooperation Movement, the leaders of the Khilafat
tensions for some time. Movement joined hands with Indian National Congress.

163. Solution: (b) 166. Solution: (d)


Exp) Option b is the correct answer. Exp) Option d is the correct answer.
Chakravarti Rajagopalachari, commonly known as C.R.
Maulana Abul Kalam Azad was the president of the Indian
Rajaji, was not a member of the Swaraj Party. He was
National Congress when it passed the Quit India resolution
member of the Indian National Congress and later founded
on 8 August 1942 in Bombay. He was also one of the senior the Swatantra Party in 1959. Rajagopalachari played a
leaders who were arrested by the British authorities on the prominent role in the Indian freedom movement, serving as
night of 9 August 1942. He remained in prison until 1945. a leader of the Congress, Premier of the Madras Presidency,
Governor-General of India, and Chief Minister of Madras
164. Solution (c) state. Srinivas Iyer, Chittaranjan Das, and Vithalbhai Patel
Exp) Option c is the correct answer. were all members of the Swaraj Party.
The Mountbatten Plan was a plan for the transfer of power Important Tips
from British rule to Indian rule in 1947. It was announced
• Srinivasa Iyengar (1874–1941) was an Indian lawyer,
by Lord Mountbatten, the last Viceroy of India, on June 3,
freedom fighter, and politician linked with the Indian
1947. The plan called for the partition of British India into
National Congress. He served as Advocate-General of
two dominions, a Hindu-majority India and a Muslim-
Madras, was a law member of the Governor’s executive
majority Pakistan. The Mountbatten Plan was accepted by
council, and broke away with leaders like Motilal
the Indian National Congress and the Muslim League, and it
Nehru and Chittaranjan Das to form the Swarajya
was approved by the British Parliament. The plan came into
Party in 1923. He mentored prominent leaders like
effect on August 15, 1947, when India and Pakistan became
K. Kamaraj. His book “Mayne’s Hindu Laws” is well-
independent dominions.
regarded. Despite his accomplishments, he faced
Important Tips criticism for the Congress’s Brahmin-dominated image
The Mountbatten Plan’s Provisions were as follows: during his leadership.

PYQ Workbook 190


HISTORY OF MODERN INDIA

• Chittaranjan Das (1870–1925), known as • The prohibition on the INC’s activities was lifted.
Deshbandhu, was an Indian freedom fighter, • The properties confiscated from Satyagrahis were
political leader, and lawyer in the Indian returned.
Independence Movement. He founded the Swaraj
Party in undivided Bengal and mentored Netaji Subhas • People living near coastal areas were allowed to gather
Chandra Bose. He defended Aurobindo Ghosh in the salt.
Alipore bomb case. He played a key role in the Non- • Outstanding fines were waived.
Cooperation Movement, promoted Hindu-Muslim • Government employees who had resigned during the
unity, and advocated national education. He founded civil disobedience movement were treated leniently.
the Congress-Khilafat Swaraj Party and was the first
mayor of Calcutta. 168. Solution: (a)
• Vithalbhai Patel (1873–1933), co-founder of the Exp) Option a is the correct answer
Swaraj Party and elder brother of Sardar Patel, was an The policy of Indian National Congress during 2nd World
influential Indian legislator. He joined the Congress
War was that the country was prepared to cooperate with
but differed from Gandhi’s leadership. He played a
the British for World War II, but on the condition that
key role in the Swaraj Party, aiming to sabotage the
there be an immediate declaration by Britain, granting India
government through councils. As President of India’s
complete independence. It also demanded the establishment
Central Legislative Assembly, he established procedural
practices and neutrality conventions in debates. of a national government. Indian National Congress (INC)
initially decided to support the British war effort in World
167. Solution: (d) War II for a number of reasons.
Exp) Option d is the correct answer. The INC was concerned about the threat posed by Nazi
The Gandhi-Irwin Pact was an agreement between Germany and its allies.
Mahatma Gandhi and the British Viceroy, Lord Irwin, to The INC also believed that it was in India’s strategic interest
end the Civil Disobedience Movement. The pact was signed to support the British war effort. The INC argued that if the
on 5 March 1931. The Pact proposed the adjournment British were defeated, India would be vulnerable to attack
of the Civil Disobedience Movement in exchange for from Japan.
the British government’s agreement to release political The INC was also hoping that by supporting the British war
prisoners, withdraw ordinances, and allow the Indian effort, it would be able to pressure the British government to
National Congress to participate in the Second Round Table grant India independence after the war.
Conference.
169. Solution: (a)
Important Tips
Exp) Option a is the correct answer.
Features of Gandhi-Irwin Pact:
The Simon Commission, formally known as the Indian
• The Indian National Congress (INC) decided to join Statutory Commission, was sent to India in 1928. Its
the Round Table Conference. primary purpose was to study and recommend potential
• The INC pledged to halt the civil disobedience administrative reforms for the British possession in
movement. India. The commission was chaired by Sir John Simon and
• The withdrawal of all laws that restricted the Congress’ consisted of seven members from the British Parliament.
activities. Although it aimed to assess constitutional reform, it faced
• Discontinuation of legal actions except for those significant criticism for its lack of Indian representation and
related to violent offenses. led to increased demands for Indian self-governance.

• Liberation of individuals imprisoned for participating 170. Solution: (b)


in civil disobedience. Exp) Option b is the correct answer.
• Elimination of the tax on salt. The slogan “Jai Hind” was given by Subhas Chandra Bose.
Outcomes of the Gandhi-Irwin Pact: “Jai Hind” became the rallying cry for the INA and a symbol
• The INC actively participated in the Second Round of patriotic fervor. It was used to inspire and motivate the
Table Conference held between September and soldiers of the INA as well as the general Indian population
December of 1931. to unite against British rule. The slogan encapsulated the
• The government committed to repealing all enacted spirit of India’s struggle for freedom and conveyed a strong
ordinances. sense of nationalism and pride.

• It agreed to release political detainees except those 171. Solution (c)


involved in violent acts. Exp) Option c is the correct answer.
• Peaceful picketing of liquor and foreign cloth shops Statement 1 is correct- The growth of socialist tendency in
was permitted. the national movement led to the foundation of the Congress

191 PYQ Workbook


HISTORY OF MODERN INDIA

Socialist Party in 1934 under the leadership of Acharya The case attracted public attention to the communist
Narendra Dev, Ram Manohar Lohia, and Jayaprakash ideology and its aims in India.
Narayan. The party was formed by a group of Congressmen
who were dissatisfied with the party’s moderate policies. 174. Solution: (d)
They wanted the Congress to adopt a more radical approach Exp) Option d is the correct answer.
to social and economic reform. Surya Sen was not associated with the Kakori Conspiracy
Statement 2 is correct- In 1936, Jawaharlal Nehru, who case. The Kakori Conspiracy case, also known as the Kakori
was the leader of the Congress Socialist Party, urged the Train Robbery, was a famous incident that took place in 1925
Congress to accept socialism as its goal and to bring itself in Kakori, near Lucknow, India. The case involved a group
closer to the peasantry and the working class. Nehru’s speech of revolutionaries who attempted to rob a train carrying
was a major turning point in the history of the Congress. It government funds.
marked the beginning of the Congress’s shift towards a more Surya Sen was a revolutionary leader who led the
socialist ideology.
Chittagong armoury raid in 1930 in Bengal.
172. Solution: (d)
175. Solution: (d)
Exp) Option d is the correct answer.
Exp) Option d is the correct answer.
Option a is correct- Women were an integral part of the Salt
According to the Mountbatten Plan, the decision regarding
March from the very beginning. Some of the women who
the partition of Punjab and Bengal was to be taken based
participated in the Salt March are- Sarojini Naidu, Kamala
on the voting of the members of the respective Legislative
Chattopadhyay, Kasturba Gandhi, Aruna Asaf Ali, and
Assemblies. The Mountbatten Plan, also known as the
Mahadevi Verma.
June 3rd Plan, was proposed by Lord Mountbatten, the last
Option b is correct- Gandhiji initially did not want women
Viceroy of India, in 1947. The plan outlined the steps for the
to participate in the Salt March, as he felt that it was too
partition of British India into two separate dominions, India
dangerous for them. However, he was eventually persuaded
and Pakistan.
by the request of Kamaladevi Chattopadhyay, a leading
Indian independence activist and social reformer. 176. Solution: (b)
Option c is correct- The march was a major news story, and Exp) Option b is the correct answer.
it was reported in newspapers and magazines all over the Jawaharlal Nehru and Sardar Patel did not represent
world. The Western press was fascinated by the nonviolent
Congress to negotiate with the mission. The correct
methods of protest used by Gandhi and his followers, and
representatives of Congress were Maulana Abul Kalam Azad
they saw the Salt March as a symbol of India’s struggle for
and C. Rajagopalachari.
independence. Some of the press where it was published
includes- New York Times, The Times of London, The 177. Solution: (b)
Guardian, and The Washington Post. Exp) Option b is the correct answer.
Option d is incorrect- The First Round Table Conference Separate electorate was not proposed by the Nehru Report.
took place from November 1930 to January 1931, while
In fact, the report rejected the idea of separate electorate for
the Dandi March, initiated by Mahatma Gandhi and his
any community, as it believed that it would harm the national
followers, occurred prior to this conference. The Dandi
unity and solidarity. The report favored joint electorate
March lasted from March 12, 1930, to April 6, 1930, and
with reservation of seats for minorities in proportion to
was a direct action against the campaign of tax resistance
their population.
and non-violent protest aimed at challenging the British
salt monopoly. It was a significant event in India’s struggle Important Tips
for independence, symbolizing civil disobedience and The Nehru report had the following recommendations:
drawing international attention to the movement.
• Dominion status for India within the British
173. Solution: (a) Commonwealth, as an interim arrangement until
Exp) Option a is the correct answer. complete independence was achieved.
• A federal form of government with a strong unitary
Kanpur Bolshevik Conspiracy case was a court case initiated
and democratic centre, and provincial autonomy for
by the British government in 1924 against some communist
the states.
activists who were accused of seeking to overthrow the
British rule by a violent revolution. • A bicameral legislature at the centre, with equal
representation for the states in the upper house and
The accused included Muzaffar Ahmad, SA Dange, Shaukat
proportional representation for the population in the
Usmani, Nalini Gupta, Singaravelu Chettiar, Ghulam
lower house.
Hussain and MN Roy. MN Roy was charged in absentia,
Ghulam Hussain turned an informer and was pardoned, and • A responsible executive at the centre and the
the rest were sentenced to four years of imprisonment. provinces, accountable to the legislature.

PYQ Workbook 192


HISTORY OF MODERN INDIA

• Adult suffrage for men and women above 21 years of HMIS Talwar was one of the ships where the mutiny started,
age, subject to certain qualifications. and the revolt spread to other naval establishments as well.
• A bill of rights that guaranteed fundamental rights Naval ratings engaged in hunger strikes as a form of protest
and freedoms to all citizens, such as freedom of speech, against the oppressive conditions and discriminatory
expression, assembly, association, religion, etc. treatment they faced.
• No state religion and no discrimination on grounds Lord Irwin, who served as the Viceroy of India from 1926
of religion, caste, creed, sex or race.
to 1931, was not directly involved in the RIN Mutiny, 1946.
• A common civil code for all citizens.
• A Supreme Court as the highest judicial authority in 181. Solution: (b)
India. Exp) Option b is the correct answer
• A Governor-General as the constitutional head of Statement 1 was part of Cabinet mission Plan: The mission
India, appointed by the British monarch on the advice proposed a complicated system for India with three tiers:
of Indian ministers
the provinces, provincial groupings and the centre.
178. Solution (c) The centre’s power was to be confined to foreign affairs,
Exp) Option c is the correct answer defence, currency and communications. The provinces
Statement 1 is incorrect: Quit India Movement was not a would keep all other powers and could establish three
rural revolt. It was a mass movement that involved people provincial groups. These provincial groups constituted of
from all walks of life, including urban workers, students, combinations of provinces and princely states. Two groups
and professionals. would be constituted by the mainly-Muslim western and
Statement 2 is incorrect: The Quit India Movement was a eastern provinces. The third group would comprise the
radical movement that called for the end of British rule. mostly-Hindu areas in the south and the centre.
Statement 3 is correct: The Quit India Movement was the Statement 2 was part of Cabinet mission Plan: The
first major movement in the Indian independence movement
proposals of the Cabinet Mission did include the formation
to be marked by widespread violence. This violence was
largely spontaneous, and was caused by the frustration of the of a Constituent Assembly. The Constituent Assembly
Indian people with British rule. was meant to be a body responsible for drafting a new
Statement 4 is correct: The British government responded constitution for India.
to the Quit India Movement with a wave of repression. Statement 3 was not part of Cabinet mission Plan:
Thousands of Indians were arrested, and many were tortured Proposals of the Cabinet Mission did not include the creation
and killed. of a sovereign Pakistan with Muslim-majority provinces. The
179. Solution (c) Cabinet Mission aimed to provide a framework for a united
Exp) Option c is the correct answer India with provisions for autonomy and representation for
Statement 1 is correct: As part of the movement, there various regions and communities.
were widespread strikes, boycotts of British goods, and
182. Solution: (a)
picketing of government institutions in urban areas.
Exp) Option a is the correct answer
Statement 2 is correct: In rural areas, there were widespread
attacks on symbols and means of colonial rule and authority, In February 1947, British Prime Minister Attlee announced
such as destroying railways, telegraph lines, and government that the British would transfer power to India by June
buildings. 1948, While other statements are correct.
Statement 3 is incorrect: The formation of Home Rule • Lord Wavell advocated total withdrawal of the British
leagues is not associated with the Quit India Movement.
from India by 31 March, 1948.
Home Rule leagues were formed earlier in 1916 and were
primarily focused on demanding self-government within the • It was Lord Mountbatten who speeded up the process of
British Empire. transfer of power.
Statement 4 is correct: The “Karnataka Method” was a • The Plan Balkan was abandoned on account of Nehru’s
system that saw many citizens of the state become part- opposition to it.
time peasant squads, tending farms throughout the day and
carrying out sabotage operations at night. 183. Solution: (d)

180. Solution (c) Exp) Option d is the correct answer


Exp) Option c is the correct answer Gandhi-Irwin Pact included the return of lands not yet
The Rin mutiny was primarily a revolt by the Indian naval sold to third parties, but it did not guarantee the return of
ratings and did not involve the INA. all confiscated lands to their original owners.

193 PYQ Workbook


HISTORY OF MODERN INDIA

Important Tips movement. The resolution stated that 26th January 1930
would be celebrated as Independence Day.
The Gandhi-Irwin Pact:
Irwin on behalf of the government agreed on: 187. Solution: (b)
• immediate release of all political prisoners not Exp) Option b is the correct answer
convicted of violence, remission of all fines not yet The All-India Depressed Classes Leaders’ Conference at
collected, return of all lands not yet sold to third parties Bombay in 1931, led by Dr. B.R. Ambedkar, demanded
• lenient treatment to those government servants who separate electorates for untouchables to ensure their
had resigned; political representation, as they believed they would be
• right to make salt in coastal villages for personal outnumbered by caste Hindus in the general electorate.
consumption (not for sale); right to peaceful and non- 188. Solution (c)
aggressive picketing; and withdrawal of emergency
Exp) Option c is the correct answer
ordinances
The viceroy, however, turned down two of Gandhi’s The Muslim League formally accepted the Cabinet Mission
demands: Plan on 6 June 1946. The plan had rejected the demand for
a sovereign Pakistan, but it did propose the creation of a
• public inquiry into police excesses, and
number of Muslim-majority provinces that could have opted
• Commutation of Bhagat Singh and his comrades’ death to join a separate Pakistan if they so desired. The Muslim
sentence to life sentence. League accepted the plan because it believed that it would
Gandhi on behalf of the Congress agreed: give Muslims a greater degree of self-determination within
• to suspend the civil disobedience movement, and to a united India.
participate in the next Round Table Conference Important Tips
184. Solution: (d) Proposals of Cabinet Mission Plan:
Exp) Option d is the correct answer. 1. The Dominion of India would be granted
Vithalbhai Patel did not belong to the group of No- independence, without any partition. The provinces
Changers but was a Swarajist who advocated for council would be divided into three groups/sections:
entry and obstruction of the government from within. • Group A: Madras, Central Provinces, UP, Bihar,
He was elected as the president of the Central Legislative Bombay and Orissa
Assembly in 1925 • Group B: Punjab, Sindh, NWFP and Baluchistan
Important Tips • Group C: Bengal and Assam
Stance of No-changers: 2. The Muslim-majority provinces were grouped into
two groups and the remaining Hindu-majority in
• The ‘No-changers’ opposed council entry
one of the groups.
• They advocated concentration on constructive work,
3. The central government at Delhi would have powers
and continuation of boycott and non-cooperation.
over the defence, foreign affairs, communications
• They also advocated quiet preparation for resumption and currency. The rest of the powers would be vested
of the suspended civil disobedience programme. with the provinces.
• This school of thought led by Vallabhbhai Patel, 4. A constituent assembly would be set up for writing a
Rajendra Prasad, C. Rajagopalachari & M.A. Ansari new constitution for the country.
came to be known as the ‘No-changers’.
5. An interim government would be established until
185. Solution: (d) a new government was formed on the basis of the
Exp) Option d is the correct answer constitution written by the constituent assembly.

The Gandhi-Irwin Pact of 1931 was an agreement between 189. Solution: (a)
Mahatma Gandhi and Lord Irwin, the Viceroy of India. Exp) Option a is the correct answer.
The pact led to the suspension of the Civil Disobedience
Rowlatt Act: It was passed in 1919. It granted the British
Movement and the Congress agreed to take part in the
government in India extensive powers to suppress political
Second Round Table Conference.
activities and arrest individuals without trial.
186. Solution (c) Salt March: Also known as the Dandi March, it took place in
Exp) Option c is the correct answer 1930. It was a nonviolent protest led by Mahatma Gandhi
On the final day of the Lahore session, 31st December against the British salt monopoly. Gandhi and his followers
1929, the Indian National Congress, under the leadership marched to the coastal village of Dandi and produced salt
of Jawaharlal Nehru, passed a historic resolution declaring from seawater, defying the salt tax imposed by the British.
Purna Swaraj as the goal of the Indian independence Chauri Chaura Incident: It occurred in 1922. During a

PYQ Workbook 194


HISTORY OF MODERN INDIA

protest against the British colonial rule in the town of Chauri elections. The Congress emerged as a dominant political
Chaura, Uttar Pradesh, a violent clash broke out between force in several provinces, including Madras, Bombay,
the protesters and the police. As a result, a police station Central Provinces, Bihar, and Orissa.
was set on fire, resulting in the deaths of several police
Statement 4 is incorrect- The 1937 elections to the provincial
officers. This event led Mahatma Gandhi to call off the Non-
legislatures in British India had significant implications
Cooperation Movement.
for various political parties. The All-India Muslim League,
Second Round Table Conference: It was held in 1931. It was despite being the second-ranking party, experienced a
a series of discussions between the British government and
setback by winning only 106 seats, which accounted for
various Indian political leaders to discuss constitutional
6.7% of the total. Surprisingly, the League’s performance
reforms and the future of India. The conference aimed to
was not strong even in provinces with a predominantly
find a common ground between the demands of the Indian
Muslim population.
National Congress and the British government.

190. Solution: (b) 192. Solution: (a)

Exp) Option b is the correct answer. Exp) Option a is the correct answer.
Statement 1 is incorrect- The Salt March, also known as the The Punjab Naujawan Bharat Sabha, founded in 1926,
Dandi March, received significant international attention. was a revolutionary organization that aimed to engage and
The Salt March, led by Mahatma Gandhi in 1930, was a mobilize the youth, peasants, and workers of Punjab in
nonviolent protest against British salt laws. Participants political activities.
walked 240 miles to produce salt from seawater, symbolizing
Important Tips
resistance and inspiring India’s struggle for independence.
Punjab Naujawan Bharat Sabha:
Statement 2 is correct- The Salt March was widely covered
by the American and European press. The progress of • It was founded by Bhagat Singh and his comrades in
the walk was closely monitored by media outlets like The March 1926 and had its headquarters in Amritsar.
New York Times, which quoted Gandhi as he criticized • The Sabha organized lectures, meetings,
the oppressive salt tax and called out the British for their demonstrations and protests to spread its ideology and
reluctance to arrest him. challenge the colonial policies.
Statement 3 is correct- The Salt March was one of the first • The Sabha was banned by the British government in
nationalist activities in which women participated in large 1929 after its involvement in the Lahore Conspiracy
numbers. Women played a crucial role in the Salt March, Case and the hunger strike of Bhagat Singh and his
defying societal norms and actively participating in the civil fellow prisoners.
disobedience movement led by Mahatma Gandhi. Women
193. Solution: (a)
like Kamaladevi Chattopadhyay participated in the Salt
March. She was trying to sell contraband Salt to Bombay Exp) Option a is the correct answer.
Stock Exchange and later arrested. The Ghadar party was a revolutionary organization formed
Statement 4 is correct- The Salt March, along with other by Indian expatriates in the United States in 1913. It aimed
civil disobedience actions, had a significant impact on the to overthrow British colonial rule in India and establish an
British government and impressed upon them the urgency to independent and democratic country. The Ghadar party,
devolve more power to Indians. Gandhi was released from formed in the USA, chose Punjab to begin its armed revolt
prison in January 1931 and Gandhi-Irwin pact was signed. in India.
It demonstrated the unity and determination of the Indian
people in their demand for self-rule. Important Tips
Punjab was strategically significant for the Ghadar
191. Solution (c)
party due to various reasons:
Exp) Option c is the correct answer.
• Punjab had a long history of nationalist movements
Statement 1 is correct- In the elections to the provincial and resistance against British rule
legislatures in 1937, only about 10 to 12 per cent of the • Punjab had a significant population of Punjabi soldiers
population had the right to vote. The right to vote was who served in the British Indian Army. The Ghadar
limited and not universal. party aimed to incite rebellion among these soldiers
Statement 2 is incorrect- The untouchables (Dalits) did and use their military training and resources to launch
have the right to vote in the elections. The Government of an armed revolt.
India Act of 1935, which regulated the elections, provided • Punjab had a strong sense of nationalism and a
voting rights to all adult males and a limited number of adult tradition of militancy among its people.
females, irrespective of caste or social status.
Statement 3 is correct- The Congress party won an absolute 194. Solution: (d)
majority in five out of eleven provinces in the 1937 Exp) Option d is the correct answer.

195 PYQ Workbook


HISTORY OF MODERN INDIA

The reason for the resignations of the Indian Ministers in Important Tips
all the provinces in the year 1939 was India was declared a
Naujawan Bharat Sabha (NBS):
party to World War II without the consent of the provincial
government. • Naujawan Bharat Sabha (NBS) was founded in
March 1926 by Bhagat Singh as a left-wing Indian
In 1939, World War II broke out, and the British government,
association.
without consulting the elected Indian representatives or
• It aimed to foment revolution against British colonial
provincial governments, declared India as a party to the war.
rule by mobilizing worker and peasant youths and
This decision was seen as a violation of the principles of self-
promoting Marxist ideas.
governance and consent.
• NBS was closely associated with the Hindustan
195. Solution: (d) Republican Association and merged with the All-
Exp) Option d is the correct answer. India Youth Federation (AIYF) of the Communist
Party of India later.
Kanaklata Barua was a girl who led a public procession
and defied the police, tried to enter the police station but • The organization had members from Hindu, Muslim,
and Sikh communities and conducted lectures, public
was shot dead during the Quit India Movement. She was
meetings, and protests.
born on 22 December 1924 in Barangabari village of Gohpur
subdivision in Assam. She was inspired by the freedom • Despite its diverse membership, NBS faced limited
struggle and joined the Mrityu Bahini (Death Squad), a support due to its radical stance on religion and
group of young volunteers who were ready to sacrifice their agrarian reform.
lives for the cause of independence. • Its popularity increased after the killing of John P.
Saunders in 1928 during a protest against the Simon
On 20 September 1942, she led a group of about 500 people,
Commission.
mostly women and children, who marched towards the
Gohpur police station with the intention of hoisting • NBS was banned in July 1929 under Section 144
the Indian National flag there. When they reached near during Bhagat Singh’s hunger strike campaign.
the police station, they were stopped by a British officer • Sohan Singh Josh, an NBS activist, was imprisoned
named R.S. Linton, who ordered them to disperse. He also for his involvement in the Meerut Conspiracy Case.
threatened to shoot them if they did not obey. Kanaklata • NBS, Kirti Kisan Party, and the Communist Party
refused to turn back and continued to move forward with the of India were the main left-wing groups in Punjab,
flag in her hand. She said, “We are not afraid of death. We are attempting alliances.
ready to die for our country.” Linton then opened fire at the • These groups faced legal restrictions under the
unarmed crowd, killing Kanaklata and another volunteer Criminal Law Amendment Act (1908) in September
named Mukunda Kakoti on the spot. 1934, leading to their prohibition.

196. Solution (c) 198. Solution: (b)


Exp) Option c is the correct answer. Exp) Option b is the correct answer.
The correct chronological order of the events is (i), (iii), (iv), The Quit India Resolution was passed by the Congress
and (ii). Working Committee on 8 August 1942 in Bombay, but it was
Event 1: The Lucknow Pact was signed on December 29, first discussed and formulated in Wardha on 14 July 1942.
1916, between the Indian National Congress and the All The resolution stated the provisions of the movement as:
India Muslim League. • An immediate end to British rule over India.
Event 3: The Jallianwala Bagh massacre took place on April • Declaration of the commitment of free India to defend
13, 1919. itself against all kinds of imperialism and fascism.
Event 4: Bal Gangadhar Tilak died on August 1, 1920. • Formation of a provisional government of India after
British withdrawal.
Event 2: The Swaraj Party was formed on January 1, 1923, by
Motilal Nehru and Chittaranjan Das. • Sanctioning of a civil disobedience movement against
British authority.
197. Solution: (d)
• The resolution was inspired by Mahatma Gandhi’s call to
Exp) Option d is the correct answer. Do or Die in his speech delivered in Bombay on 9 August
The correct chronological order of the events is: 1942 at the Gowalia Tank Maidan.
Event 4: Jallianwala Bagh Tragedy (1919) 199. Solution: (d)
Event 2: Formation of Swaraj Party (1923) Exp) Option d is the correct answer.
Event 1: Formation of Naujawan Bharat Sabha (1926) Mahatma Gandhi was the sole official representative of
Event 3: The Dandi March (1930) the Indian National Congress at the Second Round Table

PYQ Workbook 196


HISTORY OF MODERN INDIA

Conference. He was invited from India accompanied The Muslim League observed ‘Day of Deliverance’ on 22
by Sarojini Naidu and also Madan Mohan Malaviya, December 1939 when the Congress Party members who were
Ghanshyam Das Birla, Muhammad Iqbal, Sir Mirza Ismail part of the Central and provincial governments, resigned
(Diwan of Mysore), S.K. Dutta and Sir Syed Ali Imam. in mass protesting the Viceroy Linlithgow’s decision to
make India a party to the Second World War without duly
200. Solution: (a)
consulting Indians.
Exp) Option a is the correct answer.
The celebration was intended to rejoice the resignation of
The whole dispute between Subhash Chandra Bose and all members of the rival Congress party from provincial
right-wing after the Tripuri session of Congress centered and central offices. It was led by Muslim League president
around the question of formation of Congress Working Muhammad Ali Jinnah. It was a step towards achieving their
Committee. Actually the Congress working committee had goal of a separate Muslim state, which would later become
been an elected body, but S.C. Bose nominated the members Pakistan.
of this body during the Tripuri Session. This step was not
taken positively by many members of Congress. Differences 203. Solution: (d)
between S.C. Bose and others propped up. He had accused Exp) Option d is the correct answer.
the working committee leaders of being supportive of the Chaudhary Rehmat Ali and his friends were the ones who
government on the matter of federation. Leaders of working coined the word “Pakistan” in 1933. They published a
committee felt they could not work with a president who pamphlet titled “Now or Never” in which they proposed the
had publicly cast doubts on their nationalistic principles creation of a separate Muslim state in the north-western
and resigned from the working committee. Bose refused part of India. The word “Pakistan” is derived from the
to nominate a new working committee. Bose wanted an names of the five provinces of the proposed state: Punjab,
immediate struggle led by Gandhi, whereas Gandhi was firm Afghania, Kashmir, Sindh, and Baluchistan.
in his belief that the time was not ripe for struggle. Hence,
formation of Congress Working Committee was one of the Important Tips
disputable issue during and after the Tripuri session. Muhammad Iqbal- He is renowned for his renowned
poem ‘Sare Jahan Se Acha,’ which remains one of the most
201. Solution: (a) notable patriotic poems in the Urdu language. This poem
Exp) Option a is the correct answer. is often used as a marching composition during various
The Muslim League representing mostly Muslims formed national events.
their government in Bengal and Sindh after 1946 elections. Sir Syed Ahmed Khan- He established the United India
In contrast, Congress representing the majority the Patriotic Association in Aligarh in 1888.
country’s population, was to go on to form governments in Md. Ali Jinnah- He became a part of the Indian National
the provinces of Madras, Bombay, United Provinces, Bihar, Congress in 1906. Initially, he steered clear of the All India
Orissa, Central Provinces and Berar and NWFP. Muslim League due to its perceived orthodoxy. However,
Important Tips in 1913, he eventually joined the Muslim League and
assumed the presidency during the Lucknow session in
Announcement of 1946 Election:
1916.
On September 19, 1945, following negotiations between
Indian leaders and members of the 1946 Cabinet 204. Solution (c)
Mission from the United Kingdom, Viceroy Lord Wavell Exp) Option c is the correct answer.
announced that elections for provincial and central The elections of 1937 were held under the Government of
legislatures would take place from December 1945 India Act 1935, which introduced a limited form of self-
to January 1946. It was also revealed that an executive
government to India. The INC contested the elections on
council would be established, and a constitution-making
a platform of Purna Swaraj (complete independence). In
body would convene after these elections.
elections, Congress secured a clear majority in the five
Summary of 1946 Election: provinces out of eleven provinces. The Congress won 716
• Central Assembly- In the December 1945 elections out of 1,161 seats it contested. It got a clear majority in 1.
for the central assembly, Congress secured 57 seats, Bihar (98/152); 2. Central Provinces (70/112); 3. Madras
the League obtained 30, Independents secured 5, (159/215); 4. Orissa (36/60); 5. United Provinces (113/228).
Akalis won 2, and Europeans captured 8. Congress was able to form its government in two more
• Provincial Assembly- Out of 1585 total seats, provinces with support of other parties in Bombay and
Congress won 923 seats (58.23%) whereas All-India Northwest Frontier Provinces.
Muslim League secured 425 seats (26.81% of the 205. Solution: (d)
total), becoming the second-largest party.
Exp) Option d is the correct answer.
202. Solution: (a) Before 1947, 26th January was celebrated as ‘Poorna Swaraj
Exp) Option a is the correct answer. Day.’ On 19 December 1929, during the Lahore session

197 PYQ Workbook


HISTORY OF MODERN INDIA

of the Indian National Congress, a historic resolution would make its own decision, while a referendum in the
for ‘Purna Swaraj’ or total independence was passed. North-West Frontier Province (now known as Khyber
This resolution led to the declaration of 26 January 1930 Pakhtunwa) and Sylhet district of Assam was scheduled to
as Independence Day, urging Indians to celebrate and resolve their status. NWFP chose to join Pakistan, although
protest British rule nationwide. The Poorna Swaraj Day Khan Abdul Gaffar Khan boycotted the referendum. The
was celebrated with great enthusiasm all over India. People possibility of independence for Princely States was ruled out;
took part in rallies and demonstrations, and they hoisted they could either join India or Pakistan. Independence for
the Indian flag. The day was a symbol of the Indian people’s Bengal was also excluded from the plan.
desire for independence. After India gained independence in
208. Solution (c)
1947, 26th January became Republic Day.
Exp) Option c is the correct answer.
Important Tips
The Indian National Congress passed the Complete
Martyrs Day: Independence Resolution at its Lahore session in 1929.
• January 30 is recognised nationally as Martyrs’ Day This historic resolution, also known as the ‘Purna
in India, to mark the assassination of Mahatma Swaraj’ resolution, declared the goal of achieving total
Gandhi in 1948. independence for India. It was a response to the breakdown
• A number of states and regions recognise other days as of negotiations with the British over dominion status.
Martyrs’ Days locally. The Lahore session led by Jawaharlal Nehru marked a
Constitution Day: turning point in the freedom movement, advocating civil
disobedience and celebrating January 26, 1930, as the first
• Constitution Day (Samvidhāna Divasa) is celebrated
Independence Day.
in India on 26 November annually, marking the
adoption of the Constitution of India in 1949. Important Tips
• It became effective on 26 January 1950. Declared as Nagpur Congess Session-1920:
Constitution Day in 2015, it honors B. R. Ambedkar, The Indian National Congress held its annual session
key in drafting the constitution. Previously known in December 1920 in Nagpur, presided over by C.
as Law Day, this day emphasizes the constitution’s Vijayaraghavachariar. It marked a turning point as
significance and Ambedkar’s ideas. the INC adopted the Non-Cooperation Resolution,
advocating boycott of foreign goods, schools, and
206. Solution: (b) courts, and promoting Hindu-Muslim harmony. The
Exp) Option b is the correct answer. session established a 15-member Congress Working
The first name for the resolution was the Lahore resolution, Committee, shifting focus from constitutional means to
but later it became known as the Pakistan Resolution. It extraconstitutional peaceful struggle for Swaraj.
was shared on March 24, 1940. Sikandar Hayat Khan wrote Belgaon Congess Session-1924:
down this resolution, and on March 23, 1940, Fazlul Haque The 39th session of the Indian National Congress held at
talked about it. This statement was created by 25 people who Belgaum in 1924 was the only Congress session presided
were part of the working group of the All Indian Muslim over by Mahatma Gandhi. Theater performances like
League. They wrote it between March 22 and March 24, “Pavitra Khadi,” promoting Gandhian ideals, were staged,
1940. Quaid-e-Azam (Muhammad Ali Jinnah) presented it. fostering nationalist sentiment. Plays on figures like Kittur
The main point of the statement was that Muslims wanted Channamma and Sangolli Rayanna kindled anti-British
fervor, motivating more to join the Congress and creating
their own place where they could follow their religion
an environment conducive to the nationalist movement.
without any problems. This place was named the Islamic
Republic of Pakistan. Karachi Congess Session-1931:
The Karachi Session of Congress in 1931, held from
207. Solution (c) March 26-31, marked significant developments. The
Exp) Option c is the correct answer. Gandhi-Irwin Pact was endorsed, despite public anger
In February 1947, Prime Minister Clement Atlee announced due to recent executions. Gandhi faced black flags on his
way to the session. Congress dissociated from political
in the House of Commons that the British Government
violence, admired martyrs’ bravery, endorsed the Pact, and
intended to transfer power to India by June 1948. To
reiterated the goal of “Poorna Swaraj.” The session’s key
facilitate this transition, Lord Mountbatten was appointed
resolution on Fundamental Rights and Economic Policy,
as India’s Viceroy on March 24, 1947, with extensive drafted by Nehru, defined Swaraj’s meaning for common
powers. Recognizing the likelihood of India’s partition and people. It included civil rights, universal adult franchise,
the creation of Pakistan, Lord Mountbatten proposed the education, workers’ rights, women’s protection, and
partition plan on June 3, 1947, known as the Mountbatten state control of industries.
Plan. According to this plan, Punjab and Bengal Legislative
Assemblies would vote on partition in separate groups, 209. Solution: (d)
allowing Hindus and Muslims to make their choices. Sindh Exp) Option d is the correct answer

PYQ Workbook 198


HISTORY OF MODERN INDIA

Individual Satyagraha was initiated by Mahatma Gandhi Shaheed Dweep, earlier known as Neil Island is an
to affirm the right to free speech. Mahatma Gandhi had island of the Andaman Islands, located in Ritchie’s
chosen Acharya Vinoba Bhave as the first Satyagrahi to start Archipelago.
Individual Satyagraha and Jawaharlal Nehru as the second
Balidan Vedi is the State Martyr’s Memorial at
and Brahma Datt as the third. Datta was one of the inmates
Humphrey Gunj in Andaman and Nicobar Islands. It
of Gandhi’s Ashram.
stands as a memorial to honor the sacrifice of 44 Indian
Important Tips Independence League (IIL) members who were killed by
Individual satyagraha’s consequence/output: Japanese forces during World War II and buried them in
an “L” shaped mass grave.
• It helped to keep the Indian independence movement
alive during a time when the British were hoping that 212. Solution: (a)
the war would divert Indian attention away from their
Exp) Option a is the correct answer.
demands for self-rule.
Jagjivan Ram was assigned the Portfolio of the Labour
• It helped to build national unity and solidarity among
Department in the Interim Government’s Cabinet of 1946.
Indians, as people from all walks of life came together
In his role as a labor minister in post-independence, he
to participate in the movement.
introduced numerous significant labor reforms such as the
• It put pressure on the British government to make Minimum Wages Act (1948), Employees Provident Fund
concessions to the Indian people, such as the release of Act (1952), Employees Insurance Act (1948), and various
political prisoners and the lifting of censorship. other pro-worker measures. He introduced these labor
• It helped to prepare the Indian people for the Quit reforms during the early years of independent India.
India Movement of 1942, which was a more widespread
and intense form of protest. 213. Solution: (b)
Exp) Option b is the correct answer.
210. Solution (c)
Sardar Vallabhbhai Patel was entrusted with the princely
Exp) Option c is the correct answer.
portfolio in the National Provisional Government. There
The honorific “Netaji” (meaning “respected leader”) was were more than 550 princely states in India when it gained
first applied to Subhas Chandra Bose in Germany in early independence, which were not officially under British
1942. It was used by the Indian soldiers of the Indische rule. These princely states covered 48% of the land in pre-
Legion and by the German and Indian officials in the Special independent India. Out of the 552 princely states in India,
Bureau for India in Berlin. The honorific quickly spread to 549 decided to become a part of India. The other 3 states,
other parts of the world, and is now used throughout India. Hyderabad, Junagarh, and Kashmir, initially didn’t want
to join India. But eventually, they became a part of India:
211. Solution: (a)
Hyderabad through police action, Junagarh through a
Exp) Option a is the correct answer. referendum, and Kashmir through the Instrument of
Nicobar Island was renamed Swaraj Island by Subhas Accession.
Chandra Bose. The term “Swaraj” means self-rule or self-
governance in Hindi, and it encapsulates the essence of India’s Important Tips
struggle for freedom from colonial rule. This renaming Interim Government:
of Nicobar Island as Swaraj Island was a symbolic act that The National Provisional Government is also called
represented Bose’s commitment to India’s sovereignty and Interim Government which was formed on 2nd
his efforts to create a sense of national identity and unity September, 1946 under the Cabinet Mission Plan.
among the people. The 14-member interim government had the following
composition:
Important Tips
• President of the Executive Council (Viceroy and
Renaming of Islands:
Governor-General of India): Viscount Wavell (until
Keeping in mind the historic significance of the Andaman February 1947), Lord Mountbatten (from February
& Nicobar Islands and to honour the memory of Netaji 1947)
Subhas Chandra Bose, Ross Islands was renamed as
• Commander-in-Chief: Sir Claude Auchinleck
Netaji Subhash Chandra Bose Dweep by the Prime
Minister during his visit to the Island in 2018. Neil • Vice President, Also in charge of External Affairs
Island and Havelock Island were also renamed as Shaheed and Commonwealth Relations: Jawaharlal Nehru
Dweep and Swaraj Dweep. (INC)
Additional information on these Islands: • Home Affairs, Information and Broadcasting: Sardar
Vallabhbhai Patel (INC)
Swaraj Dweep, earlier known as Havelock Island, is one
of the largest islands in Ritchie’s Archipelago, to the east • Agriculture and Food: Rajendra Prasad (INC)
of Great Andaman within the Andaman Islands. • Commerce: Ibrahim Ismail Chundrigar (ML)

199 PYQ Workbook


HISTORY OF MODERN INDIA

• Defence: Baldev Singh (INC) During the Non-Cooperation Movement, Mahatma Gandhi
urged people to avoid alcohol to boycott British goods and
• Finance: Liaquat Ali Khan (ML)
institutions. This resulted in a serious loss of revenue for the
• Education and Arts: C Rajagopalachari (INC) government, which was heavily dependent on liquor taxes. To
• Health: Ghazanfar Ali Khan (ML) persuade people to start drinking again, the government of
• Labour: Jagjivan Ram (INC) Bihar circulated a list of prominent individuals who drank
alcohol. The list included many Indian leaders and officials
• Law: Jogendra Nath Mandal (ML)
who were seen as supporting the British Raj. The intention
• Railways and Communications, Post and Air: Asaf was to utilize social pressure as a means of compelling
Ali (INC). individuals to discontinue their boycott and revert to their
• Works, Mines, and Power: C H Bhabha (INC). previous drinking practices.
• Note- INC stands for Indian National Congress and 218. Solution: (b)
ML stands for Muslim League.
Exp) Option b is the correct answer.
214. Solution: (d) Lord Wavell was not a member of the Cabinet Mission.
Exp) Option d is the correct answer. The Cabinet Mission of 1946 was an initiative formulated
Jayprakash Narayan was the founder of the Azad Dasta, by Clement Attlee, the then Prime Minister of the United
a revolutionary organization that operated during the Quit Kingdom. The Cabinet Mission had three British cabinet
India Movement of 1942. He formed the Azad Dasta with the members – Pethick Lawrence, Stafford Cripps, and A.V.
Alexander. The mission aimed to discuss the transfer of
help of some amateur radio operators and Congress leaders,
power from the British government to the Indian political
and trained them in guerrilla warfare. The Azad Dasta
leadership with the aim of preserving India’s unity and
carried out several attacks on British targets and disrupted
granting its independence.
their communication and transportation systems. One of the
places where the Azad Dasta operated was Bakri Ka Tapu, an 219. Solution: (e)
island in the Gandak river in Bihar.
Exp) Option e is the correct answer.
Important Tips Gandhiji’s Salt Satyagraha aimed at securing full
Chittu Pandey: independence for India from British rule. While the
• He was a peasant leader who led a rebellion against the Satyagraha had additional purposes, like Repeal of salt laws
British in Ballia district of Uttar Pradesh during the thereby ensuring economic relief to the common people
Quit India Movement. and limiting government authority, these served as steps
toward the primary objective rather than the main aim. The
• He declared Ballia as an independent republic and
Satyagraha served as a potent instrument within the broader
hoisted the Indian flag there on 19 August 1942.
movement for Indian independence, ultimately contributing
215. Solution (c) to the realization of that aspiration.
Exp) Option c is the correct answer. Important Tips
Jai Prakash Narayan, Ram Manohar Lohia, and Acharya On January 31, 1930, Mahatma Gandhi conveyed 11
Narendra Dev founded the Congress Socialist Party in demands to British Viceroy Lord Irwin, marking a
1934. The Congress Socialist Party emerged as a socialist prelude to the Civil Disobedience Movement. These
faction within the Congress Party. Born out of differences demands included:
in ideology with Mahatma Gandhi, its objectives centered on • Removal of the salt tax, symbolizing British oppression
achieving both independence and socialism. Ram Manohar and adversely affecting the poor.
Lohia played a role in disseminating its ideas through the • Abolition of the liquor monopoly, seen as a regressive
publication of the ‘Congress Socialist’ journal. However, the tax and a social injustice.
party eventually dissolved in 1948.
• Reduction of land revenue, which was considered
216. Solution: (a) excessive and unfair to the peasantry.
Exp) Option a is the correct answer. • Prohibition of forced labor, addressing exploitation.
In Bihar, Swami Vidyanand led cultivators during the Non- • Abolition of communal electorates, seen as divisive
Cooperation Movement. He had agitated against zamindars and promoting communal violence.
and rallied peasants in support of the movement, aligning • Amnesty for political prisoners, particularly those held
their struggles with the principles of non-cooperation. He without trial.
played a significant role in advocating for and mobilizing • Withdrawal of restrictive ordinances limiting freedom
farmers in Bihar for the movement. of speech and assembly.
217. Solution: (b) • Restoration of pre-1919 press freedom, ending press
Exp) Option b is the correct answer. censorship.

PYQ Workbook 200


HISTORY OF MODERN INDIA

• Release of confiscated property belonging to those colonial India. Guided by Gandhiji, this march aimed to
involved in the independence movement. create salt by processing seawater in the coastal village of
Dandi. The march included notable freedom fighters like
• Permission for peaceful picketing of liquor and foreign
Sarojini Naidu, Rajkumari Amrit Kaur, and Kamaladevi
cloth shops as a legitimate form of protest.
Chattopadhyay.
• Recognition of the right of Indians to control their
own affairs and a call for dominion status for India, Important Tips
granting more self-government. Spread of Civil Disobedience Movement:
Satyagraha at different places and with their leaders:
220. Solution: (d)
• Malabar- K.Kelappan
Exp) Option d is the correct answer.
• Orissa- Gopalbandhu Chaudhari
Sardar Vallabhbhai Patel was given the Home Department
in the Interim Government of 1946. As the Home Affairs • Tamil Nadu- C. Rajagopalachari
Minister, he worked to restore peace in a country torn by • Assam- Strike against the Cunningham Circular.
unprecedented communal conflicts. He did this with the
223. Solution (c)
strong and efficient leadership of a skilled administrator.
Apart from the home department, he was also in charge of Exp) Option c is the correct answer.
Information and Broadcasting department. The Nehru Report was prepared by a committee headed
by Motilal Nehru. The subject of the Nehru Report was
221. Solution (c)
“Constitutional arrangements in India.” J.L. Nehru
Exp) Option c is the correct answer. was acting as a secretary. There were 9 members in this
Sheel Bhadra Yajee, an activist from Bihar, actively committee. The final report was signed by M.L. Nehru and
associated with both the All-India Forward Bloc and the J.L. Nehru, Tej Bahadur Sapru, Ali Imam, Madhav Shrihari
Indian National Army (INA) movement. In 1939, Yajee Aney, Shuaib Qureshi, Mangal Singh, Subhas Chandra
joined Subhas Chandra Bose to find the All-India Forward Bose, and G. R. Pradhan. It was a significant document that
Bloc. He also became associated with the INA movement. outlined the proposed constitutional framework for India’s
Yajee was a supporter of social transformation and believed in future governance and sought a dominion status for India
the active participation of peasants, workers, and the middle within the British Commonwealth. The report addressed
classes in the struggle for change. He played a significant various aspects of governance, representation, and rights,
role in the freedom movement and was a member of the and played a pivotal role in shaping discussions on India’s
Rajya Sabha. He authored several works and was honored constitutional reforms.
with a commemorative stamp by the Government of India. Important Tips
Important Tips Recommendations of Nehru Report:
Baikuntha Shukla (15 May 1907 – 14 May 1934) was Dominion Status and Parliamentary Government:
an Indian nationalist and revolutionary. He executed Suggested granting India Dominion Status, featuring
Phanindra Nath Ghosh, a government approver responsible a parliamentary system of government. The legislature
for Bhagat Singh, Sukhdev, and Rajguru’s hanging. Born in should be bicameral, consisting of a Senate and a House
Jalalpur, he taught at Mathurapur before joining HSRA. of Representatives.
Shukla was hanged at 27 in Gaya Central Jail on May 14, Responsible Government: Advocated responsible
1934. government at both the central and provincial levels. The
Jayaprakash Narayan, a disciple of Gandhi, was a key Governor-General and Governors should act on the advice
figure in India’s independence movement. He advocated of the executive councils, which would be collectively
“saintly politics,” led Quit India movement, and post- responsible to the legislature.
independence, co-founded the Praja Socialist Party. He Federal Structure: Proposed a federal form of government,
called for “Total Revolution” in 1974, uniting people with residual powers vested in the central authority.
against corruption and advocating change in political, Provincial Councils: Suggested provincial councils with a
social, economic, and cultural systems. His leadership 5-year term, headed by Governors who would act based on
influenced the formation of the Janata Party and shaped the advice of the provincial executive councils.
a new direction in Indian politics, emphasizing youth- Joint Electorates: Recommended doing away with
driven change and democratic participation. In 1999, separate electorates for minorities, replacing them with
he was posthumously awarded the Bharat Ratna, India’s joint electorates to discourage communal sentiments.
highest civilian award, in recognition of his social service.
Fundamental Rights: Advocated nineteen fundamental
222. Solution: (d) rights, encompassing equality for women, the right to
form unions, and universal adult suffrage.
Exp) Option d is the correct answer.
Religious Protections: Emphasized safeguarding Muslims’
The Salt Satyagraha, also known as the Salt March or
cultural and religious interests.
Dandi March, represented a form of peaceful protest during

201 PYQ Workbook


HISTORY OF MODERN INDIA

Secular State: Urged the complete separation of the state Important Tips
from religion, ensuring a secular character. The Khilafat Movement, which began in 1919, brought
the Muslims and the Hindus on a common platform
224. Solution (c)
against British rule, was the most important cause of the
Exp) Option c is the correct answer Non - Cooperation Movement.
The All India Trade Union Congress (AITUC) is the oldest The Khilafat movement was initiated under the leadership
trade union federations in India. It was established on of Ali brothers namely Mohammed Ali and Shaukat Ali.
October 31, 1920 at Bombay Presidency, British India. The The movement was started to support the Sultan of Turkey
first President of the All Indian Trade Union Congress whom The Muslim population in India considered their
religious head.
(AITUC) was Lala Lajpat Rai. It is associated with the
Communist Party of India. 227. Solution (c)

Important Tips Exp) Option c is the correct answer.

All Indian Trade Union Congress (AITUC): Tej Bahadur Sapru was an eminent lawyer, political leader,
and social reformer in India during the British Raj. He was a
The first session, the founding conference began on
member of the Indian National Congress and the All India
October 31, 1920, in Empire Theatre Bombay with Lala
Liberal Federation. He was also a member of the Round
Lajpat Rai as the founding President in which 101 delegates
Table Conferences. Sapru played an important role in the
from 64 unions with a membership of 1,40,854 from all
signing of the Gandhi-Irwin Pact. He was the mediator
over India participated with the presence of political
between Gandhi and the British Viceroy, Lord Irwin.
leaders of various shades of opinions such as Motilal
Nehru, M.A. Jinnah, Annie Besant, V.J. Patel, B.P. Wadia, Important Tips
Joseph Baptista, Lalubhai Samaldas, Jamnadas, Dwarka Chintamani:
Das, B W Wadia, R R Karandikar, Col. J.C. Wedgwood. Sir Chintamani was an Indian editor, journalist, and
progressive politician.
225. Solution: (b)
Born on the Telugu New Year’s Day (Ugadi) in Andhra
Exp) Option b is the correct answer Pradesh.
There are various phases of growth of labour unions in India He served as the Chief Editor of The Leader from 1909 to
Pre-1918: The genesis of the labour movement in India 1934.
Chintamani was appointed as the Education Minister of
1918-1925: The early trade union phase
the United Provinces of British India under the Dyarchy
1926-1939: Revolutionary phase of the trade union system of the Government of India Act 1919.
1939-1946: Period of labour activism 228. Solution: (b)
1947-present: Post-independence trade unionism Exp) Option b is the correct answer.
1926-1939: This era was marked by increasing militancy Winston Churchill was the Prime Minister of Britain
and a revolutionary approach. It also saw multiple split-ups when the Quit India Movement started in India in 1942.
He was a staunch opponent of Indian independence and
in the movement. AITUC split up multiple times paving
refused to negotiate with the Indian National Congress or
way for the formation of organisations like National Trade Mahatma Gandhi. He also ordered a harsh crackdown on the
Union Federation (NTUF) and All India Red Trade Union movement, arresting thousands of leaders and activists and
Congress(AIRTUC). However, the need for unity was felt suppressing the protests with force.
and they all merged with theAITUC in the next phase. The Important Tips
government was also receptive to the trade union movement.
Chamberlain: He was the Prime Minister of Britain
Legislations like the Trade Unions Act, 1926 and the Trade from 1937 to 1940. He is best known for his policy of
Disputes Act, 1929 gave a fillip to its growth. It bestowed appeasement towards Nazi Germany, which failed to
many rights to the unions in return for certain obligations. prevent the outbreak of World War II. He did not have
This period was marked by the dominance of the left. much involvement in the Quit India Movement, as he
resigned in May 1940 and was succeeded by Winston
226. Solution: (b) Churchill.
Exp) Option b is the correct answer Clement Attlee: He was the Prime Minister of Britain
from 1945 to 1951. He was the leader of the Labour Party,
Mahatma Gandhi supported the Khilafat movement as he saw which won a landslide victory in the 1945 general election.
it as an opportunity to bring together Hindus and Muslims He is credited with granting independence to India and
on a common platform for the nationalist movement and to Pakistan in 1947, after realizing that the British could not
unite against the British empire. afford to maintain their empire after the war.

PYQ Workbook 202


HISTORY OF MODERN INDIA

231. Solution (c)


Macdonald: He was the Prime Minister of Britain from
1924 to 1929 and from 1931 to 1935. He was the first Exp) Option c is the correct answer.
Labour Party leader to become Prime Minister, but he also The Second Round Table Conference in London was held
formed a coalition government with the Conservatives in the backdrop of the Irwin-Gandhi Pact, which was signed
during the Great Depression. on 5 March 1931. The pact was an agreement between
Mahatma Gandhi and the British Viceroy, Lord Irwin, to
229. Solution: (a) end the Civil Disobedience Movement and to allow the
Exp) Option a is the correct answer. Indian National Congress to participate in the Second Round
Congress President who negotiated with both Cripps Table Conference.
Mission and Lord Wavell was Abul Kalam Azad. He was
Important Tips
the president of the Congress from 1940 to 1946, and he
advocated for a united India with provincial autonomy. He Organisations and their representatives in the Second
also tried to persuade Muhammad Ali Jinnah, the leader of Round Table Conference:
the Muslim League, to accept the Cripps Mission’s proposals Indian National Congress- Mahatma Gandhi, Rangaswami
and the Wavell Plan, but failed. Iyengar, Madan Mohan Malaviya
230. Solution: (b) Muslims League – Md. Ali Jinnah, Aga Khan III,
Muhammad Iqbal
Exp) Option b is the correct answer.
The Non-Cooperation Movement led by Mahatma Gandhi Hindus – M R Jayakar
was halted due to the Chauri-Chaura incident. The Chauri Depressed classes – Dr. B R Ambedkar
Chaura incident was a violent protest that took place Female Representatives- Radhabai Subbarayan and
on February 4, 1922, in the town of Chauri Chaura in Sarojini Naidu
Gorakhpur district of Uttar Pradesh. A group of protesters
The First Round Table Conference occurred in London
clashed with police and set fire to a police station, killing 22
from November 12, 1930, to January 19, 1931. The
policemen. This incident shocked Mahatma Gandhi, who felt
Third Round Table Conference was held in London from
that the movement had become violent and that it was no
longer following the principles of non-violence. He called November 17, 1932, to December 24, 1932.
off the movement on February 12, 1922. 232. Solution: (a)
Important Tips Exp) Option a is the correct answer
Kakori Case: The term Sarvodaya was the title of Gandhi’s 1908
The Kakori Train Action occurred on 9 August 1925 near translation of John Ruskin’s book, Unto This Last. Later
Lucknow during the Indian independence movement. into his life, the concept of Sarvodaya became a central part
Organized by Hindustan Republican Association (HRA) of his political philosophy. He understood Sarvodaya as “the
members like Ram Prasad Bismil and Ashfaqullah Khan, welfare of all” and applied it practically into ideas such as the
it aimed to fund anti-British activities. The train was dignity of labour, equitable distribution of wealth, communal
self-sufficiency and individual freedom.
stopped, British funds were looted, and one passenger
was unintentionally killed. The event brought attention to 233. Solution: (d)
HRA’s cause.
Exp) Option d is the correct answer.
The Jallianwala Bagh massacre: At the Congress Working Committee meeting in Allahabad
The Jallianwala Bagh massacre occurred on 13 April 1919 in April-May 1947, Jawaharlal Nehru was able to secure a
in Amritsar, British India. General Dyer surrounded a majority for his plan of a Guerilla War against the Japanese.
peaceful crowd protesting the Rowlatt Act and ordered his Nehru argued that the guerrilla war would be a way for
troops to open fire, killing at least 379 and injuring over India to contribute to the Allied war effort and to weaken
1,200. The brutal event sparked outrage, eroding Indian the Japanese. Nehru argued that the guerrilla war would be a
trust in British intentions and fueling the non-cooperation way for India to assert its independence and to show the world
movement. that it was not willing to be subjugated by any foreign power.
The Muzaffarpur Conspiracy Case: His proposal gained support due to India’s unequivocal
stand against Nazism, Fascism, and imperialism during
The Muzaffarpur Conspiracy Case (1908) revolved
World War II. Nehru’s leadership, vision, and commitment
around the bomb attack on Chief Magistrate Douglas
to India’s independence and resistance against oppressive
Kingsford’s carriage by Khudiram Bose and Prafulla forces were influential in garnering support for his plan
Chaki in Muzaffarpur, Bihar. Bose was arrested while among the committee members.
Chaki committed suicide. The trial began on 21 May
1908 with Khudiram, Mrityunjay Chakraborty, and 234. Solution (c)
Kishorimohan Bandopadhyay. Despite his defense, Exp) Option c is the correct answer.
Khudiram was sentenced to death at the age of 18 and In 1943, the Azad Hind Fauz (Indian National Army)
faced the sentence with a smile. was set up in Singapore. Subhas Chandra Bose arrived in

203 PYQ Workbook


HISTORY OF MODERN INDIA

Singapore on July 2, 1943, at the invitation of Rash Bihari 236. Solution: (a)
Bose, a revolutionary freedom fighter. He took over as the Exp) Option a is the correct answer
leader of the Indian Independence League in East Asia
and was appointed President of the league. On October 21, The Cripps’ Mission declared that entire India was to be a
1943, Bose proclaimed the formation of the Provisional Dominion and after the war its constitution was to be framed
Government of Free India at Cathay Cinema Hall in by an elected body which was to be accepted by the British
Singapore. He reorganized and revitalized the Azad Hind Government. Mahatma Gandhi called Cripps Mission ‘A
Fauz with the assistance of the Japanese. Bose actively worked post-dated cheque drawn on a failing bank’ due to the
to raise funds and recruit soldiers for the Azad Hind Fauz in Cripps offer of Dominion Status after the war.
Malaya and other parts of Southeast Asia. He capitalized on
the discontent among Indian army personnel in Malaya due 237. Solution: (a)
to the way British officers had abandoned them during their Exp) Option a is the correct answer
retreat from Singapore. Many British Indian troops joined
Malabar Rebellion or Moplah Uprising or Mapillah Revolt
the Azad Hind Fauz under his leadership.
(1921) was an offshoot of Khilafat agitation movement.
Important Tips The anti-British sentiment fuelled by these agitations
Japan: The Azad Hind Fauj was never set up in Japan. affected the Muslim Mapillahs (also known as Moplahs) of
Burma: The Azad Hind Fauj was set up in Burma in 1944, south Malabar region of Kerala.
but it was originally set up in Singapore in 1943.
Important Tips
Malaya: The Azad Hind Fauj did not have a significant
presence in Malaya. Malabar Rebellion or Moplah Uprising:
• After the death of Tipu Sultan in 1799 in the Fourth
235. Solution: (a)
Anglo-Mysore War, Malabar had come under British
Exp) Option a is the correct answer. authority as part of the Madras Presidency.
Maulana Mohammad Ali was a prominent leader of the All- • The British had introduced new tenancy laws that
India Muslim League and a staunch advocate of Muslim tremendously favoured the landlords known as Janmis
separatism. He attended the First Round Table Conference
and instituted a far more exploitative system for
in London as a representative of the Muslim League. The
peasants than before.
Indian National Congress only attended the Second
Round Table conference out of all the three Round Table • The new laws deprived the peasants of all guaranteed
Conference. rights to the land, share in the produce they earlier got
and in effect rendered them landless.
Important Tips
• Most of the landlords were Namboodiri Brahmins
Organisations and their representatives in the First
while most of the tenants were Mapillah Muslims.
Round Table Conference:
• Fuelled by the fiery speeches by Muslim religious
Muslim League: Muhammad Zafarullah Khan,
Muhammad Ali Jinnah, Sir Abdul Halim Ghuznavi, leaders and anti-british sentiments, the Mopillahs
Maulana Mohammad Ali Jauhar, Raja Sher Muhammad launched a violent rebellion. Numerous actions of
Khan of Domeli, Sir Ghulam Hussain Hidayatullah, Aga violence were reported, and series of persecutions
Khan III (leader of British-Indian delegation), Muhammad were committed both against the British and the Hindu
Shafi, AK Fazlul Huq, Dr. Shafa’at Ahmad Khan. landlords.
Depressed Classes: Rettamalai Srinivasan, B. R. Ambedkar • By the end of 1921, the rebellion was crushed by the
Sikhs: Sardar Sampuran Singh, Sardar Ujjal Singh British who had raised a special battalion, the Malabar
Liberals: Tej Bahadur Sapru, V. S. Srinivasa Sastri, J. N. Special Force for the riot.
Basu, Chimanlal Harilal Setalvad, C. Y. Chintamani • In November 1921, 67 Moplah prisoners were killed
Europeans: T. F. Gavin Jones, Sir Oscar de Glanville when they were being transported in a closed freight
(Burma), Sir Hubert Carr, C. E. Wood (Madras) wagon from Tirur to the Central Prison in Podanur.
Parsis: Homi Mody, Cowasji Jehangir, Phiroze Sethna They died of suffocation. This event is called the
Wagon Tragedy.
Anglo-Indians: Henry Gidney
Justice Party: Bhaskarrao Vithojirao Jadhav, Sir A. P. 238. Solution: (b)
Patro, Arcot Ramasamy Mudaliar Exp) Option b is the correct answer.
Indian Christians: K. T. Paul (All India Conference of
Maulana Abul Kalam Azad was the president of the
Indian Christians)
Congress Working Committee in 1942, when Mahatma
Women: Radhabai Subbarayan, Begum Jahanara Gandhi discussed his Quit India Policy at the Bombay session
Shahnawaz of the All-India Congress Committee on 9 August 1942.

PYQ Workbook 204


HISTORY OF MODERN INDIA

Important Tips 1913 by Lala Hardayal.

Maulana Abul Kalam Azad: He joined the Indian National 242. Solution: (a)
Congress in 1920 and became a close associate of Mahatma Exp) Option a is the correct answer.
Gandhi. He participated in the Khilafat Movement, the
The Civil Disobedience Movement was put on hold in 1934
Non-Cooperation Movement, the Dharasana Satyagraha,
when the Congress ended the mass satyagraha. Following
and the Quit India Movement. He was elected as the
the setback of the Civil Disobedience movement, the decision
president of the Indian National Congress twice, in 1923
and 1940. He was the youngest person to hold the post at was made to shift attention towards constructive endeavors.
the age of 35. This new approach encompassed various activities such as
khadi work, advocating temple entry, fostering communal
239. Solution: (a) unity, and combating untouchability.
Exp) Option a is the correct answer. Important Tips
The slogan “Swaraj should be for common people not only The constructive programs included:
for classes” was coined by Chittaranjan Das, commonly
Khadi: The promotion of khadi, or hand-spun and hand-
known as C.R. Das. He championed a more active approach
woven cloth, as a symbol of self-reliance and economic
to achieving Swaraj (self-rule) during the 37th Indian
independence.
National Congress session in 1922. This slogan reflected his
belief that independence should benefit the masses and not Village uplift: The improvement of rural areas through
just specific classes. Das’s stance led to the formation of the education, health care, and sanitation.
“Pro-Change” faction, advocating for more direct tactics and Hindu-Muslim unity: The promotion of Hindu-Muslim
participation in government Councils. His slogan signified unity in order to counter the communalism that was being
the evolving dynamics and ideologies within the Indian fanned by the British.
nationalist movement during that period. Removal of untouchability: The campaign to end the
240. Solution: (b) practice of untouchability, which discriminated against
millions of Indians.
Exp) Option b is the correct answer.
The agitation against the Chaukidari Tax in Begusarai was a 243. Solution: (a)
significant component of the Civil Disobedience Movement. Exp) Option a is the correct answer.
As Bihar lacked access to the sea, the Salt Satyagraha, a The Second Round Table Conference failed to reach an
prominent aspect of the movement, was not feasible here. agreement on the issue of communal representation.
Instead, the local residents initiated a campaign by refusing The Indian National Congress wanted a system of joint
to pay the Chowkidari Tax, which was a levy imposed on electorates, where all Indians would vote for all seats,
farmers in 1907 to support village watchmen. This tax regardless of their religion. The Muslim League wanted a
disproportionately burdened poorer farmers, prompting a
system of separate electorates, where Muslims would vote
revolt in 1930 as part of the civil disobedience movement,
for Muslim seats and Hindus would vote for Hindu seats.
led by Swami Sahajanand Saraswati, a Hindu monk and
The British government tried to mediate between the two
social reformer. The movement achieved success in areas like
sides, but they were unable to reach a compromise. The
Bhagalpur, Munger, and Saran, resulting in the dismissal of
conference ended without any agreement being reached on
chowkidars and panchayat members responsible for their
the issue of communal representation. Other than Communal
appointments. However, the British responded with a harsh
delegation, the princes were also not too enthusiastic about a
crackdown, arresting and imprisoning many movement
federation, especially after the possibility of the formation of
leaders, including Saraswati himself, who endured several
a Congress government at the center had receded following
years of incarceration.
the suspension of the civil disobedience movement.
241. Solution: (b)
244. Solution: (d)
Exp) Option b is the correct answer.
Exp) Option d is the correct answer.
The postponement of the Non-Cooperation Movement
Gandhiji was arrested on 9 August 1942, the day after he
led to a sense of disappointment and frustration among
many Indian people. This led to a revival of revolutionary launched the Quit India Movement at the Bombay session
activities. Many young people felt that the only way to of the All-India Congress Committee. He was detained in
achieve independence was through violence. They formed Aga Khan Palace Jail in Pune, along with his wife Kasturba
secret societies and carried out acts of sabotage and terrorism. Gandhi, his secretary Mahadev Desai, and other Congress
Some of the most prominent revolutionary groups during leaders such as Sarojini Naidu and Acharya Kripalani.
this period included the Hindustan Socialist Republican Gandhiji was in the prison at Poona (Aga Khan Palace) for
Association (HSRA) and the Ghadar Party. The HSRA was almost two years, until May 1944. He suffered endless agony
founded in 1928 by Chandrashekhar Azad, Bhagat Singh, for the activities that the British did to crush the Quit India
and Sukhdev Thapar. The Ghadar Party was founded in Movement. The British Government charged Gandhi that he

205 PYQ Workbook


HISTORY OF MODERN INDIA

and his congress party were responsible for all violence and On 9 June, 1920 the Khilafat Committee at Allahabad
disorder in the country. unanimously accepted the suggestion of non-cooperation
Important Tips and asked Gandhiji to lead the movement. Subsequently,
the Khilafat Movement merged with the Non-Cooperation
Gandhiji faced personal tragedies during his
imprisonment prison at Poona (Aga Khan Palace): Movement launched by Mahatma Gandhi in 1920.
• His secretary Mahadev Desai died of a heart attack on 248. Solution (c)
15 August 1942, six days after their arrest.
Exp) Option c is the correct answer.
• His wife Kasturba Gandhi died of pneumonia on 22
February 1944, after being ill for several months. Under the leadership of leaders like Jawaharlal Nehru and
Sardar Vallabhbhai Patel, Congress adopted a significant
• Gandhiji himself suffered from malaria and was
released on medical grounds on 6 May 1944. policy aimed at abolishing landownership and land grants in
India. Finally in 1945, the Congress Working Committee
245. Solution: (a) adopted the policy of abolishing land ownership and land
Exp) Option a is the correct answer. grants to the tiller. The resolution on abolishing landlordism
Jai Prakash Narayan got the recognition of national leader was moved by Acharya Narendra Dev.
in the context of the Quit India Movement, which was a
mass civil disobedience campaign launched by Mahatma Important Tips
Gandhi and the Indian National Congress in August 1942, Congress Working Committee (CWC):
demanding an end to British rule in India. • The Congress Working Committee (CWC) is the
He was arrested by the British authorities on 9 August 1942, executive committee of the Indian National Congress.
along with other Congress leaders, but he managed to escape It was formed in December 1920 at Nagpur session of
from the Hazaribagh jail in Bihar on 9 November 1942. He INC which was headed by C. Vijayaraghavachariar.
then went underground and tried to organize a guerrilla • It is composed of senior party leaders and is
resistance against the British government. He formed an responsible for taking decisions on important policy
underground Azad Dasta (Free Force) and established and organizational matters, as well as guiding and
contacts with other revolutionaries and groups across the directing the party’s activities and campaigns at the
country. He also issued a manifesto titled The Last Battle of national level.
Freedom, in which he called for a total revolution against the • At Nagpur session of 1920 of the Indian National
British and their collaborators. Congress, some important organisational changes were
made:
246. Solution: (a)
• A congress working committee (CWC) of 15 members
Exp) Option a is the correct answer
was set up to lead the Congress from now onwards;
The Communist International was an international
• Provincial congress committees on linguistic basis
communist organization that advocated world communism.
were organised;
Manabendra Nath Roy was a delegate to the congress of
the Communist International. He was first Indian to be the • Ward committees were organised; and the entry fee
member of Communist International. was reduced to four annas.

Important Tips 249. Solution: (b)


M.N. Roy: Exp) Option b is the correct answer.
• Manabendra Nath Roy was born on 21 March 1887 in The Indian National Army (INA) was the result of Mohan
West Bengal. Singh’s efforts, who played a pivotal role in its foundation.
• He was founding member of the Communist Party of He was convinced by Fujiwara, a Japanese official, to
India. unite with the Japanese mission for the purpose of Indian
• He established the Communist Party in 1917 in Mexico. independence. Mohan Singh initially helped Fujiwara take
• At 14, Roy joined the underground revolutionary control of the situation in Alor Star, and after meetings
organisation Anushilan Samiti. After it was banned, with Japanese officials, he was convinced of the feasibility
he helped to organise the Jugantar Group under the of raising an armed Indian unit. This led to the formation
leadership of Jatin Mukherjee.
of the nucleus of what became the Ajad Hind Fauj (Indian
• He was sentenced to six years imprisonment in 1931 for
National Army) in 1942. Mohan Singh played a crucial
his involvement in 1924 Kanpur Bolshevik Conspiracy
role in recruiting Indian prisoners of war and volunteers
case.
to fight for India’s independence from British rule. While
247. Solution (c) initially playing a significant role, leadership was later taken
Exp) Option c is the correct answer over by Subhash Chandra Bose.

PYQ Workbook 206


HISTORY OF MODERN INDIA

Important Tips discuss the Wavell Plan. Differences emerged as Jinnah


insisted on exclusive Muslim League representation and veto
Formation of INA:
power. The conference failed, leading to dissolution and the
• During World War II, Indian revolutionary Rashbehari approach of Indian independence.
Bose, who had been living in Japan since 1915 to evade
INA Trials, Nov 1945: INA Trials were held against Indian
British authorities, saw an opportunity to mobilize
National Army (INA) officers who had fought against
Indians for armed resistance against British rule.
the British during World War II. The trials were highly
• With numerous Indian soldiers serving the British, controversial, and they ultimately led to the release of the
the Japanese, after defeating the British in Southeast INA officers.
Asia, convinced Captain Mohan Singh to collaborate
Naval Mutiny, Feb 1946: This was a mutiny of Indian sailors
with them for India’s freedom.
in the Royal Indian Navy. The mutiny was triggered by a
• A conference in Tokyo in 1942 led to the formation number of grievances, including low pay and poor living
of the Indian Independence League. Subsequently, in conditions. The mutiny was suppressed by the British, but it
Bangkok (June 1942), Rashbehari Bose was elected had a major impact on Indian politics.
its president, and the decision to establish the Indian
National Army (INA) was made. Captain Mohan 252. Solution: (b)
Singh became its commander with around 40,000 Exp) Option b is the correct answer.
Indian soldiers. Dr. M.A. Ansari was the President of the ‘All Parties’
• Bose, who had earlier escaped to Berlin in 1941, was Conference held in February 1928. The conference was
invited to lead the movement. In June 1943, Bose significant in Indian political history as it brought together
arrived in Tokyo and joined the INA in Singapore. various political parties, including the Indian National
Rashbehari Bose handed leadership to Subhas Bose, Congress, All-India Muslim League, All-India Hindu
resulting in the formation of the Azad Hind Sarkar. Mahasabha, and others, to discuss and deliberate on
• The Japanese later announced their plan to transfer important constitutional matters, leading to the adoption
administration of the Andaman and Nicobar Islands of the Nehru Report.
to the INA in November 1943. This marked the 253. Solution: (d)
commencement of the INA’s valiant struggle for India’s
independence. Exp) Option d is the correct answer.
In August Kranti Maidan, the Quit India Movement was
250. Solution: (d) launched by Mahatma Gandhi in 1942. It is also known
Exp) Option d is the correct answer. as Gowalia Tank Maidan and located in Central Mumbai.
The Pakistan Movement, which took place in the first half On 8 August 1942, Mahatma Gandhi delivered the Quit
of the 20th century, was a movement that wanted to create India speech decreeing that the British must quit India
a new country called Pakistan. This country would be made immediately or else there would be mass agitations. He called
from parts of British India where most people were Muslims. for resolute yet passive resistance which meant the certainty
The movement happened because Muslims under British that Gandhi foresaw the revolution, best represented by
rule wanted to have their own say in things. Muhammad his call to do or die. Gandhi made the speech to help India
Ali Jinnah, who was a lawyer and a politician, led this become independent.
movement after the All-India Muslim League passed a 254. Solution: (e)
important decision known as the Lahore Resolution on 23
Exp) Option e is the correct answer.
March 1940. Another person named Ashraf Ali Thanwi,
who was a religious scholar, also supported this idea. People Netaji Subhash Chandra Bose became the President of
like Shabbir Ahmad Usmani and Zafar Ahmad Usmani, India National Congress by defeating P. Sitaramaiya in
who were students of Thanwi, played a big role in supporting Tripuri Session of 1939. He was also the President of Indian
the creation of Pakistan based on religion. National Congress during Haripura Session of 1938 but he
was unanimously elected in that session.
251. Solution (c)
Important Tips
Exp) Option c is the correct answer.
Pattabhi Sitaramayya:
Wavell Plan, May 1945: The Wavell Plan proposed a
• He served as a member of the Constituent Assembly
restructured Viceroy’s Executive Council with a balanced
and later became the first governor of Madhya
representation of Indian communities, excluding Viceroy
Pradesh in 1956.
and Commander-in-Chief, and transfer of foreign affairs
portfolio to an Indian member. It aimed to encourage wider • In his book, History of the Indian National Congress,
Indian participation in governance while retaining a power- he provides a comprehensive account of Congress
sharing arrangement. party. Because of Dr. Pattabhi SItaramayya’s efforts to
Shimla Conference, June 1945: The Shimla Conference document history of the Indian National Congress, he
invited Indian leaders, including Gandhi and Jinnah, to is often referred to as ‘historian of the Congress’.

207 PYQ Workbook


HISTORY OF MODERN INDIA

• In 1919, he started Nationalist English Weekly Important Tips


‘Janambhumi’. Through this, he propagated the ideas of Events and their occurance:
non-violence, satyagraha, a boycott of foreign goods,
• Dandi March Event- 12 March, 1930.
advantages of the cooperative movement, etc.
• Quit India Movement Event- 8th August, 1942.
• He has also authored ‘Gandhi and Gandhism’;
‘Gandhism and Socialism’; ‘Feathers and Stones’ • Gandhi-Irwin Pact Event- 5th March, 1931.
along with several others. 257. Solution: (d)
• He strongly recommended National Banking and
Exp) Option d is the correct answer
Insurance Companies to stop the flow of the Indian
wealth to Britain. Keeping this in mind he established The centerpiece of Individual Satyagraha was Non-
Andhra Bank in 1923 and ‘Hindustan Insurance violence. The first Satyagrahi selected by Mahatma
Company’ in 1935. Gandhi for this Satyagraha was Vinoba Bhave. He was
followed nearly by 25,000 individual satyagrahis. It was
• He was elected as the President of All India Congress at
launched to protest against the British government’s
Jaipur in 1948 and in 1952.
decision to participate in World War II without consulting
255. Solution: (d) the Indian people. The movement called on individuals to
violate certain laws of the British Raj, such as the Defense
Exp) Option d is the correct answer.
of India Act, as a form of nonviolent protest. In December
The two brothers Gangaram and Kheemdev were martyred 1940, Gandhi suspended the movement and the campaign
in the Quit India Movement, which was a mass civil started again in January 1941. The aim of the movement was
disobedience campaign launched by Mahatma Gandhi and to show that nationalist patience was not due to weakness. It
the Indian National Congress in August 1942, demanding an also aims to express the feeling that people are not interested
end to British rule in India. in war and they made no distinction between nazism and the
double autocracy that ruled India. Another opportunity for
Important Tips the government to accept Congress’ demands.
Gangaram and Kheemdev:
Important Tips
• The two brothers Gangaram and Kheemdev were from
Vinobha Bhave:
the Salt region of Uttarakhand, which was also known
as Kumaon ka Bardoli, a reference to the famous • He started the Sarvodaya Movement which meant
peasant movement led by Sardar Vallabhbhai Patel in ‘Progress for all’.
Gujarat in 1928. • In 1951, Bhave started the Bhoodan Movement in
• The Salt region was known for its active participation Pochampally, Telangana.
in the freedom struggle and its resistance to the British • He was awarded the Ramon Magsaysay Award for
oppression Community Leadership in 1958.
• On September 5, 1942, at a place called Khumad, which • In 1983, he received the Bharat Ratna posthumously.
was the headquarters of the Salt Congress, a local
branch of the Indian National Congress, the British 258. Solution (c)
army opened fire on the public who were protesting Exp) Option c is the correct answer.
against the arrest of their leaders and the suppression The basic concept behind the Two-Nation Theory was that
of their movement. Hindus and Muslims were two separate nations with their
• Gangaram and Kheemdev were among the 21 freedom own distinct cultures, histories, and religious beliefs. This
fighters of Uttarakhand who were martyred in this theory was first proposed by Sir Mohammad Iqbal in 1930,
incident who argued that Muslims would never be able to achieve
equality in a united India. He believed that Muslims needed
256. Solution (c) to have their own separate state in order to protect their
Exp) Option c is the correct answer. culture and identity.
The Simon Commission was a British commission appointed 259. Solution: (b)
in 1927 to investigate the political situation in India and
Exp) Option b is the correct answer.
to make recommendations for constitutional reforms.
Mahatma Gandhi drafted a resolution in April of 1942
The commission arrived in India in 1928 and was met with
with the help of Jawaharlal Nehru and Maulana Abul
widespread protests because there were no Indian members Kalam Azad calling upon the British to Quit India. The
in the Commission. The Commission submitted its report movement demanded an end to British rule in India and
in 1930 and ignored the demands of Dominion Status for included boycott of the British government and rejection
India. It was seen as a British attempt to deny Indians self- of transactions involving the government. Various violent
government. incidents took place around the country against the British

PYQ Workbook 208


HISTORY OF MODERN INDIA

regime. The movement ended in 1945 with the release of the promotion of Hindi as the national language, and the
jailed freedom fighters. protection of civil liberties.

260. Solution: (d) 264. Solution (c)


Exp) Option d is the correct answer. Exp) Option c is the correct answer.
Indians were excluded from the Simon Commission on In 1928, Jawaharlal Nehru, along with Srinivasa Iyengar
the suggestion of Lord Irwin. He argued that no Indian and Subhash Chandra Bose, founded the Independence
member would be able to approach the task of impartially for India League. Srinivasa Iyengar served as its first
reviewing the workings of the Government of India president. This league was established by the radical wing of
Act 1919, as they would inevitably be biased or self- the Congress Party, led by Nehru, as a response to the Nehru
interested. This decision to have an all-British commission Report. The Nehru Report, despite advocating constitutional
led to widespread protests and demands for greater Indian reforms, was criticized by Congress radicals for not explicitly
representation, as Indians felt that their perspectives were demanding full independence and settling for dominion
not being considered in matters that directly concerned their status. The “Independence for India League” reflected a
governance. more assertive approach in seeking complete independence.

261. Solution: (a) 265. Solution (c)


Exp) Option a is the correct answer. Exp) Option c is the correct answer.
Option 1 is correct: The All India Muslim League which Asaf Ali held the Railways Portfolio in the Interim
was annoyed with the Congress for not sharing power with Government of 1946. As the railway minister, he made
them established the Pirpur Committee in 1938 to prepare an early move to end the practice of providing separate
a detailed report on the atrocities supposedly committed drinking water on railway platforms, previously labeled as
by the Congress ministries on Muslims. In its report, the Hindu Pani and Muslim Pani. In his personal and political
life, Ali actively worked to bridge religious differences and
committee charged the Congress with interference in the
promote unity.
religious rites, suppression of Urdu in favour of Hindi, denial
of proper representation and of the oppression of Muslims in 266. Solution: (b)
the economic sphere.
Exp) Option b is the correct answer.
Option 2 is correct: As per the findings of Sharif Report The title “Kaiser-i-Hind” means “Emperor of India” in
of 1938, congress was alleged to be pro-Hindu and anti- Urdu. It was a high honor bestowed upon individuals who
Muslim. had rendered exceptional services to the British Empire in
Option 3 is correct: A. K. Fazlul Haq’s “Muslim Sufferings India. Mahatma Gandhi was awarded the title in 1915 for
Under Congress Rule” published in 1939 concluded that his work in South Africa during the Boer War. However, he
Congress was representing Hindus voice and voice of surrendered the title in 1920 during the Non-Cooperation
eminent British personalities. Movement as a gesture of his opposition to British rule in
India.
262. Solution (c)
Exp) Option c is the correct answer. 267. Solution: (d)
Quit India Resolution was a movement launched by Exp) Option d is the correct answer.
Mahatma Gandhi on 9 August 1942, during World War II, On the historic day of 31st December 1929, Jawaharlal
demanding an end to British rule in India. The resolution Nehru unfurled the flag of India on the banks of the Ravi
was passed by the All-India Congress Committee (AICC) in River. This symbolic act of hoisting the flag embodied
Bombay (present-day Mumbai), Maharashtra, after Gandhi the spirit of the Indian people’s aspiration for complete
made a call to Do or Die in his Quit India speech. On independence. The Indian National Congress (INC)
August 8, 1942 in the meeting of AICC, Pandit Jawaharlal adopted the resolution of ‘Poorna Swaraj’ on 19th
Nehru presented Quit India Resolution and Sardar Patel December 1929 in Lahore. This resolution signified the
supported it. Congress’s unwavering dedication to attaining full self-
governance and liberating the nation from British rule.
263. Solution (c)
268. Solution: (d)
Exp) Option c is the correct answer.
Exp) Option d is the correct answer.
Ram Manohar Lohia was one of the prominent leaders of
the Indian independence movement who made regular The Hindu Mahasabha, The Communist Party of India, and
broadcasts on Congress Radio during the Quit India The Unionist Party of Punjab did not support Quit India
Movement. He was arrested several times for his anti- Movement. These parties had different reasons for opposing
British speeches and activities, especially during the Quit the movement, such as:
India Movement of 1942. He left the Congress in 1948 and • The Muslim League opposed the movement, fearing
formed his own Socialist Party in 1955. He advocated for that if the British left India at that time, the minorities
various social reforms, such as the abolition of caste system, would be oppressed by the Hindus.

209 PYQ Workbook


HISTORY OF MODERN INDIA

• The Communists did not join the movement; in the Important Tips
wake of Russia (where the communists were in power)
Works and Achievements of Swaraj Party:
being attacked by Nazi Germany, the communists began
to support the British war against Germany and the • Electoral Success: The Swarajists emerged as the
‘Imperialist War’ became the ‘People’s War’. largest party in the Central Assembly, Bombay, and
Bengal Councils in 1923, with a notable presence in
• The Hindu Mahasabha openly opposed the call for the
the U.P. Council.
Quit India Movement and boycotted officially under the
apprehension that the movement would create internal • Preserving Anti-British Sentiment: During a period
disorder and would endanger internal security during with limited mass political activities, the Swarajists
the war. played a crucial role in maintaining the spirit of anti-
British protest.
269. Solution (c) • Obstruction of British Policies: The Swarajists
• Exp) Option c is the correct answer. obstructed British policies and proposals in legislatures,
Sardar Vallabhbhai Patel was not one of the advocates who making it challenging for the British to gain approval
defended the INA officers in the Red Fort trials. The INA for their measures.
Defence Committee was established by the Indian National • Defeat of Repressive Bills: Notably, in 1928, they
Congress with the purpose of providing legal defense and defeated a bill that would have empowered the
support to the officers of the Indian National Army (INA) government to expel foreign supporters of India’s
who were facing charges during the INA Red fort trials. This freedom struggle.
committee comprised a group of prominent leaders who were • Exposing Reforms’ Weaknesses: They highlighted
dedicated to the cause. The notable individuals included the shortcomings of the Montagu-Chelmsford reforms
Jawaharlal Nehru, Tej Bahadur Sapru, Bhulabhai Desai, through fiery speeches in the Assembly, emphasizing
R.B. Badri Das, Asaf Ali, Kanwar Sir Dalip Singh, Kailash self-rule and civil liberties.
Nath Katju, Bakshi Sir Tek Chand, P.N. Sen, Inder Deo Dua, • Legislative Impact: The years 1924-25 saw numerous
Shiv Kumar Shastri, Ranbeer Chand Soni, Rajinder Narayan, Swarajist victories in the Legislative Assembly.
Sultan Yar Khan, Narayan Andley, and J.K. Khanna. They managed to reject the Budget, compelling the
270. Solution: (b) government to rely on certification powers.

Exp) Option b is the correct answer. • Questioning and Adjournment Motions: Swarajists
effectively employed adjournment motions and
Statement 1 is correct: The Non-Cooperation Movement probing questions to unveil the wrongdoings of the
led by Mahatma Gandhi gained significant momentum but colonial government, exposing its misdeeds.
was abruptly withdrawn after the Chauri Chaura incident
in 1922. Many leaders and activists felt demoralized by the 271. Solution (c)
withdrawal and believed that political engagement within the Exp) Option c is the correct answer.
legislative councils was necessary to continue the struggle
Mahatma Gandhi, Sarojini Naidu, and Madan Mohan
for self-rule.
Malviya all participated in the Second Round Table
Statement 2 is correct: The Government of India Act of Conference. Maulana Azad did not participate in the
1919 introduced limited self-government in India through Second Round Table Conference. Maulana Azad was a
the formation of legislative councils. The Swaraj Party prominent leader of the Indian National Congress, but he
decided to enter these councils and use them as platforms did not participate in the Second Round Table Conference
to oppose repressive laws, inadequate representation, and to because he was not in favor of the Gandhi-Irwin Pact. He
demand more powers for Indians in governance. While they believed that the pact gave too many concessions to the
participated, they also resisted and criticized the functioning British government.
of these councils.
272. Solution: (a)
Statement 3 is incorrect: While repression by the British
Government might have played a role in shaping the Exp) Option a is the correct answer.
context of political decisions, it is not a direct reason for Mahatma Gandhi started the Dandi March from his
the founding of the Swaraj Party. The party was formed to Sabarmati Ashram in Ahmedabad, Gujarat, on March
engage in legislative councils strategically rather than as a 12, 1930. The march was a protest against the British salt
direct response to repression. monopoly. Gandhi and his followers marched for 241 miles
Statement 4 is incorrect: While there might have been (385 km) to Dandi, Gujarat, where he broke the salt law by
some sentiment among Indians to gain experience in making salt from seawater.
administration, it was not a primary reason for the founding Important Tips
of the Swaraj Party. The main focus was on utilizing the
Various Ashram of Gandhi:
legislative councils as a platform to push for self-governance
and political rights. Pavnar Ashram was founded by Vinoba Bhave in 1934.

PYQ Workbook 210


HISTORY OF MODERN INDIA

Sewagram Ashram (Village of Service) was founded by its supreme commander. He established the INA
Gandhi in 1936 in his village shegaon. headquarters in Rangoon and initiated a process to
reorganize and strengthen the INA. The headquarters in
Ramanandiya Ashram was founded by Ramananda in the
Rangoon, a strategic location during World War II, served
15th century.
as a significant operational center for the INA’s activities in
273. Solution (c) Southeast Asia. The INA’s efforts were geared towards freeing
India from British colonial rule through military means.
Exp) Option c is the correct answer.
The slogan ‘Quit India’ was coined by Yousuf Meher Ali. The 276. Solution (c)
iconic ‘Quit India’ slogan was coined by socialist Congress Exp) Option c is the correct answer.
leader and lesser-known hero of the Indian National The Indian National Army trials, also known as the Red Fort
movement Yusuf Meher Ali in 1942. He was the founder of trials, were a series of court-martial proceedings conducted
National Militia, Bombay youth League, and the Congress by the British Indian authorities between November 1945
Socialist Party. and May 1946. These trials involved officers of the Indian
Important Tips National Army (INA) who fought for India’s independence
alongside the Japanese during World War II. Colonel Prem
Famous slogans during Indian freedom struggle:
Sahgal, Colonel Gurbaksh Singh Dhillon, and Major-
• Tum mujhe khoon do, main tumhe azaadi doonga. - General Shah Nawaz Khan faced charges of “waging war
Subhash Chandra Bose. against the King-Emperor,” murder, and abetment of
• Inqalab Zindabad. - Shaheed Bhagat Singh. murder.
• Karo ya maro (Do or die) – Mahatma Gandhi. Mohan Singh did not face these trials. Mohan Singh
• Satyameva Jayate. - Pandit Madan Mohan Malviya. was initially the leader of the INA, but in June 1943 he
was replaced due to disagreements between him and the
• Sarfaroshi ki tamanna ab hamare dil me hai–
Japanese, over both the intended size and the specific
Ramprasad Bismil
role of the INA. Due to this leadership change and his
274. Solution: (d) subsequent lack of involvement with the INA’s actions
Exp) Option d is the correct answer. under Bose’s leadership, he did not face the Red Fort trials.

The November 1931 Second Round Table Conference saw Important Tips
the participation of 31 delegates, among them Pyarelal Key Highlights of Red Fort Trials:
Nayyar, Sarojini Naidu, Annie Besant, Mahadan Mohan
• Initiated by British Indian authorities to prosecute INA
Malviya, Mahadev Desai, and B.R Ambedkar. However,
officers.
Jawaharlal Nehru chose not to take part in this conference.
• Congress, led by Jawaharlal Nehru, accepted
Important Tips responsibility for the trials. A defense committee,
Mahadev Desai: including Nehru, Bhulabhai Desai, Tej Bahadur
Sapru and others, was formed to support INA
• Close associate of Mahatma Gandhi and key figure in
soldiers.
Indian independence movement.
• Over 7,600 INA members initially faced trial, but the
• Proficient scholar in Gujarati and Hindi.
number was reduced due to evidentiary challenges.
• Played a crucial role in the Salt March and other • About ten court-martials were conducted, with the first
independence movement events. involving Prem Sahgal, Gurbaksh Singh Dhillon, and
• Tragically died in 1942 during a hunger strike in Shah Nawaz Khan. These officers had served in the
support of the Quit India Movement. British Indian Army and joined INA, facing charges
Pyarelal Nayyar: of waging war against the King-Emperor, murder, and
abetment.
• Prominent in India’s independence movement, close to
Gandhi. • Trials encompassed legal arguments in military,
constitutional, and international law, becoming a
• Proficient in Gujarati and Hindi, translated Gandhi’s notable anticolonial critique.
works.
• Lt. Col. Horilal Varma and Tej Bahadur Sapru
• Actively involved in events like the Salt March. defended the accused.
• Trusted friend who was with Gandhi during his passing • Public outcry and growing unrest led Field Marshal
in 1948. Claude Auchinleck to commute the sentences of the
first trial’s defendants, symbolizing the trials’ impact
275. Solution: (b)
on India’s independence movement.
Exp) Option b is the correct answer.
In August 1943, Netaji Subhas Chandra Bose assumed 277. Solution: (b)
leadership of the Indian National Army (INA), becoming Exp) Option b is the correct answer.

211 PYQ Workbook


HISTORY OF MODERN INDIA

Ganesh Vasudev Mavlankar (G.V. Mavlankar) held the Parliament (1947-52). In 1952, he was elected the Speaker of
Office of Speaker in Central Legislative Assembly of British the first Lok Sabha. He was also known as ‘Dada Saheb’. He
era and the Parliament of free India. He was the Speaker of was referred to as the ‘Father of the Lok Sabha’ by Jawaharlal
the Constituent Assembly (Legislative) and the Provisional Nehru.

PYQ Workbook 212


HISTORY OF MODERN INDIA

HISTORY OF MODERN INDIA


GOVERNANCE AND OTHER ASPECTS
*This unit consists of questions form Constitutional, Administrative and Judicial Developments, the
Development of press and the Development of Education under the British Rule.

5.1. UPSC CSE Previous Years’ Questions (a) Fundamental Rights


(b) Directive Principles of State Policy
1. By which one of the following Acts was the (c) Extent of Executive Power of State
Governor General of Bengal designated as (d) Conduct of Business of the Government
the Governor General of India? of India
[UPSC CSE Pre 2023]
5. By a Regulation in 1793, the District
(a) The Regulating Act
(b) The Pitt’s India Act Collector was deprived of his judicial
(c) The Charter Act of 1793 powers and made the collecting agent only.
(d) The Charter Act of 1833 What was the reason for such regulation?
[UPSC CSE Pre 2010]
2. In the Government of India Act 1919, the
(a) Lord Cornwallis felt that the District
functions of Provincial Government were
divided into “Reserved” and “Transferred” Collector’s efficiency of revenue
subjects. Which of the following were collection would enormously increase
treated as “Reserved” subjects? without the burden of other work.
1. Administration of Justice (b) Lord Cornwallis felt that judicial power
2. Local Self-Government should compulsorily be in the hands of
Europeans, while Indians can be given the
3. Land Revenue
job of revenue collection in the districts.
4. Police
(c) Lord Cornwallis was alarmed at the extent
Select the correct answer using the code given of power concentrated in the District
below: [UPSC CSE Pre 2022] Collector and felt that such absolute
(a) 2, 3 and 4 power was undesirable in one person.
(b) 1, 2 and 3 (d) The judicial work demanded a deep
(c) 1, 3 and 4 knowledge of India and a good training in
(d) 1, 2 and 4 law and Lord Cornwallis felt that District
3. Consider the following statements: Collector should be only a revenue
1. The Montagu-Chelmsford Reforms of collector.
1919 recommended granting voting
6. In collaboration with David Hare and
rights to all the women above the age of
Alexander Duff, who of the following
21.
established Hindu College at Calcutta?
2. The Government of India Act of 1935
gave women reserved seats in legislature. [UPSC CSE Pre 2009]
(a) Henry Louis Vivian Derozio
Which of the statements given above is/are
correct? [UPSC CSE Pre 2021] (b) Ishwar Chandra Vidyasagar
(c) Keshab Chandra Sen
(a) 1 only
(d) Raja Ram Mohan Roy
(b) 2 only
(c) Both 1 and 2 7. Which one of the following was a journal
(d) Neither 1 nor 2 brought out by Abul Kalam Azad?
4. The ‘Instrumentation of Instructions” [UPSC CSE Pre 2008]
contained in the Government of India (a) Al-Hilal
Act, 1935 have been incorporated in the (b) Comrade
Constitution of India in the year 1950 as: (c) The Indian Sociologist
[UPSC CSE Pre 2010] (d) Zamindar

213 PYQ Workbook


HISTORY OF MODERN INDIA

8. Who among the following started the (c) Lord Curzon


newspaper ‘Som Prakash’? (d) Lord Minto
[UPSC CSE Pre 2007] 13. Consider the following statements:
(a) Dayanand Saraswati
Some of the main features of the Government
(b) Ishwar Chandra Vidyasagar
of India Act, 1935 were the:
(c) Raja Rammohan Roy
(d) Surendranath Banerjee 1. Abolition of diarchy in the Governor’s
provinces.
9. Consider the following statements: 2. The power of the Governors to veto
1. The Charter Act, 1853 abolished East legislative action and to legislate on their
India Company monopoly of Indian own.
trade. 3. Abolition of the principle of communal
2. Under the Government of India Act, 1858 representation.
the British Parliament abolished the East Which of the statements given above is/are
India Company altogether and undertook correct? [UPSC CSE Pre 2004]
the responsibility of ruling India directly.
(a) 1 only
Which of the statements given above is/are (b) 1 and 2
correct? [UPSC CSE Pre 2006] (c) 2 and 3
(a) 1 only (d) 1, 2 and 3
(b) 2 only
(c) Both 1 and 2 14. During the colonial period in India,
(d) Neither 1 nor 2 what was the purpose of the Whitley
Commission? [UPSC CSE Pre 2003]
10. Consider the following statements: (a) To review the fitness of India for further
1. Warren Hastings was the first Governor- political reforms
General who established a regular police (b) To report on existing conditions of labour
force in India on the British pattern. and to make recommendations
2. A Supreme Court was established at (c) To draw up a plan for financial reforms
Calcutta by the Regulating Act, 1773. for India
3. The Indian Penal Code came into effect (d) To develop a comprehensive scheme for
in the year 1860. Civil Services in India
Which of the statements given above are 15. Which one of the following provisions was
correct? [UPSC CSE Pre 2005] not made in the Charter Act of 1833?
(a) 1 and 2 [UPSC CSE Pre 2003]
(b) 2 and 3 (a) The trading activities of the East India
(c) 1 and 3 Company were to be abolished.
(d) 1, 2 and 3 (b) The designation of the supreme authority
11. Consider the following statements: was to be changed as the Governor
Government of India Act, 1935 provided for– General of India-in-Council.
(c) All law-making powers to be conferred
1. provincial autonomy.
on Governor-General-in-council.
2. The establishment of the Federal Court. (d) An Indian was to be appointed as a Law
3. All India Federation at the Centre. Member of the Governor-General’s
Which of the following statements given Council.
above are correct? [UPSC CSE Pre 2005]
16. Which one of the following Acts of British
(a) 1 and 2 India strengthened the Viceroy’s authority
(b) 2 and 3 over his executive council by substituting
(c) 1 and 3 “portfolio” or departmental system for
(d) 1, 2 and 3 corporate functioning?
12. Who among the following repealed the [UPSC CSE Pre 2002]
Vernacular Press Act of 1878? (a) Indian Councils Act, 1861
[UPSC CSE Pre 2005] (b) Government of India Act, 1858
(a) Lord Ripon (c) Indian Councils Act, 1892
(b) Lord Lytton (d) Indian Councils Act, 1909

PYQ Workbook 214


HISTORY OF MODERN INDIA

17. Match list-I with list-II and select the the farmer which were not issued by many
correct answer using the codes given below of the Zamindars. The reason was:
the lists: [UPSC CSE Pre 2001]
List-I List-II (a) the Zamindars were trusted by the
farmers
(Acts of Colonial (Provisions)
(b) there was no officials check upon the
Government of
Zamindars
India)
(c) it was the responsibility of the British
A. Charter Act, 1. Set up a Board of government
1813 Control in Britain (d) the farmers were not interested in getting
to fully regulate the pattas
East India Company.
20. Who amongst the following Englishmen,
B. Regulating 2. Company’s trade
first translated Bhagavad-Gita into English?
Act, 1773 monopoly in India
was ended. [UPSC CSE Pre 2001]
(a) William Jones
C. Act of 1858 3. The power to govern
(b) Charles Wilkins
was transferred
(c) Alexander Cunningham
from the East India
(d) John Marshall
Company to the
British Crown. 21. Which one of the following is not a feature
D. Pitt’s India 4. The Company’s of the Government of India Act of 1935?
Act, 1784 directors were asked [UPSC CSE Pre 2000]
to present to the (a) Dyarchy at the Centre as well as in the
British Government provinces
all correspondence (b) A bicameral Legislature
and documents (c) Provincial Autonomy
pertaining to the (d) An All-India Federation
administration of 22. The term “imperial preference” was applied
the company. to the: [UPSC CSE Pre 1999]
Code: [UPSC CSE Pre 2002] (a) Special privileges on British imports in
A B C D India
(a) 2 4 3 1 (b) Racial discrimination by the Britishers
(b) 1 3 4 2 (c) Subordination of Indian interest to that
(c) 2 3 4 1 of the British
(d) 1 4 3 2 (d) Preference was given to British political
18. The real intention of the British to include agents over Indian Princes
the Princely States in the Federal Union 23. There was no independent development
proposed by the India Act of 1935 was to: of industries in India during British rule
[UPSC CSE Pre 2002] because of the: [UPSC CSE Pre 1999]
(a) Exercise more and direct political and (a) Absence of heavy industries
administrative. (b) Scarcity of foreign capital
(b) Involve the princes actively in the (c) Scarcity of natural resources
administration of the colony.
(d) Preference of the rich to invest in land
(c) Finally effect the complete political
and administrative take-over of all the 24. The most short-lived of all of Britain’s
princely States by the British. constitutional experiments in India was:
(d) Use the princes to counterbalance [UPSC CSE Pre 1999]
the anti-imperialist doctrines of the (a) The Indian Council Act of 1861
nationalist leaders. (b) Indian Council Act of 1892
19. Under the Permanent Settlement, 1793, the (c) Indian Council Act of 1909
Zamindars were required to issue pattas to (d) Government of India Act of 1919

215 PYQ Workbook


HISTORY OF MODERN INDIA

25. Match List-I with List-II and select the 28. The foundation of the modern educational
correct answer using the codes given below system in India was laid by:
the lists: [UPSC CSE Pre 1993]
(a) The Charter Act of 1813
List-I List-II
(b) Macaulay’s Minutes of 1835
A. Butler 1. Jallianwala Bagh (c) The Hunter Commission of 1882
Committee massacre (d) Woods Dispatch of 1854
Report
B. Hartog 2. Relationship 5.2. Other Examination Previous Years’
Committee between the Indian Questions
Report State and Paramount 29. “Can you show me one free country where
Power there are separate electorates? ..... The
C. Hunter 3. Working of Dyarchy British element is gone, but they have left
Inquiry as laid down in mischief behind”. Who among the following
Committee the Montague- said the above-mentioned statement in the
Report Chelmsford Reforms Constituent Assembly debates?
D. Muddiman 4. The growth of [U.P.P.C.S. (Pre.) 2021]
Committee education in (a) Somnath Lahiri
Report British India and (b) Jawaharlal Nehru
potentialities of its (c) Sardar Vallabh Bhai Patel
further progress (d) N.G. Ranga
Code: [UPSC CSE Pre 1997] 30. Given below are two statements, one is
A B C D labelled as Assertion (A) and the other as
Reason (R):
(a) 3 2 1 4
(b) 1 4 2 3 Assertion (A): The British Government
(c) 2 1 3 4 introduced different land revenue system in
different part of India.
(d) 2 4 1 3
Reason (R): It led to create different classes
26. Who among the following leaders did not in India peasantry.
believe in the drain theory of Dadabhai Select the correct answer from the codes
Naoroji? [UPSC CSE Pre 1996] given below. [U.P.P.C.S. (Pre) 2020]
(a) B.G. Tilak (a) Both (A) and (R) are true and (R) is the
(b) R.C. Dutt correct explanation of (A).
(c) M.G. Ranade (b) Both (A) and (R) are true but (R) is not
(d) Sir Syed Ahmad Khan the correct explanation of (A).
(c) (A) is true but (R) is false.
27. Which one of the following statements
(d) (A) is false but (R) is true.
correctly defines the term ‘Drain Theory’
as propounded by Dadabhai Naoroji? 31. Match list-I with list-II and select the
[UPSC CSE Pre 1993] correct answer using the codes given below
the lists:
(a) That the resources of the country were
being utilized in the interest of Britain. List-I List-II
(b) That a part of Indians national wealth or A. Jajmani 1. North India
total annual product was being exported B. Bara Balute 2. Karnataka
to Britain for which India got no material
returns. C. Mirasi 3. Maharashtra
(c) That the British industrialists were being D. Adabe 4. Tamil Nadu
given an opportunity to invest in India Code: [U.P.P.C.S. (Pre) 2020]
under the protection of the imperial A B C D
power. (a) 1 2 3 4
(d) That the British goods was being imported (b) 1 3 2 4
to the country making it miserable day by (c) 1 4 2 3
day. (d) 1 3 4 2

PYQ Workbook 216


HISTORY OF MODERN INDIA

32. Given below are two statements, one D. Somaprakash 4. Shivrama Madhav
labelled as Assertion (A) and the other as Paranjape
Reason (R). Code: [U.P.P.C.S. (Mains) 2017]
Assertion (A): The Asiatic Society of Bengal
A B C D
was established in the period of Warren
Hastings, and he modestly declined the offer (a) 4 1 2 3
of Presidentship of that learned body in (b) 4 1 3 2
favour of Sir William Jones. (c) 3 1 4 2
(d) 3 1 2 4
Reason (R): Warren Hastings was himself a
great scholar and an ardent orientalist who 36. Arrange the following events
used to encourage the study of Sanskrit, chronologically:
Persian and Arabic. 1. Vernacular Press Act
Select the correct answer from the codes 2. Rowlatt Act
given below: [U.P.P.C.S. (Pre) 2019] 3. Ban on Sati
(a) Both (A) and (R) are true and (R) is the 4. Woods Despatch
correct explanation of (A). Code: [U.P.P.C.S. (Mains) 2017]
(b) Both (A) and (R) are true, but (R) is not (a) 1, 2, 3, 4
the correct explanation of (A).
(b) 3, 1, 2, 4
(c) (A) is true, but (R) is false.
(c) 3, 4, 1, 2
(d) (A) is false, but (R) is true.
(d) 3, 1, 4, 2
33. Arrange the following events in their
37. Consider the following statements and
chronological order and select the correct
answer from the codes given below: select the correct answer from the code
given below the statements:
A. Hunter Commission
Assertion (A): Generally, India had a
B. Sadlar Commission favourable balance of trade during the British
C. Wood’s Despatch rule.
D. Sargeant Plan Reason (R): The drain of wealth took the
Codes: [U.P.P.C.S. (Pre) 2018] form of unrequired exports.
(a) A B D C Code: [U.P.P.C.S. (Pre) 2017]
(b) C B A D (a) Both (A) and (R) are true, and (R) is the
(c) A B C D correct explanation of (A).
(d) C A B D (b) Both (A) and (R) are true, but (R) is not
34. With which of the following Newspaper/ the correct explanation of (A).
Magazine, Mahatma Gandhi was not (c) (A) is true, but (R) is false.
associated? [U.P.P.C.S. (Mains) 2017] (d) (A) is false, but (R) is true.
(a) Indian Opinion 38. Match list-I with list-II and select the
(b) Young India correct answer using the codes given below
(c) Nav Jeevan the lists:
(d) Yugantar
List-I List-II
35. Match list-I with list-II and select the
correct answer using the codes given below (Commission) (Chairman)
the lists: A. Railway Commission 1. Anthony Mac
(1901) Donnel
List-I List-II
B. Famine Commission 2. Colin Scott
(Newspapers) (Editors)
(1899)
A. Kaal 1. Surendra Nath
C. Irrigation 3. Andrew
Banerjee
Commission (1901) Frazer
B. Bengalee 2. Gopal Krishna
D. Police Reform 4. Thomas
Gokhale
Commission (1902) Robertson
C. Sudharak 3. Dwarkanath Code: [U.P.P.C.S. (Mains) 2017]

217 PYQ Workbook


HISTORY OF MODERN INDIA

A B C D 44. Arrange the following in their chronological


(a) 3 1 4 2 order and select the correct answer with the
(b) 3 2 1 4 help of code given below:
(c) 4 1 2 3 A. Dramatic performances Act
(d) 4 2 1 3 B. Vernacular Press Act
39. With reference to ‘deindustrialization’ C. North-Western Provinces and Oudh Act
which of the following statements is/are D. Bengal Tenancy Act
correct? Code: [U.P.P.C.S. (Mains) 2016]
1. This process started in 1813. (a) A B D C
2. Abolition of monopoly trade rights of East (b) D B A C
India Company aggravated the process. (c) A B C D
Select the correct answer from the codes (d) B C D A
given below: [U.P.P.C.S. (Mains) 2017]
45. In Indian context, the attitude of
(a) 1 only ‘Paternalism in Governance’ is associated
(b) 2 only with: [U.P.P.C.S. (Mains) 2016]
(c) Both 1 and 2
(a) Thomas Munro
(d) Neither 1 nor 2
(b) Charles Grant
40. Which of the following is one of the reasons (c) Holt Mackenzie
for considering the Charter Act of 1813 (d) Macaulay
important for India? 46. Who has said it: ‘I don’t have to beg pardon
[U.P.P.C.S. (Mains) 2016] in connection with the allegation that in
(a) It banned propaganda by Christian the draft of the Constitution, a major part
Missionaries in India. of the Government of India Act, 1935, has
(b) It emphasized industrialization in India. again been reproduced’?
(c) It made a financial allocation for the [U.P.P.C.S. (Mains) 2015]
education of Indian people. (a) Dr. Rajendra Prasad
(d) It approved the development of a railway (b) Sardar Patel
system in India. (c) Jawaharlal Nehru
41. The first Hindi Newspaper ‘Udant Martand’ (d) Dr. B.R. Ambedkar
(30th May 1826) was published from- 47. The Indian Famine Code, 1883 was prepared
[U.P.P.C.S. (Mains) 2016] by: [U.P.P.C.S. (Mains) 2015]
(a) Kolkata (a) Hunter Commission
(b) Patna (b) Hartog Commission
(c) Allahabad (c) Strachey Commission
(d) Lucknow (d) Indigo Commission
42. Which of the following Acts for the first 48. British East India Company lost the
time created a functioning Legislature monopoly of Tea trade by:
Council in India? [U.P.P.C.S. (Pre) 2015]
[U.P.P.C.S. (Mains) 2016] (a) The Charter Act of 1793
(a) Charter Act of 1793 (b) The Charter Act of 1813
(b) Charter Act of 1813 (c) The Charter Act of 1833
(c) Charter Act of 1853 (d) The Charter Act of 1853
(d) Charter Act of 1833
49. In which of the following years the British
43. By which of the following Acts, the British Government finally agreed to hold the
for the first time introduced the system of Indian Civil Services (I.C.S.) examination
indirect elections in India? simultaneously in India and England?
[U.P.P.C.S. (Mains) 2016] [U.P.P.C.S. (Mains) 2014]
(a) 1909 (a) 1922
(b) 1861 (b) 1923
(c) 1867 (c) 1924
(d) 1892 (d) 1925

PYQ Workbook 218


HISTORY OF MODERN INDIA

50. Match list-I with list-II and select the (a) Mahatma Gandhi
correct answer using the codes given below (b) C.R. Das
the lists: (c) Jawahar Lal Nehru
(d) Motilal Nehru
List-I List-II
(Newspapers) (Launcher/ 56. Match list-I with list-II and select the
Publisher) correct answer using the codes given below
the lists
A. Leader 1. Madan Mohan
Malviya List-I List-II
B. Bombay 2. Firozshah Mehta (News Paper) (Language)
Chronicle A. Bharat Mitra 1. Bengali
C. Independent 3. T.M. Nair B. Rastra Mata 2. Gujarati
D. Justice 4. Motilal Nehru C. Prajamitra 3. Hindi
Code: [U.P.P.C.S. (Mains) 2014]
D. Nayak 4. Marathi
A B C D Code: [ U.P.P.C.S. (Pre) 2012]
(a) 1 2 4 3
A B C D
(b) 4 3 2 1
(c) 3 2 1 4 (a) 1 2 4 3
(d) 4 1 2 3 (b) 2 3 1 4
(c) 3 4 2 1
51. The first editor of the journal ‘Indian (d) 4 1 3 2
Opinion’ was: [U.P.P.C.S. (Pre) 2014]
57. Which of the following was established
(a) M.K. Gandhi
first? [U.P.P.C.S. (Pre) 2012]
(b) Albert West
(a) Hindu College, Calcutta
(c) Mahadev Desai
(b) Delhi College
(d) Mansukhal Nazar
(c) Mayo College
52. Who among the following had launched the (d) Muslim-Anglo Oriental College
paper ‘Qaumi Awaz’?
58. Which one of the following was the first to
[U.P.P.C.S. (Mains) 2013] be declared as Central University?
(a) Maulana Abul Kalam Azad [U.P.P.C.S. (Mains) 2011]
(b) Jawaharlal Nehru
(a) Aligarh Muslim University, Aligarh
(c) Shaukat Ali
(b) Dr. Bhimrao Ambedkar University,
(d) Khaliquzzaman
Lucknow
53. In which of the following Acts for the first- (c) Banaras Hindu University of Varanasi
time provision was made for the post of (d) University of Allahabad
Governor-General of Bengal?
59. Which one of the popular magazines of
[U.P.P.C.S. (Mains) 2013] the revolutionary period criticized the
(a) Regulating Act, 1773 Congress for various reasons?
(b) Pitt’s Indian Act, 1784 [U.P.P.C.S. (Mains) 2011]
(c) Charter Act of 1813
(a) Bangawasi
(d) Act of 1833
(b) Kaal
54. Who was associated with the formation of (c) Kesari
the Deccan Educational Society? (d) All of the above
[U.P.P.C.S. (Pre) 2013] 60. In whose reign was English Education
(a) Justice Ranade introduced in India
(b) Firoz Shah Mehta [U.P.P.C.S. (Mains) 2011]
(c) B.G. Tilak
(a) Lord William Cavendish Bentinck
(d) Dayananda Saraswati
(b) Lord Hardinge
55. The English Newspaper ‘Independent’ was (c) Lord Minto
associated to- [U.P.P.C.S. (Mains) 2012] (d) Lord Dalhousie

219 PYQ Workbook


HISTORY OF MODERN INDIA

61. Match list-I with list-II and select the 67. Who among the following introduced the
correct answer using the codes given below Vernacular Press Act?
the lists: [U.P.P.C.S. (Mains) 2007]
List-I List-II (a) Lord Lytton
(b) Lord Ripon
(Newspaper) (Founder) (c) Lord Curzon
A. Dainik Aaj 1. George Allen (d) Lord Hastings
B. The Leader 2. Jawaharlal Nehru 68. Where was the first Madarsa set up by
C. The National 3. Madan Mohan British in India? [U.P.P.C.S. (Pre) 2006]
Herald Malviya (a) Madras
(b) Bombay
D. The Pioneer 4. Shiv Prasad Gupta
(c) Aligarh
Code: [U.P.P.C.S. (Mains) 2010] (d) Calcutta
A B C D
69. The ‘Swadesh Vahini’ was edited by–
(a) 4 3 2 1
[U.P.P.C.S. (Pre) 2005]
(b) 4 1 3 2
(c) 2 3 4 1 (a) C.V. Raman Pillai
(b) C.N. Mudaliar
(d) 1 4 2 3
(c) K. Ram Krishna Pillai
62. The Permanent Settlement was introduced (d) C.R. Reddy
by- [U.P.P.S.C. (GIC) 2010]
70. Who one of the following had vigorously
(a) Lord Hastings advocated for religious education in the
(b) Lord Cornwallis Indian Universities
(c) Lord Curzon [U.P.P.C.S. (Mains) 2005]
(d) Loa William Bentinck (a) Bal Gangadhar Tilak
63. Which Act of British Government granted (b) Swami Vivekanand
Rs. One lakh for education in India for the (c) Mahatma Gandhi
first time? [U.P.P.C.S. (Mains) 2009] (d) Madan Mohan Malviya
(a) Wood’s Dispatch, 1854 71. Match list-I with list-II and select the
(b) Charter Act, 1813 correct answer using the codes given below
(c) Charter Act, 1853 the lists:
(d) Indian Council Act, 1892
List-I List-II
64. Who among the following was offered (Newspapers) (Person who started it)
membership of the Royal Asiatic Society of
Paris? [U.P.P.C.S. (Mains) 2007] A. Bombay 1. Annie Besant
Chronicle
(a) Dadabhai Naoroji
(b) Michael Madhusudan Dutta B. Common 2. Madan Mohan
(c) Raja Rammohan Roy Weal Malviya
(d) Vivekanand C. Leader 3. Feroz Shah Mehta
65. From where the ‘Indian Mirror’ newspaper D. Search Light4. Sachchidanand
was published in 1880? Sinha
[U.P.P.C.S. (Mains) 2007] Code: [U.P.P.C.S. (Spl) (Mains) 2004]
(a) Bombay A B C D
(b) Calcutta (a) 1 4 3 2
(c) Madras (b) 2 3 4 1
(d) Pondicherry (c) 3 1 2 4
66. In which one of the following languages (d) 4 2 1 3
was the first issue of the Journal Gadar 72. The first newspaper in India was:
published? [U.P.P.C.S. (Mains) 2007] [U.P.P.C.S. (Spl) (Mains) 2004]
(a) Urdu (a) Bengal Gazette
(b) Hindi (b) Hindustan Times
(c) English (c) Pioneer
(d) Marathi (d) Sambad Kaumadi

PYQ Workbook 220


HISTORY OF MODERN INDIA

73. The Marathi fortnightly ‘Bahishkrit Bharat’ (a) Permanent Settlement


was started by: [U.P.P.C.S. (Pre) 2004] (b) Mahalwari Settlement
(a) Bal Gangadhar Tilak (c) Ryotwari Settlement
(b) B.R. Ambedkar (d) None of the above
(c) V.D. Savarkar 80. Persian Weekly ‘Mirat-ul Akhbar’ was
(d) Gopal Krishna Gokhale Published by: [U.P.P.C.S. (Pre) 2000]
74. In the Hunter Commission report, special (a) Lala Lajpat Rai
emphasis was laid on the development of: (b) Raja Ram Mohan Roy
[U.P.P.C.S. (Pre) 2004] (c) Sir Syed Ahmad Khan
(a) Girls’ education (d) Maulana Shibli Nomani
(b) Higher education 81. The Supreme Court was set up for the first
(c) Primary education time in India under the:
(d) Technical education [U.P.P.C.S. (Pre) 1998]
75. Who authored the book “Poverty and the (a) Regulating Act, 1773
Unbritish Rule in India”? (b) Charter Act, 1853
[U.P.P.C.S. (Mains) 2004] (c) Government of India Act, 1935
(d) Indian Constitution Act, 1950
(a) Amartya Kumar Sen
(b) Ramesh Chandra Dutt 82. Who was the first Indian to go to jail in
(c) Gopal Krishna Gokhale performance of his duty as a journalist:
(d) Dadabhai Naoroji [U.P.P.C.S. (Pre) 1997]
76. Which of the following were the two (a) Bal Gangadhar Tilak
newspapers published by Annie Besant? (b) Dadabhai Nauroji
Select the correct answer using the code (c) Motilal Ghosh
given below: (d) Surendra Nath Banerjee
1. Commonweal 83. Which one of the following Acts empowered
2. New India the Governor-General of India to issue
3. New Hindu Ordinances? [U.P.P.C.S. (Pre) 1997]
4. The Aryans (a) Charter Act of 1833
(b) Indian Councils Act of 1861
Code: [U.P.P.C.S. (Mains) 2002]
(c) Indian Councils Act of 1892
(a) 1 and 2 (d) Indian Councils Act of 1909
(b) 1 and 3
(c) 2 and 4 84. Who affiliated himself with the English
(d) 3 and 4 Journal ‘Vande Matram’?
[U.P.P.C.S. (Pre) 1996]
77. The establishment of the first Women’s
(a) Arbind Ghosh
University in Mumbai was the result of the
(b) M. G. Ranade
effort of- [U.P.P.C.S. (Mains) 2002]
(c) S. C. Bose
(a) Dayaram Gindumal (d) Lokmanya Tilak
(b) D.K. Karve
(c) M.G. Ranade 85. The Indian Council Act of 1909 was
(d) Ramabai provided for: [U.P.P.C.S. (Pre) 1996]
(a) Dyarchy
78. Who among the following had primarily (b) Communal representation
implemented Press Censorship? (c) Federation
[U.P.P.C.S. (Pre) 2001] (d) Provincial autonomy
(a) Wellesley
(b) Hastings 86. Why is the Government of India Act, 1935
important? [U.P. Lower Sub. (Pre) 2015]
(c) John Adams
(d) Dalhousie (a) It is the main source of the Indian
Constitution
79. Sir Thomas Munro is associated with the (b) By this India got freedom
land revenue settlement: (c) Division of India is described in it
[U.P.P.C.S. (Pre) 2000] (d) End of the Princely States by this

221 PYQ Workbook


HISTORY OF MODERN INDIA

87. Sadler Commission was related to: (c) 2 3 1 4


[U.P. Lower Sub. (Spl) (Pre) 2010] (d) 3 2 1 4
(a) Judiciary 92. Which of the following election/s of the
(b) Revenue Administration Central Legislative Assembly was/were held
(c) Education under the Government of India Act, 1919?
(d) Police Administration
(A) 1926
88. Match list-I with list-II and select the (B) 1937
correct answer using the codes given below
the lists: (C) 1945
Select the correct answer using code given
List-I List-II
below: [R.A.S./R.T.S. (Pre) 2018]
A. Abul Kalam Azad 1. Bombay (a) A only
Chronicle (b) B and C
B. Feroz Shah Mehta 2. Al-Hilal (c) A and C
C. Annie Besant 3. Young India (d) A, B and C
D. Mahatma Gandhi 4. New India 93. Identify the incorrect statement about
Code: [U.P. Lower Sub. (Pre) 2008] Government of India Act 1919:
A B C D [R.A.S./R.T.S. (Pre) 2016]
(a) 2 1 4 3 (a) This Act is also known as Morley-Minto
(b) 1 2 3 4 Reforms Act.
(c) 2 1 3 4 (b) This Act separated the Central and
(d) 3 2 1 4 Provincial subjects.
(c) The Government of India Act 1919 came
89. Which Urdu newspaper was started by
into force in 1921.
Lajpat Rai from Lahore in 1920?
(d) Montague was the Secretary of State
[U.P. Lower Sub. (Spl) (Pre) 2008] for India and Lord Chelmsford was the
(a) Vande Matram Viceroy of India.
(b) People
(c) Tribune 94. Match list-I with list-II and select the
(d) Vir Arjun correct answer using the codes given below
the lists:
90. The ‘Young India’ was started as a weekly
by: [U.P. Lower Sub. (Pre) 2003] List-I List-II
(a) The Home Rule Party (Editors) (Papers/Magazines)
(b) The Extremist Party
A. S.A. Dange 1. Navyug
(c) Gadar Party
(d) Swaraj Party B. Muzaffar Ahmad 2. Inqilab
91. Match list-I with list-II and select the C. Ghulam Husain 3. Labour Kisan
correct answer using the codes given below Gazette
the lists: D. M. Singarvelu 4. The Socialist
List-I List-II Codes: [R.A.S./RTS (Pre) 2013]
(Newspapers) (Editors) A B C D
(a) 1 2 3 4
A. Hindu 1. Dadabhai Nauroji
(b) 4 1 2 3
B. Sudharak 2. Gopal Krishna (c) 2 3 1 4
Gokhle (d) 3 4 2 1
C. Voice of India 3. G. Subramaniya Iyer 95. During India’s colonial period, the theory
D. Bengalee 4. Surendra Nath of downward filtration was related to:
Banerjee [R.A.S./R.T.S.(Pre) 2013]
Code: [U.P. Lower Sub. (Pre) 2003 & 2002] (a) Railways
A B C D (b) Education
(a) 1 2 3 4 (c) Irrigation
(b) 1 3 2 4 (d) Poverty alleviation

PYQ Workbook 222


HISTORY OF MODERN INDIA

96. The first three universities in India (a) Sir Arthur Cotton
(Calcutta, Madras, and Bombay) were (b) Col. Baird Smith
established in which year? (c) Lt. Blaine
[R.A.S./R.T.S.(Pre) 2010] (d) Col. Robert Smith
(a) 1857 102. The first Chief Justice of Supreme Court
(b) 1881 established by the East India Company was:
(c) 1885 [Uttarakhand P.C.S. (Pre) 2012]
(d) 1905 (a) Elijah Impey
97. Through which newspaper published from (b) Courtney Ilbert
Kanpur, Vijay Singh Pathik made the (c) Phillip Francis
(d) None of the above
Bijoliya movement the subject of discussion
in India: [R.A.S./R.T.S. (Pre) 1996] 103. Which newspaper was started by Lokmanya
(a) Kesari Bal Gangadhar Tilak to serve the cause of
(b) Pratap India’s freedom struggle?
(c) Maratha [Uttarakhand P.C.S. (Pre) 2010]
(d) Majusha (a) Gadar
(b) Kesari
98. Which of the following Act provided for (c) Free Hindustan
communal representation in British India? (d) Swadesh Mitra
[M.P.P.C.S. (Pre) 2019]
104. Who founded the First Sanskrit
(a) Indian Councils Act, 1892
Mahavidyalaya at Varanasi?
(b) Minto-Morley Reforms, 1909
[Uttarakhand P.C.S. (Mains) 2006]
(c) Montague-Chelmsford Reforms, 1919
(d) Government of India Act, 1935 (a) Jonathan Duncan
(b) Warren Hastings
99. Which of the following Acts introduced the (c) Lorel Macaulay
‘Principle of Constitutional Autocracy’? (d) Bankim Chandra
[M.P.P.C.S. (Pre) 2017] 105. The main reason for the British Government
(a) The Indian Councils Act of 1909 to spread modern education in India during
(b) The Government of India Act of 1919 the pre-independence period was–
(c) The Government of India Act of 1935 [Uttarakhand P.C.S. (Pre) 2005]
(d) The Indian Independence Act of 1947 (a) The need for educated Indians in minor
100. Match list-I with list-II and select the administrative posts
correct answer using the codes given below (b) To promote Indian culture
the lists: (c) To modernize the Indian people so
that they could share their political
List-I List-II responsibilities
A. Bipin Chandra Pal 1. New India (d) None of the above
B. Arvind Ghosh 2. Comrade 106. Permanent Settlement and Ryotwari System
C. Brahmobandhab 3. Vande of land revenue introduced respectively in:
Upadhyay Mataram [Jharkhand P.C.S. (Pre.) 2021]
D. Mohammad Ali 4. Sandhya (a) Bengal and Madras
Code: [M.P.P.C.S. (Pre) 2008] (b) Madras and Punjab
(c) Madras and Bengal
A B C D (d) Punjab and Bengal
(a) 1 3 4 2
(b) 2 1 3 4 107. Who have appointed First Indian Education
Commission and in which year
(c) 2 1 4 3
(d) 1 3 2 4 [Jharkhand P.C.S. (Pre.) 2021]
(a) Lord Ripon in 1882
101. Who is regarded as the Pioneer of irrigation (b) Lord William Wedderburn in 1885
works in South India (c) Lord Curzon in 1905
[Uttarakhand PCS (Pre) 2016] (d) Lord Lytton in 1880

223 PYQ Workbook


HISTORY OF MODERN INDIA

108. What was the name of the newspaper 115. In 1911, who among the following
published by Mahatma Gandhi in South introduced a bill in the Imperial Legislative
Africa? [Chhattisgarh P.C.S. (Pre) 2018] Council for introduction of compulsory
(a) The Indian Opinion and free primary education in India?
(b) National Herald [CDS 2021 (I)]
(c) Leader (a) Dadabhai Naoroji
(d) The Pioneer (b) Bal Gangadhar Tilak
109. At the time of the National Movement who (c) Sir Harcourt Butler
started ‘Dash Roja’ periodical? (d) Gopal Krishna Gokhale
[Chhattisgarh P.C.S. (Pre) 2018] 116. Which one of the following statements
(a) Mohammad Ali Jinnah about the Act V of 1843 relating to Slavery
(b) Abdul Gaffar Khan in India is correct? [CDS 2021 (I)]
(c) Lala Lajpat Rai (a) It gave the masters the right to willfully
(d) Bal Gangadhar Tilak keep their slaves tied to their estates.
(b) It denied the masters the use of Courts to
110. The declaration of reforms on August 20,
assert their claims on slaves.
1917, is known as:
(c) The Law Courts and masters worked
[Chhattisgarh P.C.S. (Pre) 2011] jointy in resolving the cases of desertion.
(a) Montagu Declaration (d) The slaves became the owners of The
(b) Morely Declaration land.
(c) Minto Declaration
(d) Chelmsford Declaration 117. Which one of the following acts reserved
seats for women in Legislatures in
111. Which one of the following Viceroys was the accordance with the allocation of seats for
first to officially shift his Council to Simla different communities? [CDS 2020 (I)]
in summer season? [CDS 2023 (I)] (a) The Government of India Act, 1858
(a) John Lawrence (b) The Indian Councils Act, 1909
(b) Lord Dalhousie (c) The Government of India Act, 1919
(c) Lord Mayo (d) The Government of India Act, 1935
(d) William Bentinck
118. The creation of a Federal Court in India
112. Which one of the following pairs of was advocated by which of the following
Newspaper and Editor is not correctly Acts/Commissions? [CDS 2020 (I)]
matched? [CDS 2022 (I)] (a) The Government of India Act, 1919
(b) The Lee Commission, 1923
Newspaper Editor
(c) The Government of India Act, 1935
(a) Navjivan Mahatma Gandhi (d) The Indian Councils Act, 1909
(b) Mahratta Bal Gangadhar Tilak 119. The Hunter Commission (1882) appointed
(c) Bengalee Surendranath Banerjea to survey the State of education in India
(d) Voice of India Gopal Krishna Gokhale [CDS 2020 (I)]
(a) deprecated University education
114. Which one of the following was not included
(b) overruled the Despatch on 1854
in the terms and conditions of Subsidiary
(c) endorsed the Despatch of 1854 with
Alliance System of Lord Wellesley?
greater emphasis on primary education
[CDS 2022 (I)] (d) criticised the grants-in-aid system of
(a) The British would protect their ally. schooling
(b) The ally was free to enter into agreements
with other rulers or engage in warfare. 120. Who among the following started the Indian
(c) In the territory of the ally, a British armed Agriculture Service? [CDS 2019 (II)]
contingent would be stationed. (a) Lord Curzon
(d) The ally would have to provide resources (b) William Bentinck
for the maintenance of the British armed (c) Lord Minto
contingent. (d) Lord Rippon

PYQ Workbook 224


HISTORY OF MODERN INDIA

121. The Deccan Agriculturalists’ Relief Act of Which of the statement(s) given above is/are
1879 was enacted with which one of the correct? [CDS 2016 (I)]
following objectives? [CDS 2019 (I)] (a) Only 1
(a) Restore lands to the dispossessed peasants (b) Only 2
(b) Ensure financial assistance to peasants (c) Both 1 and 2
during social and religious occasions (d) Neither 1 nor 2
(c) Restrict the sale of land for indebtedness
to outsiders 127. Which of the following statement(s) about
(d) Give legal aid to insolvent peasants the Hastings Plan of 1772 is/are correct?
1. Each district was to have a civil and a
122. The Limitation Law, which was passed by criminal court.
the British in 1859, addressed which one of
2. The Judges were helped by native
the following issues? [CDS 2019 (I)]
assessors who were skilled in Hindu and
(a) Loan bonds would not have any legal
Islamic laws.
validity
(b) Loan bonds signed between moneylender 3. The Sadar Diwani Adalat was mainly
and ryots would have validity only for meant to settle mercantile cases exceeding
three years 10000 in value.
(c) Land bonds could not be executed by 4. These courts did not put into place any
moneylenders procedural improvements.
(d) Loan bonds would have validity for ten Select the correct answer using the codes
years given below [CDS 2015 (II)]
123. The Summary Settlement of 1856 was based (a) 1 and 2
on which one of the following assumptions? (b) 3 and 4
[CDS 2019 (I)] (c) 2 and 4
(a) The Talukdars were the rightful owners of (d) Only 2
the land 128. Which of the following sets of newspapers
(b) The Talukdars were interlopers with no reflected the concerns of educated Indian
permanent stakes in the land Muslims during the Khilafat Movement?
(c) The Talukdars could evict the peasants [CDS 2015 (I)]
from the lands
(d) The Talukdars would take a portion of (a) Comrade and Hamdard
the revenue which flowed to the State (b) Comrade and Hindustan Times
(c) Zamindar and Muslim Voice
124. Who was the founder of Mahakali Pathshala (d) Comrade, Hamdard, Zamindar and Al
in Calcutta? [CDS 2019 (I)] Hilal
(a) Her Holiness Mataji Maharani Tapaswini
(b) Sister Nivedita 129. Consider the following historical events in
(c) Madame Blavatsky India carefully:
(d) Sarojini Naidu 1. The Indian press failed to protest against
the Vernacular Press Act of 1878.
125. Who was the editor of the journal Indian 2. The Vernacular Press Act of 1878 was
Social Reformer that was started in 1890?
introduced by Viceroy Lytton against the
[CDS 2017 (II)] counsel of his own Law Member.
(a) KT Telang
(b) Veeresalingam Which of the above is/are correct?
(c) NG Chandavarkar [CAPF 2022]
(d) KN Natarajan (a) 1 only
(b) 2 only
126. Consider the following statements: (c) Both 1 and 2
1. The province of Assam was created in the (d) Neither 1 nor 2
year 1911.
2. Eleven districts comprising Assam 130. Which of the following pairs is/are correctly
were separated from the Lieutenant matched?
Governorship of Bengal and established 1. Permanent Settlement: Lord Cornwallis
as an independent administration under 2. Ryotwari Settlement: Thomas Munro
a Chief Commissioner in the year 1874. 3. Mahalwari Settlement: Holt Mackenzie

225 PYQ Workbook


HISTORY OF MODERN INDIA

Select the correct answer using the codes (a) It was proposed by Governor General
given below. [CAPF 2021] William Bentinck on the advice of
(a) 1 only Macaulay.
(b) 1 and 2 only (b) It made English the language of
(c) 2 and 3 only instruction in Indian education system
(d) 1, 2 and 3 (c) With the formal institutionalization of
131. Who among the following was the first non- English as the language of instruction,
official member to be elected as Speaker of the stage was set for a new direction of
the Central Legislative Assembly on 24-8- Indian education.
1925? [CAPF 2020] (d) To the existing Oriental Institutions
(a) Vithalbhai J Patel fresh awards of stipends to students and
(b) Muhammad Yakub the publication of classical texts were to
(c) G.V. Mavalankar continue.
(d) Sardar Hukum Singh 136. Which one of the following statements
132. The British Committee of the Indian about the colonial economy is not correct?
National Congress was founded in [CAPF 2017]
[CAPF 2020] (a) The British presence inhibited indigenous
(a) 1889 capitalism.
(b) 1892 (b) Laissez-faire actively promoted
(c) 1898 indigenous capitalism.
(d) 1901 (c) The ‘white collective monopoly’ came
133. Why did the early nationalists oppose the earliest and remained most pronounced
Council Act of 1892 ? [CAPF 2019] in Eastern India.
(a) The number of members of the Imperial (d) The Bombay hinterland was difficult
Legislative Council was increased. to penetrate before the construction of
(b) The number of members of Provincial railways
Councils was increased. 137. Which one of the following has created the
(c) The Councils were given the right to office of the Secretary of State for India?
discuss the annual budgets.
[CAPF 2017]
(d) The Act did not give the Indians control
over the public funds. (a) The Councils Act 1861
(b) The Government of India Act 1858
134. Examine these two statements carefully and (c) The Morley Minto Reforms
select the correct answer using the codes (d) Montague Chelmsford reforms
given below.
Statement I The Permanent Settlement was 138. Which one of the following statements
rarely extended to any region beyond Bengal. about land tenure system is not correct?
Statement II After 1810, the agricultural [CAPF 2017]
prices declined affecting adversely the income (a) During the British rule in India three
of the Bengal Zamindars. categories of land tenure system, viz.
Codes: [CAPF 2018] Zamindari, Mahalwari and Ryotwari were
(a) Both the statements are individually introduced.
true and Statements II is the correct (b) Under Zamindari system, land was held
explanation of Statement I by one person or at the most by a few
(b) Both the statements are individually joint owners who were responsible for the
true but Statement II is not the correct payment of land revenue.
explanation of Statement I (c) Under the Mahalwari system, the
(c) Statement I is true but Statement II is false agricultural lands belonged to the
(d) Statement I is false but Statement II is government.
true (d) Under Ryotwari system, the individual
135. Which one of the following statements holders had the permanent rights over
about the English Education Act of 1835 is land and were directly responsible for
not correct? [CAPF 2017] payment of land revenue.

PYQ Workbook 226


HISTORY OF MODERN INDIA

139. Which of the following is/are 142. Which of the following statements about the
characteristic(s) of the debate between ideology of Utilitarianism in the context of
‘Anglicists’ and ‘Orientalists’ in the 1830s? British India is/are correct? [CAPF 2016]
1. The Anglicists wanted to cut the money 1. Utilitarianism believed that the rule of
being spent on printing Sanskrit and law was necessary for societal betterment.
Arabic texts. 2. James Mill expounded the principles
2. The Anglicists wanted to cut the money of Utilitarianism in his famous book,
spent on printing Sanskrit texts but ‘Utilitarians and the Raj’.
wished to continue to print in Persian. 3. Utilitarians differed from the liberals in
3. The Orientalists wanted a stipend for the significant ways.
students of Arabic and Sanskrit. 4. Utilitarianism had distinct authoritarian
4. The Orientalists started a new Sanskrit tendencies.
College in Delhi. Select the correct answer using the code given
Select the correct answer using the code given below.
below. [CAPF 2016] (a) Only 1
(b) 1 and 2 only
(a) 1 only
(c) 3 and 4 only
(b) 1, 3 and 4
(d) 1, 3 and 4
(c) 1 and 3 only
(d) 2, 3 and 4 143. The following item consists of two
Statements I and II. Examine these two
140. Which of the following is NOT a statements carefully and select the correct
characteristic of the Permanent Settlement? answer using the code given below.
[CAPF 2016]
Statement I
(a) The Permanent Settlement vested land
India’s share of world manufacturing output
ownership rights in the Zamindar
steadily declined during the 19th and the
(b) The Permanent Settlement continued to
20th centuries.
pay attention to the customary occupancy
rights of peasants Statement II
(c) The burden of the high revenue There was no absolute decline in production
assessment was shifted to the peasants in India during the colonial rule.
under the Permanent Settlement Codes: [CAPF 2015]
(d) The condition of the actual cultivators (a) Both the statements are individually true
of the land declined under Permanent and Statement II is the correct explanation
Settlement of Statement I
(b) Both the statements are individually
141. Consider the following statements about true but Statement II is not the correct
the Morley-Minto reforms of 1909. explanation of Statement I
1. They were named after the British (c) Statement I is true but Statement II is false
Parliamentarians, Minto and Morley. (d) Statement I is false but Statement II is
2. They provided for limited self- true
government by increasing the number
144. Which of the following statements with
of elected Indians in the Legislative
regard to construction of railways in
Councils.
colonial India by the British companies is/
3. They contained provisions that ensured are NOT true?
that British officials retained their 1. The companies were guaranteed a return
majority in the Imperial Legislative of 5% on their investment by Government
Council. of India.
Which of the statements given above is/are 2. The railways were to be managed mainly
correct? [CAPF 2016] by the Government.
(a) 1 only 3. There was no system of preferential
(b) 2 and 3 only freight charges.
(c) 1 and 3 only 4. The companies were to get the land free
(d) 1, 2 and 3 from the Government.

227 PYQ Workbook


HISTORY OF MODERN INDIA

Select the correct answer using the code given (a) Zamindari System
below. [CAPF 2015] (b) Ryotwari System
(a) 1 and 3 only (c) Mahalwari Sysem
(b) 2 and 3 only (d) Dahsala System
(c) 4 only (e) None of the above/More than one of the
(d) 2, 3 and 4 above
145. Which one among the following was not a 150. From the options given below, find the
provision of the Regulating Act of 1773? correct combination of the names of editors
[CAPF 2014] of the nationalist newspapers The Hindu,
(a) It introduced parliamentary supervision Kesari, Bengalee, Hindustani, Sudharak:
over the East India Company [64th B.P.S.C. (Pre) 2018]
(b) The Government of Bengal was vested in (a) Surendra nath Benerjee, G, Subramania
a Governor General and the Council of Iyer, Bal Gangadhar Tilak, Ganga Prasad
four members Verma, Gopal krishna Gokhale
(c) It changed the power of vote in the Court (b) Bal Gangadhar Tilak, G. Subramania Iyer,
of Proprietors by removing all restrictions Surendra nath Banerjee, Ganga Prasad
on qualifications Verma, Gopal Krishna Gokhale
(d) The Governor-General was vested with a (c) G. Subramania Iyer, Bal Gangadhar tilak,
casting vote Ganga Prasad Verma, Gopal Krishna
146. The Deccan Riots Commission was Gokhale, Surendra nath banerjee
concerned with [CAPF 2014] (d) G. Subramania Iyer, Bal Gangadhar Tilak,
(a) indebtedness of the peasant Surendra Nath Banerjee, Ganga Prasad
(b) lack of law and order in the Deccan Verma, Gopal Krishna Gokhale
(c) problems with the Ryotwari system (e) None of the above/More than one of the
(d) communal riots in the Deccan above
147. In 1902, Lord Curzon appointed the 151. The post of the Indian High Commission
University Commission including two was created by which Act?
Indian members. Who were they? [63rd B.P.S.C (Pre.) 2017]
[66th B.P.S.C. Re-Exam (Pre) 2020] (a) The Indian Councils Act, 1909
(a) Bal Gangadhar Tilak and Surendranath (b) The Government of India Act, 1919
Banerjee (c) The Government of India Act, 1935
(b) Gopal Krishna Gokhale and Rashbehari (d) The Indian Independence Act, 1947
Bose (e) None of the above/More than one of the
(c) Syed Hussain Bilgrami and Surendranath above
banerjee
(d) Syed Hussain Bilgrami & Justice Gurudas 152. Who is known as the father of Dyarchy?
Banerjee [B.P.S.C. (Pre) 2016]
(e) None of the above/More than one of the (a) Lord Clive
above (b) Hector Munaro
148. About which Act, Jawaharlal Nehru had (c) Lord Macaulay
said, “We were provided with a car with all (d) Sir Leoni Cartis
brakes and no engine”? (e) None of the above/More than one of the
[66th B.P.S.C. (Pre) 2020] above
(a) Act of 1858 153. First Indian elected to the British House
(b) Act of 1909 of Commons was Dadabhai Naoroji who
(c) Act of 1919 contested on the ticket of:
(d) Act of 1935 [60th to 62nd B.P.S.C. (Pre) 2016]
(e) None of the above/More than one of the (a) Liberal Party
above (b) Labour Party
149. The system under which the peasant (c) Conservative Party
himself owns the land and is responsible for (d) Communist Party
payment of land revenue to the Government (e) None of the above/More than one of the
is known as: [65th B.P.S.C. (Pre) 2019] above

PYQ Workbook 228


HISTORY OF MODERN INDIA

154. Which journal was not associated with (a) Girish Chandra Ghosh
revolutionary activities? (b) Harish Chandra Mukherjee
[60th to 62nd B.P.S.C. (Pre) 2016] (c) S.N. Banerjee
(a) Sandhya (d) Shishir Kumar Ghose
(b) Yugantar
(c) Gadar 157. Which of the following papers was
(d) Young India essentially the mouthpiece of the policies of
(e) None of the above/More than one of the liberals? [47th B.P.S.C. (Pre) 2005]
above (a) New India
155. The Montagu-Chelmsford Report formed (b) Leader
the basis of: (c) Young India
[Jharkhand P.C.S. (Pre) 2011, 53rd to 55th (d) Free Press Journal
B.P.S.C. (Pre) 2011]
(a) the Indian Councils Act, 1909 158. Who in America started to issue newspaper
(b) the Government of India Act, 1919 ‘Free India’? [39th B.P.S.C. (Pre) 1994]
(c) the Government of India Act, 1935 (a) Ram Nath Puri
(d) the Indian Independence Act, 1947 (b) G.D. Kumar
156. Who founded the ‘Amrit Bazar Patrika’? (c) Lala Hardayal
[47th B.P.S.C. (Pre) 2005] (d) Tarak Nath Das

229 PYQ Workbook


HISTORY OF MODERN INDIA

SOLUTIONS
and to the Governors by the British Government under the
5.1. UPSC CSE Previous Years’ Questions Government of India Act of 1935. The only difference is that
1. Solution: (d) now they are instructions to the legislature and the executive.
Exp) Option d is the correct answer. 5. Solution: (c)
The Charter Act of 1833 addressed the internal challenges Exp) Option c is the correct answer.
in India and the political situation in England. It renamed
The reason for such regulation was Lord Cornwallis was
the position of the Governor-General of Bengal as the
alarmed at the extent of power concentrated in the District
Governor-General of India and vested in him all civil and
Collector and felt that such absolute power was undesirable
military power. Lord William Bentinck became the first
governor-general of India. in one person. The revenue administration was separated
from the judiciary functions by the Charter Act of 1793.
2. Solution: (c)
Important Tips
Exp) Option c is the correct answer.
The features of Charter Act, 1793:
Under the Government of India Act 1919, Subjects were
divided into two lists: ‘reserved’ which included subjects • It gave the Governor-General more powers and control
such as law and order, finance, land revenue, irrigation, etc., over the governments of the subordinate Presidencies
and ‘transferred’ subjects such as education, health, local of Bombay and Madras.
government, industry, agriculture, excise, etc. • It extended the trade monopoly of the Company in
The reserved subjects were to be administered by the India for another period of twenty years.
governor through his executive council of bureaucrats, • It stated that the Commander-in-Chief was not to be
and the transferred subjects were to be administered by a member of the Governor-General’s council, unless
ministers nominated from among the elected members of he was so appointed.
the legislative council. • The Home Government members were to be paid out
Option 1 is correct: Law and order was a reserved subject of Indian revenues, which continued up to 1919.
under the Government of India Act, 1919 • The Company, after paying the necessary expenses,
Option 2 is incorrect: local government was a transferred interest, dividends, salaries, etc., from the Indian
subject under the Government of India Act, 1919. revenues, was to pay 5 lakh pounds annually to the
British government.
Option 3 is correct: Land Revenue was a reserved subject
under the Government of India Act, 1919 • The royal approval was mandated for the appointment
of the Governor General, the Governors, and the
Option 4 is correct: Police was a reserved subject under the
Commander-in-Chief.
Government of India Act, 1919.
• Senior officials of the Company were debarred from
3. Solution: (b) leaving India without permission. Doing so was
Exp) Option b is the correct answer. treated as resignation.
Statement 1 is incorrect. The Montague Chelmsford reforms
6. Solution: (d)
of 1919 did not recommend granting voting rights to all
women above the age of 21. Although it recommended the Exp) Option d is the correct answer.
voting rights to women in limited numbers to be extended Raja Ram Mohan Roy collaborated with David Hare and
on the basis of property, tax or education. Alexander Duff to establish Hindu College in Calcutta in
Statement 2 is correct. The Government of India Act 1817. This initiative marked a significant step in introducing
1935, under part XII section 308 and under schedule I part English language education and promoting liberal education
I, specifically mentions about reservation for women in among Hindus in Bengal. The college’s foundation laid the
legislature. groundwork for modern education in the region and played
a pivotal role in shaping educational reforms.
4. Solution: (b)
Exp) Option b is the correct answer. 7. Solution: (a)

The ‘Instrumentation of Instructions” contained in the Exp) Option a is the correct answer.
Government of India Act, 1935 have been incorporated Al-Hilal (The Crescent) was an Urdu newspaper started
in the Constitution of India in the year 1950 as Directive by Maulana Abdul Kalam Azad in 1912. The newspaper’s
Principles of State Policy. In the words of Dr. B. R. first edition was published in Kolkata. The main aim of the
Ambedkar, the Directive Principles are like the instrument newspaper was to create a national consciousness among the
of instructions, which were issued to the Governor-General masses, and to encourage them to join the freedom struggle in

PYQ Workbook 230


HISTORY OF MODERN INDIA

large numbers. Al-Hilal advocated for communal harmony Statement 2 is correct: Government of India Act, 1935
and asserted that Independence for India could only be provided for the establishment of a Federal Court, which
achieved if all communities set aside their petty differences was set up in 1937.
and launch a combined struggle against colonialism.
Statement 3 is correct: Government of India Act, 1935
8. Solution: (b) provided for the establishment of an All-India Federation
Exp) Option b is the correct answer. consisting of provinces and princely states as units. The Act
divided the powers between the Centre and units in terms of
Som Prakash newspaper was started by Ishwar Chandra
three lists–Federal List, Provincial List, and Concurrent List.
Vidyasagar in 1859 under the editorship of Dwarakanath
Residuary powers were given to the Viceroy. However, the
Vidyabhushan. It was a weekly paper coming from the
federation never came into being as the princely states did
Sanskrit Press. It is famous for its contribution to the Indigo
Movement. not join it.

9. Solution: (b) 12. Solution: (a)

Exp) Option b is the correct answer. Exp) Option a is the correct answer.

Statement 1 is incorrect: The Charter Act of 1813 (not In 1882, Lord Rippon repealed the Vernacular Press Act of
the Charter Act of 1853) abolished East India Company 1878. The Vernacular Press Act (1878) was enacted to curtail
monopoly of Indian trade except trade in tea and trade with the freedom of the Indian press (non-English newspaper) and
China. Whereas Charter Act of 1833 ended the activities prevent the expression of criticism toward British policies.
of the East India Company as a commercial body and made
13. Solution: (b)
company a purely administrative body.
Exp) Option b is the correct answer.
Statement 2 is correct: Under the Government of India
Act of 1858, the British Parliament abolished the East India Statement 1 is correct: Government of India Act, 1935
Company altogether and undertook the responsibility of abolished dyarchy in the Governor’s provinces and
ruling India directly. This act transferred the powers to the introduced ‘provincial autonomy’ in its place. The Act
British Crown. introduced responsible Governments in provinces, that is,
the Governor was required to act with the advice of ministers
10. Solution: (b)
responsible to the provincial legislature. This came into
Exp) Option b is the correct answer. effect in 1937 and was discontinued in 1939.
Statement 1 is incorrect: The credit for establishing the
Statement 2 is correct: Government of India Act, 1935
modern police system in India goes to Lord Cornwallis. Lord
provides power to the Governors to veto legislative action
Cornwallis established a permanent police force to maintain
and to legislate on their own. The governor could refuse
peace and order by resurrecting and modernizing the old
assent to a bill; promulgate ordinances; and enact governor’s
Indian system of thanas (circles) in each district, each led
Acts.
by a daroga (an Indian) and a superintendent of police (SP).
Statement 2 is correct: Section 13 of the Regulating Act Statement 3 is incorrect: Government of India Act, 1935
1773 made a detailed provision for the establishment of a did not abolish the principle of communal representation.
Supreme Court of judicature at Fort William, in Calcutta. Contrary, it further extended the principle of communal
This Supreme Court consisted of a chief justice and three representation by providing separate electorates for
other judges of higher rank but inferior to the chief justice. depressed classes (Scheduled Castes), women and labour
The British King appointed the judges. (workers).
Statement 3 is correct: The Indian Penal Code (IPC) was 14. Solution: (b)
drafted by Thomas Babington Macaulay in 1860.
Exp) Option b is the correct answer.
Note: Indian Penal Code came to effect in 1862.
During the colonial period in India, the purpose of the
11. Solution: (d) Whitley Commission, officially known as the Royal
Exp) Option d is the correct answer. Commission on Labour, was to investigate and report
on the existing conditions of labor in India and make
Statement 1 is correct: Government of India Act, 1935
abolished dyarchy in the provinces and introduced recommendations for improvements. The commission,
‘provincial autonomy’ in its place. The provinces were chaired by John Henry Whitley, submitted its report in 1931.
allowed to act as autonomous units of administration in their The report highlighted the role of poverty in India’s social
defined spheres. Moreover, the Act introduced responsible and industrial problems and criticized British employers for
Governments in provinces, that is, the Governor was perpetuating these issues. It also emphasized the need for
required to act with the advice of ministers responsible to systematic collection of labor statistics and recommended
the provincial legislature. suitable legislation for this purpose.

231 PYQ Workbook


HISTORY OF MODERN INDIA

Important Tips
Important Commissions and committees during British rule in India:

Committees/Commission Year Governor Subjects of the committees


General/Viceroy

Scott-Moncrieff Commission 1901 Lord Curzon Irrigation


Fraser Commission 1902 Lord Curzon Police Reforms
Hunter Committee Report 1919 Lord Chelmsford Punjab Disturbances
Muddiman Committee 1924 Lord Reading To examine the working of Diarchy of the Montague-
Chelmsford reforms
Butler Commission 1927 Lord Irwin Indian State’s relation with the British Crown
Whitley Commission 1929 Lord Irwin Labour
Simon Commission 1928 Lord Irwin To investigate the progress of the governance scheme
and suggest new steps for reforms.
Sapru Commission 1935 Lord Linlithgow Unemployment
Chatfield Commission 1939 Lord Linlithgow Army
FIoud Commission 1940 Lord Linlithgow Tenancy in Bengal

15. Solution: (d) 17. Solution: (a)


Exp) Option d is the correct answer. Exp) Option a is the correct answer.
Under Indian Councils Act of 1909 (not the Charter Act • According to Charter Act of 1813, Company’s trade
of 1833), an Indian was to be appointed as a Law Member monopoly in India was ended. It means the Indian trade
of the Governor-General’s Council. Satyendra Sinha was was thrown open to all British merchants. However, it
appointed as a law member of viceroy’s executive council in continued the monopoly of the company over trade in tea
1909. At central government level, an Indian member was and trade with China.
taken for the first time in the Executive Council. • According to Regulating Act of 1773, the Company’s
directors were asked to present to the British Government
Important Tips
all correspondence and documents pertaining to the
The features of Charter Act, 1833: administration of the company. It strengthened the
• It made the Governor-General of Bengal as the control of the British Government over the Company by
Governor General of India. Lord William Bentick requiring the Court of Directors to report on its revenue,
was the first Governor-General of India. civil, and military affairs in India.
• Governor General of India was vested with all civil • According to Government of India Act of 1858, the
and military powers. All law-making powers to be power to govern was transferred from the East India
conferred on Governor-General-in-council. Company to the British Crown. The Company lost all
• The trading activities of the East India Company its administrative powers following the Government of
were to be abolished. It ended the activities of the India Act of 1858, and its Indian possessions and armed
East India Company as a commercial body. It became a forces were taken over by the Crown.
purely administrative body. • Pitt’s India Act of 1784 set up a Board of Control in
• It deprived the Governor of Bombay and Madras of Britain to fully regulate the East India to manage the
their legislative powers. political affairs. It empowered the Board of Control to
supervise and direct all operations of the civil and military
16. Solution: (a)
government or revenues of the British possessions in
Exp) Option a is the correct answer. India.
Indian Councils Act, 1861 of British India strengthened the
18. Solution: (d)
Viceroy’s authority over his executive council by substituting
“portfolio” or departmental system for corporate Exp) Option d is the correct answer.
functioning. Under this system, a member of the Viceroy’s Government of India Act of 1935 provided for the
council was made in-charge of one or more departments of establishment of an All-India Federation consisting of
the Government and was authorised to issue final orders on British provinces and princely states as units. The real
behalf of the council on matters of his departments. intention of the British to include the Princely States in

PYQ Workbook 232


HISTORY OF MODERN INDIA

the Federal Union proposed by the India Act of 1935 was Sir Charles Wilkins (1749 – 1836) was the first Englishman
to use the princes to counterbalance the anti-imperialist to translate the Bhagavad Gita into English. He was an
doctrines of the nationalist leaders. As the British Provinces Orientalist and a founding member of The Asiatic Society.
were in favour of bringing some democratic change while His translation of the Gita, titled ‘Bhagvat-geeta’ or
Princely states were not in favour of any democratic changes Dialogues of Krishna and Arjun, was published in 1785.
that can affect their authority. Wilkins’ work was significant not only for its translation but
also for introducing the term “Hinduism” to refer to various
19. Solution: (b) mythologies and cultures existing in India. His translation
Exp) Option b is the correct answer. had a profound influence on European perceptions of Hindu
The zamindars were required to issue pattas to the farmers philosophy and Romantic literature.
under the Permanent Settlement of 1793. However, many 21. Solution: (a)
zamindars did not issue pattas to the farmers. The reason
Exp) Option a is the correct answer.
for this was that there was no official check upon the
Statement 1 is incorrect: The Government of India Act of
zamindars. The British government did not have the
1935 abolished dyarchy in the provinces. But it provided
resources to monitor the zamindars effectively. As a result,
for the adoption of dyarchy at the Centre. Consequently,
the zamindars were able to exploit the farmers without
the federal subjects were divided into reserved subjects and
fear of reprisal. The pattas were important for the farmers
transferred subjects. However, this provision of the Act did
because they gave them a legal right to the land they were
not come into operation at all.
cultivating. Without pattas, the farmers were vulnerable to
Statement 2 is correct: It introduced bicameralism in six
eviction by the zamindars. The zamindars often used the
out of eleven provinces. Thus, the legislatures of Bengal,
threat of eviction to extract more rent from the farmers.
Bombay, Madras, Bihar, Assam and the United Provinces
Important Tips were made bicameral consisting of a legislative council
(upper house) and a legislative assembly (lower house).
Challenges of the Zamindari System:
Statement 3 is correct: It introduced ‘provincial autonomy’
• Impact on Cultivators:
in place of dyarchy in the provinces. The provinces were
• High Rents: Cultivators faced oppression and allowed to act as autonomous units of administration in their
exploitation due to exorbitant rents charged by defined spheres. Hence, the Act introduced responsible
zamindars. Governments in provinces, that is, the Governor was
• Land Insecurity: The uncertainty of land ownership required to act with the advice of ministers responsible to
under the system added to their woes. the provincial legislature.
• Indebtedness: Many cultivators had to borrow Statement 4 is correct: Government of India Act of 1935
money to pay rents, leading to cycles of debt. provided for the establishment of an All-India Federation
• Eviction Threat: Failure to pay rents resulted in consisting of provinces and princely states as units.
eviction from the land, worsening their situation. 22. Solution: (a)
• Struggles for Zamindars: Exp) Option a is the correct answer.
• High Revenue: The fixed revenue demand was The term “imperial preference” referred to the granting
often challenging for zamindars to meet. of special privileges to British imports in India during
• Risk of Loss: Zamindars failing to pay revenue the colonial period. This practice involved minimal or
risked losing their zamindari rights. no charges on British goods brought into India, while
• Lack of Incentive: Zamindars were more Indian exports to Britain faced significant duties. This
interested in collecting rents than investing in land unequal practice aimed to benefit Britain’s economy, stifling
improvement. Indian industries by flooding markets with British goods
• Limitations for the Company: and inhibiting local value addition. ‘Imperial preference’
showcased colonial economic exploitation, suppressing
• Growing Cultivation: Increased cultivation and
India’s development and autonomy through skewed trade
rising prices benefited zamindars but not the
policies, deepening economic inequality.
colonial company.
• Revenue Constraint: The company couldn’t adjust 23. Solution: (d)
the permanent revenue demand despite changing Exp) Option d is the correct answer.
economic conditions. The British colonial government did not encourage the
• Stagnant Income: The company’s income remained development of industries in India. They preferred to keep
stagnant, even as agrarian activities expanded. India as a market for British goods. The British also imposed
high tariffs on Indian goods, making it difficult for Indian
20. Solution: (b) industries to compete with British industries. In addition,
Exp) Option b is the correct answer. the British government encouraged the rich Indians to

233 PYQ Workbook


HISTORY OF MODERN INDIA

invest in land rather than in industries. This was because The Hunter Commission was established after the Jallianwala
land was seen as a safe investment and it gave the rich Bagh massacre in 1919. Led by Lord William Hunter, it found
Indians a social status. As a result of these policies, there General Dyer’s actions unjustified, but justified martial law
was no independent development of industries in India in Punjab. The majority report reprimanded Dyer for his
during British rule. The Indian economy remained largely duty concept, while the minority report questioned martial
agricultural and backward. law’s necessity and disturbance severity. Compensation was
24. Solution: (c) provided to the victims’ dependents.

Exp) Option c is the correct answer. The Muddiman Committee, led by Sir Alexander Muddiman
in 1924, addressed Indian leaders’ demands regarding the
The most short-lived of all of Britain’s constitutional
diarchy issue in the 1921 Indian Council Act. It produced
experiments in India was Indian Council Act of 1909. The
majority and minority reports, with the majority suggesting
Indian Councils Act of 1909 is also called Morley-Minto
minor changes, while the minority called for a permanent
Reforms. It lasted only for 10 years until Government of
Constitution. Lord Birkenhead favored the majority report.
India Act of 1919 was introduced.
26. Solution: (d)
Important Tips
Exp) Option d is the correct answer.
The features of Indian Council Act, 1909:
• It considerably increased the size of the legislative Sir Syed Ahmad Khan was a Muslim scholar and reformer
councils, both Central and provincial. The number who did not believe in the drain theory of Dadabhai
of members in the Central legislative council was Naoroji. He argued that the British rule had a positive
raised from 16 to 60. The number of members in the impact on the Indian economy and that the drain of wealth
provincial legislative councils was not uniform. was not as severe as Naoroji had claimed.
• The Act of 1909 created non-official majority of 27. Solution: (b)
members in all the Provincial Legislative Councils,
Exp) Option b is the correct answer.
but maintained official majority of members in the
Central Legislative Council. Drain Theory as propounded by Dadabhai Naoroji refers
• It enlarged the deliberative functions of the legislative to the concept that a part of India’s national wealth or
councils at both levels. For example, members were total annual product was being drained or exported to
allowed to ask supplementary questions, move Britain without commensurate material returns to India.
resolutions on the budget and so on. He calculated that India was losing about £150 million every
• For the first time, it provided for the association of year to Britain.
Indians with the executive councils of the Viceroy and The drain theory was based on the following factors:
Governors. Satyendra Prasad Sinha became the first • The excess of exports over imports, which meant that
Indian to join the Viceroy’s executive council. He was
India was sending more goods to Britain than it was
appointed as the Law Member.
receiving from Britain.
• It also provided for the separate representation of
• The high salaries paid to British officials in India, which
presidency corporations, chambers of commerce,
universities and zamindars. were then sent back to Britain.
• The profits made by British companies in India, which
25. Solution: (d) were also sent back to Britain.
Exp) Option d is the correct answer. • The cost of the British military in India, which was paid
The Harcourt Butler Committee, chaired by Sir Harcourt for by the Indian government.
Butler in 1923, clarified British paramountcy over princely
states in India. It reaffirmed the doctrine of paramountcy, Important Tips
provided guidelines for its application, and emphasized fair Impacts of Wealth Drain from India:
financial dealings. Princely states wouldn’t be transferred • Reduced Local Circulation of Money: Money sent
without their consent to Indian government control. abroad hindered local economic circulation, impacting
The Hartog Committee, formed in 1929 by the Simon trade, industries, and agriculture. This deprived local
Commission in British India and led by Philip Hartog, aimed businesses and individuals of the benefits of circulating
to address issues in education. It prioritized mass education, capital within India.
identified problems in primary schools like wastage and • Stifled Economic Growth: The drain impeded the
stagnation, and recommended government oversight. It also growth of vital sectors like trades, industries, and
suggested curriculum reforms for secondary and higher agriculture that relied on circulating capital. Capital
education, advocated for women’s education, and highlighted outflow to England led to a shortage of funds, hindering
the need for improved facilities. substantial industrial development.

PYQ Workbook 234


HISTORY OF MODERN INDIA

• Impoverishment of Peasantry: Draining land revenue, Features:


a key capital source, led to rural impoverishment. The • Promotion of European Education: Macaulay stressed
peasantry was disproportionately affected, denying the importance of emphasizing European science and
them potential economic growth opportunities. literature for Indians, suggesting that resources be
• Capital Shortage and Industrial Stagnation: Loss of allocated primarily to English education.
productive capital hindered India’s industrial growth, • Stipends and Education: New students entering
preventing robust manufacturing development. Capital institutions under the committee would not receive
scarcity impeded India’s own industrial revolution. stipends. However, existing professors and students
• Inhibited Capital Formation: Drainage disrupted would continue to be stipend recipients.
capital formation necessary for long-term economic • Oriental Works Funding: Government funds would
growth, curtailing India’s potential for domestic no longer support the publication of oriental works.
development.
• Future Allocation: Macaulay’s proposal directed
• Skewed Socioeconomic Structure: The drain future government expenditures exclusively to English
worsened wealth inequality, hindering middle-class literature and science education for Indians.
formation and marginalizing socioeconomic progress.
• Undermined Self-Reliance: Wealth drainage weakened
India’s internal resource generation, heightening 5.2. Other Examination Previous Years’
dependence on foreign capital. Questions
28. Solution: (b) 29. Solution: (c)
Exp) Option c is the correct answer.
Exp) Option b is the correct answer.
When debate on the inclusion of separate electorate in
The foundation of the modern educational system in
Indian Constitution was conducted, Sardar Vallabh Bhai
India was established by Macaulay’s Minutes of 1835.
Patel said in constituent assembly, “Can you show me one
In this historic document, Thomas Babington Macaulay
free country where there are separate electorates? ..... The
emphasized the importance of providing English education British element is gone, but they have left mischief behind”.
to Indians. He believed that this approach would create Patel said that there was unanimity of opinion “on the point
a class of Indians who shared British values and interests. that there should be no more separate electorates and we
This marked a significant shift in educational policy, should have joint electorates hereafter”. Patel iterated his
promoting Western sciences and literature in English as uncompromising opposition to the principle of separate
the primary focus of education in India. This approach electorates. He said, the introduction of the system of
aimed to produce individuals who were “Indian in blood and communal electorates was a poison which had entered into
color, but English in taste, opinions, morals, and intellect.” the body politic of the country.

30. Solution: (b)


Important Tips
Exp) Option b is the correct answer.
The Macaulay Minute of 1835:
Assertion (A) is true: The British Government did introduce
Objectives:
different land revenue systems in various parts of India
• Exclusivity of Western Education: The famous Lord based on local conditions, historical practices, and economic
Macaulay’s Minute settled the dispute in favor of considerations. For instance, the Permanent Settlement,
Anglicists—the limited government resources were to
Ryotwari System, and Mahalwari System were implemented
be devoted solely to the teaching of Western sciences
in different regions.
and literature in English, excluding other forms of
education. Reason (R) is true: The introduction of varied land revenue
systems did lead to the emergence of different classes among
• University Closure: He advocated for the closure
the peasantry, depending on how the systems operated and
of universities offering courses solely in eastern
the rights granted to landholders. The systems affected
philosophy.
landownership, tenancy, and economic relationships
• Downward Filtration Theory: Macaulay suggested a differently across regions.
limited number of Indians should receive government
Both (A) and (R) are true but (R) is not the correct
education, who would then disseminate knowledge
explanation of (A): While it’s true that the introduction of
to the broader population, known as the “downward
different land revenue systems resulted in the creation of
filtration” approach.
different classes within the Indian peasantry, the connection
• Indian-British Identity: His vision aimed to cultivate between the two statements is not as straightforward as cause
Indians who could align with British interests—being and effect. The creation of different classes was not the
“Indian by blood and colour, but English by likes, purpose of introducing different revenue systems; rather,
beliefs, morality, and intellect.” it was a consequence of a complex set of factors including

235 PYQ Workbook


HISTORY OF MODERN INDIA

administrative policies, economic considerations, and This explains his association with the establishment of The
social dynamics. Asiatic Society of Bengal and his choice to nominate Sir
William Jones as the president, showcasing his commitment
31. Solution: (d) to scholarly pursuits.
Exp) Option d is the correct answer.
33. Solution: (d)
The correct matching of List-I with List-II is as follows:
Exp) Option d is the correct answer.
A. Jajmani -1. North India
The correct chronological order of the events is as follows:
B. Bara Balute -3. Maharashtra
• Wood’s Despatch (1854): Charles Wood, President of
C. Mirasi - 4. Tamil Nadu
the Board of Control of the East India Company, sent the
D. Adabe - 2. Karnataka influential Wood’s Despatch in 1854 to Lord Dalhousie,
promoting English education in India with vernacular
Important Tips
languages in primary schools and English as the medium
• The jajmani system, prevalent mainly in North India,
in colleges, shaping Indian education significantly.
was an economic arrangement where lower castes
provided specific services to upper castes in return • Hunter Commission (1882): The Hunter Commission
for goods, maintaining a patron-client relationship. of 1882 made several key recommendations for the
Services were hereditary, exchanged within families, education system in British India, including: prioritizing
fostering self-sufficient village economies. literate candidates for government jobs, expanding
primary schools in underdeveloped areas, entrusting
• The Bara Balutedar system in Maharashtra comprised
local self-government bodies with primary education
twelve hereditary trades with roles like priests,
barbers, blacksmiths, and more. They received village management, encouraging private secondary schools
produce for services under a barter system. Balutedars with government funds, revising the secondary school
held privileges at ceremonies and received payments in curriculum, discouraging missionary schools, and
cash or kind. This system, akin to the jajmani system, promoting the education of girls and women.
was agriculturally linked and had hereditary rights. It • Sadler Commission (1917): The Sadler Commission of
was abolished by law in 1958. 1917, also known as the Calcutta University Commission,
• In pre-colonial South India, land was intertwined with was established to address issues in Calcutta University
social relationships, forming a ‘share distribution and Indian education. It recommended improvements
system.’ Village produce was divided into shares, in teacher training, university autonomy, separation
sustaining those in production roles, a concept known of secondary and university education, and the
as “kudavolai” in Tamil. The mirasi system, analogous establishment of boards for secondary and intermediate
to inherited rights, prevailed, fostering flexibility by education. It also emphasized the need for women’s
linking shares to roles, accommodating mobility. education and the expansion of educational institutions
Temples and Brahmins also received dues. This in towns.
indigenous, enduring mirasi system played a pivotal • Sargeant Plan (1944): The Sergeant Plan of Education,
role in Tamil Nadu’s history.
outlined in the 1944 Sargent Report, proposed a
comprehensive educational system with pre-primary
32. Solution: (a)
education for 3-6-year-olds, universal primary education
Exp) Option a is the correct answer. for ages 6-11 and 11-14, middle schools with diverse
Assertion (A) is true: The Asiatic Society of Bengal was courses, and high schools divided into academic
established on January 15, 1784, by Sir William Jones, a and technical streams. It emphasized mother tongue
British lawyer and orientalist. Warren Hastings was the instruction, teacher training, physical education, and
governor-general of Bengal at the time, and he was a great equal opportunities for students.
supporter of the society. He even declined the offer of
presidency of the society in favor of Jones, who was a more 34. Solution: (d)
qualified scholar. Exp) Option d is the correct answer.
Reason (R) is true: Warren Hastings was a prominent • Yugantar was founded by Barindra Kumar Ghosh (not
scholar and orientalist. He supported and encouraged the by Mahatma Gandhi). Barindra Kumar Ghosh aas an
study of languages like Sanskrit, Persian, and Arabic, which Indian revolutionary, journalist and a younger brother of
were crucial for understanding the cultural and historical Sri Aurobindo Ghosh. The first editor of the Jugantar was
heritage of India and other Asian regions. Bhupendranath Dutta. He played a significant role in
(R) is the correct explanation of (A): Reason (R) supports the revolutionary activities of the Indian independence
Assertion (A) by explaining that Warren Hastings, who was movement, particularly in Bengal.
a knowledgeable scholar and orientalist, actively promoted • Indian Opinion was started in 1903, Young India and
the study of languages like Sanskrit, Persian, and Arabic. Navjivan in 1919 by Gandhi.

PYQ Workbook 236


HISTORY OF MODERN INDIA

35. Solution: (a) which was implemented in 1905, making it subordinate to


Exp) Option a is the correct answer. the Government.
• Kaal: Shivram Mahadev Paranjape was the founding Following the famine of 1899-1900, Lord Curzon appointed
father and editor of the weekly newspaper Kaal. It was a famine commission led by Anthony MacDonnell. In its
founded in 1897 in Pune. report in 1901, it summarized principles of relief suggesting
• Bangalee: It was edited by several notable figures, variations wherever necessary.
including Girish Chandra Ghose and Surendranath Following the devastating famine of 1896-1900, a focus
Banerjee. It was one of the highest circulated weekly on internal administrative reforms was heightened. An
newspaper in the late 19 th and early 20 th century. Irrigation Commission under Sir Colin Scott-Moncrieff,
• Sudharak: Gopal Krishna Gokhale established the appointed by Lord Curzon, outlined plans for constructing
Servants of India Society in 1905 and edited “Sudharak”, irrigation works, with Punjab’s 2714-mile main canal being
a quarterly Journal of the Poona Sarvajanik Sabha. He a crucial element.
also started Hitavada newspaper in 1911.
The Indian Police Commission of 1902-03, led by Sir
• Shom Prakash: It was started by Ishwar Chandra Andrew Frazer and Lord Curzon, aimed to reform India’s
Vidyasagar in 1859 under the editorship of Dwarakanath police system. It proposed centralizing police intelligence,
Vidyabhushan. It was a weekly paper coming from the
separating executive and judicial roles, and establishing a
Sanskrit Press. It is remembered for its contribution to
police training school, contributing to lasting improvements
the Indigo Movement.
in policing.
36. Solution: (c)
39. Solution: (c)
Exp) Option c is the correct answer.
Exp) Option c is the correct answer.
The correct chronological order of the events is:
Statement 1 is correct: Deindustrialization in India
• Ban on Sati (1829)
commenced with the Charter Act of 1813, which terminated
• Woods Despatch (1854) the East India Company’s trade monopoly. The Act enabled
• Rowlatt Act (1919) British merchants to engage in unrestricted trade, causing
• Vernacular Press Act (1878) an influx of inexpensive British goods. The ensuing one-
sided free trade policy disadvantaged Indian industries,
37. Solution: (a)
precipitating their decline.
Exp) Option a is the correct answer.
Statement 2 is correct: The abolition of the monopoly trade
Assertion (A) is true: India did generally have a favorable rights of the East India Company aggravated the process
balance of trade during British rule. It implies that the value
of deindustrialization in India. This is because it led to
of its exports exceeded the value of its imports, resulting
increased competition for Indian industries from British
in a surplus. This was due to the significant export of raw
industries. Indian industries were not able to compete with
materials, agricultural products, and handicrafts from India
British industries, which were more efficient and had access
to Britain.
to cheaper raw materials. This led to the further decline of
Reason (R) is true: The drain of wealth from India took
Indian industries.
the form of unrequired exports, where valuable resources
and goods were exported to Britain without receiving 40. Solution: (c)
proportionate benefits in return. This economic exploitation
Exp) Option c is the correct answer.
led to a significant outflow of wealth from India, contributing
to its economic subjugation. One of the reasons for considering the Charter Act of 1813
important for India is it made a financial allocation for
(R) is the correct explanation of (A): The drain of wealth,
the education of Indian people. It provided for the spread
characterized by unrequired exports, played a crucial role in
maintaining the favorable balance of trade for Britain. India’s of western education among the inhabitants of the British
surplus exports contributed to Britain’s economic prosperity territories in India. A sum of one lakh rupees was to be
while hindering India’s own development. Thus, the drain of set aside for the revival, promotion, and encouragement of
wealth was the underlying reason behind India’s favorable literature, learning, and science among the natives of India,
trade balance during British rule. every year.

38. Solution: (c) Important Tips


Exp) Option c is the correct answer. The features of Charter Act, 1813:
In 1901, Thomas Robertson became the special • It abolished the trade monopoly of the company in India
commissioner to assess the Railway’s functioning. The i.e., the Indian trade was thrown open to all British
Robertson commission proposed the establishment of a merchants. However, it continued the monopoly of the
Railway Board with a chairman, 2 members, and 1 secretary, company over trade in tea and trade with China.

237 PYQ Workbook


HISTORY OF MODERN INDIA

• It allowed the Christian missionaries to come to India Exp) Option d is the correct answer.
for the purpose of enlightening the people. Under Indian Councils Act of 1892, the British for the first
• It authorised the Local Governments in India to time introduced the system of indirect elections in India. The
impose taxes on persons. They could also punish the act brought the provision of bringing additional members
people for not paying taxes. into the Legislative Council through the indirect election.
For the first time, the element of an election was introduced
• The regulations made by the Councils of Madras,
into the Indian political system.
Bombay, and Calcutta were now required to be laid
before the British Parliament. The constitutional Important Tips
position of the British territories in India was thus The features of Indian Councils Act, 1892:
explicitly defined for the first time.
• It increased the number of additional (non-official)
41. Solution: (a) members in the Central and provincial legislative
councils but maintained the official majority in them.
Exp) Option a is the correct answer.
• It increased the functions of legislative councils and
The first Hindi language newspaper, ‘Udant Martand’ (The
gave them the power of discussing the budget and
Rising Sun), was launched in India on May 30, 1826. Pt.
addressing questions to the executive.
Jugal Kishore Shukla produced the weekly newspaper from
Kolkata every Tuesday. • The act made a limited and indirect provision for the
use of election in filling up some of the non-official
42. Solution: (c) seats both in the Central and provincial legislative
Exp) Option c is the correct answer. councils. The word “election” was, however, not used
Charter Act of 1853 for the first time created a functioning in the Act.
Legislature Council in India. It separated, for the first time, 44. Solution: (a)
the legislative and executive functions of the Governor-
Exp) Option a is the correct answer.
General’s council. It established a separate Governor-
General’s legislative council which came to be known as the Chronological order:
Indian (Central) Legislative Council. This legislative wing • Dramatic performances Act (1876): The British
of the council functioned as a mini-Parliament, adopting Government introduced the Dramatic Performances Act
the same procedures as the British Parliament. Thus, for the in 1876 to regulate and control seditious Indian theatre
first-time legislation was treated as a special function of the used as a form of protest against colonial rule.
government. • Vernacular Press Act (1878): The Vernacular Press Act
Important Tips of 1878 in British India aimed to control the indigenous
The features of Charter Act, 1853: press, restricting criticism of British policies. Proposed
by Viceroy Lytton and unanimously passed, it enabled
• It introduced an open competition system of selection government censorship, monitoring of Vernacular
and recruitment of civil servants. The covenanted civil newspapers, and warnings for seditious content,
service was, thus, thrown open to the Indians also. excluding English-language publications.
• It introduced, for the first time, local representation • Bengal Tenancy Act (1885): The Bengal Tenancy Act
in the Indian (Central) Legislative Council. Of the of 1885 was introduced by the Bengal government in
six new legislative members of the Governor General’s response to the Pabna Agrarian Uprisings, addressing the
council, four members were appointed by the local rights of zamindars and tenants amid widespread peasant
(provincial) governments of Madras, Bombay, Bengal protests.
and Agra. • North-Western Provinces and Oudh Act (1890): The
• The Charter Act of 1853 stated that the Company North-Western Provinces and Oudh Act, 1890, aimed
would be responsible for paying the salaries of the to enhance the administration of these territories under
Board of Control members, its secretary, and other the Lieutenant Governor of the North-Western Provinces
officers. and the Chief Commissioner of Oudh by amending
• The Court of Directors, by the Act, also could alter the relevant enactments.
boundaries of the existing states and incorporate the 45. Solution: (a)
newly acquired state.
Exp) Option a is the correct answer.
• Power was given to the Court of Directors to Paternalism in Governance in the Indian context is
constitute a new Presidency. associated with Thomas Munro. He was a Scottish soldier
• The Act empowered the British Crown to appoint a and British colonial administrator who served as the governor
Law Commission in England to examine the drafts. of Madras Presidency. Munro introduced the ryotwari system
of land revenue and initiated reforms in the judicial and
43. Solution: (d) police systems during his tenure. Munro’s approach aimed to

PYQ Workbook 238


HISTORY OF MODERN INDIA

address social and administrative issues while respecting • Recommended the appointment of a famine
local customs. His governance approach aimed to protect commissioner in provinces expecting extensive
and guide the people, reflecting a paternalistic attitude. relief operations.
46. Solution: (d) • Emphasized non-official assistance, improved
Exp) Option d is the correct answer. transport, agricultural banks, irrigation, and
agricultural methods.
Dr. B.R. Ambedkar said, ‘I don’t have to beg pardon in
connection with the allegation that in the draft of the • Many recommendations accepted, but some
Constitution, a major part of the Government of India Act, measures implemented half-heartedly under Lord
1935, has again been reproduced’. Actually, many critics Curzon’s administration.
called the Constitution as a “Carbon Copy of the 1935
48. Solution: (c)
Act” or an “Amended Version of the 1935 Act”. Dr. B.R.
Ambedkar answered the above criticism in the Constituent Exp) Option c is the correct answer.
Assembly and said that I make no apologies. There is nothing The commercial monopoly of British East India Company in
to be ashamed of in borrowing. It involves no plagiarism. tea trade was ended by the Charter Act of 1833. Whereas
Nobody holds any patent rights in the fundamental ideas of Charter act of 1813 ended the monopoly of the East India
a constitution. Company in India but the company›s monopoly in trade
47. Solution: (c) with China and trade in tea remained intact.

Exp) Option c is the correct answer. 49. Solution: (a)


The Indian Famine Code of 1883 was prepared by the Exp) Option a is the correct answer.
Strachey Commission. This commission, led by Richard In 1922, the British Government finally agreed to hold the
Strachey, was appointed in 1880 by Lord Lytton, the then Indian Civil Services (I.C.S.) examination simultaneously
Viceroy of India, to formulate general principles and in India and England. Initially, the examinations for the
suggest measures for dealing with famines in India. The Indian Civil Service were conducted only in London. From
code recommended various measures, including adjusting 1922 onwards, the Indian Civil Service Examination began
wages for laborers, providing gratuitous relief to the poor,
to be held in India also.
and suspensions/remissions of land revenue and rents
during famine situations. It also laid the foundation for the 50. Solution: (a)
provincial famine codes. Exp) Option a is the correct answer.
Important Tips • The Leader: It was one of the most influential English-
Other Famine Commission: language newspapers in India during British Raj. It was
founded by Madan Mohan Malviya. It was published in
• Campbell Commission, 1866:
Allahabad.
• Famine in Odisha, Bengal, Bihar, and Madras in
• Bombay Chronicle: It was an English-language
1865-66.
newspaper founded by Sir Pherozeshah Mehta. He was
• 20 lakhs lives lost, with Odisha alone witnessing 10 a prominent lawyer who later became president of the
lakhs casualties. Indians National Congress in 1890, and member of the
• Committee led by Sir George Campbell. Legislative Council of Bombay in 1893.
• Blamed government machinery for the tragedy. • The Independent: It was an Allahabad based newspaper
• Viceroy Sir John Lawrence in office. begun by Motilal Nehru in 1919. The paper closed down
• Lyall Commission, 1896: under British repression two years later. The Independent
was started on 5 February 1919 with the primary aim of
• Famine of 1896-97 affecting nearly every province,
countering the moderate political line adopted by the
impacting 34 million people.
then leading Allahabad daily The Leader.
• Sir James Lyall chaired the commission.
• Justice: The newspaper Justice began publication from
• Recommended flexibility to existing principles. 1917. Dr. T.M. Nair was the Chief Editor until his death
• Advocated the development of irrigation in 1919. In his newspaper, Dr. Nair attacked his opponents
facilities. in the Indian National Movement and supporters of the
• Lord Elgin II was the Viceroy. Home Rule Movement.
• McDonnell Commission, 1900: 51. Solution: (d)
• Led by MacDonnell. Exp) Option d is the correct answer.
• Stressed moral strategy, early distribution of • Natal Indian Congress Secretary Mansukhlal Hiralal
advances for agriculture, and sinking temporary Nazar served as the first editor of the newspaper.
wells. Manilal Gandhi was editor of Indian Opinion for the

239 PYQ Workbook


HISTORY OF MODERN INDIA

next 36 years, making him the longest serving editor. countering the moderate political line adopted by the then
He did this for no salary as his father M.K. Gandhi. leading Allahabad daily The Leader.
• The initial tone of the newspaper was moderate. It
56. Solution: (c)
reiterated Indians as ‘loyal subjects of the King Emperor’
and reiterated faith in the British system. Exp) Option c is the correct answer.
• Bharat Mitra was a Hindi newspaper first published in
52. Solution: (b) 1877 in Kolkata, as a fortnightly. Harmukund Shastri
Exp) Option b is the correct answer. was its first editor and later Balmukund Gupta was later
Qaumi Awaz is an Urdu language newspaper started by appointed as the editor.
Jawaharlal Nehru in 1937. It is puplished by Associated • Rashtra Mata Newspaper was started by Ganpat
Journals Limited. It had temporarily suspended its Sakharam Patil in April 1888. It was published in the
publication in 2008. Marathi language.
53. Solution: (a) • Chandrakant Keni started the newspaper Prajamitra. It
Exp) Option a is the correct answer. was published in the Gujarati language.

Regulating Act, 1773 made provision for the first time and • Nayak newspaper, published from Kolkata, was among
designated the Governor of Bengal as the ‘Governor General the foremost vernacular nationalist newspapers in
of Bengal’. It also created an Executive Council of four Bengali. This newspaper extensively cover the the second
members to assist him. The first such Governor General was phase of the Civil Disobedience Movement in Bengal.
Lord Warren Hastings.
57. Solution: (b)
Important Tips Exp) Option b is the correct answer.
The features of Regulating Act, 1773: Among the options provided, Delhi College was
• It made the Governors of Bombay and Madras established first. It was founded in 1696 and is the oldest
presidencies subordinate to the Governor-General existing educational institution in India. It has played a
of Bengal, unlike earlier, when the three presidencies significant role in shaping modern education, Urdu, and
were independent of one another. Islamic learning in the country. Delhi College is now known
• It provided for the establishment of a Supreme Court as Zakir Husain Delhi College and is affiliated with the
at Calcutta (1774) comprising one chief justice and University of Delhi.
three other judges.
Important Tips
• It prohibited the servants of the Company from
• Hindu College, Calcutta, was established first among
engaging in any private trade or accepting presents or
bribes from the ‘natives’. the given options. It was founded in 1899 by Krishan
Dassji Gurwale and Pandit Deen Dayal Sharma as a
• It strengthened the control of the British Government
response to the nationalist struggle against British
over the Company by requiring the Court of Directors
rule. Originally located in Kinari Bazar, Chandni
(governing body of the Company) to report on its
Chowk, it was affiliated with Punjab University until
revenue, civil, and military affairs in India.
the University of Delhi was established in 1922. The
54. Solution: (c) college played a significant role in India’s freedom
struggle, hosting debates and discussions with
Exp) Option c is the correct answer.
prominent leaders like Mahatma Gandhi, Jawaharlal
The formation of the Deccan Educational Society was Nehru, and Subhash Chandra Bose.
associated with B.G. Tilak. In 1884, Lokmanya Bal
• Mayo College, founded in 1875 by the 6th Earl of
Gangadhar Tilak, along with Vishnushastri Chiplunkar
Mayo, Richard Bourke, is a prestigious boys-only
and Gopal Ganesh Agarkar, established the Deccan
boarding school in Ajmer, Rajasthan, India. It was
Educational Society. The society played a significant role in
conceived to be an “Indian Eton,” providing education
promoting education in Pune and beyond, and it established
to young rulers and nobles. Colonel Sir Oliver St John
institutions like Fergusson College and various schools to
became the first principal. It’s considered one of the
provide education in Western subjects.
world’s renowned boarding schools.
55. Solution: (d) • Muhammadan Anglo-Oriental College was founded
Exp) Option d is the correct answer. in 1875 by Sir Syed Ahmad Khan, initially as a
The Independent was an Allahabad based newspaper primary school, with the intention of turning it
begun by Motilal Nehru in 1919. The paper closed down to a college level institution. It was inspired by the
under British repression two years later. The Independent Cambridge education system. It started operations on
was started on 5 February 1919 with the primary aim of Queen Victoria’s 56th birthday, 24 May 1875.

PYQ Workbook 240


HISTORY OF MODERN INDIA

58. Solution: (c) Important Tips


Exp) Option c is the correct answer. Feature of The English Education Act of 1835:
Banaras Hindu University (BHU) is a central university • Legislative Implementation: Enacted by the Council
established in 1916 in Varanasi, UP. It was Centralised of India, the Act enforced a decision by Lord William
in 1916 through the Banaras Hindu University Act. Bentinck to reallocate funds as mandated by the British
Banaras Hindu University is India’s first central university. Parliament for education and literature in India.
It emerged from Central Hindu College founded by Annie
• Shift in Support: Formerly, limited backing was given
Besant. Spread across 1,370 acres, BHU is Asia’s largest
to traditional Muslim and Hindu education as well as
residential university with 30,000+ students. It’s an Institute
literature in Sanskrit and Persian. The Act redirected
of Eminence and boasts 6 institutes, 14 faculties, and 140+
support towards institutions teaching a Western
departments.
curriculum with English as the medium of instruction.
Important Tips • Impact on Language: Alongside other measures
• Aligarh Muslim University (AMU), originally promoting English in administration and law courts,
established as Muhammadan Anglo-Oriental College the Act contributed to English becoming one of
in 1875 by Sir Syed Ahmad Khan, became a central India’s languages, transcending its status as merely
university in 1920 through the Aligarh Muslim the language of foreign rulers.
University Act. • Macaulay’s Memorandum: Thomas Babington
• Babasaheb Bhimrao Ambedkar University (BBAU) is Macaulay’s influential Memorandum on (Indian)
a Central University in Lucknow, Uttar Pradesh. The Education, a catalyst for the Act, emphasized Western
university was established on 10 January 1996. The superiority and proposed English-medium higher
university has a satellite campus at Amethi too, which education to produce individuals “Indian in blood
was established in 2016. and colour, but English in taste, opinions, morals, and
• The University of Allahabad in Prayagraj, UP, is a intellect.”
central collegiate university of national importance, • Hybrid Approach: While Macaulay’s memorandum
founded on 23 Sept 1887. Originating from Muir advocated reduced support for traditional education,
Central College, it’s among India’s oldest universities, the Act exhibited a more balanced stance and led to
known as the “Oxford of the East.” Its Central University subsequent funding provisions for both traditional
status was re-established through the University of and Western education.
Allahabad Act 2005 by the Parliament of India. • Vernacular Language: Vernacular language education
received limited funding under the Act, despite
59. Solution: (d)
historical underrepresentation.
Exp) Option d is the correct answer.
• All the three magazines - Bangawasi, Kaal and Kesari 61. Solution: (a)
criticized Congress for its liberal. Exp) Option a is the correct answer.
• The Bangabasi College Magazine was first published in • Dainik Aaj: In the present-day Varanasi district, a daily
1903. It was published by the Bangabasi College Students newspaper called Aaj (Today) was established in 1920 by
Union. philanthropist Shiv Prasad Gupta. Inspired by Japan he
decided to start a nationalist newspaper in Hindi along
• Shivram Mahadev Paranjape was the founding father of
the lines of the London Times. The newspaper provided
the weekly newspaper Kaal in 1898. Kaal had ushered in
support to the Indian freedom struggle, and popularized
a new era of satirical and incisive writing. It was known
Hindi readership.
for its radical and revolutionary views and actively
advocated for revolutionary terrorism. • The Leader: It was one of the most influential English-
language newspapers in India during British Raj. It was
• Bal Gangadhar Tilak in his newpaper Kesari also
founded by Madan Mohan Malviya. It was published in
condemned the policies of congress under moderates.
Allahabad.
60. Solution: (a) • National Herald: It was established in Lucknow on 9
Exp) Option a is the correct answer. September 1938 by Jawaharlal Nehru as a tool to win
English education was introduced in India in the reign of independence. It was banned by British government in
Lord William Cavendish Bentinck, who was the Governor- 1942 during the Quit India movement. In 1938, K. Rama
General of India from 1828 to 1835. Bentinck was a strong Rao was appointed the paper’s first editor.
advocate of Western education, and he believed that it was • The Pioneer: It was started by Sir George Allen in
essential for the modernization of India. In 1835, he passed Allahabad in 1865, who was the ‘sole manager, editor-in-
the English Education Act, which made English the medium chief and distributer of the paper’. It was the first British-
of instruction in all government schools. This act marked the owned and British-edited newspaper in northern India.
beginning of the spread of English education in India. It is the only newspaper in India, which had two Nobel

241 PYQ Workbook


HISTORY OF MODERN INDIA

laureates on its staff. The two Nobel laureates were- 66. Solution: (a)
Rudyard Kipling and Sir Winston Churchill. Exp) Option a is the correct answer.
62. Solution: (b) On November 1, 1913, the Gadar party put out a weekly paper
called Gadar, meaning revolt. The Newspaper “Ghadar” was
Exp) Option b is the correct answer.
published first in Urdu later in Gurumukhi script. It was
The Permanent Settlement was introduced by Lord published from San Francisco in USA.
Cornwallis in 1793. It was a system of land revenue settlement
Important Tips
in Bengal and Bihar. Under the Permanent Settlement,
the zamindars were given the right to collect land revenue Gadar Party:
from the peasants in return for a fixed annual payment to • The Ghadar Party was a revolutionary association
the British government. The Permanent Settlement was founded by Punjabi Indians in the United States and
Canada with the aim to gaining India’s independence
intended to create a stable and reliable source of revenue for
from British rule.
the British government and to encourage the zamindars to
invest in agriculture. • Its aim was to get rid of the colonial superpower by
means of an armed struggle and to set up a national
Important Tips democratic government on the sub-continent.
The Permanent Settlement Agreement: • Gadar Party was founded in 1913 with its headquarter
at San Francisco.
• The Permanent Settlement Agreement established the
Zamindars as permanent landowners. • In 1912–1913, the Pacific Coast Hindustan
Association was formed by Indian immigrants under
• Under this arrangement, Zamindars were obligated the leadership of Lala Dayal, with Sohan Singh Bhakna
to remit 89% of the annual revenue to the state while as its president, which later came to be called the
retaining 11% as their portion. Ghadar Party.
• This settlement also granted Zamindars autonomy • Key members included Lala Har Dayal, Sohan Singh
over the internal affairs within their districts. Bhakna, Kartar Singh and Rashbehari Bose.

63. Solution: (b) 67. Solution: (a)


Exp) Option b is the correct answer. Exp) Option a is the correct answer.
The East India Company Act 1813, also known as the Lord Lytton introduced the Vernacular Press Act to
Charter Act 1813, granted Rs. One lakh for education in prevent newspapers from printing ‘seditious material’. The
India for the first time. This act renewed the charter of the European press used to publish content in favour of the
British government whereas the vernacular press was critical
British East India Company and brought about changes in its
of the government. The Vernacular Press Act (VPA) of
governance, including allowing funds for the promotion of
1878 was designed to ‘better control’ the vernacular press
literary and scientific knowledge, and permitting Christian
and effectively punish and repress “seditious writing” in
missionaries to propagate their religion and teach in
publications in oriental languages (Indian languages). The
Company territories. act came to be nicknamed “the gagging Act”.
64. Solution: (b)
Important Tips
Exp) Option b is the correct answer.
The provisions of the Vernacular Press Act, 1878:
Michael Madhusudan Dutta was a renowned Bengali • The district magistrate was empowered to call upon
poet and playwright. He is known for his efforts to blend the printer and publisher of any vernacular newspaper
Western literary styles with Bengali literature. Due to his to enter into a bond with the government.
contributions to literature and his unique approach, he • The printer and publisher could also be required to
was recognized internationally. The Royal Asiatic Society deposit security, which could be forefeited if the
of Paris, appreciating his literary endeavors, offered him regulations were contravened, and press equipment
membership. This recognition highlights his significance in could be seized if the offence re-occurred.
the literary world and his ability to bridge cultural influences • The magistrate’s action was final and no appeal could
in his work. be made in a court of law.
65. Solution: (b) • A vernacular newspaper could get exemption from
the operation of the act by submitting proofs to a
Exp) Option b is the correct answer.
government censor.
In 1861, Indian mirror was published in Calcutta, West The worst features of this act were:
Bengal. The newspaper was published in English. It was first
• Discrimination between English and vernacular press.
Indian daily paper in English. It was edited and published
by Manmohan Ghosh and Devendranath Tagore of Calcutta. • No right of appeal.

PYQ Workbook 242


HISTORY OF MODERN INDIA

68. Solution: (d) Gazette” even though it was also known as “Calcutta General
Exp) Option d is the correct answer. Advertiser.”
The first Madrasa set up by the British in India was the Important Tips
Calcutta Madrasa. It was established in Calcutta (now Other Newspapers:
Kolkata) in October 1780 by Governor General Warren
• Hindustan Times: Hindustan Times was founded in
Hastings. The purpose of this educational institution was
1924 in Delhi by Sunder Singh Lyallpuri, founder-
to provide training in Persian, Arabic, and Muslim Law to
father of the Akali movement and the Shiromani Akali
produce individuals qualified for lower posts in government
Dal. The Mahatma Gandhi inaugurated the Hindustan
offices and courts. Times press in 1924.
69. Solution: (c) • Pioneer: The Pioneer was founded in Allahabad in
Exp) Option c is the correct answer. 1865, by George Allen, an Englishman. It was brought
out three times a week from 1865 to 1869 and daily
The Vakkom Moulavi founded the weekly Swadesh Vahini
thereafter. It is the second oldest English language
or Swadeshabhimani (The Patriot) Travancore. K. Ram
newspaper in India still in circulation after The Times
krishna Pillai took over as the editor in January 1906. He of India. In 2010, The Pioneer launched its Hindi
edited Swadeshabhimani, the newspaper which became version in Lucknow.
a potent weapon against the rule of the British and the
• Sambad Kaumudi: It was a Bengali weekly newspaper
erstwhile princely state of Travancore (Kerala) and a tool for
published from Kolkata in the first half of the 19th
social transformation.
century by Ram Mohan Roy. It was first published
70. Solution: (d) in 1821. It attempted to avoid the effects of the
Exp) Option d is the correct answer. missionaries’ propaganda and actively campaigned for
the abolition of the Sati Pratha.
Madan Mohan Malviya was a great educationist and a
nationalist leader. He was the founder of Banaras Hindu 73. Solution: (b)
University (BHU), which was established in 1916. He was Exp) Option b is the correct answer.
also a strong advocate of religious education in Indian
On 3 April 1927, Ambedkar launched the Marathi
universities. He believed that religious education was
fortnightly ‘Bahishkrit Bharat’ to address the cause of the
essential for the moral and spiritual development of the
depressed classes. In 1920, Ambedkar entered into the world
students.
of newspapers. He started his first newspaper, Mooknayak,
71. Solution: (c) on January 31, 1920. It ran for three years before being
Exp) Option c is the correct answer. closed. Later, he went on to found three more newspapers
– Bahishkrut Bharat (1927-1929), Janata (1930-56), and
• Bombay Chronicle: It was an English-language
Prabuddha Bharat (1956).
newspaper founded by Sir Pherozeshah Mehta. He was
a prominent lawyer who later became president of the 74. Solution: (c)
Indians National Congress in 1890, and member of the
Exp) Option c is the correct answer.
Legislative Council of Bombay in 1893.
In the Hunter Commission report of 1882, special
• Commonweal: Annie Besant launched the Home
emphasis was placed on the development of primary
Rule League in 1916 to campaign for dominion status.
education in India. The commission, chaired by W.W.
She published New India and Commonweal. The
Hunter, reviewed the educational progress in the country
Commonweal was a weekly dealing with issues of national
and recommended measures to improve primary education.
reform and New India was a daily newspaper which for
The commission’s focus was on assessing the state of primary
fifteen years was a powerful instrument promoting Home
education and making recommendations for its extension
Rule and revolutionizing Indian journalism.
and enhancement. The commission argued that primary
• The Leader: It was one of the most influential English- education was essential for the overall development of the
language newspapers in India during British Raj. It was country and that it would help to improve the lives of the
founded by Madan Mohan Malviya. It was published in people.
Allahabad.
• Searchlight: Sachchidananda Sinha started the Important Tips
Searchlight in 1918, in Patna. It was an English biweekly, The Hunter Commission of 1882 made significant
which became a tri-weekly in 1920 and a daily in 1930. recommendations that led to key changes in British India’s
education system:
72. Solution: (a)
• Mother Tongue as Medium: Primary education should
Exp) Option a is the correct answer. use the mother tongue as the medium of instruction.
The Bengal Gazette was the title of the first newspaper in • Enhanced Teacher Training: Increased establishment
India, published on January 29, 1780, by James Augustus of training schools for the proper training of primary
Hicky during the British Raj. People just refer to it as “Hicky’s teachers.

243 PYQ Workbook


HISTORY OF MODERN INDIA

• Model Government High School: Creation of a model with issues of national reform and New India was a daily
government high school was suggested. newspaper which was a powerful instrument promoting
Home Rule and revolutionizing Indian journalism.
• Practical and Localized Curriculum: The school
curriculum should prioritize practical aspects and Important Tips
cater to local needs. Arayan: Dr. Velukkutty Arayan completed his medicine
• Local Self-Government Involvement: Primary course in both Ayurveda and Allopathy (English medicine)
education management was entrusted to district and from Madras. In 1925, under the leadership of Velukkutty
municipal boards under the Local Self Government Arayan, a Savarna Jatha (procession) was organized
Act. Separate funds were allocated for rural and urban supporting Vaikom Satyagraha. He started a newspaper
areas to ensure proper allocation. called ‘Arayan’ and strongly criticized the anti-Indian
• Promotion of Indian Participation: Missionary policies of the government. His editorials in the newspaper
schools were discouraged, encouraging Indian were a serious criticism of the government.
engagement in private schools. Aryan Path: It was a monthly periodical published from
• Addressing Female Education: Recognizing Bombay and edited first by B. P. Wadia and later by
inadequate female education outside major cities, Sophia Wadia. The journal started in January 1930 and
the commission urged the expansion of facilities for was published at least until 1978.
women’s education and put forth recommendations for
77. Solution: (b)
its improvement.
Exp) Option b is the correct answer.
• Private Secondary Schools: Private entities were
encouraged to establish secondary schools with Dhondo Keshav Karve, also known as Maharshi Karve,
government funding. Government-operated model was the driving force behind the establishment of the
schools in each district were suggested to guide these first Women’s University in Mumbai. He was a prominent
private institutions. social reformer in India, advocating for women’s education
and welfare. In 1916, he founded the Shreemati Nathibai
75. Solution: (d) Damodar Thackersey Women’s University, making it
Exp) Option d is the correct answer. the first women’s university in India. Karve’s efforts
Dadabhai Naoroji, authored the book “Poverty and Un- were instrumental in advancing women’s education and
British Rule in India” in 1901. The book presented his empowerment in the country. He was awarded the Bharat
theory of wealth drainage, revealing how resources were Ratna, India’s highest civilian award, in 1958 for his
systematically transferred from India to Britain. Naoroji significant contributions to society.
emphasized economic exploitation by detailing mechanisms 78. Solution: (a)
like excessive exports, over-employment of British officials,
Exp) Option a is the correct answer.
and interest payments. His work fueled nationalist sentiment
and the demand for self-governance. The first to impose censorship on the press in India was Lord
Wellesley. Lord Wellesley served as Governor-General of
Important Tips Bengal from 1798 to 1805. He introduced the Censorship
Indian leaders provided varying estimates of the drain’s of Press Act in 1799, which required all newspapers and
magnitude, subject to individual and yearly fluctuations: periodicals to obtain a license from the British government
• R.C. Dutt: Around £20 million, half of India’s net in order to publish. Under Lord Hastings, who held
revenue, was drained out annually in the early 20th progressive views, these limitations were loosened, and pre-
century. censorship was abolished in 1818.

• M.G. Ranade: More than one-third of India’s national Important Tips


income was taken by the government through various • Governor-General John Adam passed an ordinance
means. in December 1823 which required that all matters
• Dadabhai Naoroji: Roughly one-fourth of India’s to be printed in a press or to be published should
raised funds, approximately $12 million annually, be printed and published under a licence from the
flowed to England. Governor-General in Council.
• William Digby: His calculations suggested an annual • John Adam was a British administrator in India,
drainage of £30 million. served as the acting Governor-General of the British
East India Company in 1823.
76. Solution: (a)
Exp) Option a is the correct answer. 79. Solution: (c)
Annie Besant launched the Home Rule League in 1916 to Exp) Option c is the correct answer.
campaign for dominion status. She published New India Sir Thomas Munro is linked to the land revenue settlement
and Commonweal. The Commonweal was a weekly dealing known as the Ryotwari Settlement. This system was

PYQ Workbook 244


HISTORY OF MODERN INDIA

established in British territories of southern India, where 82. Solution: (d)


farmers directly paid land revenue to the state. Munro, along Exp) Option d is the correct answer.
with Captain Alexander Read, devised this approach in the
Surendra Nath Banerjee was the first Indian to go to
late 18th century, and Munro implemented it during his
jail in performance of his duty as a journalist. In 1883,
governorship of Madras Presidency from 1819 to 1826.
Banerjee was arrested for publishing remarks in his paper,
Important Tips in contempt of court. He became the first Indian journalist
• The Ryotwari System in British Southern India marked to be imprisoned.
a departure from the Permanent Settlement. Conceived
83. Solution: (b)
by Captain Alexander Read and implemented by Sir
Thomas Munro (1819–26), it enabled direct land Exp) Option b is the correct answer.
revenue payment from farmers to the state. Indian Councils Act of 1861 empowered the Governor-
• Named after individual cultivators called “ryots,” it General of India or Viceroy to issue Ordinances. This
granted them complete land rights, including sale act empowered the Viceroy to issue ordinances during
and lease. Ryots, ensuring rent payment, couldn’t be an emergency, without the concurrence of the legislative
evicted. council. The life of such an ordinance was six months.
• Extending from Madras to regions like Bombay,
Berar, and Assam, this system curbed middlemen Important Tips
exploitation, benefiting villagers. The features of Indian Councils Act, 1861:
• It initiated the process of decentralization by
80. Solution: (b)
restoring the legislative powers to the Bombay and
Exp) Option b is the correct answer. Madras Presidencies.
Raja Rammohun Roy started India’s first Persian newspaper • It also provided for the establishment of new
the Mirat-ul-Akhbar. The newspaper was first published in legislative councils for Bengal, North-Western
1822. It was published on a weekly basis on Fridays. The Provinces and Punjab, which were established in 1862,
newspaper Mirat-ul-Akhbar means ‘Mirror of News’. Persian 1886 and 1897, respectively.
was chosen because it was still recognised in courts, and • It empowered the Viceroy to make rules and orders
was seen as a means to reach the intelligentsia, the top for the more convenient transaction of business in
the council. It also gave recognition to the ‘portfolio’
policymakers of the country.
system, which was introduced by Lord Canning in
Important Tips 1859.
Raja Rammohan Rai:
84. Solution: (a)
Journals and Magazines written by Raja Rammohan Rai:
Exp) Option a is the correct answer.
• The Brahmanical Magazine (1821)
Sri Aurobindo was affiliated with the English journal
• The Bengali weekly
Vande Mataram. Vande Mataram, a nationalist journal
• Samvad Kaumudi (1821)
started by Bipan Chandra Pal in 1905. The Newspaper
• Mirat-ul-Akbar was influential in disseminating revolutionary ideals in
• The Persian weekly Calcutta. Aurobindo Ghosh took over the editorship of
Books: Vande Mataram later and turned it into a mouthpiece for
• Tuhfat-ul-Muwahhidin (1804) his political and revolutionary ideologies.
• Appeal To The Christian Public 85. Solution: (b)
• Vedanta Gantha (1815) Exp) Option b is the correct answer.
• A Defence of Hindu Theism (1820) The Indian Councils Act of 1909 introduced a system of
• History of Indian Philosophy (1829) communal representation for Muslims by accepting the
concept of ‘separate electorate’. Under this, the Muslim
81. Solution: (a)
members were to be elected only by Muslim voters.
Exp) Option a is the correct answer.
Important Tips
The Supreme Court was set up for the first time in India
Provisions of the Morley-Minto reforms
under the Regulating Act, 1773 at Fort William in Calcutta.
It became the British India’s highest court from 1774 until • The legislative councils at the Centre and the provinces
increased in size.
1862, when the High Court of Calcutta was established by
the Indian High Courts Act 1861. It replaced the Mayor’s • The legislative councils at the Centre and the provinces
Court of Calcutta. were to have four categories of members as follows:

245 PYQ Workbook


HISTORY OF MODERN INDIA

• Ex officio members: Governor-General and • It extended franchise. About 10 per cent of the total
members of the executive council. population got the voting right.
• Nominated official members: Government officials • It provided for the establishment of a Reserve Bank of
who were nominated by the Governor-General. India to control the currency and credit of the country.
• Nominated non-official members: nominated by • It provided for the establishment of not only a Federal
the Governor-General but were not government Public Service Commission, but also a Provincial
officials. Public Service Commission and Joint Public Service
• Elected members: elected by different categories Commission for two or more provinces.
of Indians. The elected members were elected • It provided for the establishment of a Federal Court,
indirectly. The elected members were from the which was set up in 1937.
local bodies, the chambers of commerce, landlords,
universities, traders’ communities and Muslims. 87. Solution: (c)
• In the provincial councils, non-official members were
Exp) Option c is the correct answer.
in the majority. However, since some of the non-
official members were nominated, in total, a non- The Sadler Commission, also known as the Calcutta
elected majority was there. University Commission, was related to education. It was
• Indians were given membership to the Imperial established in 1917 under the chairmanship of M.E. Sadler,
Legislative Council for the first time.
the vice-chancellor of Leeds University. The commission
• The members could discuss the budget and move
was formed to inspect and address the challenges faced by
resolutions. They could also discuss matters of public
interest. Calcutta University, but its findings and recommendations
• They could also ask supplementary questions. had a significant impact on the overall education system
• No discussions on foreign policy or on relations with in India. The commission highlighted issues related
the princely states were permitted. to university, secondary, and collegiate education and
provided recommendations for improvement.
86. Solution: (a)
Exp) Option a is the correct answer. Important Tips
The 1917 Saddler Commission presented the following
The Government of India Act, 1935 is the main source
recommendations:
of the Indian Constitution. The structural part of the
• Teacher Training: Emphasis on teacher training and
Indian Constitution is, to a large extent, derived from the
provision of adequate facilities for this purpose.
Government of India Act of 1935. About 250 provisions of
• University Autonomy: Universities should function
the 1935 Act have been included in the Indian Constitution. independently without external government control.
• Intermediate Colleges: A distinction between
Important Tips
secondary courses and university education was
The features of Government of India Act, 1935:
established. The government gained authority to
• It provided for the establishment of an All-India establish Intermediate colleges.
Federation consisting of provinces and princely states
• Separate Boards: Different boards were formed for
as units.
Intermediate and Secondary education.
• It abolished dyarchy in the provinces and introduced
• Residential Teaching College: A separate residential
‘provincial autonomy’ in its place.
teaching college was suggested in Dacca.
• It provided for the adoption of dyarchy at the Centre.
Consequently, the federal subjects were divided into • Vice-Chancellor Appointment: A full-time, paid
reserved subjects and transferred subjects. Vice-Chancellor was to be appointed for administrative
purposes.
• It introduced bicameralism in six out of eleven
provinces • Streamlined Curriculum: Intermediate Colleges
should offer varied streams like Arts, Engineering,
• It further extended the principle of communal
Science, and Industrial education.
representation by providing separate electorates for
depressed classes (Scheduled Castes), women and • Entry Criteria: Only students passing the intermediate
labour (workers). examination, not just the Matriculation Examination,
• It abolished the Council of India, established by the should be eligible for university admission.
Government of India Act of 1858. The secretary of • Course Differentiation: Differentiation between Pass
state for India was provided with a team of advisors. and Honours courses in university education.

PYQ Workbook 246


HISTORY OF MODERN INDIA

• Board Establishment: A Board of Secondary and 91. Solution: (d)


Intermediate Education with representation from Exp) Option d is the correct answer.
schools, government, universities, and Intermediate • The Hindu: It was founded in the year 1878 by G.
Colleges should be established. Subramania Iyer in Madas. The Hindu started as a
• Flexibility in Rules: Universities should have flexibility weekly in 1878 and became a daily in 1889 and from
in framing rules and regulations. then on has been steadily growing.
• Women’s Education: Establishment of a Board of • Sudharak: It was a weekly newspaper started by Ganesh
Women Education at Calcutta University to promote Agarkar in 1888. It was edited by Gopal Krishna Gokhle.
women’s education. It was published both in English as well as Marathi
languages. It was published in the city of Pune.
88. Solution: (a) • The Voice of India: In the year 1883, Dadabhai Naoroji
Exp) Option a is the correct answer. started publishing the newspaper “The Voice of India”
Maulana Abdul Kalam Azad: Al-Hilal (The Crescent) from Bombay. It aims at providing an ideological
was an Urdu newspaper started by Maulana Abdul Kalam defense of Hindu society and culture through a series of
Azad in 1912. The newspaper’s first edition was published publications. Dadabhai Naoroji also started Rast Goftar
in Kolkata. The main aim of the newspaper was to create a (“The Truth Teller”) in 1854, which was an Anglo-
national consciousness among the masses, and to encourage Gujarati paper operating in Bombay.
them to join the freedom struggle in large numbers. • Bengalee: It was an English-language newspaper. It
Pherozeshah Mehta: Bombay Chronicle was an English- was published in Kolkata from 1862 to 1931. It started
language newspaper founded by Sir Pherozeshah Mehta. as a daily newspaper and later transitioned into a
He was a prominent lawyer who later became president of weekly publication. The Bengalee was edited by several
notable figures, including Girish Chandra Ghose and
the Indians National Congress in 1890, and member of the
Surendranath Banerjee.
Legislative Council of Bombay in 1893.
Annie Besant: New India was started by Annie Besant in 92. Solution: (c)
1914 as daily newspaper to highlight issues related to the Exp) Option c is the correct answer.
Indian freedom struggle.
The elections of the Central Legislative Assembly were held
Mahatma Gandhi: The Young India, a weekly journal, was under the Government of India Act, 1919 in the years 1926
published by Mahatma Gandhi from 1919 to 1931, and used and 1945, whereas the provincial elections of 1937 were
by him to spread his ideology of non-violence and resistance held in India under the Government of India Act of 1935.
against the British. There were not any elections held for Central Legislative
Assembly in 1937.
89. Solution: (a)
Exp) Option a is the correct answer. Important Tips

Lala Lajpat Rai published the Urdu daily newspaper named General elections of 1926:
‘Vande Mataram’ from Lahore in 1920. He also published an • They were held in British India between 28 October and
English daily newspaper called ‘People’. late November 1926 to elect members of the Imperial
Legislative Council and the Provincial Legislative
Important Tips Councils.
Other Newspaper: • The Swaraj Party was victorious in Provincial Council
• People: It was English daily newspaper started by Lala elections in Bengal and Madras, and also made gains in
Lajpat Rai. Bihar and Orissa.
• Tribune: The Tribune started publication on February Provincial elections of 1937:
2, 1881, in Lahore. It was started by Sardar Dyal • Provincial elections were held in British India in the
Singh Majithia, a public-spirited philanthropist. The winter of 1936-37 as mandated by the Government of
Tribune has two sister publications, Punjabi Tribune India Act 1935.
(in Punjabi) and Dainik Tribune (in Hindi). Now, it is
• Elections were held in eleven provinces – Madras,
published from Chandigarh.
Central Provinces, Bihar, Orissa, United Provinces,
90. Solution: (a) Bombay Presidency, Assam, NWFP, Bengal, Punjab
and Sindh.
Exp) Option a is the correct answer.
• Except for Bengal, Punjab, and Sindh, the Congress
The ‘Young India’ was started as a weekly by the New-York
had fared well in other regions.
based India Home Rule League of America (IHRLA). The
IHRLA was founded by Lala Lajpat Rai, who resided in the • In Bengal, NWFP, Assam and Bombay, Congress
United States from 1914-1919. emerged as the single largest party.

247 PYQ Workbook


HISTORY OF MODERN INDIA

• The performance of Congress in reserved constituencies • It separated, for the first time, provincial budgets
was not at all satisfactory except in the labour seats. from the Central budget and authorised the provincial
General elections of 1945: legislatures to enact their budgets.

• General elections were held in British India in 94. Solution: (b)


December 1945 to elect members of the Central
Exp) Option b is the correct answer.
Legislative Assembly and the Council of State.
• S.A. Dange: In 1922, Shripad Amrit Dange started the
• The Indian National Congress emerged as the largest
first socialist journal of India called the Socialist. It
party, winning 57 of the 102 elected seats.
was the first Indian Marxist journal. He also played an
93. Solution: (a) important role in the formation of Maharashtra state.

Exp) Option a is the correct answer. • Muzaffar Ahmad: He was a young man who did not
participate in the non-cooperation movement but
The Indian Councils Act, 1909 (not Government of India published nationalist journal in Bengali named Navyug
Act 1919) is also known as Morley-Minto Reforms Act. (New Age) in 1920 together with the firebrand Bengali
Government of India Act, 1919 is known as Montagu- poet Nazrul Islam.
Chelmsford Reforms. Government of India Act 1919 • Ghulam Hussain: He was the editor of Inquilab in
separated the Central and Provincial subjects. The 1923 which was published from Lahore. He worked as a
Government of India Act 1919 came into force in 1921. lecturer in a government college in Peshawar. He was also
the secretary of the North-Western Railway Workers’
Montague was the Secretary of State for India and Lord
Union.
Chelmsford was the Viceroy of India during Government of
• M Singaravelu Chettiar: In December 1923, M
India Act 1919.
Singaravelu started a fortnightly journal called the
Important Tips Labour Kisan Gazette. He also founded Labour Kisan
Party of Hindustan on 1 May 1923 in Madras.
The features of Government of India Act, 1919:
• It relaxed the central control over the provinces 95. Solution: (b)
by demarcating and separating the central and Exp) Option b is the correct answer.
provincial subjects. The theory of downward filtration, relevant during India’s
• It further divided the provincial subjects into two colonial era, pertained to Education. The Downward
parts– transferred and reserved. The transferred Filtration Theory, as proposed by Lord Macaulay in his
subjects were to be administered by the Governor 1835 Macaulay’s Minutes, aimed to address challenges
with the aid of Ministers responsible to the legislative in implementing the new education strategy. The theory
council. The reserved subjects, on the other hand, were suggested educating a select group of individuals
to be administered by the Governor and his executive who would then disseminate knowledge to the wider
council without being responsible to the legislative population. This approach was intended to efficiently
council. This dual scheme of governance was known spread education throughout society. Macaulay’s
as ‘dyarchy’. recommendations in his historic report sought to resolve the
• It introduced, for the first time, bicameralism and Oriental-Occidental Controversy, a conflict that hindered
direct elections in the country. Thus, the Indian the effective implementation of modern education in India
legislative council was replaced by a bicameral during the colonial period.
legislature consisting of an Upper House (Council of
96. Solution: (a)
State) and a Lower House (Legislative Assembly). The
majority of members of both the Houses were chosen Exp) Option a is the correct answer.
by direct election. The first three universities in India, namely Calcutta,
• It required that the three of the six members of Madras, and Bombay, were established in the year 1857.
the Viceroy’s executive Council (other than the This development was a response to the Sir Charles
Commander-in-Chief ) were to be Indian. Wood’s Dispatch of 1854. These universities were founded
to implement educational reforms and provide higher
• It extended the principle of communal representation
education in India.
by providing separate electorates for Sikhs, Indian
Christians, Anglo-Indians and Europeans. Important Tips
• It granted franchise to a limited number of people on Wood’s Despatch, 1854 recommendations:
the basis of property, tax or education. • Provincial Departments: The Despatch suggested
• It provided for the establishment of a public creating Department of Public Instruction in Bengal’s
service commission. Hence, a Central Public Service provinces: Bombay, Madras, Punjab, and North-
Commission was set up in 1926 to recruit civil servants. Western provinces.

PYQ Workbook 248


HISTORY OF MODERN INDIA

• Mass Education Expansion: Addressing limited • Bijolia is famous for its association with the 23rd Jain
opportunities, the Despatch stressed increasing Tirthankara or Lord Parshwanath.
primary, middle, and high schools for wider access to
education. 98. Solution: (b)

• University Establishment: It proposed universities Exp) Option b is the correct answer.


in Calcutta, Bombay, and Madras, modeled after Minto-Morley Reforms, 1909 (which is the name of Indian
London University, encompassing subjects like Arabic, Council Act of 1909) provided for communal representation
Sanskrit, Persian, law, and civil engineering. in British India by accepting the concept of ‘separate
• Grant-in-Aid System: Advocating financial aid, the electorate’. Under this, the Muslim members were to be
Despatch supported a grant-in-aid system for Indian elected only by Muslim voters. Thus, the Act ‘legalized
education. communalism’ and Lord Minto came to be known as the
• Language Emphasis: The Despatch emphasized Father of Communal Electorate.
teaching English while recognizing the significance of 99. Solution: (c)
teaching Indian languages.
Exp) Option c is the correct answer.
• Women’s Education: It recommended government
The Government of India Act of 1935 introduced the
support for women’s education.
‘Principle of Constitutional Autocracy’ as the act provided
• Teacher Training: Teacher training schools were
absolute power over all provinces to the Governors and
suggested in each province, with specialized schools
Governors-General, who worked on behalf of the crown.
for engineering, medicine, and law instructors.
Both Governors and Governors-General enjoy veto power
• Graded School Network: The Despatch advocated over any legislation passed by the legislatures, but it has been
establishing a nation-wide network of graded schools. provided in the constitution or act itself.
97. Solution: (b) 100. Solution: (a)
Exp) Option b is the correct answer. Exp) Option a is the correct answer.
Vijay Singh Pathik, with the help of Ganesh Shankar Bipin Chandra Pal: He started the New India Newspaper.
Vidhyarthi, popularise Bijolian Peasant Movement in India New India had two publications. The Weekly was started by
through Pratap newspaper published from Kanpur. He Bipin Chandra Pal, while Annie Basant started New India
was a legendary freedom fighter, joined the movement in (Daily. The newspaper spread patriotic feelings among
Indians.
1916 and started leading it. He wrote several articles in the
famous newsletter Pratap at that time. British authorities Aurobindo Ghosh: Vande Mataram, a nationalist journal,
were compelled to form an enquiry commission headed by started by Bipan Chandra Pal in 1905. The Newspaper was
influential in disseminating revolutionary ideals in Calcutta.
Justice Bindu Lal Bhattacharya.
Aurobindo Ghosh took over the editorship of Vande
Important Tips Mataram later and turned it into a mouthpiece for his
Bijolia Peasant Movement (1897-1941): political and revolutionary ideologies.
• The Bijolia Peasant Movement was a peasant revolt Brahmabandhab Upadhyay: He started an extremist daily
in present-day Rajasthan’s Mewar region over the Sandhya in 1904. The newspaper condemned the British
excessive collecting of land revenue. government’s action in extremely strong words.
• The movement started in 1897, led by Sadhu Sitaram. Mohammad Ali: The Comrade was a weekly English-
• Rao Krishna Singh was the chieftain of Bijolia and language newspaper that was published and edited by
Maharana Fateh Singh the ruler of Mewar. Mohammad Ali between 1911 and 1914. Mohammad Ali
was a good orator and writer, writtten articles to various
• Krishna Singh imposed many taxes on the farmers,
newspapers.
even one on the marriage of their daughters. The
common men tried to raise their voice in the royal 101. Solution: (a)
court of Mewar but it went in vain.
Exp) Option a is the correct answer.
• Farmers were paying 84 different types of taxes at that
General Sir Arthur Thomas Cotton is regarded as the
time.
pioneer of irrigation works in South India. He dedicated his
• When Vijay Singh Pathik was forbidden from entering life to constructing irrigation and navigation canals across
the area. Then leaders like Seth Jamuna Lal, Haribhau British India. Notable projects include the Dowleswaram
Upadhyaya and Manik Lal Verma took over the Barrage, Prakasam Barrage, and the Kurnool Cuddappah
movement in their control. Canal. His contributions are honored in Andhra Pradesh and
• The farmers could get rid of the unjustified taxes only Tamil Nadu, where the Sir Arthur Cotton Museum stands as
in 1941, after a continuous struggle of 44 years. a testament to his efforts.

249 PYQ Workbook


HISTORY OF MODERN INDIA

102. Solution: (a) While these secondary reasons played a role, the
Exp) Option a is the correct answer. fundamental objective of addressing the shortage of
The first Chief Justice of Supreme Court which was educated personnel for administrative roles remained
established by the East India Company under Regulating the primary driver for the widespread promotion of
Act of 1773 was Sir Elijah Impey. For the effective working modern education by the British Government in India.
of judiciary, he made rules containing 133 articles. After
which, it was recognized as the Civil procedure of court. 106. Solution: (a)
Exp) Option a is the correct answer.
103. Solution: (b)
Permanent settlement was most prevalent in West Bengal,
Exp) Option b is the correct answer.
Bihar, Odisha, UP, Andhra Pradesh and Madhya Pradesh
Kesari is a Marathi newspaper which was founded on 4
whereas Ryotwari System was prevalent in most of southern
January 1881 by Lokmanya Bal Gangadhar Tilak. The
India, first introduced in Madras and was later extended to
newspaper threw light on the burning issues of the Indian
Freedom Movement. It was used as a mouthpiece for the Maharashtra, Berar, East Punjab, Coorg and Assam.
freedom struggle. The paper today is still one of the leading
107. Solution: (a)
dailies of Maharashtra.
Exp) Option a is the correct answer.
104. Solution: (a)
The First Indian Education Commission, known as the
Exp) Option a is the correct answer. Hunter Commission, was appointed by Viceroy Lord
The First Sanskrit Mahavidyalaya at Varanasi was founded Ripon in 1882. The commission, led by Sir William Wilson
by Jonathan Duncan in 1791. He proposed the establishment Hunter, was tasked with assessing the state of elementary
of a Sanskrit college to promote the development and
education in British territories and proposing measures to
preservation of Sanskrit language and literature. This
extend and enhance it.
initiative was supported by the governor general, Lord
Cornwallis. The first principal of the institution was John 108. Solution: (a)
Muir, and the college played a significant role in preserving
and advancing Sanskrit studies in India. The institution Exp) Option a is the correct answer.
eventually evolved and changed its status over time, On 4 th June 1903, Mahatma Gandhi released his newspaper
becoming Sampurnanand Sanskrit University in 1974. ‘Indian Opinion’ in South Africa to fight against racial
discrimination and demand civil rights for Indians in that
105. Solution: (a)
country. It was a weekly newspaper and was published in
Exp) Option a is the correct answer.
English, Hindi, Tamil and Gujarati.
The British Government’s primary motive behind
promoting modern education in India was to address 109. Solution: (b)
the challenge of staffing minor administrative positions.
Given the logistical and financial difficulties of importing Exp) Option b is the correct answer.
a sufficient number of English officials, they opted to train ‘Dash Roja’ periodical was started by Abdul Ghaffar Khan
Indians in modern education to fill these administrative during National movement. He also known as Bacha Khan
roles. or Badshah Khan, and honourably addressed as Fakhr-
Important Tips e-Afghan. He was a Pashtun independence activist, and
Other significant reasons for the British Government’s founder of the Khudai Khidmatgar resistance movement
push for modern education: against British colonial rule in India.
• Cultural Influence: The promotion of Western culture Important Tips
and values through education aimed to reshape Indian Abdul Ghaffar Khan:
society according to British ideals, fostering a more
• He was nicknamed “Frontier Gandhi” by his close
interconnected colonial relationship.
associate Amir Chand Bombwal.
• Loyalty to British Rule: By educating a class of Indians
• Ghaffar Khan met Gandhi and entered politics in
in modern subjects, the British sought to create a
1919 during agitation over the Rowlatt Acts.
segment of the population that was more aligned with
and loyal to the British Raj, further solidifying their • In 1987, he was awarded the Bharat Ratna Prize, the
control over India. highest Indian honour that can be given to civilians.
He was the first non-Indian to receive this honour.
• Economic Development: The British anticipated
that providing modern education to Indians would Khudai Khidmatgar resistance movement:
contribute to the development of the Indian economy • It was a non-violent movement against British
by equipping individuals with skills relevant to occupation of the Indian subcontinent led by Abdul
emerging industries and trades. Ghaffar Khan in the North-West Frontier Province.

PYQ Workbook 250


HISTORY OF MODERN INDIA

• Following the arrest of Khan and other leaders in 1929, Important Tips
the movement formally joined the Indian National • The Subsidiary Alliance was a system introduced by
Congress after they failed to receive support from the Lord Wellesley, the Governor-General of British India
All-India Muslim League. from 1798 to 1805.
• Members of the Khudai Khidmatgar were organised • It was a policy designed to expand and consolidate
and the men wore bright red shirts as uniforms, while British control over the Indian princely states.
the women wore black garments. • Some Provisions were:
• The Khudai Khidtmatgar opposed Partition as the • The British East India Company would guarantee
movement not being in favour of the creation of the protection to the ally against external threats.
independent nation of Pakistan. • The British would station a contingent of their
armed forces within the territory of the ally to
110. Solution: (a) ensure control and maintain stability.
Exp) Option a is the correct answer. • The ally would be responsible for providing
resources, including financial support, provisions,
On August 20, 1917, the British Government declared, for
and supplies, to sustain the British armed contingent
the first time, that its objective was the gradual introduction
stationed in their territory.
of responsible Government in India. The said declaration
was presented in the British Parliament by Edwin Samuel 114. Solution: (d)
Montagu (Secretary of State for India), popularly called Exp) Option d is the correct answer
Montagu Declaration. This declaration proposed the In 1911, Gopal Krishna Gokhale introduced a bill in
increased participation of Indians in the administration and the Imperial Legislative Council for the introduction of
the development of self-governing institutions in India. compulsory and free primary education in India. The
bill was not passed, but it helped to raise awareness of the
111. Solution: (a)
issue and paved the way for the introduction of compulsory
Exp) Option a is the correct answer. primary education in India in later years.
In 1863, the Viceroy of India, John Lawrence, decided to 115. Solution: (b)
shift the summer capital of the British Raj to Shimla.
Exp) Option b is the correct answer
Simla, located in the foothills of the Himalayas, provided
The Act V of 1843 relating to Slavery in India denied the
relief from the intense summer heat experienced in the
masters the use of Courts to assert their claims on slaves.
plains of India. Recognizing the benefits of the cool This meant that slaves could no longer be forced to stay
climate in Simla, Lord Lawrence decided to move his with their masters, and they were free to leave and seek
Council to Simla during the summer months. employment elsewhere. The Act also made it illegal to sell
or transfer slaves, and it provided for the punishment of
112. Solution: (d)
those who violated the law.
Exp) Option d is the correct answer.
116. Solution: (d)
The Voice of India newspaper was edited by Dadabhai
Exp) Option d is the correct answer
Naoroji, not Gopal Krishna Gokhale.
The Government of India Act, 1935, reserved seats for
Important Tips
women in legislatures in accordance with the allocation of
• Navjivan was a newspaper founded by Mahatma seats for different communities.
Gandhi in 1919, and he served as its editor.
117. Solution: (c)
• The Mahratta was a prominent newspaper established
by Bal Gangadhar Tilak in 1881. Exp) Option c is the correct answer
• The Bengalee was a prominent English-language The Government of India Act, 1935, provided for the creation
newspaper founded by Surendranath Banerjea in 1879. of a Federal Court, which would have original jurisdiction in
disputes between the federal government and the provinces,
and appellate jurisdiction in appeals from the High Courts.
113. Solution: (b)
The Federal Court was inaugurated on October 1, 1937,
Exp) Option b is the correct answer. and it functioned until the establishment of the Supreme
Subsidiary Alliance System restricted the ally’s freedom to Court of India in 1950.
enter into independent agreements or engage in warfare
118. Solution: (c)
with other rulers. The system aimed to establish British
Exp) Option c is the correct answer
dominance and control over the Indian princely states by
ensuring their dependence on the British for security and The Hunter Commission was appointed by the Viceroy of
administration. India, Lord Ripon, in 1882 to survey the state of education

251 PYQ Workbook


HISTORY OF MODERN INDIA

in India. The Commission’s report, which was published Important Tips


in 1883, endorsed the Despatch of 1854, and also
• The Limitation Law of 1859 was a significant step in the
recommended that greater emphasis be placed on primary
protection of ryots from exploitation by moneylenders.
education. However, the Act was not perfect. It was difficult to
Important Tips enforce, and it did not address the underlying causes
of usury, such as poverty and lack of access to credit.
Major recommendations by the Hunter Commission of
1882 resulted in the following changes in the education • The Limitation Law of 1859 was repealed in 1963 and
system of British India: replaced by the Limitation Act of 1963.
• The new Act has a number of provisions that are
• Preference was given to literate candidates for
designed to protect ryots from exploitation by
government jobs in the lower levels, along with
moneylenders. For example, the Act stipulates that
expansion of primary schools in backward districts. the maximum rate of interest that can be charged on
• District and municipal boards were entrusted with a loan is 18%.
the management of primary education under the • The Act also provides for a number of debt relief
Local Self Government Act. The funds were separated schemes that can be used by ryots to escape from the
for rural and urban areas to avoid funds earmarked for clutches of moneylenders.
rural schools being misappropriated by urban schools.
122. Solution: (b)
• Secondary schools were to be established by private
parties with funds provided by the government. Exp) Option b is the correct answer
Model schools fully run by the government were to be The Summary Settlement of 1856 was a land revenue
opened in each district to guide such private schools. settlement that was implemented in Awadh, India. The
• Secondary school curriculum was also revised with settlement was based on the assumption that the Talukdars,
academic and vocational courses diversified into or landlords, were interlopers with no permanent stakes in
different branches. the land.

• Missionary schools were discouraged, and Indian Important Tips


participation in the private school system was • The British government believed that the Talukdars
solicited by the raj. Special care was supposed to be had acquired their land through force and fraud, and
taken in advancing the education of girls and women. that they did not have the legitimate right to collect
revenue from the peasants.
119. Solution: (a) • The Summary Settlement of 1856 removed the
Exp) Option a is the correct answer Talukdars from power and replaced them with
a system of direct revenue collection from the
One of Lord Curzon’s notable contributions was the peasants.
establishment of the Imperial Agricultural Department in
• The settlement was a major blow to the Talukdars, and
1901. Under Lord Curzon’s initiative, the Indian Agriculture
it led to a great deal of resentment among them. The
Service was created to focus on improving agricultural resentment eventually boiled over in the form of the
practices, increasing agricultural production, and addressing 1857 Indian Rebellion.
agricultural issues in India.
• The Summary Settlement of 1856 was a controversial
120. Solution: (c) decision, but it had a significant impact on the history
of Awadh. The settlement led to the downfall of the
Exp) Option c is the correct answer
Talukdars, and it paved the way for the development
The Deccan Agriculturalists’ Relief Act of 1879 was of a more equitable system of land revenue collection.
enacted with the objective of restricting the sale of land for
indebtedness to outsiders. The act aimed to provide relief 123. Solution: (a)
and protection to agriculturalists in the Deccan region of Exp) Option a is the correct answer
India, who were often burdened with debt and faced the Maharani Tapaswini was a social reformer and educationist
risk of losing their land due to the inability to repay loans. who founded the Mahakali Pathshala in Calcutta in 1893.
The school was one of the first schools in India to provide
121. Solution: (b)
education for girls and women. Maharani Tapaswini was a
Exp) Option b is the correct answer strong advocate for women’s education, and she believed that
The Limitation Law of 1859 was passed by the British education was the key to women’s empowerment.
government to address the issue of usury, or excessive interest
rates, charged by moneylenders to ryots, or peasants. The Act 124. Solution: (d)
stipulated that loan bonds signed between moneylenders Exp) Option d is the correct answer.
and ryots would have validity only for three years. This K.N. Natarajan was the editor of the journal Indian Social
meant that if a moneylender did not recover his loan within Reformer, which was started in 1890. The journal aim was
three years, he would lose the right to collect the debt. to grow a small rural society into modern Indian society.

PYQ Workbook 252


HISTORY OF MODERN INDIA

125. Solution: (c) 128. Solution: (b)


Exp) Option c is the correct answer. Exp) Option b is the correct answer.
Statement 1 is correct: The province of Assam was created Statement 1 is incorrect- The Indian press did protest
in the year 1911. Prior to 1911, Assam was a part of the against the Vernacular Press Act of 1878. The Vernacular
larger Bengal Presidency. However, in 1911, the British Press Act was a law enacted in British India in 1878 to
colonial administration decided to separate Assam from curtail the freedom of the Indian-language (i.e., non-
Bengal and establish it as a separate province. English) press. The act was proposed by Lord Lytton, the
Statement 2 is correct: Eleven districts comprising Assam Viceroy of India at the time, and was unanimously passed.
were separated from the Lieutenant Governorship of The protests were led by the Indian National Congress. The
Bengal and established as an independent administration protests were successful in forcing the government to repeal
under a Chief Commissioner in the year 1874. This the Act in 1882 by the Lord Rippon.
administrative reorganization was carried out to give Assam Statement 2 is correct- The Vernacular Press Act of 1878
a more distinct and separate administrative identity. was introduced by Viceroy Lytton against the counsel of
126. Solution: (a) his own Law Member, Sir Henry Maine. The Act gave the
government the power to censor vernacular (i.e., non-
Exp) Option a is the correct answer.
English) newspapers and periodicals. The Act was met with
Statement 1 is correct: Under the Hastings Plan of 1772,
widespread protests from the Indian press and public.
it was indeed proposed that each district would have
both a civil court and a criminal court. This was done to 129. Solution: (d)
establish a more organized judicial system and ensure proper Exp) Option d is the correct answer.
administration of justice at the district level.
Statement 1 is correct- The Permanent Settlement was
Statement 2 is correct: Judges appointed under the Hastings
introduced by Lord Cornwallis during his tenure as the
Plan were assisted by native assessors who had knowledge
Governor-General of Bengal. It was implemented in 1793
of Hindu and Islamic laws. These assessors played a role
and aimed to fix the land revenue demand in perpetuity
in providing guidance and expertise in matters related to
for zamindars (landlords) in certain parts of British India,
religious and personal laws.
particularly in Bengal, Bihar, and Odisha.
Statement 3 is incorrect: The Sadar Diwani Adalat was
Statement 2 is correct- The Ryotwari Settlement was
not meant to settle mercantile cases, but to hear appeals
introduced by Thomas Munro, who served as a British
from the civil courts of the districts. It was established at
administrator in India during the 19th century. The
Calcutta and presided over by the Governor and his council.
There was no limit on the value of the cases that could be Ryotwari system involved direct settlement of land revenue
appealed to this court. with individual peasants (ryots) who were recognized as
the cultivators and became the owners of their land. It was
Statement 4 is incorrect: The Hastings Plan of 1772
implemented in areas such as Madras Presidency, Bombay
did introduce some procedural improvements in the
Presidency and some parts of Central Provinces.
administration of justice, such as abolishing unreasonable
fines, restricting the raising of rent, doing away with the Statement 3 is correct- The Mahalwari Settlement was
judicial powers of the zamindars, and establishing a regular introduced by Holt Mackenzie, a British administrator who
system of reporting and record-keeping. served as the Governor of the North-Western Provinces
during the early 19th century. The Mahalwari system
127. Solution: (d) involved the settlement of land revenue with village
Exp) Option d is the correct answer. communities (mahals) collectively, where the village was
Comrade, Hamdard, Zamindar and Al Hilal were some of the treated as the unit of revenue assessment and collection. It
newspapers that reflected the concerns of educated Indian was implemented in areas such as Punjab, North-Western
Muslims during the Khilafat Movement. Provinces, parts of Bihar, and some parts of Central
Provinces.
Important Tips
• The Comrade and Hamdard were published by 130. Solution: (a)
Maulana Mohammad Ali Jauhar, a prominent leader Exp) Option a is the correct answer.
of the movement.
Vithalbhai Patel was a Gujarati lawyer and politician
• The Zamindar was published by Maulana Zafar Ali who was one of the most prominent leaders of the Indian
Khan, another leader and journalist who supported independence movement. He was elected as the first non-
the Khilafat cause.
official member to be Speaker of the Central Legislative
• The Al Hilal was published by Abul Kalam Azad, a Assembly on 24 August 1925. He was also a member of the
prominent scholar and activist who also joined the Indian National Congress, and he served as the President of
movement the Congress in 1930 and 1931.

253 PYQ Workbook


HISTORY OF MODERN INDIA

131. Solution: (a) 135. Solution: (b)


Exp) Option a is the correct answer. Exp) Option b is the correct answer.
The British Committee of the Indian National Congress The colonial economy had the following effects on the
was established in 1889 in Britain by the Indian National Indian economy:
Congress. Its primary objective was to bring attention
Option a is correct: The British presence inhibited
to Indian issues among the British public, as the British
indigenous capitalism by imposing monopolies, tariffs, and
government was responsible for governing India. Major
restrictions on Indian industries and trade.
leaders involved in the committee included WC Banerjee,
Dadabhai Naroji, and Charles Bradlaugh. The first Option b is not correct: Laissez-faire did not actively
chairman of the committee was William Wedderburn, and promote indigenous capitalism, but rather served the
the secretary was William Digby. interests of British capitalists and traders at the expense of
Indian producers and consumers.
132. Solution: (d)
Option c is correct: The ‘white collective monopoly’ came
Exp) Option d is the correct answer. earliest and remained most pronounced in Eastern India,
The early nationalists opposed the Council Act of 1892 where the British East India Company established its first
because: foothold and exploited the resources and markets of Bengal,
The act did not give the Indians control over the public Bihar, and Orissa.
funds. The members could only ask questions on the budget, Option d is correct: The Bombay hinterland was difficult
but not vote on it or propose amendments to it. to penetrate before the construction of railways, because
The act did not give adequate representation to the Indians. of the rugged terrain, the resistance of local rulers, and the
The official majority was maintained in both central and competition from other European powers
provincial councils, and the non-official members were
mostly nominated by the governor-general or governors. 136. Solution: (b)
The principle of indirect election was also unsatisfactory, Exp) Option b is the correct answer.
as it excluded a large section of the Indian population from The office of the Secretary of State for India was created by
participating in the electoral process. The Government of India Act 1858.
133. Solution: (c) • This act transferred the power and territories of the
Exp) Option c is the correct answer. British East India Company to the British Crown after
Statement I is correct: The Permanent Settlement was rarely the Indian Rebellion of 1857.
extended to any region beyond Bengal. The Permanent • The Secretary of State for India was the British Cabinet
Settlement was an agreement between the East India minister and the political head of the India Office
Company and the Bengali landlords or zamindars to fix the responsible for the governance of British India, Aden,
land revenue in 1793. It was introduced by Lord Cornwallis and Burma.
in Bengal, Bihar and Odisha. It granted the zamindars the • He was assisted by a council of 15 members and reported
hereditary rights of ownership of land and made them to the Parliament of India.
responsible for paying a fixed amount of revenue to the
Company. 137. Solution: (c)
Statement II is not correct: After 1810, agricultural Exp) Option c is the correct answer.
prices rose, increasing the value of harvest produce, and Under the Mahalwari system, the agricultural lands did not
enlarging the income of the Bengal zamindars. Since the belong to the government, but rather to a group of village
revenue demand was fixed under the Permanent Settlement,
landlords or village communities. The Mahalwari system
the colonial state could not claim any share of this enhanced
was introduced in 1833. It was prevalent in Central Province,
income. The zamindars benefited from this situation and
North-West Frontier, Agra, Punjab and Gangetic Valley. The
accumulated wealth and power. They also invested in trade,
mahals or estates were assessed as a whole and the revenue
commerce, banking and urban development.
was distributed among the landlords or villagers according
134. Solution: (d) to their shares.
Exp) Option d is the correct answer.
138. Solution: (c)
English Education Act of 1835 was based on the
Exp) Option c is the correct answer.
recommendations of Thomas Babington Macaulay, who
argued that Western learning was superior and that English Statement 1 is correct: The Anglicists, who advocated for
should be the medium of instruction. The Act stated that the promotion of English education and western knowledge,
the existing Oriental Institutions would receive no further sought to reduce the expenditure on printing Sanskrit
support from the Company. Thus, there was no provision and Arabic texts. They believed that resources should
of fresh awards of stipends to students of existing Oriental be allocated towards promoting English education and
Institutions. modern sciences instead.

PYQ Workbook 254


HISTORY OF MODERN INDIA

Statement 2 is incorrect: The Anglicists did aim to reduce Statement 3 is correct: Utilitarians believed that the
the funding for printing Sanskrit texts, as mentioned in happiness of the majority was more important than the
Statement 1, but they did not express a desire to continue rights of the individual. Liberals, on the other hand, believed
printing in Persian. Their focus was on emphasizing that the rights of the individual were paramount.
English education and reducing resources allocated to Statement 4 is correct: Utilitarianism had distinct
classical Indian languages. authoritarian tendencies because Utilitarians believed that
Statement 3 is correct: The Orientalists believed that the government should intervene in society to promote the
Sanskrit and Arabic were important cultural and intellectual greatest good for the greatest number of people. This could
traditions, and that they should be preserved and studied. lead to the government restricting the rights of individuals
They argued that the stipends for students of Arabic and in the name of the greater good.
Sanskrit were a good investment.
142. Solution: (c)
Statement 4 is incorrect: The Orientalists started a new
Sanskrit College in Banaras not Delhi. The Banaras Sanskrit Exp) Option c is the correct answer.
College was created by Jonathan Duncan in 1791. Statement I is true: India’s share of world manufacturing
output did steadily decline during the 19th and 20th
139. Solution: (b)
centuries. In 1750, India accounted for about 22.6% of
Exp) Option b is the correct answer. world manufacturing output. By 1947, this share had
The Permanent Settlement did not pay attention to the declined to just 2.4%.
customary occupancy rights of the peasants. These rights Statement II is False: The British policies favored the
gave the peasants the right to cultivate a particular piece export of raw materials from India to Britain, while Indian
of land, even if they did not own it. Under the Permanent industries faced restrictions and unfair competition
Settlement, these rights were often ignored, and the from British goods. This resulted in the decline and even
zamindars were free to evict peasants from their land. destruction of many traditional Indian industries, leading to
140. Solution: (b) a decrease in production.
Exp) Option b is the correct answer. 143. Solution: (b)
Statement 1 is incorrect: The Morley-Minto reforms of 1909 Exp) Option b is the correct answer.
were named after the Secretary of State for India, John Morley
Statement 1 is true: The guarantee of a 5% return on
(British parliamentarian), and the Viceroy of India, Gilbert
investment was an incentive for private companies to invest
Elliot-Murray-Kynynmound, also known as Lord Minto.
in railways.
The Viceroy of India was not a British parliamentarian.
Statement 2 is not true: The railways were not to be
Statement 2 is Correct: The Morley-Minto reforms
managed mainly by the Government. The railways were built
introduced a limited form of self-government by increasing
and managed by private companies.
the number of elected Indians in the Legislative Councils at
both the central and provincial levels. Statement 3 is not true: There was a system of preferential
freight charges. The railways charged lower freight rates for
Statement 3 is Correct: The Morley-Minto reforms
goods that were considered to be of strategic importance to
maintained the principle of communal representation,
the British, such as military supplies and raw materials.
which allowed separate electorates for different religious
communities. This provision ensured that British officials Statement 4 is true: The companies were to get the land
retained their majority in the Imperial Legislative Council free from the Government as an added incentive for private
and prevented the emergence of a united Indian political companies.
front.
144. Solution: (c)
141. Solution: (d) Exp) Option c is the correct answer.
Exp) Option d is the correct answer. The Court of Proprietors was the governing body of the East
Statement 1 is correct: Utilitarianism is a philosophy that India Company, and it was made up of shareholders of the
advocates that actions are right if they promote happiness Company. The Act did not change the qualifications for
and wrong if they promote unhappiness. The rule of law voting in the Court of Proprietors, which remained the
is necessary for societal betterment because it ensures that same as they had been before the Act was passed.
everyone is treated equally under the law and that no one is
Important Tips
above the law.
Regulating Act, 1773 Act - Key Provisions
Statement 2 is incorrect: James Mill wrote a book called
“Utilitarianism” in 1861 in which he outlined the principles • Governor-General of Bengal: The post of “Governor
of Utilitarianism and argued that they should be applied to of Bengal” was changed to “Governor-General of
society. Principles of Utilitarianism were not written in the Bengal” and he was also given the responsibility to
book ‘Utilitarians and the Raj’. oversee the presidencies of Madras and Bombay.

255 PYQ Workbook


HISTORY OF MODERN INDIA

• Four Board of Administrators: Four Board of • Staff and Resources: Universities were empowered
Administrators members were selected by the to hire new staff, recruit professionals, and establish
Governor-General in Bengal (Philip Francis, Clavering, libraries. This aimed to enrich students’ knowledge
Monson, and Barwell). and learning experience.
• High Court Inauguration: The High Court was • Fellowships: The Act reduced the number of fellows
inaugurated in Calcutta. There were four judges in from 100 to 50. Universities with more than 50 fellows
total, including the Chief Justice. faced greater accountability. Fellowships were limited
• Primary & Appellate Jurisdiction: Primary and to a six-month term instead of a lifetime.
appellate jurisdiction was assigned to the Supreme • Senate Election: Senate elections were introduced, with
Court. 20 elected fellows for Madras, Bombay, and Calcutta
• Prohibited from Bribes & Gifts: Military and civilian universities, and 15 for others. The government could
officers under the Company were prohibited from intervene if the senate failed to formulate rules.
accepting any gifts, donations, or prizes from private • Syndicates and Representation: The Act recognized
businesses and Indians, according to this Act. Syndicates in the education system. All university
• Civil & Military Affairs: India’s civil and military teachers gained representation in the university Senate.
affairs as well as its revenues were required to be • Affiliation: Colleges needed affiliation with a specific
disclosed to the British Crown. university, which might not be within the local area’s
• Pay Rise: The company’s officers and staff were given jurisdiction. This aimed to strengthen the connection
a pay raise. between colleges and universities.
• Research Grants: INR 5 lakhs were allocated over five
145. Solution: (a) years to fellows for further studies and research.
Exp) Option a is the correct answer. • Governor-General Council: The Act empowered the
The Deccan Riots Commission was a commission set Governor-General Council to handle jurisdictional
matters related to universities and their territories.
up by the British government in 1875 to investigate the
causes of the Deccan Riots, a series of peasant uprisings Impact:
that took place in the Deccan region of India in 1875. The • Criticism: The Sadler Commission (1917) criticized
commission found that the main cause of the riots was the the Act for increasing governmental control over
universities.
indebtedness of the peasantry.
• Nationalist Reaction: Nationalists saw the Act as
146. Solution: (d) reinforcing imperialism and suppressing nationalist
Exp) Option d is the correct answer. sentiments. Gopal Krishna Gokhale labeled it a
“retrograde measure.”
The University Commission appointed in 1902 by Lord
• Primary Education: Lord Curzon used the Act’s
Curzon aimed to recommend reforms in university
reforms to enhance primary education, allocating
education in India. The commission, led by Law member substantial financial aid to schools between 1905 and
Thomas Raleigh, included two Indian members: Syed 1912.
Hussain Bilgrami and Justice Gurudas Banerjee. Their
recommendations covered a wide range of topics, including 147. Solution: (d)
reformation of University Senates, greater representation Exp) Option d is the correct answer.
of affiliated colleges, stricter monitoring of affiliated
About the Government of India Act of 1935, Jawaharlal
institutions, curricular reforms, education and examinations, Nehru had said, “We were provided with a car with all brakes
research, student welfare, and state scholarships. These and no engine”. He said this because the act failed to meet
recommendations led to the enactment of the Indian certain expectations of nationalists. The act was silent over
Universities Act 1904, despite strong opposition from the ‘dominion status’ or ‘swaraj’. It provided the system of
certain quarters in India. dyarchy at the Centre, which made the Governor General’s
control over reserved subjects absolute and over transferred
Important Tips
subjects very effective.
The Indian Universities Act of 1904;
• Autonomy and Activities: Universities gained the 148. Solution: (b)
authority to conduct private teaching and other Exp) Option b is the correct answer.
activities. They were responsible for teaching The Ryotwari system was a system of land revenue collection
and examination supervision. Focus on research introduced in the Madras Presidency of British India in
was emphasized, leading to the establishment of 1802 by Thomas Munro. Under the Ryotwari system, the
laboratories across disciplines. peasants were given the right to own the land they were

PYQ Workbook 256


HISTORY OF MODERN INDIA

cultivating and to pay land revenue directly to the British the British Parliament. He was a prominent scholar, writer,
government. The Ryotwari system was intended to give the and advocate for Indian rights.
peasants more incentive to improve their land and to make
153. Solution: (d)
them more prosperous.
Exp) Option d is the correct answer.
149. Solution: (d) The Young India, a weekly journal, was published by
Exp) Option d is the correct answer. Mahatma Gandhi from 1919 to 1931, and used by him to
• The Hindu: It was founded in the year 1878 by G. spread his ideology of non-violence and resistance against
Subramania Iyer in Madras. The Hindu started as a the British. Hence, it was not associated with revolutionary
weekly in 1878 and became a daily in 1889 and from activities. Whereas Sandhya, Yugantar and Gadar were
associated with promoting the revolutionary activities.
then on has been steadily growing.
• Kesari: It is a Marathi newspaper which was founded 154. Solution: (b)
on 4 January 1881 by Lokmanya Bal Gangadhar Tilak. Exp) Option b is the correct answer.
The newspaper threw light on the burning issues of the The Montagu-Chelmsford Report formed the basis of
Indian Freedom Movement. the Government of India Act, 1919. This Act is known as
• Bengalee: It was an English-language newspaper. The Montagu-Chelmsford Reforms. Montagu was the Secretary
Bengalee was edited by several notable figures, including of State for India and Lord Chelmsford was the Viceroy of
Girish Chandra Ghose and Surendranath Banerjee. India. The Government of India Act of 1919 came into force
in 1921.
• Hindustani: Gangaprasad Verma was a town planner
under British rule. He also printed a newspaper named 155. Solution: (d)
Hindustani, which was later managed by Pandit Exp) Option d is the correct answer.
Kishanprasad Kaul.
Amrit Bazar Patrika was founded by Sisir Kumar Ghosh and
• Sudharak: Gopal Krishna Gokhale established the Motilal Ghosh. It was started on 20th February, 1868 in the
Servants of India Society in 1905 and edited “Sudharak”, village of Amrita Bazar as a Bengali weekly newspaper. Sisir
a quarterly Journal of the Poona Sarvajanik Sabha. He Kumar Ghosh had declared that the primary objective of his
also started Hitavada newspaper in 1911. paper was “to awaken the people and infuse in them the noble
spirit of patriotism”. Bal Gangadhar Tilak had acknowledged
150. Solution: (b) Sisir Kumar Ghosh as his Guru, and in his newspaper Kesari,
Exp) Option b is the correct answer. he attempted to follow him.
The Government of India Act, 1919 created a new office 156. Solution: (b)
of the High Commissioner for India in London and
Exp) Option b is the correct answer.
transferred to him some of the functions hitherto performed
by the Secretary of State for India. The Leader was seen as a liberal party newspaper but it was
not merely a mouthpiece organ. The newspaper criticized
151. Solution: (d) the British Raj, but also Congress leaders and even liberal
Exp) Option d is the correct answer. colleagues. For instance, Gopal Krishna Gokhale, a liberal,
was not spared for his support to the Press Act of 1910. The
Sir Lionel Curtis is known as the father of the Dyarchy. Leader was one of the most influential English-language
Dyarchy was a system of double government introduced by newspapers in India during British Raj. It was founded
the Government of India Act (1919) for the provinces of by Madan Mohan Malviya, the paper was published in
British India. Allahabad. Under C. Y. Chintamani, a dynamic editor from
[Note: The option d mentions the name as Sir Leoni Cartis but 1909 to 1934, it acquired a large readership in North India.
the father of dyarchy is Sir Lionel Curtis. The Government
Important Tips
of India Act (1919) established the British Indian provinces’
Other Newspapers:
double-government arrangement known as Dyarchy. Lionel
George Curtis (1872-1955), a British diplomat and novelist, • New India: It was started by Annie Besant in 1914
wrote works that affected the creation of the Commonwealth as daily newspaper to highlight issues related to the
of Nations.] Indian freedom struggle.
• Young India: Lala Lajpat Rai published the Young
152. Solution: (a) India journal in 1916. It was started as a weekly
Exp) Option a is the correct answer. magazine in English but the journal was immediately
The first Indian elected to the British House of Commons banned from circulation in India and Britain by the
was Dadabhai Naoroji, who contested as a member of the Colonial Government on the grounds of ‘seditious’
Liberal Party. Known as the “Grand Old Man of India,” content. The magazine was later published by Gandhi
Naoroji served as President of the Indian National Congress from 1919 to 1931 to spread his message of swaraj and
and played a pivotal role in advocating for Indian interests in non-violence.

257 PYQ Workbook


HISTORY OF MODERN INDIA

• Free Press Journal: It is an Indian English-language 157. Solution: (d)


daily newspaper that was established in 1928 by Exp) Option d is the correct answer.
Swaminathan Sadanand. It was a supporter of the Tarak Nath Das started a monthly magazine in 1908 called
Independence movement. It is the first Indian-owned “Free Hindusthan” to condemn policies of the British Indian
government and encourage resistance. He was a professor
English daily newspaper published from Mumbai. of political science at Columbia University and a visiting
faculty in several other universities.

PYQ Workbook 258


HISTORY OF MODERN INDIA

HISTORY OF MODERN INDIA


MISCELLANEOUS
*This unit consists of questions from the Princely States, the Famous Personalities during the freedom
struggle, Post-Independence India and Other miscellaneous aspects from 1600 to 1947 CE.

6.1. UPSC CSE Previous Years’ Questions 5. With reference to Indian National
Movement, consider the following pairs:
1. Who among the following is associated with
‘Songs from Prison’, a translation of ancient Person Position held
Indian religious lyrics in English? 1. Sir Tej Bahadur President, All India
[UPSC CSE Pre 2021] Sapru Liberal Federation
(a) Bal Gangadhar Tilak Member, The
(b) Jawaharlal Nehru 2. K.C. Neogy
Constituent Assembly
(c) Mohandas Karamchand Gandhi
(d) Sarojini Naidu General Secretary,
3. P.C. Joshi Communist Party of
2. With reference to Madanapalle of Andhra India
Pradesh, which one of the following
statements is correct? Which of the pairs given above is/are correctly
[UPSC CSE Prelims 2021] matched? [UPSC CSE Pre. 2019]
(a) Pingali Venkayya designed the tricolour (a) 1 only
Indian National Flag here. (b) 1 and 2 only
(b) Pattabhi Sitaramaiah led the Quit India (c) 3 only
Movement of Andhra region from here. (d) 1, 2 and 3
(c) Rabindranath Tagore translated the
National Anthem from Bengali to English 6. Who among the following were the founders
here. of the “Hind Mazdoor Sabha” established in
(d) Madame Blavatsky and Colonel Olcott set 1948? [UPSC CSE Pre. 2018]
up headquarters of Theosophical Society (a) B. Krishna Pillai, E.M.S. Namboodiripad
first here. and K.C. George
3. The Vital-Vidhvansak, the first monthly (b) Jayaprakash Narayan, Deen Dayal
journal to have the untouchable people as Upadhyay and M.N. Roy
its target audience was published by: (c) C.P. Ramaswamy Iyer, K. Kamaraj and
[UPSC CSE Pre 2020] Veeresalingam Pantulu
(a) Gopal Baba Walangkar (d) Ashok Mehta, T.S. Ramanujam and G.G.
(b) Jyotiba Phule Mehta
(c) Mohandas Karamchand Ghandhi 7. Consider the following events:
(d) Bhimrao Ramji Ambedkar 1. The first democratically elected
4. With reference to the book “Desher Katha” communist party government formed in
written by Sakharam Ganesh Deuskar a State in India.
during the freedom struggle, consider the 2. India’s then largest bank, ‘Imperial Bank
following statements: of India’, was renamed ‘State Bank of
1. It warned against the Colonial State’s India’.
hypnotic conquest of the mind. 3. Air India was nationalised and became
2. It inspired the performance of swadeshi the national carrier.
street plays and folk songs.
4. Goa became a part of independent India.
3. The use of ‘desh’ by Deuskar was in the
specific context of the region of Bengal. Which of the following is the correct
Which of the statements given above are chronological sequence of the above events?
correct? [UPSC CSE Pre 2020] [UPSC CSE Pre. 2018]
(a) 1 and 2 only (a) 4 - 1 - 2 - 3
(b) 2 and 3 only (b) 3 - 2 - 1 - 4
(c) 1 and 3 only (c) 4 - 2 - 1 - 3
(d) 1, 2 and 3 (d) 3 - 1 - 2 – 4

259 PYQ Workbook


HISTORY OF MODERN INDIA

8. He wrote biographies of Mazzini, Garibaldi, 2. Acharya J.B. Kripalani was one of


Shivaji and Shrikrishna; stayed in America Mahatma Gandhi’s colleagues in his
for some time; and was also elected to the Champaran investigation.
Central Assembly. He was Which of the statements given above is/are
[UPSC CSE Pre. 2018] correct?
(a) Aurobindo Ghosh (a) 1 only
(b) Bipin Chandra Pal (b) 2 only
(c) Lala Lajpat Rai (c) Both 1 and 2
(d) Motilal Nehru (d) Neither 1 nor 2
9. Annie Besant was [UPSC CSE Pre. 2013] 13. Four resolutions were passed at the famous
1. responsible for starting the Home Rule Calcutta Session of Indian National
Movement Congress in 1906. The question of either
2. the founder of the Theosophical Society retention or rejection of these four
3. once the President of the Indian National resolutions became the cause of a split in
Congress Congress at the next Congress Session held
in Surat in 1907. Which one of the following
Select the correct statement/statements using was not one of those resolutions?
the codes given below:
[UPSC CSE Pre 2010]
(a) 1 only
(a) Annulment of the partition of Bengal
(b) 2 and 3 only
(b) Boycott
(c) 1 and 3 only
(c) National education
(d) 1, 2 and 3
(d) Swadeshi
10. Mahatma Gandhi said that some of his
14. Who among the following Governor-
deepest convictions were reflected in a
Generals created the Covenanted Civil
book titled, “Unto this Last’’ and the book
Service of India which later came to be
transformed his life. What was the message
known as the Indian Civil Service?
from the book that transformed Mahatma
Gandhi ? [UPSC CSE Pre. 2011] [UPSC CSE Pre 2010]
(a) Uplifting the oppressed and poor is the (a) Warren Hastings
moral responsibility of an educated man (b) Wellesley
(b) The good of individual is contained in the (c) Cornwallis
good of all (d) William Bentinck
(c) The life of celibacy and spiritual pursuit 15. Who of the following is the author of
are essential for a noble life a collection of poems called ‘Golden
(d) All the statements (a), (b) and (c) are Threshold’? [UPSC CSE Pre 2009]
correct in this context (a) Aruna Asaf Ali
11. With reference to Indian freedom struggle, (b) Annie Besant
Usha Mehta is well-known for? (c) Sarojini Naidu
(d) Vijaya Lakshmi Pandit
[UPSC CSE Pre. 2011]
(a) running the secret congress radio in the 16. Who among the following Gandhian
wake of quit India movement followers was a teacher by profession?
(b) participating in the second round table [UPSC CSE Pre 2008]
conference (a) A. N. Sinha
(c) leading a contingent of Indian national (b) Braj Kishore Prasad
army (c) J.B. Kripalani
(d) assisting in the formation of Interim (d) Rajendra Prasad
Government under Pandit Jawaharlal
17. Who among the following translated the
Nehru
autobiography of Madam Curie in Hindi?
12. Consider the following statements: [UPSC CSE Pre 2008]
[UPSC CSE Pre 2010] (a) Atal Bihari Vajpayee
1. Dr. Rajendra Prasad persuaded Mahatma (b) Lal Bahadur Shastri
Gandhi to come to Champaran to (c) Choudhary Charan Singh
investigate the problem of peasants. (d) Gobind Ballabh Pant

PYQ Workbook 260


HISTORY OF MODERN INDIA

18. Who among the following wrote the poem 24. Consider the following statements:
“Subh-e Azadi”? [UPSC CSE Pre 2008] [UPSC CSE Pre 2007]
(a) Sahir Ludhiyanvi 1. Robert Clive was the first Governor-
(b) Faiz Ahmed Faiz General of Bengal.
(c) Muhammad Iqbal 2. William Bentinck was the first Governor-
(d) Maulana Abul Kalam Azad General of India.
19. Elizabeth Hawley is well-known for her Which of the statements given above is/are
correct?
writings relating to which one of the
following? [UPSC CSE Pre 2008] (a) 1 only
(b) 2 only
(a) Historical monuments in India (c) Both 1 and 2
(b) Regional dances in India (d) Neither 1 nor 2
(c) Himalayan expeditions
(d) Wildlife in India 25. Who has written the book named ‘Gita
Rahasya’? [UPSC CSE Pre 2005]
20. Who, among the following, wrote the book (a) Mahatma Gandhi
‘Bahubivah’? [UPSC CSE Pre 2007] (b) Bal Gangadhar Tilak
(a) Raja Rammohan Roy (c) Vinoba Bhave
(b) Ishwar Chandra Vidyasagar (d) Gopal Krishna Gokhale
(c) Pandit Ramabai 26. Who among the following was a proponent
(d) Rabindranath Tagore of Fabianism as a movement?
21. The song ‘Amar Sonar Bangla’ written during [UPSC CSE Pre 2005]
the Swadeshi Movement of India inspired (a) Annie Besant
the liberation struggle of Bangladesh and (b) A.O Hume
was adopted as the National Anthem of (c) Michael Madhusudan Dutt
Bangladesh. Who wrote this song? (d) D. R. Palme Dot
[UPSC CSE Pre 2007] 27. Consider the following Viceroys of India
(a) Rajni Kanta Sen during the British rule:
(b) Dwijendralal Roy [UPSC CSE Pre 2004]
(c) Mukunda Das 1. Lord Curzon
(d) Rabindranath Tagore 2. Lord Hardinge
3. Lord Chelmsford
22. Consider the following statements:
4. Lord Irwin
1. Jawaharlal Nehru was in his fourth term
as the Prime Minister of India at the time Which one of the following is the correct
of his death. chronological order of their tenure?
(a) 1, 2, 3, 4
2. Jawaharlal Nehru represented Rae Bareilly
(b) 2, 4, 1, 3
constituency as a Member of Parliament. (c) 1, 4, 2, 3
3. The first non-Congress Prime Minister of (d) 2, 3, 1, 4
India assumed the office in the year 1977.
28. Which one of the following pairs is not
Which of the statements given above is/are correctly matched? [UPSC CSE Pre 2004]
correct? [UPSC CSE Pre 2007] (a) Pitt’s India Act: Warren Hastings
(a) 1 and 2 (b) Doctrine of Lapse: Dalhousie
(b) 3 only (c) Vernacular Press Act: Curzon
(c) 1 only (d) Ilbert Bill: Ripon
(d) 1 and 3
29. Consider the following Princely States of
23. Who wrote the book “The Story of The the British rule in India:
Integration of the Indian States”? 1. Jhansi
[UPSC CSE Pre 2007] 2. Sambalpur
(a) B.N. Rao 3. Satara
(b) C. Rajagopalachari The correct chronological order in which
(c) Krishna Menon they were annexed by the British is:
(d) V.P. Menon [UPSC CSE Pre 2004]

261 PYQ Workbook


HISTORY OF MODERN INDIA

(a) 1, 2, 3 33. Who among the following was the President


(b) 1, 3, 2 of the All-India States Peoples Conference
(c) 3, 2, 1 in 1939? [UPSC CSE Pre 2001]
(d) 3, 1, 2 (a) Jaya Prakash Narayan
30. Which one of the following statements is (b) Jawaharlal Nehru
not correct? [UPSC CSE Pre 2003] (c) Sheikh Abdullah
(a) The National Song Vande Mataram was (d) Sardar Vallabhbhai Patel
composed by Bankim Chandra Chatterji 34. A London branch of the All-India Muslim
originally in Bengali. League was published in 1908 under the
(b) The National Calendar of India based presidency of: [UPSC CSE Pre 2001]
on Saka era has its 1st Chaitra on 22nd (a) Aga Khan
March Normally & 21st March in a leap (b) Ameer Ali
year. (c) Liaquat Ali Khan
(c) The design of the National Flag of India (d) M.A. Jinnah
was adopted by the Constituent Assembly
on 22nd July 1947. 35. At the time of Partition of India, which one
(d) The song Jana-Gana-Mana, composed of the following provinces of British India
originally in Bengali by Rabindranath came forward with a plan for a United and
Tagore was adopted in its Hindi version Independent existence?
by the Constituent Assembly on 24 [UPSC CSE Pre 2000]
January 1950 as the national anthem. (a) Punjab
31. Match list-I with list-II and select the (b) Assam
correct answer using the codes given below (c) Bengal
the lists: (d) Bihar

List-I (Person) List-II (Events) 36. The native State of Tripura became involved
in the Freedom movement early in the 20th
A. Macdonald 1. Doctrine of Lapse century because: [UPSC CSE Pre 2000]
B. Linlithgow 2. Communal Award (a) The kings of Tripura were always anti-
C. Dalhousie 3. August Offer British
(b) The Bengal revolutionaries took shelter
D. Chelmsford 4. Dyarchy
in Tripura
Code: [UPSC CSE Pre 2002] (c) The tribes of the state were fiercely
A B C D freedom loving
(a) 3 2 1 4 (d) There were already some groups fighting
(b) 3 2 4 1 against the kingship and its protector, the
(c) 2 3 1 4 British
(d) 2 3 4 1
37. The last major extension of British Indian
32. With reference to the Indian freedom territory took place during the time of:
struggle, which one of the following [UPSC CSE Pre 2000]
statements is not correct? (a) Dufferin
[UPSC CSE Pre 2002] (b) Dalhousie
(a) Hakim Ajmal Khan was one of the leaders (c) Lytton
to start a nationalist and militant Ahrar (d) Curzon
movement.
(b) When the Indian National Congress was 38. Which one of the following statements is
formed, Sayyid Ahmad Khan opposed it. not correct? [UPSC CSE Pre 1999]
(c) The All-India Muslim League which was (a) ‘Neel Darpan’ was a play based on the
formed in 1906 vehemently opposed exploitation of the indigo farmers
the partition of Bengal and separate (b) The author of the play ‘Ghashiram
electorates . Kotwal’s is Vijay Tendulkar
(d) Maulana Barkat Ullah and Maulana (c) The play ‘’Navanna’ by Nabin Chandra
Obeidullah Sindhi were among those Das was based on the famine of Bengal
who formed a Provisional Government of (d) Urdu theatre used to depend heavily on
India in Kabul. Parsi theatre

PYQ Workbook 262


HISTORY OF MODERN INDIA

39. Match list-I with list-II and select the 2. Ilbert Bill: Ripon
correct answer using the codes given below 3. Ferozshah Mehta: Indian National
the lists: Congress
List-I (Books) List-II (Authors) 4. Badruddin Tayabji : Muslims League
A. The first 1. Rabindranath Select the correct answer using the codes
Indian War of Tagore given below:
Independence (a) 1, 2, 3 and 4
B. Anand Math 2. Sri Aurobindo (b) 2 and 4
(c) 1, 3 and 4
C. Life Divine 3. Bankim Chandra
Chatterjee (d) 1, 2 and 3
D. Sadhana 4. Vinayak Damodar 44. Match list-I with list-II and select the
Savarkar correct answer using the codes given below
Code: [UPSC CSE Pre 1999] the lists:
A B C D List-I List-II
(a) 4 3 2 1 A. Surat Split 1. 1929
(b) 3 4 1 2
B. Communal Award 2. 1928
(c) 4 3 1 2
(d) 3 4 2 1 C. All Party Convention 3. 1932
40. At a time when empires in Europe were D. Poorna Swaraj Resolution 4. 1907
crumbling before the might of Napoleon, 5. 1905
which one of the following Governors- Code: [UPSC CSE Pre 1997]
General kept the British flag flying high in A B C D
India? [UPSC CSE Pre 1999] (a) 4 3 1 4
(a) Lord Dalhousie (b) 4 3 2 1
(b) Lord Cornwallis (c) 2 5 4 1
(c) Lord Wellesley (d) 1 4 2 3
(d) Lord Hastings
45. “A graduate at 18, professor and associate
41. The Governor-General who followed editor of the Sudharak at 20, Secretary of
a spirited “Forward” policy towards the Sarvajanik Sabha and of the Provincial
Afghanistan was: [UPSC CSE Pre 1999]
Conference at 25, Secretary of the National
(a) Minto Congress at 29, leading witness before
(b) Dufferin an important Royal Commission at 31,
(c) Elgin Provincial legislator at 34, Imperial
(d) Lytton legislator at 36, President of the Indian
42. What is the correct sequence of the following National Congress at 39. a patriot whom
events? Mahatma Gandhi himself regarded as his
1. Tilak’s Home Rule League master”. This is how a biographer describes:
2. Kamagatamaru Incident [UPSC CSE Pre 1997]
3. Mahatma Gandhi’s arrival in India (a) Pandit Madan Mohan Malaviya
Select the correct answer using the codes (b) Mahadev Govind Ranade
given below: [UPSC CSE Pre 1998] (c) Gopal Krishna Gokhale
(a) 1, 2, 3 (d) Bal Gangadhar Tilak
(b) 3, 2, 1 46. Who among the following was associated
(c) 2, 1, 3 with suppression of thugs?
(d) 2, 3, 1 [UPSC CSE Pre 1997]
43. Which of the following pairs are correctly (a) General Henry Prendergast
matched? [UPSC CSE Pre 1998] (b) Captain Sleeman
1. Theodore Beck: Mohammadan Anglo- (c) Alexander Burres
Oriental College, Aligarh (d) Captain Robert Pemberton

263 PYQ Workbook


HISTORY OF MODERN INDIA

47. Match list-I with list-II and select the 50. Which one of the following pairs is not
correct answer using the codes given below correctly matched? [UPSC CSE Pre 1995]
the lists: (a) Jamnalal Bajaj - Satyagraha Ashram at
List-I List-II (Event) Wardha
(Period) (b) Dadabhai Naoroji - Bombay Association
(c) Lala Lajpat Rai - National School at
A. 1883 1. Announcement of Communal
Lahore
Award from Whitehall
(d) Bal Gangadhar Tilak - Satya Shodhak
B. 1906 2. Formation of the All-India Sabha
State People Conference
51. Which one of the following was an emigre
C. 1927 3. Foundation of Muslim League Communist journal of M.N. Roy?
at Dhaka
[UPSC CSE Pre 1995]
D. 1932 4. First session of National (a) Kisan Sabha
Conference at Kolkata (b) The Worker
Code: [UPSC CSE Pre 1996] (c) Vanguard
A B C D (d) Anushilan
(a) 4 3 1 2
(b) 3 4 1 2 52. What is the correct chronological sequence
(c) 4 3 2 1 of the following events in the political life
(d) 3 4 2 1 of Mahatma Gandhi?
1. Champaran Satyagrah
48. Match list-I with list-II and select the
correct answer using the codes given below 2. Ahmedabad Mill strike
the lists: 3. Kheda Satyagrah
4. Non-cooperation Movement
List-I List-II
Code: [UPSC CSE Pre 1993]
A. Surendranath 1. Hind Swaraj (a) 2, 4, 3, 1
Banerjee (b) 1, 2, 3, 4
B. M.K. Gandhi 2. The Indian (c) 4, 3, 2, 1
Struggle (d) 3, 4, 2, 1
C. Subhash 3. Autobiographical 53. “Indian cotton merchant, banker;
Chandra Bose Writings Congressman and a close associate of
D. Lajpat Rai 4. A Nation in Mahatma Gandhi.” The description fits
Making with: [UPSC CSE Pre 1993]
Code: [UPSC CSE Pre 1996] (a) G. D. Birla
A B C D (b) M. R. Jayakar
(a) 4 1 3 2 (c) Jamnalal Bajaj
(b) 1 4 3 2 (d) V. S. Shrinivas Sastri
(c) 1 4 2 3 54. At the time of India’s Independence,
(d) 4 1 2 3 Mahatma Gandhi was:
49. Consider the following statements about [UPSC CSE Pre 1993]
Jawaharlal Nehru: (a) A member of Congress Working
1. He was the president of the Congress Committee
Party in 1947. (b) Not a member of the Congress
2. He presided over the Constituent (c) The President of the Congress
Assembly. (d) The General Secretary of the Congress
3. He formed the first Congress Ministry
in United Province before India’s 55. The founder of Boy Scouts and Civil Guides
Independence. movement in India was:
Of these statements: [UPSC CSE Pre 1996] [UPSC CSE Pre 1993]
(a) 1, 2 and 3 are correct (a) Charles Andrews
(b) 1 and 3 are correct (b) Robert Montgomery
(c) 1 and 2 are correct (c) Richard Temple
(d) None is correct (d) Baden-Powell

PYQ Workbook 264


HISTORY OF MODERN INDIA

56. The first Factory Act for restricting the 60. Who had merged Sikkim in India?
working hours of women and children, [U.P.P.C.S. (Pre) 2020]
and authorizing local government to make (a) Lord Hastings
necessary rules was adopted during whose (b) Lord William Bentinck
time? (c) Lord Dalhousie
(a) Lord Lytton (d) Lord Auckland
(b) Lord Bentinck 61. “The Rise and Growth of Economic
(c) Lord Ripon Nationalism in India” was written by:
(d) Lord Canning [U.P.P.C.S. (Pre) 2019]
6.2. Other Examination Previous Years’ (a) Partha Sarthi Gupta
(b) S. Gopal
Questions
(c) B.R. Nanda
57. In the contexts of Indian history, which (d) Bipin Chandra
of the following statement is/are correct 62. With reference to the Civil Administration
regarding Vaikom Satyagraha? in 1905, which of the statements is/are
1. It was a satyagraha againsts untouchability correct?
and caste discrimination. 1. Lord Curzon decided to rearrange the
2. Mahatma Gandhi participated in this provincial boundaries.
Satyagraha. 2. A new province was constituted, called
Select the correct answer using the codes East Bengal and Assam.
given below. [U.P.P.C.S. (Pre.) 2021] Select the correct answer using the codes
(a) 1 only given below: [U.P.P.C.S. (Pre) 2019]
(b) 2 only (a) 1 only
(c) Both 1 and 2 (b) Both 1 and 2
(d) Neither 1 nor 2 (c) 2 only
(d) Neither 1 nor 2
58. Who of the following was sent as
representative of labourers by Indian 63. With reference to revolts in British India,
Government in the Washington Summit of people known as ‘Kallar’ belonged to which
I.L.O. in the year 1919? of the following region?
[U.P.P.C.S. (Pre) 2020] [U.P.P.C.S. (Mains) 2017]
(a) V.P. Wadia (a) Kolhapur
(b) N.M. Joshi (b) Midnapore
(c) Madurai
(c) C.F. Andrews
(d) Rangpur
(d) Joseph Baptista
64. Which of the following events occurred
59. Match list-I with list-II and select the first? [U.P.P.C.S. (Mains) 2017]
correct answer using the codes given below:
(a) Lord Lytton’s Delhi Darbar
List-I (Book) List-II (Writer) (b) Santhal Revolt
A. The Story of My 1. Surendranath (c) First Anglo-Sikh War
Deportation Banerjee (d) Ilbert Bill Controversy
B. Gita Rahasya 2. Maulana Abul 65. Who among the following is known as
Kalam Azad ‘Bombay Trimurti’? [UPPCS (Mains) 2017]
(a) Tilak, Gokhale, Naoroji
C. Nation in Making 3. Lala Lajpat Rai
(b) Mehta, Tilak, Tyabji
D. India Wins 4. Bal Gangadhar (c) Mehta, Telang, Tyabji
Freedom Tilak (d) Naoroji, Telang, Deshmukh
Code: [UPPCS (Pre) 2020]
66. Who is the author of “Annihilation of
A B C D Caste”? [U.P.P.C.S. (Mains) 2016]
(a) 3 4 1 2 (a) Jyotirao Phule
(b) 4 2 1 3 (b) Dr. B. R. Ambedkar
(c) 2 4 1 3 (c) Kanshi Ram
(d) 4 3 2 1 (d) Ram Mohan Roy

265 PYQ Workbook


HISTORY OF MODERN INDIA

67. In August 1947, who of the following (a) Aga Khan


leaders did not participate anywhere in (b) M.A. Jinnah
Independence Day celebrations? (c) Faiz Hussain
[U.P.P.C.S. (Mains) 2016] (d) Karim Jalal
(a) Jawaharlal Lal Nehru 74. In which of the following books did
(b) Mahatma Gandhi Mahatma Gandhi called British Parliament
(c) Vallabh Bhai Patel as sterile and prostitute?
(d) Rajendra Prasad [U.P.P.C.S. (Pre) 2014]
(a) Sarvodaya or Universal dawn
68. Who among the following used to say,
(b) An Autobiography or the Story of My
“Wrong means never take us to right ends”?
Experiments with Truth
[U.P.P.C.S. (Mains) 2015] (c) Hind Swaraj
(a) Sardar Patel (d) The Story of a Satyagrahi
(b) M.K. Gandhi
75. Which one of the following is not correctly
(c) Lala Lajpat Rai
matched? [U.P.P.C.S. (Pre) 2014]
(d) Jawaharlal Nehru
(a) Bhagat Singh : The Indian Sociologist
69. Who among the following was the strong (b) Sachindranath Sanyal : Bandi Jivan
supporter of the principle that “Which is (c) Lala Ramsaran Das : Dreamland
morally wrong, can never be politically (d) Bhagwati Charan Vohra : The Philosophy
right”? [U.P.P.C.S. (Mains) 2015] of Bomb
(a) Jawaharlal Nehru 76. Who one of the following is the biographer
(b) Sardar Patel of Jawahar Lal Nehru?
(c) M. K. Gandhi [U.P.P.C.S. (Mains) 2014]
(d) C. Rajagopalachari (a) Frank Moraes
70. Who among the following is the author of (b) Dom Moraes
(c) Louis Fischer
the book ‘The Problems of the Far East’?
(d) Webb Miller
[U.P.P.C.S. (Pre) 2015]
(a) Lawrence 77. Who of the following had written, “India’s
(b) Curzon salvation will not be achieved under the
leadership of Mahatma Gandhi”?
(c) Churchill
(d) Lytton [U.P.P.C.S. (Mains) 2014]
(a) M.A. Jinnah
71. Which one of the following is not correctly (b) Clement Richard Attlee
matched? [U.P.P.C.S. (Pre) 2015] (c) Winston Churchill
(a) Motilal Nehru - Nehru Report (d) Subhash Chandra Bose
(b) M. K. Gandhi - Champaran Movement 78. Who among the following was the first
(c) S.C. Bose - Forward Bloc Governor-General of free India? [U.P.P.C.S.
(d) M.A. Jinnah - Khilafat Movement (Mains) 2014]
72. Who addressed Gandhiji as ‘one-man (a) Lord Mountbatten
boundary force’? (b) Sir Stafford Cripps
[U.P.P.C.S. (Re. Exam) (Pre) 2015] (c) C. Rajagopalachari
(d) Dr. Rajendra Prasad
(a) Churchill
(b) Attlee 79. Who one of the following had commanded
(c) Mountbatten Mahatma Gandhi to spend the first year
(d) Simon in India ‘with his ears open but his mouth
shut’? [U.P.P.C.S. (Mains) 2014]
73. Who amongst the following presided over (a) Dadabhai Naoroji
the All-Parties Muslim Conference held at (b) Bal Gangadhar Tilak
Delhi on 31st December 1928? (c) Pherozeshah Mehta
[U.P.P.C.S. (Mains) 2015] (d) Gopal Krishna Gokhale

PYQ Workbook 266


HISTORY OF MODERN INDIA

80. Which one of the following statements is (c) Mahadev Desai


not correct about Dadabhai Naoroji? (d) Manubehn
[U.P.P.C.S. (Pre) 2014] 86. Which one of the following statements is
(a) He was the first Indian to be appointed as not correct about Rabindranath Tagore?
Professor of Mathematics and Physics at [U.P.P.C.S. (Mains) 2013]
Elphinstone College, Bombay.
(a) He sang the glories of ancient India and
(b) He was elected as the member of British
its culture.
Parliament in 1892.
(b) He considered Shivaji and Guru Govind
(c) He started a Gujarati Journal, ‘Rast
Singh as nation builders.
Goftar’.
(c) Many of his songs refute the heroism of
(d) For four times he had presided over the
Marathas.
Indian National Congress.
(d) He favoured some aspects of the Swadeshi
81. Which of the following is not correctly Movement.
matched? [U.P.P.C.S. (Mains) 2014]
87. Match list-I with list-II and select the
(a) Lord Cornwallis: Permanent Settlement correct answer using the codes given below
(b) Lord Wellesley: Subsidiary Alliance
the lists:
System
(c) Sir John Shore: Anglo-Nepal War List-I List-II
(d) Lord Hastings: Third Anglo-Maratha War A. Subhash 1. Throwing of the
82. Consider the following which concluded Chandra Bose bomb in Central
the subsidiary alliances with Lord Wellesley Assembly
and finds the correct chronological order of B. Vallabhbhai 2. Haripura Session
their treaties from the code given below: Patel of Indian National
1. Hyderabad Congress
2. Mysore C. Iqbal 3. Operation Polo
3. Oudh D. Batukeshwar 4. Allahabad Session
4. Sindhia Dutt of Muslim League,
Code: [U.P.P.C.S. (Mains) 2014] 1930
(a) 1, 2, 3, 4 Code: [UPPCS (Mains) 2013]
(b) 1, 3, 4, 2 A B C D
(c) 4, 3, 2, 1 (a) 2 3 4 1
(d) 3, 2, 1, 4 (b) 1 2 3 4
83. The Lucknow Session of Indian National (c) 4 3 2 1
Congress that took place in 1916 was (d) 3 4 2 1
presided over by: [U.P.P.C.S. (Mains) 2013] 88. Which one of the following is correctly
(a) Annie Besant matched? [U.P.P.C.S. (Mains) 2013]
(b) Lala Lajpat Rai (a) Lord Ellenborough - Annexation of
(c) Motilal Nehru Awadh
(d) A.C. Majumdar (b) Lord Dalhousie - Annexation of Sindh
84. Which one of the following is correctly (c) Lord Wellesley - Fourth Anglo-Mysore
matched? [U.P.P.C.S. (Mains) 2013] War
(a) Bhagat Singh - An Introduction to the (d) Sir John Shore - Third Anglo-Maratha
Dreamland War
(b) Subhas Chandra Bose - Bandi Jivan 89. Who among the following Governor
(c) Sachindranath Sanyal - Indian Struggle Generals of Bengal was associated with
(d) Bhagwati Charan Vohra - Why I am an Third Anglo-Mysore War?
Atheist? [U.P.P.C.S. (Mains) 2013]
85. The memoirs entitled ‘Bapu: My Mother’ (a) Warren Hastings
was written by: [U.P.P.C.S. (Mains) 2013] (b) Lord Cornwallis
(a) B.R. Nanda (c) Sir John Shore
(b) Rajkumari Amrita Kaur (d) Lord William Cavendish Bentinck

267 PYQ Workbook


HISTORY OF MODERN INDIA

90. Tinkathia System in Champaran meant: 97. The statement “I am a socialist and a
[U.P.P.C.S. (Pre) 2013] republican and I do not believe in Kings
(a) Cultivation of Indigo on the 3/20 area of and Princes” is associated with:
land [U.P.P.C.S. (Pre) 2011]
(b) Cultivation of Indigo on 3/19 area of land (a) Narendra Dev
(c) Cultivation of Indigo on 3/18 area of land (b) Achyut Patwardhan
(d) None of the above (c) Jai Prakash Narayan
(d) Jawahar Lal Nehru
91. Who among the following was the first Law
Minister of India? [U.P.P.C.S. (Mains) 2012] 98. Who was responsible for encouraging the
local self Govt. in India?
(a) M. C. Sitalwad
(b) Dr. B.R. Ambedkar [U.P.P.C.S (Pre) 2010]
(c) Kailash Nath Katju (a) Lord Mayo
(d) Rafi Ahmad Kidwai (b) Lord Lytton
(c) Lord Canning
92. Who compared Curzon’s administration in (d) Lord Ripon
India to that of Aurangzeb?
99. Anglo-Nepal War took place during the
[U.P.P.C.S. (Mains) 2012] reign of– [U.P.P.C.S. (Mains) 2010]
(a) B.G. Tilak (a) Lord Cornwallis
(b) G.K. Gokhale (b) Lord Hastings
(c) Dadabhai Naoroji (c) Lord Wellesley
(d) Annie Besant (d) Warren Hastings
93. Where did the Indian Congress got divided 100. The first communist Government of India
into two wings moderates and extremists? was established in the province of:
[U.P.P.C.S. (Mains) 2012] [U.P.P.C.S. (Pre) 2010]
(a) Surat Session, 1907 (a) Tamil Nadu
(b) Lahore Session, 1909 (b) Andhra Pradesh
(c) Calcutta Session, 1911 (c) Kerala
(d) Karachi Session, 1913 (d) West Bengal
94. The conquest of Sindh by British was 101. Who among of the following had told:
completed during the period of– “destruction is the best method of dealing
[U.P.P.C.S. (Mains) 2012] with the foreign clothes”?
(a) Lord Ellenborough [U.P.P.C.S. (Pre) 2010]
(b) Lord Hardinge (a) Rabindranath Tagore
(c) Lord Aukland (b) Mahatma Gandhi
(d) Lord Amherst (c) Chitranjan Das
(d) Subhas Chandra Bose
95. Which one of the following pairs is correctly
102. Who sang ‘Hindustan Hamara’ of Iqbal and
matched? [U.P.P.C.S. (Pre) 2011]
‘Jan-gan-man’ in the Central Assembly at
(a) Durga Das : The Life of Mahatma Gandhi midnight of 14/15 August 1947? [U.P.P.C.S.
(b) Louis Fischer : India From Curzon to (Pre) 2010]
Nehru and After (a) Rameshwari Nehru
(c) Frank Moraes : Jawaharlal Nehru - A (b) Meera Ben
Biography (c) Sucheta Kriplani
(d) Maulana Abul Kalam Azad : India (d) M.S. Subbulakshmi
Divided
103. Who of the following called Gandhiji
96. The book ‘Springing Tiger’ was written by: ‘Mahatma’ for the first time?
[U.P.P.C.S. (Mains) 2011] [U.P.P.C.S. (Mains) 2010]
(a) Hugh Toye (a) Jawahar Lal Nehru
(b) Louis Fisher (b) Madan Mohan Malviya
(c) Webb Miller (c) Rabindra Nath Tagore
(d) None of the above (d) Subhash Chandra Bose

PYQ Workbook 268


HISTORY OF MODERN INDIA

104. Which of the following is not correctly (c) C. Rajagopalachari, Rajendra Prasad, S.
matched? [U.P. P.C.S. (Pre) 2010] Radhakrishnan, Fakhruddin Ali Ahmad
(a) Larry Collins and Dominique Lapierre - (d) Rajendra Prasad, S. Radhakrishnan, Zakir
Freedom at Midnight Hussain, V. V. Giri
(b) Durga Das - India from Curzon to Nehru 109. Who had the longest tenure as the Viceroy
and After of India? [U.P.P.C.S. (Mains) 2009]
(c) K.K. Aziz - The Man Who Divided India (a) Lord Curzon
(d) Maulana Abul Kalam - India Wins (b) Lord Dufferin
Freedom (c) Lord Hardinge
(d) Lord Mayo
105. The Archaeological Survey of India was
established during the period of- 110. Which of the following is not correctly
[U.P.P.C.S. (Mains) 2010] matched? [U.P.P.C.S. (Mains) 2009]
(a) Lord Curzon (a) Unhappy India - Bal Gangadhar Tilak
(b) Lord Wellesley (b) Durgesh Nandini - Bankim Chandra
Chatterjee
(c) Lord William Bentinck
(c) India Wins Freedom - Abul Kalam Azad
(d) Warren Hastings (d) Poverty and Un-British Rule in India -
106. Match list-I with list-II and select the Dadabhai Naoroji
correct answer using the codes given below 111. The book “Mountbatten and the Partition
the lists: of India” was written by:
List-I (Authors) List-II (Works) [U.P.P.C.S. (Mains) 2009]
(a) Larry Collins and Dominique Lapierre
A. Bankim Chandra 1. Anand Math
(b) M.N. Das
Chatterjee (c) Louis Fischer
B. Michael 2. Captive Lady (d) None of the above
Madhusudan Dutt
112. Queen Victoria’s Proclamation was read out
C. Rabindranath 3. Gora by Lord Canning on 1st November 1858 at–
Tagore [U.P.P.C.S. (Mains) 2009]
D. Sarojini Naidu 4. The Broken (a) Allahabad
Wing (b) Calcutta
Code: [UPPCS (Pre) 2010] (c) Bombay
(d) Madras
A B C D
(a) 1 2 3 4 113. Who was the first person to call Mahatma
(b) 2 3 4 1 Gandhi, ‘Father of Nation’?
(c) 1 4 2 3 [U.P.P.C.S. (Pre) 2008]
(d) 4 1 3 2 (a) Jawaharlal Nehru
(b) Vallabhbhai Patel
107. In which of the following movements did (c) C. Raj Gopalachari
Mahatma Gandhi make the first use of (d) Subhash Chandra Bose
hunger strike as a weapon?
114. Who among the following was the founder
[U.P.P.C.S. (Pre) 2010]
of All India Muslim League?
(a) Non-Cooperation Movement [U.P.P.C.S. (Pre) 2007]
(b) Rowlatt Satyagraha
(a) Sir Saiyed Ahmad Khan
(c) Ahmedabad Strike (b) Sir Mohammad Iqbal
(d) Bardoli Satyagraha (c) Agha Khan
108. Which one of the following is the correct (d) Nawab Salimullah Khan
sequence of persons who occupied the 115. Who had opposed the Champaran
office of the President of India right from Satyagraha of Mahatma Gandhi?
the beginning? [U.P.P.C.S. (Pre) 2009] [U.P.P.C.S. (Mains) 2007]
(a) C. Rajagopalachari, Rajendra Prasad, (a) Rabindranath Tagore
Zakir Hussain, V. V. Giri (b) N.G Ranga
(b) Rajendra Prasad, S. Radhakrishnan, V. V. (c) Rajkumar Shukla
Giri, Zakir Hussain (d) Rajendra Prasad

269 PYQ Workbook


HISTORY OF MODERN INDIA

116. The Book ‘Mother India’ was written by: Code: [U.P.P.C.S. (Pre) 2005]
[U.P.P.C.S. (Mains) 2006] (a) 1, 2 and 3
(a) Catherine Mayo (b) 2, 3 and 4
(b) Lala Lajpat Rai (c) 1, 3 and 4
(c) Bal Gangadhar Tilak (d) 1, 2 and 4
(d) Bipin Chandra Pal 123. During the tenure of which Governor-
117. The policy of ‘Security cell’ is related with- General was the ‘Ancient Monuments
[U.P.P.C.S. (Pre) 2006] Preservation Act’ passed-
(a) Warren Hastings [U.P. P.C.S. (Mains) 2005]
(b) Lord Dalhousie (a) Lord Minto
(c) Henry Lawrence (b) Lord Linlithgow
(d) Lord Hastings (c) Lord Curzon
(d) Lord Canning
118. “Everything else can wait, but not
agriculture.” The above statement is 124. Who among the following said that “the
attributed to: [U.P.P.C.S. (Mains) 2006] most notable achievement of the British
(a) Jagjivan Ram Rule was the unification of India”?
(b) Jawahar Lal Nehru [U.P.P.C.S. (Spl) (Mains) 2004]
(c) Mahatma Gandhi (a) K.M. Panikkar
(d) Sardar Patel (b) M.N. Srinivas
119. Which one of the following was favored by (c) Rajni Kothari
Nehru but not favored by Gandhi? (d) Yogendra Singh
[U.P.P.C.S. (Pre) 2006] 125. Which one of the following sessions of
(a) Truth Indian National Congress was for the first
(b) Non-violence time attended by M.K. Gandhi:
(c) Untouchability [U.P.P.C.S. (Spl) (Mains) 2004]
(d) Heavy industrialization (a) Lucknow Session, 1916
120. Which one of the following books is (b) Calcutta Session, 1901
associated with the rise of the national (c) Amritsar Session, 1919
movement in India? (d) Nagpur Session, 1920
[U.P.P.C.S. (Mains) 2005] 126. On the basis of alleged maladministration
(a) Gitanjali which Governor-General had taken the
(b) Anandmath administration of Mysore state?
(c) Satyarth Prakash [U.P. Lower Sub. (Pre) 2004]
(d) Gita Rahasya (a) Lord Wellesley
121. Which one of the following Satyagrahas was (b) Lord Hastings
not lead by Mahatma Gandhi? (c) Lord William Bentinck
[U.P.P.C.S. (Pre) 2005] (d) Lord Harding
(a) Quit India Movement 127. Who wrote the famous song ‘Saare Jahan Se
(b) Civil Disobedience Achcha Hindustan Hamara’?
(c) Bardoli [U.P.P.C.S. (Mains) 2004]
(d) Kheda (a) Sahir Ludhianvi
122. Which of the following statements regarding (b) Sir Sayyad Ahmed Khan
Dr. B.R. Ambedkar is correct? (c) Sir Muhammad Iqbal
1. He founded Siddharth College. (d) Bahadur Shah Zafar
2. He started his journal Mook Nayak in 128. Who said, “Political Freedom is the life-
1920. breath of a nation”?
3. He founded the Depressed Class Institute [U.P.P.C.S. (Mains) 2004]
in 1922. (a) Bal Gangadhar Tilak
4. He was the first Defense Minister of India. (b) Aurobindo Ghosh
Select the correct answer from the code given (c) Bipin Chandra Pal
below the statements: (d) Mahatma Gandhi

PYQ Workbook 270


HISTORY OF MODERN INDIA

129. Which one of the following Viceroy of India 135. Match list-I with list-II and select the
was murdered by a convict in Andaman and correct answer using the codes given below
Nicobar Island while he was on tour? the lists:
[U.P.P.C.S. (Pre) 2004] List-I (Party) List-II (Founder)
(a) Lord Curzon
(b) Lord Ripon A. Azad Muslim 1. Abdul Gaffar
(c) Lord Mayo Conference Khan
(d) Lord Minto B. Khaksar Party 2. Allah Baksh
130. Which one of the following pairs is correctly C. Khudai 3. Allama Mashriqui
matched? [U.P.P.C.S. (Spl) (Mains) 2004] Khidmatgar
(a) Abul Kalam Azad - Hind Swaraj D. Krishak Praja 4. Fazlul Haq
(b) Annie Besant - New India Party
(c) Bal Gangadhar Tilak - Commonweal 5. Sikandar Hayat
(d) Mahatma Gandhi - India Wins Freedom Khan
131. Who among the following was the chief Code: [UPPCS (Mains) 2002]
architect or reconciliation between the A B C D
Extremists and the Moderates? (a) 1 3 4 5
[U.P.P.C.S. (Pre) 2004] (b) 2 3 4 5
(a) Annie Besant (c) 2 3 1 4
(b) M.A. Jinnah (d) 3 2 1 5
(c) Madam Cama 136. Name of the magazine published by
(d) Feroz Shah Mehta Mahatma Gandhi during his stay in South
132. The right to adopt heir in place of own son Africa, was: [U.P P.C.S. (Pre) 2002]
was re-established by– (a) Navjeevan
(b) India Gazette
[U.P.P.C.S. (Spl) (Mains) 2004] (c) Africaner
(a) Government of India Act of 1858 (d) Indian Opinion
(b) Lord Canning
(c) Release of Charter in 1860 137. Match list-I with list-II and select the
correct answer using the codes given below
(d) Queen’s announcement of 1858 the lists:
133. The author of the book ‘Towards Struggle’
was: [U.P.P.CS. (Mains) 2003] List-I (Authors) List-II (Works)
(a) Jawaharlal Nehru A. Mahatma Gandhi 1. India Divided
(b) Ram Manohar Lohiya B. Ram Manohar 2. India Wins
(c) Jai Prakash Narayan Lohia Freedom
(d) J.B. Kriplani
C. Dr. Rajendra 3. Hind Swaraj
134. Match list-I with list-II and select the Prasad
correct answer using the codes given below D. Abul Kalam 4. The Wheel of
the lists: Azad History
List-I List-II Code: [UPPCS (Pre) 2001]
A. Amritlal Nagar 1. Devdas A B C D
(a) 1 3 4 2
B. Sumitra Nandan 2. Geet Govind (b) 4 3 2 1
Pant (c) 3 4 1 2
C. Sharat Chandra 3. Amrit aur Vish (d) 2 3 4 1
Chatterjee 138. The prefix ‘Mahatma’ was added with the
D. Jaidev 4. Chidambara name of Gandhi: [U.P.P.C.S.(Pre) 2001]
Code: [UPPCS (Mains) 2003] (a) During Champaran Satyagraha
(b) During the Satyagraha against Rowlatt
A B C D Act
(a) 3 4 1 2 (c) In the Amritsar Session of the Indian
(b) 4 3 1 2 National Congress, 1919
(c) 4 3 2 1 (d) At the beginning of the Khilafat
(d) 3 4 2 1 Movement

271 PYQ Workbook


HISTORY OF MODERN INDIA

139. Which one of the following is correctly (a) Abolishment of untouchability and
matched? [U.P.P.C.S. (Pre) 2000] temperance
(a) Ailan-i-Haq - Bipin Chandra Pal (b) Truth and non-violence
(b) Al-Helal - Dr. Zapir Hussain (c) Khadi and Spinning wheel
(c) Tahzib-ul-Akhlaq - Sir Syed Ahmad (d) Right means and right ends
(d) Yugantar - Aurobindo Ghosh 147. Who established the judicial organization
140. Who presided over the Benaras Session of in India? [U.P.P.C.S. (Pre) 1991]
the Indian National Congress in 1905? (a) Lord Mayo
[U.P.P.C.S. (Pre) 1999] (b) Lord Cornwallis
(c) Lord Attlee
(a) Surendranath Bannerji (d) Lord Curzon
(b) Feroz Shah Mehta
(c) Gopal Krishna Gokhale 148. The transfer of capital of British India from
(d) Dinshaw Wacha Calcutta to Delhi was affected during the
period of– [U.P.P.C.S. (Pre) 1990]
141. Those who joined Mahatma Gandhi during (a) Lord Mayo
the Champaran Satyagraha included: (b) Lord Lawrence
[U.P.P.C.S. (Pre) 1999] (c) Lord Minto
(a) Vallabh Bhai Patel and Vinoba Bhave (d) Lord Hardinge
(b) Jawaharlal Nehru and Rajendra Prasad 149. Which of the following pairs is correct?
(c) Rajendra Prasad and Anugraha Narain
[66th B.P.S.C. (Pre) 2020]
Sinha
(d) Mahadev Desai and Maniben Patel (a) Vinoba Bhave - Second Individual
Satyagrahi
142. Which three Indian States delayed accession (b) C.R. Das - Deshbandhu
to India even after other princely states had (c) William Wedderburn - Congress
joined India? [U.P.P.C.S. (Pre) 1998] President in 1907
(a) Junagarh, Mysore, Jammu and Kashmir (d) Shyamji Krishna Verma- Founder of
(b) Junagarh, Hyderabad, Jammu and India House in Paris
Kashmir (e) None of the above/More than one of the
(c) Udaipur, Kapurthala, Jammu and above
Kashmir 150. Which of the following was Gandhiji’s first
(d) Hyderabad, Udaipur, Travancore Satyagraha Movement in India in which he
used Civil Disobedience?
143. Which one of the following native States
was a party to the ‘Stand-Still’ Agreement: [66th B.P.S.C. (Pre) 2020]
[U.P.P.C.S. (Pre) 1997] (a) Champaran
(b) Kheda
(a) Hyderabad (c) Ahmedabad
(b) Jammu and Kashmir (d) Rowlatt Satyagraha
(c) Junagarh (e) None of the above/More than one of the
(d) Mysore above
144. Who among the following is associated with 151. The Sarabandi Campaign of 1922 was led
the policy of ‘Masterly Inactivity’: by: [65th B.P.S.C. (Pre) 2019]
[U.P.P.C.S. (Pre) 1997] (a) Chittaranjan Das
(a) William Bentinck (b) Sardar Vallabhbhai Patel
(b) Lord Canning (c) Rajendra Prasad
(c) Lord Mayo (d) Lala Lajpat Rai
(d) John Lawrence (e) None of the above/More than one of the
above
145. The author of ‘Malgudi Days’ is:
[U.P.P.C.S. (Pre) 1993] 152. Who wrote the book, Desher Katha?
(a) K. Abbas [65th B.P.S.C. (Pre) 2019]
(b) R.K. Narayan (a) Sakharam Ganesh Deuskar
(c) Laxman Seth (b) Rajendra Prasad
(d) Mulkraj Anand (c) Nivaran Chandra
(d) Murali Mohan Prasad
146. The twin principles of Mahatma Gandhi’s (e) None of the above/More than one of the
Ram Rajya were? [U.P P.C.S. (Pre) 1993] above

PYQ Workbook 272


HISTORY OF MODERN INDIA

153. Whom did Rajendra Prasad consider as the (d) H.N. Brailsford
father of Pakistan? (e) None of the above/More than one of the
[63rd B.P.S.C (Pre.) 2017] above
(a) Md. Jinnah 159. What was the common relationship between
(b) Lord Mountbatten Katherine Mayo, Aldous Huxley, Charles
(c) Lord Minto Andrews and William Digby?
(d) Liaquat Ali Khan
(e) None of the above/More than one of the [56th to 59th B.P.S.C. (Pre) 2015]
above (a) They wrote commentaries on the
condition of India during the British rule
154. Who established the Imperial Cadet Corps? (b) They were supporters of the Indian
[63rd B.P.S.C (Pre.) 2017] National Movement
(a) Lord Minto (c) They were opponents of the Indian
(b) Lord Curzon National Movement
(c) Lord Lytton (d) They were friends of Mahatma Gandhi
(d) Lord Ripon
(e) None of the above/More than one of the 160. Who raised the demand of ‘Complete
above Independence’ for the first time in 1921?
[56th to 59th B.P.S.C. (Pre) 2015]
155. Which of the following Princely States was
(a) Maulana Muhammad Ali
not annexed by the British?
(b) Pandit Jawaharlal Nehru
[63rd B.P.S.C (Pre.) 2017] (c) Mahatma Gandhi
(a) Sind (d) Maulana Hasrat Mohani
(b) Gwalior
(c) Awadh 161. ‘New Lamps for Old’ was a series of articles
(d) Satara (1893-94) that criticized the Congress for
(e) None of the above/More than one of the being out of touch with the ‘proletariat’.
above Who was the author of these articles?
156. “In my belief, Congress is to tottering to its [56th to 59th B.P.S.C. (Pre) 2015]
fall and one of my great ambitions while in (a) Aurobindo Ghose
India is to assist it to a peaceful demise.” (b) A. O. Hume
Who wrote it? (c) G.K. Gokhale
[60th to 62nd B.P.S.C. (Pre) 2016] (d) B.G. Tilak
(a) Lord Lytton 162. Who wrote the book ‘India for Indians’?
(b) Lord Dufferin [65th B.P.S.C. (Pre) 2011]
(c) Lord Curzon (a) C. R. Das
(d) Lord Minto (b) M.G. Ranade
(e) None of the above/More than one of the
(c) V. D. Savarkar
above
(d) S.N. Banerjee
157. Which among the following viceroy’s (e) None of the above/More than one of the
period the title of ‘Rai Bahadur’ and ‘Khan above
Bahadur’ began to confer to Indian?
163. The first railway in India was laid down
[60th to 62nd B.P.S.C. (Pre) 2016]
during the period of-
(a) Lord Ripon
(b) Lord Lytton [48th to 52nd B.P.S.C. (Pre) 2008]
(c) Lord Mayo (a) Lord Dalhousie
(d) Lord Dufferin (b) Lord Curzon
(e) None of the above/More than one of the (c) Lord Wellesley
above (d) Lord Lytton
158. “The Muslims, if contented and satisfied, 164. Name the Deoband scholar who played a
would become the greatest bulwark of significant role in the freedom movement.
British power in India.” Who wrote it? [47th B.P.S.C. (Pre) 2005]
[60th to 62nd B.P.S.C. (Pre) 2016] (a) Abul Kalam Azad
(a) Herbert Risley (b) Mohammad Ali Jinnah
(b) Lord Lytton (c) Badruddin Tyabji
(c) W.W. Hunter (d) Chirag Ali

273 PYQ Workbook


HISTORY OF MODERN INDIA

165. Who among the following capitalists served 172. Who was the secretary of Mahatma Gandhi
as an AICC treasurer for many years and during Noakhali? [41st B.P.S.C. (Pre) 1996]
went to jail in 1930? (a) Nirmal Kumar Bose
[47th B.P.S.C. (Pre) 2005] (b) Mahadeo Desai
(a) G. D. Birla (c) Pyare Lal
(b) Jamnalal Bajaj (d) Ballabh Bhai Patel
(c) J. R. D. Tata 173. Who among the following abolished ‘Dual
(d) Balchand Hirachand Government’ system in Bengal?
166. Mahatma Gandhi’s close English compatriot [41st B.P.S.C. (Pre) 1996]
during the freedom movement was: (a) Robert Clive
[46th B.P.S.C. (Pre) 2004] (b) Lord Cornwallis
(a) Thomas Moore (c) Warren Hastings
(b) A. O. Hume (d) None of the above
(c) Charles Andrews 174. Who gives the Slogan ‘Jai Jawan, Jai Kisan’?
(d) William Wavell [38th B.P.S.C. (Pre) 1992]
167. Who is known as “Loknayak”? (a) Lal Bahadur Sashtri
[46th B.P.S.C. (Pre) 2004] (b) Subhash Chandra Bose
(a) Mahatma Gandhi (c) Devi Lal
(b) Subhash Chandra Bose (d) Morariji Desai
(c) Jayprakash Narayan 175. Which one of the following statements
(d) Bal Gangadhar Tilak about Princely States in India is not correct?
168. The purpose of the Butler Committee, 1927: [CAPF 2022]
[46th B.P.C.S. (Pre) 2004] (a) The British Government declared that
(a) Modernization of Indian Army States were free to join either India or
Pakistan or remain independent.
(b) Innovative modernization of Indian
(b) The decision to join either India or
Agriculture
Pakistan or remain independent was left
(c) Implementation of Sensor over National
not to the people of Princely States but to
Newspapers
the Princely Rulers.
(d) Improvement of relationships between
(c) The Ruler of Travancore first decided to
Indian Provinces and the Crown remain independent.
169. The Muslim deputation met Minto in 1906 (d) The State of Travancore finally joined
at Shimla and pleaded for– India through a plebiscite.
[46th B.P.S.C. (Pre) 2004] 176. Which of the following pairs of years and
(a) Separate electorate for Muslims historical events is correctly matched?
(b) A composite electorate [CAPF 2022]
(c) Higher representation to the Hindus (a) 1878 – Second Anglo-Afghan War
(d) Special representation to Muslim by (b) 1885 – Ilbert Bill Controversy
nomination (c) 1785 – Fox’s India Bill
170. Bihar became a separate state during the (d) 1898 – Lord Curzon becomes the Viceroy
British rule in the year: 177. When was the first All India Census
[44th B.P.S.C. (Pre) 2000] attempted? [CAPF 2022]
(a) 1905 (a) 1871
(b) 1912 (b) 1872
(c) 1936 (c) 1881
(d) 1946 (d) 1861
171. Which of the following leaders presided 178. Initially, what was the purpose of setting
over the Congress Session at Calcutta in up of the hill stations during the colonial
1906? [44th B.P.S.C. (Pre) 2000] period in India? [CAPF 2021]
(a) B.G. Tilak (a) To serve as winter capitals for the British
(b) G.K. Gokhale (b) To serve the needs of the British army
(c) Aurobindo Ghosh (c) To source timber from forests
(d) Dadabhai Naoroji (d) To set up sanatoria

PYQ Workbook 274


HISTORY OF MODERN INDIA

179. Which among the following statements Select the correct answer using the code given
about Mahatma Gandhi’s Satyagraha is/are below:
correct? [CAPF 2019] (a) 1 and 3
1. It was during the course of his campaign (b) 2 and 3
against racialism in South Africa that (c) Only 2
Gandhiji first applied Satyagraha. (d) Only 3
2. The two vital ingredients of Satyagraha 183. Which of the following statements about
are ‘truth’ and ‘non-violence’. Ishwarchandra Vidyasagar is/are not
3. The Satyagrahi resists evil by inflicting correct?
suffering on himself and not by inflicting 1. Vidyasagar’s writings significantly helped
suffering on the opponent. to develop the Bengali language.
4. In India, Satyagraha was first tried by 2. He received the title ‘Vidyasagar’ from
Gandhiji in Champaran. Presidency Sanskrit College, Calcutta.
Select the correct answer using the code given 3. He was enthusiastic about the spread of
below: women’s education.
(a) 1 and 4 4. Vidyasagar defended widow marriage
(b) Only 2 without being convinced about scriptural
(c) Only 4 support.
(d) All of these Select the correct answer using the code given
180. Which one of the following pairs of below: [CAPF 2019]
organization and their founders is not (a) 2 and 4
correctly matched? [CAPF 2019] (b) 1 and 2
(c) Only 4
(a) National Tej Bahadur Sapru (d) Only 3
Liberation and MR Jayakar
Federation 184. Which of the following statements about
India’s Independence is/are correct?
(b) Jamiat-ul Maulana Mehmood
Ulama-i Hasan Shaikh-ul- [CAPF 2018]
Hind 1. The formal transfer of power on
15th August 1947 heralding India’s
(c) Congress Bal Gangadhar Tilak Independence was announced by Lord
Democratic Mountbatten.
Party
2. Mahatma Gandhi was not present at the
(d) Congress MN Roy festivities in the capital on 15th August
Socialist Party 1947.
181. Why was Vasudeo Balwant Phadke known Select the correct answer using the codes
in history? [CAPF 2019] given below.
(a) He led a violent struggle against the (a) 1 only
British during the Revolt of 1857. (b) 2 only
(b) He was a critic of Gandhian struggle. (c) Both 1 and 2
(c) He led an armed uprising against the (d) Neither 1 nor 2
British in the 1870s. 185. Who among the following in Congress
(d) He was a radical leader of the Indian represented the socialist view? [CAPF 2018]
National Congress. (a) Rajendra Prasad
182. Which of the following statements about (b) Narendra Dev
Bhagat Singh is/are not true? [CAPF 2019] (c) Maulana Azad
1. Bhagat Singh was influenced by socialist (d) Sardar Patel
ideas 186. Who among the following has not written
2. He was a member’ of the Hindustan about partition in his works? [CAPF 2018]
Socialist Republican Association (a) Saadat Hasan Manto
3. Bhagat Singh threw a bomb in the Central (b) Bankim Chandra Chattopadhyay
Legislative Assembly in 1929 with the (c) Bhisham Sahni
objective to kill as many people as possible (d) Narendranath Mitra

275 PYQ Workbook


HISTORY OF MODERN INDIA

187. Examine these two statements carefully and (c) Bahishti Zewar
select the correct answer using the code (d) City of Djinns
given below.
192. Identify from the following the novel
1. Statement I Nationalism in India, which written by Charles Dickens that critiqued
was assigned a privileged position by its the adverse impact of industrialisation on
western educated political leadership, the poor. [CAPF 2016]
was ‘different’, but a ‘derivative discourse’
from the West. (a) The Day Before America
(b) Hard Times
2. Statement II Indian nationalism as a
(c) Creating a Nation
response to western imperialism was ‘like
(d) The Courtier
all such responses, shaped by what it was
responding to’. 193. Bal Gangadhar Tilak was associated with
Code: [CAPF 2017] [CAPF 2015]
(a) Both the statements are individually true 1. Poona Sarvajanik Sabha
and statement II is the correct explanation 2. The Age of Consent Bill
of statement I. 3. Gaurakshini Sabha
(b) Both the statements are individually 4. Atmiya Sabha
true, but statement II is not the correct
explanation of statement I. Select the correct answer using the code
(c) Statement I is true, but statement II is given below.
false. (a) 1 and 2 only
(d) Statement I is false, but statements II is (b) 1, 2 and 4
true. (c) 3 and 4 only
(d) 2 and 4 only
188. Who among the following British
ethnographers launched the Survey of India 194. Where and when did Mahatma Gandhi first
project in the 1860s? [CAPF 2016] use his method of hunger-strike to achieve
(a) Hunter a desired goal? [CAPF 2015]
(b) Dufferin (a) At Natal in South Africa in the year 1906
(c) Risley (b) At Champaran in the year 1917
(d) Thurston (c) At Ahmedabad in the year 1918
(d) At Cape Town in South Africa in the year
189. Which of the following statements about
1906
‘Barnaparichay’, a mid 19th century Bengali
text is/are correct? [CAPF 2016] 195. Which of the following were the main
1. It was written by Raja Rammohan Roy. influences on Vasudeo Balvant Phadke?
2. It was the most widely used Bengali [CAPF 2015]
primer of the time. 1. Experience of the Deccan famine of 1876-
Select correct answer using the code given 77
below. 2. Hindu revivalism
(a) Only 1 3. Theory of Drain of Wealth
(b) Only 2 4. Reformist idea of Phule
(c) Both 1 and 2 Select the correct answer using the code
(d) Neither 1 nor 2 given below.
190. Which one among the following is the oldest (a) 1, 2 and 3
Regiment of Indian Army? [CAPF 2016] (b) 2, 3 and 4
(a) Sikh Regiment (c) 1 and 3 only
(b) Kumaon Regiment (d) 2 and 4 only
(c) Punjab Regiment 196. Who among the following was NOT a
(d) Madras Regiment Communist leader in colonial India?
191. Which one of the following is the famous [CAPF 2015]
novel written in 1869 by Nazir Ahmad? (a) PC Roy
[CAPF 2016] (b) SA Dange
(a) Twilight in Delhi (c) Muzaffar Ahmad
(b) Mirat-ul-urus (d) Singaravelu

PYQ Workbook 276


HISTORY OF MODERN INDIA

197. Match List I with List II and select the 201. Between the 15th of August 1947 and the
correct answer using the code given below 26th of January 1950 when India became free
the lists. and when she declared herself a democratic
sovereign republic, Government of India
List I (Person) List II (Political functioned under the provisions of :
Party)
[CAPF 2014]
A. BR Ambedkar Pakistan Muslim (a) The Government of India Act, 1935
League (b) The Indian Independence Act, 1947
B. Sikander Hayat Krishak Praja Party (c) The Indian (Provisional Constitution)
Khan Order, 1947
C. Fazlul Huq Unionist Party (d) None of the above
D. Khaliquzzaman Independent Labour 202. Which member of the Servants of India
Party Society founded the Seva Samiti at
Code: [CAPF 2015] Allahabad in 1914? [CDS 2023 (I)]
A B C D (a) Shri Ram Bajpai
(a) 4 2 3 1 (b) Hriday Nath Kunzru
(b) 4 3 2 1 (c) S. G. Vaze
(c) 1 2 3 4 (d) Srinivas Shastri
(d) 1 3 2 4 203. Who among the following was not a woman
labour leader who organised workers in the
198. The Assam Company, consisting of strikes in the 1920s? [CDS 2022 (II)]
European tea planters, remained the virtual (a) Sarala Devi Chaudhurani
proprietor of the tea industry till 1850. (b) Ushabai Dange
In the 1850s who among the following (c) Prabhabati Devi
established two small proprietary tea (d) Anasuya Behn
gardens of his own, which were confiscated
by the state after his execution for treason 204. Who among the following founded the Bhil
Seva Mandal in 1922 [CDS 2021 (I)]
in 1858? [CAPF 2014]
(a) Dayaram Gidumal
(a) Anandaram Dhekial Phukan (b) Gurusaday Dutt
(b) Gunabhiram Barua (c) Dhondo Keshav Karve
(c) Lakhinath Bezbarua (d) Amritlal Vithaldas Thakkar
(d) Maniram Dewan
205. Which one of the following statements
199. Dinabandhu Mitra published a play in about the British Indian Medical Service
Bengali titled Neel Darpan (Blue Mirror), (IMS) is not correct? [CDS 2021 (I)]
which depicted the atrocities on the indigo (a) IMS began in 1764.
planters. The play was translated into (b) It recruited health professionals by means
English by [CAPF 2014] of a competitive examination.
(a) Reverend James Long (c) Indians were never admitted to IMS.
(b) Dwarkanath Tagore (d) The IMS was at first meant to look after
the troops.
(c) Michael Madhusudan Dutta
(d) Rabindranath Tagore 206. Who among the following was one of the
founders of the Indian Society of Oriental
200. William James, Henry Colebrooke and Art? [CDS 2020 (I)]
Nathaniel Halhed had which of the (a) Rabindranath Tagore
following common among them? (b) Abanindranath Tagore
[CAPF 2014] (c) Dwarkanath Tagore
(a) They were critics of Indian culture and (d) Bankim Chandra Chattopadhyaya
civilization 207. Who believed that the Russian designs
(b) They were judges at the courts of the East were ‘an imminent peril to the security and
Indian Company tranquility’ of the Indian Empire in 1836?
(c) They were professors teaching history [CDS 2020 (I)]
and society of South Asia (a) Lord Auckland
(d) They were linguists who tried to interpret (b) Lord Palmerston
the culture of South Asia to the East India (c) Lord Canning
Company (d) Alexander Burnes

277 PYQ Workbook


HISTORY OF MODERN INDIA

208. The creation of a Federal Court in India was (c) Theodore Beck
advocated by which of the following Acts/ (d) Sir Syed Ahmed Khan
Commissions? [CDS 2020 (I)]
215. After the First World War, the Triveni Sangh
(a) The Government of India Act, 1919 was formed by [CDS 2020 (II)]
(b) The Lee Commission, 1923
(a) the Jats and Gujjars
(c) The Government of India Act, 1935
(d) The Indian Councils Act, 1909 (b) the Rajputs and Yadavs
(c) the Jats and Yadavs
209. Who among the following gave evidence (d) the Ahirs and Kurmis
before the Joint Select Committee on the
Government of India Bill, 1919 in favour of 216. Who among the following was the first to
female franchise? [CDS 2020 (II)] accept a ministerial position in the Central
Provinces in October 1925? [CDS 2020 (II)]
1. Mrs. Annie Besant
(a) BS Moonje
2. Mrs. Sarojini Naidu
(b) MR Jayakar
3. Mrs. Hirabai Tata (c) SB Tambe
Code: (d) BN Sasmal
(a) Only 1
(b) 1 and 2 217. Who among the following formed the
(c) 2 and 3 National Liberation Federation (Liberal
(d) 1, 2 and 3 Party)? [CDS 2020 (II)]
(a) Motilal Nehru and CR Das
210. With whom did Subhas Chandra Bose (b) Muhammad Ali and CR Das
form an alliance to destroy the Holwell (c) TB Sapru and MR Jaykar
Monument in Culcutta during 1939-40? (d) MR Jayakar and CR Das
[CDS 2020 (II)]
(a) The Communist Party of India 218. Who among the following wrote The
(b) The Muslim League Philosophy of the Bomb? [CDS 2020 (II)]
(c) The Hindu Mahasabha (a) Sukhdev
(d) The Unionist Party (b) Chandrashekhar Azad
(c) Bhagwati Charan Vohra
211. Who among the following created the first (d) Bhagal Singh
All India Trade Union [CDS 2020 (II)]
(a) BP Wadia 219. At which one of the following Sessions
(b) SA Dange of the Indian National Congress was the
(c) NM Joshi resolution on Fundamental Rights and
(d) BT Ranadive Economic Policy passed? [CDS 2020 (II)]
(a) Tripuri Session
212. Which one among the following was India’s (b) Lahore Session
first trade union in the proper sense of the (c) Lucknow Session
term? [CDS 2020 (II)] (d) Karachi Session
(a) Bombay Labour Union
(b) Ahmedabad Labour Union 220. What was the Dutt-Bradley Thesis?
(c) Madras Labour Union [CDS 2019 (II)]
(d) Allahabad Labour Union (a) The Working Committee of the Indian
213. During the 19th century, who among the National Congress decided that Congress
following wrote Satapatra Series? should play a crucial role in realising the
[CDS 2020 (II)] independence of India.
(b) The Socialist Party decided to play
(a) MG Ranade
foremost part in anti-imperialist struggle.
(b) BG Tilak
(c) Revolutionary socialist Batukeshwar Dutt
(c) Bankim Chandra Chatterjee
put forth a ten-point plan to work for the
(d) GH Deshmukh
success of anti-imperialist front.
214. Who among the following founded the (d) It was a Communist Party document,
Mohammedan Anglo-Oriental Defense according to which the National Congress
Association (1893)? [CDS 2020 (II)] could play a great part and a foremost part
(a) Auckland Colvin in realising the anti-imperialist people’s
(b) Badruddin Tyabji front.

PYQ Workbook 278


HISTORY OF MODERN INDIA

221. The Khuntkatti tenure was prevalent in A B C D


which one of the following regions of India (a) 2 4 1 3
during the British Colonial Rule? (b) 2 1 4 3
[CDS 2019 (II)] (c) 3 1 4 2
(d) 3 4 1 2
(a) Bundelkhand
(b) Karnataka 226. Subhas Chandra Bose started the ‘Azad
(c) Chota Nagpur Hind Radio’ in which of the following
countries? [CDS 2018 (II)]
(d) Madras Presidency
(a) Japan
222. Which European ruler had observed, “Bear (b) Austria
in mind that the commerce of India is (c) Germany
the commerce of the world....he who can (d) Malaysia
exclusively command it is the dictator of 227. Who took over the ‘Eka Movement’ started
Europe”? [CDS 2019 (I)] by the Congress in Awadh during 1921-
(a) Queen Victoria 1922? [CDS 2018 (II)]
(b) Peter the Great of Russia (a) Bhagwan Ahir
(c) Napoleon Bonaparte (b) Madari Pasi
(c) Baba Ramchandra
(d) Gustav Adolf
(d) Shah Naeem Ata
223. In 1921, during which one of the following 228. Which organization was started at the
tours, Gandhiji shaved his head and began Haridwar Kumbh Mela in 1915?
wearing loincloth in order to identify with [CDS 2018 (II)]
the poor? [CDS 2019 (I)] (a) Sanatan Dharma Sabha
(a) Ahmedabad (b) Dev Samaj
(b) Champaran (c) Brahmin Sabha
(c) Chauri Chaura (d) Hindu Mahasabha
(d) South India 229. Which of the following statements about
Sir Syed Ahmed Khan is/are correct?
224. Simla was founded as a hill station to use
[CDS 2018 (I)]
as strategic place for billeting troops,
1. He argued that India was a federation of
guarding frontier and launching campaign ethnic communities based on common
during the course of [CDS 2019 (I)] descent.
(a) Anglo-Maralha War 2. His philosophy was very similar to that of
(b) Anglo-Burmese War the Indian National Congress.
(c) Anglo-Gurkha War 3. He imagined India as a Nation State based
(d) Anglo-Afghan War on individual citizen’s rights.
4. The curriculum at the Mohammedan
225. Match List I with List II and select the Anglo-Oriental College blended Muslim
correct answer using the codes given below. theology and European empiricism.
List I (Author) List II (Book) Select the correct answer using the codes
A. Sekhar 1. Jawaharlal Nehru : given below.
Bandyopadhyay A Biography, Vol-1, (a) Only 1
1889-1947 (b) 2 and 3
(c) Only 3
B. Sarvepalli 2. From Plassey to (d) 1 and 4
Gopal Partition: A History
of Modern India 230. Consider the following statement: “A sound
body means one which bends itself to the
C. David 3. The Ascendancy spirit and is always a ready instrument at
Hardiman of the Congress in its service.” [CDS 2018 (I)]
Uttar Pradesh, 1926-
1934 The above statement is attributed to
(a) Sardar Patel
D. Gyanendra 4. Gandhi in His Time (b) Winston Churchill
Pandey and Ours (c) Mahatma Gandhi
Codes [CDS 2019 (I)] (d) Baden-Powell

279 PYQ Workbook


HISTORY OF MODERN INDIA

231. Who among the following was the founder 235. Which of the following statements with
of Phoenix Settlement? [CDS 2018 (I)] regard to the speech of Mahatma Gandhi
(a) Mahatma Gandhi at the opening of the Banaras Hindu
(b) BR Ambedkar University are correct? [CDS 2017 (I)]
(c) Rabindranath Tagore 1. He charged the Indian elite with a lack of
(d) Swami Vivekananda concern for the labouring poor.
2. He asserted that our salvation can come
232. Which of the following was/were founded only through the farmers.
by Raja Ram Mohan Roy? [CDS 2017 (II)] 3. He highlighted the plight of the
1. Atmiya Sabha untouchables.
2. Brahmo Samaj 4. He promised to take up the cause of the
3. Prarthana Samaj mill owners of Ahmedabad.
4. Arya Samaj Select the correct answer using the code
given below
Select the correct answer using the codes (a) 1, 2, 3 and 4
given below (b) 1 and 2 only
(a) 1, 2 and 3 (c) 1, 2 and 3 only
(b) 2 only (d) 3 and 4 only
(c) 1 and 2 only 236. Match the following. [CDS 2016 (II)]
(d) 1, 3 and 4
List I (Historians) List II
233. Consider the following statements about
the different meanings of ‘Swaraj’ as (Books)
articulated by Mahatma Gandhi: A. Sumit Sarkar 1. The Rise and
[CDS 2017 (I)] Growth of
1. Swaraj is intimately linked with Economic
Ahimsa (non-violence) and Satyagraha Nationalism in
(adherence to truth). India.
2. Swaraj has two senses-one political and B. Shahid Amin 2. A Rule of
one beyond the realm of politics. Property for
3. Swaraj is something that requires time Bengal
and patience to acquire. C. Ranajit Guha 3. The Swadeshi
4. With determination, Swaraj could be Movement in
obtained easily and quickly. -- Bengal, 1903-
Which of the statement(s) given above is/are 1908
correct? D. Bipan Chandra 4. Event, Metaphor,
(a) 1 only Memory-Chauri
(b) 1 and 2 only Chaura, 1922-
(c) 3 and 4 1992.
(d) 1, 2 and 3 Codes
234. What was/were the formative influence(s) A B C D
on the philosophy of Mahatma Gandhi? (a) 3 4 2 1
1. Gandhiji was influenced by the 18th (b) 3 2 4 1
Century Pranami sect that advocated the (c) 1 2 4 3
(d) 1 4 2 3
unity of faiths.
2. Gandhiji was influenced by the 237. Which of the following statement(s)
theosophists. about Mahatma Gandhi’s South African
experiences (1893-1914) is/are true?
3. Gandhiji was an admirer of the writings
[CDS 2015 (I)]
of Romantics like Wordsworth
1. Muslim merchants were actively involved
Select the correct answer using the codes in Gandhian political movements in
given below. [CDS 2017 (I)] South Africa.
(a) 1 and 2 2. In 1906, Gandhi led a campaign in
(b) 2 and 3 Cape Town against the ordinance on
(c) 1, 2 and 3 compulsory registration and passes for
(d) 1 only Indians.

PYQ Workbook 280


HISTORY OF MODERN INDIA

3. Gandhi began his political career with 242. Consider the following statements and
struggles against the imposition of choose the correct option:
excessive taxes on Indians in Cape Town. [CGPSC (Pre) 2020]
Select the correct answer using the codes 1. J.B. Kripalani formed the Kisan Mazdoor
given below: Praja Party in July, 1952.
(a) Only 1 2. P.C. Ghosh and T. Prakasam were
(b) Only 3 associated with the Kisan Mazdoor Praja
Party.
(c) 1 and 2
Code:
(d) All of the above
(a) 1 is true, but 2 is false
238. Which of the following statement(s) is/are (b) 1 is false, but 2 is true
correct? [CDS 2015 (I)] (c) 1 and 2 both are true
1. The Marxist theory repudiates the idea of (d) 1 and 2 both are false
the atomised and alienated individual. 243. Which revolutionary made this statement:
2. Marxism upholds the idea of the natural “Avenge the blood of Indian Martyrs. Use
rights of men and individuals. your sword to liberate your motherland.
Select the correct answer using the codes Stand up against the entire Anglo-American
given below: enemy.” [Chhattisgarh P.C.S. (Pre) 2019]
(a) Ras Behari Bose
(a) Only 1 (b) Subhash Chandra Bose
(b) Only 2 (c) Captain Mohan Singh
(c) Both 1 and 2 (d) Khudiram Bose
(d) Neither 1 nor 2
244. The revolt of Vellore occur during the
239. Which of the following was/were connected regime of which Governor?
primarily to the communist ideology? [Chhattisgarh P.C.S. (Pre) 2016]
1. Kirti Kisan Party (a) Wellesley
2. Labour Swaraj Party (b) Lord Minto
Select the correct answer using the codes (c) Lord Cornwallis
(d) Sir George Barlow
given below: [CDS 2015 (I)]
(e) None of these
(a) Only 1
(b) Only 2 245. The famous book “Foundation of Indian
(c) Both 1 and 2 Culture” is authored by:
(d) Neither 1 nor 2 [Chhattisgarh P.C.S. (Pre) 2015]
(a) Rabindranath Tagore
240. Which of the following statements about (b) Shri Aurobindo
the social reformer, Raja Ram Mohan Roy, (c) Radhakrishnan
is false? [CDS 2015 (I)] (d) Bhagwan Das
(a) Ram Mohan Roy belonged to the gentry (e) None of these
class whose power has been diminished 246. Who was the first President of the Muslim
because of the imposition of the League: [Chhattisgarh P.C.S. (Pre) 2015]
Permanent Settlement (a) Agha Khan
(b) He studied both Vedantic Monism and (b) Hamid Khan
Christian Unitarianism (c) Hasan Khan
(c) He translated the Upanishads into Bengali (d) M.A. Jinnah
(d) His first organisation was the Atmiya
247. Matching the following pairs (in the context
Sabha, founded in Calcutta in 1815 of Gandhi):
241. The social ideals of Mahatma Gandhi were List-I List-II
first put forth in [CDS 2014 (I)]
A. Gandhiji took to 1. For endorsing
(a) Hind Swaraj
Yerwada prison Delhi Pact
(b) An Authobiography-The Story of My
Experiments with Truth B. He undertook 2. The civil
(c) History of the Satyagraha in South Africa fast unto death disobedience
(d) The Bhagavad Geeta According to Gandhi movement

281 PYQ Workbook


HISTORY OF MODERN INDIA

C. He was 3. Congress crisis of 253. The export of slaves from Bengal was
confronted with 1939 banned in which year?
black flags on [Jharkhand P.C.S. (Pre) 2013]
way to Karachi (a) 1764
D. He called the 4. Against (b) 1789
defeat more Communal Award (c) 1858
mine than theirs (d) 1868
Code: [Chhattisgarh PCS (Pre) 2013] 254. Public Works Department was organized in
A B C D 1845-1855 by–
(a) 2 4 1 3 [Jharkhand P.C.S. (Pre) 2013]
(b) 1 2 3 4 (a) Lord Dalhousie
(c) 1 4 2 3 (b) Lord Cornwallis
(d) 2 4 3 1 (c) George Auckland
(e) 3 4 1 2 (d) Warren Hastings
248. Which company started the First Railway 255. Who was the first Indian selected as a
Service in India? member of the British Parliament?
[Chhattisgarh P.C.S. (Pre) 2011] [Jharkhand P.C.S. (Pre) 2013]
(a) Eastern Railway (a) R.C. Dutt
(b) Great Indian Peninsula Railway
(b) Dadabhai Naoroji
(c) Madras Railway
(c) Rammohan Roy
(d) Awadh-Tirhut Railway
(d) Meghnad Desai
249. The book “Guilty Man of India’s Partition”
256. January 26,1950 was chosen for the
was authored by:
enforcement of the Constitution because:
[Chhattisgarh P.C.S. (Pre) 2003]
[Jharkhand P.C.S. (Pre) 2011]
(a) Jawaharlal Nehru
(b) Dr. Ram Manohar Lohia (a) This was an auspicious day.
(c) Maulana Abul Kalam Azad (b) ‘Quit India Movement’ was begun on this
(d) Sarojini Naidu date in 1942.
(c) Congress had celebrated 26 January 1930
250. Mahatma Gandhi set up an Ashram on the as the first Independence Day.
banks of Sabarmati near Ahmedabad is (d) None of the above.
known as:
[Chhattisgarh P.C.S. (Pre) 2003] 257. Who had proposed “Party Less Democracy’
(a) Sabarmati Ashram in India? [M.P.P.C.S. (Pre) 2019]
(b) Harijan Ashram (a) S. A. Dange
(c) Satyagraha Ashram (b) Ram Manohar Lohiya
(d) Swaraj Ashram (c) Mahatma Gandhi
(d) Jay Prakash Narayan
251. The British Monarch at the time of Indian
Independence was- 258. Who authored the book “Planned Economy
[Jharkhand P.C.S. (Pre) 2016] for India”? [M.P.P.C.S. (Pre) 2018]
(a) George V (a) M. Visvesvaraya
(b) George VI (b) J.R.D. Tata
(c) King Edward VII (c) G.D. Birla
(d) None of these (d) Pattabhi Sitaramayya
252. ‘Hill Assembly Plan’ was set up for the 259. Who was the last Governor-General of
development of Adivasis by- Independent India?
[Jharkhand P.C.S. (Pre) 2016] [M.P.P.C.S. (Pre) 2010]
(a) T. Wilkinson (a) C. Rajagopalachari
(b) Cleveland (b) Rajendra Prasad
(c) Lord William Bentinck (c) Lord Mountbatten
(d) Lord Macaulay (d) Lord Canning

PYQ Workbook 282


HISTORY OF MODERN INDIA

260. Which of the following assumed sovereign 266. What idea is given by Gandhiji for family
power at midnight of 14/15 August, 1947 planning? [M.P.P.C.S (Pre) 1990]
provisionally? [M.P.P.C.S. (Pre) 2010] (a) Self-control
(a) Central Legislative Assembly (b) Sterilization
(b) Constituent Assembly (c) Restrain
(c) Interim Government (d) Loop
(d) Chamber of Princes 267. Who wrote the book ‘Prison Diary’?
261. Which part of India remained under [M.P.P.C.S. (Pre) 1990]
Portuguese control after 15th August 1947? (a) Jai Prakash Narayan
(b) Munshi Premchand
[M.P.P.C.S. (Pre) 2010]
(c) Morarji Desai
(a) Goa (d) Atal Bihari Vajpayee
(b) Sikkim
(c) Pondicherry 268. Name the revolutionary woman, who
(d) Andaman & Nicobar participated in the Peasant Movement
of Bijoliya and was arrested. She also
262. Which of the following Governor-General participated in the 1930 Satyragrah and
used the system of Separate electoral college 1932 Civil Disobedience Movement and
to conquer Muslims and make them against was sentenced to jail:
Congress? [M.P.P.C.S. (Pre) 1997] [R.A.S./R.T.S. (Pre.) 2021]
(a) Lord Curzon (a) Rama Devi
(b) Lord Dufferin (b) Ratan Shashtri
(c) Lord Hardinge (c) Anjana Devi Chaudhary
(d) Lord Minto (d) Kishori Devi
263. Match the following female Rulers with 269. Who among the following negotiated
their State/Capital: [MPPCS (Pre) 1994] Subordinate Alliances of 1817-18 with the
Princely States of Rajputana?
List-I List-II [R.A.S./R.T.S. (Pre) 2018]
A. Rani Durgavati 1. Jhansi (a) David Ochterlony
B. Maharani Ahilya Bai 2. Holkar State (b) Charles Metcalf
(c) Arthur Wellesley
C. Maharani Lakshmi Bai 3. Gara Mandal
(d) John George
D. Begum Razia Sultan 4. Delhi
270. The revolutionary, who was not involved in
Code: Hardinge Bomb incident?
A B C D [R.A.S./R.T.S. (Pre) 2018]
(a) 3 2 1 4 (a) Master Amir Chand
(b) 1 4 2 3 (b) Bhagwati Charan Vohara
(c) 2 3 4 1 (c) Bhai Balmukund
(d) 3 4 2 1 (d) Avadh Bihari
264. The author of the novel ‘Chandrakanta’ is: 271. Who has authored ‘Gandhian Constitution
[M.P.P.C.S. (Pre) 1994] for Free India’? [RAS/RTS (Pre) 2018]
(a) Bharatendu Harishchandra (a) Aruna Asaf Ali
(b) Achyut Patwardhan
(b) Premchand
(c) Shriman Narayan Agarwal
(c) Ravindra Nath Tagore (d) Humayun Kabir
(d) Devkinandan Khatri
272. Who was the first historian who wrote
265. Which Governor- General was prosecuted about ‘Feudalism’ in Rajasthan during the
for impeachment? [M.P.P.C.S. (Pre) 1992] 19th Century? [R.A.S./R.T.S. (Pre) 2010]
(a) Warren Hastings (a) Col. James Tod
(b) Lord Clive (b) Dr. L.P. Tessitori
(c) Lord Cornwallis (c) George Grearson
(d) Lord Wellesley (d) John Thomas

283 PYQ Workbook


HISTORY OF MODERN INDIA

273. The main aim of East India Company to 280. Which one of the following was the last step
make a subsidiary alliance in Rajput states in the Gandhian strategy of Satyagraha?
was- [R.A.S./R.T.S. (Pre) 1992] [U.P. Lower Sub. (Mains) 2013]
(a) Receiving military support against (a) Boycott
enemies (b) Picket
(b) Protecting these states from Maratha- (c) Fast
Pindari invasion (d) Strike
(c) To receive funds in the form of Khiraj
(d) To establish the sovereignty of the British 281. Which of the following statements is not
true as per Gandhian Principle?
274. Which one of the following Jails was named [U.P. Lower Sub. (Mains) 2013]
as ‘Mandir’ by Gandhiji?
(a) The aim of Satyagrahi is to defeat the
[U.P. Lower Sub. (Mains) 2015] enemy
(a) Naini (b) The weapon of Satyagraha is Ahimsa
(b) Yarvada (c) Satyagrahi should be firm in his belief
(c) Cellular (Port Blair) (d) Satyagrahi should have no ill feeling
(d) Aghakhan Palace towards his enemies
275. What does ‘Gandhian Innovation’ mean? 282. As per Mahatma Gandhi politics meant–
[U.P. Lower Sub. (Mains) 2015] [U.P. Lower Sub. (Mains) 2013]
(a) To produce more
(a) Religionless politics
(b) To produce in domestic economy
(b) Activity for public welfare
(c) To produce for consumption
(c) Truthless politics
(d) To produce more from less input for more
(d) None of the above
people
276. The writer of ‘Indian National Movement: 283. Which of the following is not a feature of
The Long-Term Dynamics’ is: politics of the Gandhian model?
[U.P. Lower Sub. (Pre) 2015] [U.P. Lower Sub. (Mains) 2013]
(a) Satish Chandra (a) Ethics
(b) Bipan Chandra (b) Religion
(c) Tarachand (c) Humanity
(d) Sumit Sarkar (d) Authority
277. According to Gandhiji, the Cruelest form of 284. Gandhi’s boycott of British made products
violence is- was effective because the British considered
[U.P. Lower Sub. (Mains) 2015] India, a major:
(a) Persistence of poverty [U.P. Lower Sub. (Mains) 2013]
(b) Killing of cows (a) Shipping centre
(c) Killing of human beings (b) Industrial centre
(d) Torture of women and children (c) Market for manufactured goods
(d) Source of mineral resources
278. Where did Gandhiji adopt ‘Seva Dharma’?
[U.P. Lower Sub. (Mains) 2015] 285. Whose work has influenced the Gandhian
(a) Mumbai concept? [U.P. Lower Sub. (Mains) 2013]
(b) Shantiniketan (a) Ruskin
(c) South Africa (b) Thoreau
(d) Pune (c) Tolstoy
(d) All the above
279. Which one of the following pairs is not
correctly matched? 286. Which of the following statements is/are
[U.P. Lower Sub. (Pre) 2013] correct regarding Gandhi?
(a) Lord Dalhousie - Annexation of Awadh [U.P. Lower Sub. (Mains) 2013]
(b) Lord Dufferin - Establishment of Indian (a) Fought hard to improve the status of
National Congress casteless untouchables
(c) Lord William Bentinck - Passing of the (b) Launched the Non-Cooperation
Charter Act, 1833 Movement
(d) Lord Lytton - Beginning of first Anglo- (c) Began the Civil Disobedience Movement
Afghan War (d) All the above

PYQ Workbook 284


HISTORY OF MODERN INDIA

287. Which one of the following statements is (a) Only 1 and 2 are correct.
not correct about Gandhian economy? (b) Only 2 and 3 are correct.
[U.P. Lower Sub. (Mains) 2013] (c) Only 1, 2 and 4 are correct.
(a) He laid emphasis on the economy based (d) Only 1, 2 and 3 are correct.
on non-violence. 291. Which one of the following statements is
(b) Centralization led to exploitation and not correct? [U.P. Lower Sub. (Pre) 2013]
inequality; hence centralization is (a) Mahatma Gandhi’s autobiography
opponent of formation of non-violent was originally written in the Gujarati
society Language.
(c) He was not in favour of mechanization in (b) Saddler Commission is associated with
India Education.
(d) He did not favour mechanization in (c) Hindu College Calcutta is the first
U.S.A. institution to help in spreading English
Education in India.
288. According to M.K. Gandhi socio-economic
(d) Lala Lajpat Rai was never elected as
improvement of untouchables can be
President of Indian National Congress.
brought about:
[U.P. Lower Sub. (Mains) 2013] 292. Who wrote the book “Gokhale- my political
(a) By their temple entry guru”? [U.P. Lower Sub. (Mains) 2013]
(b) By providing grant-in-aid (a) M.A. Jinnah
(c) By earmarking funds for their socio- (b) M.K. Gandhi
economic development (c) Shaukat Ali
(d) By establishing cottage industry for them (d) C.R. Das
289. Consider the following statements: 293. M.K. Gandhi was a supporter of–
[U.P. Lower Sub. (Pre) 2013] [U.P. Lower Sub. (Pre) 2009]
1. The Arya Samaj was founded in 1875. (a) Marxist socialism
(b) Category socialism
2. ‘Al Hilal’ was published by Maulana Abul (c) Idealism
Kalam Azad. (d) Philosophical anarchism
3. Lala Lajpat Rai was not associated with
the paper ‘The People’. 294. Which one of the following statements is
4. The famous Presidency College not correct about Dadabhai Naoroji?
(former Hindu College) of Calcutta was [U.P. Lower Sub. (Pre) 2008]
established by Raja Ram Mohan Roy. (a) He wrote a book ‘Poverty and Un-British
Rule in India’
Of these statements:
(b) He worked as a Professor of Gujarati in
(a) Only 1 and 2 are correct. the University College, London
(b) Only 2 and 3 are correct. (c) He laid the foundation of woman’s
(c) Only 1, 2 and 4 are correct. education in Bombay
(d) Only 1, 2 and 3 are correct. (d) He was elected as a member of
290. Consider the following statements: British Parliament on the ticket of the
[U.P. Lower Sub. (Pre) 2013] Conservative Party
1. Asaf Ali looked after the work of Railway 295. Aurobindo Ghosh wrote:
Ministry in the interim Government [U.P. Lower Sub. (Spl) (Pre) 2008]
(1946) (a) Extremist Movement
2. ‘Ancient Monuments Preservation Act’ (b) Moderate Movement
was passed when Lord Curzon was (c) Kesari
Governor-General. (d) The Life Divine
3. The Haripura Session of the Indian 296. Who made the agreement between Muslim
National Congress was presided over by League and Congress in 1916?
C.R. Das. [U.P. Lower Sub. (Pre) 2004]
4. Swami Shraddhanand suggested a no-tax (a) B. G. Tilak
campaign as a protest against the Rowlatt (b) Gokhle
Act. (c) Annie Besant
Of these statements: (d) J. L. Nehru

285 PYQ Workbook


HISTORY OF MODERN INDIA

297. Who is the author of flag song (Jhanda 304. James Andrew Ramsay was the real name of
Geet) of India? which Governor-General of India?
[U.P Lower Sub. (Spl) (Pre) 2004] [Uttarakhand P.C.S. (Pre) 2016]
(a) Rabindranath Tagore (a) Lord Dalhousie
(b) Ramdhari Singh Dinkar (b) Lord Canning
(c) Shyamlal Parshad Gupta (c) Lord North
(d) Ganesh Shankar Vidyarthi (d) Lord Curzon
298. In whose rule the Widow Remarriage Act 305. Which one of the following Ashrams related
was implemented– to Mahatma Gandhi, is the oldest?
[U.P. Lower Sub. (Pre) 2003] [Uttarakhand P.C.S. (Pre) 2012]
(a) Lord Dalhousie (a) Sabarmati
(b) Lord Canning (b) Phoenix
(c) Sir Henry Harding (c) Wardha
(d) Lord Lawrence (d) Sadaqat
299. Which one of the following is correctly 306. Who wrote the book “A Passage to India”?
matched? [UP Lower Sub. (Pre) 2003] [Uttarakhand P.C.S. (Pre) 2010]
Writer Books (a) Jawaharlal Nehru
(b) Minoo Masani
A. Valentine Chirol 1. Indian Struggle (c) E.M. Forster
B. Rafiq Zakaria 2. The Man Who (d) None of them
Divided India 307. Soj-e-Vatan is the book written by:
C. Subhash 3. Indian Unrest [Uttarakhand P.C.S. (Pre) 2010]
Chandra Bose (a) Mahadevi Verma
D. V.D. Savarkar 4. Unhappy India (b) Premchand
(c) Sumitra Nandan Pant
300. Which of the following Movement is not (d) Suryakant Tripathi ‘Nirala’
related to Gandhiji?
[U.P. Lower Sub. (Pre) 1998] 308. Who wrote the book ‘Geographical Factors
in Indian History’?
(a) Swadeshi Movement
(b) Khilafat Movement [Uttarakhand Lower (Sub) (Pre) 2010]
(c) Individual Satyagraha (a) K.M. Panikkar
(d) Quit India Movement (b) Toynbee
(c) M.N. Srinivas
301. In which language the famous work of Raja (d) Jamna Das
Rammohan Roy ‘A Gift to Monotheists’ was
written? [Uttarakhand PCS (Pre) 2021] 309. Match the following:
(a) Persian [Uttarakhand PCS (Pre) 2010]
(b) English List-I List-II
(c) Arabic
(d) Bengali A. Mahatma Gandhi 1. Dandi March
302. In 1929 at which place Mahatma Gandhi B. Jawaharlal Nehru 2. Red Shirt
completed his translation of Anasakti Yoga? Movement
[Uttarakhand PCS (Pre) 2021] C, Khan Abdul 3. Bardoli
(a) Kausani Gaffar Khan Satyagraha
(b) Almora D. Vallabhbhai Patel 4. Demand for
(c) Nainital Complete
(d) Ranikhet Independence in
303. Who introduced ‘Financial Lahore Session
Decentralization’ in India? Code:
[Uttarakhand PCS (Pre) 2021] A B C D
(a) Lord Rippon (a) 4 1 3 2
(b) Lord Dalhousie (b) 4 3 2 1
(c) Charles Metcaff (c) 2 1 4 3
(d) Lord Mayo (d) 1 4 2 3

PYQ Workbook 286


HISTORY OF MODERN INDIA

310. Who wrote the play “Andher Nagari A B C D


Chaupat Raja”? (a) 3 4 1 2
[Uttarakhand P.C.S. (Mains) 2006] (b) 4 3 2 1
(a) Govardhan Ram M. Tripathi (c) 1 2 3 4
(b) Munshi Premchand (d) 2 1 4 3
(c) Fakir Mohan Senapati
(d) Bhartendu Harishchandra 314. Swaraj as a national demand was first made
by: [Uttarakhand P.C.S. (Pre) 2002]
311. Which of the following statements is not (a) B.G.Tilak
true for Jawaharlal Nehru (b) C.R.Das
[Uttarakhand P.C.S. (Mains) 2006] (c) Dadabhai Naoroji
(a) He was influenced by Socialism. (d) Mahatma Gandhi
(b) He was influenced by British liberalism.
(c) He was influenced by Mahatma Gandhi. 315. Surat split was led by–
(d) He was influenced by German [Uttarakhand P.C.S. (Mains) 2002]
Nationalism. (a) Hume
312. Who was the person to conduct Census in (b) Dufferin
India for the first time? (c) Tilak
[Uttarakhand P.C.S. (Mains) 2006] (d) Gandhiji
(a) Ripon 316. Integration of States was done under the
(b) Lytton leadership of:
(c) Dufferin [Uttarakhand P.C.S. (Mains) 2002]
(d) Lord Mayo (a) Maulana Azad
313. Match list-I with list-II and select the (b) Govind Ballabh Pant
correct answer using the codes given below (c) B. R. Ambedkar
the lists: (d) Sardar Patel
List-I List-II 317. Who among the following was the author of
A. Madan Mohan 1. Founder of the official history of Congress?
Malviya Home Rule [Uttarakhand P.C.S. (Pre) 2002]
League (a) C. Rajagopalachari
B. Motilal Nehru 2. Launched (b) J.B. Kripalani
Servants of (c) Pattabhi Sitaramayya
India Society (d) Sardar Patel
C. Mrs. Annie Besant 3. Founder of 318. The famous work of Bharatendu
Banaras Hindu Harishchandra is:
University [Uttarakhand P.C.S. (Mains) 2002]
D. Gopal Krishna 4. Established (a) Mayanka Manjari
Gokhale Swaraj Party (b) Bharat Durdasha
with others (c) Nutan Brahmachari
Code: [Uttarakhand PCS (Pre) 2003] (d) Chandra Kanta Santati

287 PYQ Workbook


HISTORY OF MODERN INDIA

SOLUTIONS
Important Tips
6.1. UPSC CSE Previous Years’ Questions “Desher Katha” by Sakharam Ganesh Deuskar:
1. Solution (c) Sakharam Ganesh Deuskar, a Marathi Brahmin who settled
Exp) Option c is the correct answer. in Bengal, was born in Deoghar.
Mahatma Gandhi was associated with “Songs from He published the book “Desher Katha” in 1904, which
Prison”, a translation of ancient Indian religious lyrics provided an exhaustive account of British commercial
from Upnishads and other religious scripture, during his and industrial exploitation in India.
imprisionment in Yarvada Jail in 1930s. “Desher Katha” sold ten thousand copies in its first year
2. Solution: (c) and reached its fifth edition by 1905.

Exp) Option c is the correct answer. The Bengal government banned the book in 1910 and
confiscated all copies.
The National anthem was first sung outside the Calcutta
by the Rabndranath Tagore himself during a session at the Deuskar popularized the ideas of Dadabhai Naoroji and
Besant Theosophical College in Madanapalle, Andhra M.G. Ranade, promoting Swadeshi in a popular format.
Pradesh, on 28th Feb 1919, when Tagore went to college and His book warned against the colonial state’s manipulation
sang the song. Then deputy director of the university (also of Indian minds, termed as its ‘hypnotic conquest.’
an expert in European music and wife of the Irish poet James
Despite the growth of regional and national consciousness,
Cousins), asked Tagore to create an English translation of
Bengali lacked a specific word for ‘nation.’
the song and establish the musical notation. Tagore then
translated the Bengali song to English and the tune was Deuskar used ‘desh’ to refer to the nation, infusing a
composed by Margaret with the girls in the college singing broader perspective beyond Bengal.
the chorus.
5. Solution (d)
3. Solution (a) Exp) Option d is the correct answer.
Exp) Option a is the correct answer. Pair 1 is correctly matched. Tej Bahadur Sapru was the
The first monthly journal aimed at untouchable people, President of All India Liberal Federation and was a
titled “Vital-Vidhvansak” (Destroyer of Brahmanical prominent member of Liberal Party of India.
or Ceremonial Pollution), was published by Gopal Baba
Sir Tej Bahadur Sapru played a pivotal role in the development
Walangkar in 1888. His work aimed to raise awareness
about the injustices faced by untouchables and to challenge of modern India’s constitutional and political life. He was a
the caste system’s position in society, primarily through member of the All India Congress Committee, the General
appealing to the elites and encouraging change. Walangkar’s Secretary of the Congress party and also the President of the
writings marked an early intellectual rebellion against the UP Congress Committee. He was also a member of the UP
caste system. He also wrote articles for Marathi-language Legislative Council, and thereafter of the Imperial Legislative
newspapers such as Sudharak and Deenbandhu, as well as Council. He became one of the founding members of a
composing couplets in Marathi that were intended to inspire party that eventually came to be called The National Liberal
the people. Federation of India. He left the Congress in 1919-20, when
the Congress took a turn towards mass politics.
4. Solution (a)
Pair 2 is correctly matched. K.C. Neogy was a member of
Exp) Option a is the correct answer.
the Constituent Assembly of India, and the member of the
Statement 1 is correct: “Desher Katha” by Sakharam Ganesh
first Cabinet of independent India. Neogy was a member of
Deuskar warned against the colonial state’s influence on the
the Indian National Congress and was elected as a member
minds of the people. It highlighted how the colonial state was
of the Central Legislative Assembly in 1920 representing
trying to control and manipulate the thoughts and beliefs of
the Indian population. Bengal. He was the second Finance Minister of free India.
The First Finance Commission was constituted under the
Statement 2 is correct: The book “Desher Katha” had a
chairmanship of KC Neogy for the period 1952-57 in the
significant impact in Bengal and inspired the performance of
swadeshi (indigenous) street plays and folk songs. It played year 1952.
a crucial role in preparing the ground for the Swadeshi Pair 3 is correctly matched. P.C. Joshi was the first General
movement. Secretary of the Communist Party of India. He was one of
Statement 3 is incorrect: Sakharam Ganesh Deuskar used the leading figures of the communist movement in India.
the term ‘desh’ to refer to the nation, not specifically the Born in Almora in 1907, Joshi joined the Communist Party
region of Bengal. His book had a broader perspective on the in 1929, and rose to prominence thereafter. He was arrested
exploitation of India as a whole by the colonial state. in connection with the Meerut Conspiracy Case.

PYQ Workbook 288


HISTORY OF MODERN INDIA

6. Solution (a) Home Rule League aimed at achieving self-government and


Exp) Option a is the correct answer. dominion status within British Empire.
Till independence, there was only one central trade union Statement 2 is incorrect. Theosophical Society was founded
organisation, the All India Trade Union Congress (AITUC) in New York by Madam Blavatsky and Henry Steel Olcott
which was formed in 1920 and created lot of industrial strife. and was later shifted to Adyar Chennai. The founders of this
The Congress Party therefore took the initiative in forming a organization were inspired by Indian thought and culture.
trade union centre (INTUC) . The socialists in INTUC were Statement 3 is correct. She became the first woman president
dissatisfied with the functioning of INTUC and founded of Indian National Congress when she presisded over the
Hind Mazdoor Sabha (HMS) in Howrah as a National Trade 1917 Calcutta Session.
Union Centre in India. Its founders included Basawan Singh
10. Solution (b)
(Sinha) , Ashok Mehta, R.S. Ruikar, Mani Benkara, Shibnath
Benerajee, T.S. Ramanujam, VS. Mathur, G.G. Mehta. Mr. Exp) Option b is the correct answer.
R.S. Ruikar was elected president and Ashok Mehta as its Mohandas Gandhi translated Unto This Last into Gujarati in
General Secretary. It acted as a balancing force between the 1908 under the title of Sarvodaya (Well Being of All).
INTUC and AITUC. He summarised Unto This Last’s teachings in these three
points:
7. Solution (b)
1. That the good of the individual is contained in the
Exp) Option b is the correct answer.
good of all.
01-08-1953: Air India was nationalised under the Air
2. That a lawyer’s work has the same value as the barber’s,
Corporations Act, 1953 and becomes India’s national flight.
in as much as all have the same right of earning their
01-07-1955: India’s largest bank Imperial Bank of India livelihood from their work.
renamed as State Bank of India. 3. That a life of labour, i.e., the life of the tiller of the soil
05-04-1957: First democratically elected Communist Party and the handicraftsman, is the life worth living.
government outside Europe gets to power in Kerala with CPI
leader EMS Namboodiripad as the Chief Minister. 11. Solution (b)

19-12-1961: 19 th December is observed as the Goa Liberation Exp) Option b is the correct answer.
Day. Portuguese rule was ended on this day. Usha Mehta was an active supporter of the Quit India
Movement. She started an underground radio in Bombay.
8. Solution (c) She was an important member of a small group which ran
Exp) Option c is the correct answer. the Congress Radio.
Lala Lajpat Rai was prolific writer and authored several works During Quit India, many nationalists went underground
like – “Unhappy India”, “Young India: An Interpretation”, and took to subversive activities. The main personalities
“History of Arya Samaj”, “England’s Debt to India”. He also taking up underground activity were Rammanohar Lohia,
wrote several biographies of Mazzini (Life of Mazzini) , Jayaprakash Narayan, Aruna Asaf Ali, Usha Mehta, Biju
Garibaldi, Shivaji (Chatrapati Shivaji) and Hindu god Shri Patnaik, Chhotubhai Puranik, Achyut Patwardhan, Sucheta
Krishna (Yogiraj Shrikrishna) . His purpose in selecting Kripalani and R.P. Goenka. Underground activities were
Mazzini and Garibladi was to infuse patriotic sentiment in meant to keep up popular morale by continuing to provide
the youth of Punjab, who had no access to books in English. a line of command and guidance to distribute arms and
ammunition.
Important Tips
Facts related to Lala Lajpat Rai: 12. Solution (b)
In 1917, Lala Lajpat Rai moved to the United States Exp) Option b is the correct answer.
during the First World War and founded the Indian Statement 1 is incorrect: It was not Dr. Rajendra Prasad
Home Rule League of America in New York. He stayed who persuaded Mahatma Gandhi to come to Champaran,
in the United States from 1917 to 1920. Rai returned to but Raj Kumar Shukla, a farmer from Champaran who was
India in 1920 and in the following year led the special suffering from the exploitation of the European indigo
session of the Indian Congress Party that launched the planters.
non-co-operation movement. He joined Swaraj Party in Statement 2 is correct: Acharya J.B. Kripalani was one
1926 and was elected as its Deputy Leader in the Central of Mahatma Gandhi’s colleagues in his Champaran
Legislative Assembly. investigation. Kripalani was a prominent educator, social
activist and politician who was a close associate of Gandhi
9. Solution (c)
and a longtime supporter of his ideology. He joined Gandhi
Exp) Option c is the correct answer. in Champaran along with other eminent lawyers and helped
Statement 1 is correct. Annie Besant along with Bal him mobilize the peasants for the Champaran Satyagraha,
Gangadhar Tilak had formed and led the Home Rule which was Gandhi’s first civil disobedience movement in
Movement (1916-1918) . It was based on the lines of Irish India.

289 PYQ Workbook


HISTORY OF MODERN INDIA

13. Solution (a) Important Tips


Exp) Option a is the correct answer Notable works of Sarojini Naidu:
In 1906, the session at Calcutta was presided by Dada The Golden Threshold (1905): Naidu’s debut collection of
Bhai Naoroji. The moderates choose Dada Bhai Naoroji to poems that introduced her poetic talent to the world.
preside over the Congress. But, in this session, the congress The Bird of Time: Songs of Life, Death & the Spring
was compelled by the extremists to adopt the following (1915): Another collection of Naidu’s poetry, exploring
resolutions which were accepted by the moderates with half themes of life, death, and the changing seasons.
heart. The Broken Wing: Songs of Love, Death, and the Spring
• Resolution on National Education. (1917): A poetic work delving into themes of love, loss,
and the renewal of life in spring.
• Resolution of Self Government (Swaraj).
The Song of the Palanquin Bearers (1919): Naidu’s lyrics
• Resolution on Swadeshi. set to music by Martin Shaw, reflecting the cultural and
• Resolution on Boycott musical influences of her time.

In this resolutions, the boycott of British goods was put The Sceptred Flute: Songs of India (1948): A collection
of poems showcasing Naidu’s deep connection to Indian
forward and an impassioned plea was made for cultivation of
culture and her poetic prowess.
the nationalist ideals.
King George V at his Coronation Durbar in Delhi in 16. Solution (c)
December 1911 announced the revocation of the Partition of Exp) Option c is the correct answer.
Bengal. Hence Annulment was done in 1911 and not in 1907. J.B. Kripalani was a teacher by profession among the
Hence, Option 1 is not one of those resolutions. Gandhian followers. He was a professor of English and
History at Fergusson College in Pune before joining the
Important Tips
freedom movement following Gandhi’s return from South
Dada Bhai Naoroji presided over the INC session at Africa. He came in contact with Gandhi in 1917 during
Calcutta in 1906 in which a resolution supporting the Champaran Satyagraha and became one of his loyal
program of swadeshi, boycott, and national education was
supporters.
passed.
Ras Behari Ghosh presided over the Surat session in 1907 Important Tips
in which congress was split into moderates and extremists. A. N. Sinha: He was an Indian politician and a leader
of the Indian National Congress. He was also a lawyer,
14. Solution (c) journalist, and social reformer who fought for the rights
Exp) Option c is the correct answer. of the peasants and workers in Bihar. He was known as the
“Bihar Kesari” or the Lion of Bihar. He was a member of
Cornwallis was the Governor-General of India from 1786
the Constituent Assembly of India and served as the first
to 1793, and he created the Covenanted Civil Service of Deputy Chief Minister of Bihar.
India as part of his administrative reforms. He introduced
Braj Kishore Prasad: He was an Indian lawyer and a leader
a system of merit-based recruitment and promotion for of the Indian National Congress. He was also a freedom
the civil servants, who were required to sign a covenant of fighter who participated in various movements such as
good behavior and loyalty to the Company. He also raised the Non-Cooperation Movement, the Civil Disobedience
their salaries and prohibited them from engaging in private Movement, and the Quit India Movement. He was a close
trade or accepting gifts from the Indians. He aimed to create associate of Rajendra Prasad and served as the President
a professional and efficient civil service that would serve the of the Bihar Provincial Congress Committee. He was also
interests of the Company and the British Empire. a member of the Constituent Assembly of India.

17. Solution (b)


15. Solution (c)
Exp) Option b is the correct answer.
Exp) Option c is the correct answer.
Lal Bahadur Shastri, who served as the second Prime
“The Golden Threshold” is a collection of poems by
Minister of India, translated the autobiography of Madam
Sarojini Naidu, who was often referred to as “Bharatiya
Curie into Hindi. Madam Curie was a pioneering physicist
Kokila” or “The Nightingale of India.” She was a prominent and chemist known for her groundbreaking research on
Indian poet, freedom fighter, and child prodigy. Notably, radioactivity, and her autobiography provides insights into
Naidu was the first Indian woman to serve as the President of her life and scientific contributions. Lal Bahadur Shastri’s
the Indian National Congress and the first woman to become translation made this work accessible to Hindi-speaking
a governor of an Indian state. Her poetry, known for its readers, contributing to the dissemination of scientific
lyrical beauty, was published in this collection in 1905. knowledge in India.

PYQ Workbook 290


HISTORY OF MODERN INDIA

18. Solution (b) Jivancharita (1849): A biography of Raja Rammohan


Exp) Option b is the correct answer. Roy
The poem “Subh-e Azadi” (Dawn of Freedom) was written Bodhadoy (1851): A book on ethics
by the renowned Pakistani poet Faiz Ahmed Faiz in August Upakramanika (1851): A book on Sanskrit grammar
1947. This Urdu poem reflects the poet’s sentiments about the
Rijupath (1851-52): A book on logic
partition of India and Pakistan, expressing disappointment,
anguish, and a critical atmosphere surrounding the events. Byakaran Kaumudi (1853): A book on Bengali grammar
Faiz’s poem sympathizes with those who migrated during Borno Parichoy (1854): A book on Bengali phonetics
the partition and addresses the pain and sadness associated Shakuntala (1854): A translation of the Sanskrit play
with the cost of sovereignty. It has been both praised and Shakuntala
criticized for its views and ideological style regarding the
Mahabharata (1860): A translation of the Sanskrit epic
partition.
Mahabharata
Important Tips Seetar Vanavas (1860): A translation of the Sanskrit epic
Faiz Ahmed Faiz: Ramayana
Faiz Ahmed Faiz, a prominent poet of the 20th century,
Ratnopariksha (1886): A book on philosophy
crafted verses that transcend time. His poems are a blend
of emotions, weaving themes of love, resistance, and social Bidhobabivah (1855), concerning the rights of widows
justice. to remarry.
Some of his notable poems: Bahubivah (1871) against polygamy.
“Aaj Bazaar Mein Pa-Ba Joulaan Chalo” exemplifies his Balyabivah (1871) against child marriage.
metaphoric brilliance in depicting the struggle against
oppression. 21. Solution (d)
“Aaj Ik Harf Ko Phir” portrays his fiery desire to express.
Exp) Option d is the correct answer.
“Bahaar Aayi” explores love tinged with sorrow.
“Bol” champions raising voices against injustice. The song “Amar Sonar Bangla” was written by Rabindranath
“Kab Yaad Mein Tera Saath Nahin” and “Mujhse Pehli Si Tagore in 1905 during the first partition of Bengal, which
Muhabbat Mehboob Na Maang” delve into lost love. occurred under British rule. The song served as an ode to
19. Solution (c) Mother Bengal and was intended to rekindle the spirit of
Exp) Option c is the correct answer. unity in Bengal amidst communal divisions. In 1971, during
Elizabeth Hawley is renowned for her meticulous the liberation war of Bangladesh, the first ten lines of this
chronicling of Himalayan expeditions. Although she never song were adopted as the national anthem of Bangladesh,
climbed mountains herself, her Himalayan Database serves
symbolizing its struggle for independence and the nation’s
as a comprehensive and trusted record of climbs in the
Nepalese Himalaya. She was respected for her accuracy and identity.
investigative skills, earning her the nickname “The Sherlock
Important Tips
Holmes of the Mountaineering World.” Hawley’s work was
crucial for climbers, establishing success and fatality rates Rabindranath Tagore Literary works:
and settling disputes over summit claims. His poetic journey encompasses several remarkable
collections, including “Manasi” (1890), known as “The
20. Solution (b)
Ideal One,” and “Sonar Tari” (1894), translated as “The
Exp) Option b is the correct answer. Golden Boat.” However, it was “Gitanjali” (1910), or
The book “Bahubivah” was written by Ishwar Chandra “Song Offerings,” that catapulted him to international
Vidyasagar in 1871. Ishwar Chandra Vidyasagar was a acclaim, earning him the Nobel Prize in Literature.
renowned social reformer and educator in 19th-century Tagore continued to enchant readers with “Gitimalya”
India. He advocated for various social reforms, including (1914) and “Balaka” (1916), commonly referred to as
widow remarriage and women’s education. “Bahubivah” “The Flight of Cranes.”
was one of his works that aimed to address the issue of Tagore’s literary legacy extends into the realm of drama,
polygamy, particularly among the Hindu community, and with notable plays like “Raja” (1910), exploring themes
promote social change to improve women’s rights and status. of love and darkness, and “Dakghar” (1912), which
Important Tips poignantly captures the imagination of a young boy
confined to his home. “Achalayatan” (1912) delves into
Some of the literary works of Ishwar Chandra
the human spirit’s quest for freedom, while “Muktadhara”
Vidyasagar:
(1922) addresses societal reform. “Raktakaravi” (1926),
Betaal Panchavinsati (1847): A collection of folktales or “Red Oleanders,” confronts themes of oppression and
Banglar Itihaas (1848): A history of Bengal rebellion.

291 PYQ Workbook


HISTORY OF MODERN INDIA

In addition to his dramatic works, Tagore ventured into Statement 2 is correct: William Bentinck was the first
prose and novels. “Gora” (1910) grapples with identity, Governor-General of India, which was a title given to him
social reform, and nationalism, while “Ghare-Baire” by the Charter Act of 1833 that abolished the East India
(1916), known as “The Home and the World,” delves Company’s monopoly on trade and gave more power to the
into the complexities of love, politics, and personal ideals. British Crown in India. He served as Governor-General
“Yogayog” (1929), or “Crosscurrents,” explores the clash of India from 1834 to 1835, after being Governor-General
between tradition and modernity. of the Presidency of Fort William (Bengal) from 1828 to
Beyond fiction, Tagore crafted an extensive body of 1834. He is credited with reforms in educational, social and
work, including short stories, essays, travel diaries, and judicial spheres during his tenure as Governor-General of
autobiographies. His creative talents extended to musical British India.
and dance dramas, and he was an accomplished artist,
25. Solution (b)
creating drawings and paintings.
Exp) Option b is the correct answer.
22. Solution (d)
The book ‘Gita Rahasya’ was written by Bal Gangadhar
Option d is the correct answer. Tilak, an Indian social reformer and independence
Statement 1 is correct: Jawaharlal Nehru served as the activist. This Marathi language book, written while Tilak
Prime Minister of India from August 15, 1947, when India was in prison in Mandalay, Burma, in 1915, analyzes Karma
gained independence, until his death on May 27, 1964. He yoga as found in the Bhagavad Gita. Tilak emphasized the
was in his 4 th term when he died, (1947-51, 1952-57, 1957- message of Nishkam Karmayoga (selfless action) from the
1962, 1962-64) Gita. The book comprises philosophical exposition, the Gita,
Statement 2 is incorrect: Jawaharlal Nehru represented the its translation, and commentary.
Phulpur constituency in the Lok Sabha (lower house of
Important Tips
Parliament).
Literary works of Bal Gangadhar Tilak:
Statement 3 is correct: The first non-Congress Prime
Minister of India was Morarji Desai, who assumed office Tilak founded two influential newspapers, Kesari in
in 1977 after Janata alliance won the general elections. This Marathi and Mahratta in English in 1880–1881. Kesari
marked a significant political shift in India as the Indian later became a daily newspaper and is still published today.
National Congress had been in power since independence In 1903, Tilak authored “The Arctic Home in the Vedas,”
in 1947. positing that the Vedas originated in the Arctic region and
were brought south by Aryan bards post the last ice age. In
23. Solution (d)
“The Orion,” he attempted to date the Vedas by studying
Exp) Option d is the correct answer. the positions of Nakshatras.
The book, “The Story of the Integration of the Indi-
an States,” is authored by V.P Menon. It recounts the 26. Solution (a)
complex process of integrating princely states into Exp) Option a is the correct answer.
India after independence, highlighting the collective Annie Besant was a proponent of Fabianism as a movement.
efforts and challenges faced during the period of Annie Besant was a prominent member of the Fabian Society
1947-1951. The author provides an objective account and actively promoted the ideas of Fabianism in her writings
of historical events, emphasizing the role of unsung and political activities.
heroes in this significant consolidation of the nation.
The book also sheds light on the background of the Important Tips
princely states and the intricacies of this monumen- Fabianism:
tal task. It was founded in 1884, is a form of socialism that advances
the principles of democratic framework to achieve gradual
24. Solution (b)
conversion to socialism rather than by revolutionary
Exp) Option b is the correct answer. overthrow.
Statement 1 is incorrect: The first Governor-General of It is a type of socialism through peaceful and constitutional
Bengal was Warren Hastings, who served from 1773 to method with the help of middle class.
1785.
They believe in the authority of State. State is an important
Robert Clive was not the first Governor-General of Bengal. agent to bring socialism.
Robert Clive served as Governor of Bengal twice, from 1757
Eg. Labour party in Britain is one of outcome of ideology
to 1760 and from 1764 to 1767. However, he was not the first
of Fabianism.
Governor-General of Bengal, which was a higher position
created by the Regulating Act of 1773 to oversee the affairs After Independence, Nehru was the supporter of
of the East India Company in India. Fabianism.

PYQ Workbook 292


HISTORY OF MODERN INDIA

27. Solution (a) • Ilbert Bill was a proposed law introduced by Lord Ripon
Exp) Option a is the correct answer. when he was the Viceroy of India from 1880 to 1884.
The correct chronological order of the tenure of the four The Bill aimed to remove the racial discrimination
Viceroys of India during the British rule is: in the judicial system by allowing Indian judges and
magistrates to try cases involving British subjects in
1. Lord Curzon was the Viceroy of India from 1899 to
India. The Bill was eventually modified and passed in
1905. He was known for his imperialist and controversial 1884, but it failed to satisfy either the Indians or the
policies, such as the partition of Bengal, the Indian British.
Universities Act, the Curzon-Kitchener controversy, and
the Delhi Durbar. 29. Solution (c)
Exp) Option c is the correct answer.
2. Lord Hardinge was the Viceroy of India from 1910 to
1916. He reversed the partition of Bengal in 1911. He Satara was the first princely state to be annexed by the
British in 1848 using the Doctrine of Lapse, followed by
also shifted the capital of India from Calcutta to Delhi in
Sambalpur in 1849 and Jhansi in 1853. The Doctrine of
1911. He faced the challenge of the First World War and Lapse was a policy devised by Lord Dalhousie, the governor-
its impact on India. general of India from 1848 to 1856, which stated that if a
3. Lord Chelmsford was the Viceroy of India from 1916 ruler of a dependent state died without a natural heir, the
to 1921. He introduced the Montagu-Chelmsford state would be annexed by the British. This policy was
Reforms in 1919, which granted more autonomy and widely resented by the Indian princes and contributed to the
outbreak of the Indian Rebellion of 1857.
representation to Indians in provincial legislatures. He
also faced the aftermath of the Jallianwala Bagh massacre 30. Solution (a)
in 1919, which sparked widespread protests and violence Exp) Option a is the correct answer.
across India. Vande Mataram is a poem written in Sanskrit and
4. Lord Irwin was the Viceroy of India from 1926 to 1931. Sanskritised Bengali by Bankim Chandra Chatterjee in
He initiated the Simon Commission in 1927, which the 1870s. The first two verses of the poem were adopted
as the National Song of India in October 1937 by the
was boycotted by all Indian political parties. He signed
Congress. The poem was first published in 1882 as part
the Gandhi-Irwin Pact in 1931, which ended the civil
of Chatterjee’s Bengali novel Anandmath. It is an ode to
disobedience movement and agreed to send a delegation the motherland, personified as the “mother goddess” in later
to the Second Round Table Conference in London. verses, of the people.
28. Solution (c) Important Tips
Exp) Option c is the correct answer. • The National Calendar of India, following the Saka
era, marks its first day of Chaitra on March 22 in
Vernacular Press Act: Curzon is not correctly matched, regular years and on March 21 in leap years. This
because the Vernacular Press Act was passed by Lord system was officially adopted on March 22, 1957,
Lytton, not Lord Curzon. The Vernacular Press Act was a law alongside the Gregorian calendar, for official
enacted in 1878 to regulate the publication of newspapers purposes including Gazette publications, All India
and books in Indian languages. The Act gave the British Radio broadcasts, Government-issued calendars,
government the power to censor and suppress any content and public communications.
that was deemed seditious or offensive to the British rule. • The national flag of India is a horizontal rectangular
The Act was widely criticized by the Indian nationalists and tricolour flag, the colours being of India saffron, white
journalists, who saw it as a violation of their freedom of and India green; with the Ashoka Chakra, a 24-spoke
expression and a sign of racial discrimination. wheel, in navy blue at its centre. It was adopted in its
present form during a meeting of the Constituent
Important Tips Assembly held on 22 July 1947, and it became the
• Pitt’s India Act was passed in 1784 during the tenure of official flag of the Union of India on 15 August 1947.
Warren Hastings as the Governor-General of Bengal. The National Flag of India was designed by Pingali
The Act was intended to address the shortcomings of Venkayya in 1921.
the Regulating Act of 1773 by bringing the East India • “Jana Gana Mana” is the national anthem of the
Company’s rule in India under the control of the Republic of India. It was originally composed as
British government. Bharoto Bhagyo Bidhata in Bengali by Rabindranath
Tagore on 11 December 1911. The first stanza of
• Doctrine of Lapse was an annexation policy followed the song Bharoto Bhagyo Bidhata was adopted by
by Lord Dalhousie when he was the Governor-General the Constituent Assembly of India as the National
of India from 1848 to 1856. According to this policy, Anthem on 24 January 1950. A formal rendition of
any princely state under the direct or indirect control the national anthem takes approximately 52 seconds.
of the East India Company where the ruler did not have It was first publicly sung on 27 December 1911 at the
a legal male heir would be annexed by the Company. Calcutta Session of the Indian National Congress.

293 PYQ Workbook


HISTORY OF MODERN INDIA

31. Solution (c) Pratap as president, Maulana Barkatullah, Prime Minister,


Exp) Option c is the correct answer. Maulana Ubaidullah Sindhi, Home Minister. Hence
Statement d is correct.
Option A in List-I matches Option 2 in List-II: The
Communal Award was created by the British prime 33. Solution (b)
minister Ramsay MacDonald on 16 August 1932. Also
Exp) Option b is the correct answer
known as the MacDonald Award, it was announced after
the Round Table Conference (1930–32) and extended the The 1939 Session of All India States People’s Conference was
separate electorate to depressed Classes (now known as the held at Ludhiana, Punjab under the leadership of Jawaharlal
Scheduled Caste) and other minorities. Nehru. Jawaharlal Nehru continued to hold this position
till 1946. In his presidential speech to the Conference,
Option B in List-I matches Option 3 in List-II: The August
titled “Freedom is Indivisible”, he denounced the political
Offer was an offer made by Viceroy Linlithgow in 1940
backwardness and oppression inflicted on the people. He also
promising the expansion of the Executive Council of the
asserted the INC’s commitment to a united and free India,
Viceroy of India to include more Indians, the establishment
and criticized those who dreamt of political independence
of an advisory war council, giving full weight to minority
for their states separate from India. The Ludhiana Session
opinion, and the recognition of Indians’ right to frame their
of the All India States Peoples’ Conference was a milestone
own constitution.
in the growth of the freedom movement, and signaled the
Option C in List-I matches Option 1 in List-II: The unification of the different strands of nationalism that had
Doctrine of Lapse was introduced by Lord Dalhousie. developed
According to this doctrine, if any Indian ruler dies without
leaving a male heir, his kingdom will automatically pass over Important Tips
to the British. The All India States Peoples’ Conference (AISPC) was
Option D in List-I matches Option 4 in List-II: Dyarchy the association of the Praja Mandals in the princely states
was introduced as a constitutional reform by Edwin under the British Raj. These Praja Mandals, established
Samuel Montagu (secretary of state for India, 1917–22) all over the country sought to introduce responsible
and Lord Chelmsford (viceroy of India, 1916–21). The government.
principle of dyarchy was a division of the executive branch of The first session of AISPC was held in Bombay in 1927
each provincial government into authoritarian and popularly which brought together representatives from various
responsible sections. The first was composed of executive Indian princely states.
Councilors, appointed, as before, by the crown. The second In 1939, The Indian National Congress amended its policy
was composed of ministers who were chosen by the governor to lend support to the AISPC.
from the elected members of the provincial legislature. These
latter ministers were Indians. 34. Solution (b)
Exp) Option b is the correct answer
32. Solution (c)
The London branch of the All India Muslim League was
Exp) Option c is the correct answer.
established in 1908 under the presidency of “Syed Ameer Ali”
Sir syed Ahmed Khan felt that the Indian National Congress to put pressure on British Government for Muslims cause.
did not work towards the uplifment of the Indian Muslim
community. Therefore, he opposed the Indian National 35. Solution (a)
Congress. Hence Statement b is correct. Exp) Option a is the correct answer
A group of Muslim landlords and nawabs formed the All India Only Punjab in British India proposed a united and
Muslim League at Dacca in 1906. The League supported independent existence during partition, led by Sikh
the partition of Bengal. It desired separate electorates for leader Master Tara Singh. Congress sought Punjab’s unity
Muslims, a demand conceded by the government in 1909. with India, while the Muslim League aimed for a separate
Hence Statement c is not correct. Muslim-majority state. Ultimately, Punjab was religiously
The Ahrar Movement was founded in 1910 under the partitioned. Other provinces, like Assam, Bengal, and Bihar,
leadership of Maulana Muhammad Ali Mohamed and joined India without such plans.
Hakim Ajmal Khan. Other leaders of the movement were
Iqbal, CR Das, Sir Mian Muhammad Shafi etc. These leaders 36. Solution (d)
were strictly against the Lucknow pact, which favoured Exp) Option d is the correct answer.
the representation of religious minorities in provincial The native State of Tripura became involved in the
legislatures. This weightage formula of the Lucknow pact Freedom movement early in the 20th century because there
paved the way for the Muslim majority by the late 1920s. were already some groups fighting against the kingship
Hence Statement a is correct. and its protector, the British. Raja Bir Bikram Manikya’s
The first Provisional Government of India was established in government was under British influence, and the oppression
Kabul during First World War in December 1915. It was the and exploitation faced by tribes, like the Reangs, led to
government-in-exile of Free Hindustan with Raja Mahendra uprisings against both the local rulers and the British. This

PYQ Workbook 294


HISTORY OF MODERN INDIA

involvement reflected a broader discontent with colonial Important Tips


rule and its collaborators.
“The Indian War of Independence” is an Indian
37. Solution (b) nationalist history of the 1857 revolt by Vinayak
Exp) Option b is the correct answer. Damodar Savarkar, first published in 1909. The book was
banned in British India due to its inflammatory content
Lord Dalhousie was the Governor-General of India from
1848 to 1856, and he was responsible for the last major but was printed in the Netherlands in 1909. It describes
extension of British Indian territory. He annexed many the 1857 rebellion as a unified Indian uprising against
Indian states using the Doctrine of Lapse, which stated British rule and emphasizes Hindu-Muslim unity.
that if a ruler of a dependent state died without a natural Savarkar justifies Indian acts of vengeance as reactions to
heir, the state would be annexed by the British. Some of the British oppression.
states that were annexed by this policy were Jhansi, Satara, “Sadhana: The Realization of Life” is a collection of
Jaitpur, Sambalpur, Udaipur, and Nagpur. He also annexed spiritual discourses by Rabindranath Tagore. It delves
Awadh on the basis of misgovernance, accusing the Nawab into profound questions about existence, God, and the
of neglecting his subjects and failing to pay his debts to the purpose of life. Tagore’s lucid explanations draw from
British. Upanishads, Lord Jesus, and Buddha’s teachings, making
Lord Dalhousie’s aggressive expansionism and interference it a valuable spiritual guide.
in the affairs of the Indian princes provoked resentment
and resistance among the Indians, and contributed to the 40. Solution (c)
outbreak of the Indian Rebellion of 1857, which was also Exp) Option c is the correct answer.
known as the First War of Independence or the Sepoy Mutiny.
Lord Wellesley was the Governor-General of India from 1798
38. Solution (c) to 1805, and he kept the British flag flying high in India at
Exp) Option c is the correct answer. a time when empires in Europe were crumbling before the
Statement a is correct: “Neel Darpan” was a 19th-century might of Napoleon. He pursued an aggressive policy of
Bengali play by Dinabandhu Mitra, addressing the ruthless expansion and consolidation of British power in India,
exploitation of indigo farmers by British indigo planters in which earned him the nickname of “the Indian Napoleon”.
colonial India. The play highlighted the oppressive ‘Neel’ He implemented the system of subsidiary alliances, by which
(indigo) system, shedding light on the harsh conditions and
he forced the Indian rulers to accept British protection and
exploitation faced by these impoverished farmers.
supremacy in exchange for their autonomy and sovereignty.
Statement b is correct: “Ghashiram Kotwal,” a 1972
He also fought several wars against the enemies of the British,
Marathi play by Vijay Tendulkar, serves as a political
satire and historical drama. It critiques the manipulation of such as the Marathas, the Mysoreans, and the French. He
power and ideologies, using the lives of Nana Phadnavis and also promoted the cultural and educational development of
Ghashiram Kotwal to illustrate how those in authority create India, by founding the Fort William College and supporting
and discard ideologies for their benefit. the Asiatic Society of Bengal.
Statement c is incorrect: The play “Navanna,” written by
41. Solution (d)
Bijon Bhattacharya in 1944 and staged by IPTA, explores
the devastating Bengal famine of 1943, where over 2 Exp) Option d is the correct answer.
million people perished from hunger and disease. It follows The ‘Forward’ policy of Lord Lytton towards Afghanistan
Pradhan Samaddar, a peasant, depicting the harrowing
was a misguided attempt to counter the perceived
realities and profound suffering during this tragic period,
raising awareness and funds for famine relief. Russian threat in Central Asia. The policy was based on
the assumption that the British should pursue an aggressive
Statement d is correct: Parsi theatre was the first professional
theatre in India and it was very popular among the urban policy towards Afghanistan in order to prevent the Russians
middle class. Urdu theatre borrowed many elements from from gaining influence or territory there. The policy involved
Parsi theatre, such as the use of music, dance, and spectacle. imposing a British resident and troops in Kabul, demanding
Urdu playwrights also adapted many Parsi plays into Urdu. concessions from the Afghan ruler, and provoking a war
when the Afghans resisted. The policy resulted in the Second
39. Solution (a)
Anglo-Afghan War in 1878, which ended in a stalemate and
Exp) Option a is the correct answer.
a costly retreat for the British. The policy also alienated
The correct match between the books (List-I) and their
the Afghan people, who saw the British as invaders and
respective authors (List-II) is as follows:
oppressors.
The first Indian War of Independence - Vinayak Damodar
Savarkar Important Tips
Anand Math - Bankim Chandra Chatterjee • Minto was the Governor-General of India from 1807 to
Life Divine - Sri Aurobindo 1813, and he followed a policy of non-intervention and
Sadhana - Rabindranath Tagore conciliation towards Afghanistan.

295 PYQ Workbook


HISTORY OF MODERN INDIA

• He signed a treaty with Shah Shuja, the exiled To advocate for passengers’ rights, a Shore Committee
Afghan ruler, in 1809, which recognized his claim formed in Vancouver, led by figures like Hussain Rahim,
to the throne and promised him British support in Sohan Lal Pathak, and Balwant Singh. In the USA, a
case of foreign aggression. campaign led by Barkatullah, Bhagwan Singh, Ram
• Dufferin was the Viceroy of India from 1884 to 1888, Chandra, and Sohan Singh Bhakna supported the cause.
and he followed a policy of masterly inactivity towards Despite these efforts, the ship was denied entry and forced
Afghanistan. out of Canadian waters.
• He respected the autonomy and sovereignty of After a two-month standoff, the ship was escorted back
Afghanistan and maintained friendly relations with to India and arrived at Budge Budge, Calcutta. British
its ruler, Abdur Rahman Khan. authorities viewed passengers as lawbreakers and political
agitators.
• He also settled the boundary dispute between
Afghanistan and Russia through diplomatic Upon arrival, British forces stopped the ship, leading to
negotiations. a clash resulting in the deaths of 18 passengers and the
imprisonment of others. This fueled discontent in Punjab
• Elgin was the Viceroy of India from 1894 to 1899, and
and led to political dacoities in districts like Jalandhar,
he followed a policy of cautious engagement towards
Amritsar, and Ludhiana.
Afghanistan.
The incident exposed disparities in Canadian
• He supported the reforms and modernization
immigration laws and galvanized support for the Ghadar
efforts of Abdur Rahman Khan and his successor,
Party’s revolutionary aims. Ghadarites like Barkatullah
Habibullah Khan.
and Tarak Nath Das used it to rally members for the
• He also tried to balance the interests of Britain and Ghadar movement and plan uprisings in India. Human
Russia in Central Asia through mutual agreements. rights violations highlighted the true face of British rule.
42. Solution (d) 43. Solution (d)
Exp) Option d is the correct answer. Exp) Option d is the correct answer.
The correct sequence of events is: Pair 1 is correctly matched: Theodore Beck was a Quaker
Kamagatamaru Incident (1914): The Komagata Maru and British educationalist working for the British Raj in
incident in 1914 involved the Japanese steamship carrying India. From 1883 until his death in 1899 he was Principal
376 passengers, primarily Sikhs, Muslims, and Hindus from of Muhammadan Anglo-Oriental College, which later
British India, attempting to immigrate to Canada. Most were became Aligarh Muslim University.
denied entry, leading to a riot and police firing in Calcutta, Pair 2 is correctly matched: The Ilbert Bill was a legislative
resulting in 22 deaths. act introduced in 1883 during the tenure of Viceroy
Ripon and written by Sir Courtenay Pergine Ilbert. The act
Mahatma Gandhi’s arrival in India (1915): On 9 January
stipulated that Indian judges could try Europeans.
1915, M K Gandhi reached Bombay sailing from South
Africa, having lived there for more than two decades. This Pair 3 is correctly matched: Sir Pherozeshah Merwanjee
date was chosen in 2003 to be observed as Pravasi Bharatiya Mehta was an Indian politician and lawyer from Bombay.
Divas (Non-Resident Indian Day). Mehta was one of the founding members and President
of the Indian National Congress in 1890 held at Calcutta.
Tilak’s Home Rule League (1916): In 1916, Tilak established
He was knighted by the British Government in India for his
the Indian Home Rule League with headquarters in Poona, service to the law. He became the Municipal commissioner
primarily active in Maharashtra, Karnataka, Central of Bombay Municipality in 1873 and its president four times
Provinces, and Berar. Demands included swarajya, linguistic – 1884, 1885, 1905 and 1911.
states, and vernacular education.
Pair 4 is incorrectly matched: Badruddin Tyabji was an
Important Tips Indian lawyer, activist, and politician during the British
The Komagata Maru incident of 1914: Raj. Tyabji was the first Indian to practice as a barrister
of the High Court of Bombay who served as the third
The Komagata Maru incident of 1914 involved the
President of the Indian National Congress. He was one of
Komagata Maru, a passenger ship converted from a coal-
the founding members and first Muslim president of Indian
transport steamship by businessman Gurdit Singh in Hong
National Congress. He founded the Anjuman-i-Islam College
Kong. It arrived in Vancouver with 376 passengers, mostly
in Bombay in 1874.
Sikhs, after setting sail from Hong Kong.
Canada’s stringent immigration policies required 44. Solution (b)
immigrants to arrive via a continuous journey from Exp) Option b is the correct answer.
their country of birth or citizenship. This aimed to Option A in List-I matches Option 4 in List-II: The Surat
restrict Indian immigration, as many sought works in Split of 1907 divided the Indian National Congress (INC)
Canada after traveling through other countries. into Moderates and Radicals. It resulted from differing

PYQ Workbook 296


HISTORY OF MODERN INDIA

approaches to dealing with British rule, with Moderates 46. Solution (b)
favoring peaceful negotiations and Radicals advocating Exp) Option b is the correct answer.
more aggressive methods. This division deepened tensions
between the two groups and led to significant political Captain Sleeman was the British officer who was mainly
developments in India’s struggle for independence. responsible for the suppression of the Thugs, a notorious
gang of robbers and murderers who operated in various
Option B in List-I matches Option 3 in List-II: The
Communal Award, instituted by British Prime Minister parts of India. He was appointed as the superintendent of the
Ramsay MacDonald in August 1932, extended separate Thuggee and Dacoity Department in 1835 by Lord William
electorates to depressed classes (Scheduled Castes) and other Bentinck, the Governor-General of India. He conducted
minorities, building upon the existing system introduced in a systematic campaign to eradicate the Thugs, by using
1909 and 1919. informers, spies, and confessions to identify and arrest them.
Option C in List-I matches Option 2 in List-II: The All- He also introduced a system of rewards and incentives for the
Parties Conference was a group of Indian political parties Thugs who turned into witnesses and provided information
known for organizing a committee in May 1928 to author the about their fellow gang members. He captured and convicted
Constitution of India after independence was actualized. It more than 1400 Thugs, who were either hanged or imprisoned
was chaired by Dr. M. A. Ansari. for life. He also established a special prison for the Thugs at
Option D in List-I matches Option 1 in List-II: The Jabalpur, where they were reformed and rehabilitated.
Indian National Congress, on 19 December 1929, passed the
historic ‘Purna Swaraj’ – (total independence) resolution – 47. Solution (c)
at its Lahore session. A public declaration was made on 26
Exp) Option c is the correct answer.
January 1930 – a day which the Congress Party urged Indians
to celebrate as ‘Independence Day’. Option A in List-I matches Option 4 in List-II: The first
Indian National Union session was held in Kolkata at Albert
45. Solution (c)
Hall from 28 to 30 December 1883. It was prompted by the
Exp) Option c is the correct answer
Introduction of the Criminal Procedure Amendment Bill
Born on 9 May, 1866, in Maharashtra, Gopal Krishna (1883-1884) or Ilbert Bill.
Gokhale was an Indian liberal political leader and a social
Option B in List-I matches Option 3 in List-II: The All-
reformer during the Indian Independence Movement.
He was the political master of Mahatma Gandhi. He was India Muslim League (AIML) was a political party established
graduate at the age of 18, He became professor and associate in Dhaka in 1906 when some well-known Muslim politicians
editor of the Sudharak at the age 20, He became Secretary of met the Viceroy of British India, Lord Minto, with the goal of
the Sarvajanik Sabha and of the Provincial Conference at the securing Muslim interests on the Indian subcontinent.
age of 25, He was Secretary of the National Congress at the Option C in List-I matches Option 2 in List-II: The All-India
age of 29, He was leading witness before an important Royal
States Peoples’ Conference (AISPC) was a conglomeration of
Commission at the age of 31, He was Provincial legislator
political movements in the princely states of the British Raj,
at the age of 34 and Imperial legislator at age of36, He was
which were variously called Praja Mandals or Lok Parishads.
President of the Indian National Congress at age of 39.
The first session of the organisation was held in Bombay in
Important Tips December 1927.
Gopal Krishna Gokhale
Option D in List-I matches Option 1 in List-II: British Prime
He was the president of INC in 1905 and gave the slogan Minister Ramsay MacDonald announced the Communal
of “Swadeshi”.
Award on August 16, 1932, following the recommendations
He founded the”Servants of India Society” in 1905 with of the Indian Franchise Committee (Lothian Committee). It
the help of Mahadev Govind Ranade.
aimed to establish separate electorates in British India for
In addition to learning English, he was exposed to Western various religious and social groups, including Forward Caste,
political thought and became a great admirer of theorists
Lower Caste, Muslims, Buddhists, Sikhs, Indian Christians,
such as John Stuart Mill and Edmund Burke.
Anglo-Indians, Europeans, and Untouchables (now known
He had a great influence of the social works of Justice
as the Dalits).
Mahadev Govind Ranade on his life. He was named as the
Protege Son’ i.e. Manas Putra of Justice Mahadev Govind Important Tips
Ranade. Key Highlights of Indian Franchise Committee (Lothian
Gokhale’s deposition before the Welby Commission on the Committee):
financial condition of India won him accolades. • Primary Aim: To establish a suitable foundation for
He played a leading role in bringing about Morley-Minto a system of responsible government by framing a new
Reforms in India. franchise.

297 PYQ Workbook


HISTORY OF MODERN INDIA

• Electoral Expansion: Proposed an expanded 50. Solution (d)


electorate, with 29,382,000 men and 6,620,000 women Exp) Option d is the correct answer.
eligible to choose provincial legislature members, Pair 1 is correctly matched: Jamnalal Bajaj was closely
compared to the current figures of 6,792,281 men and associated with the Satyagraha Ashram in Wardha. Inspired
315,651 women. by Mahatma Gandhi’s principles, he brought his family to live
• Federal Assembly Electorate: Recommended a federal in the Ashram and played a significant role in its activities,
assembly electorate consisting of 6,864,899 men and including serving as the president of Gandhi Seva Sangha and
1,578,151 women, a significant increase from the supporting constructive work during the freedom struggle.
existing combined total of 1,142,948. Pair 2 is correctly matched: In the year 1852, Jagannath
• Reserved Seats for Women: Suggested that between Shankarshet founded the Bombay Association. Naoroji
2 percent and 5 percent of seats in both provincial Fursungi, Sir Jamshedji Jejibhai,Vinayak Shankarshet and
legislatures and the assembly should be temporarily Dadabhai Naoroji were among its notable members. The
reserved for women. organization’s first president was Sir Jamshedji Jejibhai. It
• Educational and Property Qualifications: Proposed was founded to vent public concerns against the British.
the introduction of educational and property Pair 3 is correctly matched: Lala Lajpat Rai established the
qualifications for voters, with special provisions to National School in Lahore during the Swadeshi movement,
ensure the enfranchisement of women, untouchables, fostering the concept of national education. This institution
and labor or other minority groups. later evolved into the National College, where Bhagat
• Exclusivity to British India: The report dealt Singh, the prominent Indian freedom fighter, received his
exclusively with British India, and the princes of the education.
various states were left to decide on the method Pair 4 is incorrectly matched: Satyashodhak Samaj,
of election of their representatives to the federal founded by Jyotiba Phule in 1873, aimed to liberate
assembly. Shudra and untouchable caste people from socio-cultural
oppression. It sought to eliminate the need for priests,
48. Solution (d) promote education among marginalized communities, and
Exp) Option d is the correct answer. challenge religious and caste-based discrimination. Inspired
by an incident of caste discrimination, Phule initiated this
The correct match for the books and their respective
society to search for truth and challenge traditional beliefs
writers is as follows:
perpetuating inequality.
Surendranath Banerjee - A Nation in Making
51. Solution (c)
M.K. Gandhi - Hind Swaraj
Exp) Option c is the correct answer.
Subhash Chandra Bose - The Indian Struggle
Vanguard, an emigre Communist journal, was established
Lajpat Rai - Autobiographical Writings by M.N. Roy. In the early 1920s, it served as a platform for
Important Tips disseminating communist ideology and revolutionary ideas.
This influential journal played a pivotal role in shaping the
Lajpat Rai’s “Autobiographical Writings” is a collection
early Indian communist movement and was widely read
of autobiographical works by Lala Lajpat Rai, providing
among intellectuals and revolutionaries.
insights into his life and involvement in the Indian
independence movement. 52. Solution (b)
Exp) Option b is the correct answer.
49. Solution (d)
The correct chronological sequence of the following events
Exp) Option d is the correct answer.
in the political life of Mahatma Gandhi is
Statement 1 is incorrect: Acharya J.B Kriplani was the
• Champaran Satyagraha: This was the first Satyagraha
President of Indian National Congress at the time of partition
movement led by Gandhi in India in 1917. It was
of India in 1947. He also served in the interim government
a farmer’s uprising in Bihar against the oppressive
of India (1946–1947) and the Constituent Assembly of India.
Tinkathiya system imposed by the British planters, who
Statement 2 is incorrect: Dr Sachchidananda Sinha, the forced them to grow indigo and sell it at a low price.
oldest member, was elected as the temporary President of the • Ahmedabad Mill Strike: This was a labour strike led by
Assembly, following the French practice. Later, on December Gandhi in 1918 in Ahmedabad, Gujarat. It was a protest
11, 1946, Dr Rajendra Prasad and H C Mukherjee were against the mill owners who refused to increase the wages
elected as the President and Vice-President of the Assembly of the workers after the plague epidemic. Gandhi used
respectively. the method of hunger strike to support the workers’
Statement 3 is incorrect: The first Congress Ministry in demand for a 35% increase.
United Province was formed in july 1937 and was headed by • Kheda Satyagraha: This was a peasant movement led
Pandit Govind Ballabh Pant. by Gandhi in 1918 in Kheda district, Gujarat. It was a

PYQ Workbook 298


HISTORY OF MODERN INDIA

resistance against the British government’s decision to • Here he tried to resolve the differences between the
collect full land revenue despite the failure of crops due moderates and the extremists within the party.
to drought and famine. Gandhi mobilized the peasants
to withhold their payments and demand a suspension of 55. Solution (d)
revenue collection. Exp) Option d is the correct answer.
• Non-cooperation Movement: This was a mass movement The founder of the Boy Scouts and Civil Guides movement
launched by Gandhi in 1920, which lasted till 1922. It in India was Lord Baden-Powell. Scouting was officially
was a response to the Rowlatt Act, the Jallianwala Bagh introduced in British India in 1909 and began to expand
massacre, and the Khilafat issue. It involved boycotting with native Indian members in 1913, thanks to the efforts
foreign goods, institutions, titles, and honours, and of various individuals, including Annie Besant, George
promoting swadeshi, education, and self-governance. Arundale, and others. Lord Baden-Powell’s visit to India
in 1921 played a significant role in uniting various Scout
53. Solution (c)
organizations in India under the umbrella of The Boy
Exp) Option c is the correct answer. Scouts Association in India. This marked a crucial step in
Jamnalal Bajaj was an Indian cotton merchant, banker, the development of Scouting in the country. The movement
Congressman and a close associate of Mahatma Gandhi. aimed to instill values of leadership, outdoor skills, and
He founded the Bajaj Group of companies in the 1920s, and community service among Indian youth.
the group now has 24 companies, including six that are listed
56. Solution (c)
on the bourses. He was also a philanthropist who supported
various social causes such as education, health, and rural Exp) Option c is the correct answer.
development. Lord Ripon was the Viceroy of India from 1880 to 1884,
and he passed the first Factory Act in India in 1881. This
Important Tips
Act was based on the terms of the Factory Act of Great
• M. R. Jayakar: He was an Indian lawyer and politician Britain, which was enacted in 1837. The first Factory Act in
who served as the Vice-Chancellor of Bombay India was a landmark legislation that aimed to regulate the
University and Pune University. He was also a leader working conditions of women and children in factories.
of the Indian National Congress and a member of The Act restricted the working hours of women and
the Constituent Assembly of India. He was involved children to nine hours per day and 48 hours per week. It
in various constitutional negotiations such as the also authorized the local governments to make rules for the
Round Table Conferences, the Cripps Mission, and health, safety, and welfare of the workers, such as providing
the Cabinet Mission. ventilation, lighting, drinking water, latrines, and first-aid
• V. S. Shrinivas Sastri: He was an Indian scholar, facilities.
educator, and diplomat who served as the Minister
of Education in Madras Presidency and the Agent- 6.2. Other UPSC CSE Previous Years’
General of India in South Africa. He was also a leader Questions
of the Indian National Congress and a member of the
57. Solution (c)
Imperial Legislative Council. He was known for his
eloquence and mastery of English language. Exp) Option c is the correct answer.
Statement 1 is correct: The Vaikom Satyagraha was a
54. Solution (b)
nonviolent agitation for access to the prohibited public
Exp) Option b is the correct answer. environs of the Vaikom Temple in the Kingdom of
Mahatma Gandhi was not a member of the Indian National Travancore, which was known for its rigid and oppressive
Congress at the time of India’s independence in 1947. He had caste system. It was a movement against untouchability and
resigned from the Congress Working Committee in 1934 and caste discrimination, as the lower castes (avarnas) were not
had no formal position in the party thereafter. He remained, allowed to enter or even walk on the roads near the temple.
however, the most influential leader of the Congress and Statement 2 is correct: Mahatma Gandhi participated in
the freedom movement, and guided the party’s policies and this Satyagraha and visited Vaikom in March, 1925. He also
strategies through his moral authority. supported the cause of temple entry for all Hindus and
negotiated with the Regent Maharani Sethu Lakshmi Bai
Important Tips
of Travancore. This eventually opened the north, south
• Mahatma Gandhi joined the Congress in 1915 after and west public roads leading to the temple to all castes.
returning from South Africa, where he had fought for
the rights of Indians using nonviolent resistance or Important Tips
satyagraha The Vaikom Satyagraha was conceived by T. K.
• Mahatma Gandhi served as the president of the Madhavan, an Ezhava leader and a follower of Sri
Indian National Congress only once, in 1924-1925. He Narayana Guru, who first raised the question of temple
presided over the Belgaum session of the Congress, entry of lower castes in an editorial in Deshabhimani
which was the 39th session of the party newspaper in December 1917.

299 PYQ Workbook


HISTORY OF MODERN INDIA

The movement gained prominence and support from stated that if a ruler of a dependent state died without a
different communities and activists across India, such natural heir, the state would be annexed by the British.
as the Akalis of Punjab, Christian and Muslim leaders, This policy was widely resented by the Indian princes and
Chatampi Swamikal and Sree Narayana Guru. contributed to the outbreak of the Indian Rebellion of 1857.

The movement ended on 23rd November 1925 with a Important Tips


compromise between Gandhi and the Regent Maharani, Sikkim did not merge with India until 1975, after a
who released all those arrested and opened three public referendum in which the majority of the people voted for
roads to all castes, but kept the eastern road and the temple joining the Indian Union and abolishing the monarchy.
closed to the lower castes.
The last Chogyal (king) of Sikkim was Palden Thondup
Only in 1936, after the Temple Entry Proclamation by Namgyal, who had ascended to the throne in 1963.
Maharaja Chithira Thirunal Balarama Varma, was access
He faced opposition from pro-democracy movements
to the eastern road and entry into the temple allowed to
led by Kazi Lhendup Dorjee, who became the first Chief
the lower castes.
Minister of Sikkim in 1974.
58. Solution (b) The merger of Sikkim with India was a result of political
Exp) Option b is the correct answer. and social changes in the region, as well as the strategic
interests of India.
Narayan Malhar Joshi, also known as Nana Saheb
Joshi, represented laborers as a representative of the 61. Solution (d)
Indian Government at the Washington Summit of the
Exp) Option d is the correct answer.
International Labour Organization (ILO) in the year
1919. Joshi’s representation at the summit marked India’s “The Rise and Growth of Economic Nationalism in India”
early engagement with international labor organizations and was written by Bipin Chandra. This book provides insights
issues. He was a prominent social and political figure, co- into the economic policies of Indian national leadership
founder of the All-India Trade Union Congress, and played and their role in shaping the Indian economy during
a significant role in labor and political movements in India the period of economic nationalism. It offers a critical
during that time. perspective on the Indian economy and highlights the
contributions of early nationalists in its development. The
59. Solution (a) book is a valuable resource for understanding the economic
Exp) Option a is the correct answer. aspects of India’s struggle for independence and its
consequences.
The correct match for the books and their respective writers
is as follows: 62. Solution (b)
• The Story of My Deportation - Lala Lajpat Rai Exp) Option b is the correct answer.
• Gita Rahasya - Bal Gangadhar Tilak Statement 1 is correct: Lord Curzon was the Viceroy of India
• Nation in Making - Surendranath Banerjee from 1899 to 1905. The partition of the Bengal province
• India Wins Freedom - Maulana Abul Kalam Azad came into force during his viceroyalty on 16th October
1905. According to Curzon, after the partition, the two
Important Tips provinces would be Bengal (including modern West Bengal,
“The Story of My Deportation” is Lala Lajpat Rai’s Odisha, and Bihar) and Eastern Bengal and Assam.
personal account of his exile in Mandalay. Beyond his Statement 2 is correct: The new province of Eastern Bengal
experiences, the book delves into the reasons behind and Assam was created with its capital at Dacca (now
Punjab’s growing discontent in 1907 and reflects on Dhaka). It had a Muslim majority population of about 31
significant events during his time in Mandalay. million, and an area of about 106,000 square miles. It also
“A Nation in Making” is Surendranath Banerjee’s included the hill districts of Assam, Chittagong, and Sylhet.
autobiography, chronicling his life and political
63. Solution (c)
involvement up to 1924. It highlights key events like the
Swadeshi movement, anti-partition protests of Bengal, Option c is the correct answer.
and his role as editor of ‘The Bengalee.’ The book also The Kallar people, primarily associated with the Madurai
recognizes the contributions of other leaders in Bengal’s region, were known for their involvement in various
early nationalist movement. revolts against British India. They were historically known
as warriors and peasants and controlled chieftaincies in the
60. Solution (c)
region. British sources often referred to them as “soldiers
Exp) Option c is the correct answer. out of work.” Kallar chieftaincies-controlled areas north
Lord Dalhousie was the Governor-General of India from and west of Madurai, and they were considered sovereign
1848 to 1856, and he was responsible for the annexation of and independent. Despite attempts by the British East India
Sikkim into the British Empire in 1850. He also annexed Company to pacify them, Kallar raiding parties continued
many other Indian states using the Doctrine of Lapse, which to pose a significant threat, leading to revolts against

PYQ Workbook 300


HISTORY OF MODERN INDIA

British control in 1801. The Kallars were also known Despite Calcutta being the capital of British India,
for working as watchmen in villages, but their practices Lord Lytton, the Governor-General in 1877, selected
and abuses led to tensions and movements against their a location just outside old Delhi for the first Durbar,
authority, resulting in their classification as a “Criminal emphasizing the cultural significance of the former
Tribe” in 1918. Mughal capital to Indians.
64. Solution (c) 65. Solution (c)
Exp) Option c is the correct answer. Exp) Option c is the correct answer.
The correct chronological order of the events is: The term ‘Bombay Trimurti’ refers to the trio of prominent
1. First Anglo-Sikh War (1845-46): The First Anglo-Sikh leaders in Bombay during the late 19th and early 20th
War of 1845-46 saw the Sikh Empire’s partial subjugation, centuries who played significant roles in the Indian political
cession of Jammu & Kashmir to British suzerainty. The and social landscape. The members of the ‘Bombay
war resulted from the Sikh Empire’s internal turmoil Trimurti’ were Sir Pherozshah Mehta, K. T. Telang, and
and British military buildup near Punjab, culminating in Badruddin Tyabji. They were instrumental in advocating
battles like Ferozeshah and Sobraon. Treaty terms led to for Indian rights and reforms in the British colonial era, and
British control over the Sikh Empire’s government. their efforts contributed to the formation of the Bombay
Presidency Association and later, the Indian National
2. Santhal Revolt (1855): The Santhal Rebellion, also
Congress’s provincial committees.
known as Santhal Hool, was a revolt by the Santhal
tribal people against the British East India Company’s 66. Solution (b)
oppressive zamindari system in Bengal Presidency. The
Exp) Option b is the correct answer.
rebellion took place on 30 June 1855 and was led by four
“Annihilation of Caste” is a powerful essay written by
Murmu Brothers – Sidhu, Kanhu, Chand, and Bhairav.
Dr. B. R. Ambedkar in 1936. It was originally intended as
3. Lord Lytton’s Delhi Darbar (1877): In 1876, Queen a speech for a conference organized by the Jat-Pat Todak
Victoria in addition to her title of ‘Queen of Great Britain Mandal, a Hindu reformist group, but the organizers
and Ireland’ assumed the title of ‘Empress of India’. The objected to its content, hence it was later self-published by
Viceroy, Lord Lytton was asked to proclaim this on her Dr. Ambedkar. In this essay, he critiques the Hindu caste
behalf throughout India. He decided to hold an Imperial system, its religious texts, and the need to eradicate the
Assemblage at Delhi on 1st January 1877 for this purpose. religious beliefs that sustain caste divisions.
4. Ilbert Bill Controversy (1883): The Ilbert Bill of 1883
Important Tips
aimed to allow Indian judges to preside over trials
of British and European subjects in session courts. Some of the Books Written by Dr. Bhimrao Ambedkar:
The controversy arose from racial prejudices, with • Castes in India: Their Mechanism, Genesis and
Europeans vehemently opposing the idea of being tried Development, 1916
by Indian judges. A compromise was reached requiring • The Problem of the Rupee: its origin and its solution,
at least half the jury to be European, deepening British- 1923
Indian tensions and paving the way for the Indian • Bahishkrut Bharat (India Ostracized), 1927
National Congress and the freedom struggle.
• Janta (weekly), 1930
Important Tips • The Annihilation of Caste, 1936
Delhi Durbar: • Federation versus Freedom, 1939
The Delhi Durbar was a grand assembly organized by • Thoughts on Pakistan, 1940
the British in Delhi, inspired by Mughal terminology, to
• Ranade, Gandhi and Jinnah, 1943
commemorate the succession of an Emperor or Empress
of India. • Mr. Gandhi and Emancipation of Untouchables, 1945
Occurring thrice during British colonial rule, it took • Pakistan or Partition of India, 1945
place in 1877, 1903, and 1911 when the British Empire • State and Minorities, 1947
was at its zenith. • Who were the Shudras, 1948
Each Durbar marked a significant coronation event: the • The Untouchables, 1948
first proclaimed Queen Victoria as the Empress of India
• Buddha or Karl Max, 1956
with the title “Kaisar-i-Hind.”
• The Buddha and his Dhamma, 1957
The second confirmed King Edward VII as the Emperor
of India, while the third celebrated King George V’s • What Congress and Gandhi have done to the
ascension. Untouchables,1945
King George V was the sole monarch to personally 67. Solution (b)
attend a Durbar. Exp) Option b is the correct answer.

301 PYQ Workbook


HISTORY OF MODERN INDIA

On August 15, 1947, Mahatma Gandhi didn’t join the to appeal for a new dominion status and a federal set-up of
Independence Day celebrations. Instead, he was in Kolkata, government for the constitution of India. It also proposed for
where violent clashes between Hindus and Muslims had torn the Joint Electorates with reservation of seats for minorities
apart the city. Gandhi was deeply saddened by the communal in the legislatures. It was prepared by a committee chaired
violence and disheartened that the principles of non- by Motilal Nehru, with Jawaharlal Nehru acting as the
violence (ahimsa) and peaceful resistance (satyagraha), secretary.
which he had dedicated his life to, seemed to have failed Pair b is correctly matched: The Champaran Satyagraha
in bringing people together during these turbulent times. of 1917, led by Mahatma Gandhi, was a significant farmer’s
However, Gandhi worked tirelessly to persuade both Hindus uprising in Bihar, India, against the oppressive indigo
and Muslims to abandon violence. His presence in Kolkata planters. It marked the beginning of popular satyagraha
played a crucial role in improving the situation, and as a and influenced India’s freedom struggle, bridging the gap
result, the coming of independence was celebrated with between moderates and radicals in the fight against British
joyous communal harmony and dancing in the streets. colonial rule.
68. Solution (b) Pair c is correctly matched: The Forward Bloc, a political
Exp) Option b is the correct answer. party formed by Subhas Chandra Bose in 1939, aimed
to unite left-wing sections within the Indian National
M.K. Gandhi led the freedom movement against British
Congress. It emphasized an alternative leadership within the
colonial rule using nonviolent methods. He used to say that
Congress and involved prominent leaders like S.S. Kavishar,
“Wrong means never take us to right ends”. He believed
Lal Shankarlal, and Vishwambhar Dayalu Tripathi. Bose’s
that the means and ends are inseparable and that immoral
efforts included publishing the newspaper “Forward Bloc”
means cannot achieve moral ends.
and nationwide rallies.
Mahatma Gandhi used to say that “means and ends are two
Pair d is incorrectly matched: The Khilafat movement
sides of a coin. They cannot be separated. Immoral means
(1919–22) was a political campaign in British India led by
cannot be used to achieve moral ends. If used, they will
leaders like Shaukat Ali, Maulana Mohammad Ali Jauhar,
vitiate the end itself. Wrong means never take us to right
Hakim Ajmal Khan, and Abul Kalam Azad, with Mahatma
ends”. He believed that if we take care of our means, the end
Gandhi’s support. It protested British policies regarding
will be taken care of itself. He also said that “the cruelest
Turkey and sought to redress Turkey’s grievances. This
form of violence is persistence of poverty”.
movement also aimed to foster Hindu-Muslim unity and
69. Solution (c) ended in 1922 alongside the non-cooperation movement.
Exp) Option c is the correct answer. M.A Jinnah opposed the Khilafat movement.
Mahatma Gandhi was a strong supporter of the principle 72. Solution (c)
that “What is morally wrong can never be politically right.” Exp) Option c is the correct answer.
This reflects his unwavering commitment to nonviolence,
truth, and ethical conduct in both personal and political Lord Mountbatten was the last Viceroy of British India and
spheres. Gandhi believed that political actions and decisions the first Governor-General of independent India. He sent a
should be guided by moral values and that compromising on letter to Gandhi on August 26, 1947, in which he praised
morality for political gain was unacceptable. This principle him as a “One-man Boundary Force” for his efforts to stop
was central to his philosophy of satyagraha (nonviolent the communal violence in Bengal.
resistance) and his approach to social and political change. He wrote:
“My dear Gandhiji, in the Punjab we have 55 thousand
70. Solution (b)
soldiers and large-scale rioting on our hands. In Bengal
Exp) Option b is the correct answer. our forces consist of one man, and there is no rioting. As a
The book “The Problems of the Far East” was written serving officer, as well as an administrator, may I be allowed
by George Nathaniel Curzon. Published in 1894, the to pay my tribute to the One-man Boundary Force”.
book delves into the political, social, and economic
conditions of Japan, Korea, and China within the changing 73. Solution (a)
international landscape of the time. Curzon, later the Exp) Option a is the correct answer.
Viceroy of India, was intrigued by the cultural heritage of The All-Parties Muslim Conference held at Delhi on 31st
the Far East but remained a supporter of British imperialism. December 1928, was presided over by Sir Sultan Mohamed
The book aimed to provide a deeper understanding of these Shah, also known as Aga Khan III with Fazal Rahimtoola
regions rather than a typical travel narrative. It also explored being its secretary. He played a prominent role in the
the future prospects of the entire region. conference, which brought together representatives from
71. Solution (d) various parts of India to discuss Muslim political issues and
demands.
Exp) Option d is the correct answer.
Pair a is correctly matched: The Nehru Report of 1928 was 74. Solution (c)
a memorandum of All Parties Conference in British India Exp) Option c is the correct answer.

PYQ Workbook 302


HISTORY OF MODERN INDIA

In “Hind Swaraj,” Mahatma Gandhi referred to the British Lord Louis Francis Albert Victor Nicholas Mountbatten
Parliament as “sterile and prostitute.” He criticized the was the first Governor-General of free India. He was
Parliament for its ineffectiveness and lack of genuine appointed by King George VI on the recommendation of
concern for the well-being of the people. Gandhi believed British Prime Minister Clement Attlee. He was in charge of
that the Parliament was controlled by ministers and subject ending the British rule in India and he put forward the plan
to frequent changes in leadership, making it incapable of of Partition on June 3, 1947.
meaningful action without external pressure. He used strong
language to convey his disappointment with the British Important Tips
political system. First Governor of Bengal- Robert Clive (1757-1760 and
1765-1767)
75. Solution (a)
First Governer- General of Bengal- Warren Hasting
Exp) Option a is the correct answer.
(1773-1785)
Bhagat Singh is not associated with “The Indian
First Governor-General of India- Lord William Bentick
Sociologist.” “The Indian Sociologist” was an influential
(1828-1835)
Indian nationalist journal in the early 20th century,
edited by Shyamji Krishna Verma. It advocated for Indian First Viceroy of India- Lord Canning (1856-1862)
independence from British rule, criticized injustice and Last Governor-General of India- Lord Mountbatten
brutality, and was influenced by Herbert Spencer’s ideas. The (1947-1948)
journal became more radical over time and faced bans and First and Last Indian Governor-General of Independent
legal actions. India- C. Rajagopalachari (1948-1950)
Important Tips
79. Solution (d)
Published in early January 1930, ‘The Philosophy of
Exp) Option d is the correct answer.
the Bomb’ by Bhagwati Charan Vohra was a polemic
addressing debates within nationalist circles regarding Gopal Krishna Gokhale was an Indian political leader and
violence’s role in India’s anti-colonial struggle, defending social reformer who mentored Mahatma Gandhi during his
its significance. early years in India. He had commanded Gandhi to spend
“Dreamland” is a book in verse written by Lala Ram the first year in India ‘with his ears open but his mouth
Saran Das during his imprisonment in the early 20th shut’ when Gandhi returned from South Africa in 1915. He
century. It explores themes like revolution, God, religion, wanted Gandhi to observe and understand the conditions
violence, non-violence, spiritualism, literature, and poetry. and problems of the Indian people before launching any
Introduction of this book was written by Bhagat Singh. political campaigns.

76. Solution (a) 80. Solution (d)


Exp) Option a is the correct answer. Exp) Option d is the correct answer
The biographer of Jawaharlal Nehru is Frank Moraes. Dadabhai Naoroji was the first Indian to be appointed as a
Frank Moraes was an accomplished journalist and writer professor of Mathematics and Physics at Elphinstone College,
who had a notable career with The Times of India and other Bombay. Hence, statement 1 is correct.
publications. In addition to his journalistic work, he also
In 1886, Naoroji campaigned as a Liberal Party candidate for
wrote a biography of Jawaharlal Nehru, providing insights
the strongly Conservative seat of Holborn. In 1888, referring
into the life and career of India’s first Prime Minister.
to Naoroji’s defeat, the Conservative Party Prime Minister,
77. Solution (d) Lord Salisbury, remarked that an English constituency was
Exp) Option d is the correct answer. not ready to elect a ‘Blackman’, drawing greater notoriety to
In a speech given in 1942, Subhash Chandra Bose said, Naoroji. In 1892, he contested the seat of Central Finsbury,
“India’s salvation will not be achieved under the leadership campaigning on Gladstone’s platform of Liberalism, and was
of Mahatma Gandhi. Gandhi’s methods are outdated and successfully elected. Hence, statement 2 is correct
ineffective. We need to adopt a more radical approach if we Dadabhai Naroji had also been founder-editor of the journal
want to achieve our goal of independence.” Bose criticized Rast Goftar in Bombay in 1851. Hence, statement 3 is
Mahatma Gandhi’s leadership and non-violent methods, correct.
suggesting that they were no longer effective in achieving
Dadabhai Naroji presided over Congress three times. He was
India’s independence. He advocated for a more radical and
assertive approach, eventually leading to his formation of the Congress president during 1886, 1893, and 1906 sessions.
the Indian National Army (INA) and collaboration with Axis Hence, statement 4 is not correct.
powers during World War II to pursue India’s liberation. Important Tips
78. Solution (a) Dadabhai Naroji founded the London Zoroastrian
Exp) Option a is the correct answer. Association in 1861.

303 PYQ Workbook


HISTORY OF MODERN INDIA

He was also a founding member of the East India with Lord Wellesley in 1801. He agreed to surrender half
Association and London Indian Society and became vocal of his territory, including Rohilkhand, Gorakhpur, and
in promoting Indian rights in regard to the ICS and trade. the Doab, to the British, and to pay a huge subsidy for
maintaining a British force in his state.
He was an economist and proponent of the drain theory,
building up a detailed economic critique of British Sindhia: The Maratha chief Sindhia signed a subsidiary
imperialism in India. alliance with Lord Wellesley in 1803. He agreed to cede
some of his territories, such as Delhi, Agra, and parts of
He also established links with Irish MPs and was one of
Bundelkhand, to the British, and to pay a large indemnity for
the founders of the Indian National Congress in 1885 in
the expenses of the war.
Bombay.
In 1906, Dadabhai Naoroji stood as a candidate at Lambeth 83. Solution (d)
North but was again unsuccessful. Exp) Option d is the correct answer.
In 1907, Dadabhai Naoroji left England to retire at Versova The Lucknow session of 1916 was presided by Ambika
in Bombay, where he died in 1917. Charan Majumdar. He was an Indian Bengali politician who
served as the Indian National Congress president.
81. Solution (c)
Exp) Option c is the correct answer. Important Tips

Sir John Shore was the Governor-General of Bengal from Features of Lucknow Pact:
1793 to 1798, and he was an expert in the Bengal revenue The Lucknow Pact was an agreement between the Indian
system. However, he was not involved in the Anglo-Nepal National Congress and the Muslim League.
War, which was fought between the Kingdom of Nepal and The Lucknow Pact of 1916 was signed between Bal
the British East India Company from 1814 to 1816. The Gangadhar Tilak and Muhammad Ali Jinnah.
Anglo-Nepal War was initiated by Lord Hastings, who was The League agreed to present joint constitutional
the Governor-General of India from 1813 to 1823. Lord demands with the Congress to the government.
Hastings was also responsible for the Third Anglo-Maratha
The Congress accepted the Muslim League’s position on
War, which ended the Maratha Confederacy as a major
separate electorates.
power in India.
They made some joint demands that were:
Important Tips
Government should declare that it would confer self-
Some major wars and related governor general of British government on Indians at an early date.
India:
The representative assemblies at the central as well as
• Third Mysore War (1790-92) and Treaty of provincial level should be further expanded with an
Seringapatam (1792): Lord Cornwallis (1786-1793) elected majority and more powers given to them.
• First Burmese War (1824-1826): Lord Amherst (1823- The term of the legislative council should be five years.
1828)
The salaries of the Secretary of State for India should be
• First Afghan War (1838-42): Lord Auckland (1836- paid by the British treasury and not drawn from Indian
1842) funds.
• First Anglo-Sikh War (1845-46): Lord Hardinge I Half the members of the viceroy’s and provincial
(1844-1848) governors’ executive councils should be Indians.
• Bhutan War (1865): Lord John Lawrence (1864-1869)
84. Solution (a)
82. Solution (a) Exp) Option a is the correct answer.
Exp) Option a is the correct answer. Pair a is correctly matched: The book “Introduction to
The correct chronological order of their treaties are as the Dreamland” authored by Bhagat Singh discusses his
follows: thoughts on revolution, religion, and war. It emphasizes
Hyderabad: In 1798, it broke the Nizam’s ties with the the importance of criticism and independent thinking for
French and made it illegal for the Nizam to form alliances revolutionaries, criticizes religion for siding with tyrannical
with the Maratha without British permission. The Nizam of forces, and discusses the role of war in achieving social
Hyderabad became the first to sign the Subsidiary Alliance. change. Bhagat Singh believed that capturing power was
Mysore: After 1799, a small state of Mysore was created necessary to establish a revolutionary government focused
by Wellesley consisting of Mysore, Srirangapatna and the on mass education and constructive work.
surrounding areas. It was given to Krishnaraja Wadiyar III Pair b is incorrectly matched: “Bandi Jivan,” an
and the rest was annexed to British India. For doing this, the autobiographical account by Sachindranath Sanyal,
Wadiyar dynasty had to sign the Subsidiary Alliance. portrays his life of captivity and his role in India’s freedom
Oudh: The Nawab of Oudh entered into a subsidiary alliance struggle. It highlights revolutionary activities, imprisonment,
and the groundwork by patriots during the First World War,

PYQ Workbook 304


HISTORY OF MODERN INDIA

inspiring generations. Denunciation of Nationalism: Tagore strongly denounced


Pair c is incorrectly matched: “The Indian Struggle, 1920– nationalism as one of humanity’s greatest problems.
1942” is a two-part book by Netaji Subhas Chandra Bose, He viewed it as a mechanical and selfish endeavor,
covering the Indian independence movement from 1920 to emphasizing the need for purposeful education over blind
1942. Published in 1935 and 1942, it analyzes key events and revolution.
Bose’s vision for an independent India, including his critical Vision of East-West Unity: During his extensive travels,
views on Gandhi’s approach. Tagore developed a vision of East-West unity, striving for a
Pair d is incorrectly matched: “Why I Am an Atheist” is harmonious coexistence of cultures and nations.
an essay written by Bhagat Singh in 1930 in response to Support for Indian Independence: Despite his
his religious friends’ beliefs that his atheism stemmed from reservations, Tagore wrote songs in support of the Indian
vanity. In the essay, he explains his journey to atheism and independence movement and renounced his knighthood
questions the existence of God and the suffering in the world. in protest against the Amritsar massacre in 1919.

85. Solution (d) Conflict Resolution: He played a pivotal role in resolving


disputes within the Indian independence movement,
Exp) Option d is the correct answer.
including the issue of separate electorates for untouchables,
The memoirs titled ‘Bapu: My Mother’ were written by mediating between Gandhi and Bhimrao Ramji Ambedkar.
Manubehn, Gandhi’s grandniece. The book recounts Educational Reforms: Tagore criticized the rote-oriented
various incidents with Mahatma Gandhi, shedding light educational system introduced by the British Raj in
on his character and activities. Manu, as she is known, India. His experimental school at Santiniketan focused
was Gandhiji’s constant companion during his last mission on holistic education and became a magnet for scholars,
for Hindu-Muslim Peace in Noakhali, East Bengal, from artists, and musicians.
September 1946 until his assassination on January 30, 1948.
87. Solution (a)
86. Solution (c)
Option a is the correct answer.
Exp) Option c is the correct answer.
Option A in List-I matches Option 2 in List-II: The
Rabindranath Tagore, a celebrated poet and philosopher, 51st session of the Indian National Congress was held
did not create songs or works that refute the heroism of in Haripura village, Gujarat, in 1938, presided over by
the Marathas. In fact, he composed a poem titled “Shivaji Subhash Chandra Bose. The Congress aimed to peacefully
Utsav” in 1904, which celebrated the valiant Maratha resist the undemocratic Federal Scheme of the Government
warrior-king, Chhatrapati Shivaji. This poem highlighted of India Act, 1935, and demanded complete independence
Shivaji’s efforts to unify India and resist foreign occupation, for India, including princely states, along with civil liberties
particularly the Mughals. Tagore used Shivaji’s bravery and and political prisoners’ release. Bose won the election for the
determination as a source of inspiration for the freedom
next Congress session’s president.
struggle against British colonial rule. Tagore’s literary
contributions consistently emphasized heroism and the Option B in List-I matches Option 3 in List-II: Sardar
collective effort needed to combat oppression, including Vallabhbhai Patel played a crucial role in Operation Polo, the
British colonial rule in India. military action to integrate the princely state of Hyderabad
into India. Despite opposition from Nehru and Rajaji, Patel’s
Important Tips determination and pressure led to the successful operation
Rabindranath Tagore’s political views: in September 1948, ending Nizam’s rule and ensuring
Rabindranath Tagore’s political views were characterized Hyderabad’s accession to India.
by a complex and ambivalent stance, reflecting the Option C in List-I matches Option 4 in List-II: Muhammad
nuanced political landscape of his time: Iqbal delivered his most popular inaugural speech, known
Critique of Swadeshi Movement: He shunned the Swadeshi as the Allahabad Address, during the 25th Session of the
movement, criticizing it in his 1925 essay “The Cult of All-India Muslim League in Allahabad in 1930. In this,
the Charkha,” aligning this with elements of Gandhism he campaigned for the establishment of a Muslim state in
and the Non-Cooperation Movement. In response to north-western India.
the British proposal to partition Bengal in 1904, Tagore Option D in List-I matches Option 1 in List-II: On April
proposed an alternative solution in his lecture “Swadeshi 8, 1929, revolutionaries Bhagat Singh and Batukeshwar
Samaj,” advocating a self-help-based reorganization Dutt threw non-lethal bombs in the Central Assembly in
of rural Bengal. However, he was supportive of some Delhi. They aimed to protest British imperialism and raise
economic aspects of the Swadeshi movement, such as the
awareness about the freedom struggle. Both willingly courted
promotion of indigenous industries, but he advocated
arrest and were later sentenced to transportation for life.
for a holistic, long-term development approach rather
than endorsing short-term methods like boycotting 88. Solution (c)
foreign goods and burning foreign clothes. Exp) Option c is the correct answer.

305 PYQ Workbook


HISTORY OF MODERN INDIA

Lord Wellesley was the Governor-General of India from • Under this system, those farmers who had 1 bigha land
1798 to 1805, and he expanded the British influence and (1bigha=20 kattha) had to plant Indigo over 3 kattha.
territory in India through the Subsidiary Alliance System, If you have 2 bigha land you must have to plant Indigo
which required Indian rulers to accept a British resident over 6 kattha land.
and troops in their states and surrender their foreign policy • The price of Indigo initially was high in the market
to the British. He also initiated the Fourth Anglo-Mysore but planters who were European usually paid very low
War in 1799, which resulted in the defeat and death of Tipu prices to farmers.
Sultan, the ruler of Mysore, and the annexation of most of his • Moreover, the seed and initial cost used to be provided
kingdom by the British. by European only. It means farmers had to pay more in
Important Tips return they get less.

• Lord Ellenborough was the Governor-General of • They were all debt-ridden with less fertile land. This
India from 1842 to 1844, and he was responsible for gave birth to several revolts and movements. Indigo
the annexation of Sindh in 1843. revolt(1859) and Champaran Satyagraha(1917) are the
famous ones.
• Lord Dalhousie was the Governor-General of India
from 1848 to 1856, and he was responsible for the 91. Solution (b)
annexation of Awadh in 1856. He also introduced the Exp) Option b is the correct answer.
Doctrine of Lapse, which allowed the British to annex
Dr. B.R. Ambedkar, also known as Baba Saheb, was the
any Indian state whose ruler died without a natural
first Law Minister of independent India. He was an Indian
heir.
scholar, economist, politician, and social reformer. He was
• Sir John Shore was the Governor-General of Bengal
the chairman of the drafting committee and widely regarded
from 1793 to 1798. The Third Anglo-Maratha War,
as the father of Indian Constitution. Apart From that, he
was fought between 1817 and 1819. The Third Anglo-
is the first Indian to get a doctorate degree in Economics
Maratha War was initiated by Lord Hastings, who was
from abroad. In recognition of his contributions, he was
the Governor-General of India from 1813 to 1823.
posthumously awarded the Bharat Ratna in 1990.
89. Solution (b) Important Tips
Exp) Option b is the correct answer. Portfolio held in the first cabinet of Independent India:
Lord Cornwallis was the Governor-General of Bengal Jawaharlal Nehru: Prime Minister; External Affairs and
from 1786 to 1793, and he was associated with the Third Commonwealth Relations; Scientific Research.
Anglo-Mysore War, which took place from 1790 to 1792. Sardar Vallabhbhai Patel: Home; Information and
He declared war on Tipu Sultan, the ruler of Mysore, after Broadcasting; States.
Tipu attacked the British ally of Travancore in 1789. He also Dr Rajendra Prasad: Food and agriculture.
formed a coalition with the Maratha Empire and the Nizam
Maulana Abul Kalam Azad: Education.
of Hyderabad to oppose Tipu’s expansionist ambitions. He
led the British forces in several battles against Tipu, such as Dr John Matthai: Railways and Transport.
the Siege of Bangalore, the Battle of Arakere, and the Battle Sardar baldev Singh: Defence.
of Seringapatam. He eventually forced Tipu to sue for peace Shri Jagjivan Ram: Labour.
and sign the Treaty of Seringapatam in 1792, which resulted Mr C.H. Bhabha: Commerce.
in the loss of half of Mysore’s territory and a large war Mr Rafi Ahmad Kidwai: Communications.
indemnity to the British and their allies.
Rajkumari Amrit Kaur: Health.
90. Solution (a) Dr B.R Ambedkar: Law.
Exp) Option a is the correct answer. Shri R.K. Shanmukham Chetty: Finance.
Tinkathia System was an economic policy enforced by the Dr. Syama Prasad Mookerji: Industries and Supplies.
East India Company in India, especially in Bihar, where the Shri N. V. Gadgit: Works, Mines and Power.
peasants had to grow indigo on three out of every twenty
92. Solution (b)
parts of their land for the landlords. This system was
challenged by the Champaran Satyagraha led by Mahatma Exp) Option b is the correct answer.
Gandhi in 1917, which was the first Satyagraha movement in G.K. Gokhale was a prominent Indian nationalist leader
India and a milestone in the Indian independence movement. and a moderate member of the Indian National Congress.
He compared Lord Curzon’s administration in India to that
Important Tips of Aurangzeb, the Mughal emperor who was known for his
• British introduced the Indigo plantation in Bengal oppressive and intolerant policies towards the non-Muslims.
(Bihar was part of it ) in the 1820s. Gokhale’s comparison was meant to highlight the similarities

PYQ Workbook 306


HISTORY OF MODERN INDIA

between Curzon’s administration and Aurangzeb’s reign Pair b is incorrectly matched: India from Curzon to Nehru
in terms of their tyranny, intolerance, exploitation, and and After is a book by Durga Das, first published in 1969.
alienation of the Indian people. He also wanted to warn It is a comprehensive history of India from the late 19th
the British government that such policies would lead to century to the early 1960s, covering the period from the
discontentment, rebellion, and eventual downfall of their viceroyalty of Lord Curzon to the premiership of Jawaharlal
empire in India. Nehru.
Pair c is correctly matched: Frank Moraes’s Jawaharlal
93. Solution (a)
Nehru: A Biography is a comprehensive and well-
Exp) Option a is the correct answer. researched account of the life and times of India’s first
The Indian National Congress, which was established in Prime Minister. The book was first published in 1956.
1885. It was divided into two groups in 1907 mainly by Moraes had known Nehru personally for many years and
extremists and moderates at the Surat session of Congress. had access to a wide range of sources, including Nehru’s own
Moderates were people who wanted to resolve issues with papers and interviews with his family and friends. The book
the government through deliberations whereas Extremists is based on this extensive research and provides a detailed
believed in strikes, agitation, and boycott to force their account of Nehru’s life, from his childhood to his death.
demands. The session was held on the bank of the Tapti river Pair d is incorrectly matched: “India Divided” by Rajendra
in Surat in 1907 Prasad is a significant work that examines the issues
Important Tips surrounding the partition of India in 1947. Prasad, a
prominent figure in the Indian independence movement,
INC Session Chairman Year offers insights into the historical events and complexities
Surat Session Rash Bihari Ghosh 1907 of this crucial period, shedding light on the challenges and
Calcutta Session Shri Dadabhai Naoroji 1906 consequences of partition.

Benares Session Gopal Krishn Gokhale 1905 96. Solution (a)


Madras Session Rash Bihari Ghosh 1908 Exp) Option a is the correct answer.
“The Springing Tiger” is a historical account of the
94. Solution (a) Indian National Army written by Colonel Hugh Toye and
Exp) Option a is the correct answer. published in 1959. This book provides one of the early
The conquest of Sindh by the British was completed sympathetic Western perspectives on the INA. Toye, who
during the period of Lord Ellenborough, who was the worked as an intelligence officer in Burma during World War
Governor-General of India from 1842 to 1844. He was a II, interrogated INA soldiers. The book covers the formation
strong advocate of British expansionism and imperialism in of the INA, its role in key battles, and the alleged death of
India and pursued an aggressive policy against the rulers of Subhas Chandra Bose. It is a valuable historical resource on
Sindh, the Talpur dynasty. this aspect of India’s struggle for independence.

Important Tips 97. Solution (d)


• Lord Ellenborough ordered Sir Charles Napier to lead Exp) Option d is the correct answer.
a military campaign against the Amirs of Sindh in In a historic address at the AICC Session in Lahore on
1843, which resulted in two major battles: the Battle of December 29, 1929, Jawaharlal Nehru made the statement,
Miani and the Battle of Hyderabad. “I am a socialist and a republican, and I do not believe in
• The British forces defeated the Sindhi forces in both Kings and Princes.” This statement reflected his ideological
battles and annexed Sindh into the British Raj. stance as a socialist and his opposition to monarchy and
feudalism. Nehru emphasized the need for India to adopt
• Lord Ellenborough celebrated this victory by sending
a socialist philosophy to address poverty, inequality, and
back to England the gates of Somnath temple, which
labor rights. He also advocated for land reform and the
had been looted by Mahmud of Ghazni in 1026.
empowerment of the peasantry. This address laid the
• This act was seen as a symbol of British triumph over foundation for Nehru’s vision of a democratic and socialist
Muslim invaders and a restoration of Hindu pride. India.
95. Solution (c) 98. Solution (d)
Exp) Option c is the correct answer. Exp) Option d is the correct answer.
Pair a is incorrectly matched: “The Life of Mahatma Lord Ripon is considered as the father of local self-
Gandhi” authored by Louis Fischer, provides a balanced government in India, as he was the first British ruler to
and insightful exploration of Gandhi’s life, beliefs, and recognize the importance of empowering the local people
impact. It dispels myths, highlights Gandhi’s spiritual to manage their own affairs. He believed that local self-
growth, and emphasizes his profound influence on social government would foster a sense of responsibility and loyalty
and political change. Fischer’s work is a crucial starting point among the Indians, and would also reduce the burden on the
for understanding Gandhi and his philosophy. central administration.

307 PYQ Workbook


HISTORY OF MODERN INDIA

Important Tips 103. Solution (c)

• Lord Ripon enacted several laws and regulations to Exp) Option c is the correct answer.
promote local self-government, such as the Municipal Rabindra Nath Tagore was a poet and Nobel laureate
Act of 1882, which gave more powers and functions who bestowed the title of “Mahatma” to Gandhi in 1915
to the municipal corporations and councils, and while writing his autobiography. The word “Mahatma”
increased the representation of Indians in them. means a great soul or venerable in Sanskrit and is now used
• He encouraged the formation of district boards, taluk throughout the world to refer to Gandhi.
boards, and village panchayats, which were responsible Important Tips
for local development works such as roads, schools,
Gandhi was born on 2 October 1869 in Porbandar, Gujarat
hospitals, sanitation, etc.
and died on 30 January 1948 in New Delhi by assassination
• He supported the participation of Indians in legislative
He adopted the short dhoti woven with hand-spun yarn as
councils and civil services.
a mark of identification with India’s rural poor and lived
99. Solution (b) in a self-sufficient residential community.
Exp) Option b is the correct answer. He was the president of the Indian National Congress in
1924-1925 and was named Time Person of the Year in
The Anglo-Nepal War took place during the reign of Lord
1930.
Hastings. He was the Governor-General of the East India
Company from 1813 to 1823. He declared war on Nepal in 104. Solution (c)
April 1814, after a series of border disputes and territorial
Exp) Option c is the correct answer.
encroachments by the Nepalese Gurkhas. The war lasted for
two years and ended with the Treaty of Sugauli in March The book “The Man Who Divided India” was authored by
1816, which forced Nepal to cede some of its lands to the Rafiq Zakaria. It discusses the political career of Muhammad
British. Ali Jinnah, who played a key role in the partition of India.
It explores how Jinnah began as a messenger of Hindu-
100. Solution (c) Muslim unity but later adopted a communalist approach
Exp) Option c is the correct answer. to divide India on religious lines. The book critiques the
failure of Congress leaders like Nehru and Patel to counter
The first communist government in India was established
communalism effectively.
in the province of Kerala. E. M. S. Namboodiripad became
the first communist leader in India to head a popularly Important Tips
elected government following the elections for the Kerala “Freedom at Midnight” by Larry Collins and Dominique
Legislative Assembly in 1957. Lapierre is a non-fiction book that chronicles the events
101. Solution (b) surrounding the Indian independence movement and
partition during the last year of the British Raj, from
Exp) Option b is the correct answer. 1947 to 1948. It delves into various aspects, including
Mahatma Gandhi had told, “Destruction is the best the reactions of princely states, the partition itself,
method of dealing with the foreign clothes.” He said this the assassination of Mahatma Gandhi, and the lives of
in the context of his campaign for swadeshi or self-reliance, prominent leaders like Jawaharlal Nehru and Muhammad
which involved boycotting foreign goods and promoting Ali Jinnah.
indigenous industries. He encouraged people to burn or “India from Curzon to Nehru and After” by Durga Das
discard their foreign clothes and wear khadi or hand-spun offers a dynamic account of politics and international
cloth instead. He believed that this would not only weaken affairs, featuring insightful portraits of influential
the economic and political hold of the British over India, but leaders like Gandhi, Jinnah, Nehru, Desai, and Patel. It
also foster a sense of national pride and dignity among the emphasizes that politics is fundamentally about people
Indians. and is a valuable resource for those interested in India,
British imperialism history, or contemporary Asia.
102. Solution (d)
Exp) Option d is the correct answer. 105. Solution (a)
M.S. Subbulakshmi sang ‘Hindustan Hamara’ of Iqbal Exp) Option a is the correct answer.
and ‘Jan-gan-man’ in the Central Assembly at midnight Lord Curzon was the Viceroy of India from 1899 to 1905, and
of 14/15 August 1947. “Hindustan Hamara” is a poem he established the Archaeological Survey of India (ASI)
written by Muhammad Iqbal, a prominent Muslim poet and as a separate department in 1902. He also appointed John
philosopher expressing the love of India and its people Marshall as the first Director-General of ASI, who reformed
whereas “Jan-gan-man” is the national anthem of India. It and modernized the archaeological practices in India.
was composed by Rabindranath Tagore, a Nobel laureate
poet and philosopher. The anthem expresses the hopes and 106. Solution (a)
aspirations of the people of India. Exp) Option a is the correct answer.

PYQ Workbook 308


HISTORY OF MODERN INDIA

Matching the authors with their respective works: Gandhi’s fast had a profound impact on both the workers
Bankim Chandra Chatterjee - Anand Math and the mill owners. The workers remained united and
disciplined, while the mill owners felt guilty and ashamed.
Michael Madhusudan Dutt - Captive Lady
On the fourth day of his fast, the mill owners agreed
Rabindranath Tagore - Gora to refer the matter to a tribunal, which awarded a 35%
increase in wages to the workers.
Sarojini Naidu - The Broken Wing

Important Tips 108. Solution (d)

Anand Math (1882): “Anandamath” by Bankim Chandra Exp) Option d is the correct answer.
Chattopadhyay is a historical Bengali novel set during The correct sequence is- Rajendra Prasad (1950-1962), S.
the 1770 Bengal famine and sanyasi rebellion. It follows Radhakrishnan (1962-1967), Zakir Hussain (1967-1969), V.
the journey of a couple, Mahendra and Kalyani, who get V. Giri (1969-1974).
involved with rebel monks fighting against British rule.
Rajendra Prasad was India’s first President and initially
The novel introduces the iconic song “Vande Mataram,”
served as the Minister of Food and Agriculture. He took
a symbol of India’s struggle for freedom.
office on January 26, 1950.
Captive Lady (1849): “Captive Lady” by Michael
Sarvapalli Radhakrishnan was the first Vice President and
Madhusudan Dutt is an English poem that narrates the
love story of Prithviraj and a princess confined by King the second President of India. Teachers’ Day, celebrated
Jayachandra. It marks Dutt’s early foray into Bengali on September 5th, honors his birthday. He assumed the
literature. presidency on May 13, 1962.
Gora (1910): “Gora” by Rabindranath Tagore is a novel Zakir Husain, an economist and co-founder of Jamia Millia
set in 1880s Calcutta during the British Raj. It delves into Islamia, was India’s third President and previously the
themes of politics, religion, liberation, caste, nationalism, second Vice President. He assumed the presidency on May
and love through the intertwined stories of two couples, 13, 1967.
Gora and Sucharita, and Binoy and Lolita. V.V. Giri was an Indian politician who served as the fourth
The Broken Wing (1917): Sarojini Naidu’s poem “The President of India. Before his presidency, he held important
Broken Wing” symbolically portrays her unwavering positions in the Indian government, including serving as the
commitment to India’s independence despite personal Vice President of India. He assumed office as President on
health issues and the communal strife of the time. It August 24, 1969.
reflects her resolute spirit and patriotism.
109. Solution (a)
107. Solution (c) Exp) Option a is the correct answer.
Exp) Option c is the correct answer. Among the following option, Lord Curzon was the Viceroy
Mahatma Gandhi made the first use of hunger strike as a of India from 1899 to 1905 with his six years tenure.
However, the longest serving viceroy of India was Lord
weapon in the Ahmedabad Strike of 1918. He fasted for
Linlithgow, who served from 1936 to 1943 for seven years.
three days to support the workers of a textile mill who were
Lord Dufferin was viceroy of India from 1884 to 1888, Lord
on strike for higher wages. This was the first time Gandhi Hardinge was governor-general of India from 1844 to 1848
used fasting as a form of protest in India. and Lord Mayo was the viceroy of India from 1869 to 1872.

Important Tips Important Tips


The Ahmedabad Strike was triggered by the mill owners’ Curzon-Kitchener controversy:
decision to withdraw the plague bonus that was given • It was a major conflict between the Viceroy of India,
to the workers during the epidemic of 1917. The workers Lord Curzon, and the Commander-in-Chief of the
demanded a 50% increase in wages to cope with the British Indian Army, Lord Kitchener, in the early 20th
wartime inflation, but the mill owners were only willing to century.
offer a 20% increase. • The Curzon-Kitchener controversy arose from a
Gandhi advised the workers to go on a peaceful strike disagreement over the status and role of the military
and demand a 35% increase in wages, which he considered member of the Viceroy’s executive council, who was
reasonable and fair. responsible for advising the Viceroy on military
The strike lasted for 21 days, during which Gandhi matters and controlling the army supply and logistics.
tried to negotiate with the mill owners, but they refused • Lord Kitchener wanted to abolish this office and bring
to budge. He then decided to go on a fast unto death to all the military affairs under his own authority, while
strengthen the workers’ resolve and to put moral pressure Lord Curzon opposed this idea and defended the
on the mill owners. existing system of dual control.

309 PYQ Workbook


HISTORY OF MODERN INDIA

• The controversy ended with the resignation of Lord independence has begun…Father of our Nation! In this holy
Curzon in August 1905, after he failed to secure the war of India’s liberation, we ask for your blessings and good
support of the home government for his position. Lord wishes.”
Kitchener emerged as the victor of the conflict, and He did not agree with Gandhi’s emphasis on non-violence
he succeeded in abolishing the office of the military and swadeshi. He said, “We cannot sit still in a state of
member and reorganizing the army department passive waiting for someone else to bring us deliverance.”
according to his wishes. Despite their differences, Bose respected Gandhi and
acknowledged his influence on the Indian people. He also
110. Solution (a) sought his blessings and guidance on various occasions. He
Exp) Option a is the correct answer. addressed him as the Father of the Nation, not only to honor
“Unhappy India” authored by Lala Lajpat Rai, not Bal his role in the freedom struggle, but also to appeal to his
Gangadhar Tilak, written in response to Katherine Mayo’s moral authority and to unite the nation under a common
“Mother India,” was a powerful critique of colonial rule. cause.
Rai dismantled Mayo’s White Supremacist arguments,
114. Solution (d)
highlighting how British imperialism was the root cause of
India’s socio-economic issues. His book played a vital role in Exp) Option d is the correct answer
countering Mayo’s narrative and bolstering India’s demand The founders of the Muslim League were: Khwaja Salimullah
for self-rule. khan, Vikar-ul-Mulk, Syed Amir Ali, Syed Nabiullah, Khan
Bahadur Ghulam and Mustafa Chowdhury.
Important Tips
“Durgeshnandini” is a Bengali historical romance novel Important Tips
authored by Bankim Chandra Chattopadhyay in 1865. All India Muslim League was formed in Dhaka in
It is the first major Bengali novel in the history of Bengali Bangladesh.
literature, depicting a love triangle against the backdrop Sir Syed Ahmed Khan, the founder of the Aligarh Muslim
of Pathan-Mughal conflicts during Akbar’s reign in the University was of the belief that Muslims constituted as a
south-western region of modern-day West Bengal. The separate nation. The party followed this philosophy.
story was inspired by local legends of the Arambag region.
It was India’s first Muslim party
111. Solution (a) The party was dissolved on 14th August, 1947.
Exp) Option a is the correct answer.
115. Solution (b)
The book “Mountbatten and the Partition of India” was
Exp) Option b is the correct answer.
written by Dominique Lapierre and Larry Collins. It is a
compilation of interviews, personal reports, and documents N.G Ranga was an Indian freedom fighter, parliamentarian,
related to Lord Mountbatten’s role in the partition of India. It and a leader of the peasant movement. He was also a critic of
sheds light on the challenges and decisions that Mountbatten Mahatma Gandhi and his methods of non-violence and civil
faced, making it a valuable resource for understanding the disobedience. He opposed the Champaran Satyagraha of
complex process of India’s partition and its far-reaching Mahatma Gandhi, which was a peaceful protest against the
consequences. exploitation of the peasants by the European indigo planters
in Bihar in 1917. He argued that Gandhi’s approach was too
112. Solution (a) passive and ineffective, and that the peasants needed more
Exp) Option a is the correct answer. radical and militant action to achieve their rights.
Queen Victoria’s Proclamation was read out by Lord 116. Solution (a)
Canning, the first Viceroy of India, on November 1, 1858,
Exp) Option a is the correct answer.
at Allahabad. The proclamation announced the transfer of
power from the East India Company to the British Crown The book “Mother India” was written by American
after the Indian Rebellion of 1857. The proclamation also historian Katherine Mayo in 1927. This polemical book
promised to respect the rights and dignity of the Indian was critical of various aspects of Indian society, including
princes and people, and to refrain from interfering in their its treatment of women, untouchables, animals, and its
religious beliefs and practices. The proclamation was seen nationalistic politicians. It was written in opposition to
as a gesture of conciliation and goodwill by the British the Indian independence movement and sparked outrage
government, but it also asserted the supremacy of the British in India. The book was considered a scurrilous libel against
crown over India Hindus and Hinduism, leading to widespread protests and
the burning of the book.
113. Solution (d)
117. Solution (a)
Exp) Option d is the correct answer.
Exp) Option a is the correct answer.
Subhash Chandra Bose was the first person to call Mahatma
Gandhi as the ‘Father of the Nation’ in his radio broadcast The policy of ‘Security cell’ or the policy of siege was a
from Singapore on July 6, 1944. He said, “India’s last war of strategy adopted by Warren Hastings and Wellesley to

PYQ Workbook 310


HISTORY OF MODERN INDIA

create a buffer zone around the British territories in India Important Tips
by making alliances with or subjugating the neighbouring
Differences in Viewpoints between Gandhi and Nehru:
Indian states. The policy aimed to protect the British
Approach to Non-violence:
interests from the threats of the Afghans and the Marathas,
as well as to expand their influence and revenue. Gandhi: Embraced non-violence (Ahimsa) as a core
principle, considering it the ultimate weapon for achieving
Important Tips social and political change.
The policy of ‘Security cell’ was one of the British policies Nehru: While appreciating non-violence’s moral aspects,
towards Indian states that were implemented during he didn’t view it as a practical method in all situations
different periods of colonial rule. The other policies were: and believed in the need for coercive authority to maintain
• Subordinate separation policy (1813-1857 AD): This law and order.
policy aimed to reduce the power and influence of the Economic Vision:
Indian states by annexing them through various means,
Gandhi: Advocated for small-scale and cottage industries
such as the doctrine of lapse, subsidiary alliance, or
like Khadi to address poverty and unemployment,
alleged misrule. Some examples of this policy were
emphasizing self-sufficiency and rural development.
the annexation of Sindh, Punjab, Nagpur, Jhansi, and
Awadh Nehru: Supported heavy industrialization and large-
scale industries, aiming for rapid modernization and
• Subordinate union policy (1858-1935 AD): This
technological advancement, even if it meant embracing
policy sought to appease and co-opt the Indian states
Western science and technology.
by granting them some privileges and recognising their
rights and titles. Some examples of this policy were the Role of Religion:
Queen’s Proclamation of 1858, the Chamber of Princes Gandhi: Integrated religion and morality into politics,
in 1921, and the Indian States Agency in 1936 emphasizing truth (Satya) and non-violence (Ahimsa) as
• The policy of equal union (1935-1947 AD): This the foundation of life. He sought moral regeneration.
policy envisaged a federal structure for India, Nehru: Promoted secularism and scientific thinking,
where the Indian states would have equal status separating religion from politics. He believed the Congress
and representation with the provinces in a central should focus on political and economic spheres.
legislature and executive. However, this policy was Freedom and Equality:
never fully implemented due to various reasons, such
Gandhi: Focused on liberty and envisioned a stateless
as the outbreak of World War II, the rise of nationalism,
society with unrestricted individual freedom and minimal
and the demand for independence
state interference.
118. Solution (b) Nehru: Emphasized equality, advocating for the
rationing of freedom to ensure equitable distribution
Exp) Option b is the correct answer.
among all members of society. He believed in a strong state
The statement “Everything else can wait, but not to achieve socialist goals.
agriculture” is attributed to Jawaharlal Nehru, India’s first
Western Influence:
Prime Minister. This quote reflects Nehru’s recognition of
Gandhi: Opposed Western civilization, seeking to banish
the critical importance of agriculture in the development
it from India while retaining Britishers as friends in
and food security of India. He made this statement during a service of the country.
time of food shortages in the aftermath of India’s partition
Nehru: Wanted to drive out British colonial rule but
in 1947. Nehru emphasized the need to prioritize agricultural
appreciated Western culture and civilization.
development to address the pressing issue of food scarcity
Role of State:
and ensure the well-being of the Indian population.
Gandhi: Preferred a passive role for the state in his
119. Solution (d) vision of autonomous village republics.
Exp) Option c is the correct answer. Nehru: Advocated for state intervention and control,
especially in economic planning, to achieve socialist
Nehru favored heavy industrialization as a means to
objectives.
modernize India and believed in the importance of building
a strong industrial base for economic development. In 120. Solution (b)
contrast, Gandhi was critical of heavy industrialization, Exp) Option b is the correct answer.
emphasizing self-reliance, cottage industries, and rural The book “Anandmath,” written by Bankim Chandra
development. Gandhi’s focus was on rural empowerment Chattopadhyay, is associated with the rise of the national
and sustainability, while Nehru advocated for rapid movement in India. It was also known as the “Bible of
industrial growth and urbanization to propel India forward Bengali Patriotism.” This novel, published in 1882, played
as an industrialized nation. a significant role in inspiring and mobilizing Indians during

311 PYQ Workbook


HISTORY OF MODERN INDIA

the struggle for independence. It introduced the iconic Classes Welfare Institute (or Bahishkrit Hitakarini Sabha)
song “Vande Mataram,” which later became a rallying was launched by Dr. B.R Ambedkar in 1922.
cry for freedom fighters and was adopted as the national Statement 4 is incorrect: The first defense minister of
song of India after independence. “Anandmath” depicted independent India was Baldev Singh Chokkar, who served in
the Sannyasi Rebellion against the British East India Prime Minister Jawaharlal Nehru’s cabinet during 1947–52.
Company in the late 18th century and celebrated the spirit
of patriotism and sacrifice. Its English version is titled “The Note: As per Official Key, answer is A but only statement
Abbey of Bliss.” 1 and 2 is correct and Statement 3 and 4 are incorrect.
Dr.Babasaheb Ambedkar established “Bahishkrit
121. Solution (c) Hitkarini Sabaha (Depressed Class Institute) “ on 20 July
Exp) Option c is the correct answer. 1924 at Bombay, not in 1922.
The Bardoli Satyagraha was not led by Mahatma Gandhi, Important Tips
but by his close associate Vallabhbhai Patel, who later
Facts related to B.R Ambedkar:
became the first deputy prime minister of India. The Bardoli
Satyagraha was a peasant movement that took place in 1928 • He represented the depressed classes at the First Round
in Bardoli taluka of Gujarat. It was a protest against the Table Conference in London in 1930.
British government’s decision to increase the land revenue • In 1936, he established the Independent Labour
by 30% despite the failure of crops due to floods and famine. Party (ILP), which won 13 out of 15 reserved seats in
Patel mobilized the peasants to withhold their payments and Bombay’s provincial elections.
demand a reduction of the revenue. He also organized mass • The ILP transformed into the All India Scheduled
meetings, petitions, and boycotts to resist the government’s Castes Federation in 1942 but faced defeat in the 1946
attempts to seize their lands and properties. The movement general elections.
lasted for six months and ended with the victory of the
peasants. The government agreed to cancel the revenue hike • Ambedkar opposed civil servants’ strikes, viewing
and return all the confiscated lands and properties. them as breaches of service contracts.
• He served as a member of the Viceroy’s Executive
Important Tips
Council as a Labour member from 1942 to 1946.
Quit India Movement: This was a movement launched by
• Ambedkar initiated the weekly newspaper ‘Mooknayak’
Mahatma Gandhi in 1942 to demand an immediate end of
in January 1920 to advocate for the rights of the
the British rule in India and to prepare for a mass struggle
depressed classes.
for freedom. The movement involved various forms of
civil disobedience, strikes, boycotts, and sabotage by the • Some of his famous books include-The Untouchable:
Indian people. Who are They and Why They Have Become
Untouchables; Buddha and His Dhamma; ‘The Rise and
Civil Disobedience: This was a movement initiated by
Fall of Hindu Women’, ‘Emancipation of Untouchables’,
Mahatma Gandhi in 1930 with the famous Dandi March
‘The Evolution of Provincial Finance in British India’;
or Salt Satyagraha, in which he defied the British salt
‘Pakistan or Partition of India‘, ‘Thoughts on Linguistic
monopoly by making salt from seawater.
States’, etc.
Kheda Satyagraha: This was a peasant movement led by
• Ambedkar became the first Law Minister of Independent
Mahatma Gandhi in 1918 in Kheda district of Gujarat. It
India but resigned in 1951 due to disagreements over
was a resistance against the British government’s decision
the Hindu Code Bill with Nehru.
to collect full land revenue despite the failure of crops due
to drought and famine. • He chaired the Constituent Assembly’s Drafting
Committee and is regarded as the “father of the Indian
122. Solution (a) Constitution.”
Exp) Option a is the correct answer. • Despite a defeat in the 1952 Lok Sabha election, he
Statement 1 is correct: Siddharth College of Law was founded became a member of the Rajya Sabha from Maharashtra.
in Mumbai in 1956. It is run by the People’s Education • On October 14, 1956, he converted to Buddhism, and
Society which was formed by Dr. Babasaheb Ambedkar on he passed away on December 6, 1956, in Delhi. His
8 July 1945. followers renamed the Scheduled Caste Federation as
Statement 2 is correct: Mooknayak (The Chief of the the Republican Party of India in 1957.
Voiceless) was a fortnightly Marathi newspaper published
123. Solution (c)
on alternate Saturdays from Bombay that began in 1920 by
Babasaheb Ambedkar. It was his first endeavor as a journalist. Exp) Option c is the correct answer.
The newspaper aimed to reveal the pain and rebellion of Lord Curzon was the Viceroy of India from 1899 to 1905,
society. The first issue of Mooknayak was published on 31 and he passed the Ancient Monuments Preservation Act
January 1920. in 1904. This Act was a landmark legislation that aimed
Statement 3 is correct: In order to bring about a new socio- to regulate the preservation and protection of ancient
political awareness among the untouchables, the Depressed monuments and objects of archaeological, historical, or

PYQ Workbook 312


HISTORY OF MODERN INDIA

artistic interest in India. He initiated several projects to Important Tips


repair and renovate some of the famous monuments, such as
Lucknow Session, 1916: This was the session where
the Taj Mahal, the Qutub Minar, and the Ajanta Caves.
the Indian National Congress and the All-India Muslim
Important Tips League reached an agreement known as the Lucknow
Pact, which aimed to secure constitutional reforms and
Ancient Monuments Preservation Act: communal harmony. The session also marked the reunion
• The Act defined an ancient monument as any structure, of the moderate and extremist factions of the Congress.
erection, or monument, or any tumulus or place of The session was presided by Ambica Charan Mazumdar.
interment, or any cave, rock-sculpture, inscription Amritsar Session, 1919: This was the session where the
or monolith, which is of historical, archaeological or Indian National Congress condemned the Jallianwala
artistic interest, or any remains thereof. Bagh massacre and the martial law imposed in Punjab
• The Act gave the British government the power by the British. The session also rejected the Montague-
to declare any ancient monument as a protected Chelmsford Reforms as inadequate and disappointing,
monument, and to acquire rights in or guardianship of and endorsed the non-cooperation movement initiated
such monument from its owner or custodian. by Mahatma Gandhi. The session was presided by Motilal
• The Act also provided for the preservation of ancient Nehru.
monuments by agreement with their owners, and for Nagpur Session, 1920: This was the session where the
the compulsory purchase of ancient monuments in Indian National Congress adopted some important
certain cases. organizational changes, such as setting up a Congress
• The Act also authorized the government to control Working Committee, forming provincial committees
the traffic in antiquities and the moving of sculptures, on linguistic basis, and reducing the entry fee. The
carvings or like objects. session also confirmed the non-cooperation movement
• The Act also empowered the government to notify as its main programme and changed its goal from self-
areas as protected for archaeological excavation, and government to swaraj. The session was presided by C.
to make rules for regulating such excavation1 Vijayaraghavachariar

126. Solution (c)


124. Solution (a)
Exp) Option c is the correct answer.
Exp) Option a is the correct answer.
Lord William Bentinck was the Governor-General of India
Kavalam Madhava Panikkar, popularly known as Sardar
from 1828 to 1835, and he took over the administration
K. M. Panikkar, said “the most notable achievement of
of Mysore state in 1831 on the basis of alleged
the British Rule was the unification of India”. He wrote
maladministration by the ruler, Maharaja Krishnaraja
it in his book “India and the Indian Ocean”. Panikkar
III. He claimed that the Maharaja had failed to protect his
argued that before the British came, India was a fragmented
subjects from the oppression and extortion of his officials,
land of many kingdoms and princely states. The British
brought these different regions together under one rule, and that he had neglected the public works and education in
and they also introduced modern ideas of administration his state. He also accused him of being hostile to the British
and governance. This helped to create a sense of national and of conspiring with their enemies, such as the Marathas
identity among Indians, which was essential for the and the Pindaris. He therefore deposed the Maharaja and
country’s independence movement. placed him under British supervision in a palace in Mysore
city. He also appointed a British commissioner to manage
125. Solution (b) the affairs of the state, and introduced several reforms in the
Exp) Option b is the correct answer. administration, revenue, judiciary, and education sectors.
Mahatma Gandhi attended the Calcutta session of Indian
127. Solution (c)
National Congress in the year 1901 for the first time. He
Exp) Option c is the correct answer.
was then a young lawyer who had recently returned from
South Africa. He was impressed by the speeches of Gopal The renowned poet Sir Muhammad Iqbal wrote the
Krishna Gokhale, Dadabhai Naoroji, and others at the famous song “Saare Jahan Se Achcha Hindustan Hamara”
session. He also met some of the prominent leaders of the in 1905. This song, often referred to as “Tarana-e-Hindi,” is
Congress, such as Bal Gangadhar Tilak, Bipin Chandra Pal, a patriotic ghazal in Urdu. It played a significant role as a
and Lala Lajpat Rai. He later wrote in his autobiography, symbol of resistance against British rule during the Indian
“The Congress session in Calcutta left a deep impression on independence movement. This anthem celebrates India and
my mind and enhanced my estimation of the Congress.” The advocates for unity among people from diverse communities.
Calcutta session of 1901 was presided by Dinshaw E. Wacha, It remains a popular patriotic song in India and is informally
a Parsi leader and a close associate of Dadabhai Naoroji. regarded as the nation’s anthem.

313 PYQ Workbook


HISTORY OF MODERN INDIA

Important Tips • Two years later he fled British India and found refuge
Muhammad Iqbal, often referred to as the “Poet of in the French colony of Pondichéry (Puducherry),
the East,” made significant contributions to literature in where he devoted himself for the rest of his life to the
multiple languages, primarily Persian, Urdu, and English. development of his “integral” yoga with an aim of a
literary works of Muhammad Iqbal: fulfilled and spiritually transformed life on earth.

Persian Works: • He established the Sri Aurobindo Ashram in


Pondichéry in 1926.
“Asrar-i-Khudi” (Secrets of the Self ) in 1915, focused
on the concept of “Khudi” or “Self ” from a religious • He believed that the basic principles of matter, life,
perspective. and mind would be succeeded through terrestrial
evolution by the principle of supermind as an
“Rumuz-i-Bekhudi” (Hints of Selflessness) in 1917
intermediate power between the two spheres of the
emphasized the Islamic way of life and self-sacrifice for
infinite and the finite.
the nation.
“Payam-e-Mashriq” (The Message of the East) in 1924, • He authored various literary works, including “Bande
inspired by Goethe, highlighted the importance of Mataram,” “Bhagavad Gita and Its Message,” “The
spirituality and morality in the face of materialism. Future Evolution of Man,” “Rebirth and Karma,” and
“Savitri: A Legend and a Symbol.”
Urdu Works:
“Bang-e-Dara” (The Call of the Marching Bell) in 1924 129. Solution (c)
contained patriotic and nature-themed poems. Exp) Option c is the correct answer.
“Bal-e-Jibril” (Wings of Gabriel) in 1935, inspired by Lord Mayo was the Viceroy of India from 1869 to 1872,
Spain, conveyed strong religious passion. and he was murdered by a convict named Sher Ali Afridi
“Zarb-i-Kalim” (The Rod of Moses) in 1936, serving as his on 8 February 1872, when he was visiting the Andaman and
political manifesto, critiqued materialism and injustice. Nicobar Islands.
English Works:
Important Tips
“The Development of Metaphysics in Persia” in 1908
• Sher Ali Afridi had served in the colonial army and
explored Persian ideology and Islamic Sufism.
police, and had killed one of his enemies in Peshawar in
“The Reconstruction of Religious Thought in Islam” in 1867. He was sentenced to death, but later commuted
1930 addressed philosophy, spirituality, and political and to life imprisonment and deported to the Andaman
social problems. and Nicobar Islands.
Punjabi Works:
• He worked as a barber at Port Blair, and waited for an
Wrote poems like “Piyaara Jedi” and “Baba Bakri Wala” opportunity to kill a white official as a revenge for his
in Punjabi. sentence.
128. Solution (b) • He chose Lord Mayo as his target, and attacked him
Exp) Option b is the correct answer. when he was returning to his boat after inspecting the
prison facilities. Lord Mayo died shortly after from his
The quote, “Political freedom is the life-breath of a nation.
wounds.
To attempt social reform, educational reform, industrial
expansion, the moral improvement of the race without • Sher Ali Afridi was immediately arrested and confessed
aiming first and foremost at political freedom, is the very to his crime. He was hanged on 11 March 1872
height of ignorance and futility,” is attributed to Aurobindo
130. Solution (b)
Ghosh. This quote underscores the fundamental importance
of political freedom as the foundation upon which other Exp) Option b is the correct answer.
reforms and improvements in society, education, industry, Pair a is incorrectly matched: “Hind Swaraj or Indian
and morality can be built. Ghosh’s belief in the primacy of Home Rule,” authored by Mohandas K. Gandhi in 1909,
political freedom was a central tenet of his philosophy and explores his perspectives on Swaraj, modern civilization,
his commitment to India’s struggle for independence. mechanization, and more. Gandhi rejects European
civilization while upholding moral empire ideals. The
Important Tips
British government banned the book in 1910 for sedition.
Aurobindo Ghose:
Pair b is correctly matched: “New India” was a newspaper
• Aurobindo Ghose, born in Calcutta on August 15, 1872, founded by the Indian freedom fighter Dr. Annie Besant in
was a multifaceted figure: yogi, seer, philosopher, June 1914 to spread socialists’ views among common people.
poet, and Indian nationalist.
Pair c is incorrectly matched: In January 1914, Annie
• From 1902 to 1910 he partook in the struggle to free Besant established the weekly newspaper Commonweal
India from the British. As a result of his political with the aim of promoting and disseminating her ideas of
activities, he was imprisoned in 1908 (Alipore Bomb constitutional agitation, with the goal of compelling the
case). British to grant Home Rule to the Indian population.

PYQ Workbook 314


HISTORY OF MODERN INDIA

Pair d is incorrectly matched: “India Wins Freedom” by Quit India Movement, during which JP played a significant
Maulana Azad offers an insightful account of the partition, role, despite being in prison at the time. JP was a prominent
blending personal experiences with Indian independence. political leader and activist in India known for his
The book emphasizes politics over religion as the cause of contributions to the independence movement and his role in
partition and laments India’s post-independence missed various socio-political movements in the country.
opportunities.
134. Solution (a)
131. Solution (a) Exp) Option a is the correct answer.
Exp) Option a is the correct answer. The correct matching of authors with their works is as
Moderate and Extremist split in Surat Session 1907 and follows:
reunited in Lucknow Session in 1916 due to efforts of Amritlal Nagar - Amrit aur Vish
Annie Besant and Bal Gangadhar Tilak. Annie Besant was Sumitra Nandan Pant - Chidambara
a prominent British socialist, Theosophist, women’s rights
Sharat Chandra Chatterjee - Devdas
activist. She also started Home Rule Movement in India in
1916. These two factions formally reconciled during the Jaidev - Geet Govind
Lucknow session, 1916. Important Tips
Important Tips Amrit Aur Vish is a novel by Amritlal Nagar, first
Various factors facilitated reunion of Extremists and the published in 1966. It is one of Nagar’s most famous
Moderates: works and won him the Sahitya Akademi Award in 1967.
The novel depicts the life of a village, spanning from the
Old controversies had become meaningless after end of
late 19th century to the mid-20th century. It shows the
Swadeshi Movement.
changing shape of rural society and the transformations
Both the Moderates and the Extremists realised that the taking place within it.
split had led to political inactivity.
‘Chidambara’ is Sumitranandan Pant’s poetry collection
To allay Moderate suspicions, Tilak had declared that he that earned him the Jnanpith Award in 1968. It reflects his
supported a reform of administration and not an overthrow poetic evolution from ‘Yugvani’ to ‘Atima,’ encompassing
of the government. He also denounced acts of violence. diverse compositions representing the physical and
The death of Pherozeshah Mehta, who had led the spiritual aspects of his era. Pant aimed to create a holistic
Moderate opposition to the Extremists, facilitated the humanity by harmonizing different philosophies and life
reunion. elements in his poetry.

132. Solution (d) “Devdas” is a Bengali romance novel by Sarat Chandra


Chattopadhyay, revolving around a tragic love triangle
Exp) Option d is the correct answer. between Devdas, his childhood love Paro, and the reformed
The right to adopt heir in place of own son was re- courtesan Chandramukhi.
established by the Queen’s announcement of 1858. This The Gita Govinda is a 12th-century work by Hindu poet
announcement was made after the Indian Rebellion of 1857, Jayadeva, portraying the love between Krishna, Radha,
which resulted in the dissolution of the East India Company and gopis in Vrindavan. It consists of twelve chapters with
and the transfer of power to the British Crown. The Queen’s twenty-four divisions called prabandhas, featuring eight-
announcement declared that the British government would couplet ashtapadis. These verses have influenced Indian
respect the rights, dignity, and religious beliefs of all Indians, classical dances and are sung nightly in the Jagannatha
and that no native of India would be disqualified from temple.
holding any office or place of trust by reason of his religion,
descent, or colour. 135. Solution (c)
The announcement also stated that “all acts and proceedings Exp) Option c is the correct answer.
in courts of justice within our said territories and possessions Option A in List-I matches Option 2 in List-II: The All-
shall be done in our name” and that “all treaties made by India Azad Muslim Conference, founded in 1929 by Allah
the said Company shall be binding upon us as fully and Bakhsh Soomro, aimed to promote composite nationalism
effectually as if they had been made by us” and a united India, opposing the partition and two-nation
theory advocated by the All-India Muslim League.
133. Solution (c)
Option B in List-I matches Option 3 in List-II: The
Exp) Option c is the correct answer. Khaksar movement, a social movement based in Lahore,
The book “Towards Struggle” was written by Jai Prakash Punjab, British India, founded by Inayatullah Khan Mashriqi
Narayan (JP). This book contains a selection of his (Allama Mashriqi) in 1931, aimed to free India from British
manifestos, speeches, and other writings, providing insights rule, opposed partition, and advocated for a united, inclusive
into his ideas and program of work. It was published on India, welcoming members regardless of religion, race, caste,
August 9th, coinciding with the fourth anniversary of the or social status, emphasizing the brotherhood of mankind.

315 PYQ Workbook


HISTORY OF MODERN INDIA

Option C in List-I matches Option 1 in List-II: The Khudai “India Divided” by Rajendra Prasad is a significant work
Khidmatgar, led by Abdul Ghaffar Khan, was a Pashtun written in 1946, addressing the pressing issue of India’s
nonviolent resistance movement against British rule, focusing partition. Prasad argues against the idea of Hindus and
on social reform and later becoming political. It joined Muslims being two separate nations, advocating instead for
the Congress Party, played a role in India’s independence a secular state with cultural autonomy for diverse groups.
movement, but faced opposition from the Muslim League. He critiques various partition schemes, emphasizing their
They opposed partition and demanded Pashtunistan’s impracticality and offering an alternative solution to the
independence but faced resistance. Hindu-Muslim question.
Option D in List-I matches Option 4 in List-II: The Krishak
138. Solution (a)
Sramik Party, formed in 1929, formerly known as Nikhil
Banga Praja Samiti and Krishak Praja Party, was a significant Exp) Option a is the correct answer.
anti-feudal political party in British Bengal and later in The prefix ‘Mahatma’ was added with the name of Gandhi
East Bengal and East Pakistan. Led by A. K. Fazlul Huq, it by Rabindranath Tagore, a Nobel laureate poet and a friend
represented tenant farmers and played a key role in Bengali of Gandhi, during the Champaran Satyagraha in 1917. The
Muslim political awareness and Hindu support. It produced Champaran Satyagraha was the first Satyagraha movement
notable leaders like Abdus Sattar and Abu Hussain Sarkar. led by Gandhi in India. It was a farmer’s uprising in Bihar
against the oppressive Tinkathiya system imposed by the
136. Solution (d)
British planters, who forced them to grow indigo and sell it
Exp) Option d is the correct answer. at a low price.
The name of the magazine published by Mahatma Gandhi
139. Solution (c)
during his stay in South Africa was Indian Opinion. He
launched the newspaper in 1903 to educate the European Exp) Option c is the correct answer.
community in South Africa about the plight of Indian Pair a is incorrectly matched: “Ailan-i-Haq” or “Anal Haq”
immigrants and to fight against racial discrimination and authored by Vaikom Muhammad Basheer tells the story of
injustice. Sufi Mansur Al-Hallaj, who faced execution for proclaiming
“Anā al-Ḥaqq” (I am the Truth). Despite condemnation, he
Important Tips
faced death with unwavering conviction, emphasizing his
Navjeevan: This is a Hindi newspaper published by The spiritual union with the divine.
Associated Journals Ltd, which also publishes National
Pair b is incorrectly matched: Al-Hilal, an Urdu
Herald and Qaumi Awaz. The newspaper was founded in
newspaper by Maulana Abul Kalam Azad (1912-1914),
1947 by Mahatma Gandhi and Jawaharlal Nehru to promote
criticized British rule, urged Muslim participation in India’s
the principles of the Indian independence movement and
modern India. independence movement, and emphasized Hindu-Muslim
unity. Its impact led to greater Muslim engagement in the
India Gazette: This was an English language weekly
independence struggle, influencing leaders like Gandhi
newspaper published in Calcutta, then the capital of
and Nehru, though it was eventually shut down by British
British India. It was the second newspaper printed in
authorities in 1914.
India, after the Bengal Gazette. It was founded in 1780
by Peter Reed, a former editor of the Bengal Gazette, and Pair c is correctly matched: Tehzeeb-ul-Ikhlaq is a journal
continued until 1834 established by the Muslim reformer Sir Syed Ahmad
Khan in 1871. The journal published alternative Muslim
137. Solution (c) perspectives, written in plain language. It gave voice to the
Exp) Option c is the correct answer. publisher’s religious, social, and reforming opinions, and is
The correct match for the books and their respective writers credited with establishing him as one of the fathers of Urdu
is as follows: fiction.

Mahatma Gandhi - Hind Swaraj Pair d is incorrectly matched: Jugantar Patrika,


founded in 1906 in Calcutta by Barindra Kumar Ghosh,
Ram Manohar Lohia - The Wheel of History
Abhinash Bhattacharya, and Bhupendranath Dutt, was
Dr. Rajendra Prasad - India Divided a revolutionary Bengali newspaper associated with the
Abul Kalam Azad - India Wins Freedom Anushilan Samiti. It advocated violent resistance against
British rule, criticized the Indian National Congress, and
Important Tips
had a significant readership before shutting down in 1908
“Wheel of History” by Rammanohar Lohia compiles a due to legal challenges.
collection of insightful lectures. Lohia’s unique style delves
into topics such as purpose, materialist interpretation of 140. Solution (c)
history, class, caste, continental shifts, and the essence Exp) Option c is the correct answer.
of modern civilization, providing readers with thought- The Banaras Session of Indian National Congress, 1905 was
provoking perspectives. presided by Gopal Krishna Gokhale. Formal Proclamation

PYQ Workbook 316


HISTORY OF MODERN INDIA

of Swadeshi Movement against the British Government was envisaged a sovereign status for the state and was
done in this session. The Indian National Congress decided reluctant to accede to either of the dominions. As he
to take up the Swadeshi Call in its Benaras Session in 1905. procrastinated, the newly established state of Pakistan
sent its forces behind a front of tribal militia and moved
141. Solution (c)
menacingly towards Srinagar. It was now that the prince
Exp) Option c is the correct answer. was forced to sign an Instrument of Accession (October
Mahatma Gandhi was joined by several leaders in his 1947) with the Indian Union.
Champaran Satyagraha, which was his first civil disobedience
movement in India against the exploitation of the peasants 143. Solution (a)
by the European indigo planters. Some of the prominent Exp) Option a is the correct answer
leaders who participated in the Champaran Satyagraha with Nizam’s Dominion, Hyderabad, chose not to become a part
Gandhi were: of free India. With the departure of the British from India,
• Rajendra Prasad, who later became the first president the special relationship of paramountcy the British had with
of India. He played a key role in inviting Gandhi to Hyderabad also had to end. Governor-General Lord Louis
Champaran and mobilizing the local people for the Mountbatten signed a Standstill Agreement with the Nizam
movement. on November 29, 1947. It was agreed that the arrangements
• Anugraha Narain Sinha, who was a lawyer, politician relating to external affairs, defence and communications
and freedom fighter. He accompanied Gandhi in his will be handled by India in the same way as by the British
investigations and negotiations with the authorities and Crown. The Nizam was allowed to appoint agent-generals
the planters. in Delhi and London. The Standstill Agreement was to
• Brajkishore Prasad, who was another reputed lawyer maintain the status quo till a permanent solution to the
who fought cases for the tenants. He was one of the first future of Hyderabad was found. The relations between India
persons to contact Gandhi and persuade him to visit and Hyderabad soured and Hyderabad was integrated with
Champaran. the Union of India barely a year of the Standstill Agreement.
• J.B. Kripalani, who was a prominent educator, social Important Tips
activist and politician. He was a close associate of Gandhi Two key documents designed to be attractive to the
and a longtime supporter of his ideology. He joined rulers of princely states were. The first was the standstill
Gandhi in Champaran along with other eminent lawyers Agreement and the second was the Instrument of
and helped him mobilize the peasants for the Satyagraha. Accession.
• Raj Kumar Shukla, who was a farmer from Champaran The Standstill Agreement which confirmed that the
who was suffering from the oppression of the indigo practices and agreements that existed as between the
planters. He met Gandhi at the Lucknow session of the princely states and British India would be now continued
Indian National Congress in 1916 and persuaded him by independent India.
to visit Champaran and see the plight of the indigo
The Instrument of Accession by which the ruler of the
cultivators
princely states agreed to the accession of his kingdom to
Therefore, out of the given options, only Rajendra Prasad independent India.
and Anugraha Narain Sinha were among those who joined
Mahatma Gandhi during the Champaran Satyagraha. 144. Solution (d)
Exp) Option d is the correct answer.
142. Solution (b)
John Lawrence was the Governor-General of India from
Exp) Option b is the correct answer
1864 to 1869, and he initiated the policy of Masterly
By 15 August 1947 almost all princely states had acceded to
Inactivity. This policy meant that the British should not
India except for the three state of Junagarh, Hyderabad and
interfere in the internal affairs of Afghanistan and other
Jammu and Kashmir.
neighboring countries, and that there was no need to keep
1. Junagarh The Muslim Nawab wanted to join Pakistan but a British representative at their courts. He believed that
a Hindu majority population wanted to join the Indian this would avoid unnecessary conflicts and expenses, and
Union. In the face of repressive attitude of the nawab, that the British should focus on consolidating their power
there was a plebiscite which decided in favour of India. and developing their territories in India. He also adopted
2. Hyderabad Hyderabad wanted a sovereign status. It a cautious and conciliatory approach towards the Indian
signed a Standstill Agreement with India in November princes, and respected their rights and dignity.
1947. The violence by razakkars and supply of foreign
arms prompted Indian troops to move in 1948— 145. Solution (b)
described as “a police action to restore law and order”. Exp) Option b is the correct answer.
Hyderabad acceded in November 1949. “Malgudi Days” is a collection of short stories written by
3. Kashmir The state of Jammu and Kashmir had a Hindu R. K. Narayan and published in 1943 by Indian Thought
prince and a Muslim majority population. The prince Publications. The book includes 32 stories, all set in the

317 PYQ Workbook


HISTORY OF MODERN INDIA

fictional town of Malgudi, located in South India. Each of • He appointed Indian judges, known as munsifs
the stories portrays a facet of life in Malgudi. R. K. Narayan’s and amins, to preside over the district courts, while
writing in this collection is known for its insightful and European judges were appointed to the higher courts.
humorous depiction of everyday life in India, making it a
• He introduced a uniform code of laws for civil cases,
classic in Indian literature. The stories cover a wide range
based on Hindu and Muslim laws, and compiled by a
of themes and characters, offering readers a glimpse into
committee of pandits and qazis.
the complexities of human nature and society.
• He also reformed the criminal justice system by
146. Solution (b) establishing six circuit courts, each headed by a British
Exp) Option b is the correct answer. judge and assisted by an Indian law officer, to try
The twin principles of Mahatma Gandhi’s Ram Rajya were serious criminal cases.
truth and non-violence. This is consistent with Gandhi’s • He abolished the judicial powers of the zamindars
philosophy of satyagraha, which means holding on to truth (landlords) and transferred them to the Company’s
or the force of truth. Gandhi believed that truth and non- courts
violence are the highest values and the foundation of an
148. Solution (d)
ideal society. He said, “Truth is God and non-violence is the
means of realizing Him”. He also said, “Non-violence is the Exp) Option d is the correct answer.
law of our species as violence is the law of the brute”. Gandhi Calcutta was the capital of British India from 1772 to 1911.
practiced and preached truth and non-violence throughout On 12 December 1911, King George V proclaimed the
his life, and inspired millions of people to follow his path. transfer of the capital from Calcutta to Delhi at the climax of
the 1911 Imperial Durbar. The decision was taken during the
Important Tips Reign of Lord Harding, who was the Viceroy of India.
Ram Rajya of Gandhi is the name given to the ideal form
of governance that Mahatma Gandhi envisioned for India. 149. Solution (b)
Gandhi’s Ram Rajya implied a radical transformation of Exp) Option b is the correct answer.
the social, economic, and political structures of India. Pair 1 is incorrectly matched: Jawaharlal Nehru was the
He advocated for a decentralized and democratic system second Individual Satyagrahi on the call of Mahatma
of self-governance, where the people would have equal Gandhi. In reaction to the August Offer, Gandhiji called for
rights and opportunities, and where the state would be a limited ‘Individual Satyagraha’. It was a protest against the
minimal and non-coercive. British government and their inability to accommodate the
demands of Indians. Also, Indians wanted to demonstrate
He also envisioned a society where there would be
that though they are taking part in the war, the participation
no exploitation, oppression, or violence, and where
everyone would live in harmony with nature and each was not voluntary. Vinoba Bhave was the first individual
other. Satayagrahi.

He said, “Ramarajya of my dream ensures equal rights Pair 2 is correctly matched: Chittaranjan Das was called
alike of prince and pauper.” Deshabandhu (Friend of the Nation) because of his selfless
service to the nation and his dedication to the cause of
147. Solution (b) Indian independence. He was a lawyer, politician, and
Exp) Option b is the correct answer. social reformer who played a leading role in the Indian
independence movement.
Lord Cornwallis is credited with establishing the judicial
organization in India. He introduced significant reforms in Pair 3 is incorrectly matched: The 1907 Surat Session of
the Indian judicial system during his tenure as Governor- the Indian National Congress witnessed a significant split
between Moderates and Extremists. Dr. Rash Bihari Ghosh’s
General of India from 1786 to 1793. These reforms were
election as President, contrary to Extremists’ preference
aimed at making the judicial system more efficient, impartial,
for Lala Lajpat Rai, led to Extremists leaving the Congress,
and accessible to the people.
marking a crucial ideological divide.
Important Tips Pair 4 is incorrectly matched: The Paris Indian Society
Lord Cornwallis introduced the following changes in (India House in Paris) was an Indian nationalist organisation
the judicial system: founded in 1905 at Paris under the patronage of Madam
• He separated the executive and the judiciary functions Bhikaji Rustom Cama, Munchershah Burjorji Godrej and
of the district collectors, who were previously S. R. Rana. The organisation was opened as a branch of the
responsible for both revenue collection and civil Indian Home Rule Society founded that same year in London
under the patronage of Shyamji Krishna Varma.
justice.
• He created a new hierarchy of civil courts, with the 150. Solution (a)
district judge at the bottom, four provincial courts of Exp) Option a is the correct answer.
appeal in the middle, and the Sadar Diwani Adalat (the
The Champaran Satyagraha was the first Satyagraha
Supreme Court of Bengal) at the top. movement in 1917 led by MK Gandhi. It was a farmer’s

PYQ Workbook 318


HISTORY OF MODERN INDIA

uprising in Bihar during the British colonial period. An 155. Solution (b)
Ashram was also established in Champaran by MK Gandhi Exp) Option b is the correct answer.
Gandhi was persuaded by Raj Kumar Shukla, a peasant The princely state of Gwalior was not annexed by the
leader from Champaran, to visit the region and see the plight British, but remained under their indirect rule until 1947.
of the farmers. He arrived in Champaran in April 1917 and
started an investigation into the grievances of the farmers. The other princely states mentioned in the options
He was supported by other leaders such as Rajendra Prasad, were annexed by the British under various policies and
Anugrah Narayan Sinha, Brajkishore Prasad, and Mahadev circumstances:
Desai. He faced resistance from the British authorities, Sind: It was annexed by the British in 1843 after the Anglo-
who tried to arrest him and prevent him from meeting the Sind War, in which the British defeated the Talpur dynasty
farmers. He defied their orders and continued his work. that ruled Sind.
He also organized mass meetings, petitions, and protests to Awadh: It was annexed by the British in 1856 under the
demand justice for the farmers. Doctrine of Lapse, which allowed the British to take over
151. Solution (b) a princely state if it had no male heir or if its ruler was
incompetent or misgoverned. The annexation of Awadh was
Exp) Option b is the correct answer.
one of the causes of the Indian Rebellion of 1857.
The Sarabandi Campaign of 1922 was led by Sardar
Vallabhbhai Patel. This campaign aimed to boycott the Satara: It was annexed by the British in 1848 under the
payment of unfair taxes that burdened the poor farmers Doctrine of Lapse, after the death of its last ruler, Pratap
and peasants, particularly in the Bardoli Taluka, which Singh, who had no biological son to succeed him.
had seen civil disobedience efforts led by Mahatma Gandhi 156. Solution (c)
in 1921. Patel’s non-violent protest garnered significant
support from the Indian population, as it aimed to relieve Exp) Option c is the correct answer.
the burden of oppressive taxes on poor farmers and peasants. Lord Curzon was the Viceroy of India from 1899 to 1905,
and he wrote this statement in a letter to Secretary of State
152. Solution (a) Hamilton on November 18, 1900. He expressed his disdain
Exp) Option a is the correct answer. and contempt for the Indian National Congress, which was
The book “Desher Katha” was written by Sakharam Ganesh founded as a platform for the educated Indians to voice their
Deuskar (1869–1912), a close associate of Sri Aurobindo. demands and grievances to the British government.
This book extensively detailed the British commercial and
Important Tips
industrial exploitation of India.
Events during the tenure of Lord Curzon:
153. Solution (c)
• The partition of Bengal in 1905, which divided the
Exp) Option c is the correct answer province into two parts: Eastern Bengal and Assam,
Lord Minto was part of the Minto-Morley reforms in 1909. with a Muslim majority, and Western Bengal, with a
It introduced a system of communal representation for Hindu majority. The official reason for this partition
Muslims based on the concept of a ‘separate electorate’ in was to improve the administration and development
India. Under this system, the Muslim members were to of the region, but the real motive was to create a divide
be elected only by Muslim voters. Therefore, Rajendra and rule policy among the Hindus and Muslims,
Prasad considered Lord Minto as the father of Pakistan. • The Indian Universities Act in 1904, which reduced
Lord Minto also came to be renowned as the Father of the autonomy and representation of Indians in the
Communal Electorate. universities, and increased the control and interference
154. Solution (b) of the government in academic matters.

Exp) Option b is the correct answer. • The Curzon-Kitchener controversy in 1905, which
was a power struggle between Lord Curzon and Lord
Lord Curzon was the Viceroy of India from 1899 to 1905,
Kitchener, who was the Commander-in-Chief of the
and he established the Imperial Cadet Corps in 1901. The
British Indian Army.
Imperial Cadet Corps was a cadet corps or military school
that was founded exclusively to give officer training to the • The Delhi Durbar in 1903, which was a grand
princes and gentlemen of British India. It was founded in ceremony held in Delhi to celebrate the coronation of
November 1901 under the direct surveillance of Lord Curzon King Edward VII as the Emperor of India.
at Meerut and Dehradun. Major D. H. Cameroon was made 157. Solution (b)
its first commandant and Maharaja Pratap Singh of Idar was
made its Honorary commandant. The youths between 17 and Exp) Option b is the correct answer.
20 years were selected and admitted as Imperial Cadets and Lord Lytton was the Viceroy of India from 1876 to 1880,
their education was to be at any one of the Chief ’s colleges at and he began to confer the titles of Rai Bahadur and Khan
Rajkot, Indore, Lahore, Ajmer or Raipur. The selected cadets Bahadur to Indian subjects for their loyal and meritorious
had to join the corps at Meerut or Dehradun. services to the British Empire.

319 PYQ Workbook


HISTORY OF MODERN INDIA

Important Tips • Aldous Huxley (1894-1963) was an English writer


• The titles of Rai Bahadur and Khan Bahadur were and philosopher, known for his extensive bibliography
formal titles of honour and respect, which were encompassing novels, non-fiction, essays, and poems.
bestowed on Hindu and Muslim (and other non- He explored philosophical mysticism, universalism,
Hindu) natives of British India respectively. and authored works like “The Perennial Philosophy,”
• Translated, Rao means “prince”, and Bahadur means highlighting Western and Eastern mysticism
“brave” or “most honorable”. commonalities. He also penned “Brave New World”
(dystopia) and “Island” (utopia), envisioning alternate
• The titles were accompanied by a medal called a Title
societies.
Badge and a citation (or sanad).
• The recipients were entitled to prefix the title to their • Charles Freer Andrews (1871-1940) was an
name and were presented with a special Title Badge Anglican priest, missionary, and advocate for
and a citation (or sanad). Indian independence. He forged close bonds with
Rabindranath Tagore and Mahatma Gandhi, convincing
• It was conferred on behalf of the Government of British
Gandhi to return to India from South Africa. Gandhi
India by the Viceroy and Governor-General of India.
affectionately called him “Christ’s Faithful Apostle”
158. Solution (c) (C.F.A) and his students named him “Deenabandhu”
Exp) Option c is the correct answer (Friend of the Poor). Charles Freer Andrews, engaged
in the Christian Social Union, sought to harmonize
Sir William Wilson Hunter was a multitalented Scottish
Gospel principles with justice. He joined the Cambridge
historian, statistician, compile and member of the Indian
Mission to Delhi in 1904, opposing British racism and
Civil Service. He was of the belief that “The Muslims, if
supporting Indian political causes. Andrews aided in
contented and satisfied, would become the greatest bulwark
resolving the 1913 cotton workers’ strike in Madras.
of British power in India.”
• William Digby, a British author and humanitarian,
Important Tips worked in India during the Great Famine of 1876–78,
William Wilson Hunter is most known for The Imperial advocating for relief and criticizing British policies.
Gazetteer of India on which he started performing in He engaged in liberal politics, pushing for Indian
1869, and which was eventually published in nine volumes reform and even ran for office. Later, he established
in 1881. the Indian Political and General Agency in London to
He reached Bengal Presidency in November 1862. raise awareness of Indian issues. His private lobbying
Later, he was made assistant magistrate and collector of for the Maharaja of Kashmir led to his disconnection
Birbhum, in the lower provinces of Bengal, where he began from the Indian National Congress.
collecting local traditions and records, which formed the 160. Solution (d)
materials for his publication.
Exp) Option d is the correct answer.
He also compiled A Comparative Dictionary of the Non-
Aryan Languages of India, a glossary of dialects based The demand for ‘Complete Independence’ was raised for
mainly upon the collections of Brian Houghton Hodgson. the first time in 1921 by Maulana Hasrat Mohani. He
along with Swami Kumaranand, presented the demand
159. Solution (a) for complete independence for India at the Ahmedabad
Exp) Option a is the correct answer. Session of Congress in 1921. This marked an important
milestone in India’s struggle for freedom from British
Katherine Mayo, Aldous Huxley, Charles Andrews, and colonial rule. Maulana Hasrat Mohani is also known for
William Digby all wrote commentaries or works related to coining the famous slogan “Inquilab Zindabad” (Long live
the condition of India during the British colonial period. the revolution!).
Mayo’s “Mother India” was a critical examination of Indian
society, Huxley’s writings included reflections on Indian 161. Solution (a)
philosophy and society, Andrews was an advocate for Indian Exp) Option a is the correct answer.
independence and wrote about Indian issues, and Digby Aurobindo Ghosh authored the series of articles titled
wrote about the Great Famine in India and later worked on ‘New Lamps for Old’ (1893-94), which criticized the
Indian political reform. Congress for being out of touch with the ‘proletariat.’ In
Important Tips these articles, he discussed various aspects of the political
situation, initially critiquing the moderate methods of the
Katherine Mayo (1867-1940) was an American historian
Congress and later delving into philosophical and historical
and nativist known for her advocacy of American themes related to politics. His writings played a significant
nativism, opposition to non-white and Catholic role in enlightening the masses and contributing to the
immigration, and promotion of racist stereotypes. She freedom struggle.
also authored “Mother India” (1927), a highly critical book
on Indian society, criticized as Indophobic by Mahatma 162. Solution (a)
Gandhi and others. Exp) Option a is the correct answer.

PYQ Workbook 320


HISTORY OF MODERN INDIA

“India For Indians” is a collection of speeches by Chitta Important Tips


Ranjan Das, addressing a meeting in Calcutta where Hindus
G. D. Birla: He was an Indian industrialist and
and Muslims came together to unite for their country’s cause
philanthropist who founded the Birla Group of companies.
in 1905 during swadeshi movement. He expresses optimism
He was a supporter of Mahatma Gandhi. He participated
about their collaboration, reminiscing about the Swadeshi
in the Civil Disobedience Movement and the Quit India
movement and foreseeing a joint struggle for independence.
Movement and was imprisoned several times by the
163. Solution (a) British.
Exp) Option a is the correct answer. J. R. D. Tata: He was an Indian aviator and businessman
Lord Dalhousie was the Governor-General of India from who founded Tata Airlines, which later became Air India.
1848 to 1856, and he was responsible for the introduction He was also the chairman of Tata Sons and Tata Group,
of railways in India. He envisioned a network of railways that and expanded the business into various sectors such as
would connect the different regions of India and facilitate steel, chemicals, automobiles, hotels, and more.
trade, commerce, communication, and military operations. Balchand Hirachand: He was an Indian entrepreneur
He also believed that railways would help in the spread of and railway contractor who founded the Scindia Steam
Western civilization and culture among the Indians. Navigation Company in 1919. He was also involved in
the establishment of Hindustan Aeronautics Limited,
Important Tips Hindustan Shipyard Limited, and Walchandnagar
• Lord Dalhousie appointed a committee under Sir Industries Limited.
Macdonald Stephenson to survey and plan the railway
routes in India. 166. Solution (c)
• The first railway in India was laid down between Exp) Option c is the correct answer.
Bombay and Thane, covering a distance of 34 Charles Andrews was an English missionary and educator
kilometers. The first train ran on this route on 16 April who became a close friend and associate of Mahatma
1853, carrying 400 passengers in 14 coaches. Gandhi during the freedom movement. He met Gandhi
• The train was flagged off by Lord Dalhousie himself, in South Africa in 1914 and helped him negotiate with the
along with other dignitaries and officials. British government on various issues. He also supported
• The first railway in India marked the beginning of a Gandhi’s campaigns in India, such as the Champaran
new era of transportation and development in the Satyagraha, the Kheda Satyagraha, and the Non-Cooperation
country Movement. He was affectionately called “Deenabandhu” or
“Friend of the Poor” by Gandhi and the Indian people.
164. Solution (a)
Exp) Option a is the correct answer. Important Tips

Maulana Abul Kalam Azad, originally named Muhiyuddin • A. O. Hume: He was a Scottish civil servant,
Ahmad, was a prominent Deoband scholar who played a ornithologist, and botanist who worked in India for 32
significant role in the freedom movement of India. He was years. He was also a political reformer who founded
a versatile figure, serving as a journalist, freedom fighter, the Indian National Congress in 1885 and served as its
politician, and educationist. Azad was known for his efforts first general secretary.
to promote Hindu-Muslim unity and his opposition to the • William Wavell: He was a British army officer and
partition of India. He started influential publications like Al- commander-in-chief of the Middle East during World
Hilal and Al-Balagh, which propagated Indian nationalism War II. He later became the Viceroy of India from
and unity. Azad supported the Non-Cooperation Movement 1943 to 1947 and oversaw the negotiations for India’s
and served as the president of the Indian National Congress independence and partition.
multiple times. After independence, he became India’s first
167. Solution (c)
Education Minister and made significant contributions to
the country’s educational system. He was posthumously Exp) Option c is the correct answer.
awarded the Bharat Ratna, India’s highest civilian honor, in Jayaprakash Narayan is famously referred to as JP or Lok
1992. Nayak. He stood as an Indian freedom fighter, theorist,
socialist, and political figure. He’s recognized as the “Hero
165. Solution (b)
of Quit India Movement.”
Exp) Option b is the correct answer.
Important Tips
Jamnalal Bajaj was a capitalist and a leader of the Indian
National Congress. He served as an All-India Congress JP Narayan gained prominence for his opposition to the
Committee treasurer for many years and went to jail in former Prime Minister Indira Gandhi, a move believed
1930. He was also a close associate and follower of Mahatma to have curbed her authority during the “JP movement.”
Gandhi and supported his causes of swadeshi, khadi, and This movement coined the iconic slogan “Sinhaasan
harijan welfare. khaali karo ki janta aati hai.”

321 PYQ Workbook


HISTORY OF MODERN INDIA

He was born on 11 October 1902 in Sitabadiyara, Bihar. The Calcutta session of 1906 was presided by Dadabhai
Naoroji. In this session Congress adopted Swaraj as the Goal
In 1948, he took the helm of the Socialist Party within the of Indian people. This session of INC passed the famous 4
Congress and later established the Samajwadi Socialist resolutions, namely - swaraj, swadeshi, Boycott, and National
Party in collaboration with the Gandhian Party. Educations.
168. Solution (d) 172. Solution (c)
Exp) Option d is the correct answer. Exp) Option c is the correct answer.
The Butler Committee (1927) was set up to examine the Pyare Lal was the secretary of Mahatma Gandhi during
nature of relationship between the princely states and
the Noakhali period. He accompanied Gandhi on his peace
government. It gave the following recommendations—
mission to Noakhali and later wrote a book titled Mahatma
1. Paramountcy must remain supreme and must fulfil its Gandhi: The Last Phase, which chronicles Gandhi’s life from
obligations, adopting and defining itself according to the 1946 to 1948.
shifting necessities of time and progressive development of
states. 173. Solution (c)
2. States should not be handed over to an Indian Government Exp) Option c is the correct answer.
in British India, responsible to an Indian legislature, without Warren Hastings was the first Governor-General of Bengal
the consent of states. from 1772 to 1785, and he abolished the dual government
Thus, “paramountcy” was left undefined and this hydra- system in 1772. He replaced it with a system of direct
headed creature was left to feed on usage, Crown’s administration by the Company, which was more efficient
prerogative and the princes’ implied consent. and less corrupt. He also introduced several reforms in the
judicial, revenue, and military sectors.
169. Solution (a)
Important Tips
Exp) Option a is the correct answer.
About Warren Hastings:
The Simla deputation of 1906 was led by Aga Khan III. The
Shimla Deputation took place on 1st October, 1906. Here • He was the first Governor-General of Bengal from
Lord Minto met with a deputation of 35 Muslim leaders led 1772 to 1785, and he established a system of direct
by Aga Khan. They demanded for both separate electoral administration by the Company
constituencies for Muslims in the provincial council and • He introduced several reforms in the judicial, revenue,
imperial legislative council for them. The Muslim community and military sectors, such as the establishment of the
should be given preference while nominating the members Supreme Court of Calcutta, the abolition of the dastaks
to the Viceroy’s council. (tax-free passes), and the reorganization of the army
Important Tips • He founded the Asiatic Society of Bengal and the
Calcutta Madrasa, which promoted the study of Indian
• Aga Khan III (1877 – 1957) was the leader of the Nizari culture, languages, and literature
Ismaili sect in British India from 1885.
• He was accused of corruption, oppression, and
• He took interest in promoting the interests of the tyranny by his political enemies, such as Philip
Muslims in British India. Francis and Edmund Burke. He was impeached by the
• Aga Khan took a Muslim delegation to meet British British Parliament in 1787 on charges of misrule and
Viceroy Lord Minto in 1906. misconduct in India

170. Solution (b) 174. Solution (a)


Exp) Option b is the correct answer. Exp) Option a is the correct answer.
On 22 March 1912 ‘Bihar and Orissa’ carved out as separate Jai Jawan Jai Kisan (English: “Hail the Soldier, Hail the
province through The Bengal, Bihar and Orissa and Assam Farmer”) was a slogan coined by Lal Bahadur Shastri, the
Laws Act, 1912. March 22 is celebrated as Foundation Day second Prime Minister of India. He introduced this slogan
of Bihar. in 1965 during a public gathering in Uruwa, Prayagraj.
Shastri used this slogan to inspire both the soldiers to defend
Important Tips
India and the farmers to increase food grain production,
On 1 April 1936 Bihar and Orissa became separate addressing the challenges of a war with Pakistan and a
provinces from the Government of India Act, 1935. shortage of food grains in the country.
On 1 April 1936, Sir James David Sifton appointed the
Important Tips
first governor of Bihar whereas Muhammad Yunus was
declared the first Prime Minister of state. • Jai Jawaan, Jai Kisan, Jai Vigyan: Following the
Pokhran nuclear tests in 1998, Prime Minister Atal
In 2000, Jharkhand was carved out of Bihar.
Bihari Vajpayee extended the iconic slogan “Jai
171. Solution (d) Jawaan, Jai Kisan” by introducing “Jai Vigyan” (Hail
Exp) Option d is the correct answer. Science).

PYQ Workbook 322


HISTORY OF MODERN INDIA

• Jai Jawaan, Jai Kisan, Jai Vigyan, Jai Anusandhan: Charles James Fox’s India Bill- In November 1783,
In his speech at the 106th Indian Science Congress Charles James Fox, a prominent British Whig statesman,
in Lovely Professional University, Jalandhar, Prime introduced a bill on India which sought to abolish
Minister Narendra Modi expanded the well-known the Court of Directors and the Court of Proprietors,
slogan to “Jai Jawaan, Jai Kisan, Jai Vigyan, Jai transferring their powers to nominated commissioners.
Anusandhan,” emphasizing the crucial role of research
Lord Curzon’s Appointment- In January 1899, Lord
in national development and progress.
Curzon was appointed as the Viceroy of India which
• Jai Jawaan, Jai Kisan, Jai Vigyan, Jai Vidwan: Dr. received a peerage of Ireland and was created Baron
Kailash Chandra Mishra, the Director of the Lal Curzon of Kedleston, in the County of Derby, upon his
Bahadur Shastri Institute of Management, Delhi, appointment.
introduced the adaptation “Jai Jawaan, Jai Kisan, Jai
Vigyan, Jai Vidwan” during his speech at the acceptance 177. Solution (b)
ceremony of the 10th Lal Bahadur Shastri National Exp) Option b is the correct answer.
Award to Sunil Bharti Mittal.
The decennial Census of India, which has been conducted
175. Solution (d) 15 times as of 2011, began in 1872 and the first complete
Exp) Option d is correct answer. census took place in 1881. It was conducted during the reign
Option a is correct- Through Indian Independence Act of Lord Mayo. Since 1951, all censuses were conducted
1947, India became a sovereign and independent states. under the 1948 Census of India Act.
This act granted authority to the princely states to join either
178. Solution (b)
of the dominions or remain independent.
Option b is correct- The decision to join either India or Exp) Option b is the correct answer.
Pakistan or remain independent was left not to the people Hill stations were initially set up during the colonial period
of the princely states but to the princely rulers. This was in India primarily to serve the needs of the British army. The
because the princely states were not considered to be part British army was particularly susceptible to diseases such as
of British India, and so they were not subject to the same malaria and cholera, which were common in the plains.
democratic principles as British India. The hill stations provided a respite from these diseases and
Option c is correct- The ruler of Travancore, Maharaja helped to improve the health of the British army.
Chithira Thirunal Balarama Varma, initially decided to
remain independent. He questioned the Congress leadership Important Tips
and remained steadfast in his stance until July 1947. Hill Stations established by British in India:
However, he ultimately chose to join India after surviving Shimla - Established in 1819
an assassination attempt by a member of the Kerala Socialist
Party. Darjeeling - Established in 1835
Option d is incorrect- The State of Travancore did not join Mussoorie - Established in 1823
India through a plebiscite. Instead, the Ruler of Travancore, Ooty (Udhagamandalam) - Established in the early 19th
Chithira Thirunal Balarama Varma, decided to accede to century
India on 25th July 1947, before the formal partition of India
Nainital - Established in 1841
and the creation of Pakistan.
Mount Abu - Established in the 19th century
176. Solution (a)
Munnar - Established in the late 19th century
Exp) Option a is the correct answer.
Dalhousie - Established in 1854
The Second Anglo-Afghan War occurred from 1878 to 1880
and was a conflict between the British Empire (represented Kodaikanal - Established in the 1840s
by Lord Lytton) and Afghanistan. The war was driven by Mahabaleshwar - Established in the 1820s
British concerns over Russian influence in the region and
the desire to maintain control over Afghanistan’s foreign 179. Solution (d)
policy. The British launched a military expedition into Exp) Option d is the correct answer.
Afghanistan, deposing Sher Ali Khan and installing Yakub Statement 1 is correct: Mahatma Gandhi developed and
Khan as a more British-friendly leader. The war lasted for
employed the concept of Satyagraha during his years in
two years and involved several battles and skirmishes.
South Africa while leading the Indian community in their
Important Tips struggle against racial discrimination and injustice.
Ilbert Bill- It was proposed in 1883, aimed to allow Indian Statement 2 is correct: Gandhiji emphasized that Satyagraha
magistrates to preside over cases involving British is based on the principles of truth (Satya) and non-
subjects in India. The bill was named after its sponsor, violence (Ahimsa). These two principles were considered
Sir Courtenay Peregrine Ilbert, the legal member of the fundamental to the philosophy and practice of Satyagraha.
Viceroy’s Council. Lord Rippon was associated with the
Statement 3 is correct: In Satyagraha, the emphasis is on
Bill.
self-suffering and self-restraint rather than causing harm

323 PYQ Workbook


HISTORY OF MODERN INDIA

or suffering to the opponent. The Satyagrahi willingly marks in Bengali literature. He also wrote several books for
undergoes suffering, such as imprisonment, fasting, or other children, such as Barnaparichay, which taught the Bengali
forms of self-denial, to demonstrate their commitment to alphabet and grammar.
truth and non-violence. Statement 2 is incorrect: Ishwarchandra Vidyasagar
Statement 4 is correct: One of the early instances of received the title ‘Vidyasagar’ (meaning ‘Ocean of Learning’)
Gandhiji’s application of Satyagraha in India was during from Sanskrit College, Calcutta, where he studied from 1828
the Champaran Satyagraha in 1917. It was a movement to 1839 and excelled in Sanskrit and philosophy. He later
against the oppressive indigo plantation system in Bihar’s became the principal of Sanskrit College in 1851.
Champaran district. Statement 3 is correct: He was enthusiastic about the spread
180. Solution (d) of women’s education. He established several schools for
girls in Calcutta and other places and advocated for female
Exp) Option d is the correct answer.
literacy and empowerment. He also supported the cause of
The Congress Socialist Party was founded by a group of widow remarriage and campaigned for the Hindu Widows’
socialists such as Jaya Prakash Narayan, Achyut Patwardhan, Remarriage Act of 1856.
NG Gore, Ashok Mehta, SM Joshi, ML Dantwala in 1934.
Statement 4 is incorrect: Vidyasagar defended widow
MN Roy was the founder of the Communist Party of India
marriage by citing scriptural evidence from Hindu
in 1925.
texts such as Vedas, Upanishads, Smritis, Puranas, etc.,
The pairs of organization and their founders are correctly to counter the orthodox opposition that claimed widow
matched: remarriage was against Hindu religion. He also wrote a
• National Liberation Federation: Tej Bahadur Sapru and book called Bidhaba Bibaha Bishayak Prabandha (Treatise
MR Jayakar on Widow Remarriage) in 1855, where he presented his
• Jamiat-ul Ulama-i-Hind:Maulana Mehmood Hasan arguments based on scriptures and logic in favour of widow
Shaikh-ul-Hind remarriage.
• Congress Democratic Party: Bal Gangadhar Tilak 184. Solution (c)
181. Solution (c) Exp) Option c is the correct answer.
Exp) Option c is the correct answer. Statement 1 is correct: The formal transfer of power on
Vasudeo Balwant Phadke is known in history for leading an 15th August 1947 heralding India’s Independence was
armed uprising against the British in the 1870s. He was a announced by Lord Mountbatten. Lord Mountbatten was
revolutionary and freedom fighter who organized a group of the last Viceroy of India appointed by the British government.
rebels with the aim of overthrowing British colonial rule in He was also the first Governor-General of independent India.
India. Phadke’s uprising, known as the Phadke Conspiracy or He played a crucial role in negotiating the partition of India
the Indi Rebellion of 1879, involved attacks on British targets and Pakistan and overseeing the transfer of power from the
and attempts to incite a larger rebellion. However, his efforts British to the Indian and Pakistani governments
were ultimately unsuccessful, and Phadke was arrested and Statement 2 is correct: Mahatma Gandhi, the leader of the
sentenced to transportation for life. Indian independence movement and the advocate of non-
violence, was not a part of the celebrations in New Delhi
182. Solution (d) on 15th August 1947. He was in Calcutta, where he was
Exp) Option d is the correct answer. trying to prevent communal violence between Hindus and
Bhagat Singh did not threw a bomb in the Central Legislative Muslims that had erupted after the partition. He spent the
Assembly in 1929 with the objective to kill as many people day fasting, praying and meeting with refugees and victims
as possible. Bhagat Singh and his associate Batukeshwar of riots. He did not rejoice over India’s independence, as he
Dutt threw a bomb in the assembly on April 8, 1929, not to was deeply saddened by the partition and the bloodshed that
kill anyone but to make a loud noise to protest against the accompanied it.
passage of two repressive bills. The bomb was thrown away
185. Solution (b)
from the crowded area and only a few people were injured.
Bhagat Singh and Batukeshwar Dutt also threw leaflets in the Exp) Option b is the correct answer.
assembly hall that read “It takes a loud voice to make the Narendra Dev was the president of the Congress Socialist
deaf hear” and “Long Live Revolution” before surrendering Party (CSP), which was the socialist wing of the Congress.
themselves to the police. He joined the Indian National Congress in 1916 and became
one of the founders of the Congress Socialist Party in 1934.
183. Solution (a)
He advocated for democratic socialism, satyagraha, and
Exp) Option a is the correct answer. secularism as the principles of the independence movement
Statement 1 is correct: Vidyasagar’s writings significantly and the future of India. He was also active in the peasant
helped to develop the Bengali language. He simplified movement and served as the president of the All-India Kisan
and modernized Bengali prose and introduced punctuation Congress.

PYQ Workbook 324


HISTORY OF MODERN INDIA

Important Tips Europe and America. They also adapted and modified these
ideas to suit the Indian context and culture
Congress Socialist Party:
Statement II is correct: Statement II explains that Indian
• The Congress Socialist Party (CSP) was a socialist
nationalism was shaped by its response to Western
group within the Indian National Congress (INC) that
imperialism, which is a common phenomenon when
was founded in 1934 by socialists such as Jayaprakash
nations respond to external pressures. Many Indian leaders,
Narayan and Acharya Narendra Dev.
including Gandhi and Nehru, were educated in the West and
• They rejected the sectarian attitude of the Communist were exposed to Western political and philosophical ideas,
Party of India towards the Congress. such as democracy, freedom, and self-determination.
• They believed in Marxist ideas, Gandhism, liberal Taken together, statement II provides an explanation for
and social democracy of the west, nationalism and statement I by highlighting the connection between Indian
independence as their goal, abolition of capitalism, nationalism and its response to Western imperialism.
zamindari system and princely states.
188. Solution (a)
• Some of their prominent leaders were Ram Manohar
Lohia, Minoo Masani, Asoka Mehta, S.M. Joshi, and Exp) Option a is the correct answer.
others William Wilson Hunter, a British ethnographer and
administrator, initiated the Survey of India project in
186. Solution (b)
1870, as per Lord Mayo’s request. The survey, completed
Exp) Option b is the correct answer. in 1881, provided valuable information on India’s geography,
Bankim Chandra Chattopadhyay was a Bengali novelist, history, and culture.
poet and journalist who wrote before the partition of India.
189. Solution (b)
He is best known for his historical and nationalist novels such
as Anandamath, which contains the song Vande Mataram, Exp) Option b is the correct answer.
considered as the national song of India. He died in 1894, Statement 1 is incorrect: Barnaparichay was written by
long before the Partition happened. Ishwar Chandra Vidyasagar, not Raja Rammohan Roy.
Important Tips Statement 2 is correct: Barnaparichay was a highly influential
textbook that played a significant role in promoting basic
• Saadat Hasan Manto: He was a Pakistani writer of
education and literacy in the Bengali language during the
Kashmiri origin who wrote in Urdu. He is regarded
as one of the greatest writers of short stories in South mid-19th century.
Asian history. He wrote extensively about partition Important Tips
and its aftermath, depicting the violence, trauma, Ishwar Chandra Vidyasagar (1820-1891):
displacement and madness that it caused. Some of his
• Born in West Bengal, he earned the title ‘Vidyasagar’
famous stories on partition are Toba Tek Singh, Khol
for his Sanskrit and philosophy mastery.
Do, Thanda Gosht, etc.
• Joined Fort William College as Sanskrit department
• Bhisham Sahni: He was an Indian writer, playwright
head at 21.
and actor who wrote in Hindi. He was a witness to the
partition and its horrors. He wrote several novels and • Supported Bengali poet Michael Madhusudan
stories based on his experiences of partition, such as Dutta’s education and return to India.
Tamas, Pali, Basanti, etc. • Promoted universal education and opened Sanskrit
• Narendranath Mitra: He was an Indian writer who College to lower castes.
wrote in Bengali. He was also a witness to the partition • Established 20 model schools, revised exam patterns,
and its impact on Bengal. He wrote many stories on introduced English and Western Sciences.
partition, such as Rajani Gandha, Abhimanyu, etc. • Simplified Bengali alphabet, authored ‘Borno
187. Solution (a) Porichoy’ for learning Bengali.
Exp) Option a is the correct answer. • Key figure in Bengal Renaissance, wrote numerous
books on Bengal’s history and literature.
Statement I is correct: The statement is based on the argument
of Partha Chatterjee, a prominent postcolonial scholar. He • Social reformer, instrumental in passing the Widow
claimed that nationalism in India was a ‘derivative discourse’ Remarriage Act (1856).
from the West, but also ‘different’ in its content and form. • Advocated women’s education, fought against child
He argued that Indian nationalists adopted the political marriage and Kulin Brahmin polygamy.
and ideological framework of western nationalism, but • Authored works on social reforms including
also modified it to suit their own cultural and historical ‘Bidhobabivah,’ ‘Bahubivah,’ and ‘Balyabivah.’
context. The Indian nationalists drew inspiration from the
ancient and medieval history of India, as well as from the 190. Solution (d)
contemporary movements of nationalism and democracy in Exp) Option d is the correct answer.

325 PYQ Workbook


HISTORY OF MODERN INDIA

The Madras Regiment is the oldest infantry regiment of 194. Solution (c)
the Indian Army, originating in the 1750s as a unit of the Exp) Option c is the correct answer.
British East India Company. The regiment took part in
In 1918, Mahatma Gandhi employed a hunger strike as
numerous campaigns with the British Indian Army and the
a political weapon for the first time during a strike by
post-independence Indian Army.
mill workers in Ahmedabad. After 5 days of fasting, the
191. Solution (b) mill owners conceded to the workers’ demands, making it a
successful tactic for Gandhi.
Exp) Option b is the correct answer.
Mirat-ul-Urus is a famous novel written by Nazir Ahmad Important Tips
in 1869. It is considered one of the first novels in the Urdu Ahmedabad Mill strike, 1918
language. The title translates to “The Bride’s Mirror”
• During the dispute between Ahmedabad mill owners
in English. The novel explores the lives and experiences
and workers, the workers demanded a 50% wage
of women in 19th-century Muslim society, focusing on
increase due to wartime inflation.
the themes of marriage, family dynamics, and societal
expectations. • The mill owners offered only a 20% raise, leading to a
strike. Relations worsened, with dismissals and hiring
192. Solution (b) of replacement workers.
Exp) Option b is the correct answer. • Anusuya Sarabhai supported the workers, and Gandhi
Hard Times is a novel by Charles Dickens that was joined the cause. He advocated for a 35% wage
published in 1854. The novel explores the social and increase and went on a fast unto death, pressuring
economic problems caused by industrialization. Dickens the mill owners to refer the matter to a tribunal.
shows how the factory system dehumanizes workers and • The strike ended, and the tribunal awarded a 35%
leads to poverty and despair. Hard Times is a powerful
wage increase to the workers.
critique of the Industrial Revolution and its impact on the
poor. 195. Solution (a)
193. Solution (a) Exp) Option a is the correct answer.

Exp) Option a is the correct answer. Influence 1 is correct: The deccan famine of 1876-77 was
characterized by widespread starvation and suffering and
• Bal Gangadhar Tilak was a member of the Poona it led to Vasudev Balvant Phadke realization of the socio-
Sarvajanik Sabha. economic issues prevalent in colonial India and the plight of
• He opposed the Age of Consent Bill, introduced in the common people.
1891, arguing against its interference in Hindu religious Influence 2 is correct: Hindu revivalism, which sought to
practices. revive and promote Hindu culture, traditions, and values,
• Tilak was not associated with the Gaurakshini Sabha, had an influence on Phadke.
which focused on cow protection. Influence 3 is correct: The theory of Drain of Wealth,
• He was not associated with the Atmiya Sabha, a popularized by Dadabhai Naoroji had a significant influence
philosophical discussion circle started by Ram Mohan on Phadke, shaping his understanding of the exploitative
Roy in 1815. nature of British colonialism and motivating his nationalist
and anti-colonial activities.
Important Tips
Influence 4 is incorrect: Phadke was a revolutionary who
The Age of Consent Act, 1891:
believed in armed struggle against British rule. Phule was
• The Age of Consent Act, 1891, raised the age of a social reformer who believed in peaceful change.
consent for sexual intercourse for all girls, whether
married or unmarried, from ten to twelve years, 196. Solution (a)
making violation subject to criminal prosecution as Exp) Option a is the correct answer.
rape. P.C. Roy was not communisis leader. Sir Acharya Prafulla
• The act was introduced as a bill on January 9, 1891, by Chandra Ray was an Indian chemist, educationist,
Sir Andrew Scoble in the Legislative Council of the historian, industrialist and philanthropist. He established
Governor-General of India in Calcutta. the first modern Indian research school in chemistry
(post classical age) and is regarded as the Father of Indian
• It was prompted by the case of an eleven-year-old
Chemistry.
Bengali girl, Phulmoni Dasi, who died due to forceful
intercourse by her 35-year-old husband in 1889. Important Tips
• It received support from Indian reformers such as • SA Dange was a Marathi communist leader who was
Behramji Malabari and women social organisations. one of the founders of the Communist Party of India.
• Bal Gangadhar Tilak opposed the bill, rejecting In 1921, Dange published a pamphlet titled Gandhi Vs.
government interference in social customs, even if Lenin, a comparative study of approaches of both the
well-intentioned. leaders.

PYQ Workbook 326


HISTORY OF MODERN INDIA

• Muzaffar Ahmad was a Bengali communist leader period, the Act provided the legal and administrative
who was one of the founders of the Communist Party framework for the functioning of the Indian government.
of India. Muzaffar Ahmad was a prolific writer, and 202. Solution (b)
he wrote extensively on a variety of topics, including
Exp) Option b is the correct answer.
communism, socialism, Indian independence, and the
history of the Communist Party of India. Hriday Nath Kunzru, a prominent member of the Servants
of India Society, established the Seva Samiti at Allahabad
• Singaravelu was a Tamil communist leader who
in 1914. The Seva Samiti was a social service organization
was one of the founders of the Communist Party
aimed at addressing various social issues and promoting
of India. Some of his most notable writings include:
welfare activities in the local community.
The Plight of the Indian Peasantry (1933), The Indian
National Congress and the Working Class (1935), The 203. Solution (a)
Communist Party of India and the Indian Revolution
Exp) Option a is the correct answer.
(1
Sarala Devi Chaudhurani was a prominent figure in the
197. Solution (b) Indian independence movement and a feminist activist, but
Exp) Option b is the correct answer. she was not known for organizing workers in strikes in the
1920s.
• BR Ambedkar was the founder of the Independent
Labour Party in 1936. Ushabai Dange, Prabhabati Devi, and Anasuya Behn, were all
women labor leaders who played active roles in organizing
• Sikander Hayat Khan was the leader of the Unionist
workers and strikes during that period.
Party in the Punjab.
• Fazlul Huq was the leader of the Krishak Praja Party Important Tips
in Bengal. • Sarala Devi Chaudhurani: She was a prominent
• Khaliquzzaman was the founder of the Pakistan nationalist and social reformer who founded the
Muslim League in 1940. Bharat Stree Mahamandal, one of the first women’s
organisations in India.
198. Solution (d) • Ushabai Dange: She was a communist leader and trade
Exp) Option d is the correct answer. unionist who led several strikes of textile workers and
Maniram Dewan was an Assamese nobleman who was one mill workers in Bombay.
of the first Indians to establish tea gardens in Assam. He • Prabhabati Devi: She was a socialist leader and trade
established two small proprietary tea gardens of his own unionist who led the scavengers’ strike in 1928 and
in the 1850s. However, he was executed for treason in then the jute general strike of 1929. She worked closely
1858, and his tea gardens were confiscated by the state. with socialists and communists in this period.
199. Solution (c) • Anasuya Behn: She was a Gandhian leader and trade
unionist who founded India’s oldest union of textile
Exp) Option c is the correct answer.
workers, the Ahmedabad Textile Labour Association
Nil Darpan is a play written by Dinabandhu Mitra in 1858. (Majadoor Mahajan Sangh) in 1920
It depicts the atrocities committed by indigo planters on the
peasantry in Bengal. The play was translated into English 204. Solution (d)
by Michael Madhusudan Dutta, a Bengali poet and Exp) Option d is the correct answer.
playwright. Amritlal Vithaldas Thakkar was a social worker and
200. Solution (d) educationist who worked for the upliftment of tribal
people in Gujarat. He founded the Bhil Seva Mandal in
Exp) Option d is the correct answer.
1922 to promote education, health care, and economic
William James, Henry Colebrooke, and Nathaniel Halhed development among the Bhil tribal community.
were all linguists who tried to interpret the culture of South
Asia to the East India Company. They were all members of 205. Solution (c)
the Asiatic Society of Bengal, which was founded in 1784 Exp) Option c is the correct answer.
by Sir William Jones. The Asiatic Society was dedicated to The British Indian Medical Service (IMS) was established in
the study of Asian languages, cultures, and religions. 1764 by the East India Company. The IMS was at first meant
201. Solution (a) to look after the troops, and it recruited health professionals
by means of a competitive examination. Indians were
Exp) Option a is the correct answer.
admitted to the IMS in 1835, and the first Indian to be
After India’s independence on the 15th of August 1947, admitted was Pundit Madhusudan Gupta.
the Government of India Act, 1935 remained in force
as the working constitution until the adoption of a new 206. Solution (b)
constitution on the 26th of January 1950. During this Exp) Option b is the correct answer.

327 PYQ Workbook


HISTORY OF MODERN INDIA

Abanindranath Tagore was one of the founders of the Narayan Malhar Joshi, was a prominent Indian socialist
Indian Society of Oriental Art. The society was established and trade union leader. He is credited with creating the
in Kolkata (formerly Calcutta) in 1907 with the aim of first All India Trade Union congress in 1920 along with
promoting and preserving traditional Indian art forms Lala Lajpat Rai.
and reviving the spirit of Indian art.
Important Tips
207. Solution (b) • The All India Trade Union Congress (AITUC) was
Exp) Option b is the correct answer. established on 31st October 1920 in Bombay (now
Lord Palmerston was the Prime Minister of the United Mumbai) under the leadership of NM Joshi.
Kingdom from 1830 to 1834 and again from 1841 to 1846. • It was the first central trade union organization
He was a strong supporter of British imperialism, and he formed in India, representing the interests of workers
believed that the Russian Empire was a threat to British from various industries and sectors across the country.
interests in India. In 1836, he wrote a letter to the British
• Nearly eight years before the Indian National Congress,
Foreign Secretary, Lord Aberdeen, in which he warned that
which served as the platform for the freedom fight,
the Russian designs in Central Asia were “an imminent peril
approved a similar resolution in 1929, the AITUC
to the security and tranquility” of the Indian Empire.
supported a resolution of Swaraj (total independence
208. Solution (c) from British rule) during its second session in Jharia
Exp) Option c is the correct answer. in 1921.
The Government of India Act, 1935, provided for the creation • In the years following World War II, the AITUC
of a Federal Court, which would have original jurisdiction in played a crucial role in the creation of the World
disputes between the federal government and the provinces, Federation of Trade Unions (WTFU).
and appellate jurisdiction in appeals from the High Courts.
212. Solution (c)
The Federal Court was inaugurated on October 1, 1937,
and it functioned until the establishment of the Supreme Exp) Option c is the correct answer.
Court of India in 1950. The Madras Labour Union was founded in 1918 by B.P.
Wadia and V. K Mudaliar. It was the first trade union in
209. Solution (d)
India that was formed on modern lines. The union had
Exp) Option d is the correct answer. a clear set of objectives, including improving the working
All three women, Annie Besant, Sarojini Naidu and Hirabai conditions of the workers, raising their wages, and providing
Tata, gave evidence before the Joint Select Committee on them with social security benefits.
the Government of India Bill, 1919 in favour of female
franchise. 213. Solution (d)
Exp) Option d is the correct answer.
210. Solution (b)
Exp) Option b is the correct answer. Gopal Hari Deshmukh, also known as Lokhitawadi, was a
Marathi social reformer, writer, and journalist. He wrote a
Subhas Chandra Bose formed an alliance with the Muslim
series of articles under the pen name Lokhitawadi, which
League to destroy the Holwell Monument in Calcutta during
were published in the weekly Prabhakar. These articles
1939-40.
were later compiled into a book called Satapatra Series.
Important Tips
Important Tips
• The Holwell Monument was a memorial erected
in memory of John Zephaniah Holwell and other • The Satapatra Series was a collection of essays on
British prisoners who were tortured and killed by the social reform, education, and economic development.
Nawab of Bengal in 1756. • Deshmukh argued that India needed to undergo a
• Bose and the Muslim League argued that the period of social reform in order to achieve progress.
monument was a symbol of British oppression and • He advocated for the abolition of child marriage, the
should be destroyed. promotion of widow remarriage, and the education
• In 1940, a mob of Muslims and Hindus attacked the of women.
Holwell Monument and destroyed it. The incident • He also argued that India needed to develop its own
sparked a wave of communal violence in Calcutta. industries in order to become self-sufficient.
• The alliance between Bose and the Muslim League was • The Satapatra Series was a significant contribution
short-lived. However, it showed that Bose was willing to the social reform movement in India. It helped to
to work with other political parties in order to achieve raise awareness of the need for social reform and it
his goal of independence for India. inspired others to work for change.

211. Solution (c) 214. Solution (d)


Exp) Option c is the correct answer. Exp) Option d is the correct answer.

PYQ Workbook 328


HISTORY OF MODERN INDIA

Sir Syed Ahmed Khan, a prominent Muslim reformer, 217. Solution (c)
was the founder of the Mohammedan Anglo-Oriental Exp) Option c is the correct answer.
Defense Association (MAODA) in 1893. He founded
the organization to address the educational and political The National Liberation Federation (Liberal Party) was
concerns of the Muslim community in India and played a founded in 1919 by moderate Indian politicians such as
pivotal role in advocating for their rights and promoting Tej Bahadur Sapru and M.R. Jayakar who were dissatisfied
modern education among Muslims. with the direction of the Indian National Congress. The
party advocated for gradual constitutional reforms and
Important Tips
held a pro-British stance, arguing that the British Raj was
• Auckland Colvin was a British administrator who necessary for India’s progress and that the country was not
served as Governor of the North-Western Provinces yet ready for self-government.
from 1876 to 1880.
• Badruddin Tyabji was a Muslim lawyer and politician 218. Solution (c)
who was one of the founders of the Indian National Exp) Option c is the correct answer.
Congress. Bhagwati Charan Vohra was a revolutionary who was a
• Theodore Beck was a British educator who was member of the Hindustan Socialist Republican Association
appointed Principal of the Mohammedan Anglo- (HSRA). He was the author of the famous article titled
Oriental College (later Aligarh Muslim University) “The Philosophy of the Bomb”, which was published in
in 1877. 1930. Vohra argued that the British Raj was a brutal and
oppressive regime, and that violence was the only way to
215. Solution (d)
overthrow it.
Exp) Option d is the correct answer.
The Triveni Sangh was a political organization that was 219. Solution (d)
formed in Bihar after the First World War. It was formed Exp) Option d is the correct answer.
by the Ahirs and Kurmis, two prominent backward castes The Karachi Resolution on Fundamental Rights and
in Bihar. The leaders associated with the formation of this
Economic Policy was passed by the Indian National
front were Yadunandan Prasad Mehta, Shivpujan Singh
Congress at its Karachi session in 1931. The session was
and Jagdev Singh Yadav. The Triveni Sangh’s goal was to
presided over by Sardar Vallabhbhai Patel.
improve the social and economic conditions of the Ahirs and
Kurmis. It also advocated for their political representation. Important Tips
216. Solution (c) • Two resolutions were adopted, one on Fundamental
Rights and the other on National Economic
Exp) Option c is the correct answer.
Programme, which made the Karachi session
Shripad Balwant Tambe was the first to accept a ministerial particularly memorable.
position in the Central Provinces in October 1925. He
• The Resolution on Fundamental Rights guaranteed:
was a member of the Swaraj Party and the President of the
Central Provinces Legislative Council. He was designated as • free speech and free press
the member of the Government of Central Provinces after • right to form associations
Moropant Vishvanath Joshi’s tenure ended in the year • right to assemble
1925.
• universal adult franchise
Important Tips • equal legal rights irrespective of caste, creed and
• BS Moonje was a Hindu Mahasabha leader who was sex
elected to the Central Provinces Legislative Council • neutrality of state in religious matters
in 1923. He was a strong opponent of the Swaraj Party
• free and compulsory primary education
and he refused to accept a ministerial position in the
government. • protection to culture, language, script of minorities
and linguistic groups
• MR Jayakar was a member of the Liberal Party who
was elected to the Central Provinces Legislative • The Resolution on National Economic Programme
Council in 1923. He was a supporter of the Swaraj included:
Party and he was considered as a possible candidate for • substantial reduction in rent and revenue in the
a ministerial position. However, he ultimately declined case of landholders and peasants
the offer. • exemption from rent for uneconomic holdings
• BN Sasmal was a member of the Swaraj Party who • relief from agricultural indebtedness
was elected to the Central Provinces Legislative
• control of usury
Council in 1923. He was a close associate of Motilal
Nehru and he was considered as a possible candidate • better conditions of work including a living wage,
for a ministerial position. However, he ultimately limited hours of work and protection of women
declined the offer. workers in the industrial sector

329 PYQ Workbook


HISTORY OF MODERN INDIA

• right to workers and peasants to form unions a symbolic gesture of his commitment to the cause of non-
violence and his identification with the poor and oppressed.
• state ownership and control of key industries,
mines and means of transport 224. Solution (c)
220. Solution (d) Exp) Option c is the correct answer.
Exp) Option d is the correct answer. Simla was founded as a hill station during the course of
the Anglo-Gurkha War (1814-16). The British were looking
The Dutt-Bradley Thesis was a document written by Rajni
for a strategic location to billet troops and guard the frontier
Palme Dutt and Ben Bradley, two leaders of the Communist
against the Gurkhas. They found Simla to be an ideal
Party. The thesis argued that the Indian National Congress
location, as it was located at a high altitude and it had a
could play a great part in realizing the anti-imperialist
cool climate.
people’s front.
225. Solution (b)
Important Tips
Exp) Option b is the correct answer.
• The Dutt-Bradley Thesis was published in
International Press Correspondence (INPRECOR), A. Sekhar Bandyopadhyay - 2. From Plassey to Partition: A
the journal of Communist International, on History of Modern India
February 29, 1936. B. Sarvepalli Gopal - 1. Jawaharlal Nehru: A Biography, Vol-
• It was also called the thesis for Anti-imperialist People’s 1, 1889-1947
Front in India. C. David Hardiman - 4. Gandhi in His Time and Ours
• The Dutt-Bradley Theses primarily tried to answer the D. Gyanendra Pandey - 3. The Ascendancy of the Congress
following crucial questions: in Uttar Pradesh, 1926-1934
• How to transform the (then existing) situation?
226. Solution (c)
• How can we unite and mobilise a powerful
Exp) Option c is the correct answer.
movement of resistance to British imperialism
and for the needs of the masses? Subhas Chandra Bose started the Azad Hind Radio in
Berlin, Germany in 1942. The radio station was a tool of
• What shall be the policy of the national
representatives who are elected? propaganda against the British government, and it broadcast
news and messages to Indians living in India and abroad.
• What shall be the future line of direction of the
national struggles to defeat imperialism? 227. Solution (b)

221. Solution (c) Exp) Option b is the correct answer.

Exp) Option c is the correct answer. Madari Pasi was a peasant leader who took over the Eka
Movement in Awadh during 1921-1922. The movement
Khuntkatti tenure was a system of land tenure prevalent
was originally started by the Congress, but it took on a more
among the Munda tribes of Chota Nagpur region during radical character under Pasi’s leadership.
the British colonial rule. Under this system, the land was
owned by the entire clan or killi, and not by individuals. The Important Tips
clan members had the right to cultivate the land, and the • The Eka Movement, a peasant movement that began
produce was shared among them. in Hardoi, Bahraich, and Sitapur in late 1921, was
driven by high rents exceeding 50% of recorded rent,
222. Solution (b) oppression by rent collectors (thekedars), and the
Exp) Option b is the correct answer. practice of share rent.
Peter the Great was a 17th-century Russian emperor who • Eka meetings involved religious rituals and vows by
was known for his modernization efforts. He was also a peasants to pay recorded rent on time, refuse forced
keen observer of world affairs, and he was aware of the labor, abide by Panchayat decisions, and remain
importance of India’s economy. In a famous quote, he said, united.
“Bear in mind that the commerce of India is the commerce • Following a shift in leadership to Madari Pasi, the Eka
of the world ... he who can exclusively command it is the Movement took a more violent turn, causing a rift with
dictator of Europe.” the nationalist class. The movement ended in March
1922 due to severe repression.
223. Solution (d)
Exp) Option d is the correct answer. 228. Solution (d)
In 1921, during his tour of South India, Gandhiji shaved Exp) Option d is the correct answer.
his head and began wearing a loincloth in order to identify The Hindu Mahasabha was founded at the Haridwar
with the poor. He also gave up his Western-style clothing Kumbh Mela in 1915 by Madan Mohan Malaviya and other
and adopted the simple dress of the Indian peasant. This was Hindu leaders. The Mahasabha was a Hindu nationalist

PYQ Workbook 330


HISTORY OF MODERN INDIA

organization that aimed to protect the rights of Hindus in Brahmo Samaj, founded in 1828, was a religious and social
British India. reform movement that sought to challenge orthodox Hindu
practices and promote monotheism and social equality.
229. Solution (d)
Option 3 is incorrect- The Prarthana Samaj was founded by
Exp) Option d is the correct answer.
Atmaram Pandurang and his brother Dadoba Pandurang
Statement 1 is correct: Sir Syed Ahmed Khan advocated the in 1867.
idea that India was a composite nation consisting of various
Option 4 is incorrect- Arya Samaj was founded by Swami
ethnic communities with a shared historical and cultural
Dayananda Saraswati in 1875.
heritage.
Statement 2 is incorrect: The Indian National Congress was 233. Solution (d)
a secular organization that wanted to achieve independence Exp) Option d is the correct answer.
for India through political means. Sir Syed Ahmed Khan, on Statement 1 is correct- Gandhi believed that swaraj was
the other hand, was a Muslim leader who wanted to achieve intimately linked with ahimsa and satyagraha. He said that
progress for Muslims through education and social reform. “swaraj could not be attained through violence” and that
Statement 3 is incorrect: Sir Syed Ahmed Khan’s vision did “it could only be attained through truth and non-violence.”
not align with imagining India as a Nation State based solely Statement 2 is correct- Mahatma Gandhi believed that
on individual citizen’s rights. He believed that India should swaraj, or self-rule, was not just about political independence
be a united country with a strong central government. He from British rule. He also believed that it was about personal
also believed that Muslims should have a separate political and spiritual independence. He said that swaraj was “the
identity within India. ability to rule oneself ” and that it was “the highest form
The statement 4 is correct: Sir Syed Ahmed Khan of freedom.”
established the Mohammedan Anglo-Oriental College (later Statement 3 is correct- Gandhi believed that swaraj was
Aligarh Muslim University), which aimed to provide a something that required time and patience to acquire. He
modern education to Muslim students. The curriculum at said that “swaraj could not be obtained overnight” and that
the college combined Islamic studies and teachings with
“it would take years of hard work and sacrifice.”
a focus on Western science, philosophy, and empiricism.
Statement 4 is incorrect- Gandhi did not believe that
230. Solution (c) Swaraj could be easily or quickly obtained. He recognized
Exp) Option c is the correct answer. that it required sustained effort, perseverance, and the
participation of the masses.
“A sound body means one which bends itself to the spirit
and is always a ready instrument at its service.” This quote 234. Solution (a)
is attributed to Mahatma Gandhi. He wrote this quote in
Exp) Option a is the correct answer.
a letter to a Tamil colleague in South Africa, in 1915. The
quote reflects Gandhi’s belief in the importance of physical Statement 1 is correct- Gandhi’s exposure to various
fitness and spiritual development. He believed that a healthy religions at a young age, particularly the teachings of the
body was essential for a healthy mind, and that both were Pranami sect, which emphasized the unity of faiths, had a
necessary for spiritual growth. significant influence on his philosophy of nonviolence and
religious tolerance.
231. Solution (a) Statement 2 is correct- Gandhi’s philosophy was also
Exp) Option a is the correct answer. influenced by theosophy, which stressed spiritual
Phoenix Settlement was founded by Mahatma Gandhi development and social reform, aligning with his belief in
in 1904 in Durban, South Africa. It was a self-sufficient the interconnectedness of all life.
community that was based on the principles of Gandhi’s Statement 3 is incorrect- Gandhi’s philosophy was shaped
philosophy of non-violence and social reform. The by diverse sources such as the Bhagavad Gita, the Bible,
settlement was home to a number of Gandhi’s followers, and the writings of Leo Tolstoy and Henry David Thoreau,
and it was also a place where Gandhi could experiment which contributed to his unique philosophy of nonviolence
with his ideas about communal living and social justice. and social change.
232. Solution (c) 235. Solution (b)
Exp) Option c is the correct answer. Exp) Option b is the correct answer.
Option 1 is correct- Raja Ram Mohan Roy, a prominent Statement 1 is correct- Mahatma Gandhi charged the
social reformer of the 19th century, founded Atmiya Sabha. Indian elite with a lack of concern for the laboring poor.
Atmiya Sabha was established in 1814 as a socio-cultural In his speech, he criticized the privileged class for their
society that aimed to promote education, social reforms, detachment from the struggles and hardships faced by the
and the exchange of ideas. common working people. Gandhi believed that the elite
Option 2 is correct- Raja Ram Mohan Roy, a prominent should actively work towards uplifting the marginalized
social reformer of the 19th century, founded Brahmo Samaj. and advocating for their rights.

331 PYQ Workbook


HISTORY OF MODERN INDIA

Statement 2 is correct- Gandhi asserted that the salvation Statement 1 is correct: Marxist theory also rejects the idea
of India can come only through the farmers. He recognized of the individual as an isolated and self-interested agent,
the pivotal role of agriculture and the farmers in the and instead emphasizes the social and historical nature of
country’s progress. Gandhi believed that the well-being and human existence and the role of class struggle in shaping
empowerment of the farming community were essential for history.
the overall development and prosperity of the nation.
Statement 2 is incorrect: Marxist theory challenges the
Statement 3 is incorrect- Gandhi did not mention the liberal notion of individual rights as being opposed to
untouchables in his speech. Gandhi was a strong advocate collective rights or social obligations, and instead advocates
for the rights of the untouchables, and he spoke out against for a socialist society where individual freedom is realized
the caste system on many occasions. However, he did not
through social equality and solidarity.
mention the untouchables in his speech at the opening of the
Banaras Hindu University. 239. Solution (c)
Statement 4 is incorrect- Gandhi did not promise to take Exp) Option c is the correct answer.
up the cause of mill owners of Ahmedabad in his speech at Both Kirti Kisan Party and Labour Swaraj Party were
the Banaras Hindu University. The focus of his speech was
connected primarily to the communist ideology.
primarily on social and educational issues rather than
specific economic concerns. Important Tips
• Kirti Kisan Party:
236. Solution (a)
Exp) Option a is the correct answer. • It was a political party in India, which worked
within the Indian National Congress in 1925–1929.
Sumit Sarkar: He is a historian known for his work on the
social and political history of modern India, particularly his • It became an important front organisation for the
book “The Swadeshi Movement in - Bengal, 1903-1908” Communist Party of India and an influential force
which delves into the Swadeshi movement as a form of anti- in the Bombay labour movement
colonial resistance in Bengal. • Labour Swaraj Party:
Shahid Amin: His book “Event, Metaphor, Memory-Chauri • It was founded in Bengal on 1 November 1925, as
Chaura, 1922-1992” explores examines the Chauri Chaura the Labour Swaraj Party of the Indian National
incident and its enduring impact on Indian nationalism. Congress.
Ranajit Guha: He was an influential historian known for his • The founding leaders of the party were Kazi Nazrul
contribution to the Subaltern Studies project. His book “A Islam, Hemanta Kumar Sarkar, Qutubuddin Ahmad
Rule of Property for Bengal” examines the dynamics of land and Shamsuddin Hussain.
ownership in Bengal during the colonial period.
240. Solution (a)
Bipan Chandra: He was a prominent Indian historian who
specialized in modern Indian history. He wrote the book Exp) Option a is the correct answer.
“The Rise and Growth of Economic Nationalism in India”. Raja Ram Mohan Roy did not belong to the gentry class
whose power was diminished due to the imposition of the
237. Solution (d)
Permanent Settlement. In fact, he came from a privileged
Exp) Option d is the correct answer. and influential family of Bengal. His father, Ramkanta Roy,
Statement 1 is correct: Muslim merchants were actively was a wealthy landowner and revenue collector. Ram Mohan
involved in Gandhian political movements in South Roy himself had a comfortable upbringing and had exposure
Africa. Gandhi had a close relationship with many Muslim to education and intellectual pursuits from an early age.
merchants who supported his campaigns against racial
Option b is correct: He played a significant role in bridging
discrimination and unjust laws.
Eastern and Western philosophies and studied different
Statement 2 is correct: Mahatma Gandhi played a key religious and philosophical traditions, including Vedantic
role in leading a campaign against the Transvaal Asiatic Monism and Christian Unitarianism.
Registration Act of 1906, which required Indians in South
Option c is correct: Raja Ram Mohan Roy was also involved
Africa to carry identification passes. This campaign was a
significant moment in Gandhi’s activism and marked a shift in the translation of ancient Indian texts into vernacular
towards nonviolent civil disobedience. languages. He translated the Upanishads into Bengali to
make them more accessible to the common people.
Statement 3 is correct: One of Gandhi’s earliest political
struggles in South Africa was against the imposition of Option d is correct: His first organization was the Atmiya
discriminatory taxes and fees on the Indian community in Sabha, which he founded in Calcutta in 1815. The Atmiya
Cape Town. This marked his initial foray into activism and Sabha was a socio-cultural association that aimed to promote
fighting for the rights of the Indian community. social reforms and intellectual discussions.

238. Solution (a) 241. Solution (a)


Exp) Option a is the correct answer. Exp) Option a is the correct answer.

PYQ Workbook 332


HISTORY OF MODERN INDIA

“Hind Swaraj” is a book written by Mahatma Gandhi in He became a Japanese citizen, married, learned Japanese,
1909. It is a dialogue between the Reader and the Editor, and engaged in cultural activities while promoting India’s
where Gandhi expresses his views on various social, political, perspective.
and economic issues.
Rash Behari, along with Captain Mohan Singh and
In this book, Gandhi outlines his vision of Swaraj (self- Sardar Pritam Singh, played a key role in establishing the
rule) for India and presents his ideas on nonviolence, Indian National Army (Azad Hind Fauj) on September
passive resistance, decentralization, and the importance of 1, 1942, with the support of the Japanese.
moral values in society. Rash Behari Bose passed away on January 21, 1945,
in Tokyo, and the Japanese government posthumously
242. Solution (b)
honored him with the Second Order of Merit of the Rising
Exp) Option b is the correct answer. Sun.
Statement 1 is False: J.B. Kripalani established the Kisan
244. Solution (d)
Mazdoor Praja Party (KMPP) in June 1951. The party
was formed by Indian National Congress dissidents and Exp) Option d is the correct answer.
contested the 1951–52 Indian general election, winning ten Sir George Barlow was the acting Governor-General of India
seats across several states. In September 1952, it merged with from 1805 to 1807, and he was in charge when the Vellore
the Socialist Party to create the Praja Socialist Party. mutiny took place on 10 July 1806. He was blamed for his
negligence and indifference towards the grievances of the
Statement 2 is True: Both Prafulla Chandra Ghosh and
Indian sepoys, which led to the revolt. He was also criticized
Tanguturi Prakasam were associated with the Kisan Mazdoor
for his harsh and arbitrary measures to suppress the mutiny,
Praja Party, and they had been chief ministers of West Bengal
such as dismissing the entire Madras army and abolishing
and Madras, respectively.
the local courts. He was eventually recalled to England in
243. Solution (a) 1807 and replaced by Lord Minto.
Exp) Option a is the correct answer. 245. Solution (b)
Rash Behari Bose, the revolutionary, made the statement: Exp) Option b is the correct answer.
“Avenge the blood of Indian Martyrs. Use your sword to The famous book “The Foundations of Indian Culture”
liberate your motherland. Stand up against the entire is authored by Sri Aurobindo. This book is a collection of
Anglo-American enemy.” This statement reflected his articles that originally appeared in the monthly review Arya
strong commitment to the cause of Indian independence and from December 1918 to January 1921. Sri Aurobindo’s work
his call for armed resistance against British colonial rule. He in this book represents a synthesis of the teachings of both
played a significant role in organizing the Azad Hind Fauj the West and the East, emphasizing the concept of a Divine
(Indian National Army) and mobilizing Indian soldiers and Life on Earth and the cultural foundations of India.
citizens in the fight for freedom.
246. Solution (a)
Important Tips
Exp) Option a is the correct answer.
Rash Behari Bose:
Aga Khan was elected as life-long President of All India
Born on May 25, 1886, in Subaldaha, Bengal, Rash Behari Muslim League in March 1908. He was one of the founders
Bose was orphaned at a young age, raised by his maternal and the first president of the All-India Muslim League. He
aunt. was nominated to represent India to the League of Nations
Rash Behari’s patriotism blossomed after reading in 1932 and served as the President of the League of Nations
revolutionary literature, including Bankim Chandra from 1937-38.
Chatterjee’s “Ananda Math” and Navin Sen’s “Plasir Yudha.”
247. Solution (a)
Influenced by teacher Charu Chand in Chandernagore, he
embraced revolutionary ideals. Exp) Option a is the correct answer.

The 1905 partition of Bengal triggered Rash Behari’s Option A in List-I matches Option 2 in List-II: Mahatma
entry into revolutionary activities, led by Jatin Banerjee. Gandhi, after returning from the Second Round Table
Conference in London in 1931, relaunched the Civil
He played a crucial role in the 1912 bomb attack on Lord
Disobedience Movement. Many congress leaders were
Hardinge, Viceroy of India.
arrested as a result of this. In January 1932, Mahatma Gandhi
Rash Behari was instrumental in organizing the Ghadar and Sardar Patel were also imprisoned and sent to Yerwada
Revolution, mobilizing thousands of Indians, including Jail.
those from the Ghadar Party in America, Canada, and the
Option B in List-I matches Option 4 in List-II: Gandhiji
Far East
went on a hunger strike in Yerwada prison in 1932 to protest
Rash Behari escaped to Japan in 1915, living under the against the Communal Award, which he believed was
identity of Raja P.N.T. Tagore. discriminatory against Hindus.

333 PYQ Workbook


HISTORY OF MODERN INDIA

Option C in List-I matches Option 1 in List-II: Throughout of Britain Prime Minister Clement Atlee. He served as the
Gandhi’s route to Karachi, he was greeted with black flag King of the United Kingdom and the Dominions of the
demonstrations by the Punjab Naujawan Bharat Sabha, in British Commonwealth from December 11, 1936, until his
protest for endorsing Delhi pact and against his failure to death. He held the position of the last Emperor of India and
secure commutation of the death sentence for Bhagat and was also the first Head of the Commonwealth.
his comrades.
252. Solution (b)
Option D in List-I matches Option 3 in List-II: In 1939
Subhas Chandra Bose won the presidency against the wishes Exp) Option b is the correct answer.
of Mahatma Gandhi. Gandhi considered the defeat of his Cleveland was the district collector of Bhagalpur in
preferred candidate, Dr. Pattabhi Sitaramayya, a personal Bihar and he introduced the ‘Hill Assembly Plan’ for the
blow. He expressed that since he had persuaded Sitaramayya development of Adivasis in 1784. He placed the entire
not to withdraw his name, the defeat felt more like his own. range of hills inhabited by Pahariyas under a uniform
Gandhi acknowledged Bose’s capabilities and suggested administration of Hill assembly under hereditary hill chiefs,
that he should form his own working committee and lead called sardars. The assembly met twice a year and had wide
Congress independently. This internal division ultimately led powers including that of inflicting or rescinding capital
to Subhas Chandra Bose’s resignation from the Congress and punishment. Sardars were entrusted the duty of reporting
his pursuit of an alternative path for India’s independence. all crimes and law and order problems in the villages under
their control to the authorities. Cleveland also established
248. Solution (b) a regular market for Adivasis to sell forest produces and
Exp) Option b is the correct answer. provided arms to Paharia archers like regular police. He was
Great Indian Peninsula Railway (GIPR) was the first regarded as the ‘Father of the Pahariyas’ by them and his
railway company in India that started the first railway death in 1784 was mourned by them.
service in India on 16 April 1853. The first train ran from 253. Solution (b)
Bombay (now Mumbai) to Thane, covering a distance of 34
kilometers with 14 coaches and 400 passengers. The train Exp) Option b is the correct answer.
was flagged off by Lord Dalhousie, the Governor-General of Governor-General of Bengal declared a proclamation on
India at that time. The GIPR was incorporated in England in July 22, 1789, banning the export of slaves from the Bengal
1845 and was one of the largest and oldest railway networks Presidency. Slavery was a prevalent social institution in
in Asia. It later became part of the Central Railway zone of Bengal in the 18th century, and many slaves were exported
Indian Railways. to other regions by Dutch, Burmese, and Portuguese traders
and pirates. The proclamation of 1789 was one of the first
249. Solution (b) attempts to curb the slave trade in India. Slavery was finally
Exp) Option b is the correct answer. abolished in British India by the Indian Slavery Act of 1843.
“Guilty Men of India’s Partition” is a book authored by Dr. 254. Solution (a)
Ram Manohar Lohia. In this insightful work, Lohia discusses
the painful days leading up to the partition of India, Exp) Option a is the correct answer.
identifying the leaders and circumstances responsible for Lord Dalhousie was the Governor-General of India from
this significant event. He shares his experiences of being 1848 to 1856, and he founded the Public Works Department
sidelined and ignored by influential leaders, shedding light (PWD) as a separate branch of the Government of India
on the complex decision-making process that led to the in 1854. He established a central agency for execution of
partition of India. public works and set up Ajmer Provincial Division. He also
appointed a committee under Sir Macdonald Stephenson to
250. Solution (c) survey and plan the railway routes in India.
Exp) Option c is the correct answer. The PWD was responsible for the construction and
The Satyagraha Ashram was founded on May 25, 1915 in maintenance of roads, railways, bridges, irrigation, and other
Ahmedabad at Kochrab, when Gandhi returned from South public utility works in India. Lord Dalhousie’s initiative was
Africa, with 25 inmates. The Ashram was shifted on the aimed at improving the infrastructure and development
bank of river Sabarmati on 17 June, 1917. It was one of the of the country, as well as facilitating trade, commerce,
main centres of the Indian freedom struggle and Gandhi’s communication, and military operations.
residence until 1930 It was from here that Gandhi led the
famous Dandi March or the Salt Satyagraha in 1930. 255. Solution (b)
Exp) Option b is the correct answer.
251. Solution (b)
Dadabhai Naoroji was the first Indian selected as a member
Exp) Option b is the correct answer. of the British Parliament. He was elected as the Member of
George VI was the British monarch at the time of India’s Parliament (MP) for Finsbury Central as a candidate of
independence in 1947. He succeeded his brother, Edward the Liberal Party in the 1892 general election. Notably, he
VIII, who abdicated the throne in 1936. He played a refused to take the oath on the Bible due to his Zoroastrian
significant role in the transition of power from Britain faith and instead took the oath on his copy of the Khordeh
to India. He appointed Lord Mountbatten on the advice Avesta. During his time in Parliament, he worked to improve

PYQ Workbook 334


HISTORY OF MODERN INDIA

the situation in India and advocated for Indian issues, Free Sarvodaya Society from Party Politics: Narayan
making him a significant figure in the Indian independence emphasized the need to separate the Sarvodaya society
movement. from party politics, allowing it to operate independently.
Important Tips Invite All Parties to Participate in Sarvodaya Work: He
Other Indian origin leaders who were members of suggested inviting all political parties to participate in
British Parliament: Sarvodaya activities, fostering cooperation and ending the
divisive spirit of political parties.
• Mancherjee Merwanjee Bhownaggree KCIE (1851-
1933) was a British Conservative Party politician Candidates to Break from Parties After Election:
with Indian Parsi ancestry. He served as a Member Narayan proposed that elected candidates should
of Parliament (MP) representing the Bethnal Green dissociate themselves from their respective parties after
Northeast constituency in the United Kingdom’s winning elections. This would allow them to represent the
Parliament (1895-1906). Notably, he was the third people independently and avoid the constraints of party
British MP of Indian descent, following in the ideologies and discipline.
footsteps of David Ochterlony Dyce Sombre and his
258. Solution (a)
fellow Parsi compatriot Dadabhai Naoroji.
Exp) Option a is the correct answer.
• David Ochterlony Dyce Sombre (1808–1851), also
known as D. O. Dyce Sombre and David Dyce Sombre, M. Visvesvaraya was a noted Indian engineer, scholar,
was an Anglo-Indian held to be the first person of statesman, and the Diwan of Mysore from 1912 to 1918.
Asian descent to be elected to the British Parliament. He was a recipient of the Indian Republic’s highest honor,
He was elected to represent the Sudbury constituency the Bharat Ratna, in 1955. Visvesvaraya was also the first
in July 1841, but was removed in April 1842 due to to write about the economic planning of the country in
bribery in the election. a book titled ‘Planned Economy for India’ published in
1934. The book proposed a ten-year plan with the aim of
256. Solution (c) doubling the income of the country. His main emphasis
Exp) Option c is the correct answer. was on industrialization so as to reduce the population
On January 26, 1930, during the Lahore session of the dependent on agriculture and to increase the population
Congress under the leadership of Jawaharlal Nehru, it employed in the industry.
was declared as the “Purna Swaraj day” or first/complete
259. Solution (a)
independence day. So, to align with that significant day, the
new constitution was enacted on January 26, 1950, marking Exp) Option a is the correct answer.
the 20th anniversary of the Purna Swaraj day. Popularly known as Rajaji, Chakravarti Rajagopalachari
was the first and last Indian Governor-General of
257. Solution (d) Independent India from 1948 to 1950 and he was also first
Exp) Option d is the correct answer. Indian Governor of Madras from 1952 to 1954. He was the
Jayaprakash Narayan advocated for a “Party Less supporter of Gandhi and participated in Non-Cooperation
Democracy” in India. He believed that while parties, Movement in 1920. He also edited Gandhi’s newspaper
elections, and the constitution were crucial for democracy, ‘Young India’. He also authored book “Environmentalism:
true democracy would only flourish when moral values and A Global History”.
spiritual qualities were ingrained in the public. He argued
260. Solution (b)
that the party system often led to a disconnect between
people and their representatives, emphasizing the need for a Exp) Option b is the correct answer.
more direct and morally grounded democratic process. The Constituent Assembly provisionally assumed sovereign
power at midnight on August 14/15, 1947. In a special
Important Tips
session held in New Delhi, where Jawaharlal Nehru, India’s
Jayaprakash Narayan proposed four key suggestions for first Prime Minister, delivered a historic speech at midnight
implementing a partyless democracy: on August 14, 1947, in the Constituent Assembly. His speech
Abolish Political Parties: Narayan recommended the was brimming with inspiring ideas as the nation embarked
elimination of political parties from the democratic system. on a new journey to draft a Constitution, leading to India
Instead, he advocated for direct elections of candidates becoming a Sovereign, Socialist, Secular, Democratic, and
at all levels, from villages to the central government. Republic nation.
Each gram sabha would elect two representatives to the
electoral council for their constituency, where candidates 261. Solution (a)
for state legislatures or the central parliament would be Exp) Option a is the correct answer.
proposed and supported in open meetings, ensuring After August 15, 1947, the part of India that remained
public participation. Candidates with over 30% of the vote under Portuguese control was Goa. Goa was a Portuguese
would become representatives. colony and continued to be under Portuguese rule until

335 PYQ Workbook


HISTORY OF MODERN INDIA

it was annexed by India in December 1961. Portugal Option B in List-I matches Option 2 in List-II: Ahilya Bai
recognized Indian control over Goa in a treaty signed on Holkar was the hereditary noble queen of the Maratha
December 31, 1974, following the Carnation Revolution Empire, in early-modern India. She established Maheshwar
and the fall of the Estado Novo regime in Portugal. (in Madhya Pradesh) as the seat of Holkar Dynasty. She
defended Malwa from invaders, led armies alongside her
Important Tips
military commander Tukoji Rao Holkar, and is renowned for
Timeline of Goa’s Integration into the Indian Union: constructing numerous Hindu temples and Dharmashalas
• Post-Independence Relations (1947-1949): After across India.
India gained independence in 1947, diplomatic Option C in List-I matches Option 1 in List-II: Rani
relations were established with Portugal in 1949. Lakshmibai of Jhansi, a prominent figure in the Indian
• Deterioration of Relations (1950): Relations between Rebellion of 1857, valiantly resisted British rule. After her
India and Portugal deteriorated due to Portugal’s husband’s death and the British annexation of Jhansi, she led
refusal to surrender its enclaves on India’s west coast, the rebellion, defending Jhansi successfully but ultimately
including Goa, Daman and Diu, and Dadra and Nagar falling in battle at Gwalior in 1858.
Haveli.
Option D in List-I matches Option 4 in List-II: Razia
• Recognition as Overseas Province (1951): In 1951, Sultana, the daughter of Iltutmish, made history as the first
Portugal changed its constitution, recognizing Goa as female Muslim ruler of Delhi. Her ascension faced challenges
an overseas province rather than a colonial possession. from nobles, leading to her deposition in 1240 after a brief
This move was seen as an attempt to invoke NATO’s rule of less than four years. She made a failed attempt to
collective security clause in case of an Indian attack.
regain power and was ultimately killed.
• Diplomatic Break (1955): By 1955, diplomatic
relations between India and Portugal had been severed, 264. Solution (d)
leading to a crisis. Exp) Option d is the correct answer.
• Liberation of Goa (1961): In 1961, India launched “Chandrakanta” is an epic fantasy Hindi novel written
Operation Vijay on December 19th, leading to the by Devaki Nandan Khatri in 1888. It is the first modern
annexation of Daman and Diu, as well as Goa, and the Hindi novel and played a significant role in popularizing the
end of Portuguese rule over these enclaves. Hindi language. The story revolves around the love between
• Operation Vijay involved air, sea and land strikes for Princess Chandrakanta of Vijaygarh and Prince Virendra
over 36 hours, and was a decisive victory for India, Singh of Naugarh, set against the backdrop of rival kingdoms
ending 451 years of rule by Portugal over its remaining and magical tilisms (enchantments). The novel has inspired
exclaves in India. The engagement lasted two days, and adaptations, including a popular TV series, and has several
twenty-two Indians and thirty Portuguese were killed sequels exploring further adventures in magical realms.
in the fighting.
265. Solution (a)
262. Solution (d) Exp) Option a is the correct answer.
Exp) Option d is the correct answer. Warren Hastings was the first governor-general of
Lord Minto was the Governor-General of India from 1905 Bengal from 1772 to 1785, and he was impeached by the
to 1910, and he used the system of separate electorates to British Parliament in 1787 on charges of misconduct and
win the support of the Muslims and to create a rift between corruption during his time in Calcutta. The trial lasted for
them and the Congress. He implemented the Morley- seven years and involved 22 charges against Hastings, such
Minto Reforms in 1909, which granted separate electorates as the Rohilla War, the case of Nanda Kumar, the treatment
to Muslims for the election of members of provincial and of Raja Chait Singh of Benares, and the pressures on the
imperial legislative councils. This meant that Muslims could Begums of Oudh. Hastings defended himself vigorously and
vote only for Muslim candidates, and not for candidates from was eventually acquitted in 1795, but his reputation and
other communities. The reforms also increased the number fortune were damaged by the ordeal.
of elected Indian members in the councils, but gave them
266. Solution (a)
limited powers and functions.
Exp) Option a is the correct answer.
263. Solution (a)
The idea given by Gandhiji for family planning was self-
Exp) Option a is the correct answer. control. He believed that self-control was the best and the
Option A in List-I matches Option 3 in List-II: Rani most natural way of regulating the size of the family and
Durgavati was the ruling Queen of Garha Kingdom, also preventing unwanted pregnancies. He said, “Self-control is
called Garha Mandla or Garha Katanga from 1550 until 1564. the surest and the only method of regulating the birth-rate.
She was married to Dalpat Shah, the son of king Sangram Birth-control by contraceptives is race-suicide.” He also said,
Shah of the Garha Kingdom. She is chiefly remembered for “The use of contraceptives is likely to produce evils of which
defending her kingdom against the Mughal Empire. we have no conception.

PYQ Workbook 336


HISTORY OF MODERN INDIA

Gandhiji was opposed to any artificial or mechanical methods with Bhagat Singh and Sukhdev studied on the model of
of family planning, such as sterilization, restrain, or loop. He the Russian Socialist Revolution. In 1929, he planned bomb
considered them as immoral, unnatural, and harmful to the blast under the train of Viceroy Lord Irwin on the DelhiAgra
health and dignity of the individuals and the society. railway line. The viceroy escaped unhurt. Mahatma Gandhi
condemned the revolutionary act through his article The
267. Solution (a) Cult of Bomb.
Exp) Option a is the correct answer.
Important Tips
The book “Prison Diary” was written by Jai Prakash
Delhi Conspiracy Case:
Narayan (JP). It is a collection of JP’s writings and reflections
during his imprisonment during the Emergency period in The Delhi-Lahore conspiracy is known for the bombing of
India, which lasted from June 1975 to March 1977. In the incident Viceroy of India Lord Hardinge.
book, JP shares his thoughts, experiences, and observations The event took place when he was travelling to Delhi on
about the political environment of the time, as well as his the occasion of the relocation of the capital from Calcutta
personal struggles and loneliness while in prison. The diary to New Delhi.
provides valuable insights into the challenges faced by Rasbihari Bose is considered the pioneer of this conspiracy.
political prisoners during that period. Other important revolutionaries who were involved in this
act were:
268. Solution (a)
Basant Kumar Vishwas,
Option a is the correct answer.
Bal Mukund,
Ramadevi Choudhury, also known as Rama Devi, was
Awadh Bihari and
a revolutionary woman who played a significant role in
India’s independence movement. She actively participated Master Amir Chand
in various movements, including the Peasant Movement They were hanged on the charges of this crime.
of Bijoliya, the 1930 Satyagraha, and the 1932 Civil Rasbihari Bose escaped arrest and escaped to Japan.
Disobedience Movement. Due to her dedication to the
cause, she was arrested multiple times and sentenced to jail. 271. Solution (c)
Rama Devi was highly influenced by Mahatma Gandhi and Exp) Option c is the correct answer.
worked tirelessly to mobilize women and promote the ideals The book “Gandhian Constitution for Free India” was
of the independence struggle. authored by Shriman Narayan Agarwal. Published in 1946,
it focuses on Mahatma Gandhi’s ideals of non-violence,
269. Solution (b)
harmony, and his concepts on education, trade, and
Exp) Option b is the correct answer. property. Agarwal’s work seeks to synthesize these Gandhian
Charles Metcalf was the British Resident at Delhi who principles into a document that could inspire the direction
negotiated Subordinate Alliances of 1817-18 with the taken by independent India. The book analyzes Gandhian
princely states of Rajputana. These alliances brought the philosophy to outline insights into framing a constitution
Rajputana states under the complete subordination of the for a free India, emphasizing the importance of combining
British. fundamental rights with the duties of the people. Mahatma
Gandhi himself found the ideas in the book consistent with
Important Tips his own beliefs.
• Charles Metcalf was a British colonial administrator
272. Solution (a)
who served as the Governor-General of India from
1835 to 1836. Exp) Option a is the correct answer.
• He also played a key role in suppressing the Pindari The first historian to write about ‘Feudalism’ in Rajasthan
raids and the Third Anglo-Maratha War. during the 19th century was Col. James Tod. He documented
the contemporary geography and history of Rajputana
• He was known for his liberal and reformist policies,
and Central India, along with the history of the Rajput
such as abolishing censorship of the press and granting
clans ruling the region at that time, in his work “Annals
equal rights to Indians.
and Antiquities of Rajasthan.” His writing, published in
• He was also a scholar of Indian languages and culture, two volumes in 1829 and 1832, included illustrations and
and wrote several books and essays on Indian history engravings by notable artists and significantly influenced the
and society popular and academic understanding of Rajasthan’s history
and heroes.
270. Solution (b)
Exp) Option b is the correct answer. 273. Solution (d)
Bhagwati Charan Vohra was not involved in the Hardinge Exp) Option d is the correct answer.
Bomb incident. He was an Indian revolutionary, associated The main aim of the East India Company to make a subsidiary
with Hindustan Socialist Republican Association. He along alliance in Rajput states was to extend its political and

337 PYQ Workbook


HISTORY OF MODERN INDIA

economic control over them and to prevent any possible 275. Solution (d)
challenge from them or other powers. The subsidiary Exp) Option d is the correct answer.
alliance was a system devised by Lord Wellesley, the Gandhian Innovation means to produce more from less
Governor-General of India from 1798 to 1805, by which the input for more people. It is a term coined by Prof. C.K.
Indian rulers had to accept the presence and maintenance of Prahalad and Dr. R.A. Mashelkar to describe a type of
a British force in their territory and surrender their foreign innovation that aims to serve the needs of the poor and the
policy to the British. In return, the British promised to marginalized by creating affordable and sustainable products
protect them from any external or internal threats. and services with fewer resources.
One of the examples of Gandhian Innovation is the Jaipur
Important Tips
Foot, which is a prosthetic limb that can be fitted in less than
• The Rajput states were among the last to enter into an hour and costs only $30, compared to $8,000 for a similar
the subsidiary alliance with the British, as they had product in the West.
resisted their domination for a long time.
• They were also reluctant to give up their autonomy and 276. Solution (b)
sovereignty to the foreign power. However, they were Exp) Option b is the correct answer.
eventually forced or persuaded to sign the treaty after The book “Indian National Movement: The Long-Term
the Third Anglo-Maratha War (1817-1818), which Dynamics” is written by Bipan Chandra. This book is a
resulted in the defeat of the Maratha Confederacy and significant contribution to the study of India’s freedom
the annexation of most of its territories by the British. struggle and explores various aspects of the movement,
• The Rajput states that signed the subsidiary alliance including its ideology, program, the role of non-violence,
with the British were: Jaipur (1818), Jodhpur (1818), the relationship between leaders and the masses, and the
Udaipur (1818), Kota (1818), Bundi (1818), Bikaner movement’s capacity for ideological transformation.
(1818), Alwar (1818), Bharatpur (1826). 277. Solution (a)
Exp) Option a is the correct answer.
274. Solution (b)
Gandhi said that poverty is the worst form of violence
Exp) Option b is the correct answer.
because it deprives people of their basic human needs and
Yarvada Central Jail is the largest jail in the state of dignity. He said, “Poverty is the cruelest form of violence. It
Maharashtra. Mahatma Gandhi spent several years during strikes not only at the individual, but at his family and his
India’s struggle for freedom. He named Yarvada Jail as future generations as well.” He also said, “There is enough
Mandir. He wrote ‘Yerwada Mandir’ (Ashrama Observance) for everyone’s need, but not for everyone’s greed.”
first published in 1932, translated from Gujarati, by Valji Gandhi was deeply concerned about the plight of the
Govind Desai. poor and the oppressed in India and elsewhere. He fought
against the British colonial rule and the exploitation of
Important Tips
the Indian masses by the landlords and the capitalists. He
• Naini: Naini Central Prison is a jail located in also campaigned for social reforms such as abolition of
Allahabad, Uttar Pradesh. It was built by the British in untouchability, emancipation of women, and promotion of
1860 and was used to imprison many freedom fighters education and health. He advocated for a simple and self-
and revolutionaries such as Chandra Shekhar Azad, reliant lifestyle based on the principles of truth, non-violence,
Ram Prasad Bismil, Ashfaqulla Khan, and Roshan and service. He said, “The best way to find yourself is to
Singh. lose yourself in the service of others.”
• Cellular (Port Blair): Cellular Jail is a colonial prison
located in Port Blair, Andaman and Nicobar Islands. 278. Solution (c)
It was built by the British between 1896 and 1906 and Exp) Option c is the correct answer.
was used to exile political prisoners and activists who Mahatma Gandhi adopted the concept of ‘Seva Dharma’ or
opposed the British rule in India. It was also known the duty of service during his stay in South Africa, where
as “Kala Pani” or “Black Water” because of the harsh he rendered his services as an attendant to the helpless poor
conditions and torture faced by the inmates. patients in the charity hospitals of Rustumji. He also served
• Aghakhan Palace: Aghakhan Palace is a historical there the wounded soldiers of the Boer War. He said, “The
building located in Pune, Maharashtra. It was built by best way to find yourself is to lose yourself in the service of
Sultan Muhammed Shah Aga Khan III in 1892 and was others.”
donated to India by his son Aga Khan IV in 1969. It ‘Seva Dharma’ was one of the key principles of Gandhi’s
was used as a prison by the British during the Quit philosophy and practice. He believed that service to humanity
India Movement in 1942 and housed Mahatma Gandhi, is service to God, and that everyone should dedicate their
Kasturba Gandhi, Mahadev Desai, Sarojini Naidu, and lives to the welfare of others. He said, “Service without
others. humility is selfishness and egotism.” He also said, “Service

PYQ Workbook 338


HISTORY OF MODERN INDIA

which is rendered without joy helps neither the servant nor 281. Solution (a)
the served. But all other pleasures and possessions pale into Exp) Option a is the correct answer.
nothingness before service which is rendered in a spirit of
The statement that is not true as per Gandhian Principle
joy.”
is- The aim of Satyagrahi is to defeat the enemy. This is
279. Solution (d) because Gandhi did not consider his opponents as enemies,
Exp) Option d is the correct answer. but as friends who were misguided or ignorant of the
truth. He said, “I do not believe in short-victories over an
Lord Lytton was the Viceroy of India from 1876 to 1880,
opponent. I do not want to end his wickedness by destroying
and he was responsible for initiating the Second Anglo-
him. I want to convert him.” He also said, “The real victory
Afghan War in 1878, not the first one. The First Anglo-
is when your opponent realizes the truth and changes his
Afghan War was fought from 1839 to 1842, during the
mind.” Therefore, the aim of Satyagrahi was not to defeat the
reign of Lord Auckland as the Governor-General of India.
enemy, but to win him over by appealing to his conscience
The British invaded Afghanistan to replace the emir Dost
and reason.
Mohammad Khan, who was seen as pro-Russian, with the
exiled Shah Shuja, who was more favorable to Britain. 282. Solution (b)
Important Tips Exp) Option b is the correct answer.
• Lord Dalhousie was the Governor-General of India Mahatma Gandhi believed that politics meant the service of
from 1848 to 1856, and he annexed Awadh in 1856 the people and the nation. He said, “Politics divorced from
under the Doctrine of Lapse, which allowed the British religion has absolutely no meaning for me.” He also said,
to take over any Indian state whose ruler died without “Politics should be sacred and not a game of parties.
a natural heir. Gandhi’s politics was based on the principles of truth, non-
• Lord Dufferin was the Viceroy of India from 1884 violence, and self-rule. He fought for India’s independence
to 1888, and he favoured the formation of the Indian from the British colonial rule and for the upliftment of
National Congress in 1885 as a platform for the the oppressed and exploited sections of the society. He
educated Indians to express their grievances to the also advocated for social reforms such as abolition of
British. untouchability, emancipation of women, promotion of
• Lord William Bentinck was the Governor-General education and health, and protection of environment.
of Bengal from 1828 to 1834 and the first Governor-
283. Solution (d)
General of India from 1834 to 1835. The Charter Act of
1833 was passed during the reign of William Bentick, Exp) Option d is the correct answer.
which ended the commercial activities of the East Gandhi did not believe in the legitimacy or necessity of
India Company and made it a purely administrative any form of centralized authority or state. He said, “I
body. The Act also gave exclusive legislative powers to look upon an increase in the power of the State with the
the Governor-General in Council and stated that no greatest fear because, although while apparently doing good
native of India should be disabled from holding any by minimizing exploitation, it does the greatest harm to
office by reason of his religion. mankind by destroying individuality which lies at the root of
280. Solution (d) all progress.” He also said, “The State represents violence in a
concentrated and organized form. The individual has a soul,
Exp) Option d is the correct answer.
but as the State is a soulless machine, it can never be weaned
The strike was the last step in the Gandhian strategy of from violence to which it owes its very existence.”
satyagraha, while fasting was the first step. Gandhi termed
fasting as ‘Fire Arrow’. He used strike as a means of exerting 284. Solution (c)
pressure on the authorities and demonstrating the strength Exp) Option c is the correct answer.
and unity of the people. He also used boycott, picket, and Mahatma Gandhi’s boycott of British made products was
civil disobedience as other methods of satyagraha in India. effective because the British considered India as a major
Important Tips market for their goods. They exported various items such
as textiles, machinery, iron, steel, and salt to India and made
Satyagraha was a nonviolent resistance to evil based on the
huge profits from them. They also imposed high tariffs and
principles of truth and moral courage.
taxes on Indian goods and discouraged the development of
Gandhi coined the term in 1908 to distinguish it from indigenous industries. This resulted in the exploitation and
passive resistance, which he felt was inadequate and impoverishment of the Indian people.
foreign.
Gandhi launched the swadeshi or self-reliance movement in
He derived the concept from the ancient Indian ideal 1905, which involved boycotting foreign goods and promoting
of ahimsa (non-injury), as well as from the writings of Indian goods. He encouraged people to burn or discard their
Leo Tolstoy, Henry David Thoreau, the Bible, and the foreign clothes and wear khadi or hand-spun cloth instead.
Bhagavadgita. He also supported the growth of cottage industries, such as

339 PYQ Workbook


HISTORY OF MODERN INDIA

spinning, weaving, pottery, and leatherwork. He believed violate the principles of non-violence, truth and justice.
that this would not only weaken the economic and political He wrote: “I do not want my house to be walled in on all
hold of the British over India, but also foster a sense of sides and my windows to be stuffed. I want the cultures of all
national pride and dignity among the Indians. lands to be blown about my house as freely as possible. But I
refuse to be blown off my feet by any.”
285. Solution (d)
Exp) Option d is the correct answer. 288. Solution (d)

Mahatma Gandhi’s concept was influenced by the works of Exp) Option d is the correct answer.
Ruskin, Thoreau, and Tolstoy, who were all secular writers According to M.K. Gandhi, socio-economic improvement
and thinkers. They shared some common themes and values, of untouchables can be brought about by establishing
such as simplicity, truth, non-violence, and social justice. cottage industry for them. He believed that cottage
Gandhi read their books and essays and applied their ideas industry, especially spinning and weaving, would provide
to his own philosophy and practice. employment, income and dignity to the untouchables,
who were often denied access to other occupations by the
Important Tips caste system. He also thought that cottage industry would
Some of the works that influenced Gandhi’s concept are: promote self-reliance, simplicity and swadeshi among the
Unto This Last by John Ruskin: This was a book that untouchables and the rest of the society. He considered
Gandhi read in 1904, when he was in South Africa. It was untouchability as a sin and a blot on Hinduism, and fought
a critique of the capitalist system and a plea for a moral against it throughout his life.
economy based on the dignity of labour and the well-being
289. Solution (c)
of all.
Exp) Option c is the correct answer.
Civil Disobedience by Henry David Thoreau: This was
an essay that Gandhi read in 1907, when he was facing a Statement 1 is correct: Arya Samaj is considered a
legal crisis in South Africa. It was a defense of the right and monotheistic Indian Hindu reform movement that promotes
duty of the individual to resist unjust laws and authority. values and practices based on the belief in the infallible
authority of the Vedas. The samaj was founded by Dayanand
The Kingdom of God is Within You by Leo Tolstoy: This
Saraswati in 1875.
was a book that Gandhi read in 1908, when he was in
prison in South Africa. It was a religious and philosophical Statement 2 is correct: The Al-Hilal was an Urdu language
treatise that advocated for a life of love, peace, and non- newspaper founded by Maulana Abul Kalam Azad. The
violence based on the teachings of Jesus Christ. newspaper’s first edition was published in Kolkata in 1912. It
was used as a tool for opposing the British Raj in India. The
286. Solution (d) newspaper also supported the Indian freedom movement’s
Exp) Option d is the correct answer. cause and encouraged Indian Muslims to join the movement.
All the statements are correct regarding Gandhi and his Statement 3 is incorrect: Along with founding ‘Arya Gazette’
political life. He fought hard to improve the status of as its editor, Lala Lajpat Rai regularly contributed to several
casteless untouchables, whom he called Harijans or the major Hindi, Punjabi, English and Urdu newspapers and
children of God. He launched the Non-Cooperation magazines. He took out a Weekly newspaper “People” from
Movement in 1920, which involved boycotting foreign Lahore. He himself was the editor of this paper.
goods, institutions, titles, and honours, and promoting Statement 4 is correct: Hindu College was established on
swadeshi, education, and self-governance. He began the January 20, 1817, in Calcutta. Raja Ram Mohan Roy, the
Civil Disobedience Movement in 1930 with the famous great social reformer, headed its foundation committee.
Dandi March or Salt Satyagraha, in which he defied the
290. Solution (c)
British salt monopoly by making salt from seawater. He also
led other movements and campaigns based on the principles Exp) Option c is the correct answer.
of truth, non-violence, and self-rule. Statement 1 is correct: Asaf Ali was an Indian independence
activist and noted lawyer. He looked after the work of
287. Solution (d) Railway Ministry in the interim Government (1946).
Exp) Option d is the correct answer. Statement 2 is correct: The Ancient Monuments Preservation
Gandhi was not opposed to mechanization in general, but Act, 1904, enacted during Lord Curzon’s British India
only to the extent that it harmed human dignity, morality tenure, aimed to safeguard historic monuments, regulate
and well-being. He believed that machines should be used antiquities traffic and excavations, and protect and acquire
as a means to serve humanity, not to exploit or enslave it. He valuable archaeological, historical, or artistic artifacts and
also recognized that different countries had different levels sites through the Archaeological Survey of India.
of development and needs, and that what was suitable for Statement 3 is incorrect: The Indian National Congress
India might not be suitable for other nations. held its fifty-first session in Haripura in the Surat district
Therefore, Gandhi did not object to mechanization in in Gujarat. It was held on 19th February 1938 and presided
U.S.A. or other advanced countries, as long as it did not over by Subhash Chandra Bose.

PYQ Workbook 340


HISTORY OF MODERN INDIA

Statement 4 is correct: The British Colonial Government • The Sadler Commission, chaired by Dr. Michael E.
passed the “Rowlatt Act” which gave power to the police Sadler, was appointed by the Government of India
to arrest any person without any reason. The purpose of in 1917. It aimed to study and proposed reforms
the Act was to curb the growing nationalist upsurge in the for Calcutta University and the broader education
country. Mahatma Gandhi called upon the people to perform system. The commission recommended a 12-year
satyagraha against the act. Swami Sharaddhanand suggested school course, separating secondary and intermediate
no tax campaign (non-payment of Lagaan/Rent) in protest education, and emphasized the need for universities to
against Rowlatt Act. focus on higher education.
• English higher education in India can be said to have
Important Tips
begun with the establishment of Hindu College in
Haripura Congress Session, 1938: Calcutta in 1817, the first ‘Europanized’ institution
The 51st session of the Indian National Congress took of higher learning in Asia. The first three universities
place in Haripura, Gujarat, in 1938, under the leadership of Calcutta, Bombay and Madras, established in 1857
of Subhash Chandra Bose. were modeled on London.
Bose emphasized the need to peacefully resist the
292. Solution (b)
colonial government’s Federal Scheme, which was seen
as undemocratic and favoring British capitalism. Exp) Option b is the correct answer.
There was a heated debate during the session about the “Gokhale: My Political Guru” is a book written by
princely states, with Congress ultimately declaring its Mahatma Gandhi. In this book, Gandhi reflects on his
goal of complete independence for all of India, including relationship with Gopal Krishna Gokhale, a prominent
the princely states. leader and political mentor. He discusses the deep influence
The Congress session demanded a responsible government Gokhale had on him and his political philosophy. Gandhi
and the protection of civil liberties within the states. emphasizes Gokhale’s commitment to bringing a spiritual
and ethical dimension to politics, which greatly impacted his
Support for the release of political prisoners was also
own approach to leadership and social change.
expressed.
Recognizing the growing peasant movements, Important Tips
Congress affirmed the right of Kisan Sabhas (farmers’ Popular books written by M.K.Gandhi:
organizations) to form unions but specified that An Autobiography or The Story of My Experiments
activities conflicting with Congress principles would with Truth: This autobiography is a detailed account of
not be supported. Gandhi’s life, his experiments with truth, spirituality, and
Bose called for increased cooperation between Congress, his journey towards becoming the Mahatma.
Trade Union Congress, and peasant organizations. Hind Swaraj or Indian Home Rule: In this influential
The session concluded with the election of Subhash work, Gandhi presents his vision of self-rule for India,
Chandra Bose as the next Congress session’s president, emphasizing the importance of traditional Indian values
solidifying Congress’s commitment to India’s independence and nonviolent resistance.
struggle. Truth is God: This book delves into Gandhi’s beliefs,
291. Solution (d) focusing on truth and spirituality, rather than religious
dogma.
Exp) Option d is the correct answer.
India of My Dreams: Gandhi’s vision for an independent
Lala Lajpat Rai was elected as President of Indian National India is outlined in this collection of writings and
Congress in 1920. Congress held a special session in speeches, covering various aspects of socio-political and
Calcutta in September 1920 under the presidentship of economic life.
Lala Lajpat Rai. It met to pass the resolution of the Non- The Bhagavad Gita: Gandhi found inspiration in this
Cooperation Movement. Fueled by British actions and the revered Hindu scripture, which guided his principles of
Jallianwala Bagh massacre, it urged Indians to boycott British nonviolence and truth.
institutions and adopt Swadeshi goods, aiming for Swaraj. The Law and The Lawyers: In this thought-provoking
Important Tips book, Gandhi critiques the legal system and the role of
lawyers, advocating for truth and service above all.
• “My Experiments with Truth” stands as the
autobiographical account of Mohandas K. Gandhi, Gokhale - My Political Guru: Gandhi pays tribute to his
chronicling his life from early childhood to his initial mentor, Gopal Krishna Gokhale, sharing his experiences
endeavors in the pursuit of freedom. It was originally and learnings from this great leader.
written in the Gujarati language and was published Diet and Diet Reform: Gandhi’s experiments with diet
in weekly instalments in his journal Navjivan from and his pursuit of a balanced and healthy lifestyle are
1925 to 1929. documented in this work.

341 PYQ Workbook


HISTORY OF MODERN INDIA

Satyagraha in South Africa: This book provides insights on Earth. It was first published serially in the Arya from
into Gandhi’s experiences with Satyagraha, his nonviolent 1914 to 1919 and later in book form in 1939 and 1940, with
resistance movement, during his time in South Africa. subsequent editions.
Ashram Observances in Action: Gandhi explains the Important Tips
observances followed in his ashram, addressing themes Some of the well-known works by Sri Aurobindo:
like poverty, work, love, and truth, offering guidance for
Savitri: A Legend and A Symbol (poetry)
a purposeful life.
The Life Divine
293. Solution (d)
The Renaissance in India with A Defense of Indian Culture
Exp) Option d is the correct answer.
Hymns to the Mystic Fire
M.K. Gandhi was a supporter of philosophical anarchism. The Secret of the Veda
He believed in the principle of swaraj or self-rule, which
Essays on The Gita
meant not only political independence from British
colonialism, but also individual and collective autonomy The Synthesis of Yoga
from any form of centralized authority. He advocated for Vedic and Philological Studies
a decentralized and nonviolent society based on the values
of truth, nonviolence, and compassion. He rejected the idea 296. Solution (a)
of a state or a government as a necessary evil, and instead Exp) Option a is the correct answer.
envisioned a network of self-reliant villages and communities B. G. Tilak made the agreement between Muslim League and
that would cooperate with each other without coercion or Congress possible during Lucknow session, popularly called
domination. Lucknow pact of 1916. The Lucknow Pact of 1916 is signed
Important Tips between Bal Gangadhar Tilak and Muhammad Ali Jinnah.
Philosophical anarchism is a political philosophy that Tilak represented the Congress while framing the deal.
advocates for a society without any form of state or All-India Muslim League was led by Muhammad Ali Jinnah.
government, and instead relies on the voluntary cooperation It was adopted by the Congress at its Lucknow session on
and mutual aid of individuals and communities. December 29 and by the Muslim league on Dec. 31, 1916.
Some of the most influential philosophical anarchists in 297. Solution (c)
history are William Godwin, Henry David Thoreau, Leo
Exp) Option c is the correct answer.
Tolstoy, Emma Goldman, and Mahatma Gandhi.
The author of the flag song (Jhanda Geet) of India is
294. Solution (d) Shyamlal Parshad Gupta. This patriotic song, originally
Exp) Option d is the correct answer composed with seven stanzas, was later modified by the
Dadabhai Naoroji wrote the book called “Poverty and Un- Congress, recognizing three stanzas as the “Dhwajgeet” or
British Rule in India” where he criticised the economic flag song. It was officially accepted in the Congress session
impact of the British rule. Hence, statement 1 is correct. of 1938. Shyamlal Gupta ‘Parshad’ was a prominent poet and
In March 1856 Dadabhai Naoroji was appointed as professor patriot known for his strong expressions of nationalism. His
of Gujarati at University College in London. Hence, song played a significant role in inspiring many to sacrifice
statement 2 is correct. for the country’s freedom struggle.
In October 1849, Dadabhai Naoroji and twelve other 298. Solution (b)
Maharashtrian and Parsi Elphinstonians opened six
Exp) Option b is the correct answer.
schools under the auspices of the Students’ Literary and
Scientific Society, three for Parsi girls and three for Hindu The Widow Remarriage Act was passed on 16 July 1856,
Maharashtrian girls. Hence Statement 3 is correct. during the rule of Lord Canning, who was the Governor-
First Indian elected to the British House of Commons, General of India at that time. The act legalized the remarriage
Dadabhai Naoroji contested on the ticket of Liberal Party. of Hindu widows in all jurisdictions of India under East
Hence statement d is incorrect. India Company rule.

295. Solution (d) Important Tips


Exp) Option d is the correct answer. • The Widow Remarriage Act was drafted by Lord
Dalhousie, who was the previous Governor-General,
Aurobindo Ghosh authored “The Life Divine,” a
and passed during Lord Canning before the Indian
comprehensive work that blends Western thought and
Rebellion of 1857.
Eastern spirituality, offering original insights. It explores
themes such as human aspiration, the emergence of life • The act was enacted due to the efforts of social
from a Divine Source, evolution from matter to spirit, reformer Ishwar Chandra Vidyasagar, who advocated
human consciousness, and the spiritual destiny of life for the rights and dignity of Hindu widows.

PYQ Workbook 342


HISTORY OF MODERN INDIA

• The act provided legal safeguards against loss of Important Tips


certain forms of inheritance for remarrying a Hindu Raja Ram Mohan Roy – Literary Work
widow, though, under the Act, the widow forsook any
inheritance due her from her deceased husband. Literary Work Year
• The act was a major step towards the empowerment of
Tuhfat-ul-Muwahhidin 1804
women and the abolition of harmful customs in Hindu
society. Vedanta Gantha 1815

299. Solution (b) Kenopanishads, Translation of an abridgment 1816


Exp) Option b is the correct answer. of the Vedanta Sara, Ishopanishad
Pair a is incorrectly matched: “The Indian Struggle” The Precepts of Jesus- The Guide to Peace and 1820
authored by Subhash Chandra Bose, is a two-part book Happiness , A Defence of Hindu Theism
that chronicles India’s independence movement from 1920
to 1942, spanning key events and movements. Banned by the Bengali Grammar 1826
British, it was published in India in 1948 after independence. History of Indian Philosophy , The Universal 1829
Pair b is correctly matched: Rafiq Zakaria authored the Religion
book “The Man Who Divided India,” which provides a
Gaudiya Vyakaran 1833
critical biography of Muhammad Ali Jinnah and explores
his role in the partition of India. This book analyzes Jinnah’s
leadership and its aftermath, shedding light on the factors 302. Solution (a)
that contributed to the division of India into India and Exp) Option a is the correct answer.
Pakistan. It offers insights into the complexities of the Mahatma Gandhi completed his translation of “Anasakti
partition and its consequences Yoga” in Kausani in 1929. This translation is a part of his
Pair c is incorrectly matched: “Indian Unrest” authored by commentary on the Bhagavad Gita, where he emphasized
Valentine Sir Chirol, is a significant collection of articles the importance of renunciation of the fruits of action
addressing the complex issue of Indian unrest during the and selfless devotion as central teachings for attaining
British Empire. It offers valuable insights and comprehensive self-realization and ultimate peace. His interpretation
analysis akin to a formal investigative report on the root underscores the significance of these principles in everyday
causes and state of affairs leading to unrest in India. life and worldly pursuits.
Pair d is incorrectly matched: “Unhappy India” was 303. Solution (d)
authored by Lala Lajpat Rai. This book was a response to
Exp) Option d is the correct answer.
Katherine Mayo’s “Mother India” and aimed to counter her
arguments about India and its people, highlighting the Lord Mayo’s Resolution of 1870 on Financial
socio-economic problems caused by British colonial rule. Decentralization visualized the development of local
self-government institutions. Its resolution pertained to
300. Solution (a) financial decentralization which was a legislative devolution
Exp) Option a is the correct answer. inaugurated by the Indian Council Act of 1861. Apart
Mahatma Gandhi was not related to the Swadeshi from the annual grant from the imperial government, the
Movement, which was started to protest against the partition provincial governments were authorized to resort to local
of Bengal in 1905 and continued up to 1911. During this taxation to balance their budgets.
period, Gandhi was residing in South Africa and fighting Important Tips
against racial discrimination and oppression there. • Lord Mayo served as the 4th Viceroy of India from 12
301. Solution (a) January 1869 to 8 February 1872.
Exp) Option a is the correct answer. • He followed the policy of non-intervention.

Raja Rammohan Roy’s famous work “A Gift to Monotheists” • India’s census began during his tenure.
(Tuhfat-ul-Muwahhidin) was written in the Persian • He was assassinated at the penal settlement of Port
language, with an introduction in Arabic. This work is Blair in the Andaman Islands.
notable as it marks one of his early intellectual endeavors
304. Solution (a)
and is recognized as the first known theological statement
by him before he gained fame and notoriety as a Vedantin. Exp) Option a is the correct answer.
During that time, Bengali had not yet become the language James Andrew Ramsay was the real name of the 1st
of intellectual discourse, and Raja Rammohan Roy’s writings Marquess of Dalhousie, who served as the Governor-
were primarily in Persian and Arabic. The work was later General of India from 1848 to 1856. He was also known as the
translated into English by Maulavi Obaidullah EI Obaid Earl of Dalhousie between 1838 and 1849. He was a Scottish
and published by the Adi Brahmo Samaj in 1884. statesman and colonial administrator who is credited with

343 PYQ Workbook


HISTORY OF MODERN INDIA

creating the map of modern India through his conquests and Marabar Caves, which leads to a significant incident and its
annexations of independent provinces. He also introduced aftermath.
many reforms in India, such as railways, telegraph, postal
307. Solution (b)
service, public works, education, and administration. He is
also infamous for his policy of the Doctrine of Lapse, which Exp) Option b is the correct answer.
allowed the British to annex any Indian state whose ruler “Soz-e-Vatan” is a story collection written by Nawabrai,
died without a natural heir. This policy was widely resented who later became well-known as Munshi Premchand. It
by the Indian princes and contributed to the outbreak of the was published in 1908 and consisted of five stories in Urdu.
Indian Rebellion of 1857, one year after his retirement. The title “Soz-e-Vatan” translates to “mourning of the
country” and the stories in this collection include “The
Important Tips
Most Precious Jewel of the World,” “Sheikh Makhmur,”
• The Doctrine of Lapse was a policy of annexation that “This is My Country,” “The Reward of Mourning,” and
was applied by the British East India Company in India “Worldly Love.” The British government considered this
until 1858. collection seditious and confiscated and destroyed all its
• It stated that any princely state under the control of the copies, leading Nawabrai to adopt the pen name Munshi
Company would be annexed if the ruler died without a Premchand.
male heir or was incompetent.
Important Tips
• The policy was devised by the Court of Directors of
Notable works of Premchand:
the Company in 1834, but it was widely used by Lord
Dalhousie, who was the Governor-General of India Karmabhoomi (1932): Set in 1930, it explores the unity
from 1848 to 1856. of Hindus and Muslims against British exploitation,
foreshadowing India’s eventual partition.
• He annexed several states under this policy, such as
Satara, Jhansi, Nagpur, and Tanjore. Kafan: A poignant short story depicting the irresponsibility
and shamelessness of Ghisu and Madhav, who squander
• The policy was seen as illegitimate and disrespectful
borrowed money meant for a shroud on drinks and food.
by many Indians, and it was one of the causes of the
Indian Rebellion of 1857. Gaban (1931): This story delves into the insatiable greed
for jewels, revealing the moral and social corruption it
305. Solution (b) leads to.
Exp) Option b is the correct answer. Godan (1936): A realistic portrayal of rural India,
The oldest ashram related to Mahatma Gandhi is the addressing issues like casteism, women’s exploitation, and
Phoenix Ashram in South Africa. He established this ashram the oppression of peasant classes.
in 1904, after being inspired by the writings of Leo Tolstoy Poos Ki Raat: Depicts a farmer’s struggle to survive the
and John Ruskin. He named the ashram after a magazine harsh winter without proper clothing due to debts to
called The Phoenix, which he published from there. The moneylenders.
ashram was a place where Gandhi and his followers practiced
Nirmala (1928): An early critique of dowry practices
the principles of truth, non-violence, and self-reliance.
through the suffering of an adolescent girl married to an
Important Tips elderly man.
Wardha Ashram: This ashram was established by Thakur Ka Kuan: Focuses on caste-based discrimination
Gandhi in 1936 in a village called Segaon near Wardha in and the denial of clean drinking water to Dalits,
Maharashtra. It was later renamed as Sevagram Ashram, highlighting societal prejudice.
which means the village of service. Eidgah: A heartwarming tale of a young boy’s selfless
Sadaqat Ashram: This ashram was established by Gandhi sacrifice on Eid, choosing a gift for his grandmother over
in 1947 in Patna in Bihar. It was also known as the Bihar sweets and toys.
Vidyapeeth or the Bihar University. It was a center for Namak Ka Daroga: Tells the story of an honest salt
education and research on Gandhian philosophy and rural inspector, Vanshidhar, who remains steadfast in his
development. integrity despite the temptations of bribery.
306. Solution (c) 308. Solution (a)
Exp) Option c is the correct answer. Exp) Option a is the correct answer.
The book “A Passage to India” was written by E. M. Forster. The book “Geographical Factors in Indian History” was
This novel, published in 1924, is set against the backdrop of written by K.M. Panikkar. It delves into the significant role
the British Raj and the Indian independence movement that geography plays in shaping the historical evolution
in the 1920s. It explores the complexities of colonialism, of a nation, influencing both internal and external
racial tensions, and personal relationships in British relations. The book is divided into chapters covering
India. The story follows four main characters and delves various aspects of geography’s impact on India, including its
into their experiences and interactions during a trip to the geographical position, the role of the Himalayas, and routes

PYQ Workbook 344


HISTORY OF MODERN INDIA

of invasion. Panikkar’s work emphasizes the importance of Lord Mayo was the Viceroy of India from 1869 to 1872,
understanding India’s geographical context in the broader and he conducted the first census of India in 1872. This
historical narrative. census was the first complete and systematic enumeration
of the population of British India, covering an area of 239
309. Solution (d) million acres and a population of 254 million people. The
Exp) Option d is the correct answer. census collected data on age, sex, caste, religion, occupation,
Option A in List-I matches Option 1 in List-II: The Dandi education, infirmities, and marital status. The census also
March, led by Mahatma Gandhi in 1930, was a 24-day act recorded the number of houses, their construction, and their
of nonviolent civil disobedience against the British salt amenities. The census was conducted by local officials under
monopoly and a symbol of resistance to colonial rule. It the supervision of provincial superintendents, who were
covered 387 kilometers from Sabarmati Ashram to Dandi, mostly British officers.
inspiring millions of Indians to join.
313. Solution (a)
Option B in List-I matches Option 4 in List-II: In December
Option a is the correct answer.
1929, under the Presidency of Jawaharlal Nehru, the Lahore
Congress formalised the demand of ‘Purna Swaraj’ or full Option A in List-I matches Option 3 in List-II: Madan
independence for India. Mohan Malaviya was an Indian scholar, educational
reformer and politician notable for his role in the Indian
Option C in List-I matches Option 2 in List-II: The Red
independence movement. He was president of the Indian
Shirt movement, also known as Khudai Khidmatgar or Surkh
National Congress four times and the founder of Akhil
Posh, was a Pashtun nonviolent resistance movement against
Bharat Hindu Mahasabha. Malaviya strove to promote
British colonial rule in India’s North-West Frontier Province.
modern education among Indians and co-founded the
Led by Abdul Ghaffar Khan, it originally focused on social
Banaras Hindu University (BHU) at Varanasi in 1916,
reform, education, and ending blood feuds.
which was created under the 1915 BHU Act.
Option D in List-I matches Option 3 in List-II: The Bardoli
Option B in List-I matches Option 4 in List-II: On 1
Satyagraha, 1928 was a movement in the independence
January 1923 C. R. Das along with Motilal Nehru formed the
struggle led by Sardar Vallabhai Patel for the farmers of
“Swaraj Party” and became its leader in Bengal Assembly. C.
Bardoli against the unjust raising of taxes.
R. Das was its president and Motilal Nehru was its secretary.
310. Solution (d) The main purpose was to oppose British policies within the
Exp) Option d is the correct answer. councils, argue for reform and also demonstrate that these
councils were not truly democratic.
The play “Andher Nagari Chaupat Raja” was written by
the renowned Hindi litterateur Bhartendu Harishchandra. Option C in List-I matches Option 1 in List-II: Between
This satirical play, consisting of six acts, serves as a scathing the years 1916 and 1918, the Indian independence movement
critique of irrational and autocratic governance systems. witnessed the growth and spread of the home rule movement
It portrays how such systems eventually self-destruct due spearheaded by leaders like Bal Gangadhar Tilak and
to their own actions. Bhartendu composed this play in a Annie Besant. The aim of the home rule movement was the
single day for the Hindu National Theater of Banaras. The attainment of home rule or a dominion status for India under
play highlights issues like greed, corruption, and the the British Empire along the lines of countries like Canada
consequences of irrational rule. and Australia. This movement was carried out through the
two home rule leagues. Tilak launched the Indian Home Rule
311. Solution (d) League in April 1916 at Belgaum and Annie Besant launched
Exp) Option d is the correct answer. the Home Rule League in September 1916 at Madras.
Nehru was not influenced by German Nationalism. Option D in List-I matches Option 2 in List-II: Gopal
German nationalism was a political ideology that emerged Krishna Gokhale was a senior leader of the Indian National
in Germany in the 19th century. It was based on the belief Congress and the founder of the Servants of India Society
that the German people were a superior race and that they in 1905. The Society organized many campaigns to promote
had a right to rule over other peoples. Nehru was a staunch education, sanitation, health care, and fight the social evils
opponent of all forms of racism and discrimination, and of untouchability and discrimination, alcoholism, poverty,
he outrightly rejected German nationalism. Nehru was oppression of women and for protection of women from
influenced by socialism, British liberalism, and Gandhian domestic abuse.
philosophy. Socialism is an economic and political system
314. Solution (c)
that advocates for the common ownership of the means of
production. British liberalism is a political ideology that Exp) Option c is the correct answer
emphasizes individual liberty and limited government. Dadabhai Naoroji, also known as the ‘Grand Old Man of
Gandhian philosophy advocated for non-violence and social India’ was born on 4th September 1825. He was the founder
reform. and a member of the Indian National Congress in 1885 along
with A. O. Hume and Dinshaw Wacha. Dadabhai Naoroji
312. Solution (d) demanded Swaraj as national demand for the first time at
Exp) Option d is the correct answer. the Calcutta session of Congress in 1906.

345 PYQ Workbook


HISTORY OF MODERN INDIA

Important Tips 316. Solution (d)

Dadabhai Naoroji became the president of the Congress Exp) Option d is the correct answer
party in 1886, 1893, and 1906. Sardar Patel, who was in charge of the states’ ministry in
Dadabhai Naoroji was one of the first Indian political the interim cabinet, helped by V.P. Menon, the secretary in
activists and mentors of Mahatma Gandhi, Bal Gangadhar the ministry, was responsible for the integration of Princely
Tilak, and Gopal Krishna Gokhale. States into Indian Union. There were several states that were
adamant to not join India. Some of them thought this to be
In 1909, Mahatma Gandhi gave a wider interpretation of
the best moment to acquire independent statehood, while
Swaraj in his book Hind Swaraj.
there were others who wanted to become a part of Pakistan.
315. Solution (c) But under the able leadership of Patel all the princely states
were successfully integrated with India.
Exp) Option c is the correct answer
Surat Split of the Indian National Congress resulted in the 317. Solution (c)
party’s division into two groups, i.e., the Moderates and the Exp) Option c is the correct answer.
Extremists. The Moderates were in favor of Rash Bihari The author of the official history of the Indian National
Ghosh, while the Extremists supported Bal Gangadhar Congress was Dr. Pattabhi Sitaramayya. His book, titled
Tilak to be the president of India. Eventually, Rash Bihari “History of the Indian National Congress,” provides a
Ghosh became the President of INC. This angered the comprehensive account of the Congress party’s formation,
Extremist like Tilak and eventually Surat split happened. development, and organization from its inception in 1885
Thus, the Surat split was led by Tilak by his extremist views until 1935. This two-volume work offers insights into the
and policies. Later the Extremist were expelled from the INC. INC sessions, debates, pacts, internal developments, and
challenges faced by the party during its early years. Dr.
Important Tips Pattabhi Sitaramayya’s efforts to document the Congress’s
After effects of Surat Session : history earned him the title of the “historian of the
Following the Surat Separation, Rabindranath Tagore Congress.”
made vain attempts to bring the Moderates and Extremists 318. Solution (b)
back together.
Exp) Option b is the correct answer.
Additionally, the 1908 Allahabad convention’s adoption of
“Bharat Durdasha” is a significant work written by
resolutions to forever disqualify the Congress’s Extremist
Bharatendu Harishchandra in 1875. In this Hindi play,
section made matters worse.
Bharatendu portrays the plight of India during his time. He
The moderate-dominated Congress fell dormant after uses symbols and metaphors to shed light on the challenges
leaders like Bal Gangadhar Tilak was imprisoned, Lala faced by the country. The play addresses issues such as
Lajpat Rai went to USA, Bipin Chandra Pal retired from British colonial rule, internal discord, vices, economic
active politics and, Aurobindo ghosh went nderground till exploitation, and more. Bharatendu’s work serves as both a
Alipore conspiracy and later was tried for it, after which reflection of the problems of his era and a call to action for
he also retired from active policts and went to Puducherry. Indians to work towards positive change.

PYQ Workbook 346

You might also like